Você está na página 1de 1090

JEE MAIN

COMPLETE

CHEMISTRY
JEE MAIN

COMPLETE

CHEMISTRY
K.L. Kapoor
Formerly Associate Professor,
Hindu College,
University of Delhi, Delhi

McGraw Hill Education (India) Private Limited


CHENNAI

McGraw Hill Education Offices


Chennai New York St Louis San Francisco Auckland Bogotá Caracas
Kuala Lumpur Lisbon London Madrid Mexico City Milan Montreal
San 
Juan Santiago  Singapore Sydney Tokyo Toronto
Published by McGraw Hill Education (India) Private Limited,
444/1, Sri Ekambara Naicker Industrial Estate, Alapakkam, Porur, Chennai-600116
Complete Chemistry—JEE Main
Copyright © 2018, McGraw Hill Education (India) Private Limited.
No part of this publication may be reproduced or distributed in any form or by any means, electronic, mechanical, photocopying,
recording, or otherwise or stored in a database or retrieval system without the prior written permission of the publishers. The program
listings (if any) may be entered, stored and executed in a computer system, but they may not be reproduced for publication.
This edition can be exported from India only by the publishers,
McGraw Hill Education (India) Private Limited.
Price: `875/-

1  2 3 4 5 6 7 8 9   7085462   22 21 20 19 18

Printed and bound in India


ISBN (13):  978-93-87572-55-3
ISBN (10):  93-87572-55-2

Information contained in this work has been obtained by McGraw Hill Education (India), from sources believed to be reliable.
However, neither McGraw Hill Education (India) nor its authors guarantee the accuracy or completeness of any information
published herein, and neither McGraw Hill Education (India) nor its authors shall be responsible for any errors, omissions, or
damages arising out of use of this information. This work is published with the understanding that McGraw Hill Education
(India) and its authors are supplying information but are not attempting to render engineering or other professional services. If
such services are required, the assistance of an appropriate professional should be sought.

Typeset at Sri Krishna Graphics, Delhi and printed at

Cover Designer: Neeraj Dayal

visit us at: www.mheducation.co.in


To Our Readers...
How to Crack the JEE

T o help students preparing for the JEE Main, there was need for a book which included a variety of Multiple Choice
Questions (MCQs) designed on the basis of the prescribed syllabus for this examination. This book is an attempt
in this direction and will help students in developing a strong foundation and enough confidence to take the JEE Main.
The various topics of chemistry may be classified into three branches—Physical, Inorganic and Organic. The
book covers these three branches in 29 chapters. Each chapter starts with the synopsis of theory followed by MCQs
along with answers and hints and solutions to arrive at correct answers. Wherever needed, the chapter is divided into
sections to cover the subject in easily understandable portions to help in better grasping of the subject matter. Each
section/chapter ends with MCQs from the previous years' AIEEE and JEE Main. This will help students in getting an
idea about the types and levels of questions asked in this competitive examination. The answers and solutions to these
questions are provided separately, immediately after the questions.
The analyses of these papers (provided on the next page) reveals that in most of the cases, one question is asked
from each chapter and the entire syllabus is covered in the examination paper. This book provides extensive coverage
of the theory as well as the associated MCQs. The contents of each chapter are covered in various sections. At the end
of all the sections, more extensive MCQs based on the Entire Chapter along with their solutions are also included.
It will be beneficial for the students to adopt the present book as the reference book along with their main text book.
The MCQs included in this book should be attempted along with the class-room teaching of the subject matter. A regular
and periodical review of the theory and MCQs from this book will help students in gaining enough confidence to appear
in the JEE Main and enable them to face the challenge of successfully clearing this examination.
From the analyses of previous years’ question papers, a pattern of predominant topics emerges on which students
should pay more attention while preparing for the examination. These are:
Physical Chemistry—Entire portion
Inorganic Chemistry has been thoroughly revised and updated—Chemical families—perioidic properties,
structures of compounds containing Si, N, P, S, halogens and inert gases, d-block elements and coordination chemistry
Organic Chemistry—Stereoisomerism, SN1 and S22 Reactions, Reactions involving rearrangement, Chemistry of
typical reactions shown by phenols, aldehydes and ketones and amines, relative acidity/basicity of phenolic, Carboxylic
acids and amines, polymers, carbohydrates, stereochemistry involved in halogenation of alkenes and dehalogenation of
halogenated compounds to give alkene, reactions involving Grignard reagent and diazonium salt.

K.L. Kapoor
Trend Analysis
Physical Chemistry

  Chapter 2014 2014 2015 2016 2017


(offline) (online) (offline) (offline) (offline)
I II III IV
1. Some Basic Concepts 1 1 1 1 — 1 1 2
2. States of Matter
Gaseous State 1 2 1 — 1 1
Liquid State — — — — 1 1
Solid State 1 1 1 1 2 1 1
3. Atomic Structure 1 1 2 3 — 1 1 2
4. Chemical Bonding 1 2 3 — 2 2 1 1
5. Solutions 1 1 1 — 2 1 1
6. Chemical Thermodynamics 1 1 — 2 — 1 2 2
7. Chemical and Ionic Equilibria
Chemical Equilibria 1 1 — 1 1 1 1
Ionic Equilibria — — 2 1 1 — 2
8. Redox Reactions and Electrochemistry 1
Redox Reactions 1 — — 1 3 — 1
Electrolysis — 2 — 1 — —
Conduction 2 — — — — —
Galvanic Cells 1 — 1 — — — 1 1
9. Chemical Kinetics 1 1 2 1 1 1 1 1
10. Surface Chemistry — — — — — 2 1 1
Total 13 13 14 12 14 12 11 15

Inorganic Chemistry

Chapter 2014 2014 2015 2016 2017


(offline) (online) (offline) (offline) (offline)
I II III IV
11. Chemical Families–Periodic properties — — 1 2 1 1 1 1
12. General Principles and
Processes of Isolation of Metals 1 1 — — — 1 1
13. Hydrogen — — — — — 1
14. s-Block Elements — — — — — 1 1
(Contd...)
viii  Trend Analysis

  Chapter 2014 2014 (online) 2015 2016 2017


(offline)
15. Study of p-Block Elements 1 1 2 1 2 2 1
(Groups 13, 14 and 15)
16. Study of p-Block Elements 1 — 1 1 — 1 2 2
(Groups 16, 17 and 18)
17. d– and f– Block Elements 1 2 — 2 1 1 2
18. Coordination Chemistry
and Organometallics 2 2 1 2 2 1 2 1
19. Nuclear Chemistry — — — — — —
Total 6 6 5 8 6 9 10 4

Organic Chemistry

  Chapter 2014 2014 2015 2016 2017


(offline) (online) (offline) (offline) (offline)
I II III IV
20. Purification and Characterization of
Organic Compounds 1 — — — — 1
21. Some Basic Principles — 1 2 1 1 2 1 2
22. Hydrocarbons
Alkanes — — — 1 — —
Alkenes — — 1 1 — — 1 2
Alkynes — — 1 — —
Benzene — — — — 1 1
23. Organic Compounds Containing
Halogens 2 2 1 — 1 1 1 1
24. Organic Compounds
Containing Oxygen
Alcohols — 1 — 2 — — 1
Phenols 2 — 1 — 2 —
Ethers — 1 — — 1 —
Aldehydes & Ketones 1 2 2 — — — 1
Carboxylic Acids 1 — — 2 1 — 1
25. Organic Compounds 2 1 1 1 1 1 1 1
Containing Nitrogen
26. Synthetic & Natural Polymers 1 2 — — 1 1 1 1
27. Biomolecules & Biological
Processes 1 — — 1 1 — 1 1
28. Chemistry in Action — — 1 1 — 2 2
29. Principles Related
to Practical Chemistry — 1 1 — — — 1
Total 11 11 11 10 10 9 9 11
About JEE Main
1.  Introduction and Scheme of Examination
The Joint Entrance Examination from the year 2013 for admission to the undergraduate programmes in Engineering
is being held in two parts, JEE-Main and JEE-Advanced. Only the top 1,50,000 candidates (including all categories)
based on performance in JEE Main will qualify to appear in the JEE Advanced examination. Admissions to IITs will be
based only on category-wise All India Rank (AIR) in JEE Advanced, subject to condition that such candidates are in the
top 20 percentile categories. Admission to NITs will be based on 40% weightage for performance in Class XII board
marks (normalized) and the remainder 60% weightage would be given to performance in JEE Main and a combined All
India Rank (AIR) would be decided accordingly.
In case any State opts to admit students in the engineering Colleges affiliated to state Universities where States
require separate merit list to be provided based on relative weightages adopted by the states, then the merit list shall be
prepared with such relative weightages as may be indicated by States.

2.  Eligibility Criteria and List of Qualifying Examinations for JEE(Main) Exam
The minimum academic qualification for appearing in JEE(Main) is that the candidate must have passed in final
examination of 10+2 (Class XII) or its equivalent referred to as the qualifying examination (see below).
However, admission criteria in the concerned institution/university will be followed as prescribed by concerned
university/institution and as per the guidelines & criteria prescribed by AICTE.

Qualifying Examinations
List of Qualifying Examinations
(i) The +2 level examination in the 10+2 pattern of examination of any recognized Central/State Board of Secondary
Examination, such as Central Board of Secondary Education, New Delhi, and Council for Indian School Certificate
Examination, New Delhi
(ii) Intermediate or two-year Pre-University Examination conducted by a recognized Board/University.
(iii) Final Examination of the two-year course of the Joint Services Wing of the National Defence Academy.
(iv) Any Public School/Board/University Examination in India or in foreign countries recognized by the Association
of Indian Universities as equivalent to 10+2 system.
(v) H.S.C. Vocational Examination.
(vi) A pass grade in the Senior Secondary School Examination conducted by the National Open School with a
minimum of five subjects.
(vii) 3 or 4-year diploma recognized by AICTE or a State Board of Technical Education.
x  About JEE Main

3.  Pattern of Examination


Subject combination for each paper and type of questions in each paper are given below:

Subjects Type of Questions Duration


Paper 1 Physics, Chemistry & Mathematics Objective type questions with equal 3 Hours
weightage to Physics, Chemistry &
Mathematics
Paper 2 Mathematics – Part I Objective type questions 3 Hours
Aptitude Test – Part II & Objective type questions
Drawing Test – Part III questions to test Drawing Aptitude

Requirement of papers for different courses is given in the table below:

Course Papers
B.E/B.TECH Paper – 1
B.ARCH/B. PLANNING Paper – 2

Scoring and Negative Marking


There will be objective type questions with four options having single correct answer. For each incorrect response,
one fourth (1/4) of the total marks allotted to the question would be deducted. No deduction from the total score will,
however, be made if no response is indicated for an item in the answer sheet.
Syllabus

SECTION —A

Physical Chemistry

Unit 1  Some Basic Concepts in Chemistry


Matter and its nature, Dalton’s atomic theory; Concept of atom, molecule, element and compound; Physical quantities
and their measurements in Chemistry, precision and accuracy, significant figures, S.I. Units, dimensional analysis; Laws
of chemical combination; Atomic and molecular masses, mole concept, molar mass, percentage composition, empirical
and molecular formulae; Chemical equations and stoichiometry.

Unit 2  States of Matter


Classification of matter into solid, liquid and gaseous states.

Gaseous State: Measurable properties of gases; Gas laws—Boyle’s law, Charles’ law, Graham’s law of diffusion,
Avogadro’s law, Dalton’s law of partial pressure; Concept of Absolute scale of temperature; Ideal gas equation, Kinetic
theory of gases (only postulates); Concept of average, root mean square and most probable velocities; Real gases,
deviation from Ideal behaviour, compressibility factor, van der Waals equation, liquefaction of gases, critical constants.

Liquid State: Properties of liquids—vapour pressure, viscosity and surface tension and effect of temperature on
them (qualitative treatment only).

Solid State: Classification of solids: molecular, ionic, covalent and metallic solids, amorphous and crystalline solids
(elementary idea); Bragg’s Law and its applications, Unit cell and lattices, packing in solids (fcc, bcc and hcp lattices),
voids, calculations involving unit cell parameters, imperfection in solids; electrical, magnetic and dielectric properties.

Unit 3  Atomic Structure


Discovery of sub-atomic particles (electron, proton and neutron); Thomson and Rutherford atomic models and
their limitations; Nature of electromagnetic radiation, photoelectric effect; spectrum of hydrogen atom, Bohr model
of hydrogen atom—its postulates, derivation of the relations for energy of the electron and radii of the different
orbits, limitations of Bohr’s model; dual nature of matter, de-­Broglie’s relationship, Heisenberg uncertainty principle.
Elementary ideas of quantum mechanics, quantum mechanical model of atom, its important features, y and y2, concept
of atomic orbitals as one electron wave functions; Variation of y and y2 with r for 1s and 2s orbitals; various quantum
numbers (principal, angular momentum and magnetic quantum numbers) and their significance; shapes of s, p and
d—orbitals, electron spin and spin quantum number; rules for filling electrons in orbitals—aufbau principle, Pauli’s
exclusion principle and Hund’s rule, electronic configuration of elements, extra stability of half-filled and completely
filled orbitals.
xii  Syllabus

Unit 4  Chemical Bonding and Molecular Structure


Kossel: Lewis approach to chemical bond formation, concept of ionic and covalent bonds.

Ionic Bonding: Formation of ionic bonds, factors affecting the formation of ionic bonds; calculation of lattice
enthalpy.

Covalent Bonding: Concept of electronegativity, Fajan’s rule, dipole moment; Valence Shell Electron Pair Repulsion
(VSEPR) theory and shapes of simple molecules.

Quantum mechanical approach to covalent bonding: Valence bond theory—Its important features, concept of
hybridization involving s, p and d orbitals; Resonance.

Molecular Orbital Theory: Its important features, LCAOs, types of molecular orbitals (bonding, antibonding),
sigma and pi-bonds, molecular orbital electronic configurations of homonuclear diatomic molecules, concept of bond
order, bond length and bond energy.
Elementary idea of metallic bonding. Hydrogen bonding and its applications.

Unit 5  Chemical Thermodynamics


Fundamentals of Thermodynamics: System and surroundings, extensive and intensive properties, state functions,
types of processes.

First Law of Thermodynamics: Concept of work, heat, internal energy and enthalpy, heat capacity, molar heat
capacity, Hess’s law of constant heat summation; Enthalpies of bond dissociation, combustion, formation, atomization,
sublimation, phase transition, hydration, ionization and solution.

Second Law of Thermodynamics: Spontaneity of processes; DS of the universe and DG of the system as criteria
for spontaneity, DGo (Standard Gibbs energy change) and equilibrium constant.

Unit 6  Solutions
Different methods for expressing concentration of solution—molality, molarity, mole fraction, percentage (by volume
and mass both), vapour pressure of solutions and Raoult’s Law—Ideal and non-ideal solutions, vapour pressure—
composition plots for ideal and non-ideal solutions; Colligative properties of dilute solutions—relative lowering
of vapour pressure, depression of freezing point, elevation of boiling point and osmotic pressure; Determination of
molecular mass using colligative properties; Abnormal value of molar mass, van’t Hoff factor and its significance.

Unit 7  Equilibrium
Meaning of equilibrium, concept of dynamic equilibrium.

Equilibria Involving Physical Processes: Solid–liquid, liquid–gas and solid–gas equilibria, Henry’s law, general
characteristics of equilibrium involving physical processes.

Equilibria Involving Chemical Processes: Law of chemical equilibrium, equilibrium constants (Kp and Kc) and
their significance, significance of DG and DGo in chemical equilibria, factors affecting equilibrium concentration,
pressure, temperature, effect of catalyst; Le­Chatelier’s principle.

Ionic Equilibrium: Weak and strong electrolytes, ionization of electrolytes, various concepts of acids and bases
(Arrhenius, Bronsted—Lowry and Lewis) and their ionization, acid—base equilibria (including multistage ionization)
and ionization constants, ionization of water, pH scale, common ion effect, hydrolysis of salts and pH of their solutions,
solubility of sparingly soluble salts and solubility products, buffer solutions.
Syllabus  xiii

Unit 8  Redox Reactions and Electrochemistry


Electronic concepts of oxidation and reduction, redox reactions, oxidation number, rules for assigning oxidation
number, balancing of redox reactions.
Eectrolytic and metallic conduction, conductance in electrolytic solutions, specific and molar conductivities and their
variation with concentration: Kohlrausch’s law and its applications.
Electrochemical cells—Electrolytic and Galvanic cells, different types of electrodes, electrode potentials including
standard electrode potential, half-cell and cell reactions, emf of a Galvanic cell and its measurement; Nemst equation
and its applications; Relationship between cell potential and Gibbs’ energy change; Dry cell and lead accumulator; Fuel
cells; Corrosion and its prevention.

Unit 9  Chemical Kinetics


Rate of a chemical reaction, factors affecting the rate of reactions c­ oncentration, temperature, pressure and catalyst;
elementary and complex reactions, order and molecularity of reactions, rate law, rate constant and its units, differential
and integral forms of zero and first order reactions, their characteristics and half-lives, effect of temperature on rate of
reactions—Arrhenius theory, activation energy and its calculation, collision theory of bimolecular gaseous reactions
(no derivation).

Unit 10  Surface Chemistry

Adsorption: Physisorption and chemisorption and their characteristics, factors affecting adsorption of gases on
solids—Freundlich and Langmuir adsorption isotherms, adsorption from solutions.

Catalysis: Homogeneous and heterogeneous, activity and selectivity of solid catalysts, enzyme catalysis and its
mechanism.

Colloidal state: Distinction among true solutions, colloids and suspensions, classification of colloids—lyophilic,
lyophobic; multi molecular, macro-molecular and associated colloids (micelles), preparation and properties of
colloids—Tyndall effect, Brownian movement, electrophoresis, dialysis, coagulation and flocculation; Emulsions and
their characteristics.

SECTION — B
Inorganic Chemistry
Unit 11  Classificaton of Elements and Periodicity in Properties
Modem periodic law and present form of the periodic table, s, p, d and f block elements, periodic trends in properties
of elements­atomic and ionic radii, ionization enthalpy, electron gain enthalpy, valence, oxidation states and chemical
reactivity.

Unit 12  General Principles and Processes of Isolation of Metals


Modes of occurrence of elements in nature, minerals, ores; steps involved in the extraction of metals — concentration,
reduction (chemical. and electrolytic methods) and refining with special reference to the extraction of Al, Cu, Zn and
Fe; Thermodynamic and electrochemical principles involved in the extraction of metals.

Unit 13  Hydrogen


Position of hydrogen in periodic table, isotopes, preparation, properties and uses of hydrogen; physical and chemical
properties of water and heavy water; Structure, preparation, reactions and uses of hydrogen peroxide; Classification of
hydrides—ionic, covalent and interstitial; Hydrogen as a fuel.
xiv  Syllabus

Unit 14  s-Block Elements (Alkali and Alkaline Earth Metals)


Group - 1 and 2 Elements
General introduction, electronic configuration and general trends in physical and chemical properties of elements,
anomalous properties of the first element of each group, diagonal relationships.
Preparation and properties of some important compounds—sodium carbonate, sodium chloride, sodium
hydroxide and sodium hydrogen carbonate; Industrial uses of lime, limestone, Plaster of Paris and cement; Biological
significance of Na, K, Mg and Ca.

Unit 15  p-Block Elements

Group-13 to Group-18 Elements

General Introduction: Electronic configuration and general trends in physical and chemical properties of elements
across the periods and down the groups; unique behaviour of the first element in each group.

Groupwise study of the p-block elements


Group-13

Preparation, properties and uses of boron and aluminium; structure, properties and uses of borax, boric acid, diborane,
boron trifluoride, aluminium chloride and alums.

Group-14

Tendency for catenation; Structure, properties and uses of allotropes and oxides of carbon, silicon tetrachloride,
silicates, zeolites and silicones.

Group-15

Properties and uses of nitrogen and phosphorus; Allotrophic forms of phosphorus; Preparation, properties, structure
and uses of ammonia nitric acid, phosphine and phosphorus halides, (PCl3, PCl5); Structures of oxides and oxoacids of
nitrogen and phosphorus.

Group-16

Preparation, properties, structures and uses of dioxygen and ozone; Allotropic forms of sulphur; Preparation,
properties, structures and uses of sulphur dioxide, sulphuric acid (including its industrial preparation); Structures of
oxoacids of sulphur.
Group-17

Preparation, properties and uses of chlorine and hydrochloric acid; Trends in the acidic nature of hydrogen halides;
Structures of Interhalogen compounds and oxides and oxoacids of halogens.

Group-18

Occurrence and uses of noble gases; Structures of fluorides and oxides of xenon.

Unit 16  d – and f – Block Elements


Transition Elements
General introduction, electronic configuration, occurrence and characteristics, general trends in properties of the
first row transition elements—physical properties, ionization enthalpy, oxidation states, atomic radii, colour, catalytic
behaviour, magnetic properties, complex formation, interstitial compounds, alloy formation; Preparation, properties
and uses of K2 Cr2 O7 and KMnO4.
Syllabus  xv

Inner Transition Elements

Lanthanoids — Electronic configuration, oxidation states, chemical reactivity and lanthanoid contraction.

Actinoids — Electronic configuration and oxidation states.

Unit 17  Co-ordination Compounds


Introduction to co-ordination compounds, Werner’s theory; ligands, co-ordination number, denticity, chelation;
IUPAC nomenclature of mononuclear co-ordination compounds, isomerism; Bonding ­Valence bond approach and basic
ideas of Crystal field theory, colour and magnetic properties; importance of co-ordination compounds (in qualitative
analysis, extraction of metals and in biological systems).

Unit 18  Environmental Chemistry

Environmental Pollution: Atmospheric, water and soil.

Atmospheric Pollution: Tropospheric and stratospheric. .

Tropospheric Pollutants: Gaseous pollutants: Oxides of carbon, nitrogen and sulphur, hydrocarbons; their sources,
harmful effects and prevention; Green house effect and Global warming; Acid rain;

Particulate Pollutants: Smoke, dust, smog, fumes, mist; their sources, harmful effects and prevention.

Stratospheric Pollution: Formation and breakdown of ozone, depletion of ozone layer—its mechanism and effects.

Water Pollution: Major pollutants such as pathogens, organic wastes and chemical pollutants; their harmful effects
and prevention.
Soil Pollution: Major pollutants such as Pesticides (insecticides,. herbicides and fungicides), their harmful effects
and prevention.
Strategies to control environmental pollution.

SECTION—C
Organic Chemistry
Unit 19  Purification and Characterisation of Organic Compounds

Purification: Crystallization, sublimation, distillation, differential extraction and chromatography—principles and


their applications.

Qualitative Analysis: Detection of nitrogen, sulphur, phosphorus and halogens.

Quantitative Analysis (basic principles only) Estimation of carbon, hydrogen, nitrogen, halogens, sulphur,
phosphorus.
Calculations of empirical formulae and molecular formulae; Numerical problems in organic quantitative
analysis.

Unit 20  Some Basic Principles of Organic Chemistry


Tetravalency of carbon; Shapes of simple molecules—hybridization (s and p); Classification of organic compounds
based on functional groups: - C = C - , - C = C - and those containing halogens, oxygen, nitrogen and sulphur,
Homologous series; Isomerism - structural and stereoisomerism.
Nomenclature (Trivial and IUPAC)
xvi  Syllabus

Covalent Bond Fission—Homolytic and heterolytic: free radicals, carbocations and carbanions; stability of
carbocations and free radicals, electrophiles and nucleophiles.
Electronic Displacement in a Covalent Bond: Inductive effect, electromeric effect, resonance and hyperconjugation.
Common Types of Organic Reactions: Substitution, addition, elimination and rearrangement.

Unit 21  Hydrocarbons


Classification, isomerism, IUPAC nomenclature, general methods of preparation, properties and reactions.
Alkanes: Conformations: Sawhorse and Newman projections (of ethane); Mechanism of halogenation of alkanes.

Alkenes: Geometrical isomerism; Mechanism of electrophilic addition: addition of hydrogen, halogens, water,
hydrogen halides (Markownikoff’s and peroxide effect); Ozonolysis, oxidation, and polymerization.

Alkynes: acidic character; addition of hydrogen, halogens, water and hydrogen halides; polymerization.

Aromatic hydrocarbons: Nomenclature, benzene—structure and aromaticity; Mechanism of electrophilic


substitution: halogenation, nitration, Friedel–Craft’s alkylation and acylation, directive influence of functional group in
mono-substituted benzene.

Unit 22  Organic Compounds Containing Halogens


General methods of preparation, properties and reactions; Nature of C-X bond; Mechanisms of substitution reactions.
Uses/environmental effects of chloroform, iodoform, freons and DDT.

Unit 23  Organic Compounds Containing Oxygen


General methods of preparation, properties, reactions and uses.

Alcohols, Phenols and Ethers

Alcohols: Identification of primary, secondary and tertiary alcohols; mechanism of dehydration.

Phenols: Acidic nature, electrophilic substitution reactions: halogenation, nitration and sulphonation, Reimer -
Tiemann reaction.

Ethers: Structure.

Aldehyde and Ketones: Nature of carbonyl group; Nucleophilic addition to >C=O group, relative reactivities of
aldehydes and ketones; Important reactions such as—Nucleophilic addition reactions (addition of HCN, NH3 and its
derivatives), Grignard reagent; oxidation; reduction (Wolff Kishner and Clemmensen); acidity of a - hydrogen, aldol
condensation, Cannizzaro reaction, Haloform reaction; Chemical tests to distinguish between aldehydes and Ketones.

Carboxylic Acids: Acidic strength and factors affecting it.

Unit 24  Organic Compounds Containing Nitrogen


General methods of preparation, properties, reactions and uses.
Amines: Nomenclature, classification, structure basic character and identi-fication of primary, secondary and tertiary
amines and their basic character.

Diazonium Salts: Importance in synthetic organic chemistry.


Syllabus  xvii

Unit 25  Polymers


General introduction and classification of polymers, general methods of polymerization - addition and condensation,
copolymerization; Natural and synthetic rubber and vulcanization; some important polymers with emphasis on their
monomers and uses - polythene, nylon, polyester and bakelite.

Unit 26  Biomolecules


General introduction and importance of biomolecules.
Carbohydrates: Classification: aldoses and ketoses; monosaccharides (glucose and fructose), constituent
monosaccharides of oligosacchorides (sucrose, lactose, maltose) and polysaccharides (starch, cellulose, glycogen).
Proteins: Elementary Idea of a - amino acids, peptide bond, polypeptides; proteins: primary, secondary, tertiary and
quaternary structure (qualitative idea only), denaturation of proteins, enzymes.
Vitamins: Classification and functions.
Nucleic Acids: Chemical constitution of DNA and RNA.
Biological functions of Nucleic acids.

Unit 27  Chemistry in Everyday Life


Chemicals in Medicines: Analgesics, tranquilizers, antiseptics, disinfectants, antimicrobials, antifertility drugs,
antibiotics, antacids, antihistamins-their meaning and common examples.

Chemicals in Food: Preservatives, artificial sweetening agents-common examples.

Cleansing Agents: Soaps and detergents, cleansing action.

Unit 28  Principles Related to Practical Chemistry

∑ Detection of extra elements (N,S, halogens) in organic compounds; Detection of the following functional groups:
hydroxyl (alcoholic and phenolic), carbonyl (aldehyde and ketone), carboxyl and amino groups in organic
compounds.
∑ Chemistry involved in the preparation of the following:
Inorganic compounds: Mohr’s salt, potash alum.
Organic compounds: Acetanilide, p-nitroacetanilide, aniline yellow, iodoform.
∑ Chemistry involved in the titrimetric excercises - Acids bases and the use of indicators, oxalic-acid vs KMnO4,
Mohr’s salt vs KMnO4.
∑ Chemical principles involved in the qualitative salt analysis:
Cations: Pb2+, Cu2+, AI3+, Fe3+, Zn2+, Ni2+, Ca2+, Ba2+, Mg2+, NH4+.
Anions: CO32–, S2–, SO42–, NO2– , NO3– , CI – , Br–, I– .
(Insoluble salts excluded).
∑ Chemical principles involved in the following experiments:
1. Enthalpy of solution of CuSO4
2. Enthalpy of neutralization of strong acid and strong base.
3. Preparation of lyophilic and lyophobic sols.
4. Kinetic study of reaction of iodide ion with hydrogen peroxide at room temperature.
Contents
To Our Readers... v
Trend Analysis vii
About JEE Main ix
Syllabus xi
1. Some Basic Concepts of Chemistry 1.1–1.35
Section-1 Physical Quantities  1.1
Section-2 Significant Figures (or Digits)  1.7
Section-3 Atomic and Molecular Masses  1.9
Section-4 Laws of Chemical Combination  1.14
Section-5 Composition of a Solution  1.19
2. States of Matter 2.1–2.90
Unit-1 Gaseous State  2.1
Section-1 Ideal Gases  2.1
Section-2 Kinetic-Molecular Theory of Gases  2.7
Section-3 Real Gases  2.12
Unit-2 Liquid State  2.37
Unit-3 Solid State  2.43
Section-1 Crystal Systems  2.43
Section-2 Closest Packings of Atoms  2.54
Section-3 Structures of Ionic Compounds  2.64
Section-4 Imperfection in Solids  2.74
3. Atomic Structure 3.1–3.39
Section-1 Development of Structure of Atom  3.1
Section-2 Quantum-Mechanical Model of Atom  3.8
4. Chemical Bonding and Molecular Structure 4.1–4.42
Section-1 Bond Formation and VSEPR Theory  4.1
Section-2 VB and MO Theories 4.13
5. Solutions 5.1–5.42
Section-1 Composition of a Solution  5.1
Section-2 Liquid Solutions  5.7
Section-3 Colligative Properties  5.16
6. Chemical Thermodynamics 6.1–6.45
Section-1 Basic Definitions and First Law of Thermodynamics  6.1
xx  Contents

Section-2 Thermochemistry 6.9
Section-3 Criteria of Spontaneity  6.17
7. Chemical and Ionic Equilibria 7.1–7.75
Unit-1 Chemical Equilibrium  7.1
unit-2 Ionic Equilibrium  7.27
Section-1 Concepts of Acids and Bases  7.27
Section-2 The pH Scale and pH of Acid and Base Solutions  7.32
Section-3 Hydrolysis of Salts  7.39
Section-4 Buffer Solutions  7.44
Section-5 Solubility Product  7.48
Section-6 Acid-Base Indicators  7.53
8. Redox Reactions and Electrochemistry 8.1–8.58
Unit-1 Redox Reactions and Electrolysis  8.1
Section-1 Redox Reactions  8.1
Section-2 Electrolytic Cell  8.8
Unit-2 Electrolytic Conduction  8.18
Unit-3 Galvanic Cells  8.30
9. Chemical Kinetics 9.1–9.33
10. Surface Chemistry 10.1–10.9
11. Chemical Families–Periodic Properties 11.1–11.19
12. General Principles and Processes of Isolation of Metals 12.1–12.8
13. Hydrogen 13.1–13.7
14. S-Block Elements (Alkali and Alkaline Earth Metals) 14.1–14.25

The Group 1 Elements – Alkali Metals  14.1
The Group 2 Element – Alkaline Earth Metals  14.12
15. Study of the p-Block Elements (Groups 13, 14 and 15) 15.1–15.60
The Group 13 Elements – Boron Family  15.1
The Group 14 Elements – Carbon Family  15.16
The Group 15 Elements  15.35
16. Study of the p-Block Elements (Groups 16, 17 and 18) 16.1–16.48
The Group 16 Elements  16.1
The Group 17 Elements  16.22
The Group 18 Elements  16.41
17. d– and f–Block Elements 17.1–17.28
18. Coordination Chemistry and Organometallics 18.1–18.26
19. Nuclear Chemistry 19.1–19.8
20. Purification and Characterization of Organic Compounds 20.1–20.9
21. Some Basic Principles 21.1–21.46
Contents  xxi

22. Hydrocarbons 22.1–22.61


Section-1 Alkanes   22.1
Section-2 Alkenes   22.13
Section-3 Alkynes   22.31
Section-4 Benzene 22.43
Section-5 Sources of Hydrocarbons  22.58
23. Organic Compounds Containing Halogens (Haloalkanes and Haloarenes) 23.1–23.14
Section-1 Haloalkanes 23.1
Section-2 Haloarenes 23.5
24. Organic Compounds Containing Oxygen (Alcohols, Phenols, Ethers, Aldehydes,
Ketones, Carboxylic Acids and their Derivatives) 24.1–24.73
Section-1 Alcohols 24.1
Section-2 Phenols 24.16
Section-3 Ethers 24.31
Section-4 Aldehydes and Ketones  24.34
Section-5 Carboxylic Acids  24.55
25. Organic Compounds Containing Nitrogen (Cyanides, Isocyanides,
Nitrocompounds and Amines) 25.1–25.24
Section-1 Cyanides and Isocyanides  25.1
Section-2 Nitro Compounds  25.2
Section-3 Amines 25.4
26. Synthetic and Natural Polymers 26.1–26.14
27. Biomolecules and Biological Processes 27.1–27.24
28. Chemistry in Action 28.1–28.13
29. Principles Related to Practical Chemistry 29.1–29.15
Annexure AN.1–AN.15
Practice Test Paper–I PI.1–PI.3
Practice Test Paper–II PII.1–PII.4
Practice Test Paper–III PIII.1–PIII.5
Practice Test Paper–IV PIV.1–PIV.4
Practice Test Paper–V PV.1–PV.4
JEE (Main) ChemistryOffline Solved Paper—2017  JEEC.1-JEEC.8
JEE (Main) ChemistryOnline Solved Paper 1—2017  JEEC.1-JEEC.8
JEE (Main) ChemistryOnline Solved Paper 2—2017 JEEC.1-JEEC.9
1
Some Basic Concepts of
Chemistry

Section 1 Physical Quantities

In physical sciences, we commonly deal with quantities such as pressure, volume, mass, temperature, current, etc. These
quantities are known as physical quantities. A physical quantity has two components, namely, numerical value and its
unit, and is written as
Physical quantity = (Numerical value) (Unit)

The International Union of Pure and Applied Chemistry (IUPAC) has


recommended the use of seven physical quantities having their own dimensions. Their dimensions are completely
independent of one another and it is for this reason, these are known as dimensionally independent physical quantities.
These physical quantities along with their recommended symbols are given in Table 1.

Seven Base Physical Quantities


Physical quantity Symbol
Length l
Mass m
Time t
Electric current I
Thermodynamic temperature T
Amount of substance n
Luminous intensity Iv
Of the seven physical quantities, luminous intensity is not needed in physical chemistry. It is used in optical
photometry and is, therefore, included here, only for the sake of completeness.

International System of Units, commonly abbreviated as SI. The SI units of seven base physical quantities are listed
in Table 2.
Complete Chemistry—JEE Main

Seven Base Physical Quantities


Physical quantity Name of SI unit Symbol for SI unit
Length metre m
Mass kilogram kg
Time second s
Electric current ampere A
Thermodynamic temperature kelvin K
Amount of substance mole mol
Luminous intensity candela cd

The SI base units stated in Table 2 have been precisely

The metre is the length of path travelled by light in vacuum during a time interval of 1/299 792 458 of a second.
The kilogram is the unit of mass; it is equal to the mass of the international prototype of the kilogram.
This prototype is a polished cylinder of platinum-iridium alloy which was chosen for its durability and resistance to
corrosion. The cylinder is kept at the International Bureau of Weights and Measures in a suburb of Paris, France.
The second is the duration of 9 192 631 770 periods of the radiation corresponding to the transition between

of negligible cross-section, and placed 1 metre apart in vacuum, would produce between these conductors a force equal
to 2 ¥ 10–7 newton per metre of length.
The kelvin, unit of thermodynamic temperature, is the fraction 1/273.16 of the thermodynamic temperature
of the triple point of water.
The mole is the amount of substance of a system which contains as many elementary entities as there are atoms
in 0.012 kilogram of carbon-12.
The candela is the luminous intensity, in a give direction, of a source that emits monochromatic radiation of
frequency 540 ¥ 1012 hertz and that has a radiant intensity in that direction of (1/683) watt per steradian.
Of the seven base physical quantities, it is worth highlighting the physical
quantity ‘amount of substance’ along with its unit ‘mol’. This is because a basic change in our conventional way of
referring to this quantity is required. Suppose we have 0.5 mol of a substance (say, hydrogen). Conventionally, we
speak or write it as
Number of moles of hydrogen = 0.5 (or moles of hydrogen = 0.5 mol)
Now suppose we have 1 kg of hydrogen. We never speak or write it as
Number of kg of hydrogen = 1
since we are not accustomed to speak or write it like this. Instead, we say
Mass of hydrogen = 1 kg
Many such examples can be cited. For example, we never say
Number of kg of apples = 1 Number of litre of milk = 1
Number of dozen of apples = 1 Number of metre of a line = 1
A mole of a substance is a collection of 6.022 ¥ 1023 particles of that substance. It is like a dozen which is a collection
of 12 articles. When we do not refer to a collection of 12 articles as
Number of dozen of articles = 1
why should we refer to a collection of 6.022 ¥ 1023 articles as
Number of moles of articles =1?
So, some inconsistency has arisen in referring to these two quantities which basically belong to the same category.
To avoid this, IUPAC has recommended the use of the phrase ‘amount of substance’ for a physical quantity whose unit
is ‘mol’. More precisely, the word ‘amount’ is exclusively reserved whenever the quantity is to be expressed in terms
of moles. It is like using the words mass for kg, volume for m3 (or L) and length for metre. Thus, the use of the phrase
‘number of moles equal to 0.5’ should be completely replaced by ‘amount of substance equal to 0.5 mol’. It may be
mentioned that to write
Amount of substance = 0.5 g
is not acceptable as the word ‘amount’ is exclusively reserved for the unit ‘mol’ and not for ‘g’.
Some Basic Concepts of Chemistry

entities of that substance. The proportionality factor is the same for all substances and is equal to l/NA, where NA is
Avogadro constant which has a value of
NA = 6.022 ¥ 1023 mol–1
( Earlier, NA was commonly referred to as Avogadro number. But it is not a pure number as it has both a
numerical value as well as a unit. So, It is a physical quantity and is called Avogadro constant.)
Mathematically, the amount of substance is given as
n = N / NA
where N NA is Avogadro constant. For example, 3.011.5 ¥ 1023
molecules of dihydrogen will contain
Amount of dihydrogen = 3.011 ¥ 1023 / (6.022 ¥ 1023 mol–1) = 0.5 mol

Physical quantities other than base physical quantities are known as derived physical quantities. These may be expressed
in terms of base physical quantities by multiplication and division. Table 3 records some of the derived physical
quantities.
SI derived units of a few physical quantities
Physical quantity SI unit Symbol for SI unit
area (length)2 square metre m2
volume (length)3 cubic metre m3
density mass/volume kilogram per cubic metre kg m–3
speed distance/time metre per second m s–1
acceleration speed/time metre per square second m s–2
heat capacity Dq/Dt — J K–1
(l/m)(Dq/Dt) — J kg–1 K–1
molar heat capacity (l/n)(Dq/Dt) — J K–1 mol–1
amount concentration amount of substance/volume of solution mole per cubic metre mol m–3
molality amount of substance/mass of solvent mole per kilogram mol kg–1

Some physical quantities have been assigned special names and symbols. These are described in Table 4.

Special names and symbols for certain SI derived units


Physical quantity Name of SI unit Symbol for SI unit
frequency — Hertz Hz s–1
force mass ¥ accleration Newton N kg m s–2
pressure force/(length)2 Pascal Pa kg m–1 s–2 (=N m–2)
energy force ¥ length Joule J kg m2 s–2 (= N m)
power energy/time Watt W kg m2 s–3 (= J s–1)
electric charge current ¥ time Coulomb C As
electric potential difference — Volt V kg m2 s–3 A–1 (= J A–1 s–1)
electric resistance — Ohm W kg m2 s–3 A–2 (= V A–1)
electric conductance — Siemens S kg–1 m–2 s3 A2 (= A V–1 = W–1)
— Tesla T kg s–2 A–1
Complete Chemistry—JEE Main

–2
g is known as

Fraction Symbol Multiple Symbol


-1
10 deci d 10 deca da
-2 102
10 centi c hecto h
10-3 milli m 103 kilo k
10 -6
micro m 10 6
mega M
10-9 nano n 10 9
giga G
-12 1012
10 pico p tera T
10-15 femto f 1015 peta P
-18 18
10 atto a 10 exa E

unit is treated the same way as any other quantity in an algebraic operation is treated. For example, in algebra, each of
the following expressions represents one and the same thing.

=6 =3 = 1.5 ¥ 4 =2¥3 =1.5 =4


2 4 1.5
and so on.
Similarly, the following expressions represent one and the same thing,
V V
V = 25.0 cm3 = 25.0 = 1 cm3
cm3 25.0
and so on.

Note that writing an expression of the type V/cm3 = 25.0 is very convenient while writing the headings in tables and
as labels on the axes of graphs.
A physical expression should also be dimensionally correct. For example, the conversion
expression of Celsius temperature to kelvin temperature may be written as

T = qC + 273.15
This expression is numerically correct but not dimensionally as the unit of qc is °C and that of T is K. One can add
or subtract two physical quantities if they have the same unit. Thus, a correct conversion equation would be
T/K = qC/°C + 273.15
For example, for 25 °C, we would have
T/K= 25 °C/°C + 273.15 = 25 + 273.15 = 298.15 or T = 298.15 K
A few other examples are

Êhˆ Ê Aˆ E Ê pˆ D H
log ÊÁ ˆ˜ = log ÊÁ ˆ˜ -
k A Ea
; ln Á ˜ = ln Á ˜ + ; ln Á ˜ = - vap + C
Ë k∞ ¯ Ë k ∞ ¯ 2.303RT Ë h∞ ¯ Ë h∞ ¯ RT Ë p∞ ¯ RT
Note that the division by k°, h° and p° (which stand for the corresponding unit physical quantities) make the
expression within the logarithm brackets unitless.
Some Basic Concepts of Chemistry

Other examples are


pH = –log {[H+]/mol dm–3}; pK°w = –log [Kw/(mol dm–3)2}; pKa° = –log {Ka/mol dm–3}
The use of standard equilibrium constant K° (= K/(mol dm–3)Ân ) would avoid the division by units.
The conversion of one unit to another may be carried out by . It involves
the following steps.
Desired unit
Arrange the identity so as to have =1
Given unit
Multiply the given value by the above identity and simplify the expression
Convert 57.8 m into cm unit.

102 cm Ê 102 cm ˆ
(57.8 m) Á = 57.8 ¥ 102 cm
Ë 1 m ˜¯
Identify expression =1 Hence,
1m

Convert 1.5 g cm–3 in terms of kg m–3.

1 kg 1m (1.5 g) (1 kg/103 g)
Identity expressions = 1 and 2 =1 Hence, 1.5 g cm–3 = t = 1.5 ¥ 103 kg m–3
3
10 g 10 cm (1 cm3 ) (1 m/102 cm)3

unit itself.
For example, 1 kg m–3 = 1 (103 g) (102 cm)–3 = 10–3 g cm–3
It is adviseable to use the complete value of a physical quantity (= numerical value ¥ unit)
while evaluating its value from the given expression. If SI Units are used for every physical quantity, the result of the
expression will also come out in terms of SI base unit.
Volume of one mole of an ideal gas at 27 °C and 1 atm pressure.
Expression to be used V = nRT/p
Here T = (27 + 273.15) K = 300.15 K
p = 1 atm = 101. 325 ¥ 103 Pa
R = 8.314 J K–1 mol–1
(1 mol) (8.314 J K -1 mol-1 ) (300.15 K)
Hence V= = 2.463 ¥ 10–2 J Pa–1
(101.325 ¥ 103 Pa)

J Pa–1 = (kg m2 s–2) (kg m–1 s–2)–1 = m3 and thus V = 2.463 ¥ 10–2 m3 = 2.463 ¥ 10–2 (10 dm)3 = 24.63 dm3

MULTIPLE CHOICE QUESTIONS ON SECTION 1

Identify the correct choice in the following questions.


1. The symbol for SI unit kg m–1 s–2 is
(a) J (b) N (c) Pa (d) C
2. Which one of the following is the base physical quantity?
Complete Chemistry—JEE Main

3. Which of the following unit represent the joule unit?


2 s–2 (c) Pa m–3
(a) kg m (b) N m (d) C V
4. The value of Avogardo constant is
(a) 6.022 ¥ 1022 (b) 6.022 ¥ 1023 atoms (c) 6.022 ¥ 1023 mol–1 (d) 6.022 ¥ 1022 mol–1
5. Which of the following conversion units is correct?
(a) 1 Pa = 1 kg m s–2 (b) 1 J = 1 kg m2 s–2 (c) 1 C = 1 A s–1 (d) 1 N = 1 kg m–1 s2
18 is

(a) tera (b) peta (c) exa (d) atto


–15 is

(a) nano (b) pico (c) femto (d) giga


8. One gigametre stands for
(a) 106 (b) 109 m (c) 1012 m (d) 1015 m
9. Which of the following expressions is dimensionally correct?
T t
(a) = c + 273.15 (b) pH = –log [H+]
K ∞C
T Ê tF ˆ 5
(c) = Á - 32˜ ÊÁ ˆ˜ + 273.15 (d)
TF Ê 9 ˆ Ê tc ˆ
= Á ˜ Á ˜ + 32
K Ë ∞F ¯ Ë 9¯ °F Ë 5 ¯ Ë ∞C ¯

10. Which one of the followings is the correct conversion expression of 1 J?


2 s–2 (c) 1 N m–2 (d) 1 Pa m3
(a) 1 kg m (b) 1 N m

ANSWERS
1. (c) 2. (b) 3. (c) 4. (c) 5. (b) 6. (d)
7. (c) 8. (b) 9. (b) 10. (c)

HINTS AND SOLUTIONS


mass × acceleration force
1. kg m–1 s–2 = kg m s–2/m2 = = = pressure. The SI symbol of pressure is pascal (Pa)
area area
2. Electric current is the base physical quantity.
3. The unit Pa m3 represents joule.
4. Avogadro constant is 6.022 ¥ 1023 mol–1.
¥ acceleration. Its unit will be
N = (kg) (m s–2).

Pa = (kg m s–2) (m–2) = kg m–1 s–2.


¥ time. Its unit will be C = A s
¥ distance = N m = (kg m s–2)m = kg m2 s–2.
10. See Q.5
Some Basic Concepts of Chemistry

Section 2 Significant Figures (or Digits)

The numerical value of a physical quantity is determined experimentally. Due to experimental error and the limitation

include all those digits which are known with certainty, and the last digit indicates the uncertain value. The total number

cant.
3. Zeros at the end of a number without a decimal point are ambiguous. For example, 7500 may have two or

digit and ending with the digit that has uncertain value.

Number Number
0.0002 1 3.800 4
0.0038 2 3.080 4
0.0380 3 3.0805 5
0.3800 4 0.3805 4

The counting of discrete variables (such as peas, pencils, erasers, and so on) leads to an exact

An algebraic operation may involve numerical quantities of

1. In an arithmetic operation involving addition and/or subtraction, the answer should include a factor of uncertainty
equal to the maximum uncertainty present in the numbers being added and/or subtracted.

3. In an arithmetic operation involving mixed manipulations (addition, subtraction, multiplication and division),

If the digit following the last digit to be retained is more than 5, the last digit to be retained is increased by one.

If the digit following the last digit to be retained is less than 5, the last digit is left unchanged.
For
Complete Chemistry—JEE Main

digit by 1 if it is odd and leave it unchanged if it is even.


For

A number may be a small or large. To express the number in a compact manner,

N ¥ 10 n
where N is a number with a single non-zero digit to the left of the decimal point and n is an integer. For example,
150 is written as 1.50 ¥ 102
Avogadro constant as 6.022 ¥ 1023 mol–1
Planck’s constant as 6.626 ¥ 10–34 J s

16 .0 ¨ involves maximum 67 . 69 ¨ involves maximum


0 . 094 uncertainty – 2 . 113 uncertainty
+ 0 . 13 65 . 577 Æ rounded off to 65.58
16 . 224 Æ rounded off to 16.2

0 . 0 8 3 ¨ minimum significant
¥1 0 . 1 digits
0 . 8 3 8 3 Æ rounded off to 0.84

0.0154
= 0.000017440 Æ rounded off to 0.0000174.
883 In scientific notation, this number
is written as 1.74 ¥ 10–5

Let =
94 678 È 1 - 1 ˘
(2.303)(8.314) ÍÎ 298 308 ˙˚

È 94 678 ˘ È 308 - 298 ˘


=Í ˙ ÍÎ 298 ¥ 308 ˙˚ = 0.538739
Î ( 2 . 303)(8 . 314 ) ˚

The second calculation involves division and subtraction. The actual operation involves

1 1
= 0.003 36 and = 0.003 25 ; 0.003 36–0.003 25 = 0.000 11
298 308

Since the answer involves two sig


digits. Hence, the answer will be 0.54.
Some Basic Concepts of Chemistry

MULTIPLE CHOICE QUESTIONS ON SECTION 2

Identify the correct choice in the following questions.

(a) 1 (b) 2 (c) 3 (d) 4


2. The radius of a hydrogen atom is 5.29 ¥ 10 –11 m and that of a proton is 1.5 ¥10–15 m. The ratio of the radius of

(a) 3.526 ¥ 104 (b) 35.266 ¥ 103 (c) 3.5 ¥ 104 (d) 3.526 666 7 ¥ 103
3. If p = 3.141 59, then the value of 4p
(a) 0.001 964 (b) 1.96 ¥ 10–3 (c) 1.963 5 ¥ 10–3 (d) 3.526 666 7 ¥ 10–2
4. The answer to the expression 1.561 ¥ 103 – 1.80 ¥ 102 + 2.02 ¥ 104
(a) 2.16 ¥ 104 (b) 2.158 ¥ 104 (c) 2.158 1 ¥ 104 (d) 21.59 ¥ 104

(a) 3 (b) 4 (c) 5 (d) 6

ANSWERS
1. (c) 2. (c) 3. (b) 4. (a) 5. (a)

HINTS AND SOLUTIONS


123
300
-11
5.29 ¥ 10 m
2. -15
= 3.5 ¥ 104
1.5 ¥ 10 m

4. The result will carry 2 digits after decimal.

Section 3 Atomic and Molecular Masses

The IUPAC has recommended the following terms while dealing with atomic and
molecular masses.

The relative atomic mass of an element is the ratio of the average mass per

of an element to 1/12 of the mass of an atom of the nuclide carbon-12, i.e.


mass of an atom
A r= (1)
(1/12) mass of an atom of 12 C
The relative molecular mass of a compound is the ratio of the average

carbon-12, i.e.
mass of a molecule
Mr = (2)
(1/12) mass of an atom of 12 C
Complete Chemistry—JEE Main

The quantities Ar and Mr are formerly known as atomic weight and molecular weight, respectively. It may be noted
that Ar and Mr carry no units as these are simply the ratio of two masses.

The quantity (1/12) mass of an atom 12

mu, unit; u, also commonly abbreviated as amu). Hence

ma (12 C) (0.012 kg mol-1 ) /(6.022 ¥ 1023 mol-1 )


1 amu = 1 u = = = 1.661 ¥ 10–27 kg
12 12
= 1.661 ¥ 10–24 g

ma). It is simply a mass and thus has the unit of mass (i.e. g or kg). From Eq. (1), it follows that
mass of an atom, ma = Ar mu

molecular mass. It is simply a mass and thus carries the unit of mass (i.e. g or kg). For Eq. (2), it follows that
mass of a molecule, mf = Mr mu

composition is known as a molar mass. Mathematically, we write it as


m
M=
n
Since the unit of mass (m) is kg or g, and that of amount of substance (n) is mol, it follows that the unit of molar
mass is kg mol–1 or g mol–1.
If a system has N entities (atoms or molecules),
we have

For elements For Compounds


m = N ma m = N mf
n = N/NA n = N/NA
m N ma m N mf
Mm = = = N A ma = N A ( Ar mu ) = Ar ( N A mu ) Mm = = = N A mf = N A ( M r mu ) = M r ( N A mu )
n N /N A n N/N A

= Ar ( I g mol–1) = Mr ( I g mol–1)

In words, the relative atomic (or molecular) mass is the numerical value of the molar mass expressed in g mol–1.

We will have
Relative atomic mass of sodium = 23
Atomic mass of sodium = 23 u = 23 (1.66 ¥ 10–27 kg) = 3.82 ¥ 10–26 kg
Molar mass of sodium = 23 g mol–1
Relative molecular mass of carbon dioxide = 44
Molecular mass of carbon dioxide = 44 u = 44 (1.66 ¥ 10–27 kg) = 7.30 ¥ 10–26 kg
Molar mass of carbon dioxide = 44 g mol–1
Quite often, we do not specify the units of atomic, molecular and molar masses. Hence, our statements are not exact and
precise. In other words, a mere replacement of the term weight by mass without carrying the associated unit does not
complete our adoption of IUPAC recommendations. So, whenever we write or speak of atomic, molecular and molar
masses, we must state the associated units.
It may be noted that the terms gram atomic weight, gram molecular weight, gram formular weight, gram atom, gram
molecule, etc., are obsolete terms and should thus be abandoned.
Some Basic Concepts of Chemistry

An atom or a molecule in nature exists in its natural


isotopic composition. The terms mentioned above for atomic or molecular masses refer to an average value of this
natural isotopic composition. For example, natural magnesium consists of three isotopes as mentioned in the following.
24Mg ma = 23.985 u 78.99% 25Mg ma = 24.986 u 10.00%
26
Mg ma = 25.983 u 11.01%
The average atomic mass of Mg is
ma(Mg) = (0.789 9 ¥ 23.985 + 0.100 0 ¥ 24.986 + 0.110 1 ¥ 25.983) u = (18.946 + 2.498 6 + 2.861) u
= 24.306 u

As mentioned earlier, 1 mol of a substance


contains 6. 022 ¥ 1023 constituent particles (atoms or molecules or ions). This fact is expressed by Avogadrao constant
which has value of 6.022 ¥ 1023 mol–1 It is expressed by the symbol of NA.
For a given amount of substance, say n, the number of constituent particles will be given as
N = n NA
For example, The number of molecules in 0.5 mol glucose is
N = (0.5 mol) (6.022 ¥ 1023 mol–1) = 3.011 ¥ 1023

The amount (n) of a substance in a given mass (m) may be calculated


from the expression
m
n=
M
where M is the molar mass of the substance.
The molar mass of a compound is sum of molar masses
of its constituent atoms. For example, the molar mass of H2SO4 is
MH2SO4 = 2 MH + MS + 4 MO = (2 ¥ 1 + 32 + 4 ¥ 6) g mol–1 = 98 g mol–1
The per cent of H, S and O is H2SO4 may be calculated by the expression
No. of its atom × Molar mass of atom
Mass per cent of element = ¥ 100
Molar mass of the compoound
For example, in H2SO4 we have
2 ¥ 1 g mol-1
Mass per cent of H = ¥ 100 = 2.04 %
98 g mol-1

1 ¥ 32 g mol-1
Mass per cent of S = ¥ 100 = 32.65 %
98 g mol-1
4 ¥ 16 g mol-1
Mass per cent of O = ¥ 100 = 65.31 %
98 g mol-1

The chemical formula of a compound may be determined from the mass percentages of atoms present in it. The latter
are ascertained experimentally by employing appropriate procedures. With these percentages known, one can proceed
to determine the molecular formula as indicated in the following.
1. Take the mass of each element equal to its mass percentage and divide this by the corresponding molar mass
(if the mass is taken in grams) or molecular mass (if the mass is taken in terms of atomic mass unit). This gives
the relative amounts (if mass is taken in gram) or number of atoms (if mass is taken in atomic mass unit) of
different elements present in the compound.
Complete Chemistry—JEE Main

2. Divide the amounts (or number of atoms) of elements by the smallest amount (or number of atoms) to give a
simple relative ratio of atoms present in the compound.
3. If the relative ratio of atoms involves noninteger(s), then multiply all the simple ratios by a suitable number to
get a whole number ratio for each element.
4. An empirical formula is written by taking as many atoms as given by the whole number ratio.
The molecular formula represents the actual number of atoms of each element present in a molecule of the compound.
The molecular formula is either the same as the empirical formula or a simple multiple of the empirical formula, i.e.,
Molecular formula = (Empirical formula)n
where n = 1, 2, 3 .... The value of n is equal to the ratio of molar mass of the compound and molar empirical formula
mass, i.e.
Molar mass
n=
Molar empirical formula mass

–1.

To determine empirical formula from the given data, we proceed as follows.


Element Per cent Mass of Amount of atom Simple relative ratio of Relative whole number
element taken atoms of ratio of atoms
47.5 g 3.95 mol
C 47.5 47.5 g = 3.95 mol = 2.80 2.8 ¥ 5 = 14
12 g mol -1 1.41 mol

2.54 g 2.54 mol


H 2.54 2.54 g -1
= 2.54 mol = 1.80 1.8 ¥ 5 = 9
1 g mol 1.41 mol

50.0 g 1.41 mol


Cl 50.0 50.0 g -1
= 1.41 mol =1 1¥5=5
35.5 g mol 1.41 mol

Hence, the required empirical formula is C14H9Cl5.


The above calculations in short may be carried out as follows.
47.5 2.54 50.0 3.95 2.54 1.41
: : 3.95 : 2.54 : 1.41 :: : : :: 2.80 : 1.80 : 1
12 1 35.5 1.41 1.41 1.41
¥ ¥ ¥
Empirical Formula molar mass = (14 ¥ 12 + 9 ¥ 1 + 5 ¥ 35.5) g mol–1 = 354.5 g mol–1
Since, the empirical formula molar mass is the same as the molar mass, the molecular formula of DDT is C14H9Cl5.
A gaseous hydrocarbon gives upon combustion 3.08 g CO2 and 0.72 g H2O. Determine its empirical
formula.
MC Ê MC ˆ 1 mCO2
Mass of C in the mass mCO2 = ¥ mCO2 Amount of C in mCO2 = Á ¥ mCO2 ˜ =
M CO2 Ë M CO2 ¯ M C M CO2
Similarly,
Ê 2M H ˆ 1 2mH 2O
Amount of H in the mass mH2O = Á ¥ mH 2O ˜ =
Ë M H2O ¯ M H M H2O
For the given data, we have
3.08 g 2 ¥ 0.72 g
Amount of C = = 0.07 mol Amount of H = = 0.08 mol
44 g mol-1 18 g mol-1
Empirical Formula C7H8.
Some Basic Concepts of Chemistry

MULTIPLE CHOICE QUESTIONS ON SECTION 3

Identify the correct choice in the following questions.

(a) 1.661 ¥ 10–27 g (b) 1.661 ¥ 10–27 kg (c) 1.661 ¥ 10–25 g (d) 1.661 ¥ 10–25 kg
2. The relative atomic mass of sodium is 23. Which of the following statements is correct about sodium?
(a) Atomic mass of sodium is 23 u.
(b) Atomic mass of sodium is 3.82 ¥ 10–26 kg.
(c) Molar mass of sodium is 23 g mol–1
(d) The number of atoms in 24 kg of sodium is 6.022 ¥ 1023.
3. Chlorine exists as 35Cl (atomic mass = 34.9688 u; 75.77%) and 37Cl (atomic mass = 36.965 9 u; 24.23%). The
atomic mass of natural existing chlorine is
(a) 35.452 7 u (b) 36.423 6 u (c) 34.123 5 u (d) 32.345 8 u
4. 3.9 g of an organic compound on combustion gives 13.2 g of CO2 and 2.7 g of H2O. The empirical formula of the
compound is
(a) CH (b) CH2 (c) C2H4 (d) C6H6
5. Which one of the following has maximum number of atoms?
(a) 16 g of C (b) 20 g of Na (c) 45 g of S (d) 15 g of N
6. An organic compound contains 20.0% C, 6.66% H, 47.33% N and the rest was oxygen. Its molar mass is 60 g
mol–1. The molecular formula of the compound is
(a) CH4N2O (b) CH2NO (c) C2H6NO (d) CH18NO
7. Mass per cent of Na in Na2CO3 is
(a) 21.52% (b) 31.20% (c) 38.20% (d) 43.40%
–1) and 30.1 mass% of oxygen, its molecular
8. If an iron oxide has 69.9 mass% of Fe (molar mass = 56. 0 g mol
formula will be
(a) FeO (b) Fe2O3 (c) Fe3O4 (d) Fe2O6
9. The number of oxygen atoms in 24.9 g of CuSO4◊5H2O is (molar mass of Cu = 63 g mol–1)
(a) 2.41 ¥ 1024 (b) 3.01 ¥ 1024 (c) 5.42 ¥ 1023 (d) 5.42 ¥ 1024
10. A compound contains 11.99% N, 13.70% O, 9.25% B and 65.06% F. Its empirical formula is (molar mass of B is
10.8 g mol–1)
(a) NOBF2 (b) NOBF4 (c) N2OF2 (d) NO2F2

ANSWERS
1. (b) 2. (d) 3. (a) 4. (a) 5. (c) 6. (a)
7. (d) 8. (b) 9. (c) 10. (b)

HINTS AND SOLUTIONS


ma (12 C) M (12 C) / N A (0.012 kg mol-1 ) /(6.022 ¥ 1023 mol-1 )
1. mu = = = = 1.661 ¥ 10–27 kg
12 12 12
2. The number of atoms in 23 g of Na will be equal to 6.022 ¥ 1023.
Complete Chemistry—JEE Main

3. ma = (0.757 7 ¥ 34.968 8 + 0.242 3 ¥ 36.965 9) u = 35.452 7 u


13.2 2 ¥ 7.2
44 15
5. Larger the amount of an element, larger the number of atoms.
n(C) = m(C) / M(C) = 16 g/12 g mol–1 = 1. 333 mol n(Na) = m(Na) / M(Na) = 20 g/ 23 g mol–1 = 0.870 mol
n(S) = m(S) / M(S) = 45 g/ 32 g mol–1 = 1.406 mol n(N) = m(N)/ M(N) = 15 g/14 g mol–1 = 1.071 mol
20 47.33 26.01 6.66
: : :
12 14 16 1
Empirical formula CN2OH4
Molar empirical mass is 60 g mol–1 (same as the given molar mass). Hence, Molecular formula is CN2OH4.
2M Na 46
7. Mass per cent of Na = ¥ 100 = ¥ 100% = 43.40%
M Na 2CO3 106
69.9 30.1 1.881
: Molecular formula Fe2O3
56.0 16.0 1.248
9. Molar mass of CuSO4·5H2O = (63 + 32 + 4 ¥ 16 + 5 ¥ 18) g mol–1 = 249 g mol–1
m 24.9 g
Amount of CuSO4·5H2O in the given mass is n = = = 0.1 mol
M 249 g mol-1
Number of oxygen atoms = 9 ¥ (0.1 mol) (6.022 ¥ 1023 mol–1 ) = 5.42 ¥ 1023
11.99 13.7 9.25 65.06
: : :
14 16 10.8 19
4

Section 4 Laws of Chemical Combination

Based on the study of chemical reactions the following laws have been established.
( ) This law states that the mass is conserved
in a chemical reaction.
This law states that all pure samples (drawn from
different sources) of the same compound contain the same elements combined in the same proportion by mass.
This law states that the two elements, A and B

versa) are in the ratio of small whole numbers.


This law was established by Gay Lussac. According to this law, the volume of
reactants and products involved in gaseous reactions are related to each other by small integers, provided the volumes
are measured at the same temperature and pressure conditions.
For example, in the reaction 2H2(g) + O2(g) Æ 2H2O(g), we have
2 Volume of hydrogen combines with 1 volume of oxygen to give 2 volume of water.
This law follows from the fact that under similar conditions of temperature and pressure, equal volumes of all gases
contain equal number of particle (Avogadro’s hypothesis). The volume occupied by 1 mol gas at standard condition of
temperature (0 °C) and pressure (1 atm), abbreviated as STP, is 22.4 L. Thus, in the above example, 2 ¥ 22.4 L of H2
combines of 22.4 L of O2 to give 2 ¥ 22.4 L of H2O(g).
Some Basic Concepts of Chemistry

The laws of chemical combination may be explained on the basis of Dalton

1. Matter is composed of atoms which cannot be created or destroyed.


2. All the atoms of one element are alike (i.e. they have the same size, shape and mass) but are different from
those of other element.
3. In compound, atoms combine together in the ratio of small whole numbers and are held by chemical forces.
The smallest entity of a compound is known as a molecule.

The branch of chemistry which deals with mass relationships in chemical reactions is called stoichiometry. This branch

1. Conservation of mass 2. The relative masses of atoms 3. The concept of the mole
According to the law of conservation of mass, the total mass of the products formed in a chemical equation is
equal to the total mass of reactants that are consumed during the progress of the equation.
The law of conservation of mass implies that the number of atoms of each kind must be the same on both sides of
a chemical equation. An equation satisfying this criterion is known as a balanced chemical equation.
A balanced chemical equation provides quantitative information regarding the consumption of reactants and
creation of products.
The numbers which appear before the chemical symbols and which balance the equation (with the understanding

proportional to the number of molecules or the amounts of the constituents that change during the reaction. For
a general case
nAA+nBB Æ nCC + nDD (1)
where nA,nB,nC and nD
The progress of a reaction is described in terms of a physical quantity known as (symbol, x,
pronounced xi)’. It is expressed as
DnA Dn Dn Dn (2)
- =- B = C = D =x
nA nB nC nD
where Dn represents the change in the amount of the substance. The negative and positive signs in the above expressions
are due to the fact that DnA and DnB are negative (i.e. their amounts decrease with the progress of reaction) and DnC
and DnD are postive (i.e. their amounts increase with the progress of reaction). The unit of extent of reaction (x) is mol.

Decrease in the amount of A is – DnA = nA mol Increase in the amount of C is DnC = nC mol
Decrease in the amount of B is – DnB = nB mol Increase in the amount of D is DnD = nD mol
In other words, we say that nA mol of A on reacting with nB mol of B gives nC mol of C and nD mol of D.
For example, for the reaction Pb(NO3)2 + 2KI Æ PbI2 + 2KNO3, we have
1 mol of Pb(NO3)2 on reacting with 2 mol of KI gives 1 mol of PbI2 and 2 mol of KNO3.
In terms of masses consumed/produced, in the reaction
Pb(NO3)2 + 2KI Æ PbI2 + 2KNO3
Molar mass 331.2 g mol–1 166 g mol–1 461.0 g mol–1 101.1 g mol–1
we have 331.2 g of Pb(NO3)2 on reacting with 2 ¥ 166 g of KI gives 461.0 g of PbI2 and 2 ¥ 101.1 g of KNO3.
In terms of molecules, we have
1 molecule of Pb(NO3)2 on reacting 2 molecules of KI gives 1 molecule of PbI2 and 2 molecules of KNO3
In terms of molecular masses, we have
331.2 u of Pb(NO3)2 on reacting with 2 ¥ 166 u of KI gives 461.0 u of PbI2 and 2 ¥ 101.1 u of KNO3.
A given reaction may be initiated with any amounts of reactants, but the consumption of reactants
and production of products will be governed by the relation similar to that given by Eq. (2), that is, the relative amounts
Complete Chemistry—JEE Main

appeared in the balanced chemical equation.


As the reaction proceeds, the amounts of reactants continue to decrease. The reaction continues to proceed till the
amount of one of the reactants is exhausted. This reactant is known as .
To determine the limiting reagent, we proceed as follows.
Determine the maximum value of extent of reaction with the initial amount of each of the reactants.
Initial amount of the reactant
xmax =
Corresponding stoichiomeetric coefficient
. The amounts of products
formed will be governed by this limited reagent. For example,
Let the reaction 2NaOH + H2SO4 Æ Na2SO4 + 2H2O be started with 100 g each of NaOH and H2SO4. We will have
Reactants NaOH H2SO4
Initial mass, m0 100 g 100 g
Molar mass, M 40 g mol–1 98 g mol–1
m0 100 100
Initial amounts, n0 = mol = 2.5 mol mol = 1.02 mol
M 40 98
n0 2.5 mol 1.02 mol
xmax = = 1.25 mol = 1.02 mol
n 2 1
Since, the extent of reaction is smaller for H2SO4, this reagent will act as limiting reagent. The amounts of products,
Na2SO4 and H2O, will be decided by this reagent as its amount is exhausted earlier. Thus, we will have
Amount of NaOH unreacted = (n0 – nxmax) = (2.50 – 2 ¥ 1.02)mol = 0.46 mol
Amount of Na2SO4 = nxmax = (1) (1.02 mol) = 1.02 mol
Amount of H2O = nxmax = (2) (1.02 mol) = 2.04 mol

MULTIPLE CHOICE QUESTIONS ON SECTION 4

Identify the correct choice in the following questions.

(a) Lavoisier (b) Proust (c) Dalton (b) Gay Lussac


2. For the chemical reaction nAA + nBB ÆnCC + nDD, the extent of reaction is given by the expression
Dn Dn Dn Dn Dn Dn Dn Dn
(a) x = A = C (b) x = A = - C (c) x = - A = C (d) x = - A = - C
nA nC nA nC n A n C nA nC
3. For the reaction 2A(g) + 3B(g) Æ 4C(g) + 5D(g)
n0 1.5 mol 2.0 mol
the amounts of A, B, C and D when the reaction has proceeded to the extent = 0.15 mol, respectively, are
(a) 1.0 mol, 1.25 mol, 1.0 mol and 1.25 mol (b) 1.2 mol, 1.55 mol, 0.60 mol and 0.75 mol
(c) 0.5 mol, 0.5 mol, 2.0 mol and 2.5 mol (d) 0.9 mol, 1.1 mol, 1.2 mol and 1.5 mol
4. Consider the reaction N2(g) + 3H2(g) Æ 2NH3(g)
n0 1.5 mol 1.5 mol
With the progress of reaction it is found that there is a formation of 0.8 mol of NH3. At this stage, the extent of
reaction is
(a) 0.2 mol (b) 0.3 mol (c) 0.4 mol (d) 0.5 mol
5. For the reaction N2(g) + 3H2(g) Æ 2NH3(g)
m0 1400 g 250 g
Some Basic Concepts of Chemistry

which of the following statement is correct?


(a) N2(g) acts as a limiting reagent
(b) H2(g) acts as a limiting reagent
(c) The extent of reaction when the reaction is over is 41.67 mol
(d) At the end of reaction, the mass of NH3 formed is 1416.78 g
6. In the reaction N2H4 + 3O2 Æ 2NO2 + 2H2O, the mass of O2 required to combine with 745 g of N2H4 will be
(a) 2120 g (b) 2235 g (c) 2436 g (d) 2510 g
7. A mixture of 50.0 g of S and 100.0 g Cl2 reacts of form S2Cl2. The mass of S2Cl2 formed will be
(a) 150.0 g (b) 105.5 g (c) 121.0 g (d) 135.1 g
8. In the reaction Fe2O3(s) + 3C(s) Æ 2 Fe(s) + 3CO(g), 453 kg of iron was obtained from 752 kg of a sample
–1)
of Fe2O3. The perentage of Fe2O3
(a) 75% (b) 80% (c) 86% (d) 92%
9. In the reaction 2NH4Cl(s) + Ca(OH)2(s) Æ CaCl2(s) + 2NH3(g) + 2H2O(g), the mass of NH3 formed by heating
–1)
a mixture containing 10.0 g each of NH4Cl and Ca(OH)2
(a) 1.5 g (b) 2.5 g (c) 3.2 g (d) 4.6 g
10. 1.8 g of Mg is burnt in a closed vessel which contains 0.8 g of oxygen. Which of the following statements is
correct? Given ; M(Mg) = 24.3 g mol–1.
(a) 0.05 mol of MgO is formed
(b) 0.8 g of Mg is left behind
(c) Oxygen is completely used in the reaction
(d) The extent of reaction at the completion of reaction is 0.025 mol.

ANSWERS
1. (a) 2. (c) 3. (b) 4. (c) 5. (a) 6. (b)
7. (b) 8. (c) 9. (c) 10. (b)

HINTS AND SOLUTIONS


1. Law of conservation of mass was established by Lavoisier.
Dn Dn
2. The extent of reaction is x=- A = C
nA nC
3. We have
Dn
- A = x fi –DnA = nAx = 2 ¥ 0.15 mol = 0.30 mol. Hence nA = n0,A + DnA = 1.5 mol – 0.30 mol = 1.20 mol
nA
Dn
- B = x fi –DnB = nBx = 3 ¥ 0.15 mol = 0.45 mol. nB = n0,B + DnB = 2.0 mol – 0.45 mol = 1.55 mol
nB
nC = DnC = nCx = 4 ¥ 0.15 mol = 0.60 mol and nD = DnD = nDx = 5 ¥ 0.15 mol = 0.75 mol
DnNH3 0.8 mol
4. We have x= = = 0.4 mol
n NH3 2
m0 ( N 2 ) 1400 g m0 (H 2 ) 250 g
5. n0(N2) = = = 50.0 mol n0(H2) = = = 125.0 mol
M ( N 2 ) 28 g mol-1 M (H 2 ) 2 g mol-1
n0 ( N 2 ) 50.0 mol n0 (H 2 ) 125.0 mol
xmax (N2) = = = 50.0 mol xmax(H2) = = = 41.67 mol
n (N2 ) 1 n (H 2 ) 3
Since xmax(H2) < xmax(N2), H2 acts as a limiting agent.
Complete Chemistry—JEE Main

The extent of reaction when the reaction is over will be xmax(H2), i.e. 41.67 mol
DnNH3 = nNH3 xmax = 2 ¥41.67 mol = 83.34 mol
mNH3 = (DnNH3) (MNH3) = (83.34 mol) (17 g mol–1) = 1416.78 g
6. We have N2H4 + 3O2 Æ 2NO2 + 2H2O
32 g 96 g
Ê 96 g ˆ
Ë 32 g ˜¯
Mass of O2 required = Á (745 g) = 2235 g

7. The reaction is 2S + Cl2 Æ S2Cl2


n0 50.0 g 100.0 g

50.0 g 100.0 g
n0 (S) = = 1.5625 mol n0 (Cl2 ) = = 1.4084 mol
32 g mol-1 71.0 g mol-1

n0 (S) 1.5625 mol n0 (Cl2 ) 1.4084 mol


xmax (S) = = = 0.7812 mol xmax (Cl2 ) = = = 1.4084 mol
n (S) 2 n (Cl2 ) 1
S acts as limiting reagent. Hence
Dn(S2Cl2) = n(S2Cl2) xmax(S) = (1)(0.7812 mol) = 0.7812 mol
Dm(S2Cl2) = Dn(S2Cl2) M(S2Cl2) = (0.7812 mol)(135 g mol–1) = 105.5 g
8. We have Fe2O3(s) + 3C(s) Æ 2Fe(s) + 3CO(g)
Molar mass 160 g mol–1 56 g mol–1
2 ¥ 56 g of Fe(s) will be obtained from 160 g of Fe2O3. Mass of Fe2O3 from which 453 kg of Fe is obtained will
be
160 g
m= ¥ 453 kg = 647.14 kg
2 ¥ 56 g
647.14 kg
Hence, Per cent purity of sample = ¥ 100 = 86%
752 kg
m NH 4Cl / M NH 4Cl 10.0 g / 53.5 g mol-1
9. We have xmax ( NH 4 Cl) = = = 0.0935 mol
n NH 4Cl 2
mCa(OH)2 / M Ca(OH)2 10 g / 74 g mol-1
xmax (Ca (OH)2 ) = = = 0.1351 mol
n Ca(OH)2 1
Since xmax(NH4Cl) < xmax (Ca(OH)2), the limiting reagent is NH4Cl.
DnNH3 = nNH3 xmax(NH4Cl) = 2 ¥ 0.0935 mol = 0.1870 mol
mNH3 = DnNH3 MNH3 = (0.1870 mol) (17 g mol–1) = 3.18 g
10. The reaction is 2Mg + O2 Æ 2MgO
mMg / M Mg 1.8 g / 24.3 g mol-1
xmax (Mg) = = = 0.037 mol
n Mg 2

0.8 g / 32 g mol-1
xmax (O 2 ) = = 0.025 mol
1
Since xmax(O2) < xmax(Mg), the limiting reagent is oxygen.
DnMgO = nMgOxmax(O2) = 2(0.025 mol) = 0.05 mol
Some Basic Concepts of Chemistry

Ê 2M Mg ˆ 48.6
Mass of Mg consumed = Á ˜ mO2 = 32 ¥ 0.8 g = 1.22 g
Ë O2 ¯
M

Mass of Mg left behind = 1.8 g – 1.22 g = 0.58 g

Section 5 Composition of a Solution

Most of Chemical reactions occur in solution. It is important to know their concentrations. Some of the terms used in

Mass of solute
Mass percentage of solute = ¥ 100
Mass of solution
A solution contains 15 g of sucrose in 100 g water. Calculate the mass percentage of sucrose.
Mass of solute, m2 = 15 g Mass of solvent, m1 = 100 g
Mass of solution, m = m1 + m2 = 115 g
m2 Ê 15 g ˆ
Mass percentage of sucrose = ¥ 100 = Á ¥ 100 = 13.04%
m1 + m2 Ë 115 g ˜¯
Throughout, the subscripts 1 and 2 represent solvent and solute, respectively.

Amount of solute n2
Amount fraction of solute = =
Amount of (solvent + solute) n1 + n2
Amount of solvent n1
Amount fraction of solvent = =
Amount of (solvent + solute) n1 + n2
The sum of amount fractions of solute and solvent is equal to 1.
Calculate the amount of fractions of solute and solvent containing 18.0 g of glucose (molar mass = 180
g mol–1) and 90.0 g of water.

m2 18.0 g m1 90.0 g
Amount of solute, n2 = = = 0.10 mol Amount of solvent, n1 = = = 5.00 mol
M 2 180 g mol-1 M1 18.0 g mol-1
n2 0.10 mol
Amount fraction of glucose, 2=
= = 0.020
n1 + n2 (5.00 + 0.10) mol

Amount fraction of water, 1 =1– 2 = 1 – 0.020 = 0.980

Amount of solute n2
Molarity = i.e. M=
Volume of solution in dm3 V
Since the unit of amount of solute is mol, the unit of molarity is mol dm–3, i.e. mol L–1. This unit is commonly
abbreviated by the symbol M (roman style) and is spelled as molar.
Molarity is temperature dependent as the volume of solution varies with temperature.
Complete Chemistry—JEE Main

Calculate the molarity of glucose containing 18 g of glucose (molar mass = 180 g mol–1) in 90 mL of an
aqueous solution.
m2 18 g
Amount of glucose, n2 = = = 0.1 mol Volume of solution, V = 90 mL = 0. 090 L
M 2 180 g mol-1
n2 0.1 mol
Molarity of glucose, M = = = 1.1 mol L–1
V 0.090 L

Amount of solute
Molality =
Mass of solvent in kg
Since the unit of amount of solute is mol, the unit of molality is mol kg–1. This unit is spelled as molal.
The molality is temperature independent quantity.
10.6 g of sodium carbonate (molar mass = 106 g mol–1) is present in 100 mL of aqueous solution. If
density of solution is 1.018 g mL–1, calculate the molality of sodium carbonate in solution.
n 10.6 g
Amount of solute, n2 = 2 = = 0.10 mol
M 2 106 g mol-1
Mass of solution, m = rV = (1.018 g mL–1)(100 mL) = 101.8 g
Mass of solvent, m1 = mass of (solution – solute) = 101.8 g –10.6 g = 91.2 g = 91.2 ¥ 10–3 kg
n2 0.1 mol
Molality of sodium carbonate, m = = = 1.096 mol kg–1
m1 91.2 ¥ 10-3 kg

MULTIPLE CHOICE QUESTIONS ON SECTION 5

Identify the correct choice in the following questions.


1. The mass of sodium carbonate (Na2CO3) to prepare 400 mL of 0.275 molar aqueous solution is
(a) 5.83 g (b) 11.66 g (c) 17.49 g (d) 18.20 g
2. If the density of CH3OH is 0.80 kg L–1, the volume of methanol to prepare 2.5 L of 0.25 M aqueous solution is
(a) 25.0 mL (b) 32.0 mL (c) 45.0 mL (d) 56.0 mL
3. The mass of CaCO3(s) that reacts completely with 50 mL of 0.75 M HCl is
(a) 3.750 g (b) 2.788 g (c) 2.020 g (d) 1.875 g
4. The mass of ethanol (molar mass = 46 g mol–1) to be added to 1.0 kg of water so as to have its amount fraction
equal to 0.2 is
(a) 319.5 g (b) 432.1g (c) 638.9 g (d) 719.3 g
5. The density of a 2.0 M solution of acetic acid in water is 1.01 g cm–3. The molality of acetic acid in solution is
(a) 2.12 mol kg–1 (b) 2.25 mol kg–1 (c) 2.50 mol kg–1 (d) 3.0 mol kg–1
6. A solution of acetic acid has molarity equal to 1.35 M and molality equal to 1.45 mol kg–1. The density of solution
will be
(a) 1.251 g mL–1 (b) 1.125 g mL–1 (c) 1.012 g mL–1 (d) 0.994 g mL–1
7. The density of a 3 molal solution of sodium thiosulphate (Na2S2O3·5H2O) solution is 1.25 g cm–3. The percentage
by mass of sodium thiosulphate is
(a) 35.25% (b) 39.20% (c) 42.67% (d) 46.67%
–3. If the density of solution is 1.14 g cm–3, the molality
8. The molarity of a sulphuric acid solution is 2.32 mol dm
of the solution will be
(a) 2.54 mol kg–1 (b) 2.25 mol kg–1 (c) 2.62 mol kg–1 (c) 1.98 mol kg–1
Some Basic Concepts of Chemistry

9. If 20.0 cm3 of 1.0 M CaCl2 and 60.0 cm3 of 0.20 M CaCl2 are mixed, the molarity of the resultant solution is
(a) 0.80 M (b) 0.60 M (c) 0.40 M (d) 0.20 M
10. The concentration of ethanol in the solution called 86-proof vodka is 6.5 M. If the density of the solution is
0.95 g cm–3, the amount fraction of ethanol in vodka is
(a) 0.304 (b) 0.252 (c) 0.205 (d) 0.152

ANSWERS
1. (b) 2. (a) 3. (d) 4. (c) 5. (b) 6. (c)
7. (c) 8. (a) 9. (c) 10. (d)

HINTS AND SOLUTIONS


1. Since molarity, M = n2/V, we have
Amount of Na2CO3 required, n2 = MV = (0.275 mol L–1) (0.40 L) = 0.11 mol
Mass of Na2CO3 required, m = n2 M2 = (0.11 mol) (106 g mol–1) = 11.66 g
2. Since molarity, M = n2/V, we have
Amount of CH3OH required, n2 = MV = (0.25 mol L–1) (2.5 L) = 0.625 mol
Mass of CH3OH required, m2 = n2M2 = (0.625 mol) (32 g mol–1) = 20.0 g = 20.0 ¥ 10–3 kg
m2 20.0 ¥ 10-3 kg
Volume of CH3OH required, V = = = 0.025 L = 25.0 mL
r 0.80 kg L-1
3. The reaction is CaCO3 + 2HCl Æ CaCl2 + CO2 + H2O
The amount of HCl in 50 mL (= 0.050 L) of 0.75 M solution is
n = MV = (0.75 mol L–1) (0.050 L) = 0.0375 mol
3 will be 0.0375 mol/2 = 0.01875 mol
Mass CaCO3 required will be m = nM = (0.01875 mol) (100 g mol–1) = 1.875 g.
2 = n2/(n1 + n2)

Hence, 2 (n1 + n2) = n2 or 2n 1 = (1 – 2)n2 or 2n 1 = 1n 2 or n2 = 2n 1 1

m1 1000 g
Now, n1 = = = 55.56 mol . Hence, n2 = (0.2) (55.56 mol)/(0.8) = 13.89 mol
M1 18 g mol-1
Mass of ethanol required is m2 = n2M2 = (13.89 mol) (46 g mol–1) = 638.9 g
5. For 2.0 M solution, we will have
Amount of acetic acid, n2 = 2.0 mol Volume of solution, V = 1000 cm3
Mass of solution, m = rV = (1.01 g cm ) (1000 cm3) = 1010 g
–3

Mass of acetic acid, m2 = n2M2 = (2.0 mol) (60 g mol–1) = 120 g


Mass of water in 1010 g of solution m1 = 1010 g – 120 g = 890 g = 0.890 kg
n2 2.0 mol
Molality of acetic acid = = = 2.25 mol kg–1
m1 0.890 kg
6. For 1.35 M solution, we will have n2 = 1.35 mol and V = 1000 mL
–1
The solution is also 1.45 mol kg . The mass of solvent to have 1.35 mol of solute will be
Ê 1.35 mol ˆ
m1 = Á = 0.93103 kg = 931.03 g
Ë 1.45 mol kg -1 ˜¯
The mass of solution will be m = m1 + n2M2 = 931.03 g + (1.35 mol) (60 g mol–1) = 1012.03 g
m 1012.03 g
The density of the solution will be r= = = 1.012 g mL-1
V 1000 mL
Complete Chemistry—JEE Main

7. Molar mass of Na2S2O3·5H2O is M = (2 ¥ 23 + 2 ¥ 32 + 3 ¥ 16 + 5 ¥ 18) g mol–1 = 248 g mol –1


The mass of 3 mol of Na2S2O3·5H2O is m2 = (3 mol) (248 g mol–1) = 744 g
The mass of solution will be m = m1 + m2 = 1000 g + 744 g = 1744 g
m2 Ê 744 g ˆ
¥ 100 = Á ¥ 100 = 42.67 %
Ë 1744 g ˜¯
Mass percentage of sodium thiosulphate =
m
8. For 1 L of solution, we have n2 = 2.32 mol and V = 1000 cm3
Mass of solution, m = Vr = (1000 cm3) (1.14 g cm–3) = 1140 g
Mass of sulphuric acid, m2 = n2M2 = (2.32 mol) (98 g mol–1) = 227.36 g
Mass of solvent, m1 = m – m2 = 1140 g – 227.36 g = 912.64 g = 0.913 kg
n2 2.32 mol
Molality of solution = = = 2.54 mol kg–1
m1 0.913 kg
9. The molarity of the resultant solution will be
M1V1 + M 2V2 (1.0 M )(20 cm3 ) + (0.20 M )(60.0 cm3 ) 20 + 12
M= = = M = 0.40 M
V1 + V2 (20 cm + 60 cm )
3 3
80
10. For 1 L of vodka, we have n2 = 6.5 mol and V = 1000 cm3
Mass of solution, m = Vr = (1000 cm3) (0.95 g cm–3) = 950 g
Mass of ethanol, m2 = n2M2 = (6.5 mol) (46 g mol–1) = 299 g
Mass of water, m1 = m – m2 = 950 g – 299 g = 651 g
m 651 g
Amount of water, n1 = 1 = = 36.167 mol
M1 18 g mol-1
n2 6.5
Amount fraction of ethanol, 2 = = = 0.152
n1 + n2 (36.167 + 6.5)
Some Basic Concepts of Chemistry

MULTIPLE CHOICE QUESTIONS FOR THE ENTIRE CHAPTER

Identify the correct choice in the following questions.

1. The number of atoms present in 0.05 g of water is


(a) 1.67 ¥ 1023 (b) 1.67 ¥ 1022 (c) 5.02 ¥ 1021 (d) 1.67 ¥ 1021
2. The value of Avogadro constant is
(a) 6.022 ¥ 1023 (b) 6.022 ¥ 1023 mol–1 (c) 6.022 ¥ 1023 mol (d) 6.022 ¥ 10–23 mol-1
3. A certain compound has the molecular formula X4O6. If 10.0 g of the compound contains 5.62 g of X, the atomic
mass of X is
(a) 62.0 amu (b) 48.0 amu (c) 32.0 amu (d) 30.8 amu
4. Select the quantity of NO2
(a) 100 amu (b) 1.0 ¥ 10–3 g (c) 7.0 ¥ 1022 molecules (d) 8.0 ¥ 10–1 mol
5. The atomic mass unit is equal to
(a) 1.66 ¥ 10–24 kg (b) 1.66 ¥ 10–25 kg (c) 1.66 ¥ 10–26 kg (d) 1.66 ¥ 10–27 kg
6. Boron occurs in two varieties, namely, 10 11

atomic mass of naturally occurring element is reported as 10.82 amu. The per cent of 10B in this naturally occurring
boron is
(a) 10 (b) 19 (c) 29 (d) 35
7 6
7. Lithium occurs in two isotopes, namely,
exists 7.4% of 6Li in naturally occurring lithium, then its atomic mass will be
(a) 6.2 amu (b) 6.5 amu (c) 6.94 amu (d) 7.2 amu
8. In SI units, the atomic mass unit is represented by the symbol
(a) g (b) kg (c) u (d) mg
9. If ml is the mass of 2 neutrons + 2 protons + 2 electrons and m2 is the mass of an a-particle + 2 electrons, then
(a) m1 > m2 (b) m1 < m2
(c) ml = m2 (d) m1> may > or < m2 depending of its physical state.
10. The numerical values of molar mass and relative molar mass are identical when the former is expressed in
(a) kg mol–1 (b) g mol–1 (c) mg mol–1 (d) cg mol–1

11. Which of the following expressions is dimensionally correct?


(a) T = t + 273.15 (b) T/K = t/°C + 273.15 (c) T = t + 273.15 K (d) T = t + 273.15 °C
12. The Celsius and Fahrenheit temperatures corresponding to 64.15 K respectively are
(a) –209 °C, –334.2 °F (b) –20.9 °C, –34.4 °F (c) 209 °C, 344.2 °F (d) 20.9 °C, 34.4 °F
13. Which of the following expressions is not dimensionally correct?
(a) h = pr4pt/8lV (b) m = ItM/F |ne |
2 2 2 2
(c) E = –2p m(e /4pe0) /n h 2 (d) Kc = Kp(RT)Dvg
where the various symbols have their usual meanings.
14. Which of the following conversion units is correct?
(a) J Pa–1 ∫ m3 (b) (J s2 kg–1)1/2 ∫ m (c) J kg–1 m–1 ∫ m s–1 (d) J1/2 kg–1/2 m–1 = Hz
15. In the IUPAC recommendations, the number of dimensionally independent physical quantities is
(a) 1 (b) 3 (c) 5 (d) 7
16. The unit of ‘amount of substance’ is
(a) g (b) cm3 (c) kg (d) mol
Complete Chemistry—JEE Main

17. The length 109 m may be represented as


(a) 10 km (b) 100 Mm (c) 1 Gm (d) 1 Tm
18. One micrometer stands for
(a) 10–3 m (b) 10–6 m (c) 10–9 m (d) 10–12 m
19. Which of the following is the unit of energy?
(a) kg m2 s–2 (b) N m (c) Pa m3 (d) Pa m–1
20. The unit Siemens is of physical quantity
(a) Resistance (b) Conductance (c) Molar conductivity (d) Conductivity
18 is

(a) tera (b) peta (c) exa (d) atto


22. Which of the following physical quantities is a base physical quantity?
(a) Mass (b) Time
(c) Electric charge (d) Thermodynamic temperature
23. Which of the following physical quantities is a derived physical quantity?
(a) Electric current (b) Speed (c) Molar heat capacity (d) Electric resistance
24. Which of the following physical quantities has SI unit of kg m2 s–3 A–1?
(a) Electric resistance (b) Electric potential (c) Power (d) Energy
2 s–3 A–2?
25. Which of the following physical quantities has SI unit of kg m
(a) Electric resistance (b) Electric potential (c) Power (d) Energy

(a) J A–1 s–1 (b) kg–1 m–2 s3 A2 (c) kg m2 s–2 (d) kg s–2 A–1
12 and its symbol respectively are

(a) mega, M (b) giga, G (c) tera, T (d) peta, P


15 and its symbol respectively are

(a) Giga,G (b) Peta, P (c) Tera, T (d) Exa, E


–12 and its symbol respectively are

(a) nano, n (b) atto, a (c) femto, f (d) pico, p

(a) 10-9 (b) 10–12 (c) 10–15 (d) 10–18

(a) 109 (b) 1012 (c) 1015 (d) 1018


32. One femtometer stands for
(a) 10–3 m (b) 10–15 m (c) 10–9 m (d) 10–18 m

33. The volume of concentrated sulphuric acid (98 mass % H2SO4, density 1.84 g cm–3) required to prepare 5 dm3
of 0.5 mol dm–3 solution of sulphuric acid is
(a) 68 cm3 (b) 136 cm3 (c) 204 cm3 (d) 272 cm3
–3
34. The molarity of concentrated sulphuric acid (r = 1.834 g cm ) containing 95% of H2SO4 by mass is
(a) 4.44 M (b) 8.88 M (c) 13.32 M (d) 17.78 M
35. Molarity of liquid water at 4 °C is
(a) 5.56 mol L–1 (b) 55.56 mol L–1 (c) 0.556 mol L–1 (d) 18 mol L–1
36. The molarity of 20.0 mass % H2SO4 solution of density 1.14 g cm–3 is
(a) 1.52 mol dm–3 (b) 2.02 mol dm–3 (c) 2.32 mol dm–3 (d) 2.82 mol dm–3
–1
) containing 36.5 mass per cent of HCl
required for preparing 2 L of 0.1 M hydrochloric acid solution is about
(a) 15.4 mL (b) 16.7 mL (c) 17.5 mL (d) 22.2 mL
Some Basic Concepts of Chemistry

–1) containing 70 mass percent

of acid to prepare 1.0 M solution would be


(a) 120.6 mL (b) 130.6 mL (c) 135.6 mL (d) 145.6 mL
39. The volume of 0.25 M NaOH to be added to 250 mL of 0.15 M NaOH so that the resultant solution is 0.2 M
would be
(a) 250 mL (b) 350 mL (c) 450 mL (d) 550 mL
40. Which of the concentration units is temperature dependent?
(a) Molality (b) Amount fraction (c) Molarity (d) Parts per million
41. Medicinal alcohol is an aqueous solution of C2H6O that contains half alcohol and half water by volume. If this
contains 44.29 % alcohol by mass, the density of pure alcohol is about
(a) 0.795 g/mL (b) 0.895 g/mL (c) 0.443 g/mL (d) 0.557 g/mL
42. The expression converting molality (m) into molarity (M) is
(a) m = M(r + MM2) (b) m = M/(r – MM2) (c) m = (r + MM2)/M (d) m = (r – MM2)/M
where r and M2 are density of solution and molar mass of the solute, respectively.

43. If in a reaction HNO3 is reduced to NO, the mass of HNO3 absorbing one mole of electrons would be
(a) 12.6 g (b) 21.0 g (c) 31.5 g (d) 63.0 g
– is reduced to NH +, the mass of NO– absorbing one mole of electrons would be
44. If in a given reaction NO3 4 3
(a) 31.0 g (b) 12.4 g (c) 6.29 g (d) 7.75 g
–3 and an atomic mass of 55.85 amu. The volume occupied by 1 mol of Fe is
45. Iron has a density of 7.86 g cm
(a) 0.141 cm3 mol–1 (b) 7.11 cm3 mol–1 (c) 4.28 ¥ 104 cm3 mol–1 (d) 22.8 cm3 mol–1
– +
46. The stoichiometry numbers of Cu2O, NO 3 and H in the balanced chemical equation of the reaction
Cu2O + NO–3 + H+ æÆ Cu2+ + NO + H2O, respectively, are
(a) 2, 4, 4 (b) 3, 2, 14 (c) 2, 3, 6 (d) 4, 2, 6
2– +
47. The stoichiometry numbers of Cr2O 7 , CH3CHO, H in the balanced chemical equation of the reaction
7 + CH3CHO + H æÆ Cr + CH3COOH + H2O, respectively, are
Cr2O 2– + 3+

(a) 1, 2, 4 (b) 1, 4, 2 (c) 1, 3, 8 (d) 1, 4, 6


48. The stoichiometry numbers of MnO–4, Pb2+ and H2O in the balanced chemical equation of the reaction
Mn2+ + PbO2 + H+ æÆ MnO4– + Pb2+ + H2O, respectively, are
(a) 2, 5, 4 (b) 1, 4, 2 (c) 1, 2, 6 (d) 1, 2, 3
49. The mass of H2O2 that is completely oxidised by 30.2 g of KMnO4 (molar mass = 158 g mol–1) in acidic medium is
(a) 12 g (b) 15 g (c) 17 g (d) 1 g
–1
50. The volume V of K2Cr2O7 (molar mass 294 g mol ) solution (mass concentration 0.005 g per mL) is equivalent
to 35.0 mL of 0.02 M KMnO4 (molar mass 158 g mol–1), when these solutions are used in the titration in acid
solution. The value of V would be
(a) 46.75 mL (b) 34.3 mL (c) 29.05 mL (d) 22.5 mL
–1
51. Iron pyrite, FeS2 ), is converted into ferric oxide and sulphur dioxide. The mass of
residue left behind from 1.08 g iron pyrite ignited to a constant mass in air would be
(a) 0.72 g (b) 0.85 g (c) 0.54 g (d) 0.46 g
52. 0.5 g of sample of an alloy gave 0.30 g of Mg2P2O7 (molar mass of Mg and P are 24 g mol–1 and 31 g mol–1,
respectively). The percentage of Mg in the alloy would be
(a) 10.5% (b) 12.97% (c) 15.23% (d) 18.31%
53. A sample of NaClO3 is converted by heat to NaCl with a loss of 0.16 g of oxygen. The residue is dissolved in
–1
) obtained will be
(a) 0.35 g (b) 0.41 g (c) 0.48 g (d) 0.54 g
–1
54. A 0.50 g sample containing only anhydrous FeCl3 ) and AlCl3
mol–1 –1
). The mass of FeCl3 in the sample is
(a) 0.3126 g (b) 0.4157 g (c) 0.2345 g (d) 0.1567 g
Complete Chemistry—JEE Main

55. One gram of a mixture of Fe3O4 and Fe2O3 is treated with H2


3Fe2O3 + H2 Æ 2 Fe3O4 + H2O
The sample weighs 0.97 g after the above reaction is completed. The per cent of Fe2O3 in the mixture is about
(a) 16 (b) 14 (c) 12 (d) 90

56. The combustion of 4.24 mg of an organic compound produces 8.45 mg of CO2 and 3.46 mg of water. The mass
percentages of C and H in the compound, respectively, are
(a) 54.4, 9.1 (b) 9.1, 54.4 (c) 27.2, 18.2 (d) 18.2, 27.2
57. A 2.0 g mixtute of Na2CO3 and NaHCO3 suffered a loss of 0.12 g on heating. The percentage of Na2CO3 in the
mixture is
(a) 83.8 (b) 16.2 (c) 38.8 (d) 61.2
58. A compound contains 48.98% C, 2.72% H and 48.32% Cl by mass. Its empirical formula is
(a) C13H8Cl5 (b) C3H2Cl (c) C12H10Cl5 (d) C15H12Cl6
59. An organic compound weighing 0.778 g was subjected to Kjeldahl’s method to determine its nitrogen content.
The evolved ammonia was absorbed in 100 cm3 of 1 M HC1. The excess acid required 147.4 cm3 of 0.5 M KOH
for complete neutralisation. The mass per cent of nitrogen in the compound was
(a) 23.67 (b) 32.67 (c) 47.33 (d) 74.33
60. An organic compound contains 20.0% C, 6.66% H, 47.33% N and the rest was oxygen. Its molar mass is 60 g
mol–1. The molecular formula of the compound is
(a) CH4N2O (b) CH2NO2 (c) C2H6NO (d) CH18NO
61. The simplest formula of the compound containing 32.5% K, 0.839% H, 26.7% S and 39.9% O by mass is
(a) KHSO2 (b) KHSO3 (c) KHSO4 (d) K2H2S2O7
62. The simplest formula of a compound which contains 85.6% C and 14.4% H by mass is
(a) CH (b) CH2 (c) C2H3 (d) CH3
63. The simplest formula of a compound containing 21.9% Mg, 27.8% P and 50.3% O by mass is
(a) Mg2P3O5 (b) MgP2O4 (c) Mg2P2O7 (d) Mg3PO4
64. The mass of Na2CO3 (91.2% purity) required for the neutralisation of 45.6 mL of 0.25 M HC1 solution is
(a) 0.6625 g (b) 0.4265 g (c) 0.5765 g (d) 0.8473 g
65. The sulphur in 1.0 g sample of steel is burned to sulphur dioxide and absorbed in 50.0 mL of 0.1 M sodium
hydroxide solution. The excess sodium hydroxide requires 24.0 mL of 0.15 M hydrochloric acid solution for
neutralization. The percentage of sulphur in the sample is
(a) 1.50% (b) 2.24% (c) 3.25% (d) 4.5%
66. A sample of organic material weighing 1.058 g is decomposed by the Kjeldahl method. The ammonia is distilled
into 50.0 mL of 0.100 M HCl solution. The excess remaining acid required 10.10 mL of 0.098 M NaOH solution.
The percentage of nitrogen in the sample would be
(a) 5.45% (b) 10.90% (c) 12.4% (d) 16.7%

ANSWERS
1. (c) 2. (b) 3. (d) 4. (d) 5. (d) 6. (b)
7. (c) 8. (c) 9. (a) 10. (b) 11. (b) 12. (a)
13. (d) 14. (c) 15. (d) 16. (d) 17. (c) 18. (b)
19. (d) 20. (b) 21. (c) 22. (c) 23. (a) 24. (b)
25. (a) 26. (d) 27. (c) 28. (b) 29. (d) 30. (d)
31. (a) 32. (b) 33. (b) 34. (d) 35. (b) 36. (c)
37. (b) 38. (d) 39. (a) 40. (c) 41. (a) 42. (b)
Some Basic Concepts of Chemistry

43. (b) 44. (d) 45. (b) 46. (b) 47. (c) 48. (a)
49. (c) 50. (b) 51. (a) 52. (b) 53. (c) 54. (a)
55. (d) 56. (a) 57. (a) 58. (b) 59. (c) 60. (a)
61. (b) 62. (b) 63. (c) 64. (a) 65. (b) 66. (a)

HINTS AND SOLUTIONS


Ê 0.05 g ˆ
1. Number of atoms = 3 Á (6.022 ¥ 1023 mol–1) = 5.02 ¥ 1021
Ë 18 g mol-1 ˜¯
4
3. If is atomic mass, then ¥ 10 g = 5.62 g which gives = 30.8 u
4 + 96 u
Ê 1g ˆ
23 ˜ = 1.66 ¥ 10
4. 100 amu = (100) Á –22 g
Ë 6.022 ¥ 10 ¯
7.0 ¥ 1022
Mass of 7.0 ¥ 1022 molecules = ¥ 46 g = 5.35 g
6.022 ¥ 1023
Mass of 8.0 ¥ 10–1 mol = 0.8 ¥ 46 g = 36.8 g
6. If is the fraction of 10B, we have (10.01 amu) + (1 – ) (11.01 amu) = 10.82 amu
This gives = 0.19. Hence, percentage is 19%.
7. We have, M = [(7.4) (6.00 amu) + (100 – 7.4) (7.00 amu)]/100 = 6.93 amu
9. In the formation of 42He from its constituents, there occurs the loss of mass. This is known as mass defect.
12. tC = (64.15 – 273.15)°C = –209 °C; and tF = {(–209 ¥ 1.8) + 32}°F = –344.2 °F

Unit of work Unit of (force ¥ distance) kg m s -2 (m)


Unit of potential = = = = kg m 2 s -3 A -1
Unit of charge Unit of (current ¥ time) As
I = E/R. Hence,
Unit of potential kg m 2 s -3 A -1
Unit of resistance = = = kg m 2 s -3 A -2
Unit of current A
F = qBu. Hence,
Unit of force kg m s -2
Unit of B = = = kg s -2 A -1
(Unit of charge) (Unit of velocity) (A s) (m s -1 )
m(100 / 98) (n2 M 2 )(100 / 98) ( MV ) M 2 (100 / 98)
33. V = = =
r r r
= {(0.5 mol dm–3) (5 dm3) (98 g mol–1) (100/98)}/(1.84 g cm–3) = 136 cm3
95 g / 98 g mol-1
34. Molarity = = 0.01778 mol cm–3 = 17.78 mol dm–3
100 g / 1.834 g cm -3
35. One litre of water weighs 1000 g (assuming density equal to 1.00 g/mL).
m / M 1000 g / 18 g mol-1
Hence, Molarity = = = 55.56 mol L-1
V 1L
m 100 g
36. Volume of 100 g solution is V= = -1
= 87.72 cm3
r 1.14 g cm
m 20.0 g
Amount of sulphuric acid in 100 g solution is n= = = 0.207 mol
M 98 g mol-1
Complete Chemistry—JEE Main

n 0.207 mol
Molarity of sulphuric acid is M= = -3
= 2.32 mol dm -3
V 87.72 ¥ 10 dm 3

m 36.5 g
37. In 100 g of HCl solution, the amount of HCl is n = = = 1.0 mol
M 36.5 g mol-1
m 100 g
Volume of 100 g of HCl solution, V= = = 83.3 mL
r 1.20 g mL-1
The amount of HCl in 2 L of 0.1 M HCl solution, n2 = MV = (0.1 mol L–1) (2 L) = 0.2 mol
Ê 83.3 mL ˆ
Hence, the volume of solution containing 0.2 mol HC1 is V= Á (0.2 mol) = 16.66 mL
Ë 1.0 mol ˜¯
38. Mass of 10.0 mL of nitric acid, m = Vr = (10.0 mL)(1.40 g mL–1) = 14.0 g
Mass of nitric acid in this solution = (14.0 g) (70/100) = 9.8 g
Amount of nitric acid, n = m/M = 9.8 g/63 g mol–1 = 0.1556 mol
n Ê 0.1556 mol ˆ
Molarity of solution M =
Ë 0.01 L ˜¯
= Á = 15.56 M
V
This solution is to be diluted to prepare 1.0 M solution.
(10 mL) (15.56 M )
Volume of the resultant solution = = 155.6 mL
(1.0 M )
Volume of water to be added = 155.6 mL – 10.0 mL = 145.6 mL
39. Let V be the volume of 0.25 M NaOH solution. Total amount of NaOH after mixing the two solutions is
n = V (0.25 mol L–1) + (0.25 L) (0.15 mol L–1)
Total volume of the solution = V + 0.25 L
V (0.25 mol L-1 ) + (0.25 L)(0.15 mol L-1 )
Molarity of the resultant solution M=
V + 0.25 L
V (0.25 mol L-1 ) + (0.25 L)(0.15 mol L-1 )
Equating this to 0.2 M, we get M= = 0.2 mol L-1
V + 0.25 L
Solving for V, we get V = 0.25 L = 250 mL
41. The mass of water per 100 g of medicinal alcohol is 55.71 g. Its volume will be 55.71 cm3. Thus, density of alcohol
= 44.29 g / 55.71 cm3 = 0.795 g cm–3
+5 +2
43. We have H N O3 ææ ÆNO
62 g mol-1
Change in oxidation number = –3. Equivalent mass of HNO3 = -1
= 21 g eq -1
+5 -3 3 eq mol
44. We have N O3- ææ Æ N H 4+
62 g mol-1
Change in oxidation number = –8. Equivalent mass of NO–3 = -1
= 7.75 g eq -1
8 eq mol
Molar mass 55.85 g mol-1
45. Molar volume = = = 7.11 cm3 mol-1
density 7.86 g cm -3
49. The reaction is 2KMnO4 + 5H2O2 + 6H+ æÆ 2Mn2+ + 5O2 + 8H2O + 2K+
2 ¥ 151 g of KMnO4 reacts completely with 5 ¥ 34 g of H2O2
5 ¥ 34 g H 2 O 2
30.2 g of KMnO4 reacts completely with (30.2 g KMnO 4 ) = 17 g H 2 O 2
2 ¥ 151 g KMnO 4
50. The reactions are
1 14 1 7 1 8 1 4
Cr2 O72- + H + + e- ææ
Æ Cr 3+ + H 2 O and MnO -4 + H + + e - ææ
Æ Mn 2+ + H 2 O
6 6 3 6 5 5 5 5

n ÊÁ Cr2 O72- ˆ˜ ∫ n ÊÁ MnO -4 ˆ˜ i.e. 6n(Cr2 O72- ) = 5n(MnO 4- )


1 1
Thus,
Ë6 ¯ Ë5 ¯
Some Basic Concepts of Chemistry

Now n(KMnO4) = VM = (0.035 L) (0.02 mol L–1) = 0.0007 mol


Ê 0.005 g mL-1 ˆ
= V ÊÁ mol mL-1 ˆ˜
0.005
n(K2CrO7) = VM = V Á -1 ˜ Ë 294 ¯
Ë 294 g mol ¯
Ê 0.005 mol mL-1 ˆ = 5(0.0007 mol) Ê 5 ˆ Ê 0.0007 ¥ 294 ˆ mL = 34.3 mL
Thus 6V Á ˜¯ or V = ÁË ˜¯ ÁË ˜¯
Ë 294 6 0.005
51. Molar mass of FeS2 = (56 + 2 ¥ 32) g mol–1 = 120 g mol–1
Molar mass of Fe2O3 = (2 ¥ 56 + 3 ¥ 16) g mol–1 = 160 g mol–1
11
The reaction is 2FeS2 + O æÆ Fe2O3 + 4SO2
2 2
160
Hence, the mass of residue will be ¥ 1.08 g = 0.72 g
2 ¥ 120
52. Molar mass of Mg2P2O7 = (2 ¥ 24 + 2 ¥ 31 + 7 ¥ 16) g mol–1 = 222 g mol–1
2 ¥ 24
Mass of Mg in the given Mg2P2O7 = ¥ 0.30 g = 0.06486 g
222
0.064 86 g
Mass percentage of Mg in alloy = ¥ 100 = 12.97%
0.5 g
3
53. The reaction is NaClO3 ææ Æ NaCl + O 2
2
0.16 g
Amount of O2 released = = 0.005 mol
32 g mol-1
2
Amount of NaCl formed or AgCl precipitated = ¥ 0.005 mol
3
Ê 2 ˆ
Mass of AgCl precipitated = Á ¥ 0.005 mol˜ (108 + 35.5) g mol–1 = 0.48 g
Ë3 ¯
m 1.435 g
54. Amount of AgCl obtained, n = = = 0.01 mol
M 143.5 g mol-1
Let be the mass of FeCl3, then
0.50 g -
Amount of FeCl3 = and Amount of A1Cl3 =
162 g mol -1
133.5 g mol-1
È 0.50 g - ˘
Hence, 3Í -1
+ -1 ˙
= 0.01 mol
Î162 g mol 133.5 g mol ˚
Solving for , we get
1 È 0.01 ¥ 162 ¥ 133.5
= - 81˘˙ g = 0.3126 g
(-28.5) ÍÎ 3 ˚
55. Molar mass of Fe2O3 = (2 ¥ 56 + 3 ¥ 16) g mol–1 = 160 g mol–1
Molar mass of Fe3O4 = (3 ¥ 56 + 4 ¥ 16) g mol–1 = 232 g mol–1
Loss of mass in the reaction due to conversion of Fe2O3 to Fe3O4 is (2 ¥ 232 – 3 ¥ 160)g mol–1 = –16 g mol–1
16 0.03 ¥ 480
If is the mass of Fe2O3 in the given mixture, then = (1 – 0.97)g or = g = 0.9 g
3 ¥ 160 16
Per cent of Fe2O3 in the mixture = 0.9 ¥ 100 = 90%
Ê 12 ˆ Ê 8.45 ˆ 100 = 54.4 Ê 2 ˆ Ê 3.46 ˆ 100 = 9.1
56. Mass per cent of C = ÁË ˜¯ ÁË ˜ %
Ë 18 ¯ Ë 4.24 ˜¯
Mass per cent of H = Á ˜ Á %
44 4.24 ¯
57. Loss is due to the reaction
2 NaHCO3 Æ Na2CO3 + H2O + CO2
2 ¥ 84 g 106 g
which involves the loss of 62 g (= 2 ¥ 84 g – 106 g). If is the mass of NaHCO3 in the mixture, then
Complete Chemistry—JEE Main

Ê 168 g ˆ (2.0 - 0.325)g


=Á (0.12 g) = 0.325 g ¥ 100 = 83.8
Ë 62 g ˜¯
Per cent Na2CO3 =
2.0 g
48.98 2.72 48.32
: :
12 1 35.5
Empirical formula = C3H2Cl
59. Amount of HCl to start with n1 = VM = (100 ¥ 10–3 L) (1.0 mol L–1) = 0.100 mol
Amount of excess of HCl n2 = VM = (147.4 ¥ 10–3 L) (0.5 mol L–1) = 0.0737 mol
Amount of N in the compound = Amount of NH3 evolved = Amount of acid consumed, n = n1 – n2
= (0.100 – 0.0737) mol = 0.0263 mol
Mass of N is the compound mN = nM = (0.0263 mol) (14 g mol–1) = 0.3682 g
mN 0.3682 g
Per cent of N = ¥ 100 = ¥ 100 = 47.33%
morganic 0.778 g
20.0 6.66 47.33 26.01
: : :
12 1 14 16
Empirical formula = CH4N2O; Molar empirical formula mass = 60 g mol–1
Molecular formula = CH4N2O
32.5 0.039 26.7 39.9
: : :
39 1 32 16
Empirical formula = KHSO3
85.6 14.4
: 2
12 1
21.9 27.8 50.3
: :
24.3 31.0 16
2P 2O 7
64. The reaction is Na2CO3 + 2HC1 Æ 2NaCl + H2CO3
Amount of HCl to be neutralised = VM = (45.6 ¥ 10–3 L) (0.25 mol L–1) = 11.4 ¥ 10–3 mol
Amount of Na2CO3 required = 0.5 ¥ 11.4 ¥ 10–3 mol = 5.7 ¥ 10–3 mol
Mass of Na2CO3 required = (5.7 ¥ 10–3 mol) (106 g mol–1) (100/91.2) = 0.6625 g
65. The reaction of NaOH and SO2 is 2NaOH + SO2 Æ Na2SO3 + H2O
Amount of NaOH to start with = VM = (50 ¥ 10–3 L) (0.1 mol L–1) = 5 ¥ 10–3 mol
Amount of NaOH after the absorption of SO2 = (24.0 ¥ 10–3 L) (0.15 mol L–1) = 3.6 ¥ 10–3 mol
Amount of NaOH used in absorbing SO2 = 5 ¥ 10–3 mol – 3.6 ¥ 10–3 mol = 1.4 ¥ 10–3 mol
Amount of SO2 formed = (0.5) (1.4 ¥ 10–3 mol) = 7.0 ¥ 10–4 mol
Amount of S in the steel = 7.0 ¥ 10–4 mol
(7.0 ¥ 10-4 mol) (32 g mol-1 )
Mass percent of S = ¥ 100 = 2.24%
1.0 g
66. Amount of HCl to start with = (50.0 ¥ 10–3 L) (0.10 mol L–1) = 50.0 ¥ 10–4 mol
Amount of excess HCl = (10.0 ¥ 10–3 L) (0.098 mol L–1) = 9.8 ¥ 10–4 mol
Amount of NH3 evolved or amount of N in the compound = (50.0 – 9.8) ¥ 10–4 mol = 41.2 ¥ 10–4 mol
Mass of N in the compound = (41.2 ¥ 10–4 mol) (14 g mol–1) = 0.05768 g
0.05768
Percentage of N in the compound = ¥ 100 = 5.45%
1.058
Some Basic Concepts of Chemistry

MULTIPLE CHOICE QUESTIONS FROM AIEEE AND JEE MAIN

1. If we consider that 1/6, in place of 1/12, mass of carbon atom is taken to be the relative atomic mass unit, the mass
of one mole of a substance will
(a) Remain unchanged (b) Be a function of the molecular mass of the substance
(c) Decrease twice (d) Increase twice

+ 520 mL of 1.2 M
(a) 1,344 M M (c) 1.20 M (d) 1.50 M
3. How many moles of magnesium phosphate, Mg3(PO4)2 will contain 0.25 mole of oxygen atoms?
(a) 2.5 ¥ 10–2 (b) 3.125 ¥ 10–2 (c) 0.02 (d) 1.25 ¥ 10–2
4. Density of a 2.05 M solution of acetic acid water is 1.02 g/mL. The molality of the solution is
(a) 0.44 mol kg–1 (b) 1.14 mol kg–1 (c) 3.28 mol kg–1 (d) 2.28 mol kg–1
5. In the reaction, 2Al (s) + 6HCl(aq) Æ 2Al3+(aq) + 6Cl–(aq) + 3H2(g)
(a) 6 L HC1(aq) is consumed for every 3 L H2(g) produced
(b) 33.6 L H2(g) is produced regardless of temperature and pressure for every mole Al that reacts
(c) 67.2 L H2(g) at STP is produced for every mole Al that reacts
(d) 11.2 L H2(g) at STP is produced for every mole HCl(aq) consumed
6. The density (in g mL–1) of a 3.60 M sulphuric acid solution that is 29% H2SO4 (molar mass = 98 g mol–1) by mass
will be
(a) 1.64 (b) 1.88 (c) 1.22 (d) 1.45
7. The density of a solution prepared by dissolving 120 g urea (molecular mass = 60 u) in 1000 g of water is
1.15 g mL–1. The molarity of the solution is
(a) 0.50 M (b) 1.78 M (c) 1.02 M (d) 2.05 M
8. The molarity of a solution obtained by mixing 750 mL of 0.5 M HC1 and 250 mL of 2 M HC1 will be
(a) 0.875 M (b) 1.00 M (c) 1.75 M (d) 0.975 M
9. A gaseous hydrocarbon gives upon combustion 0.72 g of water and 3.08 g of CO2. The empirical formula of the
hydrocarbon is
(a) C2H4 (b) C3H4 (c) C6H5 (d) C7H8

is

11. Dissolving 120 g of a compound of relative molar mass 60 in 1000 g of water gave a solution of density
1.12 g/mL. The molarity of the solution is
(a) 1.00 M (b) 2.00 M (c) 2.50 M (d) 4.00 M
12. The mass of oxygen in 3.6 mol of water is
(a) 115.2 g (b) 57.6 g (c) 28.8 g (d) 18.4 g
13. A gaseous compound of nitrogen and hydrogen contains 12.5 mass% of hydrogen. The density of the compound
relative to hydrogen is 16. The molecular formula of the compound is
(a) NH2 (b) N3H (c) NH3 (d) N2H4
14. The mass of BaSO4 formed upon mixing 100 mL of 20.8% BaCl2 solution with 50 mL of 9.8% H2SO4 solution

(a) 23.3 g (b) 11.65 g (c) 30.6 g (d) 33.2 g


15. A sample of a hydrate of barium chloride weighing 61 g was heated until all the water of hydration is removed.

(a) BaCl ◊ H2O (b) BaCl2 ◊ 2H2O (c) BaCl2 ◊ 3H2O (d) BaCl2 ◊ 4H2O
Complete Chemistry—JEE Main

16. A + 2B + 3C AB2C3. Reaction of 6.0 g of A, 6.0 ¥ 1023 atoms of B, and 0.036 mol of C yields 4.8 g of compound
AB2C3. If the atomic masses of A and C are 60 and 80 amu, respectively, the atomic mass B of is (Avogadro
constant = 6 ¥ 1023 mol–1)
(a) 70 amu (b) 60 amu (c) 30 amu (d) 40 amu
17. The molecular formula of a commercial resin used for exchanging ions in water softening is C8H7SO3Na (molecular
2+
ions by the resin when expressed in mole per gram
resin?
(a) 1/103 (b) 1/206 (c) 2/309 (d) 1/412
18. 5 L of an alkane requires 25 L of oxygen for its complete combustion. If all volumes are measured at constant

(a) Isobutane (b) Ethane (c) Butane (d) Propane


19. An organic compound contains C, H and S. The minimum molecular weight of the compound containing 8%

(a) 600 (b) 200 (c) 400 (d) 300


20. The amount of arsenic pentasulphide that can be obtained when 35.5 g arsenic acid is treated with excess H2S in

(a) 0.25 mol (b) 0.50 mol (c) 0.333 mol (d) 0.125 mol


in aqueous

(a) 400 mL (b) 600 mL (c) 200 mL (d) 800 mL


22. At 300 K and 1 atm, 15 mL of a gaseous hydrocarbon requires 375 mL air containing 20% O2 by volume for
complete combustion. After combustion the gases occupy 330 mL. Assuming that the water formed is in liquid
form and the volumes were measured at the same temperature and pressure, the formula of the hydrocarbon is
(a) C3H8 (b) C4H8 (c) C4H10 (d) C3H6

ANSWERS
1. (a) 2. (a) 3. (b) 4. (d) 5. (d) 6. (c)
7. (d) 8. (a) 9. (d) 10. (a) 11. (b) 12. (b)
13. (d) 14. (b) 15. (b) 16. (c) 17. (d) 18. (d)
19. (c) 20. (d) 21. (a) 22. None

HINTS AND SOLUTIONS


1. The atomic mass unit will become twice and the relative molar mass will become half such that the product of
these two remains unchanged.
2. Total amount of solute = (0.480 ¥ 1.5 + 0.520 ¥ 1.2) mol = (0.720 + 0.624) mol = 1.344 mol
Total volume of solution = (480 + 520) mL = 1000 mL = 1 L
Molarity of solution = 1.344 mol/l L = 1.344 M
3. In Mg3(PO4)2, there are 8 atoms of oxygen. Hence, 0.25 mol of oxygen atoms will be contained in
(1 mol/8 mol) (0.25 mol) = 3.125 ¥ 10–2 mol of Mg3(PO4)2.
4. In 1 L of solution, we have
r(acetic acid soln) = 1.02 g/mL = 1020 g/L m(acetic acid) = (2.05 mol/L) (60 g mol–1) = 123 g/L
m(water) = ( 1020 – 123) g/L = 897 g/L
Molality of acetic acid solution = (2.05 mol)/(0.897 kg) = 2.28 mol kg–1
Some Basic Concepts of Chemistry

5. We have 2Al(s) + 6HCl(aq) Æ 2Al3+(aq) + 6Cl–(aq) + 3H2(g)


2 mol 6 mol 3 mol
3 ¥ 22.4 L at STP
3 ¥ 22.4 L
For 1 mol of HCl, the volume of H2 gas evolved will be = 11.2 L
6
6. In 100 g solution, 29 g H2SO4 is present. The volume of this solution as determined from the given molarity is
n (29 g / 98 g mol-1 )
V= = = 0.0822 L ∫ 82.2 mL
M 3.60 mol L-1
m 100 g
Hence, density of the solution is r = = = 1.22 g mL-1
V 82.2 mL
m 120 g
7. Amount of urea dissolved, n = = = 2 mol
M m 60 g mol-1
Total mass of solution, m = 120 g + 1000 g = 1120 g
m 1120 g
Volume of solution, V = = = 973.9 mL = 0.9739 L
r 1.15 g mL-1
n 2 mol
Molarity of solution, M = = = 2.05 mol L-1
V 0.9739 L
8. Amount of HCl in the resultant solution is
n = MlVl + M2V2 = (0.5 M) (750 ¥ 10–3 L) + (2 M) (250 ¥ 10–3 L) = 875 ¥ 10–3 mol
Total volume of the resultant solution is V = Vl + V2 = (750 + 250) ¥ 10–3 L = 1 L
n 875 ¥ 10-3 mol
Molarity of the resultant solution is M = = = 0.875 mol L-1 = 0.875 M
V 1L
Ê 2M H ˆ 2
9. Mass of H in the obtained water is ÁM ˜ mH 2O = 18 ¥ 0.72 g = 0.08 g
Ë H2O ¯
m 0.08 g
Amount of H in the molecule is nH = H = = 0.08 mol
M H 1 g mol-1
Ê MC ˆ 12
Mass of C in the obtained CO2 is ÁM ˜ mCO2 = 44 ¥ 3.08 g = 0.84 g
Ë CO2 ¯
mC 0.84 g
Amount of C in the molecule is nC = = = 0.07 mol
M C 12 g mol-1
The ratio of amounts of C and H in the molecule is nC nH
Hence, the empirical formula of the compound is C7H8.
10. Let there, be 1 g of O2, then mass of N2 will be 4 g.
1g 4g
Amount of O2 = = (1 / 32) mol Amount of N2 = = (1 / 7) mol
32 g mol-1 28 g mol-1
1 1
Ratio of their amounts is : :: 7 : 32 The same ratio holds good for the number of molecules.
32 7
11. Total mass of the solution = 120 g + 1000 g = 1120 g
m 1120 g
Volume of the solution, V = = = 1000 mL = 1.0 L
r 1.12 g mL-1
m 120 g
Amount of compound, n = = = 2.0 mol
M 60 g mol-1
Complete Chemistry—JEE Main

n 2.0 mol
Molarity of solution, M = = = 2.0 M
V 1.0 L
12. Mass of oxygen, m = nM = (3.6 mol) (16 g mol–1) = 57.6 g
87.5 12.5
14 1
2 Molar empirical formula mass = 16 g mol–1
Molar mass = ( 2 ¥ vapour density) g mol–1 = 2 ¥ 16 g mol–1
Molecular formula (NH2), i.e. N2H4.

m 20.8 g
14. Amount of BaCl2 in 100 mL of 20.8% BaCl2 = = = 0.1 mol
M (137 + 2 ¥ 35.5) g mol-1

m (9.8 / 2) g
Amount of H2SO4 in 50 mL of 9.8% H2SO4 = = = 0.05 mol
M 98 g mol-1
From the reaction
BaCl2 + H2SO4 Æ BaSO4 + 2HCl
0.1 mol 0.05 mol
We conclude that 0.05 mol of BaSO4 will be formed
Hence m(BaSO4) = (0.05 mol) (233 g mol–1) = 11.65 g
m 52 g
15. Amount of anhydrous BaCl2, n = = = 0.25 mol
M (137 + 2 ¥ 35.5) g mol-1
Let the hydrated barium chloride be BaCl2 ◊ H2O. The mass of 0.25 mol of this salt will be equal to 61 g (given)
Hence,
(0.25 mol) (208 + ¥ 18)g mol–1 = 61 g
Solving for , we get = 2. Hence, the formula of hydrated barium chloride is BaCl2 ◊ 2H2O

A + 2B + 3C AB2C3
6.0 g 6.0 ¥ 1023 atoms 0.036 mol 4.8 g
6
= mol 1 mol
60
Maximum extent
0.1 mol 0.5 mol 0.012 mol
of reaction
The limiting reagent is the atom C. The amount of AB2C3 formed will also be 0.012 mol. Mass of AB2C3 formed
is 4.8 g. Hence (0.012 ¥ 60 + (0.024 mol) MB + 036 ¥ 80) g = 4.8 g
This gives MB = 50 g mol–1. Thus, atomic mass of B is 50 amu.
17. The exchanging reaction is 2 C8H7SO3Na + Ca2+Æ(C8H7SO3)2Ca + 2Na+
For one mole of Ca2+ ions, two moles of resin is required. Thus, the maximum uptake of Ca2+ ions will be
1 mol Ca2+/2 mol resin ∫1 mole Ca2+/(2 × 206) g resin ∫ (1/412) mol Ca2+/g of resin
18. For the alkane CnH2n+2, the combustion reaction is
Ê 3n + 1ˆ
CnH2n + 2 + Á O Æ n CO2 + (n + 1) H2O
Ë 2 ˜¯ 2
For 5 L of CnH2n+2, the volume of oxygen required is (5 L) {(3n + 1)/2}. Equating this to 25 L, we get
Some Basic Concepts of Chemistry

Ê 3n + 1ˆ
(5 L) Á = 25 L
Ë 2 ˜¯
This gives n = 3. Hence, the alkane is propane.
19. 8% sulphur implies that 8 g of sulphur is present in 100 g of the compound. The minimum molecular weight of
the compound is obtained when 32 g of sulphur (i.e. one mole of sulphur) is present. Thus minimum molecules
Ê 100 ˆ
weight = Á
Ë 8 ˜¯
(32) = 400
20. Molar mass of arsenic acid, H3AsO4, is M = (3 + 74.92 + 4 × 16) g mol–1 = 143.42 g mol–1
Arsenic pentasulphide is As2S5.
2 mol of H3AsO4 will provide 1 mol of As2S5.
1 Ê 35.5 ˆ
35.5 g (= (35.5/143.92) mol) of H3AsO4 will provide Á ˜ mol of As2O5 which is 0.123 mol.
2 Ë 143.92 ¯
21. 0.04 mol of OH– implies 0.04 eq of OH–.
Hence N1V1 = 0.04 eq i.e. (0.1 eq L–1) V1 = 0.04 eq
0.04
Hence, V1 = L = 0.4 L ∫ 400 mL
0.1
Ê 20 ˆ
Ë 100 ˜¯
22. Volume of O2 present in the given volume of air = Á (375 mL) = 75 mL

Volume of air without O2 = 375 mL – 75 mL = 300 mL


The combustion reaction may be represented as

C Hy + ÊÁ x + ˆ˜ O2
y y
Æ CO2 + HO
Ë 4¯ 2 2
15 mL 75 mL (330 – 300) mL
= 30 mL
Obviously, the reaction is
y
C Hy + 5O2 Æ 2CO2 + HO
2 2
Thus =2 and y/4 = 5 – = 5 – 2 = 3 i.e. y = 12
The hydrocarbon is C2H12, which theoretically, is not possible.
Had the volume of gas after the combustion of hydrocarbon been 345 mL, the hydrocarbon would be C3H8
as seen from the following calculations.
Ê yˆ y
C Hy + ÁË x + ˜¯ O2 Æ CO2 + HO
4 2 2
15 mL 75 mL (345 – 300) mL
= 45 mL
The value of x will be 45 mL/15 mL = 3 and that of y will be
y 75 mL
+ = =5
4 15 mL
y = 4(5 – ) = 4( 5 – 3) = 8.
The net reaction will be C3H8 + 5O2 Æ 3 CO2 + 4 H2O
2
States of Matter

UNIT 1 GASEOUS STATE

SECTION 1 Ideal Gases

Experimentally Derived Gaseous Laws


Boyle’s Law

1 K
Vμ V= or pV = K (1)
p p
where K

p lV l = p 2V 2 (2)
where Vl and V2 are volumes at pressures p1 and p2

Fig. 1 Fig. 2

Charles Law

Vt = a + bt (3)
2.2 Complete Chemistry—JEE Main

where t a and b
a and b are
V0 / 273.15
b= and a=V
1 ∞C

Ê V0 / 273.15 ˆ t / ∞C ˆ Ê 273.15 + t / ∞C ˆ
Vt = V + Á ˜ t = V0 ÊÁ1 + ˜¯ = V0 ÁË ˜ (4)
Ë 1 ∞C ¯ Ë 273.15 273.15 ¯

T t
K ∞

Ê T / K ˆ = Ê V0 ˆ
VT = V0 Á ˜ T = KT
Ë 273.15 ¯ Ë 273.15 K ˜¯
Á (6)

where K

Fig. 3 Fig. 4

Gay-Lussac’s Law

pt = a + bt and pT = KT (7)
pT = KT

Fig. 5 Fig. 6
States of Matter 2.3

Equation of State

p1 , V1 , T1 æStep
æææ (1)
Æ p2 ,V ¢, T1 æStep
æææ (2)
Æ p2 ,V2 , T2

Step 1 p 1V 1 = p 2V ¢ fi V ¢ = p1V 1/p2


Step 2 V ¢/T1 = V2/T2 fi V ¢ = V2T1/T2

p1V1 V2T1 p1V1 p2V2 pV
= or = or =K (8)
p2 T2 T1 T2 T
where K
Universal Gas Constant K † K = nR where n
R

pV = nRT (9)

(pressure) (volume)
R=
(amount of gas) (kelvin temperature)
R are
(force/length 2 ) (length)3 (force)(llength ) work (or energy)
R= = =
(amount of gas) (kelvin) (amount of gas)(kelvin) (amount of gas) (kelvin)
R

Concept of an Ideal Gas and Value of Gas Constant When


H2
T = 273.15 K

He

22.414
Ideal gas
ideal gas.
pV
pV/atm L

2, N 2 2 p
p N2
O2
same value of pV
p/atm

Fig. 7

pV 22.414 atm L
R= = –1
mol–1
nT (1 mol) (273.15 K )
3 3
p V , we have


p and T V
K
2.4 Complete Chemistry—JEE Main

R 3) K–1 mol–1 3 K–1 mol–1 3 K–1 mol–1


3 K–1 mol–1
3, we also have R –1 mol–1
R
–1 –1
R mol

Dalton’s Law of Partial Pressures

p = p1 + p2
where pV = (n1 + n2 + · · ·) RT ; p1V = n1RT ; p2V = n2RT · · ·
p1 n1 n p2 n2 n
also = = 1 = x1 , = = 2 = x2 (11)
p n1 + n2 +  ntotal p n1 + n2 +  ntotal

where x1, x2, …

such that
ptotal = p + pwater (12)

Graham’s Law of Diffusion

T and p, we have
r2 r1 r2 M1
= or = (13)
r1 r2 r1 M2

1/ 2
r2 Ê p2 ˆ Ê M1 ˆ
= Á ˜Á
Ë p1 ¯ Ë M 2 ˜¯
(14)
r1
p1 and p2

Illustration

–1 –1
M M 4
4
p =x p=
4 +1
1
p 4=x 4p=
4 +1
1/ 2
nHe r Ê p ˆ Ê M CH 4 ˆ Ê 16 bar ˆ Ê 16 g mol -1 ˆ
= He = Á He ˜ Á ˜ = 16
Ë 4 bar ˜¯ ÁË 4 g mol -1 ˜¯
= Á
nCH 4 rCH 4 Ë pCH 4 ¯ Ë M He ¯
States of Matter 2.5

Avogadro’s Law

¥ 23

as
NA ¥ 23 mol–1
Density of a Gas pV = nRT
pV = nRT = (m/M) RT
where m M

m pM
r= =
V RT

MULTIPLE CHOICE QUESTIONS ON SECTION 1

pV = K K does not depend upon

pV versus p

p = a + bt, where b
(a) p p p /273 K (d) a/273 K
where p
pV)p = not correct?
¥ 2 dm3 3 ¥ 3 cm3
R not correct?
¥ 7 –1 mol–1 –1
mol–1
–1 mol–1 –1 mol–1
3

¥ 23 ¥ 22 ¥ 22 ¥ 23
–1
–3 –3 –3 –3
–1
2 2

2 2 2

(a) p > pN > p p >p > pN (c) pN > p >p (d) pN > p >p
2 2 2 2 2 2 2 2 2 2 2 2
3
3 4

3 3 3 3
2.6 Complete Chemistry—JEE Main

–1
–1 –1 –1 –1
3 3

¥ 8 ¥ 7 ¥ 7 ¥ 6

ANSWERS

HINTS AND SOLUTIONS


K
pV = constant at all pressures, the plot of pV versus p
b = (∂p/∂t)V = (p b=p
pV)p
∫ 3
¥ 2
dm3 ¥ 2 3 –2

¥ 3
cm3
3 –1
m)3 3

–1
R mol–1 –1
mol–1 –1
mol–1
-3
pV (6.0 atm)(500 ¥ 10 dm ) 3
n= =
RT (0.082 dm3 atm K -1 mol-1 )(300 K )
N = nNA ¥ 23
mol–1 ¥ 22

pM (1 atm)(124 g mol -1 )
r= = –3
RT (0.082 dm3 atm K -1 mol -1 ) (620 K )
rRT (0.50 g L-1 )(0.082 L atm K -1mol-1 )(300 K )
r = pM/RT, we have Mav = = –1
p (1 atm)
If x
M av = x M H 2 + (1 - x) M O2 = M O2 + ( M H 2 - M O2 ) x
M av - M O2 12.3 g mol -1 - 32 g mol -1
or x= =
M H 2 - M O2 2 g mol -1 - 32 g mol -1

m (O 2 ) 25 g m( N 2 ) 35 g
n 2) = = = 0.78 mol ; n(N2) = = = 1.25 mol
M (O 2 ) 32 g mol-1 M ( N 2 ) 28 g mol-1

m(CO 2 ) 40 g
n 2) = = = 0.909 mol
M (CO 2 ) 44 g mol-1
States of Matter 2.7

3 4 x
1/ 2
rNH3 ÊM ˆ 36.5 ˆ 1 / 2
= ÊÁ
x/t
= Á HCl ˜ i.e. ˜ = 1.465
rHCl Ë M NH3 ¯ (3 m - x) / t Ë 17 ¯

or x = (3 m – x x x
p2 = p – pwater
p2V2/T2 = p1V1/T1
Ê p1 ˆ Ê T2 ˆ Ê 760 ˆ Ê 300 ˆ (125 cm3 ) = 144.0 cm3
V2 = Á ˜ Á ˜ V1 = ÁË ˜Á ˜
Ë p2 ¯ Ë T1 ¯ 725 ¯ Ë 273 ¯
Ê p ˆÊT ˆ Ê 700 ˆ Ê 273 ˆ (1.43 g L-1 )
r = pM/RT r2 = Á 2 ˜ Á 1 ˜ r1 = Á –1
Ë p 1 ¯ Ë T2 ¯ Ë 760 ˜¯ ÁË 290 ˜¯

p 3) = (p 3 p
pV {(10-10 / 760) atm} (10-3 L)
n= = -1 -1
= 5.35 ¥ 10-18 mol
RT (0.082 L atm K mol )(300 K )

N = n NA ¥ –18
¥ 23
mol–1 ¥ 6

pV (2 p ) (0.85 V )
=
348 K T

T fi t

SECTION 2 Kinetic-Molecular Theory of Gases

Kinetic Theory of Gases

1 È 1 m(u 2 + u 2 +  + u 2 ) ˘ μ T
N ÍÎ 2 1 2 N ˙
˚
where N m u 1, u 2
2.8 Complete Chemistry—JEE Main

1
mu 2 = KT (16)
2

where K u2
u12 + u22 +  + u N2
u2 =
N (17)
Kinetic Gas Equation
1
pV = m N u2 (18)
3
Derivation of Gaseous Laws

N Á m u 2 ˆ˜ = NKT
2 Ê1 2
pV = (19)
3 Ë 2 ¯ 3
Boyle’s Law T, we have
pV = constant
Charles Law p, we have

Ê 2 NK ˆ
V= Á T
Ë 3 p ˜¯
or V = (constant) T

Gay-Lussac’s Law V, we have

Ê 2 NK ˆ T
Ë 3 V ˜¯
p= Á or p = (constant) T

Avogadro’s Law p and T, we have

Ê 2 KT ˆ
V= Á N
Ë 3 p ˜¯
or V = (constant) N

Expression of Root Mean Square Speed pVm = RT and M = mNA

1 1
pVm = (mN A )u 2 or RT = M u2
3 3

3RT 3RT
u2 = or urms = u2 =
M M
Average Kinetic Energy of Gaseous Molecules
1
KE = mu 2
2

1 Ê 3RT ˆ 1 Ê 3RT ˆ 3 Ê R ˆ 3
KE = mÁ ˜= m = T = kBT
2 Ë M ¯ 2 ÁË mN A ˜¯ 2 ÁË N A ˜¯
(21)
2

R 8.314 J K -1 mol -1
where k k = = = 1.38 ¥ 10-23 J K -1
N A 6.022 ¥ 1023 mol -1
States of Matter 2.9

3 3
N A (KE) = ( N A kB )T = RT (22)
2 2
Graham’s Law of Diffusion

rμ μ 1/ M
u2

r2 p2 M1
=
r1 p1 M2
Average Speed of Molecules
u + u2 +  + u N
u= 1
N

8RT 8kBT
u= =
pM pm
Comparison of Different Speeds ump, u and urms
2 RT 8RT 3RT 8
: : :: 2 : : 3 :: 1.414 : 1.595 : 1.732 :: 1 : 1.128 : 1.224
M pM M p

Distribution of Molecular Speeds

M ˆ3/ 2
= 4p ÊÁ
dNu
Mu2/2RT)*u2 du
Ë 2pRT ˜¯
(23)
N
Nu/N u and u
+ du M
N) (dNu/du) Fig. 8
u T1 and T2 T2 > T1
General Characteristics of Fig. 8

represented as ump ump Nu/du)/du


2 RT 2kBT
ump = = (24)
M m

Mu2/2RT
ump

upon the value of M/T M at temperature 2T


2 molecules at temperature T
2 molecules at temperature 2T
2.10 Complete Chemistry—JEE Main

2 RT 8RT 3RT
ump u urms : : 2 : 8/ p : 3
M pM M
ump < u < urms

MULTIPLE CHOICE QUESTIONS ON SECTION 2

1 1 1 2
(a) pV = mNu
2
pV = mNu 2 (c) pV = mNump
2
(d) pV = mNu
3 3 3 3

(a) T T2 (c) T (d) T 3/2

(a) 3r / p 3p / r (c) 3 pr (d) 3RT / m


where p, r and m

f 1, f 2 and f 3

(a) f1 > f2 > f3 f1 > f3 > f2 (c) f2 > f3 > f1 (d) f3 > f2 > f1
E 2 2 2
(a) E (N2) > E 2) > E 2 E (N2) < E 2) < E 2)
(c) E (N2) = E 2) = E 2) (d) E 2) > E (N2) > E 2)
2 not correct?
–1 –1 –1 –1
–1 –1 –1
f1, f2 and f3 ,

(a) f1 > f2 > f3 f1 < f2 < f3 (c) f1 > f3 > f2 (d) f2 > f1 > f3
f1 and f2

(a) f1 f2 f1 decreases and f2


f1 and f2 f1 and f2 decrease

(a)
States of Matter 2.11

not correct?
(a) 8 RT / pM 8 kBT / pmf (c) 8 p / pr (d) 8r / pp

(a) ¥ ¥ 32)

ANSWERS

HINTS AND SOLUTIONS


1
pV = mNu 2
3
RT/M
3RT 3 pVm 3p 3p
urms = = = =
M M M / Vm r

3RT2 3RT1 3R
- = ( T2 - T1 )
M M M
T2 - T1
373 K - 273 K = 2.79 K ; 473 K - 373 K = 2.44 K

573 K - 473 K = 2.19 K ; 673 K - 573 K = 2.00 K


3RT2 3RT1
=2 fi T2 = 2 T1 fi T2 = 4T1 = 4(300 K )
M M
T2 t2

3RT2 2 RT1
= fi 3T2 = 2T1 fi T2 = (2 / 3)(300 K ) = 200 K
M M

n ÊÁ RT ˆ˜
3
Ë2 ¯
8g 8g
Amount of N2 = = 0.286 mol = = 0.25 mol
28 g mol-1 2
32 g mol-1
8g
= = 0.182 mol
2
44 g mol-1

3 3
E = RT = (8.314 J K -1 mol-1 )(300 K ) = 3741.3 J mol-1
2 2
3 3 -1 -1 -1
E = RT = (1.987 cal K mol )(300 K ) = 894.15 cal mol
2 2
2.12 Complete Chemistry—JEE Main

3 3
E = RT = (0.082 L atm K -1 mol-1 )(300 K ) = 36.9 L atm mol-1
2 2
3
E = RT = 3741.3 J mol-1 ∫ 3741.3 kPa dm3 mol-1 ¥ –1 –1
2

1/ 2
2 RT Ê 2(8.314 J K -1 mol -1 ) (300 K ) ˆ
ump = =Á ˜¯ = 394.8 m s -1
M Ë 0.032 kg mol -1

f1 f2
2 RT1 / M T1 300 K
= = = 0.866
2 RT2 / M T2 400 K

8RT 8 pVm 8p 8p 8kBT


u= = = = Also u=
pM pM p( M / Vm ) pr p mf

(3 / 2) RT1 T1 300 K
= = = 0.75
(3 / 2) RT2 T2 400 K

SECTION 3 Real Gases

Deviation from Ideal Behaviour

Vm, real gas Vm pVm


Z= = =
Vm, ideal gas RT/p RT

Z
Z versus p

(1) Z 2
2

temperature, the pV 2 2


States of Matter 2.13

T4
T1 > T2 > T3 > T4 T3
T2
T1
1.0
Ideal gas

Z
0 200 400 600
p/101.325 kPa
Fig. 9 Fig. 10

pV = nRT

Van Der Waals Equation of State for a Real Gas

Causes of Deviations from Ideal Behaviour

Evidence for Molecular Attractions and Molecular Volume

Derivation of Van der Waals Equation


p V = nRT.

Correction for Volume p V = nRT, V


b

V = Vr – nb (26)
b
r, then
Ê 4 3ˆ
b = 4NA Á p r ˜
Ë3 ¯
Correction for Molecular Attraction

Decrease in velocity of a single molecule


Fig. 11
2.14 Complete Chemistry—JEE Main

n
μ
V
( ) Number of molecules colliding per second with the side of the vessel

n/V

n2
-Dp μ ÊÁ ˆ˜ ÊÁ ˆ˜
n n
i.e - Dp μ
ËV ¯ ËV ¯ V2

n2
-Dp = a (27)
V2
where a

n2 a
p = pr p = pr + (28)
V2

Ê n2 a ˆ
ÁË p + V 2 ˜¯ (V - nb) = nRT (29)

a and b

Units of van der Waals Constants a and b n2a/V2 a


6 mol–2 6 mol–2

3 mol–1
nb b
dm3 mol–1 or cm3 mol–1
Comment on the values of a and b b as
a

For example,
b 3 mol–1; a 6 mol–2
2
b 3 mol–1; a 6 mol–2
2
b 3 mol–1; a 6 mol–2
2

Illustration 3
2
6 mol–2 and b
a 2 2) =
3 mol–1
We have
n V 3; T a 6 mol–2 b ¥ –3 dm3 mol–1

nRT n2 a
p= - 2
V - nb V
nRT (2.0 mol)(8.314 kPa dm3 K -1 mol-1 )(300 K )
Now =
V - nb (12.0 dm3 ) - (2.0 mol) (0.0427 dm3 mol-1 )

n2 a (2.0 mol) 2 (364 kPa dm6 mol-2 )


=
V2 (12.0 dm3 ) 2
States of Matter 2.15

nRT (2.0 mol)(8.314 kPa dm6 K -1 mol -1 )(300 K )


p= =
V 12.0 dm3
Applicability of van der Waals Equation

Ê a ˆ
ÁË p + V 2 ˜¯ (Vm - b) = RT
m

b Vm

Ê a ˆ a a
ÁË p + V 2 ˜¯ Vm = RT or pVm +
Vm
= RT or Z=1–
Vm RT
(31)
m

Z
a/VmRT V p Z
p

When p Vm b Vm
2
a/V m p

pb
p(Vm – b) = RT or Z=1+ (32)
RT
Z

Vm
a/Vm2 b Vm

a
p(Vm – b) = RT a/V m2
Z p
Virial Equation of State
pVm B C
Z= =1+ + + (33)
RT Vm Vm2

where B, C

Ê a ˆ RT a
ÁË p + V 2 ˜¯ (Vm - b) = RT or p= - 2
Vm - b Vm
m

-1
pVm Vm a Ê b ˆ a È b Ê b ˆ
2
˘ a
= - = Á1 - - = Í1 + + Á ˜ + ˙ -
Vm - b Vm RT Ë Vm ˜¯
or Z=
RT Vm RT ÎÍ Vm Ë Vm ¯ ˙˚ Vm RT
2
Ê a ˆ 1 Ê b ˆ
= 1 + ÁË b - ˜ + +
RT ¯ Vm ÁË Vm ˜¯
(34)
2.16 Complete Chemistry—JEE Main

Boyle Temperature
a
B=b-
RT
a
T =
Rb

Conditions for a Real Gas to Behave Ideally


nRT n2 a
p= - 2 (36)
V - nb V
b a

nRT
p= (37)
V

nRT nRT n2 a n2 a nRT nRT


= - 2 or = - (38)
V V - nb V V 2
V - nb V
a
b
to the constant a b

T and V a and b

RTb a
a= and b= (39)
1 - nb / V RT + na / V
T a and b
nab
V=
a - RTb
V a and b
a na
T= - (41)
Rb RV
V na/RV
a/Rb
a
T = (42)
Rb
Comparison of van der Waals Pressure of a Gas and the Corresponding Ideal Gas Pressure
preal < p preal > p

RTb a
preal < p a> or b<
1 - nb / V RT + na / V
RTb a
preal > p a< or b>
1 - nb / V RT + na / V
States of Matter 2.17

Critical Constants
Andrews Isotherms
p-V

T4

Fig. 12
constant pressure p
V p

vapour pressure

(4) At temperature Tc Tc

Tc

pc
the temperature Tc
Vc
temperature Tc pc
Characteristics of Critical Isotherm

critical constants
the temperature Tc

Fig. 13

Continuity of State
surface of discontinuity
2.18 Complete Chemistry—JEE Main

Critical Constants and van der Waals Constants

Ê a ˆ Ê RT ˆ 2 a ab
Vm3 - Á b + Vm + Vm - =0
ÁË p + V 2 ˜¯ (Vm - b) = RT Ë ˜
p ¯ p p
(43)
m

(Vm – Vc ) 3 Vm3 - Vc3 - 3VcVm2 + 3Vc2Vm = 0 (44)

Ê RT ˆ a ab
Vm3 - Á b + c ˜ Vm2 + Vm - =0
Ë pc ¯ pc pc

Vm

Vc = 3b; pc = a/27b2 and Tc = 8a/27Rb (46)


pV/RT

pcVc 3
= (47)
RTc 8

Inversion Temperature

2a
T = (48)
Rb

MULTIPLE CHOICE QUESTIONS ON SECTION 3

Z
(a) Z T and p Z T and p
(c) Z T and p (d) Z T and p and Z < 1 at lower T and p
a and b
factor Z,
(a) a Z > 1 and b Z a Z < 1 and b Z>1
a and b Z a and b Z<1
States of Matter 2.19

Z
(a) Z Z<1
(c) Z = 1 (d) Z > 1 or Z
3 mol–1
b

6 mol–2 and b 3 mol–1


a

a not correct?
(a) a = 27 pc b2 a = 3pcVc 2 (a) a = 27(RTc)2/64pc (d) a = 27 RVcTc/8
6 mol–2, and b 3 mol–1
a=

T) and pressure (p) are


(a) T > Tc and p > pc T = Tc and p < pc T < Tc and p > pc (d) T > Tc and p < pc
where Tc and pc
6
a mol–2 and b 3
mol–1

preal and p

(a) preal p
preal p
(c) preal p
(d) preal p
T and V a b

Ê V ˆ V ˆ Ê b ˆ Ê b ˆ
a = RTb ÊÁ (c) a = RT Á (d) a = RT ÁË ˜
Ë V + nb ˜¯ Ë V + nb ˜¯
(a) a = RTb Á
Ë V - nb ˜¯ V - nb ¯
T and V b a

a a aRT a
(a) b = b= (c) b = (d) b =
RT + na/V RT - na/V V - na RT - n 2 a / V
T a and b
n2 a n2 a nba nba
(a) V = V= (c) V = (d) V =
a - nRTb a + nRTb a - RTb a + RTb
V a and b
a na a na a n2 a a n2 a
(a) T = + T= - (c) T = - (d) T = -
Rb RV Rb RV Rb RV Rb RV 2
2.20 Complete Chemistry—JEE Main

preal and p
preal > p
RTb a RTb a
(a) a < b< (c) a < (d) b =
1 - nb / V RT + na / V 1 - nb / V RT + na / V
preal and p
preal < p
RTb RTb a a
(a) a < a> (c) b > b=
1 - nb / V 1 - nb / V RT + na / V RT - na / V

a 6 mol–2 and b 3 mol–1) at T =


3 3 K–1 mol–1)
V R

ANSWERS

HINTS AND SOLUTIONS


Z T and p

Ê a ˆ a ab Z=
pVm
=1-
a
+
pb ab
+ 2
ÁË p + V 2 ˜¯ (Vm - b) = RT or pVm + - pb - 2 = RT ;
Vm Vm RT Vm RT RT Vm RT
m

a pb
ab / Vm2 RT Z = 1- +
Vm RT RT
a Z < 1 and b Z>1
a a
Z = 1- and ZB = 1 -
Vm RT Vm RTB

a Ê 1 1ˆ a Ê 1 1ˆ
Z - ZB = - or Z -1 = -
Vm R ÁË TB T ˜¯ Vm R ÁË TB T ˜¯

T>T , Z Z>1
Ê 4 3ˆ
b = 4 NA Á pr ˜
Ë3 ¯
1/ 3 1/ 3
3 ¥ 25.12 cm3 mol -1
1/ 3
Ê 3b ˆ È ˘ 3ˆ
 ÊÁ
1 -8
r= Á =Í ˙ cm ˜¯ = 1.36 ¥ 10 cm = 13.6 nm
Ë 16 N A p ˜¯ -1
Î16(6.022 ¥ 10 mol )(3.14) ˚
23 Ë 4 ¥ 1023

8a 8(27.0 kPa dm6 mol-2 )


Tc = =
27 Rb 27(8.314 kPa dm3 K -1mol-1 )(0.024 dm3 mol-1 )

2
Ê 3RTc ˆ 27( RTc )2
a= 3 pcVc2 = 3 pc (3b) = 27 pc b
2 2;
a = 3 pcVc2 = 3 pc Á =
Ë 8 pc ˜¯ 64 pc
States of Matter 2.21

Ê 3RTc ˆ 2 9
a = 3 pcVc2 = 3 Á ˜ (Vc ) = RTcVc
Ë 8Vc ¯ 8

a 364.0 kPa dm6 mol -2


TB = =
Rb (8.314 kPa dm3 K -1 mol -1 ) (0.0364 dm3 mol -1 )


T £ Tc and p ≥ pc

2a 2(127.2 kPa dm6 mol -2 )


T = =
Rb (8.314 kPa dm3 K -1 mol -1 )(0.0318 dm3 mol -1 )

nRT n2 a
p= - 2
V - nb V
nRT (2.0 mol)(8.32 kPa dm3 K -1 mol -1 )(300 K )
Now =
V - nb (12.1 dm3 ) - (2.0 moll)(0.05 dm3 mol -1 )

n 2 a (2.0 mol) 2 (366 kPa dm6 mol -2 )


=
V2 (12.1 dm3 ) 2
p

MULTIPLE CHOICE QUESTIONS ON ENTIRE UNIT

Gaseous Laws

Ê 273.15 + t / ∞C ˆ Ê V0 ˆ
Vt=V ÁË ˜¯ (c) VT = Á T
273.15 K ˜¯
(a) Vt = a + bt (d) Vt = V t
273.15 Ë
1
(d) pV = mNu
2
(a) pV = nRT pM = rRT (c) p = rRT
3
not correct?
–1
(a) R mor–1 R 3
K–1 mol–1
3 –1
(c) R K mol–1 (d) R 3 –1
K mol–1

–1 –1 –1 –1
2.22 Complete Chemistry—JEE Main

V/T versus T
V/T versus T

V versus p
V versus p
pV versus p
pV versus p

2V 3V 2N 3N
(a) p = E p= E (c) p = E (d) p = E
3N 2N 3V 2V

(a) r2 = r1(T1 p2/T2 p1) r2 = r1 (T2T1/p2 p1) (c) r2 = r1 (p2T2/p1T1) (d) r2 = r1 (p1T1/p2T2)
–2 3
R
¥ –2) (mm3) K–1 mol–1 ¥ 12 –2) (mm3) K–1 mol–1
¥ 14 –2) (mm3) K–1 mol–1 ¥ 16 –2) (mm3) K–1 mol–1

¥ –12

¥ ¥ 11
¥ 12
¥ 13

–3 –3 –3 –3
–1
–1 –3 –3 –3
3

–1 –1 –1 –1
–3

2 3 4
–3
2

3 3 3

3 2 4

3 æÆ N2 2 2 4 æÆ N2 2
2 4

rA/r

(a) (pA /p ) (MA/M )1/2 MA/M ) (pA/p )1/2 (c) (pA/p ) (M /MA)1/2 (d) (MA/M ) (p /pA)1/2

r r M = 3M r and M

(a) p p p p =4 (c) p p =6 (d) p p = 1/6


States of Matter 2.23

2 2 4

r1 p1 M1 r1 p1 M2 r1 p1 M1 r1 p1 M 2
(a) = = (c) = (d) =
r2 p2 M2 r2 p2 M1 r2 p2 M 2 r2 p2 M1
R correct?
3K–1 mol–1 3 K–1 mol–1
–1 mol–1 ¥ 12 –2) (mm3) K–1 mol–1

2 2 2 2
r(A) = 2r( ) and 2M(A) = M( ), the
p(A) and p

r
r

(a) 1 3 31 (c) 1/3 (d) 3/1


2

2 2
pVm)p Æ
3 3 3 3

Speeds and Kinetic Energy

(a) 3RT/M 3kBT/m (c) 2RT/M (d) 3k T/m

(a) 3RT/M 2RT/M (c) 8RT / pM (d) 8kBT/pM

(a) 3kBT/m 2kBT/m (c) 8kBT/pm (d) 3RT/M


2.24 Complete Chemistry—JEE Main

3 1
(a) ¥ 8.314 ¥ 273 J mol-1 ¥ 8.314 ¥ 273 J mol-1
2 2
3 8.314 ¥ 273 ˆ
(c) 3 ¥ ¥ –1 (d) ÊÁ ˆ˜ ÊÁ
Ë 2 ¯ Ë 6.022 ¥ 1023 ˜¯
J

Ê3 ˆ
(a) ÊÁ RT ˆ˜ mol
3
RT) mol (c) Á kBT ˜ mol (d) (3 k T) mol
Ë2 ¯ Ë2 ¯

(a) (urms) 2
> (urms) 4
> (urms) 3
> (urms) 2
urms) 2
< (urms) 4
< (urms) 3
< (urms) 2
(c) (urms) 4
< (urms) 4
> (urms) 3
> (urms) 2
(d) (urms) 2
> (urms) 4
< (urms) 3
< (urms) 2
urms), uav
speed (ump
(a) ump uav urms ump urms uav
(c) uav ump ums uav urms ump?

2 2 2
E)
(a) E 2) = E 2) = E 2 E 2) >E 2) >E 2)
(c) E 2 ) < E 2) < E 2) (d) E 2 >E
) 2 <E
) 2)

(a) (1/2)k T k T (c) (3/2)k T (d) RT

–1 –1 –1 –1

–3

(a) 1414 ms–1 –1 –1


(d) 866 ms–1

–1
–1 –1 –1 –1

urms = 3 p/r .
temperature
(a) urms μ p urms μ 1/ r (c) urms μ pVm (d) urms μ p / Vm

7
(a) T 2) = T(N2) T 2) > T(N2) (c) T 2) < T(N2) (d) T 2) = 7 T(N2)

CV, m = (3/2) R
States of Matter 2.25

Cp, m = (3/2) R
Cp, m and CV, m R
Cp, m and CV, m R
2

Real Gases

b r

Ê 4 3ˆ
(a) b = ÊÁ p r 3ˆ˜ N A b = ÊÁ p r 3ˆ˜ (d) b = 4 ÊÁ p r 3 ˆ˜ N A
4 4 4
(c) b = 2 Á p r ˜ N A
Ë3 ¯ Ë3 ¯ Ë3 ¯ Ë3 ¯

Ê p + na ˆ (V - nb) = nRT
(a) ÊÁ p + 2 ˆ˜ (V - nb) = nRT
na
ÁË ˜
Ë V ¯ V¯
Ê n2 a ˆ Ê n2 a ˆ
(c) Á p + (V - nb) = nRT (d) Á p + 2 ˜ (V - nb) = nRT
Ë V ˜¯ Ë V ¯
a
3 3
mol–1 6 mol–2

Z(= pVm /RT )


pressure?
(a) Z = l – a/Vm RT Z = 1 + a/Vm RT (c) Z = 1 + pb/RT (d) Z = 1 – pb/RT
Z(= pVm/RT)
pressure?
(a) Z = 1 – a/Vm RT Z = 1 + a/Vm RT (c) Z = 1 + pb/RT (d) Z = 1 – pb/RT

(a) T = aR/b T = ab/R (c) T = a/Rb (d) T = aRb

(a) T = 2 a/Rb T = a/2Rb (c) T = 2 aR/b (d) T = 2 aRb

T and low p T p (c) low T and low p (d) low T p

(a) low T p T and low p T p T and low p

(a) low T and low p T p T p T and low p


6
a 2, N2 3 4 atm
mol
2 2 3 4
2.26 Complete Chemistry—JEE Main

(a) Vm – b p + a/V 2m (c) RT (d) 1/RT


Vm
3 3
3

Z) versus p
At low temperature and low pressure, Z

(a) the constant a b b a


a and b a and b

(a) m3 3 mol–1 3

a?
2 3 2
b?
2 2 2 2
p, Z versus pressure p

p, Z versus pressure p 2

l = l/( 2 p s 2 N*) where s N*

(a) l μ p/T l μ (p/T)2


(c) l μ T/p (d) l T and p
b
(a) Vc = b Vc = 2b (c) Vc = 3b (d) Vc = 4b
not correct?
(a) Tc = 8a/(27Rb) pc = a (3b2) (c) pcVc = (3/8) RTc (d) Vc = 3b
States of Matter 2.27

f1 and f2 c and c + dc 2 at 2T 2 and T,

(a) f1 > f2 f1 < f2


(c) f1 = f2 (d) f1 and f2
Miscellaneous Questions

H 2 (g ) He(g ) N 2 (g ) O 2 (g )
(I) (II) (III) (IV)
T (=

decreased to
2.28 Complete Chemistry—JEE Main

g Cp, m and CV, m

Z p

ANSWERS

(d) (c)

HINTS AND SOLUTIONS


rH 2 M He
= = 2 = 1.414
rHe M H2
Ê 725 mmHg ˆ -1
pVM ÁË 760 mmHg ¥ 1 atm˜¯ (0.75 L)(71 g mol )
pV = nRT =
m
RT m= = = 2.11 g
M RT (0.082 atm L K -1 mol -1 )(293 K )
States of Matter 2.29

pM (1 atm)(28 g mol -1 )
r= = = 1.25 g L-1
RT (0.082 L atm K -1mol -1 )(273 K )
V= V + at and V = KT
pV = K

pV = mNu 2 = N ÊÁ mu 2 ˆ˜ . p = ÊÁ ˆ˜ E
1 2 1 2 N
3 3 Ë2 ¯ 3ËV ¯
m pM r2 Ê p2 ˆ Ê T1 ˆ
pV = nRT = r= =
r1 ÁË T2 ˜¯ ÁË p1 ˜¯
RT
M RT
–2 –2 2 cm)–2 –4 –1 s–2

–3 –2 ¥ 6 –1 s–2 ¥ 10 –2

Vm 3 mol–1 ¥ 6 mm3 mol–1

pV (1.103 ¥ 1010 dyn m -2 )(22.414 ¥ 106 mm3 mol -1 ) –2)


R= = ¥ 14 (mm3) K–1 mol–1
T (273 K )
pVN A (101.325 ¥ 10 -12 kPa )(1 dm3 )(6.022 ¥ 1023 mol -1 )
N= = = 7.244 ¥ 1010
RT (8.314 kPa dm3 K -1 mol -1 )(101.325 K )

pM (3 ¥ 101.325 kPa ) (17 g mol -1 )


r= = = 2.07 g dm -3
RT (8.314 kPa dm3 K -1 mol -1 )(300 K )

Ê p ˆÊT ˆ Ê 700 Torr ˆ Ê 273 K ˆ


r2 = r1 Á 2 ˜ Á 1 ˜ = (1.43 g L-1 ) Á = 1.20 g L-1
Ë T2 ¯ Ë p1 ¯ Ë 300 K ˜¯ ÁË 760 Torr ˜¯

r RT (0.001 kg dm -3 )(8.314 kPa dm3 K -1 mol -1 )(300 K )


M = = = 0.0246 kg mol -1 = 24.6 g mol -1
p (101.325 kPa )
r RT (2.52 g dm -3 )(8.314 kPa dm -3 K -1 mol -1 )(600 K )
M = = = 124 g mol -1
p (101.325 kPa )
124
=4
31

r RT (0.543 g dm -3 )(8.314 kPa dm3 K -1 mol -1 )(300 K )


M av = = = 13.37 g mol -1
p (101.325 kPa )
–1
If x x M + (1 – x)M 2
x(4) + (1 – x
x
2
2.68
¥ 100 = 20.2
2.68 + 10.56
ÊT ˆÊ p ˆ Ê 300 K ˆ Ê 760 Torr ˆ
V2 = V1 Á 2 ˜ Á 1 ˜ = (136.5 cm3 ) Á = 162.9 cm3
Ë p2 ¯ Ë T1 ¯ Ë (725 - 25) Torr ¯ ÁË 273 K ˜¯
˜

x 2 4
x + 2(1 – x)] mol = (2 + x) mol
Now pV = nRT
2.30 Complete Chemistry—JEE Main

p1 n1T1 Ê 0.5 atm ˆ = Ê 1 ˆ Ê 300 K ˆ 4.5


- 2 = 0.25
= ÁË ˜ Á ˜
4.5 atm ¯ Ë 2 + x ¯ ÁË 1200 K ˜¯
; or x=
p2 n2T2 0.50 ¥ 4

2 4

r μ1/ M Also, r μpA


where pA

1/ 2
rA pA Ê mB ˆ
=
rB pB ÁË M A ˜¯

pV = nRT = (m/M)RT fi pM = (m/V) RT = rRT


pX r M Y Ê rX ˆ Ê M Y ˆ
= X = =2¥3=6
pY M X rY ÁË rY ˜¯ ÁË M X ˜¯
n
–1 –1
(a) n n
–1 –1
(c) n = 1 mol (d) n

r1 Ê 1 ˆ n /t 1 n 1
r1 = (M2/M1)1/2r2 –1 –1)1/2r = (1/4)r2 fi =Á ˜ fi 1 = fi 1 =
2
r2 Ë 4 ¯ n2 / t 4 n2 4
n1 = n2 –1)/4= (3/16) mol and m1 –1
–1 mol–1
R
3 –1 mol–1 3 –1 mol–1
R
R 3 atm K–1 mol–1 3 atm K–1 mol–1
R –1 mol–1 –1 mol–1 –2) 3 mm)3 K–1 mol–1
¥ 14 –2) (mm3) K–1 mol–1

pA r M B rA M B
p = rRT/M = A = = (2) (2) = 4
pB M A rB rB M A

r1 T2 Êr ˆ Ê r ˆ
p = rRT/M = fi T2 = Á 1 ˜ T1 = Á (300 K ) = 600 K
r2 T1 Ë r2 ¯ Ë 0.5 r ˜¯

u1 r2 1
urms = 3 p / r , we get = =
u2 r1 3
1/ 3
r1 t2 M2 t = 4 / 2 (5 s) = 5 2 s
= =
r2 t1 M1

t = 32 / 2 (5 s) = 20 s t = 28 / 2 (5 s) = 5 14 s

t = 44 / 2 (5 s) = 5 22 s
States of Matter 2.31

pVm)pÆ 3 3 3

RT
Ê 3 RT ˆ
ÁË ˜¯ = (3 mol) RT
2
urms = 3RT / M , urms μ 1/ M . M H 2 < M CH 4 < M NH3 < M CO2

ump = 2 RT / M , uav = 8 RT / pM , urms = 3RT / M

ump : uav : urms :: 2 : 8 / p : 3 :: 1.414 : 1.595 : 1.732 :: 1 : 1.128 : 1.224

urms = 3RT/M ; urms p or V at


constant temperature as pV

1/ 2
3RT È 3(8.314 J K -1 mol -1 ) (480 K ) ˘
urms = =Í ˙ = 499.4 m s -1 ª 500 m s -1
M Î 48 ¥ 10 -3 kg mol -1 ˚

2 RT 2 R (300 K )
=2 . T
M M
3RT 3p 3 ¥ 100 ¥ 103 Pa
urms = = = = 1732 m s -1
M r 0.1 kg m -3

È 3 (8.314 J mol-1 K -1 ) (300 K ) ˘


ÍÎ 2 ˙˚

uav = 8 RT / pM , we have

(uav )927 º C È (927 + 273) K ˘ Ê 1200 ˆ 1 / 2


= =Á ˜ =2 fi (uav )927ºC = 2(0.3 m s -1 ) = 0.6 m s -1
(uav ) 27 º C ÍÎ (27 + 273) K ˙˚ Ë 300 ¯

p/r urms = 3RT / M we have


urms μ RT = pVm

T (327 + 273) K
urms = 3RT / M , we have = fi T ∫
4 16
T (H 2 ) T (N2 )
(urms )H 2 = 7 (urms ) N 2 = ( 7 )2 or T 2) = T(N2)/2
2 28

CV, m = (3/2) RT and


Cp, m RT
u1 T M
urms = 3RT / M , we have = 1 2
u2 T2 M1
2
ÊM ˆÊu ˆ 64 1 2
T2 = Á 2 ˜ Á 2 ˜ T1 = ÊÁ ˆ˜ ÊÁ ˆ˜ (300 K ) = 1200 K
Ë M1 ¯ Ë u1 ¯ Ë 4 ¯ Ë 2¯
2.32 Complete Chemistry—JEE Main

b
a/V 2m 2 ) (V – b) = RT
p + a/V m m p
pVm pb
p(Vm – b) = RT fi pVm – pb = RT fi Z= =1+
RT RT
b Vm

Vm, real,/V Vm, real > V V 3 mol–1


a
Z =1- Z
Vm RT
Z = 1 + pb/RT. Z

b Vm
Ê a ˆ pVm a
ÁË p + V 2 ˜¯ (Vm ) = RT or pVm = RT - a / Vm or Z = RT = 1 - V RT
m m

N N pN A p
pV = nRT = RT fi N * = = =
NA V RT kT
kT
l= l μ T/p
2ps 2 p
c and c + dc
3/ 2
dN 1 Ê M ˆ
= 4p Á exp( - Mc 2 / RT )c 2
Ë 2 p RT ˜¯
f=
N dc
M 2)/2T =M 2)/T, hence, f

N = nNA = (m/M)NA fi N μ (1/M

crms = 3RT / M fi crms μ 1 / M ,


n(3/2) RT = (m/M) (3/2) RT fi μ1/M,
a

2 b
Tc and pc
Tc and pc

M ˆ 3 / 2 - Mu 2 / 2 RT 2
= 4p ÊÁ
dN 1
Ë 2p RT ˜¯
e u
N du
M ˆ 3 / 2 -1
= 4p ÊÁ
dN 1
ump = 2 RT / M , e (2 RT / M ) μ M1/2
N du Ë 2pRT ˜¯

T = a/Rb T μ a/b a
r μ1/ M
a
T = 2a/Rb
(mkB t )1 / 2
h= , where s =2r μ b1/3 h μ m1/2/b1/3
p3 / 2s 2
least value of m1/2/b1/3
States of Matter 2.33

–1) = –1
RT 3R T
RT –1
) = 3R –1
T ¥ 32/28) K = 343 K
(crms )H 2 = 3R (300 K ) /(0.002 kg mol -1 ) and (cav ) He = 8R (300 K ) /(p ¥ 0.004 kg mol -1 )
(crms )H 2 (3 / 2)
= >1
(caV ) He 8 /(4p)

g g
Tc
Z = 1 + pb/ b
Z = 1 – a/Vm RT a

MULTIPLE CHOICE QUESTIONS FROM AIEEE AND JEE-MAIN

313 / 293 (c) 313/293 (d) 2 [2004]


b

[2004]
not

[2005]

1 273 1 1
(a) 2 ¥ (c) (d) [2007]
3 3 298 3 2
–4 dm3 3

–1
2 R mol–1
¥ –3 ¥ –3 ¥ –2 ¥ –2 mol [2010]
a b
(a) a 2> a 2 6 b 2 <b 2 6 a and b 2> a and b 2 6
(c) a and b 2< a and b 2 6 (d) a 2 <a 2 6 b 2 >b 2 6
[2011]

[2011]
2.34 Complete Chemistry—JEE Main

(a) 1 + RT/pb pb/RT (d) 1 – pb/RT [2012]


c c
c
(a) c*: c : c = 1.225 : 1.128 : 1 c*: c : c = 1.128 : 1.225 : 1
(c) c*: c : c = 1 : 1.128 : 1.225 (d) c*: c : c = 1 : 1.225 : 1.128 [2013]
Z

(a) Z = 1 – a/VmRT Z = 1 – pb/RT (c) Z = 1 + pb/RT (d) Z = 1 + RT/pb [2014]

[2003]

n 2
- a ÊÁ ˆ˜
nRT
p=
V - nb ËV ¯
p = nRT/V, when

[2014]

[2014, online]

[2014, online]
3
2
3
2

[2014, online]
not

[2015, online]

[2015, online]
V p and temperature T1 are

T2 pf
T1 T1 T1 T2
pi, V pi, V pf, V pf, V
States of Matter 2.35

Ê T1 ˆ Ê T2 ˆ Ê TT ˆ Ê TT ˆ
(a) 2p Á p Á (c) 2p Á 1 2 ˜ (d) p Á 1 2 ˜ [2016]
Ë T1 + T2 ˜¯ Ë T1 + T2 ˜¯ Ë T1 + T2 ¯ Ë T1 + T2 ¯
2 u

(a) 2u u (c) 4u (d) u/2 [2016, online]

(a) 1 + pb/RT pb/RT (c) 1 – pb/RT (d) 1 – b/VRT


[2016, online]

ANSWERS

HINTS AND SOLUTIONS


k
(KE) 40 º C (273 + 40) K 313
= =
(KE) 20 º C (273 + 20) K 293
b

–1 –1 m
2
m m nO2 (m / 32 g) 1 / 32 1
nCH 4 = and nO2 = xO2 = = = =
16 g mol -1 32 g mol-1 nCH 4 + nO (m / 16 g) + (m / 32 g) 3 / 32 3
2

pV (3170 Pa )(10 -3 m3 )
n= = ¥ –3
mol
RT (8.314 J K -1 mol -1 )(300 K )
a
a 2 a 2) >
a 2 6
b
b 2 6 2 b 2) <b 2 6

crms = 3RT / M ; cav = 8RT / pM ; cmp = 2 RT / M

Ê a ˆ
ÁË p + V 2 ˜¯ (Vm - b ) = RT
m

a / Vm2 << p p(Vm – b) = RT or pVm – pb = RT


pVm pb
Z= =1+
RT RT
2.36 Complete Chemistry—JEE Main

2 RT 8 RT 3RT
c*: c : c :: : : :: 2 : 8 / p : 3 :: 1 : 4 / p : 3 / 2 :: 1 : 1.128 : 1.225
M pM M
Ê a2 ˆ
ÁË p + 2 ˜¯ (Vm - b) = RT
Vm
At low pressure, Vm b Vm
Ê a ˆ pVm a
=1-
ÁË p + V 2 ˜¯ Vm = RT or Z=
RT Vm RT
m

nb and a(n/V)2
3 RT 3 R(300 K )
-1
= T
32 g mol 4 g mol -1

Vi pi (750.0 mL)(840 mmHg)


Vf = = = 1750 mL = 1.750 L
pf (360 mmHg)
Note

1/ 2
rO2 Ê M SO2 ˆ 64 1/ 2
=Á ˜ = ÊÁ ˆ˜ = 1.414 ; V ¥V = 3 3
rSO2 Ë M O2 ¯ Ë 32 ¯ 2 2

piV piV Ê pV ˆ
n1 + n2 = + = 2Á i ˜
RT1 RT1 Ë RT1 ¯
pf V pf V pVÊ1 1ˆ p V ÊT +T ˆ
n1 + n2 = + = f Á + ˜ = f Á 2 1˜
RT1 RT2 R Ë T1 T2 ¯ R Ë T1T2 ¯

pf V Ê T2 + T1 ˆ pV Ê T2 ˆ
= 2 i pf = 2p Á
Á
R Ë T1T2 ¯ ˜ RT1 Ë T1 + T 2 ˜¯

3RT
urms =
M
T M

3R(2T ) 3RT
urms = = 2
M /2 M
Ê a ˆ
Á p + 2 ˜ (Vm – b) = RT
Ë Vm ¯
a/V m2 p
p(Vm – b) = RT
pVm pb
Z= =1+
RT RT
States of Matter 2.37

UNIT 2 LIQUID STATE

Intermolecular Forces

Van der Waals Forces

Dipole–Dipole Forces

1
E μ
r3

Dipole-Induced Dipole Forces

1
E μ
r6

Fig. 1

Dispersion Forces

Explanation

1
E μ
r6

Fig. 2
2.38 Complete Chemistry—JEE Main

A Few Facts Regarding Dispersion Forces

Examples
2 2 2
Ion-Dipole Forces

1
E μ
r2

Ion-Induced Dipole Forces

1
E μ
r4

Fig. 3

Hydrogen Bond

Explanation Fig. 4a

Characteristics of the Hydrogen Bond

–1

Examples
Fig. 4b
3 2
States of Matter 2.39

3 2

Fig. 5

Fig. 6

Vapour Pressure

p2 D vap H m Ê 1 1 ˆ
log = - -
p1 2.303R ÁË T2 T1 ˜¯
where DvapHm

Surface Tension

–1 3 –1
g

g (Mv)2/3 = k(tc – t – 6)
where M v tc
2.40 Complete Chemistry—JEE Main

1
g = r r gl
2
where g r r g
l

mg = (2 p r) g
Fig. 7
where r r,

We have
m1g = (2p r)g1 mwg = (2p r)gw
gw = (mw/m1)g

V = nv = n(m/r) fi m = Vr/n

g1 = (r1/rw) (nw/n1) gw

Viscosity

–2 –2

h=A E/RT)
where A E
A

p r 4 p1 t1 p r 4 pw t w
h1 = and hw =
8l v 8l v
Fig. 8
Ê p t ˆ
h1 = Á 1 1 ˜ hw
Ë pw t w ¯
p
States of Matter 2.41

Ê r t ˆ
h1 = Á 1 1 ˜ hw
Ë rw tw ¯

mg
mg = 6p h r v fi h=
6p r v

where r v m

MULTIPLE CHOICE QUESTIONS ON ENTIRE UNIT

h
h versus T h versus 1/T
(c) h = E/RT h versus 1/T
not true?
–2 2 s–1

(c) N m–2 –1 N m–2 s

not correct?

D vap H m 1
log p = + constant
2.303 R T

not correct?

–1

not correct?

g (Mv)2/3 = k(tc – t

(d) 1 N m–1 3 –1

not correct?
2.42 Complete Chemistry—JEE Main

not correct?

h=A E/RT)

[2015]

[2016, online]

ANSWERS

HINTS AND SOLUTIONS

p = DvapHm RT) + constant


h=A Ea/RT
States of Matter 2.43

UNIT 3 SOLID STATE

SECTION 1 Crystal Systems

Crystalline Solids

Amorphous Solids

Molecular Crystals (or van der Waals Crystals)

2 4, I 2 and most of the

Ionic Crystals

Covalent Crystals
2.44 Complete Chemistry—JEE Main

Structure of diamond Structure of graphite


Fig. 1

Metallic Crystals

Arrangement of Constituents in a Crystal—Space Lattice

Unit Cell

Unit Cell in 1-Dimensional Lattice

Unit Cell in 2-Dimensional Lattice

a a a a

b b b b
a=b aπb a=b a=b aπb
– = 90° – = 90° – = 60° – π 90°, 60° – π 90°, 60°
(I) (II) (III) (IV) (V)

Fig. 2

Unit Cell in 3-Dimensional Lattice

three vectors a, b and c

Fig. 3
States of Matter 2.45

Seven Crystal Systems


1

Fourteen Bravais Lattices

Primitive Unit Cell


primitive unit cell
Nonprimitive Unit Cells
triad axis
(four of this type)
nonprimitive unit cell
Fig. 4

Crystal Faces

Bragg’s Law Fig. 5

nl = 2d q where n = 1,2,3, … Fig. 6

Derivation of Bragg’s law


de + ef = be sin q + be sin q = d q+d q = 2d q

Fig. 7
2.46 Complete Chemistry—JEE Main

Table 1 Seven Crystal Systems

Crystal Minimum Parallelopiped Unit Cell Examples


System symmetry of dimensions
rotational axis

aπbπc 4 2
1
aπbπgπ K2 2 7

triclinic

aπbπc
2
a=b= ºπg Na2
4 2
4 2

aπbπc 4,
222
a=b=g= º 3, K2 4

a=b=c
3
a=b=gπ º 3

a=b=c
3333
a=b=g= º 2

a=bπc
4
a=b=g= º 2 4

a=bπc 2
6
a = b = º, g = º 2

Note
States of Matter 2.47

Table 2 Fourteen Bravais lattices


System Bravais lattices Number

P I F

P I

P I C F

P C

one each
1+1+1
2.48 Complete Chemistry—JEE Main

nl = 2d q (1)
where n d l q
l and q

Study of Cubic Systems


Primitive Cubic System
Arrangement of Points
Arrangement of Atoms in the Unit Cell.

Number of Atoms per Unit Cell

8 ÊÁ atomˆ˜
1
Ë8 ¯

8
7 6

3 4
1 2

Fig. 8 Fig. 9 Fig. 10

Unit Cell Sliced out of Crystal Lattice.


(v) Size of Atom Relative to Edge Length. 2r = a
Coordination Number of an Atom.

Fig. 11 Fig. 12
States of Matter 2.49

Density of Element
General Expression
N
Ê M ˆ
m = m aN = Á N
Ë N A ˜¯
a
V = a3
m (M / NA ) N
r= = (42)
V a3
M / NA
N r=
a3
Void Volume in Unit Cell r=a
4
a3 p (a/2)3
3
(4 / 3) p (a / 2)3 p
=
a3 6

Body Centred Cubic System


Arrangement of Points

Arrangement of Atoms in the Unit cell.

Number of Atoms per Unit Cell.


Ê1 ˆ
Share from corner atoms = 8 ÁË atom ˜¯ Fig. 13 Fig. 14
8

Share from the atom at the centre = 1 atom

Unit cell sliced out of the crystal Lattice

(v) Size of Atom Relative to Edge Length

r + 2r + r = 3a or 4r = 3a
Coordination Number of an Atom
eight Fig. 15
Density of Element N
N ( M/N A ) 2( M/N A )
r= 2
=
a a3
Void Volume in Unit Cell r= 3a
2.50 Complete Chemistry—JEE Main

a3
È4 Ê 3 ˆ3˘
2Í pÁ a˜ ˙
ÍÎ 3 Ë 4 ¯ ˙˚

3p
8

Face-Centred Cubic System


Arrangement of points.

Arrangement of Atoms in the Unit


Cell.

Number of Atoms per Unit Cell. Fig. 16 Fig. 17

Ê1 ˆ
Share from corner atoms = 8 Á atom˜ = 1 atom
Ë8 ¯

6 ÊÁ atomˆ˜ = 3 atoms
1
Ë2 ¯
Fig. 18
Unit Cell Sliced out of the Crystal Lattice.
(v) Size of Atom Relative of Edge Length.
r + 2r + r = 2a r= 2a
Coordination Number of Atoms.

Density of Element. N
N ( M / N A ) 4( M / N A )
r= =
a3 a3
Void volume in the Unit Cell.
r = 2a
Fig. 19
a3

È 4 Ê 2 ˆ3 ˘ 2p
4Í pÁ a˜ ˙
ÍÎ 3 Ë 4 ¯ ˙˚ 6
States of Matter 2.51

MULTIPLE CHOICE QUESTIONS ON SECTION 1

3 2
2 3
3
2

2
a π b π c and a π b π g

aI /a

3
2.52 Complete Chemistry—JEE Main

–1

–3 –3 –3 –3

–1 –3

–1

–3 –3 –3 –3

r rI r

(a) l = d q nl = d q (c) nl = 2d q (d) l = nd q

ANSWERS

HINTS AND SOLUTIONS

3 2

a π b π c and a π b π g
States of Matter 2.53

3aI = 4r and 2aF = 4r 3aI = 2aF or aI/a = 2/3


a3 3 fi a r= 2a
r = a/2 2 nm / 2 2
4r = 3a fi r = ( 3 / 4)a = (0.433)(350 pm) = 151.6 pm
a 286.4 pm
= = 2.31
r 124 pm

a/r = 4 / 2 a/r = 4 / 3

a 300 pm
= = 2.83
r 212 pm / 2

N (M / NA ) (1)(240.88 g mol -1 ) (6.022 ¥ 1023 mol -1 )


r= = –3
a3 (2 ¥ 150 ¥ 10 -10 cm)3

N (M / NA ) (1)( M / N A ) N (M / NA ) (2)( M / N A )
r = 3
= rI = =
a ( 2 r )3 a 3
[(4 / 3 )r ]3

rP 1 ( 4 / 3 )3
= 3 = 0.769
rI ( 2 ) 2

r = N ( M / N A ) / a3 = N ( M / N A ) /(2 2r )3, we have

1/ 3
1 È 4(197.0 g mol -1 ) /(6.022 ¥ 1023 mol -1 ) ˘
1/ 3
1 È N (M / NA ) ˘
r= Í ˙ = Í ˙
2 2Î r ˚ 2 2Î 19.7 g cm -3 ˚

1
= (4.05 ¥ 10-8 cm) = 1.432 ¥ 10 -8 cm = 143.2 pm
2 2

N (M / NA ) (2)(48.0 g mol -1 ) /(6.022 ¥ 1023 mol -1 )


r= = –3
a3 ÈÎ (4 / 3 )(100 ¥ 10 -10
cm) ˘˚
3

3
N1 ( M / N A ) N2 (M / NA ) r1 N Êa ˆ
r1 = and r2 = = 1 Á 2˜
a13 a23 r2 N 2 Ë a1 ¯

3
rP 1 È (4 3 ) r ˘
= ÊÁ ˆ˜ Í
rI Ë 2 ¯ Î 2r ˙˚

3
rP Ê 1 ˆ È 2 2r ˘
= ÁË ˜¯ Í ˙
rF 4 Î 2r ˚

3
rI Ê 2 ˆ È 2 2r ˘
= ÁË ˜¯ Í ˙
rF 4 Î 2r ˚
2.54 Complete Chemistry—JEE Main

r = N(M/NA)/a3, we have

(1) ( N / N A ) (2) ( M / N A ) (4) ( M / N A )


r = 3 ; rI = 3 and r =
( 2r ) ÈÎ(4 3 ) r ˘˚ ( 2 2 r )3

1 2 4
r rI r 3
: :
2 ( 4 / 3 ) ( 2 / 2 )3
3

nl = 2d q

nl (1)(250 pm)
d= =
2 sin q (2)(sin 30º )

SECTION 2 Closest Packings of Atoms

Closest Packing in Elements

twelve

Two Types of Packing

Fig. 20

Fig. 21
States of Matter 2.55

Hexagonal Closest-Packed Structure hexagonal closest-packed structure

Fig. 22

Cubical Closest-Packing cubical closest-packing face-centred cubic


packing

Fig. 23
2.56 Complete Chemistry—JEE Main

ab r 2r
ad = ad = =
cos 30º ( 3 / 2) 3

C cd

(ac) 2 = (ad )2 + (cd ) 2

2r 2 8r 2
(cd )2 = (ac) 2 - (ad )2 = (2r )2 - ÊÁ ˆ˜ = (2r ) 2 ÊÁ ˆ˜ =
2
or
Ë 3¯ Ë 3¯ 3

Fig. 24 Fig. 25 Fig. 26

C 2 2r 4r 8r
= cd = = C=
2 3 6 6

r r
3r

6 ÍÈ ( 3r )(2r ) ˙˘ = 6 3r 2
1
Î2 ˚

2 Ê 8 ˆ
= (6 3r ) ÁË r ˜ = 24 2r 3
6 ¯
States of Matter 2.57

Number of spheres belonging to this prism we have

Volume occupied by Atoms 6 È (4/3) pr 3 ˘˚


= Î
2
f= = p
Volume of hexagon (24 2 )r 3 6

Types of Voids in Closest Packing of Atoms

Tetrahedral Void

Number of Voids per Closest packed Atom

Fig. 27 Fig. 28

Number of Tetrahedral Voids

a
3 ( a / 2) / 2 = 3a tetrahedral
2( 3a / 4) = 3a / 2
2.58 Complete Chemistry—JEE Main

Fig. 29a Fig. 29b

Fig. 30a Fig. 30b Fig. 30c

Placement of Voids in Closest-structure of Atoms

Second closest-packed layer

Tetrahedral holes

Octahedral holes

Tetrahedral holes

First closest-packed layer

Fig. 31
States of Matter 2.59

Size of Atom Placed in Tetrahedral Void A tetrahedral void is formed when atoms
A are placed in the alternate corners of a cube as shown in Fig. 32. The centre of the
cube is the tetrahedral void. In closest-structure of A atoms, if an atom B is placed in
the tetrahedral void such that it touches all the four A atoms, we will have
rA + rA = 2 a (Atoms A touch each other along the face-diagonal of the cube of
edge length a)
rA + 2rB+ rA = 3 a (Atoms A and B touch each other along the body-diagonal of the
cube of edge length a)

Hence 2rB = 3 a – 2 rA = 3 a – 2 a = 0.318 a


or rB = 0.159 a
Fig. 32
rB 0.159a
The ratio rB / rA is = = 0.159 ¥ 1.414 = 0.225 (10)
rA a/ 2
Size of Atom placed in Octahedral Void The centre M in Fig. 33 is an octahedral void. In closest structure of A
atoms, if an atom B is placed in the octahedral void such that it touches all the six A atoms, we will have
rA + 2rB + rA = a
In the face-centred cubic unit cell, atoms A touch each other along the face
diagonal. Hence, 4rA = 2a or 2rA = a/ 2 . This also follows from the
smaller cube,
2 2
Ê aˆ + Ê aˆ = a
rA + rA = ÁË ˜¯ ÁË ˜¯
2 2 2

a Ê 2 - 1ˆ 0.414
= aÁ = a
2 ˜¯
Hence, 2rB = a – 2rA = a –
2 Ë 2
0.414
or rB = a
2 2 Fig. 33

rB (0.414 / 2 2)
The ration rB/rA is = a = 0.414 (11)
rA a/2 2

Formula of Compound and Computation of Void Volume

Illustration 1 In a cubical closest packing, atoms A occupy lattice points and atoms B occupy tetrahedral voids. What
is the simplest formula of the compound and fraction of void volume per uint volume of unit cell?
Number of atoms A = 1 + 3 =4
(from corners) (from face-centres)
Number of atoms B = Number of tetrahedra voids = 8
Hence, the formula is A4B8 or the simplest formula is AB2.
Volume occupied by atoms in one unit cell

u = 4 ÊÁ prA3 ˆ˜ + 8 ÊÁ prB3 ˆ˜
4 4
Ë3 ¯ Ë3 ¯
Since rB/rA = 0.225, we have

u = 4 ÊÁ prA3 ˆ˜ + 8 ÈÍ p(0.225rA )3 ˘˙ = 4 ÊÁ prA3 ˆ˜ ÈÎ1 + 2(0.225)3 ˘˚ = 4 ÊÁ p rA3 ˆ˜ (1.023)


4 4 4 4
Ë3 ¯ Î3 ˚ Ë 3 ¯ Ë 3 ¯

Volume of unit cell is V = a3 = ( 2 2 rA)3 = 16 2 r3A


2.60 Complete Chemistry—JEE Main

4 Ê prA3 ˆ (1.023)
4
u Ë3 ¯ 1.023p
Fraction of volume occupied is f= = = = 0.758
V 3
16 2rA 3 2
Fraction of void volume = 1 – 0.758 = 0.242
Illustration 2 In a face-centred cubic arrangement of atoms A, atoms B occupy octahedral voids. What is the simplest
formula and the fraction of volume of unit cell occupied by atoms?
Solution: Number of atoms A = 4
Number of atoms B = Number of octahedral voids = 4
Hence, the formula is A4B4 or the simplest formula is AB.
Volume occupied by atoms in one unit cell is

u = 4 ÊÁ prA3 ˆ˜ + 4 ÊÁ prB3 ˆ˜
4 4
Ë3 ¯ Ë3 ¯
Since rB/rA = 0.414, we have

u = 4 ÊÁ prA3 ˆ˜ + 4 ÈÍ p(0.414 rA )3 ˘˙ = 4 ÊÁ prA3 ˆ˜ ÈÎ1 + (0.414)3 ˘˚ = 4 ÊÁ prA3 ˆ˜ (1.071)


4 4 4 4
Ë3 ¯ Î3 ˚ Ë 3 ¯ Ë 3 ¯
3
Volume of unit cell is V = a3 = (2 2 rA)3 = 16 2 rA

4 Ê prA3 ˆ (1.071)
4
u Ë3 ¯ 1.071 p
Fraction of volume occupied is f= = = = 0.793
V 16 2rA3 3 2

Illustration 3 In a cubical-closest packing of atoms A, 3/4th of tetrahedral voids are occupied by atoms B. What is
the simplest formula and the fraction of volume per unit cell is occupied?
Solution: Number of atoms A = 4

3 3
Number of atoms B = (Tetrahedral voids) = (8) = 6
4 4
Hence, the formula is A4B6 or the simplest formula is A2B3.

u = 4 ÊÁ prA3 ˆ˜ + 6 ÊÁ prB3 ˆ˜
4 4
Volume occupied by atoms is
Ë3 ¯ Ë3 ¯
Since rB/rA = 0.225, we have
3
u = 4 ÊÁ prA3 ˆ˜ + 6 ÊÁ p(0.225 rA )ˆ˜ = 4 ÊÁ prA3 ˆ˜ [1 + 1.5(0.225)3 ] = 4 ÊÁ prA3 ˆ˜ (1.017)
4 4 4 4
Ë3 ¯ Ë3 ¯ Ë3 ¯ Ë3 ¯
3
Volume of unit cell is V = a3 = (2 2 rA)3 = 16 2 rA

4 Ê prA3 ˆ (1.017)
4
u Ë3 ¯ 1.107p
Fraction of volume occupied is f= = = = 0.753
V 16 2rA3 3 2
Illustration 4 In a face-centred cubic arrangement of A and B atoms, where A atoms are at the corners of the unit cell
and B atoms at the face-centres, one of the A atoms is missing from one corner in each unit cell. What is the simplest
formula of the compound?
Number of Atoms A from the corner of the unit cell = 7/8
Number of atoms B from the faces of the unit cell = 3
States of Matter 2.61

7
Thus A : B : : : 3 so, that we can write the formula as A7/8 B3.
8
Hence, the simplest formula is A7 B24.

Illustration In a face-centred unit cell with all the position occupied by A atoms, the body-centred octahedral hole in
it is occupied by an atom B of an appropriate size. For such a crystal, calculate the void space per unit volume of unit
cell. Also predict the formula of the compound.
Let a be the edge length of the cube so that
4rA = 2 a or a = 2 2 rA (atoms A touch along the face-diagonal of the tube)
Now, since the atom B is occupied in the body-centred octahedral hole, it is obvious that
2rA + 2rB = a (atom B touches atoms A situated at the opposite face of the cube)
or 2rA + 2rB = 2 2 rA
rB
or 2rB = (2 2 –2)rA or = 2 – 1 =1.414 – 1.0 = 0.414
rA
3
Volume of the cube = a3 = 16 2 rA
Since there are 4 atoms of A in the face-centred cube, we will have
4 4 4
Volume occupied by A and B = 4 ¥ prA3 + prB3 = p(4rA3 + rB3 )
3 3 3
Volume occupied per unit volume of unit cell
4
p(4rA3 + rB3 )
p ÏÔ Ê rB ˆ ¸Ô
3
3 3.14 3.14 ¥ 4.071
f= = Ì4 + Á ˜ ˝ = {4 + (0.414)3 } = = 0.7534
16 2rA 3
12 2 ÔÓ Ë rA ¯ Ô˛ 12 ¥ 1.414 12 ¥ 1.414

Void space per unit volume of unit cell = 24.66%


There are four A atoms and one B atom, the formula of the compound is A4 B.

MULTIPLE CHOICE QUESTIONS ON SECTION 2

1. In closest packing of atoms, the number of atoms surrounding an atom in one layer is
(a) 2 (b) 4 (c) 6 (d) 8
2. The coordination number of an atom in closest packing of atoms is
(a) 4 (b) 6 (c) 8 (d) 12
3. The number of atoms in the unit cells of cubical-closest and hexagonal-closest packings, respectively, are
(a) 4,4 (b) 4,6 (c) 6,4 (d) 6,6
4. The distance between two closest layers of atoms (radius r) in hexagonal-closest packing is
(a) 2 r/ 6 (b) 4 r/ 6 (c) 6 r (d) 8 r/ 6
5. Volume of unit cell of hexagonal-closest packings of atoms (radius r) is
(a) 12 2 r3 (b) 16 2 r3 (c) 20 2 r3 (d) 24 2 r3
6. The void volume in the unit cell of hexagonal-cubical packing of atoms is
(a) 25.95% (b) 51.9% (c) 74.05% (d) 80.2%
7. The numbers of tetrahedral and octahedral holes per atom in cubical-closest packings of atoms, respectively, are
(a) 1 and 1 (b) 1 and 2 (c) 2 and 1 (d) 2 and 2
8. In the unit cell of cubical-closest packing, octahedral voids exist at
(a) centre of face-centred cube only (b) centre of faces of face-centred cube only
(c) centre of edges of cube only (d) centre of cube and centre of edges
2.62 Complete Chemistry—JEE Main

9. In the unit cell of hexagonal-closest packing of atoms, the number of atoms not shared amongst the neighbouring
unit is
(a) 1 (b) 2 (c) 3 (d) 4
10. In the unit cell of cubical-closest packing of atoms, the number of atoms not shared amongst the neighbouring
unit cells is
(a) 0 (b) 1 (c) 2 (d) 4
11. In the unit cell of cubical-closest packing of atoms, the number of atoms shared between two unit cells is
(a) 2 (b) 4 (c) 6 (d) 8
12. The unit cell of hexagonal-closest packing of atoms is
(a) primitive unit cells (b) body-centred unit cell
(c) face-centred unit cell (d) end-centred unit cell
13. In the unit cell of hexagonal-closest packing of atoms, the number of atoms contributed from the corner atoms is
(a) 1 (b) 2 (c) 3 (d) 4
14. In the unit cell of hexagonal-closest packing of atoms, the number of atoms contributed from the end-centred
atoms is
(a) 1 (b) 2 (c) 3 (d) 4
15. A compound of A, B and C crystallizes in cubical-closest packing with A occupying lattice points, B occupying one
half of octahedral voids and C occupying one fourth of tetrahedral voids. The simples formula of the compound
is
(a) A2BC (b) A2B2C (c) A2B2C2 (d) A2B4C3
16. A compound of A and B crystallizes in hexagonal-closest packing with A occupying lattice points and B occupying
3/4 of tetrahedral voids. The simplest formula of the compound is
(a) AB (b) AB2 (c) A2B2 (d) A3B2
17. A compound of A and B, atoms A crystallize as cubical-closest packing with one atom missing form one of the
faces of unit cell and atoms B occupy octahedral voids. The simples formula of the compound is
(a) A2B6 (b) A3B4 (c) A8B7 (d) A7B8
18. The fraction of volume occupied per unit cell of the compound described in Q.15 is
(a) 0.702 (b) 0.771 (c) 0.80 (d) 0.692
19. The fraction of volume occupied per unit cell of the compound described in Q.16 is
(a) 0.75 (b) 0.80 (c) 0.86 (d) 0.90
20. The fraction of volume occupied per unit cell of the compound described in Q.17 is
(a) 0.75 (b) 0.80 (c) 0.88 (d) 0.93

ANSWERS
1. (c) 2. (d) 3. (b) 4. (b) 5. (d) 6. (a)
7. (c) 8. (d) 9. (c) 10. (a) 11. (c) 12. (d)
13. (b) 14. (a) 15. (a) 16. (c) 17. (d) 18. (b)
19. (a) 20. (d)

HINTS AND SOLUTIONS


1. There are 6 atoms around a single atom in a layer of closest packing of atoms.
2. The coordination number is 12. Around a single atom, there are 6 atoms in the same layer, 3 atoms in each of the
above and below layers.
3. The number of atoms in cubical-closest packing is 4 while that in hexagonal-closest packing is 6.
States of Matter 2.63

4. The distance is 4 r/ 6 . See text


5. The volume of hexagonal unit cell is 24 2 r 3. See text.
6. The per cent of void volume is 25.95. See text.
7. There are 2 tetrahedral and 1 octahedral holes per atom of cubical-closest packings of atoms. See text.
8. Octahedral voids exist at the centre of face-centred cube and at the centre of 12 edges.
9. There are 3 atoms which lie in the central layer and exclusively belong to the unit cell.
10. In cubical-closest packing, all atoms are shared amongst neighbouring unit cells.
11. Atoms occupying the centre of 6 faces of cube are shared between the two unit cells.
12. Hexagonal-closest packing is end-centred because atoms occupy centres of two opposite faces.
13. There are 6 atoms in the top layer and 6 atoms in the bottom layer. Each of these is shared amongst six unit cell
giving 2 atoms from the corners.
14. End-centred atoms are shared between two unit cells. There are 2 such end-centres, hence, atoms contributed is
one.
1 1
15. Number of atoms A = 4; Number of atoms B = (4) = 2; Number of atoms C = (8) = 2
2 4
Hence, formula is A4B2C2 or simplest formula is A2BC.
3 3
16. Number of Atoms A = 6 Number of Aoms B = (tetrahedral units) = (12) = 9
4 4
The formula is A6B9 or the simplest formula is A2B3.
7
17. Number of atoms A = 8(1/8) + 5 (1/2) =
2
(from corner) (from face corners)
Number of atoms B = Number of octahedral voids = 4
Hence the formula is A7/2B4. i.e. A7B8.

u = 4 ÊÁ prA3 ˆ˜ + 2 ÊÁ prB3 ˆ˜ + 2 ÊÁ prC3 ˆ˜


4 4 4
18. Volume expressed by atoms is
Ë3 ¯ Ë3 ¯ Ë3 ¯

For octahedral voids, rB = 0.414 rA For tetrahedral voids, rC = 0.225 rA

u = 4 ÊÁ prA3 ˆ˜ + 2 ÈÍ p(0.414rA )3 ˘˙ + 2 ÈÍ p(0.225rA3 ˘˙ = ÊÁ prA3 ˆ˜ [4 + 2(0.414)3 + 2 (0.225)3)


4 4 4 4
Ë3 ¯ Î3 ˚ Î3 ˚ Ë3 ¯

= ÊÁ prA3 ˆ˜ ( 4 + 0.142 + 0.023) = 4.165 ÊÁ prA3 ˆ˜


4 4
Ë3 ¯ Ë3 ¯

Volume of unit cell, V = 16 2 rA3


Hence, fraction of volume per unit cell occupied by atoms is

4.165 Ê prA3 ˆ
4
u Ë3 ¯ 4.165p
f= = = = 0.771
V 16 2rA3 12 2

u = 6 ÊÁ prA3 ˆ˜ + 9 ÊÁ prB3 ˆ˜
4 4
19. Volume occupied by atoms is
Ë3 ¯ Ë3 ¯

For tetrahedral voids, rB = 0.225 rA. Hence

u = 6 ÊÁ prA3 ˆ˜ + 9 ÊÁ (0.225rA )3 ˆ˜ = ÊÁ prA3 ˆ˜ [6 + 9(0.225)3] = ÊÁ prA3 ˆ˜ (6.102)


4 4 4 4
Ë3 ¯ Ë3 ¯ Ë 3 ¯ Ë 3 ¯

Volume of unit cell hexagonal-closest packing is V = 24 2rA3


2.64 Complete Chemistry—JEE Main

Ê 4 pr 3 ˆ (6.102)
u Ë 3 A¯ 6.102p
Thus, fraction of volume occupied is f= = 3
= = 0.753 or 75.3%
V 24 2rA 18 2
7 Ê 4 3ˆ Ê 4 3ˆ
20. Volume occupied by atoms is u= ÁË prA ˜¯ + 4 ÁË prB ˜¯
2 3 3
7 Ê 4 3ˆ È4 3˘
For octahedral void, rB = 0.414 rA. Hence u= ÁË prA ˜¯ + 4 Í p (0.414 rA ) ˙
2 3 Î3 ˚

= ÊÁ prA3 ˆ˜ ÈÍ + 4(0.414)3 ˘˙ = ÊÁ prA3 ˆ˜ (3.78)


4 7 4
Ë3 ¯ Î2 ˚ Ë3 ¯

Volume occupied by unit cell is V = 16 2rA3


Fraction of volume occupied is

Ê 4 pr 3 ˆ (3.78)
u Ë 3 A¯ 3.78p
f= = 3
= = 0.932 or 93.2%
V 16 2rA 9 2

SECTION 3 Structures of Ionic Compounds

In ionic crystals, the structural units are held in position by electrostatic forces. In general, cations have only anions
nearest neighbours and vice versa. The coordination numbers commonly encountered in ionic crystals are 8, 6 and 4
(Fig 34)

Fig. 34

Radius Ratio
The arrangement of the ions in a crystal is determined by the relative sizes of cations
and anions.

is fairly large so that the anions touch the cation and still maintain their distances from
each other. As the cation gets smaller, the anions come closer to each other and reaches
a stage where they just touch each other. If the size of the cation gets still smaller,
anions will overlap each other leading to more repulsion and instability to the octahedral

fewer cation-anions bonds with larger anion-anion distance.


Fig. 35
States of Matter 2.65

In general, the arrangement of ions in an ionic crystal is determined by the limiting value of radius ratio, rc/ra, where
rc and ra are the radii of cation and anion, respectively. The arrangement of ions is expected to be stable till the ratio, rc/
ra, corresponds to the arrangement where anions, besides touching cation, also touch each other. In other words, anions
may be considered to form closest structures of CCP or HCP and cation occupy octahedral or tetrahedral voids in it.
Ratio of Eight-Coordinated Arrangement of Ions Eight coordinated arrangement of ions is the body-centred
cubic packing as shown in Fig. 36. For closest structure anions touch along the edge of cube and anion-cation-anion
touch each other along the body diagonal of the cube.

Hence 2ra = a and 2ra + 2rc= a 3


rc
This gives = 3 - 1 = 0.732
ra
Radius Ration of Octahedral and Tetrahedral Arrangements
As determined earlier, in case of closest structure of elements, the

rB
= 0.414 for octahedral void (Eq. 11)
rA
rB
and = 0.225 for tetrahedral void (Eq. 10) Fig. 36
rA

For octahedral arrangement rc/ra = 0.414


For tetrahedral arrangement rc/ra = 0.225
This amount to the fact that if rc/ra ≥ 0.414, the octahedral arrangement is stable and if rc/ra £ 0.414, this arrangement
is not stable. The more stable arrangement will be a tetrahedral till the rc/ra ≥ 0.225. If rc/ra is still smaller than 0.225,
the more stable arrangement is triangular arrangement with fewer cation-anion bond with larger anion-anion distance.
Radius Ratio for Triangular Arrangement

CD = rc + ra: CE = ra
–ECD = 30∞ and –CED = 90∞
CE CE
Hence cos 30∞ = or CD =
CD cos 30°
ra
i. e. rc + ra =
3/2
rc 2
This gives = - 1 = 0.155
ra 3
Fig. 37
If rc/ra
Summary of the Critical Ratio Thus, the critical ratios are

These ratios are approximate in number, since in actual calculation, we must also consider the mutual repulsion and
attraction of like and unlike charges, respectively, as well as the polarizaion of one ion by another.
Chlorides bromides and iodies of Li+, K+, Rb+ have structures like that of Cl– (Br– and I–) of sodium, i.e. face-centred
cubic crystal. But the chloride, bromide and iodide of Cs have the structure of a body-centred cubic crystal. It follows
that each Cs+ ion is surrounded by eight chloride ions and vice versa since the ratio of rc/ra increases in going from
RbCl to CsCl.
2.66 Complete Chemistry—JEE Main

Structure of Common Ionic Compounds


It is impossible for both anions and cations to have closest-packed structures but if one of the ions is much larger than

this structure.
Body-centred Unit Cell Anions occupy the corners of a cube and cation occupies body-centre of the cube.
(Fig. 39)
Coordination number of cations = 8 and Coordination number of anions = 8
There is one cation and one anion in a unit cell. Examples are CsCl, CsBr and CsI
The crystal lattice of the compound having body-centered unit cell may be considered as the half-way interpenetraion
of two separate primitive cube lattices one occupied by cations and the other by anions only (Fig. 40)

Fig. 39 Fig. 40

Fraction of Occupied Area per Unit Cell involving Closest-Packing of Ions


4 4
Volume occupied by ions is u = pra3 + prc3
3 3
For closest-packing, if cation occupies body-centre, then
Edge length of cube, a = 2ra and Body diagonal of cube, 3 a = 2ra + 2rc
This requires that rc/ra = 3 – 1 = 0.732
4 4 4
Thus u= p Èra3 + (0.732ra )3 ˘˚ = pra3 ÈÎ1 + 0.7323 ˘˚ = pra3 (1.392)
3 Î 3 3
3 3 3
Volume of unit cell is V = a = (2ra) = 8 r a
Ê 4 pr 3 ˆ (1.392)
u Ë3 a ¯ 1.392p
Fraction of volume occupied per unit cell is f= = 3
= = 0.728 i.e. 72.8% (9)
V 8ra 6

Density of Body-Centred Unit Cell Two anions touch a cation along the body diagonal of the cube. Hence
ra + 2rc + ra = 3a i.e. ra + rc = ( 3 /2)a
There are one anion (contribution from the corners of the cube) and one cation (at the body-centre of the cube) per
unit cell, i.e., there is a contribution of one molecule per unit cell.
States of Matter 2.67

(1)( M MX / N A ) (1)( M MX / N A )
Hence r= 3
= 3
(10)
ÎÈ2(ra + rc ) / 3 ˘˚
a

Illustration CsCl crystallizes in body-centred cubic lattice. If radii of Cs+ (molar mass = 133 g mol–1) and Cl– (molar
mass 35.5 g mol–1) are 181 pm and 167 pm, respectively. Calculate the density of CsCl.
We have

(1)( M CsCl / N A ) (1)[(133 + 35.5) g mol-1 ]/(6.022 ¥ 1023 mol-1 ) 2.798 ¥ 10-22 g
r= = = = 4.31 g cm–3
6.489 ¥ 10-23 cm3
3 3
ÈÎ 2(rc + ra ) / 3 ˘˚ ÈÎ2{(181 + 167) ¥ 10-10 cm}} / 3 ˘˚

Rock Salt Unit Cell


Anions alone have a face-centred cubic unit cell and the cations are present in the octahedral holes. When this occurs,
the cations by themselves form a face-centred cubic unit cell; hence the rock salt unit cell contains alternate cations and
anions, each occupying a face-centred cubical agreement. The crystal lattice of rock-salt structure may be visualised
as the interpenetration of two separate face-centred lattices one occupied by cations only and the other by anions only
(Fig. 40)
If anions form cubical-closest packing as shown in Fig. 41, then we have
1. anion-cation-anion touch each other along the edge of the cube. Hence
r a + 2 r c+ r a = a or rc + ra = a/2
2. anion-anion-anion touch each other along the face diagonal of the cube. Hence
r a + 2 r a+ r a = 2 a or ra = a/2 2

Fig. 40 Fig. 41

General formula
Number of cations: 1 central atom wholly within the cell
3 from 12 edges cations, each being shared by 4 unit cells
Total: 4 cations per unit cell
Number of anion: 1 from 8 corners anions, each being shared by 8 unit cells
3 from 6 anions situated in the centre of 6 faces, each being shared by two unit cells.
Total: 4 anions per unit cell
Thus, the formula is M4X4, or MX
Examples are MgO, CaO. SrO, BaO and all alkali halides except CsCl, CsBr, and Csl.
2.68 Complete Chemistry—JEE Main

Fraction of Occupied Area per Unit Cell Involving Closest-Packing of Anions

u = 4 ÊÁ pra3 ˆ˜ + 4 ÊÁ prc3 ˆ˜
4 4
Volume occupied by ions is
Ë3 ¯ Ë3 ¯
For Octahedral void, rc = 0.414 ra.

u = 4 ÊÁ pra3 ˆ˜ + 4 ÊÁ prc3 ˆ˜ = 4 ÊÁ pra3 ˆ˜ ÈÎ1 + (0.414)3 ˘˚ = 4 ÊÁ pra3 ˆ˜ (1.071)


4 4 4 4
Hence
Ë3 ¯ Ë3 ¯ Ë3 ¯ Ë3 ¯
3
Volume of unit cell is V = a3 = (2 2 ra)3 =16 2 ra

Hence, fraction of area occupied is

4 Ê pra3 ˆ (1.071)
4
u Ë3 ¯ 1.071 p
f= = = = 0.792 (10)
V 3
(16 2ra ) 3 2
Density of Rock-Salt Structure If rc and ra are the radii of cation and anion, it follows from Fig. 41 that
a = 2 rc + 2 ra
There are 4 cations and 4 anions per unit cell. Thus, the density of rock-salt structure will be
4( M MX / N A ) ( M MX / N A )
r= = (11)
a 3
2(rc + ra )3

Illustration Calculate the density of sodium chloride if r(Na+) = 95 pm and r (Cl–) =181 pm.
Solution: M NaCl = (23 +35) g mol–1 = 58.5 g mol–1
rNa+ +rCl– = (95 + 181) pm = 276 pm = 276 ¥ 10–12 m = 276 ¥ 10–10 cm
M NaCl / N A (58.5 g mol-1 ) /(6.022 ¥ 1023 mol-1 )
r= = = 2.31 g cm–3
2( rNa+ + rCl- ) 2(276 ¥ 10-10 cm)3
Note: Since r(Na +)/r(Cl–) = 95 pm/181 pm = 0.525 (which is greater than 0.414 for the cubical-closest packing of Cl–
ions), it follows that Cl– ions will form face-centred cubic lattice with edge length larger than the expected edge length
for cubical-closest packing.
Zinc Blende Unit Cell
Anions occupy face-centred cubic points and cations occupy half of the tetrahedral
voids (Fig. 42)
Coordination number of cations = 4
Coordination number of anions = 4
There are 4 cations and 4 anions in a unit cell. Hence, the formula of zinc blende
structure is M4X4 or MX Fig. 42
Example is BeO
Fraction of Area Occupied per Unit Cell Since the distance of a tetrahedral void from the corner of a cube is 3 a/4
(See Fig 29b), it follows from Fig. 42 that

ra + rc = 3 a/4 or a = 4(ra + rc) / 3

u = 4 ÊÁ p ra3 ˆ˜ + 4 ÊÁ p rc3 ˆ˜ = 4 ÊÁ pˆ˜ (ra3 + rc3 )


4 4 4
Volume occupied by ions is
Ë3 ¯ Ë3 ¯ Ë3 ¯

64
Volume occupied by unit cell is V = a3 = [4 (ra + rc) / 3 ]3 = (ra + rc )3
3 3
States of Matter 2.69

4 p Ê (ra3 + rc3 )ˆ
4
u Ë3 ¯ 3 È ra3 + rc3 ˘
Fraction of volume occupied is f= = = pÍ ˙
V 64 4 Î (ra + rc )3 ˚
( ra + rc )3
3 3
Note: Diamond also has zinc blende structure with 8 carbon atoms per unit cell. In this case, fraction of volume is

3 È r3 + r3 ˘ 3 2r 3 3p
f= pÍ 3˙ = p = = 0.34 ; (where r is the radius of carbon atom)
4 Î (r + r ) ˚ 4 ( 2 r )3 16

Expression of Density Let MX be the compound having zinc-blende structure .We have
NM MX / N A (4)( M MX / N A ) (3 3 )( M MX / N A )
r= = =
a 3
[4(ra + rc ) / 3 )3 16(ra + rc )3
Note: The expression of density of diamond becomes
(3 3 )(2M C / N A ) 3 3(M C / NA )
r= 3
=
16(2rC ) 64rC3
Anti-Fluorite Unit Cell
Anions occupy face-centred cubic points and cations occupy tetrahedral
void (Fig. 43)
Coordination number of cation = 4
Coordination number of anions = 8
There are 8 cations and 4 anions per unit cell.
The formula is M2X
Examples are Li2O, Na2O, K2O and RbO2 Fig. 43
+ and O2– are 152 pm and 126 pm,
Illustration The compound K2
determine edge length of unit cell, density of K2O and fraction of volume per unit cell occupied by K2O.

tetrahedral void lies at a distance 3a / 4 from the corner (Fig. 29b), it follows that
r+ + r– = 3a / 4

4(r+ + r- ) 4(152 pm + 126 pm)


Hence a= = = 642 pm
3 3
There are 4 molecules of K2O in a unit cell, hence, we have
NM 4(2 ¥ 39 + 16) g mol-1
r= = = 2.36 g cm–3
N A a3 (6.022 ¥ 1023 mol-1 )(642 ¥ 10-10 cm)3
Volume occupied by ions
4
u = 4 ÊÁ pra3 ˆ˜ + 8 ÊÁ prc3 ˆ˜ = p(4ra + 8rc ) = (3.14) [4(126 pm)3 + 8(152 pm)3] = 1.511 ¥ 108 pm3
4 4 3 3 4
Ë3 ¯ Ë3 ¯ 3 3
Volume of unit cell V = a3 = (642 pm)3 = 2.646 ¥ 108 pm3
u 1.511 ¥ 108 pm3
Hence, fraction of volume occupied is f= = = 0.57
V 2.646 ¥ 108 pm3
Fluorite Unit Cell
Cations occupy face-centred cubic unit cell and anions occupy tetrahedral voids.
Coordination number of cations = 8 and Coordination number of anions = 4
There are 4 cations and 8 anions in a unit cell. The formula is MX2. Examples are UO2 and ThO2.
2.70 Complete Chemistry—JEE Main

Corundum Structure
Anions from hexagonal packing and cations occupy 2/3 of octahedral voids.
Coordination number of cations = 6
Coordination number of anions = 4
There are 6 anions and 4 cations per unit cell. The formula is M2X3.
Examples are Fe2O3, Al2O3, and Cr2O3
Spinel Structure
A spinel is an important class of oxides containing two types of metal ions with the oxide ions arranged in face-centred
cubic packing. The normal spinel has one-eighth of the tetrahedral holes occupied by divalent metal ions (A2+) and
one-half of the octahedral holes occupied by trivalent metal ions (B3+). In a unit cell, we have.
1 1
Number of divalent metal ions, A2+ = (8) =1 Number of trivalent metal ions, B3+ = (4) =2
8 2
Number of oxide ions, O2– = 4
Hence, the formula of normal spinel is AB2O4 Examples are ZnAl2O4, MgAl2O4 and ZnFe2O4.

MULTIPLE CHOICE QUESTIONS ON SECTION 3

1. Which of the following coordination numbers is not found in ionic crystals?


(a) 4 (b) 5 (c) 6 (d) 8
2. In an ionic structure, the radius ratio, rc/ra, is 0.66. The expected arrangement of anions around a cation is
(a) cubic (b) octahedral (c) tetrahedral (d) triangular
3. In an ionic compound, anions from cubical-closest packing and cations occupy half of the tetrahedral void. The
unit cell of this compound is known as

(a) anions form face-centred cubic unit cell and cations occupy tetrahedral voids
(b) cations form face-centred cubic unit cell and anions occupy tetrahedral voids.
(c) anions form face-centred cubic unit cell and cations occupy octahedral voids.
(d) cations form face-centred cubic unit cell and anions occupy octahedral voids.
5. In corundum structure, cations occupy
(a) 1/3 of octahedral voids (b) 2/3 of octahedral voids
(c) 1/4 of octahedral voids (d) 1/2 of octahedral voids
6. In rock-salt structure (edge length = a), the distance between centres of cation and anion is
(a) a (b) a/2 (c) ( 3 / 2 )a (d) a / 2
7. Fraction of volume occupied in closest-packed rock-salt unit cell is
(a) 0.65 (b) 0.70 (c) 0.79 (d) 0.85
8. The rock-salt structure may be considered as interpenetration of
(a) primitive cubic lattice of cation and primitive cubic lattice of anions
(b) body-centred cubic lattice of cations and body-centred cubic lattice of anion
(c) face-centred cubic lattice of cations and face-centred cubic lattice of anions
(d) half-way interpretation of primitive cubic lattice of cations and primitive cubic lattice of anions
9. The compound CsCl has
(a) body-centred cubic unit cell (b) rock-salt unit cell
States of Matter 2.71

10. MgO has closest-packed rock-salt unit cell. If ionic size of Mg2+ ion is 72 pm, the size of O 2-
2 will be about
(a) 320 pm (b) 243.9 pm (c) 173.9 pm (d) 122.4 pm
11. The ionic radius of M+ is 100 pm. The volume of unit cell of the compound of M+X– having closest-packed rock
salt structure would be about
(a) 3.19 ¥ 10–22 cm3 (b) 3.19 ¥ 10–25 cm3 (c) 3.19 ¥ 10–28 cm3 (d) 3.19 ¥ 10–30 cm3
12. The compound Fe2O3 has

13. The compound CsCl is


(a) 8:8 coordinated compound (b) 6:6 coordinated compound
(c) 4:4 coordinated compound (d) 4:6 coordinated compound
14. If ionic radii of Cs+ and Cl– ions are 181 pm and 167 pm, respectively . The edge length of the unit cell of CsCl
will be about
(a) 201 pm (b) 340 pm (c) 375 pm (d) 402 pm
2+ 2–
15. The ionic radii of Mg and O are 86 pm and 126 pm, respectively. the edge length of the unit cell of MgO will
be
(a) 212 pm (b) 350 pm (c) 424 pm (d) 521 pm
2+ ion is 60 pm, the edge length of the unit
16. The compound MO has zinc-blende closest unit cell. If the radius M
cell of MX is
(a) 754.3 pm (b) 650 pm (c) 525 pm (d) 402.2 pm
17. The compound MO (molar mass = 100 g mol–1) has zinc-blende closest unit cell. If the radius of M2+ is 69.4 pm,
the density of MO will be
(a) 2.5 g cm–3 (b) 1.5 g cm–3 (c) 0.75 g cm–3 (d) 1.0 g cm–3
+ is 90 pm, the edge length of unit cell of
18. The compound M2
M2O is
(a) 1.13 nm (b) 2.26 nm (c) 3.39 nm (d) 4.52 nm
19. The compound M2O (molar mass = 200 g mol–1 + is 79.55

pm, the density of M2O would be


(a) 0.67 g mol–1 (b) 1.33 g mol–1 (c) 2.00 g mol–1 (d) 2.67 g mol–1
20. The formula of spinel structure is AB2O4. If A2+ and B3+
packing of O2– ions (r = 126 pm), the sizes of A2+ and B3+, respectively, are about
(a) 52.3 pm, 28.4 pm (b) 28.4 pm, 52.2 pm (c) 89.1 pm, 72.7 pm (d) 72.7 pm, 89.1 pm
2+) = 59 pm, r(O2–) = 126 pm and M(Be) = 9 g mol–1, answer the
The compound BeO has zinc blende unit cell. If r(Be
following three questions
21. The edge length of unit cell is
(a) 213.62 pm (b) 314.82 pm (c) 427.24 pm (d) 540.2 pm
22. The density of BeO is
(a) 2.13 g cm–3 (b) 3.14 g cm–3 (c) 4.27 g cm–3 (d) 5.40 g cm–3
23. The fraction of volume per unit cell occupied by BeO is
(a) 0.30 (b) 0.47 (c) 0.55 (d) 0.72
2+ and B3+ occupy, respectively.
24. In spinel structures, the ions A
1 1 1 1
(a) of tetrahedral and of octahedral voids (b) of tetrahedral and of octahedral voids
8 2 4 2
1 1 1 1
(c) of tetrahedral and of octahedral voids (d) of tetrahedral and of octahedral voids
2 4 4 4
25. Diamond has the structure similar to zinc blende. The number carbon atoms per unit cell will be
(a) 2 (b) 4 (c) 6 (d) 8
2.72 Complete Chemistry—JEE Main

26. The fraction of volume occupied by carbon in diamond unit cell is


(a) 0.17 (b) 0.25 (c) 0.34 (d) 0.49
27. The compound MX (molar mass = 100 g mol–1) occurs in zinc blende structure. If rc = 80 pm and ra = 120 pm,
its density will be
(a) 6.25 g cm–3 (b) 8.11 g cm–3 (c) 9.56 g cm–3 (d) 10.4 g cm–3
28. Carbon has zinc-blende type unit cell. If the atomic radius of C is 77 pm, the edge length of face-centred unit cell
is
(a) 355.7 pm (b) 308.0 pm (c) 231.0 pm (d) 375.6 pm
29. A metal has zinc blende type unit cell. If the atomic radius of this metal is 86.6 pm, then the number of atoms in
a cube of edge length of 1 cm is about
(a) 0.62 ¥ 1023 atoms (b) 1.25 ¥ 1023 atoms (c) 2.50 ¥ 1023 atoms (d) 5.0 ¥ 1023 atoms
30. An ionic compound MX has rock-salt structure. If rc= 120 pm and ra= 180 pm, then the number of molecules of
MX in a cube of edge length of 1 cm is about
(a) 1.85 ¥ 1020 molecules (b) 1.85 ¥ 1021 molecules (c) 1.85 ¥ 1022 molecules (d) 1.85 ¥ 1023 molecules

ANSWERS
1. (b) 2. (b) 3. (d) 4. (b) 5. (b) 6. (b)
7. (c) 8. (c) 9. (a) 10. (c) 11. (a) 12. (d)
13. (a) 14. (d) 15. (c) 16. (a) 17. (d) 18. (a)
19. (b) 20. (b) 21. (c) 22. (a) 23. (b) 24. (a)
25. (d) 26. (c) 27. (b) 28. (a) 29. (b) 30. (c)

HINTS AND SOLUTIONS


1. In ionic compounds, coordination number 5 is not found.
2. If rc/ra is in between 0.414 and 0.732, the arrangement of anions around cations is octahedral.
3. In zinc-sulphide unit cell, anions form cubical-closest packing and cations occupy half of tetrahedral voids.

5. In corundum, structure, 2/3rd of octahedral voids are occupied by cations.


6. In rock-salt structure, anion-cation occupy the edge of cube. Hence
ra + 2rc + ra = a or rc + ra = a/2
7. Fraction of volume occupied in closest-packed rock salt is 0.79. See Text.
8. Rock-salt structure may be considered as interpenetration of face-centred cubic lattice of cations and face-centred
cubic lattice of anions.
9. For rCs+/rCl– > 0.732, hence, CsCl forms body-centred cubic unit cell.
10. For closest-packed rock salt structure where cations occupy octahedral voids, rc/ra = 0.414. Hence,
ra = rc/0.414 = 72 pm/0.414 = 173.9 pm.
11. If rc = 100 pm, then ra = 100 pm/0.414 = 241.5 pm. Edge length of cube, a = 2 (rc+ ra) = 683 pm
a3 = (683 pm)3 = 318.6 ¥ 106 pm3 = (318.6 ¥ 106) (10–10 cm)3 = 318.6 ¥ 10–24 cm3 = 3.19 ¥ 10–22 cm3.
12. Fe2O3 has corundum unit cell.
13. The coordination number of Cs+ an Cl– are 8 and 8, respectively, as CsCl occurs in body-centre cubic unit cell.
14. CsCl forms body-centred cubic unit cell. Hence, 2(rc + ra) = 3a
2 (rc + ra ) 2(181 + 167)
This gives a= = pm = 401.8 pm
3 3
States of Matter 2.73

15. For MgO, rc/ra = 86 pm/126 pm = 0.682. It is more than 0.414 but less than 0.732. Thus MgO occurs as rock-salt
structure. The edge length unit cell is 2(ra + rc ) = 2(86 +126) pm = 424 pm
16. In zinc-blende closest unit cell, cations occupy tetrahedral voids for which rc/ra = 0.225. Hence, ra = rc /0.225
= 60 pm/0.225 = 266.67 pm. Anions occupy face centred cubic positions. Hence, 4ra = 2 a or a = 2 2 ra =
(2 2 ) (2.667 pm) = 754.3 pm.
17. ra = rc /0.225 = 69.4 pm/0.225 =308.4 pm; a = 2 2 ra = 2 2 (308.4 pm) = 872.4 pm
NM 4(100 g mol-1 ) 400
r= = -1 -10
= g cm -3 = 1.0 g cm -3
NA a 3
(6.022 ¥ 10 mol )(872.4 ¥ 10 cm)
23 3
399.84
Hence, ra = rc /0.225 = 90 pm/0.225 = 400 pm
Anions form face-centred cubic unit cell. Hence a = 2 2 ra = 2 2 (400 pm) =1131.4 pm =1.13 nm
19. ra = rc /0.225 = (79.55 pm)/0.225 =353.56 pm; a = 2 2 ra = 2 2 (353.56 pm) =1000 pm
There are 4 molecules of M2
NM 4(200 g mol-1 )
Hence r= = =1.33 g mol–1
N A a3 (6.022 ¥ 1023 mol-1 )(1000 ¥ 10-10 cm)3
20. A2+ occupy tetrahedral voids and B3+ occupy octahedral voids
Hence rA2+ = 0.225 ¥ 126 pm = 28.4 pm and r B3+ = 0.414 ¥ 126 pm = 52.2 pm
4(r+ + r- ) 4 (59 pm + 126 pm )
21. We have 2 (r++ r–) = 3 (a/2). Hence a = = = 427.24 pm
3 3
22. There are 4 molecules of BeO in a unit cell, Hence
NM 4(9 + 16) g mol-1 )
r= = = 2.13 g cm–3
N A a3 (6.022 ¥ 1023 mol-1 )(427.24 ¥ 10-10 cm)3
23. Volume occupied by ions is

u = 4 ÊÁ p ra3 ˆ˜ + 4 ÊÁ p rb3 ˆ˜ = 4 ÊÁ ˆ˜ (3.14) [ (126 pm)3 + (59 pm)3] = 3.69 ¥ 107 pm3
4 4 4
Ë3 ¯ Ë3 ¯ Ë 3¯
Volume of unit cell is V = a3 = (427.24 pm)3 = 7.80 ¥ 107 pm3
u 3.69 ¥ 107 pm3
Fraction of volume occupied is f= = = 0.47
V 7.80 ¥ 107 pm3
24. In spinel structure, 1/8 of tetrahedral voids are occupied by A2+ and 1/2 of octahedral voids are occupied by B3+.
25. Zinc blende unit cell, carbon atoms will occupy face-centred cubic unit cell with half of tetrahedral voids occupied
by carbon atoms.
Carbon atoms form face-centred cubic unit cell = 4. Carbon atoms from half of tetrahedral voids = 4
Total number of carbon atoms = 8
26. The fraction of volume occupied by carbon in diamond unit cell is 3p / 16 which is 0.34. See text.
27. We have

4( M MX / N A ) 3 3 ( M MX / N A ) (3 3 )[(120 g mol-1 ) /(6.022 ¥ 1023 mol-1 )]


r= = =
[4(ra + rc ) / 3 ]3 16(ra + rc )3 16 [(120 + 80) ¥ 10-10 cm]3

(3 3 )(20 ¥ 10-23 ) g
= = 8.11 g cm–3
16 ¥ 8.0 ¥ 10-24 cm3
28. The distance between corner and tetrahedral void is 3a / 4 .
Hence 2rC = 3a /14. This gives a = 8rC / 3 = 8(77 pm)/1.732 = 355.7 pm
2.74 Complete Chemistry—JEE Main

3 a/4 . Hence, a = 8rM / 3 = 8 ¥ 86.6 ¥10 cm/1.732 = 400 ¥ 10 cm


29. 2rM = –10 –10

u = a3 = (4.00 ¥10–8 cm)3 = 6.4 ¥ 10–23 cm3


Ê 8 atoms ˆ V = Ê 8 atoms ˆ (1 cm3 )
N = ÁË ˜ ÁË 3˜ = 1.25 ¥ 1023 atoms
u ¯ 6.4 ¥ 10 cm ¯
-23

30. The edge length of unit cell is a = 2(ra + rc) = 2 (180 + 120) pm = 600 pm = 600 ¥ 10–10 cm
The volume of unit cell is u = (600 ¥ 10–10 cm)3 = 2.16 ¥ 10–22 cm3
Ê 4 molecules ˆ (1 cm)3
The number of molecules in V = 1 cm3 is N = ÁË ˜ = 1.85 ¥ 1022 molecules
2.16 ¥ 10-22 cm3 ¯

SECTION 4 Imperfection in Solids

The term ‘imperfection in solids’ implies the departure from the perfect periodic arrangement of atoms, molecules
or ions in the structure of crystalline substance. The perfect arrangement of the species is expected to exist only at
the lowest temperature of 0 K. At higher temperatures, the crystalline substances exhibit departure from the ideal

mentioned in the following.


1. STOICHIOMETRIC DEFECT
This defect does not change the stoichiometry of solids. This includes
(i) Vacancy defect—presence of vacancies in the lattice points.
(ii) Interstitial defect—dislocation of species from lattice point to interstitial site.
In ionic solids, the above two defects do occur with the constraints of electrical neutrality of ionic solids.
Schottky Defect This is vacancy defect. To maintain electrical neutrality, both cations and anions are missing from
lattice points (Fig. 44).
The schottky defects are more common in ionic compounds with high coordination
numbers and with similar sizes of cations and anions. The absence of ions leads to
decrease in the density of solid as compared to perfect crystals. Examples are
NaCl and CsCl
Frenkel Defect This is interstitial defect. In a solid involving large differences in Fig. 44 Schottky defect
sizes of cations and anions, the smaller-sized ions shift from the regular position to the
interstitial positions. (Fig. 45)
The density of crystals exhibiting Frenkel defect remains unchanged. Examples are
ZnS, AgCl, AgBr and AgI.
Alkali halides do not show Frenkel effect as the ions cannot occupy interstitial sites.
2. IMPURITY DEFECTS
In impurity defect, some of the lattice points are occupied by ions other than the expected
ion. For example, if molten NaCl is crystallized in the presence of SrCl2, some of Na+
lattice sites are occupied by Sr2+ ions. To maintain electrical neutrality, some of the lattice
points are unoccupied (Fig. 46). The creation of cationic vacancies imparts electrical Fig. 45 Frenkel defect
conductivity to the ionic crystals.
3. NON-STOICHIOMETRIC DEFECTS

quantity as compared to the expected stoichiometric compound. Two categories of non-

Fig. 46
States of Matter 2.75

The preparation of FeO always leads to


non-stoichiometric compound Feo.95O. This compound contains both Fe2+ and
Fe3+ ions. To maintains electrical neutrality, Fe2+ ions from three lattice points are
replaced by two Fe3+ ions at two sites leaving third site vacant (Fig. 47).

Illustration 1 The composition of a sample of wustite is Fe0.93O. What


percentage of iron exists as Fe(III). Fig. 47

For every 1 mol of oxygen atoms, 0.93 mol of iron atoms is present. If x is the amount of Fe(III) present, then
Amount of positive charge = x(+3) + (0.93 mol – x)(+2) = 1.86 mol + x
Amount of negative charge = (1 mol) (2)
To maintain electrical neutrality, we have
1.86 mol + x = (1 mol)2 This gives x = 0.14 mol
Percentage of Fe(III) = (0.14 mol/0.93 mol) ¥100 = 15.1%

Illustration 2 A sample of wustite FexO contains 25 mol% of Fe(III). What is the value of x and simple formula
of FexO.

Amount of Fe(III) = 0.25x Amount of Fe(II) = 0.75x


To maintain electrical neutrality, we have
(0.25x)(3) + (0.75x)(2) = 2 This gives x = 2/2.25 = 8/9
Hence, the compound is Fe8/9O.
Metal Exess Defect due to Anionic Vacancies
(i) Heating of solid NaCl in the atmosphere of sodium metal vapours causes the
deposition of sodium on the surface of solid. The Cl– ions from the bulk of solid
moves to the surface and combines with sodium to give sodium chloride. The
electrons set free here move to the bulk and occupy anionic vacancies (Fig. 48).
These trapped electrons in the anionic vacancies are referred to as F-centres Fig. 48
and is responsible for the interesting properties to the crystal such as photoconductors, n-type semiconductor
and characteristic colours. For example, the excess of K+ in KCl imparts violet colour and Li+ in LiCl imparts
pink colour.
(ii) The strong heating of ZnO leads to the formation of a yellow-coloured non-stiochiometric compound. Zinc
oxide loses oxygen reversibly on heating ( ZnO Æ Zn + 12 O 2 + 2e - ) . The excess of Zn2+ ions in crystal move
to interstitial sites and electrons occupy the neighbouring interstitial positions. Because of free electrons, the
electrical conductivity is enhanced.

Electrical Properties

Conductors These are good conductors of electricity. Their conductivity ranges between 104 S m–1 and 107 S m–1
(where S stands for Siemens which is ohm–1).
Insulators These are poor conductor of electricity. Their conductivity ranges between 10–20 S m–1 and 10–10 S m–1
Semiconductors These lie in between conductors and insulator and have conductivity ranges from 10–6 S m–1 to
10 S m –1.
4

Band Theory
These conductivity of solids depends on the valence electrons in their constituent atoms. The conductivity ability has
been explained on the basis of band theory. The later involves the formation of molecular orbitals from the valence
atomic orbitals of atoms. Because of very large number of atoms, the energies of one set of molecular orbitals are very
close to each and constitute what is called band of molecular orbitals.
2.76 Complete Chemistry—JEE Main

Metals contain one, two or three electrons in the outermost shell of their atoms. Their atomic orbitals form partially-

Nonmetals contain 5,6 or 7 electrons in the outermost shell of their atoms. Their atomic orbitals form atmost fully-

Elements of Group 14 of periodic table having atoms with 4 valence electrons constitute semiconductors. In this
case, the gap between lower-energy band and conduction band is small (Fig. 49 c) and electrons can jump from lower-
energy band to conduction band and their number increases with increase in temperature. This causes an increase in
conductivity of semiconductors with increase in temperature.

Fig. 49

Enhancement of Conductivity of Semiconductors Silicon and germanium are the examples of semicounductors.
Their conductivity is too low for any practical use. However their conductivity can be increased by introducing a very
small quantity of atoms having 3 or 5 electrons in the outermost shell. This process is called dopping.
n-Type Semiconductors Si and Ge belong to Group 14 of periodic table. When dopping is carried out with P or As
(which belong to Group 15 with 5 valence electrons), some of the Si or Ge is replaced by P or As. In the latter atom,

available for conduction causing an increase in the conductivity of dopped semiconductor. In n-type semiconductor, n
stands for negatively charged free electron.
p-Type Semiconductor If dopping is done with the atoms containing 3 valence electrons (i.e. elements from Group
13 of periodic table such as boron, gallium or indium), the resultant semiconductor is known as p-type semiconductor.
Here all the three electrons of dopped atom are involved in the bonding with neighbouring atoms and a hole exist in

is generated. The latter moves in the opposite direction of the movement of electrons. This movement of electrons or
hole enhances the conductivity of semiconductor.

Magnetic Properties of Solids

This alignment produces a net magnetization on the microscopic level.


The orbital motion of electrons around nucleus also generates tiny magnets but their contribution to total magnetization

The magnetic moment due to the spinning of electrons is given by


m = n(n + 2) m B
States of Matter 2.77

where n is the number of unpaired electrons and mB is the base unit of magnetic moment and is known as Bohr magneton.
Its value is 9.274 ¥ 10–24 J T–1.

Paramagnetic Substnace
Examples are O2, Cu2+ and Fe3+.

Diamagnetic Substance
Examples are N2, H2O, and NaCl.
These substance are repelled by the external

Ferromagnetic Substance The substances


which can be permanently magnetized by the

substance. Examples are Fe, Co, Ni and CrO2. These


substances are strongly attracted towards magnetic

(Fig. 49a) and this alignment remains unaffected


Fig. 2.49
Antiferromagnetic Substance In these substances, half of the magnetic moment vectors align in one direction and

no magnetic properties (Fig. 49b). Example is MnO.


Ferrimagnetic Substance In these substances, the alignment of magnetic moment vector in opposite direction is
uneven resulting into a net magnetic moment (Fig. 49c). Examples are Fe3O4 and ferrites with general formula MFe2O4
where M is Mg2+, Cu2+, Zn2+, etc.

Dielectric Properties

generates an induced dipole moment in the constituent of the substance. In general, the induce dipole moment is directly

pind μ E or pind = a E
where a is known as polarizability of the constituent molecules. The induced dipole moments may align in the direction

Ferroelectric substance The induced dipole moments are aligned to induce a net dipole moment. Examples are
barium titnate (BaTiO3), sodium potassium tartrate (Rochelle salt) and potassium dihydrogen phosphate (KH2PO4)
Antiferroelectric substance The induced dipoles are alternatively aligned in the opposite directions resulting into
no dipole moment. Example is lead zirconate (PbZrO3).
Piezoelectricity The crystal in which alignment of induced dipoles generates net dipole moment show the
phenomenon of piezoelectricity. There occurs emission of electrons when a stress is applied to the crystal.
Pyroelectricity: Some of the polar crystal emit electrons when these are heated.
Superconductivity: A substance offering no resistance to the passage of electrons is said to be superconductor. Most
metals become superconducting at temperature of about 2-5 K. At this temperature, the substance becomes diamagnetic.
Examples are Ba0.7K0.3BiO3(30 K) and YBa2Cu3O7(90K).
2.78 Complete Chemistry—JEE Main

MULTIPLE CHOICE QUESTIONS ON SECTION 4

Identify the correct choice in the following questions.


1. Schottky defect is
(a) vacancy defect (b) interstitial defect (c) impurity defect (d) stoichiometric defect
2. Frenkel defect is
(a) vacancy defect (b) interstitial defect (c) impurity defect (d) stoichiometric defect
3. Iron oxide preparation always leads to nonstoichiometric compound. It involves

4. In the compound Fe0.85O, the percentage of iron existing as Fe(III) is


(a) 25.3% (b) 30.3% (c) 35.3% (d) 40.3%
5. A sample of wustite FexO contanis 15 mol% of Fe(III). The sample has the molecular formula of
(a) Fe0.93O (b) Fe0.90O (c) Fe0.86O (d) Fe0.82O
6. Heating of sodium chloride in the presence of sodium vapour creates F-centres which represent
(a) cationic vacancies (b) anionic vacancies (c) octahedral voids (d) tetrahedral voids
6 S m–1
7. A substance has conductivity of 10
(a) conductor (b) insulator (c) semiconductor (d) superconductor

(a) conductor (b) insulator (c) semiconductor (d) superconductor


9. The magnetic moment of an atom or molecule or ion is given by the expression
(a) n(n + 1) mB (b) n(n + 2) mB (c) n mB (d) (n+1) mB
where n is number of unpaired electrons and mB is Bohr magneton.
10. The value of Bohr magneton is
(a) 9.274 ¥ 10–21 J T–1 (b) 9.274 ¥ 10–22 J T–1 (c) 9.274 ¥ 10–23 J T–1 (d) 9.274 ¥ 10–24 J T–1
11. Which of the following material can be permanently magnetized?
(a) Paramagnetic substance
(b) Diamagnetic substance
(c) Ferromagnetic substance

12. Which of the following substance does not have magnetic properties?
(a) Paramagnetic substance (b) Ferromagnetic substance
(c) Antiferromagnetic substance (d) Ferrimagnetic substance
13. Which of the following facts regarding the effect of increasing temperature on the conductivity of a substance is
correct?
(a) Increases for metals and decreases for semiconductors
(b) Decreases for metals and increases of semiconductors
(c) Increases for both metals and semiconductors
(d) Decreases for both metals and semiconductors
14. The amount of cationic vacancies per mole of Fe0.88O is
(a) 0.10 mol (b) 0.11 mol (c) 0.12 mol (d) 0.14 mol
15. The amount of cationic vacancies per mole of a nonstoichiometric iron oxide is 0.14 mol. The formula of the
compound is
(a) Fe0.75O (b) Fe0.82O (c) Fe0.86O (d) Fe0.93O
States of Matter 2.79

ANSWERS
1. (b) 2. (b) 3. (a) 4. (c) 5. (a) 6. (b)
7. (a) 8. (c) 9. (b) 10. (d) 11. (c) 12. (c)
13. (b) 14. (c) 15. (c)

HINTS AND SOLUTIONS


1. Schottky defect is vacancy defect.
2. Frenkel defect is interstitial defect.

4. For every 1 mol of oxygen atoms, 0.85 mol of iron atoms is present. If x is the amount of Fe(III) present, then the
electrical neutrality requires that
x(+3) + (0.85 mol – x) (2) = (1 mol) 2
This gives x = 0.3 mol
Percentage of Fe(III) = (0.3 mol/0.85 mol) ¥ 100 = 35.3%
5. We will have Amount of positive charge = Amount of negative charge
(0.15 mol) x (+3) + (0.85 mol) x (+2) = (1 mol) (2)
2
This gives x = = 0.93. The compound is Fe0.93O
2.15
6. F–centres are anionic vacancies.
7. Conductors has conductivity in the range 104 – 107 S m–1
8. Conductivity of semiconductor increases with increase in temperature.
9. The expression of magnetic moment is n(n + 2) mB
10. mB = 9.274 ¥ 10–24 J T–1

12. Antiferromagnetic substance do not have magnetic properties.


13. For metals, conductivity decreases while for semiconductors, it increases.
14. In non-stoichiometric iron oxide, 3 mol of Fe2+ ions are replaced by 2 mol of Fe3+ ions and 1 mol of cationic
vacancies is created. If x is the amount of Fe(III) present in the given compound, then
x(+3) + (0.88 mol –x) (+2) = (1 mol) (2)
This gives x = 0.24 mol.
(III) (II)
Hence, the given compound is Fe0.24 Fe0.64 O . The cationic vacancies will be 0.24 mol/2 = 0.12 mol
15. 0.14 mol of cationic vacancy implies the presence of 0.28 mol of Fe(III). If x is the amount of Fe(II), then electrical
neutrality requires that
(0.28 mol) (+3) + x (2) = (1 mol) (2)
(III) (II)
This gives x = 0.58 mol of Fe(II). Hence the formula is Fe0.28 Fe0.58 O , i.e. Fe0.86O
2.80 Complete Chemistry—JEE Main

MULTIPLE CHOICE QUESTIONS ON ENTIRE UNIT

Identify the correct choice in the following questions.


Crystal Systems
1. In the primitive unit cell, the points are present at the
(a) corners of the unit cell (b) centre of the unit cell
(c) centre of each face of the unit cell (d) one set of faces of the unit cell
2. Which of the following statements is not correct?
(a) There are seven crystal systems
(b) There are fourteen Bravais lattices
(c) There are one primitive and two nonprimitive cubic unit cells
(d) In the body-centred unit cell, the points besides present at the comers are also present at the centre of all faces
3. Molecular solids are usually
(a) good electrical conductors (b) quite hard
(c) quite brittle (d) volatile
4. The number of nearest neighbours around each particle in a face-centred cubic lattice is
(a) 4 (b) 6 (c) 8 (d) 12
5. Bragg equation for the scattering of X-rays by crystals is
(a) nl = d sin q (b) nd= l sin q (c) nl = 2d sin q (d) nd = 2l sin q
6. Which of the following dimensions represents a cubic unit cell?
(a) a = b = c; a = b = g = 90° (b) a = b π c; a = b = g = 90°
(c) a = b = c; a = b = g π 90° (d) a = b π c; a = b = 90°, g = 120°
7. Which of the following dimensions represents a tetragonal unit cell?
(a) a = b = c; a = b = g = 90° (b) a = b π c; a = b = g = 90°
(c) a = b = c; a = b = g π 90° (d) a = b π c; a = b = 90°, g = 120°
8. Which of the following dimensions represents a trigonal unit cell?
(a) a = b = c; a = b = g = 90° (b) a = b π c; a = b = g = 90°
(c) a = b = c; a = b = g π 90° (d) a = b π c; a = b = 90°, g = 120°
9. Which of the following dimensions represents a hexagonal unit cell?
(a) a = b = c; a = b = g = 90° (b) a = b π c; a = b = g = 90°
(c) a = b = c; a = b = g π 90° (d) a = b π c; a = b = 90°, g = 120°
10. Which of the following dimensions represents a triclinic unit cell?
(a) a = b = c; a = b = g = 90° (b) a = b π c; a = b = g = 90°
(c) a π b π c; a = b = 90° π g (d) a π b π c; a π b π g π 90°
11. Which of the following dimensions represents a monoclinic unit cell?
(a) a = b = c; a = b = g = 90° (b) a = b π c; a = b = g = 90°
(c) a π b π c; a = b = 90° π g (d) a π b π c; a π b π g π 90°
12. Which of the following dimensions represents a orthorhomic unit cell?
(a) a π b π c; a = b = g = 90° (b) a π b π c; a = b = 90° π g
(c) a π b π c; a π b π g = 90° (d) a = b π c; a = b = 90° π g
13. Which of the following units can be used to built the structure of diamond?
(a) Tetrahedral (b) Octahedral (c) Hexagonal (d) Cubic
14. Which of the following units can be used to built the structure of graphite?
(a) Tetrahedral (b) Octahedral (c) Hexagonal (d) Cubic
States of Matter 2.81

15. The unit cell present in the crystal lattice of diamond is


(a) face-centred cube (b) tetragonal (c) hexagonal (d) trigonal
16. Which of the following statements is not correct?
(a) The molecular crystals are soft and possess comparatively low melting points.
(b) The ionic crystals have high melting points.
(c) Covalent crystals are poor conductor of heat and electricity.
(d) Metallic crystals are not ductile and malleable.
17. In a molecular crystal the forces holding together the atoms within a given molecule are
(a) covalent bonds (b) ionic bonds (c) metallic bonds (d) van der Waals forces
18. Ionic solids tend to be
(a) good electrical conductors (b) soft
(c) volatile (d) brittle
19. Which of the following Bravais lattices does not exist in the crystal lattice having cubic unit cell?
(a) Primitive (b) Body-centred (c) Face-centred (d) End-centred
20. The number of Bravais lattices in the crystal lattice having orthorhombic unit cell is
(a) 1 (b) 2 (c) 3 (d) 4
21. The number of Bravais lattices in the crystal lattice having tetragonal unit cell is
(a) 1 (b) 2 (c) 3 (d) 4
22. Which of the following lattices having the indicated unit cell has only primitive Bravais lattice?
(a) Monoclinic (b) Trigonal (c) Cubic (d) Tetragonal
23. Which of the following lattices does not have only primitive Bravais lattice?
(a) Lattice with unit cell triclinic (b) Lattice with unit cell hexagonal
(c) Lattice with unit cell monoclinic (d) Lattice with unit cell trigonal
24. The number of nearest neighbours around each particle in a primitive cubic lattice is
(a) 4 (b) 6 (c) 8 (d) 12
25. Which of the following groups of crystal systems has the crystallographic axes a = b = c?
(a) Trigonal, Hexagonal (b) Trigonal, Cubic
(c) Trigonal, Hexagonal, Cubic (d) Orthorhombic, Cubic
26. Which of the following groups of crystal systems has the crystallographic axes a = b π c?
(a) Hexagonal, Tetragonal (b) Hexagonal, Monoclinic
(c) Hexagonal, Orthorhombic (d) Cubic, Orthorhombic
27. Which of the following groups of crystal systems has the crystallographic axes a π b π c?
(a) Trigonal, Triclinic (b) Tetragonal, Monoclinic
(c) Trigonal, Tetragonal (d) Triclinic, Monoclinic
28. Which of the following groups of crystal systems has the crystallographic anlges a = b = g = 90°?
(a) Tetragonal, Cubic, Orthorhombic (b) Tetragonal, Cubic, Monoclinic
(c) Cubic, Orthorhombic, Jrigonal (d) Hexagonal, Cubic, Orthorhombic
29. Which of the following groups of crystal systems has the crystallographic angles a = b = 90° π g ?
(a) Trigonal, Tetragonal (b) Hexagonal, Monoclinic
(c) Tetragonal, Hexagonal (d) Triclinic, Monoclinic
30. Which of the following crystal systems has crystallographic angles a π b π g ?
(a) Monoclinic (b) Triclinic (c) Trigonal (d) Tetragonal
31. Which of the following crystal systems has the largest number of Bravais lattices?
(a) Cubic (b) Tetragonal (c) Orthorhomic (d) Hexagonal
32. Which of the following groups of crystal systems has only two Bravais lattices?
(a) Tetragonal, Monoclinic (b) Tetragonal, Cubic
(c) Tetragonal, Orthorhombic (d) Orthorhombic, Hexagonal
2.82 Complete Chemistry—JEE Main

33. Which of the following crystal systems has three Bravais lattices?
(a) Tetragonal (b) Orthorhombic (c) Cubic (d) Monoclinic
34. The number of lattices having only one Bravais lattice is
(a) one (b) two (c) three (d) four
Of the three cubic Bravais lattices, primitive (P), body-centred (I) and face-centred (F), of elements, answer the
following questions.
35. The number of atoms per unit cell are
(a) P-one; I-two; F-four (b) P-one; I-four; F-two
(c) P-two; I-one; F-four (d) P-two; I-four; one
Packing in Crystalline Solids
36. The unit cell present in ABC ABC... packing of atoms is
(a) hexagonal (b) tetragonal (c) face-centred cubic (d) primitive cube
37. The unit cell present in ABAB... packing of atoms is
(a) hexagonal (b) tetragonal (c) face-centred cube (d) primitive cube
38. The factor which makes a solid to have a low density is
(a) close-packing (b) high atomic mass
(c) high occupancy of tetrahedral holes (d) large atomic radius
39. The closest-packed layers in the face-centred cubic unit cell are perpendicular to
(a) the face of the cell (b) the face diagonal of the cell
(c) edges of the cell (d) the body diagonal of the cell
40. If the same type of atoms are packed in hexagonal closest packing (HCP) and cubical closest packing (CCP)
separately, then
(a) density of HCP will be greater than CCP
(b) density of HCP will be smaller than CCP
(c) density of HCP will be equal to CCP
(d) density of HCP may be equal or greater than or smaller than CCP depending upon the temperature of the
system
41. In the closest packing of atoms,
(a) the size of tetrahedral void is greater than that octahedral void
(b) the size of tetrahedral void is smaller than that of octahedral void
(c) the size of tetrahedral void is equal to that of octahedral void
(d) the sizes of tetrahedral and octahedral voids vary depending upon their locations
42. In the closest packing of atoms, there are
(a) one tetrahedral void and two octahedral voids per atom
(b) two tetrahedral voids and one octahedral void per atom
(c) two of each of tetrahedral and octahedral voids
(d) three of each of tetrahedral and octahedral voids
43. In the closest packing of atoms A (radius ra) the radius of atom C (radius rc
hole in terms of ra is
(a) rc = 0.414 ra (b) rc = 0.225 ra (c) rc = 0.732 ra (d) rc = 0.155 ra
44. In the closest packing of atoms A (radius ra) the radius of atom C (radius rc)
in terms of ra is
(a) rc = 0.732 ra (b) rc = 0.414 ra (c) rc = 0.225 ra (d) rc = 0.155 ra
45. In the primitive cubic unit cell of closest packed atoms, the radius of atom (r) is related to the edge length (a) of
unit cell by the expression
(a) r = a/2 (b) r = a/2 2 (c) r = 3 a/4 (d) r = 4a/ 3
States of Matter 2.83

46. In the body-centred cubic unit cell of closest packed atoms, the radius of atom (r) is related to the edge length (a)
of unit cell by the expression
(a) r = a/2 (b) r = a/2 2 (c) r= 3 a/4 (d) r = 2 a
47. In the face-centred cubic unit cell of closest packed atoms, the radius of atom (r) is related to the edge length (a)
of unit cell by the expression
(a) r = a/2 (b) r = a 2 (c) r = a/2 2 (d) 3 a/4
48. The fraction of volume occupied in a primitive cubic unit cell is
(a) 0.52 (b) 0.48 (c) 0.55 (d) 0.45
49. The fraction of volume occupied in a body-centred cubic unit cell is
(a) 0.32 (b) 0.68 (c) 0.35 (d) 0.65
50. The fraction of volume occupied in a face-centred cubic unit cell is
(a) 0.26 (b) 0.30 (c) 0.74 (d) 0.70
51. The coordination number of atoms packed in hexagonal close-packed structure is
(a) 4 (b) 6 (c) 8 (d) 12
52. The radius of atom is related to the edge length of unit cell by the relations
(a) (P) r = a/4; (I) r = 3 a/2; (F) r = a/2 2 (b) (P) r = a/2; (I) r = 3 a/4; (F) r = a/2 2
(c) (P) r = 3 a/4; (I) r = a/2; (F) r = a / 2 (d) (P) r = a/4; (I) r = 3 a/2; (F) r = a/ 2
53. The coordination number of atoms are
(a) P : I: F : : 6 : 6 : 8 (b) P : I: F : : 8 : 6 : 6
(c) P : I: F : : 6 : 8 : 6 (d) P : I: F : : 6 : 6 : 6
54. The per cent void volume per unit cell are
(a) P : I : F : : 25.95 : 31.98 : 47.64 (b) P : I : F : : 52.36 : 68.02 : 74.05
(c) P : I : F : : 47.64 : 25.95 : 31.98 (d) P : I: F : : 47.64 : 31.98 : 25.95
55. The portion of edge-length not occupied by atoms are
(a) P : I : F : : 0 : a (1 – 3 /2) : a(1 – 1/ 2 ) (b) P : I : F : : 0 : a (1 –1/ 2 ) : a(1 – 3 /2)
(c) P : I : F : : a (1 – 1/ 2 ) : 0 : a(l – 4 3 /2) (d) P : I: F : : a (1 – 3 /2) : a(1 – 1/ 2 ) : 0
56. Diamond has face-centred cubic unit cell. The number of carbon atoms per unit cell is
(a) 2 (b) 4 (c) 6 (d) 8
57. An element crystallizes in hexagonal unit cell. The number of atoms per unit cell is
(a) 2 (b) 4 (c) 6 (d) 8
Ionic Solids
58. In the normal spinels some of the tetrahedral holes in the close-packed structure are occupied by
(a) oxide ions (b) dipositive ions (c) tripositive ions (d) tetrapositive ions
59. Which of the following substances does not crystallize in the rock-salt structure?
(a) NaCl (b) KC1 (c) MgO (d) CsCl
60. Which of the following statements is true in the rock-salt structure of an ionic compound?
(a) Coordination number of cation is four whereas that of anion is six
(b) Coordination number of cation is six whereas that of anion is four
(c) Coordination number of each cation and anion is four
(d) Coordination number of each cation and anion is six
61. Which of the following statements is true in the body-centred cubic structure of an ionic compound?
(a) Coordination number of each cation and anion is eight
(b) Coordination number of each cation and anion is six
(c) Coordination number of each cation and anion is four
(d) Coordination number of cation is six whereas that of anion is four
2.84 Complete Chemistry—JEE Main

62. The general formula, of an ionic compound crystallizing in body-centred cubic structure is
(a) AB (b) AB2 (c) A2B (d) AB3
63. The general formula of an ionic compound crystallizing in rock-salt structure is
(a) AB (b) AB2 (c) AB3 (d) A3B
64. Which of the following statements is true in the zinc-blende type structure of an ionic compound?
(a) Coordination number of each cation and anion is eight
(b) Coordination number of each cation and anion is six
(c) Coordination number of each cation and anion is four
(d) Coordination number cation is four whereas that of anion is six
65. The general formula of an ionic compound crystallizing in zinc-blende structure is
(a) AB (b) AB2 (c) A2B (d) AB3

(a) Coordination number of anion is four and that of cation is eight


(b) Coordination number of anion is eight and that of cation is four
(c) Coordination number of each cation and anion is four
(d) Coordination number of each cation and anion is eight

(a) AB (b) AB2 (c) A2B (d) AB3

(a) Coordination number of anion is four and that of cation is eight


(b) Coordination number of anion is eight and that of cation is four
(c) Coordination number of each cation and anion is four
(d) Coordination number of each cation and anion is eight

(a) AB (b) AB2 (c) A2B (d) AB3


70. If the anions (B) form hexagonal closest packing and cations (A) occupy only 2/3 octahedral holes in it, then the
general formula of the compound is
(a) AB (b) AB2 (c) A2 B3 (d) A3 B2
71. In spinel structure, oxides ions are cubical-closest packed, whereas l/8th of tetrahedral holes are occupied by
cations A2+ and 1/2 of octahedral holes are occupied by cations B3+ ions. The general formula of the compound
having spinel structure is
(a) A2BO4 (b) AB2O4 (c) A2B4O (d) A4B2O
72. In a cubic unit cell, seven of eight comers are occupied by atom A and centres of faces are occupied by B. The
general formula of the substance having this type structure would be
(a) A7B6 (b) A7B24 (c) A7B12 (d) A7 B36
73. An ionic compound is expected to have octahedral structure if rc/ra lies in the range of
(a) 0.414 to 0.732 (b) 0.732 to 0.82 (c) 0.225 to 0.414 (d) 0.155 to 0.225
74. An ionic compound is expected to have tetrahedral structure if rc/ra
(a) is more than 0.732 (b) lies in the range of 0.414 to 0.732
(c) lies in the range of 0.225 to 0.414 (d) lies in the range of 0.155 to 0.225
75. The structure of an ionic compound is expected to be cubic if rc/ra
(a) is greater than 0.732 (b) lies in the range of 0.414 to 0.732
(c) lies in the range of 0.225 to 0.414 (d) lies in the range of 0.155 to 0.225
76. Which of the following expressions is true in case of a sodium chloride unit cell?
(a) rc + ra = a (b) rc + ra = a/2 (c) rc + ra = 2a (d) rc + ra = 2 a
where rc, ra and a have their usual meanings
States of Matter 2.85

77. Ammonium chloride crystallizes in a body-centred cubic lattice with a unit distance of 387 pm. If the size of
Cl– ion is 181 pm, the size of NH4+ ion would be
(a) 206 pm (b) 116 pm (c) 174 pm (d) 154 pm
78. The edge length of sodium chloride unit cell is 564 pm. If the size of CI– ion is 181 pm, the size of Na+ ions would
be
(a) 282 pm (b) 383 pm (c) 101pm (d) 167 pm
79. A compound formed by elements A and B crystallises in a cubic structure where A atoms are at the corners of a
cube and B atoms are at the face centres of the cube. The formula of the compound is
(a) AB4 (b) AB3 (c) AB2 (d) AB
80. The 8:8 type packing is present in
(a) NaCl (b) CaF2 (c) CsCl (d) KC1
81. An ionic compound AxBy crystallized in hexagonal unit cell in which the anions B occupy the lattice points and
the cations A occupy 2/3rd octahedral voids. The simplest formula of the compound is
(a) AB2 (b) A2B3 (c) A3B4 (d) A2B
82. An ionic compound AxByCz crystallizes in cubical-closest packing in which the ions A occupy the lattice points,
the ions B occupy one-eighth of tetrahedral voids and the ions C occupy one-half of octahedral voids. The simplest
formula of the compound is
(a) A4BC2 (b) A4B2C (c) A2BC4 (d) A2B4C
Density of Crystalline Solids
83. The expression to compute the density of cubic crystal is
NÊ M ˆ a3 Ê M ˆ a3 Ê N A ˆ a3 Ê M ˆ
(a) r = 3 Á ˜ (b) r = Á ˜ (c) r = Á ˜ (d) r =
a Ë NA ¯ N Ë NA ¯ NË M ¯ 6 ÁË N A ˜¯
where the various symbols have their usual meanings.
84. The number of atoms associated with a single primitive cubic unit cell is
(a) 1 (b) 2 (c) 4 (d) 8
85. The number of atoms associated with a single face-centred cubic unit cell is
(a) 1 (b) 2 (c) 4 (d) 8
86. The number of atoms associated with a single body-centred cubic unit cell is
(a) 1 (b) 2 (c) 4 (d) 8
87. The unit cell length of sodium chloride crystal is 564 pm. Its density would be
(a) 2.165 g cm–3 (b) 4.330 g cm–3 (c) 1.082 g cm–3 (d) 3.247 g cm–3
–1
88. The cubic unit cell of aluminium (molar mass 27.0 g mol ) has an edge length of 405 pm. Its density is 2.70 g
cm–3. The type of unit cell is
(a) Primitive (b) Face-centred (c) Body-centred (d) End-centred
89. A substance which has face-centred cubic crystal has a density of 1.984 g cm–3 and the edge length of unit cell
equal to 630 pm. The molar mass of the substance is
(a) 37.35 g mol–1 (b) 56.02 g mol–1 (c) 74.70 g mol–1 (d) 65.36 g mol–1
90. If the atomic mass of elements are the same in the three cubic lattices, then the order of densities is
(a) rP < rF < rI (b) rP < rI < rF (c) rF < rP < rI (d) rF < rI < rP
91. Potassium has a body-centred cubic structure with the nearest neighbour distance 452 pm. Its relative atomic
mass is 39. Its density would be
(a) 0.71 g cm–3 (b) 0.81 g cm–3 (c) 0.91 g cm–3 (d) 1.01 g cm–3
92. A metal crystallises in two cubic phases, FCC and BCC with unit length equal to 350 pm and 300 pm, respectively.
The ratio of densities of FCC and BCC is about
(a) 1.26 (b) 1.36 (c) 1.46 (d) 1.66
2.86 Complete Chemistry—JEE Main

93. The pyknometric density of sodium chloride crystal is 2.16 g cm–3 while its X-ray density is 2.18 g cm–3. The
fraction of unoccupied site in sodium chloride crystal is
(a) 0.0043 (b) 0.0056 (c) 0.0093 (d) 0.0082
Point Defects, Dielectric Properties, etc.
94. Which of the following statements is not correct?
(a) The density of the crystal exhibiting Schottky defect is less as compared to that of the perfect crystal
(b) The density of the crystal exhibiting Frenkel defect is less as compared to that of the perfect crystal
(c) The Schottky defects are more common in ionic compounds with high coordination numbers and where the
sizes of ions are small
(d) In alkali halides, Frenkel defects are not found.
95. Which of the following statements is not correct?
(a) Silver bromide shows both types of Schottky and Frenkel defects
(b) The compound ferrous oxide can be prepared in its stoichiometric composition
(c) The creation of cationic holes in ionic crystals imparts electrical conductivity to such crystals
(d) Heating of zinc oxide leads to the formation of a yellow coloured nonstoichiometric compound.
96. Which of the following is expected to be diamagnetic?
(a) TiO (b) TiO2 (c) Ti2O3 (d) VO
97. Which of the following is expected to be paramagnetic?
(a) TiO (b) TiO2 (c) NaCl (d) Benzene
98. An electron trapped in an anionic site in a crystal is called
(a) F-centre (b) Frenkel defect (c) Schottky defect (d) Interstitial defect
99. In a ferromagnetic material
(a) all the magnetic moment vectors are aligned in one direction.
(b) half of the magnetic moment vectors point in one direction and the rest in the opposite direction.
(c) all the magnetic moment vectors are randomly oriented.
(d) is characterised by small magnetic moment.
100. In an antiferromagnetic material
(a) all the magnetic moment vectors are aligned in one direction.
(b) half of the magnetic moment vectors point in one direction and the rest in opposite direction.
(c) all the magnetic moment vectors are randomly oriented.
(d) is characterised by a very large magnetic moment.
101. In a ferrimagnetic material
(a) all the magnetic moment vectors point in one direction.
(b) all the magnetic moment vectors are randomly oriented.
(c) contains equal number of magnetic moment vectors oriented in opposite directions.
(d) contains unequal number of magnetic moment vectors in opposite directions.

ANSWERS
1. (a) 2. (d) 3. (d) 4. (d) 5. (c) 6. (a)
7. (b) 8. (c) 9. (d) 10. (d) 11. (c) 12. (a)
13. (a) 14. (c) 15. (a) 16. (d) 17. (a) 18. (d)
19. (d) 20. (d) 21. (b) 22. (b) 23. (c) 24. (b)
25. (b) 26. (a) 27. (d) 28. (a) 29. (b) 30. (b)
31. (c) 32. (a) 33. (c) 34. (c) 35. (a) 36. (c)
37. (a) 38. (d) 39. (d) 40. (c) 41. (b) 42. (b)
States of Matter 2.87

43. (a) 44. (c) 45. (a) 46. (c) 47. (c) 48. (a)
49. (b) 50. (c) 51. (d) 52. (b) 53. (c) 54. (d)
55. (a) 56. (d) 57. (c) 58. (b) 59. (d) 60. (d)
61. (a) 62. (a) 63. (a) 64. (c) 65. (a) 66. (a)
67. (b) 68. (b) 69. (c) 70. (c) 71. (b) 72. (b)
73. (a) 74. (c) 75. (a) 76. (b) 77. (d) 78. (c)
79. (b) 80. (c) 81. (b) 82. (a) 83. (a) 84. (a)
85. (c) 86. (b) 87. (a) 88. (b) 89. (c) 90. (b)
91. (c) 92. (a) 93. (c) 94. (b) 95. (b) 96. (b)
97. (a) 98. (a) 99. (a) 100. (b) 101. (d)

HINTS AND SOLUTIONS


48. f = (4/3) pr3/a3 = (4/3) p(a/2)3 /a3 = p/6 =0.52
77. Body diagonal of cube = 3 a = (1.732) (387 pm) = 670.3 pm. Thus 2(r+ + r–) = 670.3 pm
670.3
which gives r+ = pm – 181 pm = 154 pm
2
564
78. 2(r+ + r–) = 564 pm; r+ = pm –181 pm = 101 pm
2
NÊ M ˆ 4 Ê 58.5 g mol-1 ˆ
87. r = 3 Á ˜ = = 2.16 ¥ 106 g m–3 = 2.l6 g cm–3
a Ë N A ¯ (564 ¥ 10-12)3 ÁË 6.022 ¥ 1023 mol-1 ˜¯
Ê 6.022 ¥ 1023 mol-1 ˆ
88. N = r a3 ÊÁ A ˆ˜ = (2.70 g cm -3 )(405 ¥ 10-10 cm)3 Á
N
=4
Ë M ¯ Ë 27.0 g mol-1 ˜¯
The unit cell is face-centred cube.
r a3 N A (1.984 g cm -3 )(630 ¥ 10-10 cm3 )(6.022 ¥ 1023 mol-1 )
89. M = = = 74.7 g mol-1
N 4
NM (2)(29 g mol -1 )
91. r = = = 9.11 ¥ 105 g m -3 = 0.91 g cm -3
a3 N A (2 ¥ 452 ¥ 10 -12 m / 3 )3 (6.022 ¥ 1023 mol -1 )

92. For FCC, N = 4 and for BCC, N =2. Hence


r (FCC) Ê N ˆ Ê a3 ˆ Ê 4 ˆ Ê (3.0)3 ˆ
= Á 3˜ = = 1.26
r (BCC) Ë a ¯ FCC ÁË N ˜¯ BCC ÁË (3.5)3 ˜¯ ÁË 2 ˜¯

( M / r1 ) - ( M / r2 ) (1 / r1 ) - (1 / r2 ) r2 - r1 r2 2..18
93. f = = = = -1 = - 1 = 1.00926 - 1 = 0.00926
( M / r2 ) (1 / r2 ) r1 r1 2.16

MULTIPLE CHOICE QUESTIONS FROM AIEEE AND JEE MAIN

1. What type of crystal defects is indicated in the diagram below?


Na+ Cl– Na+ Cl– Na+ Cl–
Cl– Cl– Na+ Na+
Na+ Cl– Cl– Na+ Cl–
Cl Na Cl Na+
+ – Na+
(a) Frenkel defect (b) Schottky defect
(c) Interstital defect (d) Frenkel and Schottky defects [2004]
2.88 Complete Chemistry—JEE Main

2. An ionic compound has a unit cell consisting of A ions at the corners of a cube and B ions on the centres of the
faces of the cube. The empirical formula for this compound will be
(a) AB (b) AB3 (c) A3B (d) A2B [2005]
3. Total volume of atoms present in a face-centred cubic unit cell of a metal (r is atomic radius)
16 20 3 24 3 12
(a) pr3 (b) pr (c) pr (d) pr3 [2006]
3 3 3 3
4. In a compound, atoms of element Y form cubical-closest packing and those of elements X occupy 2/3 of tetrahedral
voids. The formula of the compound will be.
(a) X3Y (b) X4Y3 (c) X2Y3 (d) X2Y [2008]
5. Copper crystallises in fcc with a unit cell length of 361 pm. What is the radius of copper atom?
(a) 157 pm (b) 181 pm (c) 108 pm (d) 127 pm [2009], [2011]
6. The edge length of a face-centred cubic cell of an ionic substance is 508 pm. If the radius of the cation is 110 pm,
the radius of the anion is
(a) 144 pm (b) 288 pm (c) 398 pm (d) 618 pm [2010]
7. Percentage of free space in cubic close-packed structure and in body-centred packed structure are respectively
(a) 48% and 26% (b) 30% and 26% (c) 26% and 32% (d) 32% and 48% [2011]
8. In a face-centred lattice, atom A occupies the corner positions and atom B occupies the faced centre position. If
one atom of B is missing from one of the face-centred points, the formula of the compound is
(a) A2B5 (b) A2B
(c) AB2 (d) A2B3 [2011], [2014 online]
9. Lithium forms body centred cubic structure. The length of the side of its unit cell is 351 pm. Atomic radius of
lithium will be
(a) 175 pm (b) 300 pm (c) 240 pm (d) 152 pm [2012]
10. Which of the following exists as covalent crystals in the solid state?
(a) Iodine (b) Silicon (c) Sulphur (d) Phosphorus [2012]
11. CsCl crystallises in body centred cubic lattice. If a is its edge length then which of the following expression is
correct?
(a) rCs+ + rCl– =3a/2 (b) rCs+ + rCl– = 3 a/2 (c) rCs+ + rCl– = 3 a (d) rCs+ + rCl– = 3a [2014]
12. How many unit cells are present in a cube-shaped ideal crystal of NaCl of mass 1.00 g?
(a) 1.71 ¥ 1021 (b) 2.57 ¥ 1021 (c) 5.14 ¥ 1021 (d) 1.28 ¥ 1021 [2003]
13. The total number of octahedral void(s) per atom present in a cubical close-packed structure is
(a) 2 (b) 4 (c) 1 (d) 3 [2014 online]
14. The appearance of colour in solid alkali metal halides is generally due to
(a) Schottky defect (b) Frenkel defect
(c) Interstitial position (d) F-centres [2014 online]
15. In a monoclinic unit cell, the relation of sides and angles are respectively
(a) a = b π c and a = b = g = 90° (b) a π b π c and a = b = g = 90°
(c) a π b π c and a = g = b = 90° = a (d) a π b π c and a π b π g π 90° [2014 online]
16. Sodium metal crystallizes in a body centred cubic lattice with a unit cell edge of 4.29 Å. The radius of sodium
atom is approximately
(a) 1.86 Å (b) 3.22 Å (c) 5.72 Å (d) 0.93 Å [2015]

ANSWERS
1. (b) 2. (b) 3. (a) 4. (b) 5. (d) 6. (a)
7. (c) 8. (a) 9. (d) 10. (b) 11. (b) 12. (b)
13. (b) 14. (d) 15. (c) 16. (a)
States of Matter 2.89

HINTS AND SOLUTIONS


1. Missing of species from their expected location gives rise to Schottky defect.
2. Each corner of a cube is shared amongst eight cubic unit cells. The contribution of atoms A at the eight corners
of a cubic unit cell is as follows.

Contribution from corners = 8 ÊÁ ˆ˜ = 1


1
Ë 8¯
Each face of a cube is common between two unit cells. The contribution of atoms B at the centre of six faces of
a cubic unit cell is as follows.

Contribution from corners = 6 ÊÁ ˆ˜ = 3


1
Ë 2¯
Hence, the empirical formula of the compound would be AB3.
3. In a face-centred cubic unit cell, there are four atoms. Hence, the volume occupied by atoms is 4(4/3 pr3), i.e.
(16/3) pr3.
4. In closest-packed atoms, there are two tetrahedral voids per closest-packed atom. Hence, the formula of the
compound will be YX2(2/3) i.e. YX4/3 or X4Y3
Alternatively, there are four Y per unit cell and 8(2/3) X. Hence, the formula is Y4X16/3 i.e. Y12X16 or Y3X4.
5. For a face-centred cubic unit cell, atoms touch each other along the face-diagonal of the cube. Hence
4r= 2a
This gives r = ( 2 ) (361 pm)/4 = 127.6 pm
6. In the face-centred cubic cell of an ionic substance, the cation-anion touch each other along the edges of the cube.
The example of such an ionic compound is sodium chloride. Hence,
2r+ + 2r– = a fi 2(110 pm) + 2r– = 508 pm. This gives r_ = 144 pm
7. In cubical-closest packing, atoms touch each other along the face diagonal of the cube. Hence, 4r = 2 a. The
number of atoms in a unit cell is four. Hence
Volume of a cube = a3
È4 Ê 2 ˆ3˘
Volume occupied by atoms = Í p Á
4 a˜ ˙ ; Fraction of volume occupied = 2p = 0.74
ÍÎ 3 Ë 4 ¯ ˙˚ 6
Percentage of void volume = (1– 0.74) (100) = 26%
In a body-centred cube, atoms touch each other along the body-diagonal of the cube. Hence, 4r = 3 a
The number of atoms in a unit cell is two. Hence
Volume of a cube = a3
È 4 Ê 3a ˆ 3 ˘ 3p
Volume occupied by atoms = 2 Í p Á ˜¯ ˙ ; Fraction of volume occupied = = 0.68
ÍÎ 3 Ë 4 ˙˚ 8
Percentage of void volume = (1 – 0.68) (100) = 32%

8. There are eight comers and six faces in a cube. Each atom at the comer is shared amongst eight unit cells and by
the two unit cells at the centre of each face. Thus, contributions of A and B from a unit cell are as follows.
Contribution of atom A = 8(1/8) = 1 and Contribution of atom B = 5(1/2) = 5/2
Hence, the formula of the compound is AB5/2 i.e. A2B5.
9. In body centred cubic structure, atoms (radius: r) touch each other along the body diagonal of the cube (edge
length : a). Thus 4r = 3 a
Ê 3ˆ Ê 1.732 ˆ (351 pm) = 152 pm
r = Á ˜ a = ÁË ˜
Ë ¯
4 4 ¯
2.90 Complete Chemistry—JEE Main

10. Iodine, sulphur and phosphorus exist I2, S6 and P4, respectively. Silicon does exist as covalent crystal in the solid
state.
11. In a body-centred cubic arrangements, ions touch each other along the body diagonal.
Hence, 2rCs+ + 2 rCl– = 3 a or rCs+ + rCl– = 3 a/ 2
12. The are four molecules of NaCl per unit cell. Hence, the mass of one unit cell is
m = 4(23 + 35.5) g/6.022 ¥ 1023
The number of unit cell in 1.00 g of crystal will be
6.022 ¥ 1023
N= = 2.57 ¥ 1021
4 ¥ (23 + 35.5)
13. There are 4 octahedral voids. These are situated at the centres of cube and at the centres of edges.
14. The electrons, which occupy the anion vacancy are called F-centres. This imparts colour to the alkali metal halides.
15. For monoclinic unit cell, the structural parameters are a π b π c and a = b = 90° π g
16. In a body centred cubic lattice, atoms touch each other along the body diagonal of the cube. Hence
4r = 3a or r = ( 3 / 4) a ( 3 / 4)(4.29Å) = 1.86 Å
3
Atomic Structure

SECTION 1 Development of Structure of Atom

The word ‘atomic’ has its origin in an adjective in Greek meaning ‘not divisible’. Based on experimental evidence,
Dalton (1808) proposed that matter is composed of atoms which cannot be created, divided or destroyed.
At the end of nineteenth century, experimental evidence established that an atom can be subdivided. It is made up of
three subatomic particles, namely, protons neutrons and electrons. Their main characteristics are as follows.
Mass Charge Symbol
Proton 1.672 ¥ 10 –27 kg 1.6022 ¥10–19 C p
Neutron 1.675 ¥ 10–27 kg No charge n
Electron 9.1096 ¥ 10–31 kg –1.6022 ¥ 10–19 C e
After the discovery of proton, neutron and electron, their arrangement in atom was proposed as described in the
following.
Thomson Model (1898)
An atom is positively-charged sphere of radius about 10–10 m in which negatively-charged electrons are uniformly
embedded so as to have an overall electrically neutral atom.
Rutherford Model
In 1910, Rutherford carried out an experiment in which a thin sheet of gold foil was bombarded with a beam of
a-particles (He2+). He observed that :
(i) Most of a
(ii) A few of a

Based on these observations, Rutherford proposed the following model of an atom:


An atom is made up of tiny positively-charged nucleus containing practically whole of mass of the atom. Electrons
are revolving around the nucleus with high speeds in circular path, known as orbits.
Rutherford also concluded that:
(i) The radii of nuclei of atoms are of the order of 10–15 m to 10–14 m, and
(ii) The radii of outer region where electrons move is of the order of 10–10 m.
Thus, most of space in an atom is emptly and the volume occupied by the nucleus is less than 10–12 of the total
volume of an atom.
Subsequently, it was discovered that the nucleus of an atom is made up of two particles, namely, positively-charged
protons and electrically-neutral neutrons.
3.2 Complete Chemistry—JEE Main

Rutherford model of an atom could not be sustained based on the following fact.
In order to maintain circular motion of electron around the nucleus, it is subjected to acceleration. According to
Maxwell theory of electromagnetic radiation a charged particle when subjected to acceleration should lose energy in
the form of electromagnetic radiation. Hence, the electron will lose energy continuously and eventually will fall into
the nucleus. Calculations have shown that only 10–8 s will be required for the electron to fall in the nucleus.
Rutherford model also do not explain the discrete spectra shown by the atoms. In 1885, Balmer analyzed the spectra
shown by hydrogen atom and established the expression
Ê n2 ˆ
l /nm = 364.56 Á 2 2 2 ˜
Ë n2 - n1 ¯
for the observed spectra in the visible and ultraviolet region of radiations. The value of n1= 2 and n2 can take values
3,4, . . ., etc.
Bohr Model
In order to explain the stability of hydrogen and hydrogen-like species (such as He+, Li2+, . . ., etc.), Niels Bohr, in 1913,
proposed the following two assumptions:
1. The electrons in an atom revolve around the nucleus only in certain allowed circular orbits without losing any
energy.
2. The electron can jump from one of the orbits to another and can there by gain or lose energy equivalent to the
difference in energy of the two involved orbits in the form of electromagnetic radiation such that
DE = hn = h (c/l) (1)
where
DE is the energy difference between the two orbits.
n (or l) is the frequency (or wavelength) of the radiation and c is the speed of light ( 3 ¥ 108 m s–1).
Criterion for stability of the atom The stability of the atom is explained based on the expression
|attractive (centripetal) force| = centrifugal force
( Ze)(e) mu2
i.e. = (2)
( 4p e 0 ) r 2 r
where Z is the atomic number of the nucleus, e is the elementary charge, u is the speed of electron in the orbit of radius
r, and eo is the permittivity of a vacuum and has a value of 8.854 ¥ 10–12 C2 N–1 m–2.
Postulate for Allowed Stationary Orbits
quantum restriction on the angular momentum of the revolving electrons.

mu r = n ÊÁ ˆ˜
h
(3)
Ë 2p ¯
where n has integral values 1,2,3, . . ., and is known as quantum number. The symbol h is Planck’s constant and has a
value of 6.626 ¥ 10–34 J s.
Radius of Permitted Orbits Equation (2) gives
Z e2
= mu2
( 4p e 0 ) r
Eliminating u by using Eq. (3), we get

Z e2 2
= m Èn ÊÁ h ˆ˜ 1 ˘
( 4p e 0 ) r Í Ë 2p ¯ m r ˙
Î ˚
È h2 ˘
This gives r = n2 Í 2 ˙ (4)
Î 4p m ( Z e / 4p e 0 ) ˚
2
Atomic Structure 3.3

Energy of electron in a orbit We have


1 Z e2
E = kinetic energy + potential energy = mu 2 -
2 ( 4p e 0 ) r

1 1 1 (mu )2
E= mu 2 - mu 2 = - mu 2 = - (5)
2 2 2 m
Eliminating mu by using Eq. (3), we get

1 È Ê h ˆ 1 ˘2
E= - nÁ ˜
2m ÍÎ Ë 2p ¯ r ˙˚
Substituting the expression of r from Eq. (4), we get

1 È Ê h ˆ 4p 2 m( Ze2 / 4p e 0 )2 ˘ 1 È 2p 2 m( Z e2 / 4pe 0 )2 ˘
E= - Ín Á ˜ ˙ =- 2 Í ˙ (6)
2m Î Ë 2p ¯ 2 2
n h ˚ n Î h2 ˚

Speed of Electron in an orbit Substituting E = – (1/2)mu2 from Eq. (5) in Eq. (6), we get

1 1 È 2p 2 m ( Z e 2 / 4p e 0 ) 2 ˘
- mu 2 = - 2 Í ˙
2 n Î h2 ˚

2p ( Ze / 4p e 0 )
This gives u= (7)
nh
Points to Remember:
n2 Z2 Z
rμ ; E μ– 2 ; u=
Z n n
E = –KE, PE = –2KE and PE = 2E
where KE and PE are kinetic energy and potential energy, respectively
Discrete spectra of Hydrogen Atom The energy difference between two Bohr orbits is

2p 2 m( Ze2 / 4p e 0 )2 Ê 1 1ˆ
DE = E2 – E1 = Á - 2˜ (8)
h 2
Ë n1 n2 ¯
2

The frequency and wavenumber of radiation having energy DE are

DE 2p 2 m( Ze2 / 4p e 0 )2 Ê 1 1ˆ
n= = Á - 2˜ (9)
h h 2
Ë n1 n2 ¯
2

~ 1 DE 2p 2 m( Ze2 / 4p e 0 )2 Ê 1 1ˆ Ê 1 1ˆ
n= = = Á - 2 ˜ = Z 2R • Á 2 - 2 ˜ (10)
l hc 3
hc Ë n1 n2 ¯
2 Ë n1 n2 ¯

2p 2 m( Ze2 / 4pe 0 )2
where R• = and is known as Rydberg constant. Its value is
h3 c
R• = 1.097 37 ¥107 m–1
The spectra shown by hydrogen atom can be explained by assigning different values of n1 and n2 in Eq. (10) keeping
in mind that n2 > n1, These are shown in Table 1 and Fig. 1
3.4 Complete Chemistry—JEE Main

Table.1
Spectral Values of Region of radiation
series
n1 n2
Lyman 1 2,3,4, . . . ultraviolet
Balmer 2 3,4,5, . . . Visible
Paschen 3 4,5,6, . . . Near infrared
Brackett 4 5,6, . . . Infrared
Pfund 5 6,7, . . . Far infrared

Ionization Energy of Hydrogen Atom For


determining ionization energy, we set n1 = 1 and n2 =
• . This gives
2 p 2 m (e 2 / 4 p e 0 ) 2
IE = = 2.182 ¥10–18 J
h2
For one mole of hydrogen atom
IE = NA(2.182 ¥10–18 J) = (6.022 ¥1023 mol–1) (2.182 ¥
10–18 J) = 1314 ¥ 103 J mol–1 = 1314 kJ mol–1
Bohr theory is inadequate
in explaining the experimental facts of multi-electronic Fig. 1

The Bohr theory was not in agreement with the principles discovered later on. These are wave-particle duality by
de-Broglie and the uncertainty principle of Heisenberg.
In the light of these facts, Bohr theory was replaced by the current quantum theory.
Illustrative problems
1.
Given: R• = 1.0974 ¥ 107 m–1
n1= 2 and n2 = 3. Hence
hc Ê 1 1ˆ
DE = = R• hc Á 2 - 2 ˜
l Ë n1 n2 ¯
1 Ê 1 1ˆ Ê 1 1ˆ
or = R• Á 2 - 2 ˜ = (1.0974 ¥ 107 m -1 ) Á - ˜ = 1.524 ¥ 106 m–1
l Ë n1 n2 ¯ Ë 4 9¯

l = 1/(1.52 ¥ 106 m–1) = 6.56 ¥ 10–7 m = 656 nm


2. What transition in the hydrogen atom will have the same energy as the transition in He+ ion from the level
n = 4 to the level n = 2 .
Ê 1 1ˆ Ê 1 1ˆ 3
For helium ion DE = Z2R•hc Á 2 - 2 ˜ = (22)R•hc Á 2 - 2 ˜ = R•hc
Ë n1 n2 ¯ Ë 2 4 ¯ 4

Ê 1 1ˆ
For hydrogen atom DE= R•hc Á 2 - 2 ˜
Ë n1 n2 ¯

Obviously n1 = 1 and n2 = 2. Hence, the transition n = 2 to n =1 in hydrogen atom will have the same wavelength
as the transition n = 4 to n = 2 in He+ ion.
3. Calculate the radius of n = 2 orbit in Li2+ ion. Given : e2/4p e0 = 2.30 ¥ 10–28 N m2.
Atomic Structure 3.5

È h2 ˘ È (6.626 ¥ 10-34 J s)2 ˘


r = n2 Í ˙ = (2 2)
Í - - ˙
Î (4)(3.14) (9.1 ¥ 10 kg )(3 ¥ 2.30 ¥ 10 N m ) ˚
2 31 28 2
Î 4p m( Ze / 4p e 0 ) ˚
2 2

= 7.09 ¥ 10–11 m =70.9 pm.


4. p e0 =1.112 ¥ 10–10 C2 N–1 m–2.
2p( Ze / 4p e 0 )
2
The expression of speed is u =
nh
For H-atom, Z = 1. Hence
2(3.14)(1)(1.6 ¥ 10-19 C) 2 /(1.112 ¥ 10-10 C2 N -1 m -2 )
u= = 2.18 ¥ 106 m s–1
(1)(6.626 ¥ 10-34 J s)
+
5. ion.
Given : 4p e0 =1.112 ¥ 10 –10 2
C N m–1 –2

The speed of electron is


u = 2p( Ze / 4p e 0 )
2

nh
2(3.14)(2)(1.6 ¥ 10-19 C) 2 /(1.112 ¥ 10-10 C2 N -1 m -2 )
=
(1)(6.626 ¥ 10-34 J s)
= 4.36¥106 m s–1
Ê h2 ˆ
r = n2 Á 2
Ë 4p m( Ze / 4p e 0 ) ˜¯
2

The value Ze2/4p e0 is

Z e2 (2)(1.6 ¥ 10-19 C)2


= = 4.60 ¥ 10–28 N m2
4p e 0 (1.112 ¥ 10-10 C2 N -1 m -2 )

È (6.626 ¥ 10-34 J s) 2 ˘
Now r = (1)2 Í -31 -28 2 ˙ = 2.66 ¥ 10
–11 m
Î (4)(3.14) (9.1 ¥ 10 kg )(4.60 ¥ 10 N m ) ˚
2

The number of revolution made is


u 4.36 ¥ 106 m s -1
n= = -11
= 2.61 ¥ 1016 s -1
2pr 2(3.14)(2.66 ¥ 10 m)

MULTIPLE CHOICE QUESTIONS ON SECTION 1

1. Isotopes contain
(a) same number of neutrons (b) same number of protons
(c) same mass number (d) same number of electrons
2. Isotones contains
(a) same number of neutrons (b) same number of protons
(c) same number of electrons (d) same mass number
3. The ratio of E2 – E1 and E5 – E4 of orbits in hydrogen-like species is
(a) 108/7 (b) 100/3 (c) 152/3 (d) 50/3
4. Which of the following orbit of He+ ion will have the same energy as that of second orbit of hydrogen atom?
(a) n = 1 (b) n = 2 (c) n = 3 (d) n = 4
3.6 Complete Chemistry—JEE Main

5. The ratio of kinetic energy and total energy of an electron in Bohr orbit of hydrogen-like species is
(a) 1/2 (b) 1/4 (c) –1 (d) –1/2
6. The ratio of potential energy and total energy of an electron in Bohr orbits of hydrogen atom is
(a) 2 (b) 1/2 (c) 1 (d) –1/2
7. The wavelength of radiation emitted when an electron jump from n = 4 to n = 2 energy levels of hydrogen atom
is (Given: Rydberg constant = 1.097 ¥107 m–1)
(a) 342 nm (b) 252 nm (c) 486 nm (d) 890 nm
8. The number of possible spectral lines emitted when electron in n = 4 Bohr orbit reaches to n = 1 Bohr orbit is
(a) 1 (b) 2 (c) 4 (d) 6
+, the energy E of an electron in nth orbit of hydrogen atom
9. If E1 2
will be
(a) E1/2n (b) E1/2n2 (c) E1/4n2 (d) E1/8n2
10. For which of the following hydrogen-like species, the wavelength of radiation emitted due to electron shifts from
= 2 to n = 1, has a maximum value?
(a) H (b) He+ (c) Li2+ (d) Be3+
11. For which of the following electronic transitions in hydrogen-like species, the frequency of emitted radiation will
be maximum?
(a) n = 2 to n = 1 (b) n = 3 to n = 2 (c) n = 4 to n = 2 (d) n = 6 to n = 4
+ having n = 1 is about (Given : e2/ 4p e = 2.30 ¥ 10–28 N m2)
0
(a) 53.4 pm (b) 106.8 pm (c) 26.7 pm (d) 42.1 pm
n =1) of hydrogen atom is 53.4 pm. The Bohr orbit having n = 3 in Li2+ will be
(a) 53.4 pm (b) 106.8 pm (c) 120.1 pm (d) 160.2 pm

also permitted for the electron in Bohr orbits of hydrogen atom?


(a) –6.8 ev (b) –4.53 ev (c) –3.4 ev (d) –2.27 ev
¥ 105 m s–1. The speed of electron in the n = 2 level of Li2+
ion will be
(a) 7.27 ¥105 m s–1 (b) 1.09 ¥106 m s–1 (c) 1.45 ¥106 m s–1 (d) 1.82 ¥106 m s–1
16. The ionization energy of hydrogen atom is 1314 kJ mol–1. The energy of n = 3 electronic level in this atom is
(a) –2.182 ¥ 10–18 J (b) –4.27 ¥10–18 J (c) –2.42 ¥ 10–19 J (d) –2.182¥10–19 J
17. The ionization energy of hydrogen atom is 1314 kJ mol–1. The ionization energy of He+ ion is expected to be
(a) 1314 kJ mol–1 (b) 2628 kJ mol–1 (C) 3952 kJ mol–1 (D) 5256 kJ mol–1

ANSWERS
1. (b) 2. (a) 3. (b) 4. (d) 5. (c) 6. (a)
7. (c) 8. (d) 9. (c) 10. (a) 11. (a) 12. (c)
13. (d) 14. (c) 15. (b) 16. (c) 17. (d)

HINTS AND SOLUTIONS


1. Isotopes contain same number of protons.
2. Isotones contain same number of neutrons.

Z2 Z2 3 2 Z2 Z2 9 2 E2 - E1 3 400 100
3. E2 – E1 μ – + = Z ; E5 – E4 μ – + = Z ; = ¥ =
4 1 4 25 16 400 E5 - E4 4 9 3
Atomic Structure 3.7

4. Since E μ – Z2/n2, the orbit n = 4 of He+ will have the same energy as that of n = 2 of hydrogen atom
5. The ratio KE/E = –1
6. E = –KE = PE/2 Thus PE/E = 2
1 Ê 1 1ˆ Ê 1 1ˆ
= R• Á 2 - 2 ˜ = (1.097 ¥ 107 m -1 ) Á 2 - 2 ˜ = (1.097 ¥ 107 mol-1 ) ÊÁ ˆ˜
3
7.
l Ë n1 n2 ¯ Ë 2 4 ¯ Ë 16 ¯
l = 4.86 ¥ 10–7 m = 486 nm
8. The possible spectral emissions correspond to the transfer n = 4 to n = 3, n = 4 to n =2, n = 4 to n = 1, n = 3 to
n = 2, n = 3 to n =1 and n = 2 to n = 1. There are 6 emissions.
9. Since En μ – Z2/n2, we have

22 12 1 E1/ 4 E1
E1(He+) μ – = –4 and En(H) = - = - 2 . Hence En(H) = 2
=
12 n 2
n n 4n 2
Ê 1 1ˆ 4
= Z 2 R• Á 2 - 2 ˜ = Z 2 R• ÊÁ 2 - 2 ˆ˜ = Z 2 R• . Hence,
1 1 1 3
10. l= .
l Ë n1 n2 ¯ Ë1 2 ¯ 4 3Z 2 R•
Lesser the value of Z (i.e. hydrogen atom) larger the wavelength.
Ê 1 1ˆ
11. We have n/Z2R•c = Á 2 - 2 ˜ . Hence
Ë n1 n2 ¯
1 1 1 1 3 108 1 1 1 1 5 20
Choice a 2
- 2 = 2- 2 = = Choice b 2
- 2 = 2- 2 = =
n1 n2 1 2 4 144 n1 n2 2 3 36 144

1 1 1 1 3 27 1 1 1 1 5 5
Choice c 2
- 2 = 2- 2 = = Choice d 2
- 2 = 2- 2 = =
n1 n2 2 4 16 144 n1 n2 4 6 144 144
Obviously, the choice a is correct.

È h2 ˘ È (6.626 ¥ 10-34 J s) 2 ˘
12. r = n2 Í 2 ˙ = (1)
2
Í -31 -28 2 ˙
Î 4 p m ( Ze 2
/ 4 p e 0 ˚
) Î 4(3.14) (9.1 ¥ 10 kg )(2 ¥ 2.30 ¥ 10 N m ) ˚
2

= 2.66 ¥ 10–11 m = 26.6 pm.


13. Since r μ n2/Z, we have
rLi2+ (32 / 3)
= 2 =3 fi rLi2+ = 3rH = 3 ¥ 53.4 pm = 160.2 pm
rH 1 /1
14. Since E μ – Z2/n2, we can have E = –3.4 eV for n = 2

15. Since u μ Z/n, we will have


u (Li 2+ ) 3 / 2 Ê 3ˆ Ê 3ˆ -1
u (Li+) = ÁË ˜¯ u(H) = ÁË ˜¯ (7.27 ¥ 10 m s ) =1.09 ¥ 106 m s–1
5
= fi
u (H) 1/1 2 2
(1314 ¥ 103 J mol-1 )
16. Ionization energy per hydrogen atom is IE = = 2.182 ¥10–18 J
(6.022 ¥ 1023 mol-1 )
The energy of n = 1 orbit in hydrogen atom will be E1 = –2.182 ¥10–18 J

E1 2.182 ¥ 10-18 J
The energy of n = 3 orbit will be E3 = =- = –2.42 ¥ 10–19 J
32 9
E1 (He+ ) - 4 / 1
17. Since E μ – Z2/n2, we have = =4 fi E1(He+) = 4 E1(H)
E1 (H) -1 / 1
Hence, IE(He+) = 4 IE(H) = 4 ¥ 1314 kJ mol–1 = 5256 kJ mol–1
3.8 Complete Chemistry—JEE Main

SECTION 2 Quantum-Mechanical Model of Atom

The quantum-mechanical model of an atom came into existence after the discovery of wave-particle duality and
uncertainty principle.
Wave-Particle Duality
A subatomic particle can display wave nature as well as particle nature. This dual behaviour is inter-related by de-
Broglie relation, according to which, we have
h h
p= i.e. mu =
l l
where the momentum p ( = mu) represents the particle nature and the wavelength l represents the wave nature of the
subatomic particle.
Illustration de–Broglie wavelength of an electron travelling with a speed of 1% of the speed of light.
h (6.626 ¥ 10-34 J s)
l= = = 2.43 ¥ 10–10 m = 243 pm
mu (9.1 ¥ 10-31 kg )(3 ¥ 106 m s -1 )
(Note For particles of larger mass (say, 1 g), l
The uncertainty Principle
According to this principle postulated by Heisenberg, we have,
It is not possible to design an experiment with the help of which one can determine simultaneously the precise values
of both position and momentum of subatomic particles.
Mathematically, the uncertainty principle is expressed as
h
Dp Dx ≥
4p
where D p and D x are the root mean square uncertainties in momentum and position of the particle, respectively and h
is Planck’s constant.
Illustration Uncertainty in speed of an electron if the uncertainty in position is 5 pm.
h h
Dp Dx ≥ i.e. (m Du ) Dx ≥
4p 4p

h Ê 1 ˆ Ê 6.626 ¥ 10-34 J s ˆ Ê 1 ˆ
Hence Du ≥ = = 1.16¥107 m s–1
Á ˜
4p Ë m D x ¯
Á
Ë 4 (3.14) ¯ Ë (9.1 ¥ 10 kg)(5 ¥ 10 m) ˜¯
˜ Á -31 -12

Comment ¥ 106 m s–1. The uncertainty of


locating its position by 5 pm leads to much larger uncertainty in its speed. Thus, talking the precise position and speed
of electron in Bohr theory has no meaning.
(Note For particles of larger mass (say. 1 g), the uncertainties in position and velocity are very small and one can talk
about their precise position as well as its speed.)
Dual Nature of Radiation
Even the radiations have dual nature of wave as well as that of particle. This was established by Albert Einstein in 1905.
In wave nature, a radiation is characterised by its wavelength (l) or frequency (n) or wave number (n~). These are
inter-related through the expressions
c 1
n= and n~ =
l l
where c is the speed of light.
In particle nature, a radiation is considered to be a stream of photons, each carrying energy equal to hn, i.e.
E = hn = hc/l
Atomic Structure 3.9

Photoelectric Effect
The photoelectric effect (discovered by Heinrich Hertz) involves the
instantaneous emission of electrons when a clean metal plate in vacuum is
exposed to a beam of ultraviolet radiation (Fig. 2)
Main Observations of the Experiment
1. Electrons are emitted only when the plate is irradiated with radiation
of frequency equal to or greater than some minimum frequency, known
as threshold frequency.
2. The number of electrons emitted is proportional to the intensity of
incident radiation.
3. The kinetic energy of emitted electrons depends on the frequency of
the incident radiation and it increases linearly with increase in the Fig. 2
frequency of incident radiation (Fig. 3)
Explanation of Photoelectric Effect When a photon of incident radiation
collides with the electron of metal, the electron acquires energy equal to the
energy of the photon. Thus, the emitted electron carries energy as given by the
expression
1
hn = KE + PE= mu2 + IE
2
where KE, PE and IE stand for kinetic energy, potential energy and ionization
energy, respectively. The latter is
IE = hn0
where n0 is known as the threshold frequency of the metal. Hence Fig. 3
1
KE = mu2 = h(n – n0)
2
that is, the kinetic energy of emitted electrons varies linearly with the frequency of the incident radiation (Fig. 3). The
slope of linear plot is Planck’s constant.
Stopping Potential In Fig. 2, if the potential applied to the electrodes is increased, a stage is reached when no
electron reaches to the negative electrode. At this stage, kinetic energy of electrons becomes zero and is thus given by
the expression
KE = |eV0|
where V0 is known as stopping potential.
Illustration The velocity and stopping potential of electrons when a metal (threshold frequency = 1.5 ¥1016 s–1) is
exposed to the radiation of wavelength 15 nm.
c 3 ¥ 108 m s -1
n= = = 2.0 ¥ 106 m s–1
l 15 ¥ 10-9 m
1
mu 2 = h(n – n0) = (6.626 ¥ 10–34 J s) (0.5 ¥ 1016 s–1) = 3.3 ¥ 10–18 J
2
Ê 2 ¥ 3.3 ¥ 10-18 J ˆ KE Ê 3.3 ¥ 10-18 J ˆ
u= Á = 2.7 ¥ 106 m s–1 =Á
Ë 9.1 ¥ 10-31 kg ˜¯
and V0 = = 20.63 V
e Ë 1.60 ¥ 10-19 C ˜¯
Quantum-Mechanical Behaviour of Electron in an Atom

in terms of probable distribution based on the wave nature of the electron.


The wave nature of the electron is described by wave function which are the solutions of Schrödinger equation.
3.10 Complete Chemistry—JEE Main

Ê ∂2 ∂2 ∂2 ˆ 8p 2 me
+ +
ÁË ∂x 2 ∂y 2 ∂z 2 ˜¯ Y + (E - V ) = 0
h2
where V is the potential energy of the electron in hydrogen-like species and is
given as
Ze2
V = ( 4p e ) r
0
The Schrödinger equation can be solved precisely for hydrogen-like species
For this, the Schrödinger equation is expressed in spherical polar system of
coordinates (Fig. 4) and its solutions have the form
Yr,q,j = Rr Qq Fj Fig. 4
where R, Q and F are r -, q - and j - dependent mathematical expressions, respectively.
Solution of r-Dependent Equation The r - dependent equation provides the variations in the function R with the
distance r of electron from the nucleus. The solution of R provides more than one r-dependent behaviour involving two
constants n and l. These are the quantum numbers and have the following values.
n 1, 2, 3, . . .
l 0, 1, 2, . . ., (n – 1) ;(a total n values)
A few expressions of Rn,l are as follows.

1/ 2 3/ 2
ÊZˆ Ê Z ˆ Ê Zr ˆ
R1, 0 = 2 Á ˜
Ë a0 ¯
exp(–Zr/a0) R2, 0 = 2 Á
Ë 2a0 ˜¯ ÁË 2 - a ˜¯ exp(–Zr/2a0)
0

3/ 2
1 Ê Z ˆ Ê Zr ˆ
R2, 1 = Á ˜ ÁË a ˜¯ exp(–Zr/2a0)
3 Ë 2a0 ¯ 0

Solution of q - Dependent Equation The q - dependent equation provides the variation in the function Q with the
angle q. The solution of Q provides more than one q-dependent solutions involving two constants l and |m|, where the
quantum number m takes the value
m 0, ±1, ±2, . . ., ±l ;(a total of 2l +1 values)
A few expression of Ql, |m| are as follows.
Q0,0 = 1 / 2 Q1,0 = ( 6 / 2) cos q Q1,±1 = ( 3 / 2)sin q
Solution of j-Dependent Equation The j-dependent equation provides variation in the function F with the angle
j. The solution of F provides more than one j-dependent solutions involving the constant m.
A few expressions of m are as follows:
F0 = 1 / 2 p F+1 = (1 / 2p ) exp(ij ) F–1 = (1 / 2p ) exp(-ij )
Orbitals and Quantum Numbers The solution of Schrödinger equation may be represented as
Yn, l, m(r, q, j) = Rn, l (r) Ql, |m|(q)Fm(j)
where the constants n, l and m are known as quantum numbers. Their values, as described above, are:
n 1, 2, 3, . . .
l 0, 1, 2, . . .,(n – 1) ;(a total of n values)
m 0, ±1, ±2, . . ., ± l ;(a total of 2l + 1 values)
Note The maximum value of l is limited by n and that of m by l.
Each solution of Schrödinger equation represents an orbital of an atom.

Quantum Number n In hydrogen-like species the energy of electron in an atom is governed by the quantum number
n and is known as principal quantum number. The energy expression is
Atomic Structure 3.11

1 Ê 2p 2 m( Ze2 / 4p e 0 )2 ˆ
E= -
n 2 ÁË h2 ˜¯

Note This expression is the same as that obtained in Bohr theory.


The different energy shells are designated by the symbols K, L, M, . . . depending upon the value of n equal to 1, 2,
3, . . ., respectively.
Quantum Number l The quantum number l, known as azimuthal or subsidiary quantum number, characterises the
angular momentum of the electron by the expression
Ê hˆ
L = l (l + 1) Á ˜ ;(h is Planck’s constant)
Ë 2p ¯
The value of l is designated by the symbols s, p, d, f, g, . . ., depending upon the values of l equal to 0, 1, 2, 3, 4, . . .,
respectively.
Quantum Number m The quantum number m, known as magnetic quantum number, characterises the z-component
of angular momentum of the electron by the expression

Lz = m ÊÁ ˆ˜
h
Ë 2p ¯
The z
The relation between l and m is

m ÊÁ ˆ˜ = ÊÁ l (l + 1) ˆ˜ cos q
h h
Lz = L cos q i.e
Ë 2p ¯ Ë 2p ¯
This gives m= l (l + 1) cos q

Designation of an Orbital An orbital is designated by writing the value of principal quantum number n followed by
the symbol prescribed to the azimuthal quantum number l. The value of m is added as the subscript to the symbol of l.

Illustrations

n=1 l=0 m=0 1s 1 orbital


n=2 l=0 m=0 2s
l=1 m = +1 2p+1
4 orbitals
m=0 2p0
m = –1 2p–1
In general, the number of orbitals in the give value of principal quantum number n is n2. All these orbitals are
degenerate, that is, they have same energy.
Spin Quantum Number An electron in an orbital spin around its own axis. This spinning produces angular
momentum whose value is given by the expression

s ( s + 1) ÊÁ ˆ˜
h
L=
Ë 2p ¯
where s is known as spin quantum number and has a value of 1/2.
The z-component of this angular momentum is given by
Ê hˆ
Lz = ms ÁË 2p ˜¯
where ms, known as magnetic spin quantum number, is either +1/2 or –1/2.
The electron having ms = 1/2 is labelled as a-spin and is represented by a vertical arrow pointing upward i.e. by (≠).
The electron having ms = –1/2 is labelled as b-spin and is represented by a vertical arrow pointing downward i.e. by
(Ø).
3.12 Complete Chemistry—JEE Main

Probable Distribution of Electron Around the Nucleus


Y 2n, l, m(=R2n , l Ql2, |m|
Fm2 ) evaluated at the given point.
Probable Distribution of s-orbitals For s-orbitals, Y 2n,0,0 μ R2n,0 as the angular dependence has a constant value.
1s orbital: The plots of R1,0 versus r and R21, 0 versus r are shown in Figs 5a and 5b.

Fig. 5a Fig. 5b
In Figs 5a and 5b, the symbol a0 represents Bohr radius.
Dot-Population Picture In dot-population, the value of relative
probability at a given location is shown by the density of dots near that
location. This picture gives the most realistic description of the electron’s
time average distribution in the atom.
Equal-Probability Contours In equal-probability contours, the points
having identical probability are joined. The probability contour enclosing
90 to 95% probability is known as shape of orbital.
Figures 5c and 5d display dot-population picture and shape of orbital of Fig. 5c Fig. 5d
1s orbital.
Radial Distribution Function
nucleus, the total amount of the dot population near the nucleus is very small owing to the small volume near the nucleus.
In order to visualise the total amount of the dot population within a spherical
shell placed at radii r and r + dr from the nucleus, the radial distribution function

F = (volume of the spherical shell) (probability density)


= (4 p r2dr) (R2)
Thus, the radial distribution functi
the electron in a spherical shell of thickness dr located at the distance r from
the nucleus.
Radial Function of 1s Orbital For 1s orbital, radial distribution function
is shown in Fig. 5e. A maximum is observed at a0 which is exactly the same as
that of Bohr radius.
2s orbital: For 2s orbital, the plots of R2,0, R22,0 and 4pr2R22,0 versus r are
shown in Figs 6a, 6b and 6c. Fig. 5e
Atomic Structure 3.13

Fig. 6a Fig. 6b Fig. 6c

Nodal Point The point where the function R has a zero value is known as spherical nodal point. R2,0 has one nodal
point at r = 2a0.
In general, the spherical nodal points in Rn, l function is equal fo (n – l – 1)
Maxima in Radial Distribution Function In the plot of 4pr2R22, 0 versur r, there are two maxima at about 0.04 nm
and 0.3 nm. The maxima at 0.04 nm is smaller than that at 0.3 nm. In general, the number of maxima in the radial
distribution plot of s orbital is equal to the principal quantum number n. Of these, the maximum value is observed
at the largest value of r.†
Dot-Population Picture and Equal-Probability contour
For 2s orbitals, these are shown in Figs. 6d and 6e.

Probable Distribution of 2p Orbitals


r-Dependence of 2p orbitals For 2p orbital, n = 2 and l
= 1. Since r-dependent function Rn, l depends only on n and l,
the same function will hold good for all the three values of m,
namely, +1, 0 and –1.
2 Fig. 6d Fig. 6e
Figures 7a, 7b and 7c show the plots of R2,1, R2, 1 and
2R2 versur r, respectively.
4pr 2, 1

Fig. 7a Fig. 7b Fig. 7c

† With the increase in the value of n, the largest maximum lies farther away from the nucleus. This implies that the size of orbital increases
with increase in value of the principal quantum number n.
3.14 Complete Chemistry—JEE Main

Notable features of these plots are:


There is no nodal points. This is in consistent with the general formula of n – l –1.
R2, 1 has a zero value at r = 0 in contrast to R2, 0 which has a maximum value at r = 0.
There is one maximum in the plot of 4pr2R2, 2 at r = 4a
1 0
d(r2R2.1
2 )/dr equal to zero.

We have R2,1 μ r e-r / 2 a0


d 2 2 d 4 - r / a0 Ê r4 ˆ
Hence (r R2, 1 ) = (r e ) = Á 4r 3 - ˜ e - r / a 0 = 0
dr dr Ë a0 ¯
This gives r = 4a0
Angular Dependence of 2p orbitals The angular dependence of an orbital is governed by the functions Q and F.
For 2p orbitals, we have
6 1
Q1, 0 = cosq F0 =
2 2p

3 1 1
Q1, | ±1 | = sinq F+1 = e ij and F-1 = e - ij
2 2p 2p
Plot of Q1,0F0 or 2pz orbital There is no j
dependence as F0 does not involve angle j. Two-
dimensional plot of the function Q1,0F0 versus q gives
two circles in xz-plane (or yz-plane), one just above
the xy-nodal plane and the other just below it (Fig.
8a).
The shape of angular probability distribution is
given by the plot of (Q1,0F0 )2 versus q. This is shown
in Fig. 8b.
Note The number of nodal surfaces in the
angular distribution function is equal to the value
of quantum number l.
Shape of 2p 0 orbital The shape (i.e. 90%
probability contour diagram) is obtained by combining
the r-dependence and angular dependence functions. Fig. 8a Fig. 8b
Figure 8c displays the shape of 2p0 orbital. It consists of two spheroidal lobes pointing in z-direction. Thus, 2p0 orbital
is also known as 2pz orbital. The nucleus is situated between the two lobes and lie in the xy-nodal plane.

Plots of 2px and 2py orbitals Since the function F+1 and F–1 carry
the imaginary quantity i = -1 , these are converted into real functions
by taking the following linear combinations.
1 1 1 1
Fx = ( F+1 + F-1 ) = (eij + e-ij ) = cos j
2 2 2p p
-i -i 1 1
Fy = ( F+1 - F-1 ) = (eij - e-ij ) = sin j
2 2 2p p
When the functions are Fx and Fy are separately combined with Q1, |±1|
function and plotted against q j equal to 0 and p,
respectively), one gets the shapes similar to that shown in Fig. 8a, with
the directions along x– and y– axes, respectively. Fig. 8c
Atomic Structure 3.15

Shpes of 2px and 2py The shapes of 2px and 2py orbitals are of the appearance as shown in Fig. 8c, the two lobes
point along x– and y– axis, respectively. (Figs 8d and 8e)

Fig. 8d 2px orbital Fig. 8e 2py orbital

Shapes of 3d orbitals
Proceeding similarly as in the case of 2p orbitals, one can draw the shapes of 3d orbitals. These are shown in Figs 9
(a-e). These include 2(=l

axes (3dz2 and 3dx2 – y2 ), and (ii) orbitals having maximum probability distribution in between the two axes (3dxy, dxz
and 3dyz).

Fig. 9a 3dz2 Fig. 9b 3dxy

Fig. 9c 3dyz Fig. 9d 3dxz


3.16 Complete Chemistry—JEE Main

Fig. 9e 3dx2 – y2
Summary of Number and Types of Orbitals in an Atom
In an atom, we have
1. Various quantum shells are governed by the principal quantum number, n, which can take values of 1,2,3, . . .,
These shells are designated as K,L,M, . . ., and so on.
2. Each quantum shell consists of sub-shells which are characterized by the azimuthal quantum number, l. The
number of sub-shells is equal to the value of n as l can take values of 0, 1, . . ., (n – 1). These sub-shells are
designated as s, p, d, f, g, . . ., and so on.
3. Each sub-shells consists of orbitals which are characterized by the magnetic quantum number m. The number
of orbitals is equal to 2l +1, since m can take values of +l through zero up to –l, i.e. +l, + (l – 1), . . ., 0. . .,
–( l – 1), –l.
4. The total number of orbitals in a quantum shell is equal to n2.
Illustration Permitted and not permitted orbitals in an atom.
(i) n = 0; l = 0; m = 0 Not permitted as n cannot be equal to zero
(ii) n = 2; l = 1, m = +1 Permissible
(iii) n = 1; l = 1, m = 0 Not permitted as l cannot be equal to n.
(iv) n = 1; l = 0, m = –1 Not permitted as |m| cannot be greater that l.
(v) n = 2; l = 1, m = 1 Permissible
General Comments on Quantum Numbers
Quantum numbers n decides the size and energy of the orbital
Quantum numbers l decides the shape of the orbital
Quantum numbers m decides the orientation of the orbital
General Comment about the Orbitals in a Multi-electron Atom
Though the energy of orbitals in hydrogen-like species is decided by the quantum number n only, but in a multi-
electron atoms, it is decided by both the quantum numbers n and l. This is primarily due to electronic repulsions
in the atom. In general
The energy of an orbital increases with increase in the value of n + l. For the two orbitals with the same value of
n + l, the orbital having higher value of n has the higher energy.
The angular orientation of orbitals in the multi-electron atoms are the same as those in hydrogen-like species. Thus, the
scheme of designation of orbitals remains the same, for example 1s, 2s, 2px, 2py, 2pz, 3s, 3px...etc.

1. Aufbau Principle The word aufbau is a German word which means ‘building up’. The principle is:

The two rules to predict the relative energies of various orbitals as mentioned earlier are:
(i) Energy increases with increase in the value of n + l.
(ii) For the same value of n + l, the lower value of n has lower energy.
Guided by these, the order of energies of orbitals are shown in Fig. 10
Atomic Structure 3.17

Fig. 10
2. Pauli Exclusion Principle The principle is:
No two electrons in an atom have the same values for all the four quantum numbers n, l, m and ms.
According to this principle, only two electrons can be accommodated in an orbital since for the same orbital, the
quantum numbers, n, l, and m have the same value and thus ms will differ having the values of +1/2 and –1/2, respectively.
3. Hund’s Rule: The rule is:
Electrons with the same spin enter degenerate orbitals (which have identical energies) one by one till all of them are singly
occupied. This is followed by pairing of electrons with opposite spins.
Illustration
Filling of 2p orbitals

Fig. 11
Explanation By occupying different orbitals with the same spin, electron-electron repulsive interactions is minimised
because the magnetic dipoles generated by the spinning of electrons will have like poles farther away from each other
causing lesser repulsion between them.
The exception is

Illustrations:

24Cr (1s)2 (2s)2 (2p)6 (3s)2 (3p)6 (4s)2 (3d)4 24Cr (1s)2 (2s)2 (2p)6 (3s)2 (3p)6 (4s)1 (3d)5
29Cu (1s)2 (2s)2 (2p)6 (3s)2 (3p)6 (4s)2 (3d)9 29Cu (1s)2 (2s)2 (2p)6 (3s)2 (3p)6 (4s)1 (3d)10
Note:

each other and their order of energies is reversed once these are occupied by electrons.
3.18 Complete Chemistry—JEE Main

MULTIPLE-CHOICE QUESTIONS ON SECTION 2

1. If the kinetic energy of a sub-atomic particle is increased 8 times, its de-Broglie wavelength becomes x times the
original wavelength. The value of x is
(a) 1/8 (b) 1/4 (c) 1/ 2 2 (d) 1/ 2
2. The stopping potential of the electrons emitted in a photoelectric experiment is V. The de-Broglie wavelength of
the electron when it is emitted from the metal surface will be
(a) h 2 / 2emV (b) h / 2emV (c) h 2 / 2emV (d) h / 2emV
3. The de-Broglie wavelength of the particle A is four times the wavelength of the particle B. If mass of A is two
times the mass of B, then the ratio of kinetic energy of A and that of B will be
(a) 1/16 (b) 1/8 (c) 1/32 (d) 1/ 32
4. The de-Broglie wavelength of electron in Bohr orbits displays the following trend with increase in the value of
quantum number n.
(a) Increase (b) Decrease (c) No change (d) Cannot be predicted
5. The de-Broglie wavelength of a proton travelling with a speed of 0.1% of the speed of light will be
(a) 1.32 pm (b) 1.32 nm (c) 2.62 pm (d) 5.14 nm
6. If the uncertainty in locating the position of a proton (m = 1.67 ¥ 10 –27 kg) is 5 mm, the uncertainty in its speed

will be
(a) ≥ 9.3 ¥ 10–3 m s–1 (b) ≥ 6.3 ¥ 10–3 m s–1 (c) ≥ 6.3 ¥ 10–2 m s–1 (d) £ 6.3 ¥ 10–2 m s–1
7. The kinetic energy of electrons in a photoelectric effect is 1.3252 ¥ 10–19 J. If the threshold frequency of the metal
is 5.50 ¥ 1014 Hz, the frequency of incident radiation is
(a) 4.50 ¥ 1014 Hz (b) 6.50 ¥ 1014 Hz (c) 7.50 ¥ 1014 Hz (d) 8.50 ¥ 1014 Hz
8. A metal (threshold frequency 8.0 ¥ 10 15 Hz) is irradiated with the radiation of frequency 10.40 ¥ 1015 Hz. The

stopping potential of the emitted electrons is about


(a) 67.6 V (b) 82.1 V (c) 120.1 V (d) 99.4 V
9. The number of spherical nodes in an orbital of hydrogen-like species is
(a) n – l (b) n – l – 1 (c) n + l – 1 (d) n – l – 2
10. The number of angular nodes in an orbital of hydrogen-like species is
(a) n (b) l (c) n – l (d) n + l
11. The total number of orbitals in M shell of an atom is
(a) 1 (b) 4 (c) 9 (d) 16
12. Which of the following orbitals is not permitted in an atom?
(a) n = 2, l = 0, m = 0 (b) n = 2, l = 1, m = 0 (c) n = 2, l = 0, m = 1 (d) n = 3, l = 2, m = 1
13. If q is the angle made by orbital angular momentum with the z-axis, then the quantum number l and m of an atom
are related to each other through the expression
(a) m = l cos q (b) m ={l(l + 1)}cos q (c) l = m cos q (d) m = l (l + 1) cos q
14. The total number of nodes in an orbital is equal to the value of
(a) n (b) n – 1 (c) n – 2 (d) n + l – 1
15. In a sub-shell of an atom, the total number of allowed orbitals is equal to the value of
(a) l (b) l + 1 (c) l + 2 (d) 2 l + 1
16. Which of the following species travelling with the same speed will have maximum de-Broglie wavelength?
(a) H (b) D (c) T (d) a–particle
17. Consider the ground state of Cu atom (Z = 29). The number of electrons with the azimuthal quantum numbers l
= 1 and 2 are, respectively,
(a) 12, 5 (b) 12, 10 (c) 16, 6 (d) 14, 8
Atomic Structure 3.19

same energy?
(i) n = 2, l = 1, m = 0 (ii) n = 3, l = 1, m = 1 (iii) n = 4, l = 0, m = 0 (iv) n = 3, l = 1, m = 0
(a) (i) and (ii) (b) (i) and (iii) (c) (ii) and (iii) (d) (ii) and (iv)
19. Which of the following arrangement of orbitals having quantum numbers
(i) n = 4, l = 2 (ii) n = 5, l = 1 (iii) n = 4, l = 3
is correct regarding their relative energies?
(a) (i) < (ii) < (iii) (b) (i) < (iii) < (ii) (c) (ii) < (i) < (iii) (d) (ii) < (iii) < (i)
24Cr is
(a) [Ar] (3d)5 (4s)1 (b) [Ar] (3d)4 (4s)2 (c) [Ar] (3d)6 (4s)0 (d) [Ar] (4d)5 (4s)1
47Ag is
(a) 3d (b) 4d (c) 5d (d) 6d
22. Which of the following statements is correct?

(b) The electron densities in the xy and yz planes are zero in 3dxz orbital.
(c) The electron density in the xy plane in 3dz2 orbital is zero.
(d) The electron density in the xy plane in 3dxy orbital is zero.
23. The number of unpaired electrons in 27Co is
(a) 2 (b) 3 (c) 4 (d) 5
24. The numbers of spherical and angular nodes in 4f orbitals, respectively, are
(a) 1, 3 (b) 1, 4 (c) 2, 3 (d) 0, 3
25. Which of the following sets represents isoelectronic species?
(a) K+, Ca2+, Sc3+ (b) Na+, F–, V3+ (c) K+, Cl–, Mg2+ (d) Cr3+, Fe2+, Co3+

ANSWERS
1. (c) 2. (b) 3. (c) 4. (b) 5. (a) 6. (b)
7. (c) 8. (d) 9. (b) 10. (b) 11. (c) 12. (c)
13. (d) 14. (b) 15. (d) 16. (a) 17. (b) 18. (d)
19. (a) 20. (a) 21. (b) 22. (b) 23. (b) 24. (d)
25. (a)

HINTS AND SOLUTIONS


1. de-Broglie relation is mu = h/l. The kinetic energy becomes

1 h2 T2 l1
2
8T1 l12 2 l12 l
T= mu 2 = Hence, = fi = fi l 2 = fi l2 = 1
2 2 ml 2 T1 l2 2
T 1 l2
2
8 2 2
2. The kinetic energy of the electron emitted from the metal surface will be
1 p2 h
fi l=
h
mu 2 = eV fi = eV fi p 2 = 2 m eV Hence, 2 m eV =
l
2 2m 2 m eV
TA mB lB2 Ê 1 ˆ Ê 1 ˆ 2 1
3. The ratio will be = =Á ˜Á ˜ =
TB mA lA2 Ë 2 ¯ Ë 4 ¯ 32
4. Since in Bohr theory p = n(h/2p), increase in p with increase in the value of n will be associated with decrease in
de-Broglie wavelength. In fact, the expression of wavelength is l = 2p/n .
3.20 Complete Chemistry—JEE Main

h (6.626 ¥ 10-34 J s)
5. l = = = 1.32 ¥ 10–12 m = 1.32 pm
mu (1.672 ¥ 10-27 kg)(3 ¥ 105 m s -1 )

h Ê 1 ˆ (6.626 ¥ 10-34 J s)
6. Du ≥ = = 6.3 ¥ 10–3 m s–1
4p ÁË mp Dx ˜¯ (4)(3.14)(1.67 ¥ 10-27 kg)(5 ¥ 10-6 m)

7. Since KE = hn – hn0, we have


KE (1.3252 ¥ 10-19 J )
n= + n0 = + 5.50 ¥ 1014 s -1 = 2.0 ¥ 1014 s–1 + 5.50 ¥ 1014 s–1 = 7.50 ¥ 1014 s–1
h (6.626 ¥ 10-34 J s)
8. |eV| = hn – hn0
h(v - n 0 ) (6.626 ¥ 10-34 J s)(2.40 ¥ 1016 s -1 )
V= = = 99.4 V
e (1.6 ¥ 10-19 C)
9. The number of spherical nodes (i.e. nodes in the plot of Rn, l versus r) is n – l – 1.
10. The number of angular node is equal to the value of l (azimuthal quantum number).
11. For M shell of an atom, n = 3. The total number of orbital in a shell is equal to the value of n2.
12. The value of m cannot be greater than the value of l.
13. L = l (l + 1) (h/2p), Lz = m(h/2p) and Lz =L cos q. Hence, m= l (l + 1) cos q
14. Total number of nodes = (spherical + angular) nodes = n – l – 1 + l = n – 1
15. The total number of orbitals in a sub-shell is equal to the number of allowed values of magnetic quantum number.
16. de-Broglie wave length is inversely proportional to the mass of the species travelling with the same speed. (l =
h/mu)
2 2 6 2 6 10 1
29Cu is (1s) (2s) (2p) (3s) (3p) (3d) (4s)
l =1 corresponds to p orbital. The number of electrons in p orbitals is 6 + 6 = 12
l =2 corresponds to d orbital. The number of electrons in d orbitals is 10
18. Orbitals having the same value of n + l will have the same energy irrespective of the value of m.
19. Larger the value of n + l, larger the energy. For the same value of n + l, larger the value of n, larger the energy.
5 1
24Cr is [Ar] (3d) (4s)
atom.
10(5s)1.
47
22. 3dxz involves xy and yz nodal planes.
7 2
27Co is [Ar] (3d) (4s) . Thus, it contains 3 unpaired electrons in 3d orbitals

24. Spherical nodes = n – l – 1 = 4 – 3 –1 = 0 Angular nodes = l = 3


25. K , Ca , Sc all contain 18 electrons. The electrons in rest of species are: Na+(10), F–(10), V3+(20), Cl–(18),
+ + 3+

Mg2+(10), Cr5+(21), Fe+(24) and Co3+(24).


Atomic Structure 3.21

MULTIPLE CHOICE QUESTIONS FOR THE ENTIRE CHAPTER

General Characteristics
1. The approximate radii of the nuclei of atoms lie in the range of
(a) 10 12 m 10 13 m (b) 10 13 m 10 14 m (c) 10 14 m 10 15 m (d) 10 15 m 10 16 m
2. The increasing order for the values of e/m (charge/mass) for electron (e), proton (p), neutron (n) and alpha particle
(a) is
(a) e, p, n, a (b) n, p, e, a (c) n, p, a, e (d) n, a, p, e
3. The fundamental particle responsible for keeping the components of nucleus together is
(a) meson (b) neutron (c) positron (d) antiproton
4. An isotone of 7632 Ge is
(a) 77
32 Ge (b) 77
33As (c) 77
34Se (d) 76
34Se
5. The number of neutrons in dipositive zinc ion (Z = 30) with mass number 70 is
(a) 34 (b) 36 (c) 38 (d) 40
6. The mass of a neutron is
(a) equal to the mass of a proton (b) greater than the mass of a proton
(c) smaller than the mass of a proton (d) equal to the sum of masses of a proton and an electron
7. The mass of one mole of electrons is
(a) 0.548 g (b) 0.548 mg (c) 0.548 cg (d) 0.548 dg
8. The number of electrons carrying a total charge of 1 C is
(a) 6.023 ¥ 1023 (b) 6.25 ¥ 1018 (c) 6.24 ¥ 1020 (d) 6.023 ¥ 1022
9. The binding energy of 3Li atom is 6.258 ¥ 10
7 –12 J. If atomic masses of electron, proton and neutron are
0.0005 u, 1.0073 u and 1.0086 u, respectively, the actual mass of Li atom is
(a) 6.899 u (b) 6.985 u (c) 7.016 u (d) 7.312 u
10. Naturally occurring copper is assigned atomic mass equal to 63.546 u. If it consists of two isotopes 63Cu (atomic
mass = 62.930 u) and 65Cu (atomic mass = 64.928 u), then mass percentage of isotope 65Cu in the naturally
occurring copper is
(a) 30.83 (b) 41.15 (c) 56.2 (d) 59.1
Bohr Theory
11. The expression for the Bohr radius of hydrogen-like species is
È h2 ˘ 1 È h2 ˘
(a) n2 Í 2 ˙ (b) Í 2 ˙
(
ÍÎ 4p m Ze 4p e 0 ˙˚
2
) n2 (
ÍÎ 4p m Ze 4pe 0 ˙˚
2
)
È 4p 2 m h 2 ˘ 1 È 4p 2 m h 2 ˘
(c) n2 Í 2 ˙ (d) 2 Í 2 ˙
Î Ze 4 p e 0 ˚ n Î Ze 4 pe 0 ˚
12. The expression for the energy of electron in the hydrogen-like species is
È 2 ( ) ˘ È 2 ( ) ˘
2 2
1 Í 2p m Ze 4pe 0 1 Í 2p m Ze 4pe 0
2 2
(a) 2 ˙ (b) - 2 ˙
n Í h2 ˙ n Í h2 ˙
Î ˚ Î ˚
È ˘ È ˘
1 Í 2p 2 m h 2 ˙ 1 Í 2p 2 m h 2 ˙
(c) (d) -
( ) ( )
2˙ 2˙
n 2 Í Ze2 4 pe n 2 Í Ze2 4 pe
Î 0 ˚ Î 0 ˚
3.22 Complete Chemistry—JEE Main

13. The expression of Rydberg constant is


2p 2 m (e 2 4p e 0 ) (e 2 4p e 0 )
2 2
2p 2 m h3 c 2 p 2 m h3 c
(a) (b) (c) (d)
(e 2 4p e 0 ) 2p 2 m h3 c e 2 4p e 0
2
h3 c

14. Which of the following atoms has the largest atomic radius?
(a) 3Li (b) 5B (c) 7N (d) 8F
15. The speed of an electron in the innermost orbit of the hydrogen atom (Bohr radius = 52.9 pm,
me = 9.11 10–31 kg) is
(a) 2.19 104 m s–1 (b) 2.19 106 m s–1 (c) 2.19 107 m s–1 (d) 2.19 108 m s–1
16. The ionization energy of hydrogen atom in the ground state is 2.17 10–11 erg. The ionization energy of Li2+ in
the ground state will be
(a) 1.953 10–15 J (b) 1.953 10–16 J (c) 1.953 10–17 J (d) 1.953 10–18 J
17.
(a) 52.9 cm (b) 52.9 nm (c) 52.9 pm (d) 52.9 Å
18. With increasing quantum number, the energy difference between adjacent levels of hydrogen atom
(a) increases (b) decreases

19. The wavelength of radiation required to remove the electron of hydrogen atom (ionization energy 21.7 10–12
erg) from n = 2 orbit to n = • is
(a) 3.664 10–4 cm (b) 3.664 10–5 cm (c) 3.664 10–6 cm (d) 3.664 10–7 cm
20. The angular momentum of an electron in the second Bohr orbit is given by
(a) 1(h/2p) (b) 2 (h/2p) (c) 2(h/2p) (d) 2(2 + 1) (h/2p)
21. The energy difference between the two orbits of a hydrogen atom is
Ê 1 1ˆ
DE = R•hc Á 2 - 2 ˜
Ë n1 n2 ¯
For the Lyman series, the values of n1 and n2 are given as
(a) n1 = 1, n2 = 2, 3, 4, … (b) n1 = 2, n2 = 3, 4, 5, …
(c) n1 = 3, n2 = 4, 5, 6, … (d) n1 = 4, n2 = 5, 6, 7, …
22. For the Balmer series, the values of n1 and n2 in the expression
Ê 1 1ˆ
DE = R•hc Á 2 - 2 ˜
Ë n1 n2 ¯
are
(a) n1 = 1, n2 = 2, 3, 4, … (b) n1 = 2, n2 = 3, 4, 5, …
(c) n1 = 3, n2 = 4, 5, 6, … (d) n1 = 4, n2 = 5, 6, 7, …
23. For the Paschen series, the values of n1 and n2 in the expression
Ê 1 1ˆ
DE = R•hc Á 2 - 2 ˜
Ë n1 n2 ¯
are
(a) n1 = 1, n2 = 2, 3, 4, … (b) n1 = 2, n2 = 3, 4, 5, …
(c) n1 = 3, n2 = 4, 5, 6, … (d) n1 = 4, n2 = 5, 6, …
24. For the Brackett series, the values of n1 and n2 in the expression
Ê 1 1ˆ
DE = R•hc Á 2 - 2 ˜
Ë n1 n2 ¯
are
(a) n1 = 1, n2 = 2, 3, 4, … (b) n1 = 2, n2 = 3, 4, 5, …
(c) n1 = 3, n2 = 4, 5, 6, … (d) n1 = 4, n2 = 5, 6, 7, …
Atomic Structure 3.23

25. For the Pfund series, the values of n1 and n2 in the expression
Ê 1 1ˆ
DE = R•hc Á 2 - 2 ˜
Ë n1 n2 ¯
are
(a) n1 = 2, n2 = 3, 4, 5, … (b) n1 = 3, n2 = 4, 5, 6, …
(c) n1 = 4, n2 = 5, 6, 7, … (d) n1 = 5, n2 = 6, 7, …
26. Which of the following statements is not true?
(a) Lyman spectral series of hydrogen atom lies in the ultraviolet region of electromagnetic radiation
(b) Balmer spectral series of hydrogen atom lies in the visible region of electromagnetic radiation
(c) Pashen spectral series of hydrogen atom lies in the visible region of electromagnetic radiation
(d) Brackett spectral series of hydrogen atom lies in the infrared region of electromagnetic radiation
27. 106 m s–1. Its velocity in the second
orbit would be
(a) 1.10 106 m s–1 (b) 4.38 106 m s–1 (c) 5.5 105 m s–1 (d) 8.76 106 m s–1
28. 10–18 J. Its energy in the second
orbit would be
(a) – 1.09 10–18 J (b) – 4.36 10–18 J (c) – 5.45 10–19 J (d) – 8.72 10–18 J
29. Hydrogen atom in its ground state is excited by a radiation of wavelength 97.26 nm. The longest wavelength it
emits is
(a) 1250 nm (b) 1400 nm (c) 1875 nm (d) 2050 nm
30. The ground state electronic energy in Be3+ ion is
(a) 3.49 ¥ 10–14 J (b) 3.49 ¥ 10–15 J (c) 3.49 ¥ 10–16 J (d) 3.49 ¥ 10–17 J
31. In Bohr theory, the ratio of kinetic energy and potential energy of an electron in any of orbit of hydrogen atom is
(a) 1/2 (b) – 1/2 (c) 1/4 (d) – 1/4
32. Which of the following statements about Bohr theory of hydrogen atom is not correct?
(a) Centripetal force acting on the electron is Ze2/(4p e0 r).
(b) Centrifugal force acting on the electron is mv2/r
(c) Angular momentum of electron is n (h/2p)
(d) Bohr radius is equal to 52.9 pm.
33. The expression of speed of electron in Bohr radii is given by
n ( h / 2p ) 2p ( Ze2 / 4p e 0 )
(a) v = (b) v =
Ze2 /(4pe 0 ) nh
n 2 ( h / 2p ) 4p 2 ( Ze2 / 42p e 0 )
(c) v = (d) v =
Ze2 /(4pe 0 ) n2 h2
34. Which of the following statements regarding Bohr theory of hydrogen atom is not correct?
(a) Kinetic energy of an electron in an orbit is half of magnitude of its potential energy.
(b) Kinetic energy of an electron in an orbit is equal to the magnitude of its potential energy.
(c) Total energy of an electron is negative of its kinetic energy.
(d) Potential energy of an electron in an orbit is equal to – mv2.
35. The ionization energy of one mole of hydrogen atoms is about
(a) 1313 J (b) 1313 kJ (c) 1616 J (d) 1616 kJ
36. In Bohr theory, which of the following expression for the energy of an electron in Bohr orbit is correct?
(a) E μ Z/n2 (b) E μ – Z/n2 (c) E μ – n2/Z (d) E μ – Z2/n2
37. In Bohr theory, which of the following expression for the speed of an electron in Bohr orbit is correct?
(a) v μ Z/n (b) v μ n/Z (c) v μ Z2/n2 (d) v μ n2/Z2
3.24 Complete Chemistry—JEE Main

38. In Bohr theory, which of the following expression for the radii of Bohr orbits is correct?
(a) r μ n/Z (b) r μ n2/Z2 (c) r μ n2/Z (d) r μ Z2/n2
39. Which of the following spectral series of hydrogen atom lies in the ultraviolet region of electromagnetic radiation?
(a) Lyman (b) Balmer (c) Paschen (d) Brackett
40. Which of the following spectral series of hydrogen atom lies in the far infrared region of electromagnetic radiation?
(a) Lyman (b) Balmer (c) Paschen (d) Pfund
41. Which of the following spectral series of hydrogen atom lies in the visible region of electromagnetic radiation?
(a) Lyman (b) Balmer (c) Paschen (d) Brackett
+ will have the same wavelength as that
42. Which of the following electronic transitions in the line spectrum of He

(a) 4 ¨æ 2 (b) 2 ¨æ 1 (c) 3 ¨æ 1 (d) 4 ¨æ 1


43. The kinetic energy of an electron in He+ is maximum when it is present in the orbit having
(a) n = 1 (b) n = 2 (c) n = 3 (d) n = •
44. The longest wavelength in Balmer series is
(a) 656.5 nm (b) 770 nm (c) 360 nm (d) 450 nm

(a) Stark effect (b) Zeeman effect (c) Bohr effect (d) Anti-Zeeman effect
46. In Bohr theory of He+, the energy difference between two successive energy levels
(a) increases with increase in the value of n
(b) decreases with increase in the value of n
(c) remains constant with increase in the value of n
(d) initially increases followed by a decrease with increase in the value of n
2+ ion will be
r
(a) 3r (b) r/3 (c) 9r (d) r/9
2+ ion relative that of electron in the third Bohr orbit of H atom is

(a) 1 (b) 2 (c) 4 (d) 9


Quantum Numbers and Orbitals
49. The shape of an orbital is governed by the
(a) principal quantum number (b) azimuthal quantum number
(c) magnetic quantum number (d) spin quantum number
50. The orientation of an orbital in an atom is governed by the
(a) principal quantum number (b) azimuthal quantum number
(c) magnetic quantum number (d) spin quantum number
51. For a given value of principal quantum number n, the number of allowed values of azimuthal quantum number l,
is given by
(a) n 2 (b) n (c) n 1 (d) n2
52. For a given value of azimuthal quantum number l, the number of allowed values of magnetic quantum number m,
is given by
(a) l + 1 (b) l + 2 (c) 2l + 1 (d) 2l + 2
53. Which of the following sets of quantum numbers is not allowed?
(a) n = 2, l = 0, m = + 1 (b) n = 2, l = 1, m = + 1
(c) n = 2, l = 0, m = 0 (d) n = 2, l = 1, m = 1
54. The value of Rydberg constant is
(a) 1.09678 105 m–1 (b) 1.09678 106 m–1
7 –1
(c) 1.09678 10 m (d) 1.09678 108 m–1
Atomic Structure 3.25

55. Which of the following orbitals is symmetric about the y-axis?


(a) px (b) py (c) dyz (d) dxy

(a) n = 3 (b) l = 2 (c) m = 2 (d) s = 1/2


57. Which of the following quantum numbers n, l, m and ms, respectively, is not possible?
(a) 2, 1, 0, – 1/2 (b) 2, 2, 0, 1/2 (c) 3, 2, 1, 1/2 (d) 3, 2, 0, 1/2

58. How many unpaired electrons are present in Ni2+ (Z = 28)?


(a) 0 (b) 2 (c) 4 (d) 8
not theoretically possible?
(a) 2d5 (b) 3p6 (c) 4s1 (d) 4f12
60. The maximum number of electrons that can be accommodated in a quantum shell is equal to
(a) n (b) n2 (c) 2n2 (d) n(n + 1)
61. The number of unpaired electrons present in the ground state of Cr is
(a) 3 (b) 4 (c) 5 (d) 6
62. Which of the electrons is most tightly bound to the nucleus?
(a) 4s (b) 4p (c) 4d (d) 4f
63. Which of the following atoms would be expected to be most paramagnetic?
(a) 3Li (b) 4Be (c) 5B (d) 6C
64. Tick the correct statement on the aufbau principle?
(a) (n – 1)d subshell is always lower in energy than ns subshell
(b) (n – 1)d subshell always has energy more than ns subshell
(c) 5d is lower in energy than 4f
(d) 5f is lower in energy than 7s
Z = 58) is
10 (5s)2 (5p)6 (4f)3 (6s)1
(a) (4d) (b) (5d)10 (5s)2 (5p)6 (5d)1 (6s)2
10 2 6 3 2
(c) (4d) (5s) (5p) (5d) (6s) (d) (4d)10 (5s)2 (5p)6 (4f)2 (5d)0 (6s)2
66. Aufbau order is not violated in

67. Which of the following statements is not correct?

(c) Aufbau order is not obeyed in cases where energy difference between ns and (n – 1)d subshells is larger.

5
(4s)1 is
(a) 24 (b) 25 (c) 26 (d) 27
69. The number of electrons present in the l = 2 orbital of Cr is
(a) 3 (b) 4 (c) 5 (d) 6
70. Which of the isoelectronic species has more electrons than neutrons?
(a) O2– (b) Ne (c) Na+ (d) Mg2+
71. The number of electrons with l = 2 in an atom having atomic number 23 is
(a) 2 (b) 3 (c) 4 (d) 5
3.26 Complete Chemistry—JEE Main

72. The ion having 18 electrons in the outermost shell is


(a) Cu+ (Z = 29) (b) Al 3+ (Z = 13) (c) K+ (Z = 19) (d) As3+ (Z = 33)
73. The number of unpaired electrons present in an atom with atomic number 23 is
(a) 1 (b) 2 (c) 3 (d) 5
Quantum Mechanical Approach

(a) at the nucleus


(b) at the Bohr radius
(c) at a large distance from the nucleus

75. The number of nodal points in the plot of Rn, 0 versus r is equal to the value of
(a) n (b) l (c) n – l (d) n – (l + 1)
76. For 2s orbital, the nodal surface exists at the distance
(a) a0 from the nucleus (b) 1.5 a0 from the nucleus
(c) 2 a0
yz plane?
(a) px (b) py (c) pz (d) dyz
78. The number of radial nodes in the 3p probability density distribution is
(a) 0 (b) 1 (c) 2 (d) 3
79. The angular dependence of an orbital is decided by the
(a) principal quantum number only (b) azimuthal quantum number only
(c) magnetic quantum number only (d) both azimuthal and magnetic quantum numbers
80. Which of the following symbols correctly represents an orbitals in an atom?
(a) n, l, m = Rn l m (b) n, l, m = Rn, l l m (c) n, l, m = Rn, l l, m m (d) n, l, m = Rn, l l, |m| m
81. The angular momentum of an electron in an atomic orbital is governed by the
(a) principal quantum number (b) azimuthal quantum number
(c) magnetic quantum number (d) spin quantum number
82. The z-component of angular momentum of an electron in an atomic orbital is governed by the
(a) azimuthal quantum number (b) principal quantum number
(c) magnetic quantum number (d) spin quantum number
83. An orbital with l = 0 is
(a) symmetrical about the nucleus (b) symmetrical about the x-axis only
(c) symmetrical about the y-axis only (d) symmetrical about the z-axis only
84. The energy of an orbital in a multielectron atom depends on the
(a) principal quantum number only
(b) principal and azimuthal quantum numbers only
(c) principal, azimuthal and magnetic quantum numbers
(d) principal, azimuthal, magnetic and spin quantum numbers
85. For silver metal, threshold frequency for the emission of photoelectron is 1.13 1017 s–1. The kinetic energy of
electrons emitted when silver is irradiated with wavelength 1.5 nm is
(a) 5.76 10–17 J (b) 5.76 10–16 J (c) 5.76 10–15 J (d) 5.76 10–14 J
86. The binding energy of electrons in a metal is 193 kJ mol–1. The threshold frequency of the metal is
(a) 4.83 1010 Hz (b) 4.83 1012 Hz (c) 4.83 1014 Hz (d) 4.83 1016 Hz
87. The momentum of a particle having de Broglie wavelength of 0.1 nm is
(a) 6.626 10–21 kg m s–1 (b) 6.626 10–22 kg m s–1
(c) 6.626 10–23 kg m s–1 (d) 6.626 10–24 kg m s–1
Atomic Structure 3.27

88. An electron is accelerated through a potential difference of 500 V. Its de Broglie wavelength would be
(a) 55 pm (b) 5.5 pm (c) 0.55 pm (d) 55 nm
89. A proton is accelerated to one tenth of the velocity of light. If its velocity can be measured with a precision of 1%,
then the uncertainty in its position is equal to or greater than
(a) 1.93 nm (b) 19.3 nm (c) 19.3 pm (d) 193 pm
90. The angular momentum of an electron in f orbital is
(a) 12 (h / 2p) (b) 6 (h / 2p) (c) 2 (h / 2p) (d) 20 (h / 2p)
91. Which of the following statements on the atomic wave function is not correct?
(a) may be a real valued wave function
(b) may be in some cases be a complex function
(c)
(d)
92. Which of the following statements on the square of atomic wave function at a point near to the nucleus is not
correct?
(a) 2 may be positive, negative or imaginary
(b) 2 is proportional to electron density
(c) 2
(d) 2 is a normalized wave function
93. Which of the following statements on quantum numbers is not correct?
(a) Quantum numbers n, l, m and ms are needed to describe an electron in an atom completely.
(b) Quantum numbers n, l, m and s are obtained by solving the Schrödinger wave equation.
(c) A subshell in an atom can be designated with two quantum numbers n and l.
(d) The maximum value of l is equal to n – 1 and that of m is ± l.
94. Which of the following statements is not correct?
(a) The wave function depicting the dependence on r involves two quantum numbers n and l.
(b) The wave function depicting the angular dependence involves two quantum numbers l and m.
(c) The spin quantum number is not the outcome of the Schrödinger equation.
(d) The lowest energy state of an atom corresponds to n = 0.
95. In an orbital, the signs of lobes indicate the
(a) sign of the wave function (b) sign of the probability distribution
(c) presence or absence of electron (d) sign of charge
96. f orbitals are characterised by the quantum number
(a) n = 4 (b) l = 3 (c) m = 3 (d) s = 1/2
97. Which of the following sets of quantum numbers is not allowed?
(a) n = 2, l = 1, m = 1 (b) n = 2, l = 1, m = 0 (c) n = 2, l = 2, m = 1 (d) n = 3, l = 2, m = – 1
98. In the plot of r Y versus r for 1s orbital of hydrogen atom, maximum occurs at
2 2

(a) r = 0 (b) r = a0 (Bohr radius) (c) r = 2a0 (d) r = •


99. The plot of r2Y 2 versus r for 2s orbital of hydrogen atom exhibits
(a) one maximum (b) two maxima (c) no maximum (d) three maxima
100. The angular momentum of an electron in p orbitals is equal to
(a) zero (b) (h/2p) (c) 2 (h/2p) (d) 6 (h/2p)
101. The angular momentum of an electron in d orbitals is equal to
(a) zero (b) (h/2p) (c) 2 (h/2p) (d) 6 (h/2p)
102. The pz orbital corresponds to the magnetic quantum m equal to
(a) 0 (b) + 1 (c) – 1 (d) + 2
3.28 Complete Chemistry—JEE Main

103. The magnetic quantum number corresponding to dz2 orbital is


(a) + 2 (b) + 1 (c) 0 (d) – 2
104. For which of the following species, the electronic distribution is spherically symmetrical?
(a) H (b) Na (c) Cl – (d) B

(a) at the nucleus


(b) at a distance equal to Bohr radius from the nucleus

(d) at a distance equal to twice of the Bohr radius from the nucleus.
106. A dyz orbital has
(a) no nodal plane (b) one nodal plane in the yz plane
(c) two nodal planes in the xy and xz planes (d) Three nodal planes in the xy, xz and yz planes.
107. A dx2 – y2 orbital has
(a) no nodal plane (b) one nodal plane in the xy plane
(c) two nodal planes in the xz and yz planes (d) two nodal planes at angles 45° to x and y axes.
108. A dyz orbital is directed
(a) along the y– and z–axes (b) along the x– and y–axes
(c) along the centre of y– and z–axes (d) along the centre of x– and y–axes
109. A dxy orbital is directed
(a) along the x– and y–axes (b) along the z–axis
(c) along the centre of x and z–axes (d) along the centre of x– and y–axes
110. If an electron is to be located within 0.1 nm, the approximate uncertainty in its speed will be about
(a) 10–5 m s–1 (b) 6 ¥ 105 m s–1 (c) 10–2 m s–1 (d) 102 m s–1
111. The number of radial nodes in 4s orbital is
(a) 2 (b) 3 (c) 4 (d) 5
112. The wavelength of an electron accelerated by 100 V of potential difference is about
(a) 122.8 pm (b) 61.4 pm (c) 30.7 pm (d) 15.35 pm
The r-dependent wave function of hydrogen atom is given by the expression
Ê 2r 2r 2 ˆ
R = (constant) Á 3 - + exp (– r/3 a0)
Ë a0 9 a02 ˜¯
Answer the following four questions.
113. The above wave function depends on the quantum numbers
(a) n (b) n and l (c) n, l and m (d) n, l, m and s
114. The number of nodes in this wave functions is/are
(a) 0 (b) 1 (c) 2 (d) 3
115. The above wave function represents
(a) 1s orbital (b) 2s orbital (c) 3s orbital (d) 3p orbital
116. One of the nodes lies at the distance r from the nucleus, where r is about
(a) 1.5a0 (b) 2.5a0 (c) 5.5a0 (d) 7.1a0
117. One of the maximum value of R2 lies approximately at
(a) r = 2.0a0 (b) r = 4.0a0 (c) r = 11.47a0 (d) r = 12.5a0
The r-dependent wave function of hydrogen atom is given by the expression
Ê 2r r 2 ˆ
R = (constant) Á - 2 ˜ exp (r/3a0)
Ë a0 3a0 ¯
Answers the following questions.
Atomic Structure 3.29

118. The number of nodes (besides at r = 0) in this wave function is


(a) 0 (b) 1 (c) 2 (d) 3
119. The above wave function represents
(a) 2s (b) 2p (c) 3p (d) 3s
120. The node (besides at r = 0) occurs at
(a) 2a0 (b) 4a0 (c) 6a0 (d) 8a0

ANSWERS
1. (c) 2. (d) 3. (a) 4. (b) 5. (d) 6. (b)
7. (b) 8. (b) 9. (c) 10. (a) 11. (a) 12. (b)
13. (b) 14. (a) 15. (b) 16. (c) 17. (c) 18. (b)
19. (b) 20. (c) 21. (a) 22. (b) 23. (c) 24. (d)
25. (d) 26. (c) 27. (a) 28. (c) 29. (c) 30. (d)
31. (b) 32. (a) 33. (b) 34. (b) 35. (b) 36. (d)
37. (a) 38. (c) 39. (a) 40. (d) 41. (b) 42. (a)
43. (a) 44. (a) 45. (b) 46. (b) 47. (b) 48. (d)
49. (b) 50. (c) 51. (b) 52. (c) 53. (a) 54. (c)
55. (b) 56. (b) 57. (b) 58. (b) 59. (a) 60. (c)
61. (d) 62. (a) 63. (d) 64. (b) 65. (d) 66. (d)
67. (c) 68. (c) 69. (c) 70. (a) 71. (b) 72. (a)
73. (c) 74. (a) 75. (d) 76. (c) 77. (a) 78. (b)
79. (d) 80. (d) 81. (b) 82. (c) 83. (a) 84. (b)
85. (a) 86. (c) 87. (d) 88. (a) 89. (d) 90. (a)
91. (d) 92. (a) 93. (b) 94. (d) 95. (a) 96. (b)
97. (c) 98. (b) 99. (b) 100. (c) 101. (d) 102. (a)
103. (c) 104. (c) 105. (a) 106. (c) 107. (d) 108. (c)
109. (d) 110. (b) 111. (b) 112. (a) 113. (b) 114. (c)
115. (c) 116. (d) 117. (c) 118. (b) 119. (c) 120. (c)

HINTS AND SOLUTIONS


4. Isotones have same number of neutrons (= mass number – atomic number).
7. m = NA me = (6.022 ¥ 1023 mol–1) (9.1 ¥ 10–31 kg) = 5.48 ¥ 10–7 kg ∫∫ 0.548 mg
8. N = 1 C/e = 1 C/(1.6 ¥ 10–19 C) = 6.25 ¥ 1018
9. 73Li contains, 3p, 4n and 3e. Their sum of masses is
m¢ = [3(1.0073 u) + 4 (1.0086) + 3(.0005)] u = 7.0578 u
Mass corresponding to mass defect is
D m = E/c2 = (6.258 ¥ 10–12 J)/(3 ¥ 108 m s–1)2 = 6.953 ¥ 10–29 kg
In atomic unit, we have
Dm 6.953 ¥ 10-29
D m¢ = = u = 0.0419 u
1.66 ¥ 10-27 kg / u 1.66 ¥ 10-27
3.30 Complete Chemistry—JEE Main

Actual mass of Li is
m = m¢ – D m¢ = (7.0578 – 0.0419 u) = 7.0159 u
10. We have
63.546 u = x (62.930 u) + (1 – x) (64.928 u)
64.928 - 63.546
Hence x= = 0.69
64.928 - 62.930
Mass per cent of 65Cu = 0.31 ¥ 100 = 31%

14. Atomic size decreases along the period.


È (
˘ )
2
2p Ê e 2 ˆ 2 (3.14) 1.602 ¥ 10-19 C
15. d = = Í ˙ = 2.19 106 m s–1
h ÁË 4 pe 0 ˜¯ ( )
6.626 ¥ 10-34 J s Í1.112 ¥ 10-10 C2 N -1 m -2 ˙
Î ˚

16. ELi = Z2EH = (32) (2.17 10–11 erg) = 1.953 10–10 erg = 1.953 10–17 J

( )(
È 8.854 ¥ 10-12 C2 N -1 m -2 6.626 ¥ 10-34 J s ) ˘
2
È h2 ˘ Ê e h2 ˆ
2
17. r = n Í 2 ˙ = n 2 Á 0 2 ˜ = 12 Í ˙
(
ÍÎ 4 p m e 4 p e 0
2
) ˙˚ Ë pme ¯ Í
Î ( )(
(3.14) 9.1 ¥ 10-31 kg 1.602 ¥ 10-19 C
2
) ˙
˚

= 5.29 10–11 m = 52.9 pm

È1 1˘ Ê 1 1ˆ
18. DE = R•hc Í 2 - 2 ˙ ; ÁË n 2 - n 2 ˜¯ decreases with increase in the value of n1 and n2 (= n1 + 1)
Î n1 n2 ˚ 1 2

1 1
19. DE• 2 = DE• 1 = (21.7 10–12 erg) = 5.425 10–12 erg = 5.425 10–19 J
22 4
hc
=
(
6.626 ¥ 10-34 J s 3 ¥ 108 m s -1 )( )
l= 10–7 m 10–5 cm
DE 5.425 ¥ 10-19 J( = 3.664
) 3.664

26. Paschen spectral series lies in the near infrared region of electromagnetic radiation.
27. The expression of velocity is

1 Ê 1 Ze2 ˆ v2 n1
106 m s–1) ÊÁ ˆ˜ = 1.10
1
d= ; Thus = d = (2.19 106 m s–1
n ÁË 2 h e 0 ˜¯ v1 n2 Ë 2¯

28. The expression of energy is

Ê n2 ˆ
10–18 J) ÊÁ ˆ˜ = – 5.45
1 1
E= (constant). Thus E2 = E1 Á 12 ˜ = (– 2.18 10–19 J
n2 Ë n2 ¯ Ë 4¯

hc (6.626 ¥ 10-34 J s) (3 ¥ 108 m s -1 )


29. DE = = = 2.0438 ¥ 10–18 J
l (97.26 ¥ 10-9 m)

Ê1 1ˆ
Also DE = (2.18 ¥ 10–18 J) Á 2 - 2 ˜
Ë1 n2 ¯
1 Ê DE ˆ 2.0436 ¥ 10-18
Hence =1– Á ˜ =1– = 1 – 0.9374
2
n2 Ë 2.18 ¥ 10-18 J¯ 2.18 ¥ 10-18

n2 = 1 / (1 - 0.9374)  4
Atomic Structure 3.31

The transition n2 = 4 Æ n1 = 3 will emit the longest wavelength. Hence

DE = (2.18 ¥ 10–18 J) ÊÁ 2 - 2 ˆ˜ = 1.06 ¥ 10–19 J


1 1
Ë3 4 ¯
hc (6.626 ¥ 10-34 J s) (3 ¥ 108 m s -1 )
l= = = 1.875 ¥ 10–6 m = 1875 nm
DE (1.06 ¥ 10-19 J)
30. Ground state energy of Be3+ = Z2EH = (16) (2.18 ¥ 10–18 J) = 3.49 ¥ 10–17 J
1 Z e2 KE = 1
31. KE = mu 2 and PE = = – mu2. Therefore -
2 (4 p e 0 )r PE 2
32. Centripetal force is Ze2/(4 p e0 r2).

35. IE = NAR • hc = (6.022 ¥ 1023 mol–1) (1.09678 ¥ 107 m–1) (6.626 ¥ 10–34 J s) (3 ¥ 10 8 m s–1)
= 1.313 ¥ 106 J mol –1 = 1313 kJ mol –1
42. First line in the Lyman series of hydrogen atom corresponds to

DE = R• hc ÊÁ 2 - 2 ˆ˜ =
1 1 3
R•hc
Ë1 2 ¯ 4
For He +, the spectral line occurs at
Ê 1 1ˆ
DE = Z2 R• hc Á 2 - 2 ˜ = (22)R• hc ÊÁ 2 - 2 ˆ˜ =
1 1 5
R•hc
Ë n1 n 2 ¯ Ë2 4 ¯ 36
43. Velocity of electron decreases with increase in the value of n in Bohr theory. The electron will have maximum
kinetic energy in the n = 1 orbit.

44. For Balmer series, the longest wavelength corresponds to the transition 3 ¨ 2.
Ê 1 1ˆ
DE = R• hc Á 2 - 2 ˜ = R• hc ÊÁ 2 - 2 ˆ˜ =
1 1 5
R• hc
Ë n1 n 2 ¯ Ë2 3 ¯ 36
hc 36 36
l= = = = 6.565 ¥ 10–7 m = 656.6 nm
DE 5 R• 1.09678 ¥ 107 m -1
46. The expression of energy is
K
E = - 2 ; where K is constant
n
È 1 1˘ (2n + 1) K
Hence DE = En + 1 – En = – K Í - 2˙ =
Î ( n + 1)
2
n ˚ [n ( n + 1)]2

For n=1 DE = 3K/4 = 0.75 K


n=2 DE = 5K/36 = 0.14 K
n=3 DE = 7K/144 = 0.049 K
47. The expression of radius is
n2 r 2+ (12 / 3) K 1
rn = K; where K is constant. Hence Li = 2 =
Z rH+ (1 /1) K 3
48. The expression of speed is
Z u (Li 2 + ) 3 K /1
u= K; where K is constant. Hence = =9
n u (H) (1/ 3) K
51. The allowed values of l are 0, 1, …, (n – 1), a total of n values.
3.32 Complete Chemistry—JEE Main

52. The allowed values of m are 0, ± 1, ± 2, …, ± l, a total of 2l + 1 values.


53. m > l is not allowed.
57. The value of l cannot exceed n – 1.
2+ (1s)2(2s)2(2p)6(3s)2(3p)6(3d)8.
58. 28Ni (electron = 26) is
59. For 2d orbital, l > n, which is not allowed.
60. The number of orbitals in a shell is equal to n2. Each orbital can accommodate two electrons.
2(2s)2(2p)6(3s)2(3p)6(3d)5 (4s)1.
61.
62. Energy increases with increase in n + l value and thus the electron is less tightly bound.
63. The most paramagnetic atom would contain larger number of unpaired electrons.
64. Energy increases with increase in the value of n + l
1 1 2
58Ce is (4f) (5d) (6s) . But due to very similar energies of 4f and
5d orbitals in all the lanthanides excepting Gd (4f75d16s2) and 71Lu (4f14, 5d1, 6s2), (5d)1 electron is shifted to (4f)
2 0 2
58Ce is (4f) (5d) (6s) .
2 2 6 2 6 5 1
(2s) (2p) (3s) (3p) (3d) (4s) .
70. Each species contains 10 electrons. The number of neutrons are as follows
O2– : 16 – 8 = 8 Ne : 20 – 10 = 10 Na+ : 23 – 11 = 12 Mg2+ : 24 – 12 = 12
2 (2s)2 (2p)6 (3s)2 (3p)6 (3d)3 (4s)2
Z = 23) is (1s)
For l = 2, we have d orbital.

Cu+ (1s)2 (2s)2 (2p)6 (3s)2 (3p)6 (3d)10 K+ (1s)2 (2s)2 (2p)6 (3s)2 (3p)6
Al3+ (1s)2 (2s)2 (2p)6 33As
3+
(1s)2 (2s)2 (2p)6 (3s)2 (3p)6 (3d)10 (4s)2
2
(2s)2 (2p)6 (3s)2 (3p)6 (3d)3 (4s)2
The number of unpaired electrons will be 3 in (3d)3
74. The probability implies the value of y 21s. The function y has a maximum value at r = 0.
75. The number of nodal point in the plot of Rn, 0 versus r is n – (l + 1). For example in R1, 0 (i.e. 1s orbital) has no
node, and R2, 0 (i.e. 2s orbital) has one node.
76. At r = 2a0, there is a node as the value of function R2, 0 is zero at this distance
78. The number of radial node is equal to n – (l + 1). For 3p orbital, n = 3 and l = 1.
79. n,l,m = Rn,l { l,|m| m}: the terms within the brackets represents angular dependence.
80. The function R depends on the quantum numbers n and l, the function Q depends on l and | m | and the function
F depends on m.
Ê h ˆ
81. The expression is L = l (l + 1) Á .
Ë 2 p ˜¯

Lz = m ÊÁ ˆ˜ .
h
82. The expression is
Ë 2p ¯
83. An orbital with l = 0 involves no dependence on the angles q and j. Thus, it is symmetrical about the nucleus.
84. The energy of a single electron species such as H, He+, and Li2+ depends only the principal quantum number
whereas for a multielectron species, it depends on the principal as well as azimuthal quantum number.
85. Energy carried by one photon of wavelength 1.5 nm is

E1 =
hc
=
( )(
6.626 ¥ 10-34 J s 3 ¥ 108 m s -1 )
= 1.325 10–16 J
l (
1.5 ¥ 10-9 m )
Energy carried by one photon of frequency 1.13 1017 s–1 is
E2 = h = (6.626 10–34 J s) (1.13 1017 s–1) = 7.49 10–17 J
Kinetic energy of a single electron is
DE = E1 – E2 = (13.25 – 7.49) 10–17 J = 5.76 10–17 J
Atomic Structure 3.33

86. Binding energy of electron per atom of the metal


193 ¥ 103 J mol-1
Ebind = = 3.20 10–19 J
6.022 ¥ 1023 mol-1
Ebind 3.20 ¥ 10-19 J
= = = 4.83 1014 s–1
h 6.626 ¥ 10-34 J s
h 6.626 ¥ 10-34 J s
87. p = = = 6.626 10–24 J s m–1 = 6.626 10–24 kg m s–1
l 0.1 ¥ 10-9 m

1 2 p2 p2
88. KE = mv = Thus = eV or p= 2 m eV
2 2m 2m
h h 6.626 ¥ 10-34 J s
Now l= = =
( )( )
1/2
p 2 m eV ÈÎ( 2) 9.1 ¥ 10-31kg 1.6 ¥ 10-19 C (500 V )˘˚
= 5.49 10–11 m = 54.9 pm
1 Ê h ˆ 6.626 ¥ 10-34 J s
89. Dx ≥ = = 1.93 ¥ 10–10 m
Dp ÁË 4 p ˜¯ {(9.1 ¥ 10-31 kg )(3 ¥ 105 m s -1 )}(4)(3.14)

91. It is Y 2 and not Y


92. Y 2 cannot be imaginary. Y 2 is positive for a real wave function and Y *Y (where Y * is the complex conjugate
of Y) is positive.
93. The quantum number s is not the outcome of the solution of Schrödinger equation.
94. The value of n = 0 is not allowed.
95. The signs of lobes of an orbital indicates the sign of wave function.

2(2s)2(2p)6(3s)2(3p)6.
is (1s)
h /4 p (6.626 ¥ 10-34 J s) / (4 ¥ 3.14159)
110. Dv = = = 5.79 ¥ 105 m s–1
mD x (9.1 ¥ 10-31 kg) (10-10 m)
111. The number of radial nodes = n – l – 1 = 4 – 0 – 1 = 3
112. We have E = eV and E2 = p2/2m
Hence p= 2meV = [(2) (9.1 ¥ 10–31 kg) (1.6 ¥ 10–19 C) (100 V)]1/2 = 5.396 ¥ 10–24 kg m s–1
h 6.626 ¥ 10-34 J s
l= = -24 -1
= 1.228 ¥ 10–10 m = 122.8 pm
p 5.396 ¥ 10 kg m s
113. The R-part of the wave function depends on the quantum number n and l.
114. At the nodal point, R = 0. This will be possible if 3 – 2 (r/a0) + 2r2/9a 02 = 0. This will give two values of r. Hence,
there are 2 nodes.
115. The orbital is 3s, since it has two nodes.
116. Writing the quadratic expression as 2r¢2 – 18r¢ + 27 = 0
18 ± 182 - 8 ¥ 27 18 ± 108 18 ± 10.39
Its roots are r¢ = = = = 1.90, 7.10
4 4 4
R with respect to r will be equal to zero.
dR d ÈÏ Ê 2r 2r 2 ˆ ¸ - r / 3a0 ˘ ÈÊ 2 4r ˆ Ê 2r 2r 2 ˆ Ê 1 ˆ ˘ – r/3a0
= ÍÌconst ¥ Á 3 - + 2 ˜ ˝e ˙ = ÍÁ - + 2 ˜ + Á 3 - + ÁË - 3a ˜¯ ˙ e
dr dr Î Ó Ë a0 9a0 ¯ ˛ ˚ ÎË a0 9a0 ¯ Ë a0 9a02 ˜¯ 0 ˚
3.34 Complete Chemistry—JEE Main

2 4r 1 2r 2r 2
Hence - + 2 - + 2 - =0
a0 9a0 a0 3a0 27a03
2
2r 2 10r Ê rˆ Ê rˆ
or - +3=0 or 2 Á ˜ - 30 Á ˜ + 81 = 0
2
27a0 9a0 Ë a0 ¯ Ë a0 ¯
Solving for r/a0, we get
r 30 ± 900 - 8 ¥ 81 30 ± 252 30 ± 15.87
= = = = 3.53, 11.46
a0 4 4 4
118. At node, R = 0. There will be one node.
119. The orbital is 3p.
r Ê r ˆ
120. Á 2- = 0. This gives r = 6a0.
a0 Ë 3a0 ˜¯

MULTIPLE CHOICE QUESTIONS FROM AIEEE AND JEE MAIN

1. The number of d electrons retained in Fe2+ (atomic number 26) ion is


(a) 3 (b) 4 (c) 5 (d) 6 [2003]
2. Which of the following groupings represents a collection of isoelectronic species?
(Atomic numbers of Cs in 55 and that of Br is 35.)
(a) Ca2+ Cs+, Br– (b) Na+, Ca2+, Mg2+ (c) N3–, F–, Na+ (d) Be, Al3+, Cl– [2003]
3. The orbital angular momentum for an electron revolving in an orbital is given by l (l + 1)(h/ 2p ). The angular
momentum for an s-electron will be given by
(a) 2 (h/ 2p) (b) (1/2) (h/2p) (c) zero (d) (h/2p) [2003]
4. The de Broglie wavelength of a tennis ball of mass 60 g moving with a velocity of 10 m/s is approximately
(a) 10–25 m (b) 10–33 m (c) 10–31 m (d) 10–16 m [2003]
5. In Bohr series of lines of hydrogen spectrum, the third line from the red end corresponds to which one of the
following inter-orbit jumps of the electron for Bohr orbits in atom of hydrogen?
(a) 2 Æ 5 (b) 3 Æ 2 (c) 5 Æ 2 (d) 4 Æ 1 [2003]
6. Which of the following sets of quantum number is correct for an electron in 4f orbital?
(a) n = 4, l = 3, m = +4, s = +1/2 (b) n = 4, l = 4, m = –4, s = –1/2
(c) n = 4, l = 3, m = +1, s = +1/2 (d) n = 3, l = 2, m = –2, s = +1/2 [2004]
7. Consider the ground state of Cr atom (Z = 24). The numbers of electrons with the azimuthal quantum numbers, l
= 1 and 2 are, respectively
(a) 12 and 4 (b) 12 and 5 (c) 16 and 4 (d) 16 and 5 [2004]

1, would be (Rydberg constant = 1.097 ¥ 107 m–1)


(a) 91 nm (b) 192 nm (c) 406 nm (d) 9.2 ¥ 10–8 nm [2004]
9. Which one of the following sets of ions represents the collection of isoelectronic species?
(a) K+, Ca2+, Sc3+, Cl– (b) Na+, Ca2+, Sc3+, F– (c) K+, Cl2–, Mg2+, Sc3+ (d) Na+, Mg2+, Al3+, Cl–
(Atomic numbers: F = 9, Cl = 17, Na =11, Mg = 12, Al = 13, K = 19, Ca = 20, Sc = 21) [2004]
10. In a multi-electron atom, which of the following orbitals described by the three quantum numbers will have the

(i) n = 1, l = 0, m = 0 (ii) n = 2, l = 0, m = 0 (iii) n = 2, l = 1, m = 1 (iv) n = 3, l = 2, m = 1


(v) n = 3, l = 2, m = 0
Atomic Structure 3.35

(a) (ii) and (iv) (b) (iv) and (v) (c) (i) and (ii) (d) (ii) and (iii) [2005]
11. Of the following sets which one does not contain isoelectronic species?
(a) PO43–, SO42–, ClO4– (b) CN–, N2, C22– (c) SO32–, CO32–, NO–3 (d) BO33–, CO32–, NO–3
[2005]
12. Which of the following statements in relation to hydrogen atom is correct?
(a) 3s and 3p orbitals are of lower energy than 3d orbital
(b) 3s, 3p and 3d orbitals all have the same energy
(c) 3s orbital is lower in energy than 3p orbital
(d) 3p orbital is lower in energy than 3d orbitals [2005]
13. According to Bohr’s theory, the angular momentum of an electron in 5th orbit is
(a) 2.5 h/p (b) 25 h/p (c) 2.5 h/p (d) 10 h/p [2006]
14. Uncertainty in the position of an electrons (mass = 9.1 ¥ 10–31 kg) moving with a velocity 300 m s–1, accurate upto
0.001% will be
(a) 3.84 ¥ 10–2 m (b) 19.2 ¥ 10–2 m (c) 5.76 ¥ 10–2 m (d) 1.92 ¥ 10–2 m [2006]
15. Which one of the following sets of ions, represents a collection of isoelectronic species?
(a) Li+, Na+, Mg2+, Ca2+ (b) K+, Cl–, Ca2+, Sc3+ (c) Ba2+, Sr2+, K+, Ca2+ (d) N3–, O2–, F–, S2–
[2006]
16. Which of the following sets of quantum numbers represents the highest energy of electron in an atom?
(a) n = 3, l = 1, m =1, s = +1/2 (b) n = 3, l = 2, m =1, s = +1/2
(c) n = 4, l = 0, m =1, s = +1/2 (d) n = 3, l = 1, m =1, s = +1/2 [2007]
17. The ionization enthalpy of hydrogen atom is 1.312 ¥ 10 6 J mol–1. The energy required to excite the electron in the

atom from n = 1 to n = 2 is
(a) 9.84 ¥ 105 J mol–1 (b) 8.51 ¥ 10–2 J mol–1 (c) 6.56 ¥ 105 J mol–1 (d) 7.56 ¥ 105 J mol–1
[2008]
18. In an atom, an electron is moving with a speed of 600 m/s with an accuracy of 0.005%. Certainty to which the
position of the electron can be located is (h = 6.6 ¥ 10–34 kg m2 s–1, mass of an electron, me = 9.1 ¥ 10–31 kg)
(a) 1.92 ¥ 10–3 m (b) 3.84 ¥ 10–3 m (c) 1.52 ¥ 10–3 m (d) 5.10 ¥ 10–3 m [2009]
19. Calculate the wavelength associated with a proton moving at 1.0 ¥ 103 m s–1. (Mass of a proton =1.67 ¥ 10–27 kg
and h = 6.63 ¥ 10–34 J s).
(a) 2.5 nm (b) 14.0 nm (c) 0.032 nm (d) 0.40 nm [2009]
20. Ionization energy of He is 1.96 ¥ 10 J atom
+ –18 –1 2+
n = 1) of Li is
(a) 8.82 ¥ 10 J atom
–17 –1
(b) 4.41 ¥ 10 J atom
–16 –1
(c) –4.41 ¥ 10 J atom
–17 –1
(d) –2.2 ¥ 10–15 J atom–1
[2010]
21. A gas absorbs a photon of 355 nm and emits two wavelengths. If one of the emissions is at 680 nm, the other is at
(a) 518 nm (b) 1035 nm (c) 325 nm (d) 743 nm
[2011 (cancelled)]
22. The frequency of light emitted for the transition n = 4 to n = 2 of He+ is equal to the transition in H atom
corresponding to which of the following?
(a) n = 3 to n = 1 (b) n = 2 to n = 1 (c) n = 3 to n = 2 (d) n = 4 to n = 3 [2011]
n and l
(i) n = 4, l = 1 (b) n = 4, l = 0 (c) n = 3, l = 2 (d) n = 3, l = 1
can be placed in order of increasing energy is
(a) (iii) < (iv) < (ii) < (i) (b) (iv) < (ii) < (iii) < (i) (c) (ii) < (iv) < (i) < (iii) (d) (i) < (iii) < (ii) < (iv)
[2012]
24. Energy of and electron is given by E = –(2.178 ¥ 10 J) (Z /n ). Wavelength of light required to excite an electron
–18 2 2

in an hydrogen atom from level n = 1 to n = 2 will be: (h = 6.62 ¥ 10–34 J s and c = 3.0 ¥ 108 m s–1)
(a) 1.214 ¥ 10–7 m (b) 2.816 ¥ 10–7 m (c) 6.500 ¥ 10–7 m (d) 8.500 ¥ 10–7 m [2013]
3.36 Complete Chemistry—JEE Main

25. The correct set of four quantum numbers for the valence electrons of rubidium atom (Z = 37) is
(a) 5, 0, 1, +1/2 (b) 5, 0, 0, +1/2 (c) 5, 1, 0, +1/2 (d) 5, 1, 1, +1/2 [2014]

state of Li2+ is
(a) –27.2 eV (b) 30.6 eV (c) –30.6 eV (d) 27.2 eV [2014, online]
27. If l0 and l1 are the threshold wavelength and wavelength of incident light, the velocity of photoelectron ejected
from the metal surface is
2h 2hc 2hc Ê l0 - l ˆ 2h Ê 1 1 ˆ
( l0 - l ) ( l0 - l ) Á ˜ -
m ÁË l0 l ˜¯
(a) (b) (c) (d)
m m m Ë ll0 ¯
[2014, online]
Ê 1 1 ˆ
28. Based on the equation DE = –(2.0 ¥ 10–18 J) Á 2 - 2 ˜
Ë n2 n1 ¯
the wavelength of the light that must be absorbed to excite hydrogen electron from level n = 1 to level n = 2 will
be (Given: h = 6.625 ¥ 10–34 J s, c = 3 ¥ 108 m s–1)
(a) 1.325 ¥ 10–7 m (b) 1.325 ¥ 10–10 m (c) 2.650 ¥ 10–7 m (d) 5.300 ¥ 10–10 m
[2014, online]
29. The de-Brogile wavelength of a particle of mass 6.63 g moving with a velocity of 100 m s–1 is
(a) 10–33 m (b) 10–35 m (c) 10–31 m (d) 10–25 m [2014, online]
30. Excited hydrogen atom emits light in the ultraviolet region at 2.47 ¥ 1015 Hz. With this frequency, the energy of
a single photon is (Given : h = 6.63 ¥ 10–34 J s)
(a) 8.041 ¥ 10–40 J (b) 2.680 ¥ 10–19 J (c) 1.640 ¥ 10–18 J (d) 6.111 ¥ 10–17 J
[2014, online]
31. Ionization energy of gaseous Na atoms is 499.5 kJ mol–1. The lowest possible frequency of light that ionizes a
sodium atom is (Given : h = 6.626 ¥ 10–34 J s, NA = 6.022 ¥ 1023 mol–1)
(a) 7.50 ¥ 104 s–1 (b) 4.76 ¥ 1014 s–1 (c) 3.15 ¥ 1015 s–1 (d) 1.24 ¥ 1015 s–1
[2014, online]
32. If the principal quantum n
(a) ns Æ np Æ (n – 1)d Æ (n – 2)f
(b) ns Æ (n – 2)f Æ (n – 1)d Æ np
(c) ns Æ (n – 1)d Æ (n – 2)f Æ np
(d) ns Æ (n – 2)f Æ np Æ (n – 1)d [2015, online]
33. At temperature T, the average kinetic energy of any particle is (3/2) kT. The de Broglie wavelength follows the
order:
(a) Thermal proton > Visible photon > Thermal electron
(b) Thermal proton > Thermal electron > Visible photon
(c) Visible photon > Thermal electron > Thermal neutron
(d) Visible photon > Thermal neutron > Thermal electron [2015, online]
34. Which of the following is the energy of a possible excited state of hydrogen?
(a) +13.6 eV (b) –6.8 eV (c) –3.4 eV (d) +6.8 eV [2015]
35. The total number of orbitals associated with the principal quantum number 5 is
(a) 20 (b) 25 (c) 10 (d) 5 [2016, online]

ANSWERS
1. (d) 2. (c) 3. (c) 4. (b) 5. (a) 6. (c)
7. (b) 8. (a) 9. (a) 10. (b) 11. (c) 12. (b)
Atomic Structure 3.37

13. (a) 14. (d) 15. (b) 16. (b) 17. (a) 18. (a)
19. (d) 20. (c) 21. (d) 22. (b) 23. (b) 24. (a)
25. (b) 26. (c) 27. (c) 28. (a) 29. (a) 30. (c)
31. (d) 32. (b) 33. (c) 34. (c) 35. (b)

HINTS AND SOLUTIONS


2
(2s)2 (2p)6 (3s)2 (3p)6 (3d)6 (4s)2.
With the removal of 4s electrons, Fe2+ is formed. Hence, 3d orbitals retain 6 electrons.
2. All the three species in choice c contain 10 electrons.
h (6.63 ¥ 10-34 J s)
4. l = = = 1.1 ¥ 10-33 m
mu (60 ¥ 10-3 kg)(10 m s -1 )
6. For a 4f orbital, we have
Principal quantum number, n = 4; Azimuthal quantum number, l = 3
Magnetic quantum number, m = any one value from +3, +2, +1, 0. –1, –2, –3
For an electron in this orbital, spin quantum number, s = either +1/2 or –1/2
24Cr is (1s)2 (2s)2 (2p)6 (3s)2 (3p)6 (3d)5 (4s)1
l = 1 corresponds to p orbital. The number of electrons in p orbitals is 6 + 6 = 12
l = 2 corresponds to d orbital. The number of electrons in d orbitals is 5

 = R Ê 1 - 1 ˆ = R ÊÁ 1 - 1 ˆ˜ = R ;
8. We have DE l=
1
= 0.91 ¥ 10-7 m = 91nm
•Á 2
Ë n1 n22 ˜¯
•Ë 2 • •
1 •2 ¯ R
9. The isoelectronic species have the same number of electrons. Each of the species K+, Ca2+, Sc3+ and Cl– has the
same number (=18) of electrons.
n + l possess the same electronic
energy.
11. The number of electrons of each species in the choice a is 50.
The number of electrons of each species in the choice b is 14.
The number of electrons of each species in the choice c are: SO32–(42), CO32–(32) and NO3– (42).
The number of electrons of each species in the choice d is 32.
12. For a single electron species, the energy of electron depends only on the principal quantum number. Hence, 3s,
3p and 3d orbitals will have the same energy.
13. In Bohr’s theory, the angular momentum of electron is given by L = n(h/2p), where n is the quantum number. For
5th orbit, n = 5.
14. p = mv = (9.1 ¥ 10–31 kg) (300 m s–1) = 27.3 ¥ 10–29 kg m s–1
Dp = (0.001%)p = (10–5) (27.3 ¥ 10–29 kg m s–1) = 27.3 ¥ 10–34 kg m s–1
1 Ê hˆ 1 Ê 6.626 ¥ 10-34 J s ˆ
Dx = ÁË ˜¯ = ˜¯ = 0.0192 m
Dp 4p (27.3 ¥ 10-34 kg m s -1 ) ÁË 4 ¥ 3.14
15. 19K+, 17Cl–, 20Ca2+ and 21Sc3+ have the same number of 18 electrons.
16. Larger the value of n + l, higher the energy of atomic orbital.
17. The energy difference between the two orbits of hydrogen atom is given by
Ê 1 1ˆ
DE = N A R• hc Á 2 - 2 ˜
Ë n1 n2 ¯
For ionization, n1 = 1 and n2 = •. Hence, NAR•hc = 1.312 ¥ 106 J mol–1
For n =1 Æ n = 2, the energy difference is
3.38 Complete Chemistry—JEE Main

3 3
DE = ( N A R• hc) = ¥ 1.312 ¥ 106 J mol-1 = 9.84 ¥ 105 J mol-1
4 4
18. According to the uncertainty principle Dp Dx ≥ (h/4p) i.e (m Du) Dx ≥ (h/4p)
h 1 (6.6 ¥ 10-34 kg m 2 s -1 )
Hence, Dx ≥ = -31 -1
= 1.92 ¥ 10-3 m
4p m( Du ) 4(22 / 7)(9.1 ¥ 10 kg){(600 m s )(0.005 / 100)}

h h (6.63 ¥ 10-34 J s)
19. We have l= = = = 4.0 ¥ 10-10 m = 0.40 nm
p mu (1.67 ¥ 10-27 kg)(1.0 ¥ 103 m s -1 )

20. (IE)He+ = Z12 R• hc = 19.6 ¥ 10-18 J


19.6 ¥ 10-18 J 19.6 ¥ 10-18 J
This gives R• hc = =
Z12 4
(9)(19.6 ¥ 10-18 J/ 4)
Z 22 R• hc
( E1 ) Li2 + = - =- = -4.41 ¥ 10-17 J
n12 (12 )
1 1 1 (355 nm)(680 nm)
21. The conversion of energy required that = + or l = = 743 nm
355 nm 680 nm l (680 - 355) nm
Ê 1 1ˆ
22. The expression of emission of special transitions is DE = Z 2 R• hc Á 2 - 2 ˜
Ë n2 n1 ¯
Ê 1 1ˆ 3
For the given transition in He+, we have DE = (2) 2 R• hc Á 2 - 2 ˜ = R• hc
Ë2 4 ¯ 4
In hydrogen atom, this transition will correspond to the transition n2 = 1 ¨ n1 = 2, since
Ê1 1ˆ 3
DE = (1)2 R• hc Á 2 - 2 ˜ = R• hc
Ë1 2 ¯ 4
23. Energy of an electron in an atom increases with increase in the value of n + l. For the same value of n + l, the
energy is larger for the larger value of n. Hence, we have
Choice (i); n+l=4+1=5 Choice (iii); n+l=3+2=5
Choice (ii); n+l=4+0=4 Choice (iv); n+l=3+1=4
Hence, the correct order of increasing energy is (iv) < (ii) < (iii) < (i)

24. DE = E2 - E1 = (-2.178 ¥ 10-18 J) ÊÁ - ˆ˜ = 1.6335 ¥ 10-18 J


1 1
Ë 4 1¯

hc (6.62 ¥ 10-34 J)(3 ¥ 108 m s -1 )


l= = -18
= 1.214 ¥ 10-7 m
DE (1.6335 ¥ 10 J)
1
. Thus it has n = 5,
l = 0, m = 0 and ms = +1/2
Z2
26. The energy of electron in Bohr orbits is E=- R• (where R• is Rydberg constant)
n2
E = –R• = –13.6 eV
2
3 9 9
For Li2+ (Z = 3), we have E=-
R = - 2 R• = - 2 (13.6 eV)
2 •
n n n
When n = 2, E = –30.6 eV(given choice).
27. E0 = hc/l0 and E = hc/l
1/ 2
1 Ê 1 1 ˆ hc(l0 - l ) ; È 2hc (l0 - l ) ˘
mu 2 = E - E0 = hc Á - ˜ = u=Í
2 Ë l l0 ¯ ll0 Î m ll0 ˙˚
Atomic Structure 3.39

Ê 1 1ˆ -18 Ê 1 1ˆ -18 -18


28. DE = -(2.0 ¥ 10-18 J) Á 2 - 2 ˜ = -(2.0 ¥ 10 J) ÁË - ˜¯ = (2.0 ¥ 10 J)(3 / 4) = 1.5 ¥ 10 J
Ë n2 n1 ¯ 4 1

hc (6.625 ¥ 10-34 J s)(3 ¥ 108 m s -1 )


l= = = 1.325 ¥ 10 –7 m
DE (1.5 ¥ 10-18 J)

h (6.626 ¥ 10-34 J s)
29. l = = = 10-33 m
mu (6.63 ¥ 10-3 kg)(100 m s -1 )
30. E = hn = (6.63 ¥ 10-34 J s)(2.46 ¥ 1015 s -1 ) = 1.64 ¥ 10-18 J

31. Ionization energy per gaseous atom is


Ê 495.5 ¥ 103 J mol-1 ˆ -19 E (8.228 ¥ 10-19 J)
E=Á = 8 .228 ¥ 10 J; n = = = 1.24 ¥ 1015 s -1
Ë 6.022 ¥ 1023 mol-1 ˜¯ h (6.626 ¥ 10-34 J s)
32. The guiding principle is
Energy increases with increase in the value of n + l. For the same value of n + l, smaller the value of n, lesser the
energy
For the given orbitals, we have
ns np (n – 1)d (n – 2)f
lÆ 0 1 2 3
n+l n n+1 n+1 n+1

ns Æ (n – 2)f Æ (n – 1)d Æ np
33. Lesser the mass of particle, larger its de Broglie wavelength (l = h/mu). Hence, the order of wavelength is
Visible photon > Thermal electron > Thermal neutron
34. The energy of orbits in hydrogen atom is given by the expression
13.6 eV
En =
n2
For n = 2, E2 = –3.4 eV
35. The number of orbitals in the principal quantum number n is n2. Hence, answer is 25.
4
Chemical Bonding and
Molecular Structure

SECTION 1 Bond Formation and VSEPR Theory

The formation of a chemical bond between two atoms implies that the system consisting of these two atoms at stable
internuclear distance is energetically more stable than the two isolated atoms. A general study on the reactivity of
different elements revealed that noble gases have little tendency to combine with other elements. This leads to the
ns)2 np)6

Kössel and Lewis Theory of Chemical Combination


According to this theory, atoms can combine either by transfer of outer-shell electrons, known as valence electrons,

eight electrons) in their respective valence shells.

formation of ionic bond between the two involved atoms.


Representation of a Bond by Lewis Structure In Lewis structure, a bond between the two atoms is shown by
Lewis electron-dot symbols in which valence electrons are shown by dots around the letter symbol of the atom. The
dots are placed as follows.
Place a single dot on the four sides of the letter symbol followed by the second dot till all the valence-electrons have
been accounted for.
Illustrations:

Lithium Beryllium Boron Carbon


1 Li 2 Be 2 1 B C
2 2

Nitrogen Oxygen Fluorine Neon


2 3 N 2 4 O 2 5
F 2 6 Ne

Formation of Covalent Bond(s) A covalent bond involves mutual sharing of valence electrons between two atoms.
The sharing of two, four and six electrons leads to the formation of a single, double and triple bond, respectively.
A covalent bond is formed if the atoms have lesser number of valence electrons as compared to the nearby noble

formation of covalent bond is


4.2 Complete Chemistry—JEE Main

Electronegative element + Electronegative element æÆ Covalent bond


Exception to the octect rule is the hydrogen atom which can accommodate only two electrons which corresponds to
2) atom.

Illustrations
Formation of Single Bond(s)
Cl2 Cl + Cl Cl Cl or Cl Cl

Cl2 2H + O H O H or H O H

NH3 3H + N H N H or H N H
H
H

H
H
CH4 4H + C H C H or H C H
H
H
Formation of Double Bond(s)

O2 O + O O O or O O

CO2 O + C + O O C O or O C O

C 2H 4 4 H + 2 C H C C H or H C C H
H H
H H
Formation of Triple Bond(s)

N2 N + N N N or N N

C 2H 2 2 H + 2 C H C C H or H C C H

Exceptions to the Octect Rule The octect rule is generally obeyed by the elements of second and third periods
with the following exceptions.
The Incomplete Octect

BeCl2 Cl + Be + Cl Cl Be Cl
4 electrons
Cl
AlCl3 3 Cl + Al Cl Al Cl

6 electrons

Odd-Electron Molecules All atoms of a compound containing odd number of electrons will not satisfy octect rule
as even number of electrons are required for pairing of electrons.
Chemical Bonding and Molecular Structure 4.3

NO N + O N + O N O

7 electrons

NO2 2 O + N O + N + O O N O

+ –
O N O

7 electrons

The Expanded Octect Elements of third period and beyond can accommodate more than 8 electrons due to the
availability of vacant d orbitals.

Cl

PCl5 5 Cl + P 5 Cl + P Cl Cl
P

Cl Cl

F F

6 6 F + 6 F + F F

F F

Coordinate Covalent (or Dative) Bond If a pair of electrons shared between two atoms comes exclusively from

). Once a coordinate bond is formed, it behaves like


a covalent bond.

H F H F

H3N—BF3 H N + B F H N B F

H F H F

Writing a Lewis Structure


The structure of a molecule or ion may be written by following the steps listed below.
1. Calculate the total numbers of valence electrons of the atoms in the molecule. For an anion, add the number of
negative charges and for a cation, subtract the number of positive charges.
2. Write the skeleton structure of the molecule or ion connecting every bonded pair of atoms by a single bond,
i.e. a pair of electron dots.

If there are fewer than eight electrons on the central atom, move one or two pairs of electrons from a surrounding

Illustration Lewis structure of COCl2.


Step 1 Valence electrons are 4 + 6 + 2 ¥ 7 = 24
4.4 Complete Chemistry—JEE Main

Step 2 Carbon being the most electropositive atoms occupies the central position to which other atoms are bounded.
O
Cl C Cl
Step 3 Assign 8 electrons each to surrounding atoms
O
Cl C Cl
Step 4 There were 24 valence electrons and all of them have been distributed. However, the central C atom has only
6 electrons. In order that this atom also has 8 electrons, move one pair of electrons from O to the bond connecting C
atom, thus forming a double bond.
O
O O
Cl C Cl Cl C Cl or Cl C Cl

Formal Charge and Lewis Structure


The formal charge on an atom is the difference between the valence electrons in an isolated atom and the number of
electrons assigned to that atoms in a Lewis structure. The equation for computing formal charge is
1
2 bonding) electrons in a Lewis structure
The sum of the formal charges of atoms in a Lewis structure is equal to the charge on the molecular species.
Cl C Cl
Illustration COCl2 molecule Lewis structure
O
Atom Valence electrons Electrons in Lewis structure Formal Charge
in a free atom Nonbonding bonding
Cl 7 6 2 1
+ 2 ¥ 2) =
O 6 4 4 1
+ 2 ¥ 4) =
1
C 4 8 + 2 ¥ 8) =

Utility of Formal Charge Computing formal charge of atoms in a molecule or ion helps deciding a plausible Lewis
structure of the species. The guiding principles are as follows.
Amongst the several Lewis structures, the species having the lowest magnitude of formal charge is the preferred
structure.
Amongst Lewis structures having similar distribution of formal charges, the one having negative formal charges
on the more electronegative atoms is the preferred structure.

Polar Covalent Bond


Each atom in a molecule has its own ability to attract the bonded pair of electrons. This ability is known as electronegativity.
2, O2, F2, Cl2, etc.) is shared equally by both

of electrons is closer to the atom having larger electronegativity. Consequently, this atom acquires a partial negative
charge while the other atom acquires equal partial positive charge. Because of the charge separation, the covalent bond
between these two atoms is said to be a polar covalent bond.
Dipole Moment The polarization of bonded pair of electrons between two atoms is expressed in terms of physical
m
m dq r
where d q is the partial charge separation between two atoms and r is the distance between the two atoms.
Chemical Bonding and Molecular Structure 4.5

Representation of Dipole Moment Dipole moment is a vector quantity, i.e. it has magnitude as well as direction.
In chemistry, dipole moment is indicated by the crossed arrow as shown in the following.†
positive end negative end
that is, it is directed from positive end to the negative end.
Unit of Dipole Moment dq r) = C m

–18
esu cm. This value of dipole moment is known as 1 debye

Ï Ê 1.6 ¥ 10-19 C ˆ ¸
–18 –18
) Ì(1esu ) Á ˝ –2
m) = 3.33 ¥
Ë 4.8 ¥ 10-10 esu ˜¯ ˛
Cm
Ó
Dipole Moment of a Polyatomic Molecule Each bond in a molecule has a dipole moment, known as bond
moment. The dipole moment of a molecule is obtained by the vector addition of these bond moments.

Illustration The bond moment of O H bond is 1.52 D. The bond angle of H2


H2O molecule will be H
mH2O = 2mOH O
Nonpolar Polyatomic Molecule H
The dipole moment of a nonpolar polyatomic molecule is zero inspite of the fact that the bond moments of the molecule
is not zero. This is due to the fact that the individual bond moments in the molecule is symmetrically placed so that
their vector additions is zero.
Illustration
F Cl

F Be F B
F C O C O
Cl Cl
F
Cl
Comparison of Dipole Moments of NH3 and NF3
Both NH3 and NF3 have pyramdial shapes with lone pair of electrons on nitrogen atom.

N N
H H F F
H F
In NH3, orbital dipole acts in the same direction as the sum of bond vectors of the three N H bond bonds.
In NF3, orbital dipole acts in the opposite direction to the sum of bond vectors of the three N F bond bonds.
These facts make the dipole moment of NH3 m = 1.57 D) larger than that of NF3 m

Per Cent Ionic Character of a Polar Band The per cent ionic character of a polar band A
m AB
Per cent ionic character = ¥ 100
mionic
where mionic = e rAB

† In actual practise, the dipole moment is represented as a simple arrow pointing from negative end to the positive end, that is
negative end positive end
In chemistry, the crossed arrow is used which indicates the direction of the shift in electronegativity in the molecule.
4.6 Complete Chemistry—JEE Main

Illustration The bond moment of O H bond is 1.52 D. If bond length O


will be
mOH (1.52 D) (3.33 ¥ 10-30 C m / 1 D)
Per cent ionic character = ¥ 100 = ¥ 100
e rOH (1.6 ¥ 10-19 C) (95 ¥ 10-12 m)

Concept of Resonance
distribution of electrons over a given skeleton of atoms in a molecule. None of the individual structures adequately
explains the characteristics of the molecule. However, these can be explained if the actual structure of the molecule is
considered as the superposition of individual structures. This phenomenon is known as resonance and the individual
) inserted between the
resonating structures.
Illustrations

written as
O

O
O

O
O bond lengths equal in size.
O 2– O 2– O 2– O 2–

C C C written as C
O O
O

O
O

O
O

Formation of an Ionic Bond

leads to the formation of positive and negative ions, respectively. The electrostatic attraction between the positive and
negative ions results in the formation of an ionic bond between the involved ions.
Illustrations

Na + Cl Na+ + Cl
1 2 5 2 6


F + Mg + F Mg2+ + 2 F
2 5 2 2 5 2 6

Energies Involved in the Formation of One Molecule of Sodium Chloride The formation of Na+Cl–

Æ Na+ – Ei = 8.24 ¥ J
– Æ Cl– Eea = –5.78 ¥ J
+ – Æ Na+Cl– PE
where PE is the potential energy in the formation of ionic bond. This is evaluated by the expression
QQ
PE = 1 2
(4pe 0 )r
where Q1 = –Q2 ¥ C and r = rNa+ + rCl– Q1 and Q2 as point charges,
we have
(1.60 ¥ 10-19 C)(-1.60 ¥ 10-19 C)
PE = = - 8.34 ¥ 10-19 J
(4)(3.14)(8.854 ¥ 10-12 C2 N -1 m -2 )(276 ¥ 10-12 m)
Chemical Bonding and Molecular Structure 4.7

Æ Na+Cl– DE =?
we have DE = Ei + DEea ¥ J = –5.88 ¥
Comment The negative value of DE indicates that the formation of an isolated ionic bond Na+Cl–

Essential Requirement for the formation of an Ionic Bond For DE


Ei
Ei
Hence
Electropositive element + Electronegative element Ionic Bond

Formation of 1 mol of Solid Ionic Compound from Constituent Elements Taking an example of sodium
chloride, we have the following steps in the formation of solid compound.
Æ DH1 –1
1
2 Cl2 Æ DH2 –1

Æ Na+ –
DH3 –1
– Æ Cl–
DH4 –1
+ – Æ Na Cl
+ – DH5 = –788 kJ mol–1
1
2 Cl2 Æ Na+ DH = –313 kJ mol–1
DH is negative, the formation of solid NaCl is energetically favourable. From the values of DH’s listed above, it

Æ Na+ –

the ionic solid.

compound into gaseous constituent ions. Larger the value of lattice energy, more stable the ionic compound.

listed above for the formation of solid ionic compound. In this cycle, DH
DH1, DH2, DH3 and DH4 from the value of DH gives the value of DH5. The lattice energy is negative of the value of DH5.
Fajan Rules An ionic compound has partial covalent character and vice versa. The partial covalency in an ionic
compound my be explained qualitatively with the help of Fajan rules described in the following.
High Charge and Small Size of the Cation
cationic electronic charge to penetrate partially into the anionic electronic cloud resulting into the partial covalent
bond character to the ionic bond.
High Charge and Large Size of the Anion The electronic cloud of such an anion is most easily polarized by

For two cations of the same size and charge, the cations of electronic
n – 1)dx ns
n – 1)s2 n – 1)p6 ns
nucleus by the electronic cloud of transition metal ions as compared to that in the alkali and alkaline metal ions.
Illustrations Hg2+ ion has larger polarizing effect than Ca2+ ion. Lithium salts have more covalent character than
the alkali salts. I– ion is more easily polarized than Cl– ion by Ag+ ion.

The Valence Shell Electron Pair Repulsion (VSEPR) Theory


4.8 Complete Chemistry—JEE Main

The number of electron pairs in the valence shell of the central atom of a molecule decides the shape of the

A multiple bond is treated as if it is a single electron pair.


The repulsive interaction of electron pairs decrease in the order

Table 1 Shapes of some molecules on the basis of VSEPR model

Molecule Number of valence Basic shape †


electrons electron pairs bounding pairs lone pairs
around the
central atom
BeCl2 4 2 2 Linear

Triangular
BCl3 6 3 3
planar

CH4 8 4 4

NH3 8 4 3 1 Tetrahedron

H 2O 8 4 2 2

PF5 5 5
Trigonal
4 5 4 1
bypyramid
CIF3 5 3 2

6 12 6 6
IF5 12 6 5 1 Octahedron

A few examples of molecules containing lone pair electrons along with their geometry are described in the following.

N O
O O H H H H
H

† The actual shapes of molecules containing lone pairs is a little distorted from the basic shape. This is due to the fact that lone pair-bonded
pair repulsion is larger than bonded pair-bounded repulsion.
Chemical Bonding and Molecular Structure 4.9

F F
F

F Cl
F
F F

Note: Lone pair equatorial position as it involves only two lp-bp Note: Lone pair occupy equatorial positions so as to have
repulsion. It is not occupied in axial position as it will involve lesser lp-bp repulsions
three lp-bp repulsions

Note:

F O
F F F F F F
Xe I Xe
F F F F F F

F I F F F
O
F I Cl F Xe Xe

F O
F I F F F

MULTIPLE CHOICE QUESTIONS ON SECTION 1

Identify the correct choice in the following questions.


1. Which of the following statements regarding Hg2+ 2+
2+ ion has more polarizing effect on an anion as compared to Ca2+ ion
2+ ion has less polarizing effect on an anion as compared to Ca2+ ion
2+ ion and Ca2+ have equal polarizing effect on an anion
2+ ion and Ca2+have no polarizing effect on an anion

2. The increasing order of covalency in silver halides is

3. The increasing order of melting point of carbonates of alkaline-earth metals is


3 3 3 3 3 3
3 3 3 3 3 3
4. Which of the following resonating structures of N2O is feasible in its linear structure?

N N O N N O N N O N N O
– +
+ – 2– + +
5. Which of the following statements regarding BrF3 molecule is correct?

Br
4.10 Complete Chemistry—JEE Main

6. Which of the following pairs of species are not isostructural?


+ + – + , IF
4, PF 4 4 and NH 4 2, I 3 5 5
7. Which of the following order regarding bond angle O N O in the given species is correct?
+ – – + + – + –
2 2 2 2 2 NO 2 2 2 2 2 2 2
8. Which of the following facts regarding Fajan’s rules is not correct?

– is not correct?
3

3 is correct?
F bond has the same length
Cl

11. The structure of ICl4 is


Cl Cl
Cl Cl Cl
Cl Cl
I I I
I
ClCl Cl Cl
Cl Cl Cl
Cl Cl
12. Which of the following statements is correct?
2 2 has nonlinear structures
2 2 has linear structures
2 and CO2 has linear structures
2 and CO2 has non-linear structures
13. The shape of XeF4 is

14. Which one of the following compounds of xenon has trigonal bipryramid structure?
3 2F 2 4 3F 2
15. The number of pairs of electrons around I in IF3, IF5 and IF7 ,respectively, are

16. Which of the following species has pyramidal shape?


2– –
3 3 3 3
17. Which of the following species has linear structure?
– +
2 2 2
18. Which of the following statements regarding bond angles in the given molecules is correct?
3 3 and OF2 2 3 3 and OF2 2
3 3 and OF2 2 3 3 and OF2 2

3 3 3 3 3 3 3 3 3 3 3 3
Chemical Bonding and Molecular Structure 4.11

s – bond and 1 p s – bond, 1 p – bond and 1 coordinate bond

ANSWERS

HINTS AND SOLUTIONS


1. Hg2+ ion belongs to transition metal and Ca2+ ion to alkaline-earth metal. Thus Hg2+ ion has more polarizing effect
on an anion.
2. As the size of halide is increased, it is more polarizable by Ag+ and hence larger is the covalency of silver halide.
2+) of Group 2 of periodic table, the polarizing power on CO2– is decreased. The
3
MCO3 becomes more ionic, hence their melting point increases.

F
5. There are 28 valence electrons. These are distributed as F Br
F F

Hence, its structure is pentagonal bipyramid with two lone pairs occupying equatorial positions. Br F
Due to lone pair-bonding pair, the angle F Br
6. XeF4 has 6 pairs of electrons around Xe while PF+4 has 4 pairs of electrons around P. F

+ –
O N O O N O O N O
NO2+ NO2 NO–2

NO–2
The molecule NO2
bonding electrons move away so as to reduce the repulsion between them.

F
F
3 is Cl

F
4.12 Complete Chemistry—JEE Main

11. There are six pairs of electron around I. These are oriented to make an octahedron. The two lane pairs occupy
axial positions. F F
13. Valence electrons of XeF4 = 8 + 4 ¥ 7 = 36. Lewis structure Xe
F F
There are 6 pairs of electrons around. These are directed to form octahedron. The four F atoms occupy apical
positions, therefore, the shape of XeF4 is square planar.
14. XeO3 XeO2F2 XeOF4 XeO3F2

8 + 3 ¥ 6 = 26 8 + 2 ¥ 6 + 2 ¥ 7 = 34 8 + 6 + 4 ¥ 7 = 42 8+3¥6+2¥

F F
O O F O F O F
Xe O Xe
Xe Xe
O O F
F F O O

OF F O F
F F
Xe Xe Xe Xe
O F O O O
O O O F F
Pyramidal Trigonal bipyramid Trigonal bipyramid

15. IF3 IF5 IF7


Valence electrons = 4 ¥ 7 =28 Valence electrons = 6 ¥ 7 = 42 Valence electrons = 8 ¥ 7 = 56
F F F
F F F F
I
I F I F
F F
F F F F

3 CO32– PCl3 NO–3


Valence electrons = 24 Valence electrons = 24 Valence electrons = 26 Valence electrons = 24

O Trigonal Cl Trigonal
planar P Cl planar
O
Cl
O

3-pairs of electrons
P
triagonal planar Cl Cl
Cl
Pyrimidal

17. NCO– NO–2 2 OF2

N C O O N O F O F
2-pairs of electrons 3-pairs of electrons 4-pairs of electrons
on C, inear structure on N, not linear on O
Chemical Bonding and Molecular Structure 4.13

18. Bond angle in NF3 is smaller than in NH3. Also bond angle in OF2 is smaller than in OH2. This is primarily due
to ionicity of N F and O
away from N and O nuclei. This causes expansion of lone pair causing bond angles F N F and F O F
smaller than those in H N H and H O 3 3 2
2

– +
C O.

SECTION 2 VB and MO Theories

Valence Bond Theory


The quantitative description of chemical bond is provided by the quantum mechanical theories. Two theories, namely,

The essential guidelines of VB method are as follows.


A molecule is considered to be a collection of atoms with electrons occupying their respective atomic orbitals.
The formation of molecule is analysed in terms of interactions amongst electrons-electrons, electrons-nuclei and
nuclei-nuclei.
For a molecule to be stable, the electrostatic attractions must predominate over the electrostatic repulsion. The
difference in these two is released in the form of heat. Thus, a molecule is energetically more stable than the
individual atoms.
Potential Energy Diagram
The potential energy diagram display the variation of potential energy during the formation of chemical bond between
two atoms when these are brought close to each other from a larger distance.
The description of potential energy variation of H2 A HB) is as follows.
At larger distance, DVinteraction A and HB exists.
As the atoms HA and HB are brought near to each other,
there occurs a net decrease in potential energy as the

B and electron B – nucleus A) are greater than the

B and nucleus A and nucleus B)


The decrease in potential energy is continued till the
atoms HA and HB are brought up to stable internuclear
distance.
As the atoms are brought more near to each other, an
increase in potential energy is observed as the repulsive
interactions become greater than the attractive
interactions.
These interactions are shown in Fig. 1 which explain the
formation of HA–HB bond at the stable internuclear distance.

Essential Criteria for the Formation of Covalent


Bond Fig. 1

If the formation of H2

2 molecule = 74 pm
A B) overlap each other in the bonded region.
4.14 Complete Chemistry—JEE Main

Electron associated with atom HA can go to the atom HB and vice versa through the overlap region. Also in the overlap

two electrons must have opposite spins.

internuclear distance.
2. Each of the two orbitals must possess one unpaired electron with opposite spin.

Fig. 2

Stablility of the Molecule The intervening electronic charge between the two nulcei has an affect of decreasing
nuclear repulsion and maximises electron-nuclei attractions. This lead to the stable H2 molecule.

Sigma an Pi Bonds
In a molecule, two types of overlapping of orbitals having directional characteristics may be distinguished.
End to End Overlap—Sigma Bond (s Bond) In this overlapping, the electronic charge is concentrated between

Sideways Overlap—Pi Bond (p Bond) In this overlapping, the electronic charge is concentrated above the

Fig. 3 Fig. 4
Illustrations
2 2 2 1 1.
Oxygen molecule x) y) z) There are two
2p atomic orbital, each containing one electron. Thus it can form two bonds s and p

Fig. 5
Chemical Bonding and Molecular Structure 4.15

2 2 1 1 1
Nitrogen molecule x) y) z) . There are
three 2p orbitals, each containing one electron. Thus it can form three bonds one s and two p

Fig. 6
The concept of Hydridization
According to the valence band theory.
2
4 , 2s2) should form no chemical bond as it does not contain any unpaired electron.
2 2 1
5 , 2s , 2p ) should form a single bond as it contains only one unpaired electron.
2 2 1 1
6 , 2s , 2px , 2py ) should form two bond as it contains two unpaired electrons.
Experimentally it if found that Be is divalent, B is trivalent and C is tetravalent,
To explain this, the concept of hydridization is introduce. In this concept we have
Two or more atomic orbitals of the same atom mix each other to provide a new set of identical number of degenerate
orbitals. These orbitals, known as hybrid orbitals, are completely identical in size, shape and orientations.
Kind of Hybridization
sp Hybridization One s orbital and one p oribtal of the same valence shell of an atom are mixed to provide

indicates the sign of wave function of the orbital.

Fig. 7
2 2
sp Hybridization In sp
valence shell of an atom combine to give three degenerate equivalent sp2 hybrid orbitals. These are oriented at an

Fig. 8
4.16 Complete Chemistry—JEE Main

sp3 Hybridization In sp3


valence shell of an atom combine to give four degenerate equivalent sp3 hybrid orbitals. These four orbitals are directed
¢

Fig. 9

Hybrid Orbitals involving d orbitals


sp2d or dsp2 Hybridization The resultant four

In dsp2, d orbital belongs to penultimate shell while in sp2d, it belong to the valence shell.
sp3d or dsp3 Hybridization

sp3d2 or d2sp3 Hybridization The resultant six hybrid orbitals are directed to the corners of regular octahedron

sp3d3 or d3sp3 Hybridization The resultant seven orbitals are directed to the corners of a regular pentagonal

xy, dxz and dyz.

Fig. 10 Fig. 11 Fig. 12 Fig. 13

Comment on Hybridization
The concept of hybridizaton follows from molecular geometry and not vice versa since the choice of appropriate

is known.
Chemical Bonding and Molecular Structure 4.17

A Few Illustrative Examples Involving Hybridization


Methane It involves sp3 Ethene It involves sp2

Fig. 15

Fig. 14
Ethyne Ammonia It involves sp3 hybrid orbitals of nitrogen.
¢

Fig. 16
Fig. 17

Water It involves sp3 ¢

Fig. 18
Note:
4.18 Complete Chemistry—JEE Main

[Ni(CN)4]2– It involves dsp2 hybrid orbitals of Ni2+ 2–

4s 4p
2+
28Ni

dsp2 hybridization Fig. 19

PCl5 It involves sp3

3s 3p 3d

15P

sp3d hybridization
Fig. 20

SF6 It involves sp3d2 21)


3s 3p 3d

16

sp3d2 hybridization

Fig. 21
Molecular Orbital Theory
Molecular orbital theory provides the explanation for the formation of bond in a molecule on the lines very similar to
those of atomic orbitals. The essential guidelines of this theory are as follows.
Like atomic orbitals in an atom, there exists molecular orbitals in a molecule. The only difference is that an atomic

around more than one nucleus and thus belongs to the molecule as a whole).

Electrons in a molecule occupy molecular orbitals in accordance with aufbau principle, Pauli’s exclusion principle
and Hund’s rule.

the electron.

Molecular Orbitals Built as a Linear Combination of Atomic Orbitals


Consider the formation and breaking of hydrogen molecule shown in Fig. 22.
a b) transform into molecular orbitals Y1 as the two atoms are brought near to each
other and vice versa.

Fig. 22
Chemical Bonding and Molecular Structure 4.19

Thus, molecular orbitals may be constructed as the linear combination of valence atomic orbitals.
a b), two combinations can be taken as shown in the following.
Y1 a b
Y2 a b
Thus, the linear combination of two atomic orbitals leads to two molecular orbitals. In general, the number of
molecular orbitals formed is always equal to the number of atomic orbitals being combined.
Bonding Molecular Orbital Consider the molecular orbitals Y1 a b
of electron in an orbital is described by the square wave function, we have
Y12 a)}
2
b)}
2
a b)}
)} 2 )} 2 give the probability of electron associated with atoms a and b, respectively. The term
a b
a b )} enhances the probability in the bonded region of the molecule, causing a decrease in the nucleus-
nucleus repulsion relative to increase in electron-nuclei attractions. This make the orbital more stable relative to those
of atomic orbitals. Hence, this orbital is known as bonding molecular orbital.
Antibonding Molecular Orbital For the molecular orbital Y2 a b), the probability distribution is
Y 22 a)}
2
b)}
2
a b)}

a b)} diminishes the probability in the bonded region of the molecule causing increase in
nuclear-nuclear repulsion relative to electron-nuclei attraction. This make the orbital less stable relative to those of
atomic orbitals. Hence, this orbital is known as antibonding molecular orbital.

Shapes of Molecular Orbitals Y1 and Y2 The shapes of molecular orbitals of Y1 and Y2 are as follows.
Bonding Molecular Orbital Y1 The positive combination of atomic orbitals in bonding molecular orbital is said to

Fig. 23
AntiBonding Orbital Y2 The negative combination of atomic orbitals in antibonding molecular orbital is said to be

Fig. 24

Energies of Molecular Orbitals Y1 and Y2 The energies of bonding orbital Y1 and antibonding orbital Y2 relative
a b) are shown in Fig. 25.
The decrease in energy for a bonding molecular orbital and increase in energy for an antibonding orbital depend on
extent of overlapping between the involved atomic orbitals. The larger the overlapping, the greater are the stability and
instability of bonding and antibonding orbitals, respectively.
4.20 Complete Chemistry—JEE Main

Fig. 25

Guidelines for Constructing Molecular Orbitals

difference of the two atomic orbitals.


Example
The extent of mixing decreases with the decrease in the
overlap of atomic orbitals placed at the stable internuclear
distance. If the overlap is zero, there is no mixing of
atomic orbitals.
Example z

Only the valence atomic orbital are considered for the


construction of molecular orbitals. The inner core atomic
orbitals are strongly bounded to their respective nuclei
and thus are considered as nonbonding orbitals.
Fig. 4.26
Example In the compounds of carbon, we have
1s2 2s2 2p1x 2p1y 2p z
considered as
nonbonding
orbital participate in the formation of
molecular orbital

Sigma and Pi Molecular Orbitals

Sigma Molecular Orbital In this orbital, electron density is concentrated symmetrically around the line joining
s.
Examples

Fig. 27
Chemical Bonding and Molecular Structure 4.21

Pi Molecular Orbital In this orbital, electron density is concentrated above and below the line joining the two
p.
Examples

Fig. 28

Designation of Molecular Orbital The designation of a molecular orbital starts by starting its s or p nature
followed by the atomic orbitals into which it separates at larger distance. The antibonding orbital is designated by
placing an asterisk on the symbol s or p.

Molecular Orbitals of Homonuclear Diatomic Molecules of Second Period The valence atomic orbitals to be
considered are
x y z
x y z

1. Combination Involving 2s(A) and 2s(B)

Fig. 29a
2. Combination Involving 2pz(A) and 2pz(B)

Fig. 29b
4.22 Complete Chemistry—JEE Main

3. Combination Involving 2px(A) and 2px(B)

Fig. 29c

4. Combination Involving 2py(A) and 2py(B)


The molecular orbitals formed are similar to p2px
and p*2px in the direction of y-axis.

Relative Energies of Molecular Orbitals-


Correlation
Diagram
diagram expected for the orbitals of atoms of second
period. In this diagram, E s E p2p) since the
end-to-end overlap of 2p orbitals is expected to be
larger than side-ways overlap.

found to be applicable only for O2 and F2 molecules.


The relative energies of molecular orbitals is
s s s2pz p2px = p2py p*2px = p*2py
s*2pz
For the molecules Li2 to N2, the energies difference

Es E p2p) Fig. 30 Correlation diagram for O2 and F2


The relative energies of molecular orbitals is
s s p2px = p2py s2pz p*2px = p*2py s*2pz

Electronic Structure and Molecular Characteristics


The following guidelines are helpful in describing molecular characteristics.
1. The valence electrons are distributed among the molecular orbitals in accordance with aufbau principle, Pauli’s
exclusion principle, and Hund’s rule.
Aufbau principle:
Pauli exclusion principle: No more than two electrons can occupy a single molecular orbital. The two electrons
have opposite spins.
Hund’s rule: The degenerate orbitals are singly occupied by electrons with parallel spins followed by double
occupancy.
Chemical Bonding and Molecular Structure 4.23

Fig. 31 Correlation diagram for Li2 to N2

bond order
one half of the net excess of bonding electrons, i.e.
Number of (bonding - antibonding) electrons
Bond order =
2
3. The strength of a bond depends on the bond order of the molecule. The larger the bond order, the stronger the
bond and larger the dissociation energy of the molecule.
4. Addition of an electron in the bonding orbital or removal of an electron form the antibonding orbital increases
bond order and hence increases stability of a molecule.

bond order and hence decreases stability of a molecule.


6. Paramagnetism in a substance is due to the presence of unpaired electrons in its molecules.
Explanation
Paramagnetic Substance A substance which is attracted towards a magnet is known as paramagnetic
substance.
Paramagnetism The phenomenon of attracting a substance towards a magnet is known

molecules of a substance.
Diamagnetic Substance A substance containing no unpaired electrons is known as diamagentic
substance. The pairing of electrons annule the tiny magnets internally.
7. Bond length is inversely related to the bond order.
4.24 Complete Chemistry—JEE Main

Molecular Characteristics
Species Valence Bond Order † Characteristics
electrons
H2 2 s1s)2 =1
Diamagnetic as there is no unpaired
2 electron
H+2 1 s1s)1 1 2, bond length increases,
=
2 2 bond dissociation energy decreases
Paramagnetic as there is one unpaired
electron.
He2 4 s1s)2 s*1s)2 2–2 Does not exist as bond order is zero
2
He+2 3 s1s)2 s*1s)1 2–1 1 Expected to exist, paramagnetic
=
2 2
Li2 2 s2s)2 =1
Diamagnetic
2
Be2 4 s2s)2 s*2s)2 2–2 Does not exist
2
B2 6 s2s)2 s*2s)2 p2px)1 p*2py)1 4–2
=1
Paramagnetic
2
C2 8 s2s)2 s*2s)2 p2px)2 p2py)2 6–2
=2
Diamagnetic
2
N2 s2s)2 s*2s)2 p2px)2 p2py)2 s2pz)2 8–2
=3 s and 2p
2 bonds)
Maximum bond dissociation energy
Minimum bond length, Diamagnetic
N2+ s2s)2 s*2s)2 p2px)2 p2py)2 s2pz)1 7–2 1 2 , bond dissociation
=2
2 2 energy decreases, bond length increases
Paramagnetic
N2– 11 s2s) 2 s*2s) 2 p2p x ) 2 p2p y ) 2 s2p z ) 2 8 – 3 1
=2 — same —
p*2px)1 2 2
O2 12 s2s) 2 s*2s) 2 s2p z ) 2 p2p x ) 2 p2p y ) 2 8 – 4 = 2 Paramagnetic as it contains two unpaired
p*2px)1 p*2py)1 2 electrons

O2+ 11 s2s) 2 s*2s) 2 s2p z ) 2 p2p x ) 2 p2p y ) 2 8 – 3 = 2.5 2, bond order is increased.
p*2px)1 2 Thus bond dissociation energy increases
and bond length decreases
Paramagnetic
Note: These effects are just opposite to those observed in going from N2 to N2+.
O2– 13 s2s) 2 s*2s) 2 s2p z ) 2 p2p x ) 2 p2p y ) 2 8 – 5 1 2, bond order is decreased.
=1
p*2px)2 p*2py)1 2 2 Thus, bond dissociation energy decreases
and bond length increases
Note: The species O2– is expected to be less stable than O+2 due to larger electron-electron repulsion.
F2 14 s2s) 2 s*2s) 2 s2p z ) 2 p2p x ) 2 p2p y ) 2 8 – 6 = 1 Diamagnetic
p*2px)2 p*2py)2 2
Ne2 16 s2s) 2 s*2s) 2 s2p z ) 2 p2p x ) 2 p2p y ) 2 8 – 8 Does not exist
p*2px)2 p*2py)2 s*2pz)2 2


Chemical Bonding and Molecular Structure 4.25

Heteronuclear Diatomic Molecules


Figure 32 displays the correlation diagram for heteronuclear
diatomic molecules. The sequence of energy levels is very

electronegative than the atom A.


Examples
NO
s2s)2 s*2s)2 p2px)2 p2py)2 s2pz)2 p*2px)1

NO+
s2s)2 s*2s)2 p2px)2 p2py)2 s2pz)2

lesser bond length.

Fig. 32

MULTIPLE CHOICE QUESTION ON SECTION 2

Identify the correct choice in the following questions.


1. The hybridization of chlorine orbitals in the compound ClF3 is
3 2 3 3d 2

2. The hybridization of iodine orbitals in the compound IF7 is


3 3d 2 2d 3 3d 3

3. The hybridization of nitrogen orbitals in the compound NO3– anion is


2 3 2
d
3
4. Which of the following ions does not involve sp hybridization?
– – 3– –
2 2 4 4
3 is
2 3 2
d
6. The hybridization of P in PO43– is
2 3 2
d
7. The hybridization of Xe in XeO3 is
2 3 2 2

8. The hybridization of Xe in XeOF4 is


3 3 2 2 3 4
d d
not correct?
4.26 Complete Chemistry—JEE Main

2–
2 ion, respectively are

11. The bond order of CN– is

12. Which of the following pairs of species has identical value of bond order?
+, N + –, O + and O+2
2 2 2 2
13. Which one of the following is expected to have maximum bond length?
+ 2+ –

14. Which of the following is expected to be diamagnetic?


+ +
2 2 2 2

– + – +
2 2 2 2
16. Which one of the following species will have maximum bond dissociation energy?
_
+ – 2
2 2 2 2
17. Which of the following statements regarding carbon monoxide is correct?

ANSWERS

HINTS AND SOLUTIONS


1. sp3d hybridization is involved.
3s 3p 3d

sp3d

2. sp3d3 hybridization is involved.


5s 5p 5d

sp3d3
3. NO3– has trigonal planar shape. It involves sp2 hybridization of N.
4. ICl–2 involves sp3 – ion is
5s 5p 5d

sp3d hybridization
Two single electrons form s bond with two chlorine atoms. There are three lone pairs of electrons.
Chemical Bonding and Molecular Structure 4.27

3 is
3s 3p 3d

sp2 hybridization Form double bonds with


Form s–bonds oxygen atoms
with oxygen
– to form 3– ion is
3s 3p 3d

sp3 hybridization Form double bonds


Form s–bonds with four with oxygen atoms
oxygen atoms

5s 5p 5d

sp3 hybridization These form 3 p–bonds


The unpaired electrons form s bonds with oxygen
with three oxygen atoms

5s 5p 5d

sp3d2 hybridization Form p–double bonds


The unpaired electrons form s–bonds with with oxygen
four F atoms and one oxygen atom

2–
2 s2s)2 s*2s)2 s2pz)2 p2px)2 p2py)2 p*2px)2 p*2py)2
Bonding electrons = 8 Antibonding electrons = 6
11. Valence electrons in CN– s2s)2 s*2s)2 p2px)2 p2py)2 s2pz)2
(Bonding - antibonding) electrons 8 - 2
Bond order = = =3
2 2
12. Valence electrons of N2 s2s)2 s*2s)2 p2px)2 p2py)2 s2pz)2

Valence electrons of NO+ s2s)2 s*2s)2 p2px)2 p2py)2 s2pz)2

Valence electrons of CN– +

Valence electrons of N–2


Valence electrons of O2+ s2s)2 s*2s)2 s2pz)2 p2px)2 p2py)2 p*2px)1

s2s) s*2s) p2px) p2py) s2pz) p*2px)


+
NO
NO2+
4.28 Complete Chemistry—JEE Main

NO–
– will having maximum bond length.

15. Bond orders are O–2 2


+
2
– +
2
16. C2 s 2s) s*2s) p*2px) p2py)
2 2 2 2
+
C2
C–2 s2s)2 s*2s)2 p2px)2 p2py)2 s2py)1
2–
C2
Larger the bond order, larger the dissociation energy.
– +
17. The CO molecule is represented as C ¨ O
Dipole moment is low due to pull of bonding electrons towards more electronegative oxygen atom.

MULTIPLE CHOICE QUESTIONS ON THE ENTIRE CHAPTER

General Characteristics
1. Which of the following molecules does not follow the octet rule?
2 3 2
2. In which one of the following compounds does hydrogen bonding occur?

3. Which of the following compounds would show evidence of the strongest hydrogen bonding?

4. A covalent bond is most likely to be formed between two elements which

5. A covalent bond is formed between

6. An electrovalent bond is formed between

7. Which of the following orbitals of a diatomic molecule AB oriented along x-axis will not have positive overlap?
x x x z
8. Which of the following occurs when two hydrogen atoms bond with each other?

3 2 2 2O

is about

11. Which of the following unit conversion of dipole moment is correct?


Cm
D
Chemical Bonding and Molecular Structure 4.29

12. Which of the following unit conversion is correct?


¥ –12 ¥ –14 C m
¥ –16 ¥ –18 C m
13. o-Nitrophenol is more volatile than p-nitrophenol. It is due to
o o-nitrophenol
p p-nitrophenol
14. The maximum number of hydrogen bonds that can be formed by water molecules is

15. Which of the following interactions is not a part of van der Waals interactions?

16. Which one of the followings contributes maximum to the van der Waals interactions?

VSEPR and Hybridization

5 which contains 5 bonding pairs and one lone pair has a shape of

21. Hybridization involves

4 molecule involves
2 3 3d orbitals
4 ]2– is

24. The geometry arrangement of F atoms around P in PF5 is

25. The geometrical arrangement of NH3 3+


3) 6] is

26. The sp3d hybridization of central atom of a molecule would lead to

27. The sp3d2 hybridization of central atom of a molecule would lead to

28. The bond angle in PH3 is expected to be


4.30 Complete Chemistry—JEE Main

+
2, NO3– and NH 4+ respectively are
2, sp3 3, sp2 2, sp, sp3 3, sp2, sp

2– 2–
4 4 4 2Cl2
31. Which of the following species involves sp2 hybridization?
2 2 2 2
32. The species ClO2– involves
2 3 2 hybridization
33. Which of the following species involves sp3 hybridization?

3 3 3 3
34. The species ClO3– involves
2 3 3 hybridization
35. Which of the following statements is correct?
3 involves sp3 hybridization and NF3 involves sp2 hybridization
3 involves sp2 hybridization and NF3 involves sp3 hybridization
3 and NF3 involve sp3 hybridization
3 and NF3 involve sp2 hybridization
36. The hybridization in OF2 is
2 3 2

2 is 145 pm and its bond moment is 5.8 ¥ C m. Its per cent ionic character
is about

38. Which of the following species involves the smallest bond angle?
3 2 2 4

2 2 2 2
Molecular Orbitals
not true?

between different atoms are considered

41. Ethyne molecule contains


bonds and 1 bond
bonds and 2 bonds and 3 bonds
42. Which of the following molecules is paramagnetic?
2 2 2 2
43. In which of the following species, the bond length is expected to be the greatest?
2– – +
2 2 2 2
44. The bond order of O2 molecule is

45. The bond order of N2 molecule is


Chemical Bonding and Molecular Structure 4.31

46. The number of molecular orbitals obtained by mixing two atomic orbitals from each of the two atoms is

47. Which of the following order of energies of molecular orbitals of N2 is correct?


E 2px) = E 2py E E *2px) = E *2py)
E 2px) = E 2py E E *2px) = E *2py)
E 2px) = E 2py E E *2px) = E *2py)
E 2px) = E 2py E E *2px) = E *2py)
48. Which of the following order of energies of molecular orbitals of O2 is correct?
E 2px) = E 2py E E *2px) = E *2py)
E 2px) = E 2py E E *2px) = E *2py)
E 2px) = E 2py E E *2px) = E *2py)
E 2px) = E 2py E E *2px) = E *2py)
not correct from the viewpoint of molecular orbital?
2 is not a stable molecule
+
2 is not stable but He 2 is expected to exist
2 is maximum amongst the homonuclear diatomic molecules belonging to the second period
2 molecule is
E E E 2px) = E 2py E E *2px) = E *2py E *2p)

– + – + – + – +
2 2 2 2 2 2 2 2 2 2 2 2
51. Which of the following orders regarding the bond length is correct?
– + – + – + – +
2 2 2 2 2 2 2 2 2 2 2 2
52. Which of the following facts regarding change in bond length is correct?
+ +
2 to N2 , decreases in going from O2 to O2
+ +
2 to N2 , increases in going from O2 to O2
+ +
2 to N2 and O2 to O2
+ +
2 to N2 and O2 to O2
53. Which of the following orders is correct for the bond dissociation energy of O2, O2+, O2– and O22–?
+ – 2– + – 2–
2 2 2 2 2 2 2 2
+ – 2– + – 2–
2 2 2 2 2 2 2 2
54. Which of the following species has the shortest bond length?
+ – 2–
2 2 2 2
55. Which of the following species has the largest bond length?
+ – 2–
2 2 2 2

ANSWERS
4.32 Complete Chemistry—JEE Main

HINTS AND SOLUTIONS


1. Boron in BCl3 does not satisfy octect rule. There are 6 electrons around B.
2. Hydrogen attached to the electronegative nitrogen shows hydrogen bonding.
3. Larger the number of —OH groups, the more extensive the hydrogen bondings.
4. Atoms having similar electronegativity form covalent bond.

6. An electrovalent bond is formed between electropositive and electronegative atoms


z
8. Potential energy is lowered.

(1.03 ¥ 3.3356 ¥ 10-30 C m)


= ¥ 100 = 16.9%
(1.602 ¥ 10-19 kg)(127 ¥ 10-12 m)

11. 1 D = 3.3356 ¥ C m.
12. 4.8 ¥ ¥ C. Hence
Ê 1.602 ¥ 10-19 C ˆ -2
1 esu cm = Á (10 m) = 3.3356 ¥ –12 C m
Ë 4.8 ¥ 10-10 esu ˜¯
13. o–Nitrophenol involves intramolecular whereas p-nitrophenol involves intramolecular hydrogen bondings.
14. Water molecule is involved in four hydrogen bondings.
15. Ion-dipole interaction is not counted in van der Waals forces.
16. Dispersion forces have maximum contribution.
17. Four lone pairs of electrons lead to square planar arrangement.
18. Three lone pairs of electrons lead to triangular arrangement.

21. Hybridization involves mixing atomic orbitals of the same atom.


Cl Cl
4 ¥ 7) electrons. These are distributed as
Cl Cl
3d

3d 4s 4p
2+
23. Ni ion
in complex
dsp2 hybridization
F F
24. PF5 ¥ 5 ¥ 7) valence electrons. These are distributed as P
F F
F
Chemical Bonding and Molecular Structure 4.33

25. There are 6 ligands attached to Cu. There arrangement will be octahedron.
26. sp3d hybridization leads to trigonal bipramide geometry.
27. sp3d2 hybridization leads to octahedral geometry.
28. PH3 ¥ 1) valence electrons. There are distributed as H P H. There are 4 pairs of electrons around
H
P.
+
has 16 valence electrons. There arrangement will be O N
2 O . Two pairs of electrons around N implies

linear geometry involving sp hybridization. NO 3 ¥ 3 ¥ 6 + 1) valence electrons. These are arranged
as O N O . There are three pairs of electrons around N. Its geometry will be trigonal planar and will result
O
from sp2 hybridization. NH +4 has sp3 hybridization.
2– 2–
4 4 2Cl2 have 4 pairs of electrons around the central atoms which lead to tetrahedral geometry.
4
3
2 hybridization is
involved.
32. ClO–2 has four pairs of electrons around Cl which leads to tetrahedral geometry, hence sp3 hybridization is
involved.
33. Only PCl3 has 26 valence electrons and P has four pairs of electrons around it which leads to tetrahedral geometry
to PCl3. Hence, P involves sp3 hybridization.
34. The species ClO3– has 26 valence electrons with four pairs of electrons around Cl leading to tetrahedral geometry,
hence sp3 hybridization
35. BF3 has 24 valence electrons with three pairs of electrons around B leading to trigonal planar geometry, hence sp2
hybridization.
NF3 has 26 valence electrons with four pairs of electrons around N leading to tetrahedral geometry, hence sp3
hybridization.
3
36. OF2
hybridization.
5.8 ¥ 10 -30 C m
37. Per cent ionic character = ¥ 100 = 25%
(1.602 ¥ 10 -19 C)(145 ¥ 10 -12 m)
38. H2O will have minimum bond angle due to repulsion between lone pair-bonding pair electrons. It will be smaller
than NH3 as the latter has only one lone pair while H2O has two lone pairs.
2
2 involves sp

s-bonds and two p-bonds.


42. Only O2 has two unpaired electrons and hence paramagnetic.
43. The bond order of O2–
2 is the minimum and thus it has largest bond distance.
44. The bond order of O2
45. The bond order of N2
46. These will be as many molecular orbitals as the number of atomic orbitals being mixed.
4.34 Complete Chemistry—JEE Main

54. N2 has the shortest bond length as its bond order is largest.
55. N2–
2

MULTIPLE CHOICE QUESTIONS FROM AIEEE AND JEE MAIN

1. The pair of species having identical shapes for molecules is


5, IF5 4 4 2, CO2 BF3, PCl3 [2003]
2. Which of the following pair of molecules will have permanent dipole moments for both members?
2 and CO2 4 and NO2 2 and CO2 2 and O3 [2003]
3. Which one of the following compounds has the smallest bond angle in its molecule?
3 2 2O 2 [2003]
2 3 , BF 3 4 is
2 4 3 3 3 2 4 3
2 3 4 3 2 3 3 4 [2004]
5. +
The bond order in NO is 2.5 while that in NO is 3. Which of the following statements is true for these two species?
+ +
+ is equal to that in NO [2004]
3BO3) are respectively
2 and sp2 2 and sp3 3 and sp2 sp3 and sp3 [2004]
7. Which one of the following has the regular tetrahedral structure?
2–
4 4 4 4] [2004]

3 3 2 hybridization sp3d2 hybridization


[2004]

+ – + [2005]
2 2 2 2

[2005]
11. Based on lattice energy and other consideration which one of the following alkali metal chlorides is expected to
have the highest melting point?
[2005]
4 , CF 4 and XeF 4 are

[2005]
13. The number and type of bonds between two carbon atoms in calcium carbide are

[2005][2011][2014]

[2005]

2– + [2006]
2 2 2 2
Chemical Bonding and Molecular Structure 4.35

not equal?

4 4 4 4 [2006]
17. In which of the following ionization processes, the bond order has increased and the magnetic behaviour has
changed?
2 Æ C2 Æ NO+ 2 Æ O2 2 Æ N2
+ + +
[2007]
18. Which of the following hydrogen bonds is the strongest?
H… H… H…O O H… F [2007]

2– +
2 2 2 O [2007]

the increasing order of the polarizing power of the cationic species, K+, Ca2+, Mg2+, Be2+?
2+ 2+ + 2+ 2+ + 2+ 2+
+ 2+ 2+ 2+
Ca2+ 2+ 2+ +
[2007]
21. Which one of the following pairs of species have the same bond order?
+
and CN+ –
and NO+ –
and CN+ O2– and CN– [2008]
–1 –1
22. The bond dissociation energy of B—F in BF3 is 646 kJ mol whereas that of C—F in CF4 is 515 kJ mol . The

p – pp interaction between B and F in BF3 whereas there is no possibility of such interaction


between C and F in CF4.
p – pp interaction between B and F in BF3 than that between C and F in CF4.

s bond between B and F in BF3 as compared to that between C and F in CF4. [2009]
23. Using MO theory predict which of the following species has the shortest bond length?
2– 2+ O2+ [2009]
2 2 2
24. Among the following maximum covalent character is shown by the compound
2 2
2 AlCl3 [20011 (Cancelled)]
25. Which of the following has maximum number of lone pairs associated with Xe?
3 4 6 XeF2 [2011]
26. The structure of IF7 is

[20011 (Cancelled)]
27. The hybridization of orbital of N atom in NO3–, NO2+, and NH+4, respectively
2, sp3 2, sp3
2, sp, sp3 3, sp2 [20011 (Cancelled)]
28. The molecule having smallest bond angle is
3 3 3 3 [2012]

2– and NO–3 and Br5 3– [2013]


3 3 4 5 6 6

2 2 2 2 [2013]
not likely to exist?
+, He22– –, He22– 2+, He2 – He2+
2 2 2 2, [2013]
– Li+2
2, Li 2 and
+ – Li+2 – – + Li–2 + [2013]
2 2 2 2 2 2 2 2 2 2
33. Which one of the following properties is not shown by NO?

It combines with oxygen to form nitrogen dioxide [2014]


4.36 Complete Chemistry—JEE Main

– + O22– [2014, online]


2 2 2
35. –
The correct order of bond dissociation energy among N2, O2, O 2 is shown in which of the following arrangements?
– – – O2 –
2 2 2 2 2 2 2 2 2 2 2
[2014, online]
not correctly stated?
7 5
3 ICl3 [2014, online]
3H 4
3 2 2 sp2 and sp3
[2014, online]
38. Which of the following molecules is paramagnetic?
2 O3 [2014, online]

Assertion 2, electron density is increased between the nuclei


Reason The bonding MO is yA + yB, which shows the destructive interference of combining electron waves

[2015, online]
4

e ¥ esu) [2015, online]

42. Choose the incorrect formula out of the four compounds for an element X below.
2Cl3 2O 3 2 4) 3 4 [2015, online]
43. The group of molecules having identical shape is
5, IF5, XeO2F2 3, PCl3, XeO3
+
4, XeF4, CCl4 3, XeOF2, XeF 3 [2016, online]
2 2 6 2 6?

2 [2016, online]

3 4 3 2O [2016, online]
46. The species in which the N atom is in a state of sp hybridization is
– – + [2016]
2 3 2 2

ANSWERS
Chemical Bonding and Molecular Structure 4.37

HINTS AND SOLUTIONS


1. Both XeF2 and CO2 have linear structures.
2. Both NO2 and O3 have bent structure. Their dipole moment will not be zero.
3. The bond angles of NH3 2, H2O and H2 ¢
4. Bond angle is H2 3 ¢
4 ¢. Bond angle in BF3 is

5. The bond order and bond length have inverse dependence, i.e. the larger the bond order, shorter the bond length.
+

6. Boric acid is a planar molecule as boron involves sp hybridization and oxygen involves sp3 hybridization.
2

7. XeF4 ¥ 7) valence electrons. These are distributed as shown in Fig. 1a. There are six pairs of
electrons around Xe. These are distributed octahedrally. Thus, the shape of XeF4 is shown in Fig. 1b.

F F F F
Xe xe
F F
F F
Fig 1a Fig 1b

4 ¥ 7) electrons. These are distributed as shown in Fig. 2a.

F
F F F

F F F F
Fig 2a Fig 2b

BF–4 ¥ 7 + 1) electrons. These are distributed as shown in Fig. 3a. There are four pairs of electrons
around B. They assume tetrahedral geometry. The shape of BF–4

F
F F
B B
F
F F F F
Fig 3a Fig 3b
2–
4] , we have
3d 4s 4p
28Ni atom

Ni2+ ion

Ni2+ ion is complex

dsp2 hybridization
2
2– NC CN
4] is shown in Fig. 4.
Ni
Thus BF–4 has the regular tetrahedral structure.
NC CN
Fig. 4
4.38 Complete Chemistry—JEE Main

3d 2 hybridization. There
2, dsp3 sp3d2
are 4, 6 and 12 such angles in dsp and hybridizations, respectively.

H2+ s1s)1 He+2 s1s)2 s*1s)1


H2–
s1s) s*1s)
2 1 H2 s1s)2
A diamagnetic substance does not possess unpaired electron. Dihydrogen does not possess unpaired electron;
hence it is diamagnetic.

11. Melting point of LiCl is less than that of NaCl and later it generally decreases on descending the group. The low
melting point of LiCl is due to its covalent character.
4 = 6 + 4 ¥ 7 = 34. These are distributed as follows.

F F
4 will be trigonal bipyramide with

F F

Number of Valence electrons in CF4 = 4 + 4 ¥ 7 = 32. These are distributed as follows.


F F
There are four paired of electrons around C. The shape of CF4 is tetrahedral. There is no lone
C
pair of electrons on the C atom.
F F

Obviously the choice b is correct. The molecule of XeF4 will have different geometry with two lone pairs of
¥ 7) valence electrons and these are distributed as follows.

F F
Xe The shape of XeF4 will be square planar.
F F

13. The carbide in calcium carbide is [C C]2–, which involves one sigma and two pi bonds.
3PO2) is
O

There are two hydrogen atoms attached to P atom. H P H

OH

O 2; s2s)2 s*2s)2 s2pz)2 p2px)2 p2py)2 p*2px)1 p*2py)1


O22–; s2s)2 s*2s)2 s2pz)2 p2px)2 p2py)2 p*2px)2 p*2py)2
B 2; s2s)2 s*2s)2 p2px)1 p2py)1
N2+; s2s)2 s*2s)2 p2px)2 p2py)2 s2pz)1

No. of valence electrons Lone pairs around the Arrangement of lone pairs
central atom
BF–4 32 4 Tetrahedral
4 34 5 Trigonal bipyramide
4 32 4 Tetrahdral
XeF4 36 6 Octahedral
geometry: square planar)
Chemical Bonding and Molecular Structure 4.39

4, all bonds will not be identical as its structure is


F F

F
F
17. The order of increasing energy of molecular orbitals is
s2s) s*2s) p2px) p2py) s2pz) p*2px p*2py) s*2pz)
The valence electrons in C2, NO, O2 and N2

s2s)2 s*2s)2 p2px)2 p2py)2 s2pz)2 p*2px)1


In converting NO to NO+, the electron is removed from antibonding molecular orbital and the resultant ion
possesses no unpaired of electrons.
18. Larger the electronegativity of atom, stronger the hydrogen bond.

O22–; s2s)2 s*2s)2 s2pz)2 p2px)2 p2py)2 p*2px)2 p*2py)2


O2+ and NO s2s)2 s*2s)2 s2pz)2 p2px)2 p2py)2 p*2px)1
O 2; s2s)2 s*2s)2 s2pz)2 p2px)2 p2py)2 p*2px)1 p*2py)1
2–
Hence, O2 is diamagnetic as it does not contain unpaired electron.

of the cation.

NO+ –

CN+ 4+5–1=8 O –2 6 + 6 + 1 = 13
+ and CN– have the same number of valence electrons and thus are expected to have the same
The species NO

p –pp interaction between B and F due to back bonding of lone pair of electrons on F to
an empty p orbital on B. There is no such interaction in CF4.
F
B F
F

O22+ s2s)2 s*2s)2 s2p)2 p2px)2 p2py)2; BO = 3


O2+ KK s2s) s*2s) s2p) p2px) p2py) p*2px) ;
2 2 2 2 2 1
BO = 2.5
O2– KK s2s)2 s*2s)2 s2p)2 p2px)2 p2py)2 p*2px)2 p*2py)1; BO = 1.5
O22– KK s2s)2 s*2s)2 s2p)2 p2px)2 p2py)2 p*2px)2 p*2py)2
The bond length is inversely proportional to the bond order. Hence O22+ is expected to have shortest bond length.

hence larger covalent character to the bond between cation and anion.
25. We have
1
XeO3 ¥ 6 = 26 Lone pair of electrons around Xe = ¥ 8) =1
2
1
XeF4 ¥ 7 = 36 Lone pair of electrons around Xe = ¥ 8) =3
2
1
XeF6 ¥ Lone pair of electrons around Xe = ¥ 8) =1
2
1
XeF2 ¥ 7 = 22 Lone pair of electrons around Xe = ¥ 8) =3
2
4.40 Complete Chemistry—JEE Main

¥ 7) valence electrons in IF7. There with be 7 pairs of electrons around iodine. These will be
oriented in the directions of pentagonal bipryamid as shown in the following structure.
F
F F
I F
F
F
F
27. The ion NO–3 ¥ 6 + 1) valence electrons. These will be distributed as

O N O There are 3 electron-pairs is around N. The shape of NO3– will be trigonal planar. Hence sp2
O hybridization of N orbitals is involved.

The ion NO+2 ¥ 6 – 1) valence electrons. These will be distributed as


+ There are 2 electron pairs around N. The shape of NO+2 will be linear. Hence, sp hybridization of
O N O N orbitals is involved.

It can be shown that the NH+4 involves sp3 ¥


1 – 1). These are distributed as
H +
These are four electron-pairs around N. The shape of NH4+ will be tetrahedral. Hence, sp3 hybridization
H N H
of N orbitals is involved.
H
28. The bond angle Cl A
increase in the size of the central atom A. Thus, out of NCl3, PCl3, AsCl3 3, the latter is expected to have
the smallest bond angle.
5 and BrF5), the number of paired of electrons around the central atoms of the two compounds
in each pair are identical. Hence, the compounds in each pair have identical structures. In PF5 and BrF5, the number

F
F F F
F P Br
F F F
F F
Five paired of electrons around P, dsp3 3 2
d
hybridization of P orbitals hybridization of Br orbitals

C2 s2s)2 s*2s)2 p2px)2 p2py)2


N2 s2s)2 s*2s)2 p2px)2 p2py)2 s*2pz)2
O2 s2s)2 s*2s)2 s2pz)2 p2px)2 p2py)2 p*2px)1 p*2py)1
2 s3s)2 s*3s)2 p3px)2 p3py)2 s*3pz)2 p*3px)1 p*3py)1
The molecules C2 and N2 do not possess unpaired electrons. Hence, these are expected to exhibit diamagnetic
behaviour.
31. The species H22+ 2
+ –
2, Li 2, Li 2 and their bond orders are
Li2 s2s) 2 Li 2 s2s)
+ 1 Li–2 s2s)2 s*2s)1

The species Li–2 will have larger electronic repulsion as compared to Li+2 and thus will be less stable.
+ –
Thus, the stability order is Li2 2 2.
Chemical Bonding and Molecular Structure 4.41

s2s)2 s*2s)2 p2px)2 p2py)2 s2p)2 p*2px)1


Molecule is paramagnetic as it contains one unpaired electron.
(Bonding - antibonding) electrons 8 - 3 1
Bond order = = =2
2 2 2
NO is a neutral oxide. It combines with O2 to give NO2.

N2 s2s)2 s*2s)2 p2px)2 p2py)2 s2pz)2


N22+ s2s)2 s*2s)2 p2px)2 p2py)2
O2– s2s)2 s*2s)2 s2pz)2 p2px)2 p2py)2 p*2px)2, p*2py)1
O22– s2s)2 s*2s)2 s2pz)2 p2px)2 p2py)2 p*2px)2 p*2py)2

N2 s2s)2 s*2s)2 p2px)2 p2py)2 s2pz)2; BO = 3


O2 KK s2s) s*2s) s2pz) p2px) p2py) p*2px) p*2py) ; BO = 2
2 2 2 2 2 1 1

O2– KK s2s)2 s*2s)2 s2pz)2 p2px)2 p2py)2 p*2px)2 p*2py)1


The larger the bond order, larger the dissociation energy.
F
36. IF7; Valence electrons = 7 + 7 ¥ 7 = 56 F F
F I
F F
F
F F
BrF5; Valence electrons = 7 + 5 ¥ 7 = 42 F F F F
Br Br
F F F F

F Five pairs of electrons around Br F


BrF3; Valence electrons = 7 + 3 ¥ 7 = 28 F Br
F
Br
F
F
Cl Cl Cl
ICl3 I I Planar dimer
Cl Cl Cl

1 2 1
37. The allene is H2C C CH2. Carbon labelled as 1 are sp2 hybridized and carbon labelled as 2 is sp
hybridized.
38. The molecule NO contains odd number of valence electrons. It is expected to be paramagnetic.

4 ¥ 7). These are distributed as follows.


F F
Xe
F F
O
There are 6 pairs of electrons around Xe. Their orientations will be square bipyramidal. The geometry of XeOF4
4.42 Complete Chemistry—JEE Main

is square pyramid as the lone pair of electrons occupy apical position


F F
Xe
F F
O
p = qr, we have
p 0.38 ¥ 10-18 esu cm
q= = = 2.35 ¥ –11 esu
r 1.617 ¥ 10-18 cm
2.35 ¥ 10-11 esu
Fractional charge on each atom =
4.802 ¥ 10-10 esu
42. The incorrect formula is X2Cl3.
43. Only the species given in the choice d have the same number of valence electrons of 28.
+
F F F
F Cl O Xe F Xe
F F F

lone pair of electrons occupy the equatorial positions giving T-shape to the molecules.
44. Na+ and F– 2 2 6.

K+, Cl– and Ca2+ 2 2 6 2 6.

45. CH4
pair-bonding pair repulsion, the angle H — X — H is decreased from the expected bond angle of tetrahedral
¢.

46. Species Number of valence Distribution of valence Number of lone pairs in Hybridization
electrons electrons central atom
-
NO–2 5 + 2 ¥ 6 + 1 = 18 3 sp2
O N O
-
NO–3 5 + 3 ¥ 6 + 1 = 24 3 sp2
O N O
O
NO2 5 + 2 ¥ 6 = 17 O N O
NO+2 5 + 2 ¥ 6 – 1 = 16 + 2 sp
O N O

Thus, N is NO+2 has two lone pairs of electrons around it and is thus sp hybridized.
5
Solutions

SECTION 1 Composition of a Solution

Solutions are homogeneous mixtures of two or more than two constituents. The constituent present in largest amount
is called the solvent while those present in lesser amounts are known as solutes†.

Composition of a solution In order to deal with a solution, it is necessary to know its composition. This may be
expressed in the following ways.
(i) Mass percentage The mass fraction of B in solution is
Mass of B in solution
wB = (1)
Total mass of solution
Mass percentage of B = wB × 100
(ii) Volume percentage The volume fraction of B in a solution is
Volume of liquid B
fB = (2)
Volume of solution
Volume percentage of B = fB ¥ 100
Note: Total volume of the solution may or may not be equal to the sum of volumes of its constituents depending
upon the fact whether the solution is ideal or not.
(iii) Mass by Volume Percentage For this, we have
Mass of constituent B
Mass by volume percentage of B = × 100 (3)
Volume of solution
(iv) Parts per million For this, we have
Number of parts of B
Parts per million of B = ¥ 106 (4)
Total number of parts of all constituennts
It is abbreviated as ppm. Parts per million may refer to mass to mass, volume to volume and mass to volume
way of expressing composition.
(v) Amount (or Mole) Fraction
Amount of constituent B in the solution nB n
xB = = = B (5)
Total amount of consstituents in the solution  B nB ntotal

† Throughout this unit, the solvent is represented by the subscript 1 and solute by the subscripts 2,3,....., and so on.
5.2 Complete Chemistry—JEE Main

The sum of amount (or mole) fractions of constituents in a solution is 1.


Ê n ˆ Â n
 B xB =  B Á B ˜ = B B = 1 (6)
Ë Â B nB ¯ Â B nB
Amount (or mole fraction) is independent of temperature.
(vi) Molarity
Amount of solute in solution n
M= 3
= 2 (V expressed in dm3, i.e. L) (7)
Volume of solution expressed inn dm (i.e. L) V
The unit of molarity is mol dm–3 ( i.e. mol L–1). It is often represented by the symbol M. The molarity of a
solution depends on temperature owing to the variation of volume with temperature.
(vii) Molality
Amount of solute n
m= = 2 (m1 expressed in kg) (8)
Mass of solvent expressed in kg m1
The molality carries the unit of mol kg–1 . It is independent of temperature.
Illustration Calculate the mole fraction of solute, molality and molarity of a solution containing 24 mass per cent of
acetic acid. The density of solution is 1.05 g cm–3.
In 100 g of solution, we have
Mass of acetic acid, m2 = 24 g; Mass of water, m1 = 76 g = 0.76 kg
m2 24 g m 76 g
Amount of acetic acid, n2 = = -1
= 0.40 mol ; Amount of water, n1 = 1 = = 4.22 mol
M 2 60 g mol M1 18 g mol-1
n2 0.40 mol
Mole fraction of acetic acid is x2 = = = 0.087
n1 + n2 4.22 mol + 0.40 mol
n2 0.40 mol
Molality of solution is m= = = 5.26 mol kg -1
m1 0.076 kg
To calculate molarity, we need to know volume of the solution, which is
Mass of solution 100 g
V= = = 95.24 cm3
Density of solution 1.05 g cm -3
n 0.40 mol
Molarity of solution is M = 2 = 3
= 4.20 mol dm -3 (i.e. 4.20 mol L–1 or 4.20 M)
V 0.09524 dm
Interconversion Expressions
Let we represent
Amount of solute = n2 Molar mass of solute = M2 Mass of solute, m2 = n2M2
Amount of solvent = n1 Molar mass of solvent = M1 Mass of solvent, m1 = n1M1
m1 + m2 n1M1 + n2 M 2
Density of solution = r Volume of solution, V = =
r r
n2
Molarity of solution, M = ; (V expressed in dm3) (9)
V
n2
Molality of solution, m = ;(m1 expressed in kg) (10)
m1
n2
Mole fraction of solute, x2 = (11)
n1 + n2
Molarity into Molality and Vice Versa We have
n2 n2 n2 / n1 M1 m mr
M= = = = = (12)
V (n1 M1 + n2 M 2 ) / r [1 + (n2 / n1 M1 ) M 2 ] / r (1 + mM 2 ) / r 1 + mM 2
Solutions 5.3

M
Rearranging Eq. (12), we get m= (13)
r - MM 2
Molarity into Mole Fraction and Vice Versa We have
n2 n2 (n2 / n1 ) r
M= = =
V (n1M1 + n2 M 2 ) / r M1 + (n2 / n1 ) M 2
n2 MM1
This gives =
n1 r - MM 2
n2 n /n MM1 / ( r - MM 2 ) MM1
Now x2 = = 2 1 = = (14)
n1 + n2 1 + n2 / n1 1 + MM1 / ( r - MM 2 ) M ( M1 - M 2 ) + r
x2 r x2 r x2 r
Rearranging Eq. (14), we get M = = = (15)
M1 - x2 ( M1 - M 2 ) (1 - x2 ) M1 + x2 M 2 x1M1 + x2 M 2

Molality into Mole Fraction and vice versa We have


n2 (n2 / n1 ) n /(m1 / M1 ) (n2 / m1 ) M1
x2 = = = 2 =
n1 + n2 1 + (n2 / n1 ) 1 + n2 /(m1 / M1 ) 1 + (n2 / m1 ) M1
mM1
which in view of Eq. (10) becomes x2 = (16)
1 + mM1
x2 x
Rearranging Eq. (16), we get m= = 2 (17)
M1 (1 - x2 ) x1M1

MULTIPLE CHOICE QUESTIONS ON SECTION 1

1. An antifreeze solution is prepared by mixing 217.0 g of ethylene glycol ( C2H6O2) with 200 g water. The molality
of solution is
(a) 15.5 mol kg–1 (b) 17.5 mol kg–1 (c) 18.5 mol kg–1 (d) 20.1 mol kg–1
2. If the density of a solution containing 217.0 g ethylene glycol ( C2H6O2) in 200 g water is 1.10 g cm–3, the molarity
of the solution is
(a) 8.8 mol dm–3 (b) 10.1 mol dm–3 (c) 9.23 mol dm–3 (d) 12.12 mol dm–3
3. A solution of glucose (C6H12O6) is 18% by mass. The mole fraction of glucose will be
(a) 0.18 (b) 0.25 (c) 0.0214 (d) 0.015
4. The mass of an equimolar mixture of Na2CO3 and NaHCO3 is 1.0 g. The volume of 0.1 M solution of HCl required
to react completely with this mixture is
(a) 52.1 mL (b) 104.2 mL (c) 125.2 mL (d) 156.3 ml
5. A solution contains 441.0 g of H2SO4 in 1 L of solution. If the density of solution is 1.25 g mL–1, the molarity and
molality of the solution, respectively, are
(a) 4.5 mol L–1 and 5.56 mol kg–1 (b) 4.5 mol L–1 and 4.5 mol kg–1
(c) 5.56 mol L–1 and 4.5 mol kg–1 (d) 5.56 mol L–1 and 5.56 mol kg–1
6. A solution of HCl is prepared by mixing 60 mL of 0.12 M HCl and 40 mL of 0.25 M HCl solution. The molarity
of HCl in the resultant solution is
(a) 0.185 M (b) 0.150 M (c) 0.20 M (d) 0.172 M
7. The mole fraction of glucose in water is 0.2. The molality of the solution will be
(a) 10.2 mol kg–1 (b) 12.4 mol kg–1 (c) 14.2 mol kg–1 (d) 16.6 mol kg–1
–1
8. The density of 40% by mass of acetic acid is 1.25 g mL . The molarity of the solution is
(a) 5.2 M (b) 6.8 M (c) 7.5 M (d) 8.33 M
5.4 Complete Chemistry—JEE Main

9. Mass of Mohr’s salt (molar mass 392 g mol–1) required to prepare 250 mL of 0.15 M solution is
(a) 9.8 g (b) 12.7 g (c) 14.7 g (d) 16.8 g
10. The mole fraction of glucose in 0.15 M solution (density = 1.10 g mL–1) is
(a) 2.40 ¥ 10–3 (b) 2.0 ¥ 10–3 (c) 2.82 ¥ 10–3 (d) 2.40 ¥ 10–2
11. The mole fraction of methanol (CH3OH) in aqueous solution is 0.10. If density of the solution is 0.97 g mL–1, the
molarity of the solution will be
(a) 4.0 M (b) 4.5 M (c) 5.0 M (d) 5.5 M
–3
12. The density of 1.50 M solution of acetic acid (CH3COOH) is 1.05 g cm . The molality of the solution will be
(a) 1.126 mol kg–1 (b) 1.36 mol kg–1 (c) 1.46 mol kg–1 (d) 1.56 mol kg–1
–1
13. The molality of an aqueous solution of methanol (CH3OH) is 1.44 mol kg . The mole fraction of methanol in the
solution will be
(a) 0.0253 (b) 0.0213 (c) 0.0273 (d) 0.0198
–1
14. The density of 1.40 molal solution of acetic acid (CH3COOH) is 1.084 g mL . The molarity of solution will be
(a) 1.30 M (b) 1.40 M (c) 1.50 M (d) 1.60 M
15. The molality m of a solution is related to its molarity M by the expression
M M M /r r - MM 2
(a) m = (b) m = (c) m = (d) m =
r - MM 2 r + MM 2 r + MM 2 M
where r is the density of solution and M2 is the molar mass of the solute.
16. The molarity M of a solution is related to the mole fraction x2 of solute by the expression
x2 r x2 r
(a) M = (b) M =
M1 + x2 ( M1 - M 2 ) M1 - x2 ( M1 - M 2 )
x2 r x2 r
(c) M = (d) M =
M1 + x2 ( M1 + M 2 ) M1 - x2 ( M1 + M 2 )
where r is the density of solution and M1 and M2 are the molar masses of solvent and solute, respectively.
17. The molality (m) of a solution in terms of mole fractions of solvent (x1) and solute (x2) is given by the expression
x1 x1M1 x2 M1 x2
(a) m = (b) m = (c) m = (d) m =
x2 M1 x2 x1 x1 M1
where M1 is the molar mass of the solvent.

ANSWERS
1. (b) 2. (c) 3. (c) 4. (d) 5. (a) 6. (d)
7. (c) 8. (d) 9. (c) 10. (b) 11. (c) 12. (d)
13. (a) 14. (b) 15. (a) 16. (b) 17. (d)

HINTS AND SOLUTIONS


1. Molar mass of C2H6O2 = 62 g mol–1
m 217.0 g
Amount of C2H6O2, n = = = 3.5 mol
M 62 g mol-1
n 3.5 mol
Molality of solution, m = = = 17.5 mol kg -1
m1 0.200 kg
2. Total mass of solution, m = 417.0 g
m 417.0 g
Volume of solution, V = = = 379.1 cm3
r 1.10 g cm -3
Solutions 5.5

m 217.0 g
Amount of C2H602, n = = = 3.5 mol
M 62 g mol-1
n 3.5 mol
Molarity of solution, M = = = 9.23 mol dm -3
V 379.1 ¥ 10-3 dm3
3. Molar mass of C6H12O6, M2 =180 g mol–1
Mass of glucose in the 100 g solution, m2 = 18 g
m2 18 g
Amount of glucose in 100 g solution, n2 = = = 0.1 mol
M1 180 g mol-1
Mass of water in 100 g solution, m1 = 82 g
m1 82 g
Amount of water in 100 g solution, n 1= = = 4.56 mol
M1 18 g mol-1
n2 0.1 mol
Mole fraction of glucose, x2 = = = 0.0214
n1 + n2 (4.56 + 0.1) mol
4. Let n be the amount of each of Na2CO3 and NaHCO3 in the given 1.0 g of mixture. We will have
n M Na 2CO3 + n M NaHCO3 = 1.0 g
i.e, n(108 g mol–1 + 84 g mol–1) = 1.0 g
1.0 g 1.0
or, n= -1
= mol
192 g mol 192
From the reaction Na2CO3 + 2HCl Æ 2NaCl + CO2 + H2O
(1.0/192) mol (2.0/192) mol
NaHCO3 + HCl Æ NaCl + CO2 + H2O
(1.0/192) mol (1.0/192) mol
2.0 1.0 3.0
Amount of HCl required = mol + mol = mol
192 192 192

Volume of 0.1 M HCl required, V = n / M = ÊÁ molˆ˜ /(0.1 mol L-1 ) = 0.1563 L = 156.3 m L
3.0
Ë 192 ¯
5. Mass of 1 L solution, m = rV = (1.25 g mL–1) (1000 mL) = 1250 g
Mass of water in solution, m1 = m – m2 = (1250 – 441) g = 809 g = 0. 809 kg
m2 441 g
Amount of H2SO4 in solution, n2 = = = 4.5 mol
M 2 98 g mol-1
n2 4.5 mol
Molarity of solution, M = = = 4.5 mol L-1
V 1L
n2 4.5 mol
Molality of solution, m = = = 5.56 mol kg -1
m1 0.809 kg

6. The molarity of resultant solution will be


M1V1 + M 2V2 (0.12 M )(60 m L ) + (0.25 M )(40 m L )
M= = = 0.172 M
V1 + V2 (100 m L )
7. Mole fraction of glucose = 0.2 implies that
Amount of glucose, n2 = 0.2 mol; Amount of water, n1 = 0.8 mol
Mass of water, m1= n1M1= (0.8 mol) (18 g mol–1) =14.4 g = 14.4 ¥ 10–3 kg
n2 0.2 mol
Molality of solution, m = = -3
= 14.18 mol kg -1
m1 14.4 ¥ 10 kg
5.6 Complete Chemistry—JEE Main

8. 40 mass percent of acetic acid implies that


Mass of acetic acid, m2 = 40 g; Mass of water, m1 = 60 g
m2 40 g
Amount of acetic acid, n2 = = = (2/3) mol
M 2 60 g mol-1
m1 + m2 100 g
Volume of solution, V = = = 80 m L
r 1.25 g m L-1
n2 (2 / 3) mol
Molarity of solution, M = = = 8.33 M
V 80 ¥ 10-3 L
M = n2/V = (m2/M2)/V, we get
m2 = MVM2 = (0.15 mol L–1) (0.250 L) (392 g mol–1) =14.7 g
10. Let we have
Amount of glucose, n2 = 0.15 mol; Volume of solution, V = 1000 mL
Mass of glucose, m2 = n2M2 = (0.15 mol) (180 g mol–1) = 27.0 g
Mass of solution, m = Vr = (1000 mL) (1.10 g mL–1) = 1100 g
Mass of water, m1 = m1 – m2 = 1100 g – 27 g = 1073 g
m 1073 g
Amount of water, n1 = 1 = = 59.61 mol
M1 18 g mol-1
n2 0.15 mol
Mole fraction of glucose, x2 = = = 0.002
n1 + n2 (59.61 + 0.15) mol
11. For x2 = 0.10, let we have n2 = 0.1 mol and n1 = 0.9 mol
The mass of solution will be
m = n1M1 + n2M2 = (0.9 mol) (18 g mol–1) + (0.1 mol) (32 mol–1) =19.4 g
The volume of solution will be
m 19.4 g
V= = = 20.0 ml = 20 ¥ 10 -3 L
r 0.97 g m L-1
The molarity of solution will be
n 0.1 mol
M= 2 = = 5.0 mol L-1
V 20 ¥ 10-3 L
12. Let there be 1 L of solution. we have n2 = 1.5 mol and V=1L
Mass of solution, m = Vr= (103 cm ) (1.05 g cm–3) = 1050 g
3

Mass of acetic acid, m2 = n2M2 = (1.5 mol) (60 g mol–1) = 90.0 g


Mass of water, m1 = m – m2 = 1050 g – 90 g = 960 g = 0.960 kg
n 1.5 mol
Molality of solution, m= 2 = = 1.56 mol kg -1
m1 0.960 kg
13. Let the mass of solvent be 1 kg. We will have n2 =1.44 mol and m1 = 1000 g
m1 1000 g
The amount of water will be n1 = = = 55.56 mol
M1 18 g mol-1
n2 1.44 mol
The mole fraction of the solute will be x2 = = = 0.0253
n1 + n2 (55.56 + 1.44) mol
14. Let the mass of solvent be 1 kg. We will have m1 = 1000 g and n2 = 1.40 mol
Mass of acetic acid, m2 = n2M2 = (1.40 mol) (60 g mol–1) = 8.40 g
Mass of solution, m = m1 + m2 = 1000 g + 84 g = 1084 g
m 1084 g
Volume of solution, V= = = 1000 m L = 1L
r 1.084 g m L-1
n2 1.40 mol
Molarity of solution M= = = 1.40 M
V 1L
Solutions 5.7

SECTION 2 Liquid Solutions

When a solute (solid, gas or liquid) is dissolved in a liquid solvent, a solution is formed.
Solid in Liquid
In general, it is found that like dissolves like. For example, an ionic compound (such as sodium compound) or a polar
compound (such as hydrogen chloride) is soluble in polar solvent water whereas these are insoluble in nonpolar solvent
benzene. Similarly, naphthalene is soluble in benzene while it is insoluble in water.
The solubility of a solid in a solvent depends on temperature. For an endothermic dissolution process (DH = +ve),
solubility increases with increase in temperature and for an exothermic process (DH = –ve), solubility decreases with
increase in temperatures.
A solution becomes saturated when no more solute dissolves in the solution. At a saturation stage, a dynamic
equilibrium exists between undissolved solute and solute in solution.

Gas in Liquid
Gases are soluble in liquids. For a given pair of gas and liquid, solubility depends on temperature and pressure. It is
found that the solubility of a gas in a liquid increases with ease of liquefaction of the gas. For example, in any solvent,
H2 and He are much less soluble than CO2, NH3 and SO2. Solubility is very much enhanced if there occurs a chemical
interaction between the gas and the liquid.
Effect of Pressure—Henry’s Law The solubility of a gas in a liquid increases with the increase in external
pressure, provided the gas does not react chemically with the liquid. According
to Henry’s law, the mass of dissolved gas in a given volume of solvent is
proportional to the partial pressure of the gas with which it is in equilibrium
(Fig. 1), i.e.
m2 μ p or m2 = kp2 (18)
where k is constant of proportionality.
Alternative Form of Henry’s Law If m2 is the mass of the gas dissolved
in the mass m1 of the solvent at gaseous pressure p2, we have
m2 = kp2
Dividing throughout by m1, we get
m2 Ê k ˆ
p2
m1 ÁË m1 ˜¯
= Fig. 1

Dividing the masses with the respective molar masses, we get


m2 / M 2 Ê kM1 ˆ n2
p2 or = k ¢p2
m1 / M1 ÁË m1M 2 ˜¯
= (19)
n1
Since the solution of a gas in a liquid is a dilute solution, we can approximate mole fraction x2 as shown in the following.
n2 n
x=  2 (as n2<< n1)
n1 + n2 n1

p = ÊÁ ˆ˜ x2 = kH x2
1
x2 = k ¢p or (20) †
Ë k¢¯
where kH is known as Henry’s law constant. It has the unit of pressure, e.g. Pa. According to Eq. (20), the partial
pressure of the gas is proportional to the mole fraction of the gas in the solution. Henry’s law is a characteristic

† Henry’s law is sometimes expressed in terms of molalities or concentrations and then the units of Henry’s law constant are Pa kg mol–1
or Pa m3 mol–1, respectively.
5.8 Complete Chemistry—JEE Main

constant for the gas. From Eq. (20), it follows that for the given external pressure of a gas, its solubility is inversely
proportional to its Henry law constant, i.e. higher the value of kH, lower the solubility.
Effect of Temperature Solubility of a gas in a liquid decreases exponentially with increase in temperature. The
dissolved gas is present in liquid phase. The gas is assumed to condense in liquid and since the process of condensation
is an exothermic process (DsolH = –ve) , it follows from Le Chatelier’s principle that the solubility will decrease with
increase in temperature.
Applicability of Henry’s law Henry’s law is applicable provided the solubility of gas is low so that the resultant
solution is dilute. Henry’s law is not applicable if the gas reacts chemically with the solvent.
Interpretation of Henry’s Law Equation (20) can be interpreted in either of the following ways.
(i) x2 is the solubility of the gas under the pressure p of the gas
(ii) p is pressure of gas when the mole fraction of the gas in liquid phase is x2. In this case, gas acts as solute and
liquid in which gas is dissolved is the solvent. The resultant solution is dilute.
Illustration Determine the solubility in mol L–1 of O2
Given: x(O2) = 0.2 in atmosphere and kH = 7.13 kbar.
Partial pressure of O2 in atmospheric pressure is pO2 = xO2 patm = (0.2) (1.01325 bar)
pO2 (0.2) (1.01325 bar)
Amount fraction of O2 in water is xO2 = k = = 2.84 ¥ 10-6
H 71.3 ¥ 10 bar
3

For the solubility in mol L–1 , we take Vsolution = 1 L =103 mL


Since the solution in dilute, we assume that Vwater = Vsolution = 103 mL
Mass of water (assuming its density is 1 g mL–1) in the volume 103 mL, m1 = Vr = (103 mL) (1 g mL–1) = 103 g
m1 103 g
Amount of water is n1 = = = 55.56 mol
M1 18 g mol-1
Since x2 = n2/(n1 + n2), we have
n2 n2
2.84 ¥10–6 =
(55.56 mol) + n2 (55.56 mol)
Solving for n2, we get n2 = (55.56 mol) (2.84 ¥ 10–6) = 1.584 ¥ 10–4 mol
Hence, Molarity of solution = 1.584 ¥ 10–4 mol L–1

Liquid in Liquid
In this section, a binary liquid solution formed from the two volatile liquids A and B is considered. Two types of liquid

Ideal Liquid Solution


If the molecular interactions A· · ·A are identical with those of B· · ·B and thus are also identical with those of
A· · ·B and also if the molecular volumes of A and B are more or less identical, the solution formed is said to be an
ideal solution. For such a solution DmixH = 0 and DmixV = 0, i.e. enthalpy and volume of mixing of two liquids are zero.
The values of DmixG = –ve and DmixS = +ve.
Since, both the liquids are volatile, both of them have their own vapour pressures and thus total vapour pressure is
sum of their partial pressures.
Raoult’ Law The partial pressures of the liquids A and B in a liquid solution follow Raoult’s law in the entire range
of composition, according to which, we have
pA = xA p*A and xB p*B (21)
where xA and xB are the respective mole fractions of A and B in liquid phase and p*A and p*B are their respective vapour
pressures of the pure liquids.
Solutions 5.9

Diagrammatic Variations of Vapour Pressures


Figure 2 displays the variations of pA and pB with the mole fractions
of A and B, in liquid phase, respectively. The total vapour pressures,
as given by Dalton’s law of partial pressures, is
ptotal = pA + pB = xA p*A + xB p*B
Since xA + xB =1, we have
ptotal = xA p*A + (1 – xA) p*B = p*B + (p*A – p*B)xA (22a)
In terms of xB, we have
ptotal = (1 – xB) p*A + xB p*B = p*A + (p*B – p*A)xB (22b)
The composition of vapour phase is
p p
yA = A and yB = B (23)
ptotal ptotal
Vapour Composition Versus Liquid Composition we have
pA xA p*A
yA = =
ptotal p*B + ( p*A - p*B ) xA Fig. 2

The reciprocal of this expression is

1 p*B + ( p*A - p*B ) xA p*B 1 p* - p*B


= = + A (24)
yA xA p*A p*A xA p*A
Thus 1/yA varies linearly with respect to 1/xA (Fig. 3). The expression 1/xA
versus 1/yA is
1 p*A 1 p*B - p*A
= + (25)
xA p*B yA p*B
Vapour Pressure Versus Vapour Composition We have
pA x p*
yA = = A A
ptotal ptotal
1 y A 1
or =
ptotal p*A xA
Using Eq. (25), we get Fig. 3

1 yA Ê p*A 1 p* - p*A ˆ 1 Ê 1 1 ˆ
= Á + B ˜= +Á - ˜ yA (26)
ptotal p*A Ë p*B yA p*B ¯ p*B Ë p*A p*B ¯
Thus 1/ptotat varies linearly with respect to yA (Fig. 4).
Examples of Ideal Liquid solutions Some of the known examples
which form nearly ideal solutions are :
(i) Ethylene dibromide and propylene bromide at 358 K.
(ii) Ethyl bromide and ethyl iodide at 303 K
(iii) n-Butyl chloride and n-butyl bromide at 323 K
(iv) n-Hexane and n-heptane at 303 K
Illustration Heptane and octane form an ideal solution. At 373 K, the
vapour pressure of the two liquids are 105.2 kPa and 46.8 kPa, respectively.
Determine the vapour pressure and vapour compositions of a liquid mixture
of 26.0 g heptane and 35.3 g of octane.
Molar mass of heptane (C7H16), M1 = 100 g mol–1
Molar mass of Octane (C8H18), M2 = 114 g mol–1 Fig. 4
5.10 Complete Chemistry—JEE Main

m1 26.0 g
Amount of heptane n1= = = 0.26 mol
M1 100 g mol-1
m2 35.3 g
Amount of octane, n2= = = 0.31 mol
M 2 114 g mol-1
n1 0.26 mol
Mole fraction of heptane, x1 = = = 0.46
n1 + n2 (0.26 + 0.31) mol
Mole fraction of octane, x2 = 1 – x1 = 1 – 0.46 = 0.54
Vapour pressure of solution,
p = x1p*1 + x1p*2 = (0.46) (105.2 kPa) + (0.54) (46.8 kPa) = 48.39 kPa + 25.27 kPa = 73.66 kPa
p1 48.39 kPa
Amount fraction of heptane in vapour phase, y1 = = = 0.66
p 73.66 kPa
Amount fraction of octane in vapour phase, y2 = 1 - y1 = 1 - 0.66 = 0.34
Nonideal Solutions
If the molecular interactions A···A, B···B and A···B are not identical, the
solution formed is known as nonideal solution. For these solution, DmixH π 0
and DmixV π 0. The values of DmixG = –ve and DS = +ve. Two types of nonideal

Solutions Exhibiting Positive Deviation from Raoult’s Law


If the molecular interactions A· · ·B are weaker than those of A· · ·A and
B· · ·B, the solution exhibits positive deviation from Raoult’s law. In this case,
the vapour pressure curves of the constituents A, B and the solution lie above
those of ideals lines (Fig. 5). The positive deviation arises from the fact that
the tendency of molecules of each kind to escape from the solution to vapour
is greater than that prevailing in an ideal solution. For this type of solutions,
DmixH > 0
Examples are:
(i) Carbon tetrachloride and heptane
(ii) Ethyl ether and acetone Fig. 5
(iii) Heptane and ethyl alcohol
(iv) Benzene and acetone
Solutions Exhibiting Negative Deviation from Raoult’s Law
If the molecular interactions A· · ·B are stronger than those of A· · ·A and
B· · ·B, the solution exhibits negative deviation from Raoult’s law. In this
case, the vapour pressure curves of the constituents A, B and the solution lie
below those of ideal lines (Fig. 6). The negative deviation arises from the fact
that the tendency of molecules of each kind to escape from the solution into
the vapour is weaker than that prevailing in and ideal solution. For this type
of solution DmixH < 0.
Examples are:
(i) Pyridine and formic (or acetic or propanic) acid
(ii) Mixture of halomethane (e.g. chloroform) with an oxygen or nitrogen
compound (e.g. a ketone, ether, ester or amine)
(iii) Aqueous solutions of HCl, HNO3 and perchloro acids
Common feature of Nonideal Solutions Before describing the common
feature of nonideal solutions displayed in Fig. 5 and Fig. 6, it is worth correlating
Henry’s law and Raoult’s law.
Fig. 6
Solutions 5.11

Correlation Between Henry’s Law and Raoult’s Law Henry’s law is


p2 = kH x2 (20)
This law is applicable to solute in dilute solution. In a case, where Henry’s law is applicable over the entire range of
concentration starting from x2 = 0 (pure solvent) to x =1 (pure liquid solute), we will have
p2 p*2
kH = =
x2 1
where p* is the vapour pressure of pure liquid solute (x2 = 1). With this, Eq. (20) becomes
p2 = p*2 x2 (21)
which is Raoult’s law. The Raoult’s law is a special case of Henry’s law; all systems which obey Raoult’s law must
satisfy Henry’s law, but the reverse is not true unless Henry’s law is applicable over the entire range of concentration.
In general, if Henry’s law is applicable to solute in the limited range of dilute solution (known as ideally dilute
solution), Raoult’s law is applicable to solvent in the same range of composition. For nonideal solutions shown in Figs
(5) and (6), the associated Raoult’s law line and Henry’ law line are also shown. It may be seen that Raoult’s law line
meets the partial vapours pressure curve tangentially in the range xAÆ1 and Henry’s law line meets the curve in the
region xAÆ0 . The same is true for the component B.
In general, we have
Ideal solution kH,A = pA* and kH,B = p*B (23 a)
Nonideal solutions
Positive deviation kH,A > pA* and kH,B > p*B (23 b)
Negative deviation k < p* and k < p*B
H,A A H,B (23 c)
Formation of Azeotropic Solution If the deviations from Raoult law are large, a maximum/minimum in the vapour
pressure versus composition curve is observed. Because of increase/decrease in vapour pressure, the boiling point versus
composition curve of the solution exhibits a minimum/maximum at which composition of liquid and vapour phases are
identical (Figs. 7 and 8). The composition corresponding to this solution is known as azeotropic composition and the
corresponding temperature is known as azeotropic temperature.

Fig. 7 Fig. 8

MULTIPLE CHOICE QUESTIONS ON SECTION 2

Identify the correct choice in the following questions.


1. Henry’s law for the solubility of gas in water is not applicable to the gas
(a) H2 (b) O2 (c) N2 (d) NH3
5.12 Complete Chemistry—JEE Main

2. The Henry’s law constant does not depend upon


(a) nature of solute (b) nature of solvent
(c) temperature of the system (d) external pressure of the gas on the system
3. Show that Henry’s law (p = kH x2) for the dissolution of a gas in a solvent when expressed in molality (m) for the
solution is approximately given as
(a) p = (kH M1)m (b) p = (kH /M1)m (c) p = (kH M2)m (d) p = (kH /M2)m
where M1, and M2 are the molar masses of solvent and gas, respectively.
4. Water saturated with air (20% O2 and 80% N2) at 1 atm and 298 K contains 8.9 ¥ 10–3 g L–1 dissolved oxygen. Its
solubility under 25 atm pressure of O2 will be
(a) 8.9 ¥ 10–3 g L–1 (b) 1.1125 g L–1 (c) 1.5642 g L–1 (d) 1.834 ¥ 10–2 g L–1
5. The solubility of a gas in a liquid for the given external pressure
(a) increases linearly with increase in temperature
(b) decreases linearly with increase in temperature
(c) increases exponentially with increase in temperature
(d) decreases exponentially with increase in temperature
6. The solubility of N2 in blood at 37 ºC and at a partial pressure of 0.80 atm is 5.6 × 10–4 mol L–1. The Henry’s law
constant for this system is
(a) 1.43 × 103 atm L mol–1 (b) 1.62×103 atm L mol–1
(c) 5.6 × 10–4 atm L mol–1 (d) 2.86×102 atm L mol–1
7. The solubility (x2) of gases in a given liquid under the constant external partial pressures is
(a) directly proportional to Henry’s law constant
(b) inversely proportional to Henry’s law constant
(c) directly proportional to inverse of Henry’s law constant
(d) does not depend on Henry’s law constant of the gas
8. Henry’s law constants at 25 ºC for N2 and O2 in water are 4.34 × 105 Torr/(g N2/100 g H2O) and 1.93 ¥105 Torr/
(g O2/100 g H2O), respectively. If the pN 2 = 608 Torr and pO2 = 152 Torr above the solution, the ratio of mass of
O2 to N2 in water
(a) is larger than the corresponding ratio in air
(b) is smaller than the corresponding ratio in air
(c) is equal to the corresponding ratio in air
(d) cannot be compared with the corresponding ratio in air
9. For an ideal binary liquid solution, which of the following facts is not correct?
(a) DmixH = 0 (b) DmixV = 0 (c) DmixG < 0 (d) DmixS < 0
10. For an ideal solution of liquids A and B, which of the facts is correct?
(a) kH, A > pA* and kH, B > p*B (b) kH, A < pA* and kH, B < p*B
(c) kH, A = pA* and kH, B = p*B (d) kH, A > pA* and kH, B < p*B
where kH¢s stand for Henry’s law constants and p*¢s are the vapour pressures of pure components.
11. For a binary ideal liquid solution of A (having vapour pressure pA* ) and B (having vapour pressure p*B ), xA and
yA are the mole fractions of A in liquid and vapour phases, respectively. Which of the following facts holds good?
(a) yA varies linearly with xA (b) yA varies linearly with 1/xA
(c) 1/yA varies linearly with xA (d) 1/yA varies linearly with 1/xA
12. Liquids A and B have vapour pressures pA* and p*B , respectively, with pA* > p*B . These liquids from an ideal
solution. If xA and yA are mole fractions of A in liquid and vapour phases then the slope of
(a) the linear plot of yA versus xA is positive (b) the linear plot of yA versus xA is negative
(c) the linear plot of 1/yA versus 1/xA is positive (d) the linear plot of 1/yA versus 1/xA is negative
13. Liquids A and B have vapour pressures pA* and p*B , respectively, with pA* > p*B . The plot of 1/ptotal versus yA
(mole fraction of A in vapour phase)
(a) is linear with positive slope (b) is linear with negative slope
(c) increases nonlinearly (c) decreases nonlinearly
Solutions 5.13

14. Methanol (p* = 90 mmHg at 300 K) and ethanol (p* = 51 mmHg at 300 K) form very nearly an ideal solution.
The total vapour pressure of a solution obtained by mixing 23 g ethanol with 32 g methanol would be
(a) 17 mmHg (b) 60 mmHg (c) 77 mmHg (d) 82 mmHg
15. An ideal solution of benzene (p* = 75 mmHg) and toluene (p* = 22 mmHg) has a vapour pressure of 50 mmHg.
The mole fraction of benzene in liquid and vapour phases, respectively are
(a) 0.53, 0.795 (b) 0.795, 0.53 (c) 0.47, 0.205 (d) 0.205, 0.47
16. An ideal liquid solution of 1 mol of A and 3 mol of B has the vapour pressure of 550 Torr. If one more mole of B
is added, its vapours pressure becomes 560 mmHg. The vapour pressures of pure A and pure B, respectively are
(a) 600 Torr and 400 Torr (b) 400 Torr and 600 Torr (c) 650 Torr and 350 Torr (d) 350 Torr and 650 Torr
17. An ideal solution of liquid A ( pA* = 100 mmHg) and liquid B ( p*B = 30 mmHg) has 0.6 mole fraction of A in the
vapour phase. The mole fraction of A in the liquid phase will be
(a) 0.21 (b) 0.25 (c) 0.31 (d) 0.41
p * p*
18. An ideal solution of liquid A ( A = 100 mmHg) and liquid B ( B = 30 mmHg) has 0.4 mole fraction of A in the
vapour phase. The total vapour pressure of the solution is
(a) 35.27 mmHg (b) 41.67 mmHg (c) 52.0 mmHg (d) 58.2 mmHg
19. A binary liquid solution exhibits positive deviation from ideal behaviour. Which of the following facts hold good?
(a) DmixH > 0 and DmixG < 0 (b) DmixH < 0 and DmixG < 0
(c) DmixH > 0 and DmixG > 0 (d) DmixH < 0 and DmixG > 0
20. A binary liquid solution exhibits negative deviation from ideal behaviour. Which of the following facts hold good?
(a) DmixH > 0 and DmixS > 0 (b) DmixH > 0 and DmixS < 0
(c) DmixH < 0 and DmixS > 0 (d) DmixH < 0 and DmixS < 0
21. Which of the facts regarding dilute solution of a nonideal binary liquid solution is correct?
(a) Solute follows Raoult’s law and solvent follows Henry’s law
(b) Solute follows Henry’s law and solvent follows Raoult’s law.
(c) Both solute and solvent follows Raoult’s law
(d) Both solute and solvent follows Henry’s law
22. Which of the following facts is correct for a binary solution of liquid A (vapour pressure pA* ) and liquid B (vapour
pressure p*B ) exhibiting positive deviation from ideality?
(a) kH, A > pA* and kH,B < p*B (b) kH,A < pA* and kH,B < p*B
(c) kH, A > pA* and kH,B > p*B (d) kH,A < pA* and kH,B > p*B
23. For a binary liquid solution of A and B, the mole fraction of A in liquid and vapour phases are 0.70 and 0.35 at
600 Torr external pressure. The vapour pressure of A and B, respectively, are
(a) 1300 Torr and 300 Torr (b) 450 Torr and 950 Torr
(b) 950 Torr and 450 Torr (d) 300 Torr and 1300 Torr
24. The mole fraction of liquid A in a binary liquid solution of A (vapour pressure pA* = 300 Torr) and B (vapour
pressure, p*B = 800 Torr) is 0.6. The external pressure on this is slowly reduced from 760 Torr. The pressure at

(a) 400 Torr and 0.24 (b) 500 Torr and 0.50 (c) 500 Torr and 0.36 (d) 600 Torr and 0.25
25. The mole fraction of liquid A in a binary liquid solution of A ( pA* = 300 Torr) and B ( p*B = 800 Torr) is 0.6. The
pressure at which last droplet of liquid is vapouried and the mole fraction of A in this last droplet, respectively, are
(a) 400 Torr and 0.80 (b) 400 Torr and 0.20
(c) 500 Torr and 0.75 (d) 500 Torr and 0.45
26. A solution of liquids A and B exhibits positive deviation from Raoult’s law provided
(a) Molecular interactions A· · ·A, B· · ·B and A· · ·B are identical
(b) Molecular interactions A· · ·B are larger than those between A· · ·A and B· · ·B
(c) Molecular interactions A· · ·B are weaker than those between A· · ·A and B· · ·B
(d) Molecular interactions A· · ·A are larger than those between A· · ·A and weaker than those between B· · ·B
5.14 Complete Chemistry—JEE Main

ANSWERS
1. (d) 2. (d) 3. (a) 4. (b) 5. (d) 6. (a)
7. (b) 8. (a) 9. (d) 10. (c) 11. (d) 12. (c)
13. (b) 14. (c) 15. (a) 16. (b) 17. (c) 18. (b)
19. (b) 20. (a) 21. (b) 22. (c) 23. (d) 24. (c)
25. (b) 26. (c)

HINTS AND SOLUTIONS


1. NH3 interacts with water.
2. The Henry’s law constant does not depend upon the external pressure of the gas on the system.
3. The expression of mole fraction in terms of molality of the solution is
mM1
x2 =
1 + mM1
For a dilute solution mM1<<1. Hence x2  mM1
Hence, p = kH x2 = (kHM1)m
4. Since p = kHc, we have 0.20 atm = kH (8.9 ¥ 10–3 g L–1) and 25 atm = kHc
Hence, c = (8.9 ¥ 10–3 g L–1) (25 atm)/0.20 atm = 1.1125 g L–1
5. The solubility of a gas in a liquid follows the expression c = c0 exp(–DH/RT) with DH = –ve
On increasing temperature, the exponential term decreases.
6. Henry’s law in terms of concentration of dissolved gas is p = kHc. Hence,
p 0.80 atm
kH = = = 1.43 ¥ 103 atm L mol–1
c 5.6 ¥ 10-4 mol L-1
7. Henry’s law is p = kH x2. It is obvious that x2 is inversely dependent on kH.
8. We have
For N2 608 Torr = [4.34 ¥ 105 Torr/(g N2/100 g H2O)] mN 2
For O2 152 Torr = [1.93 ¥ 105 Torr/(g O2/100 g H2O] mO2
mO2 (152 / 1.93 ¥ 105 ) g 7.88 ¥ 10-4 g
Hence, in water = = = 0.56
mN 2 (608 / 4.34 ¥ 105 ) g 1.40 ¥ 10-3 g
mO2152 Torr
The corresponding ratio in air = 608 Torr = 0.25
mN 2
9. There is an increase in entropy during the formation of binary liquid solution.
m1 32 g
14. Amount of methanol, n1 = = =1.0 mol
M1 32 g mol-1
m 23 g
Amount of ethanol, n2 = 2 = = 0.5 mol
M2 46 g mol-1
n1 1.0 mol
Mole fraction of methanol, x1 = = = 2/3
n1 + n2 (1.0 + 0.5) mol
Total Vapour pressure, p = x1 p1* + x2 p*2 = [(2/3)(90) + (1/3) (51)] mmHg = 77 mmHg
15. We have ptotal = xb p*b + xt p*t = xb p*b + (1 – xb) p*t = p*t + ( p*b – p*t ) xb
ptotal - p*t (50 - 22) mmHg
Hence, xb = = = 0.53
*
pb - pt * (75 - 22) mmHg
Solutions 5.15

pb = xb p*b = (0.53) (75 mmHg) = 39.75 mmHg


pb 39.75 mmHg
yb = = = 0.795
ptotal 50 mmHg
nA 1 mol 1 1 3
16. Mole fractions are xA = = = and xB = 1 – =
nA + nB (1 + 3) mol 4 4 4
1 * 3
Hence, 550 Torr = p + p* (1)
4 A 4 B
When one more mole of B is added, then
1 mol 1 1 4
x¢A = = and x¢B = 1– =
(1 + 4) mol 5 5 5
1 * 4
Hence, 560 mmHg = pA + p*B (2)
5 5
Solving pA* and p*B from Eqs (1) and (2), we get pA* = 400 Torr and p*B = 600 Torr
pA xA p*A xA p*A
17. We have yA = = =
pA + pB x p* + (1 - x ) p* p* + ( p* - p* ) x
A A A B B A B A

yA p*B (0.69) (30)


This gives xA = = = 0.31
p*A + ( p*B - p*A ) yA 100 + (30 - 100) (0.60)
18. We have
pA pA x p*
yA = fi ptotal = = A A
ptotal yA yA
Also ptotal = xA pA* + xB p*B = p*B + ( pA* – p*B )xA
Equating the two expressions of ptotal, we get
xA p*A xA (100 mmHg)
= p*B + ( pA* – p*B )xA or = 30 mmHg + [(100 –30) mmHg] xA
yA 0.4
30 1
or xA(250) = 30 + 70 (xA) or xA = =
180 6
Ê 1ˆ Ê 5ˆ
ptotal = xA pA* + xB p*B = Á ˜ (100 mmHg) + Ë 6 ¯ (30 mmHg) = 16.67 mmHg + 25 mmHg = 41.67 mmHg.
Ë 6¯
19. Positive deviation from ideality implies that the molecular interactions A· · ·B are weaker than those of A· · ·A and
B· · ·B. More energy is required in breaking A· · ·A and B· · ·B interactions as compared to the energy released
in making A· · ·B interactions. Hence, the formation of liquid solution will be exothermic, i.e. DmixH < 0, DmixG
will be negative as the formation of liquid solution is spontaneous.
20. Negative deviation from ideality implies that the molecular interactions A· · ·B are stronger than those of A· · ·A
and B· · ·B. More energy is relased in making A· · ·B interactions as compared to the energy required to break
A· · ·A and B· · ·B interactions. Hence, the formation of liquid solution is accompanied with DmixH > 0. Dmix S is
always positive in the formation of a solution.
pA x p* y p
23. We have yA = = A A fi pA* = A total
ptotal ptotal xA
(0.35) (600 Torr ) yB ptotal (0.65) (600 Torr )
Thus pA* = = 300 Torr Similarly, p*B = = = 1300 Torr
(0.70) xB (0.30)
bble of vapour is formed will be
p = x p* + x p* = (0.6) (300 Torr) + (0.4) (800 Torr) = 500 Torr
A A B B
5.16 Complete Chemistry—JEE Main

xA p*A (0.6)(300 Torr )


The composition of this vapour will be yA == (500 Torr ) = 0.36
p
25. When the last droplet of liquid is vapourized, the composition of vapour formed will be yA= 0.60 (given composition
of the liquid solution).
pA xA p*A xA p*A
Hence, yA = p = =
xA p*A + xB p*B p* + ( p* - p* ) x
B A B A

xA (300 Torr )
i.e 0.60 =
800 Torr + (300 Torr - 800 Torr ) xA
Solving for xA, we get xA = 0.80.
The pressure of the vapour will be p = xA pA* + xB p*B = (0.80) (300 Torr) + (0.20) (800 Torr) = 400 Torr
26. The escaping tendencies of A and B are enhanced if molecular interactions A· · ·B are weaker than those between
A· · ·A and B· · ·B.

SECTION 3 Colligative Properties

In this section, colligative properties of a binary liquid solution formed by dissolving a nonvolatile solute in a liquid

species relative to the total number of species present in the solution.

Relative Lowering of Vapour Pressure


In a solution containing a nonvolatile solute, the vapour pressure of solution is only due to solvent molecules. For an
ideal solution, this is given
p = x p*
1 1 1 (21)†
where the subscript 1 represents solvent.
The relative lowering of vapour pressure is
p1* - p1 p1* - x1 p1
= = 1 – x1 = x2 (24)
p1* p1*
D p1
or - = x2 (where Dp1 = p1 – p1*) (25)
p*
1

Determination of Molar Mass of Solute For a dilute solution, we have


n2 n (m2 / M 2 )
x2 =  2 =
n1 + n2 n1 (m1 / M1 )
Dp1 (m2 / M 2 ) m2
Hence, -  or M2 = (26)
p1* (m1 / M1 ) (m1 / M1 )(-Dp1 / p1* )
Elevation of Boiling Point
A liquid starts boiling when its vapour pressure becomes equal to the external pressure on the liquid. The temperature
at which this occurs is known as boiling point.

† The physical explanation of Eq. (21) is that the number of solvent molecules available at the surface of solution, which are responsible
to produce vapour pressure, is decreased as compared to those available in a pure liquid solvent
Solutions 5.17

The vapour pressure of a solution containing a nonvolatile solute is


lesser than that of pure solvent. The variation in the vapour pressures
of solvent and solution with the temperature are shown in Fig. 9. From
Fig 9, it is obvious that there occurs an elevation of boiling point when
a nonvolatile solute is dissolved in a liquid solvent.
Since lowering of vapour pressure of a solution relative to a pure
solvent depends upon the mole fraction, x2, of solute in solution, it is
expected that the elevation of boiling point also depends on x2, i.e.
DTb μ x2
It is customary to write DTb in terms molality, m, of the solution. For
a dilute solution
x2 μ m
Hence, DTb μ m or DTb μ Kb m (27)
Fig. 9

where Kb is known as boiling point elevation constant. The unit of Kb is


Unit of DTb K
Unit of Kb = = = K kg mol–1
Unit of m (i.e. molality) mol kg -1
The more vigorous treatment reveals that
M1 RTb*2
Kb = (28)
D vap H m
where M1 is the molar mass of solvent, Tb* is boiling point of the solvent and DvapHm is the molar enthalpy of
vapourization of the solvent. From Eq. (28), it follows that Kb is a characteristic constant of the solvent only.
Illustration If DvapHm(water) = 40.658 kJ mol–1, determine boiling point elevation constant for water.
M1 RTb*2 (0.018 kg mol-1 )(8.314 J K -1 mol-1 )(373.15 K ) 2
Kb = = = 0.512 K Kg mol–1
D vap H m (40.658 ¥ 103 J mol-1 )
Determination of Molar Mass of Solute The molality m of the solution is
n2 (m2 / M 2 )
m= m =
1 m1
Ê m / M2 ˆ K b m2
Hence, DTb = Kb Á 2 or M2 = (29)
Ë m1 ˜¯ DTb m1

Illustrations
1. A solution contains 20.0 g of glucose (C6H12O6) dissolved in 100 g water. Calculate its normal boiling point. Given:
Kb(water) = 0.512 K kg mol–1
m 20.0 g 1
Amount of glucose, n2 = 2 = -1 = mol
M 2 180 g mol 9

n2 (1 / 9) mol
Molality of solution, m = = = 1.11 mol kg–1
m1 100 ¥ 10-3 kg
Elevation in boiling point, DTb = Kb m = (0.512 K kg mol–1) (1.11 mol kg–1) = 0.57 K
DTb = 0.57 K implies DTC = 0.57 ºC.
*
The normal boiling point of the solution is tb = tb + Dtb = 100 ºC + 0.57 ºC = 100.57 ºC
2. A solution of 12.50 g of nonelectrolyte in 150.0 g of water gives boiling point elevation of 0.63 ºC. Calculate the
molar mass of nonelectrolyte. Given : Kb(water) = 0.512 K kg mol–1
5.18 Complete Chemistry—JEE Main

Ê m2 ˆ Ê K b ˆ Ê 12.50 g ˆ Ê 0.512 K kg mol -1 ˆ –1 –1


M2 = Á ˜ Á ÁË 150.0 g ˜¯ Á
Ë m1 ¯ Ë DTb ˜¯
= ˜¯ = 0.0677 kg mol = 67.7 g mol
Ë 0.63 K

Depression of Freezing Point


The temperature at which the vapour pressure of solid phase
of a substance becomes equal to the vapour pressure of its
liquid phase is known as the freezing point of the substance.
The variation of vapour pressures of solid solvent, liquid
solvent and solution containing a nonvolatile solute, are shown
in Fig. 10 along with the freezing points of pure solvent and
solution. From Fig. 10, it follows that there occurs a depression

DTf = Tf – Tf*
The depression in freezing point is
–DTf = Tf* – Tf
From Fig. 10, it follows that the depression in freezing point
is directly proportional to the lowering of vapour pressure of
the solvent when a nonvolatile solute is dissolved in it, i.e.
–DTf μ (– Dp)
Also, from the relative lowering of vapour pressure. We have
(– Dp) μ x2 Fig. 10
Thus –DTf μ x2
It is customary to write –DTf in terms of molality of solution. For a dilute solution x2 μ m, hence
–DTf μ m or – DTf = Kf m (30)
where Kf is known as freezing point depression constant. The unit of Kf is
Unit of DTf K
Unit of Kf = = = K kg mol–1
Unit of m (i.e. molality) mol kg -1
The more rigorous treatment reveals that
*2
Kf = M1 RTf (31)
D fus H m
where M1 is the molar mass of solvent, Tf* is the freezing point of the solvent and DfusHm is the molar enthalpy of fusion
of solid solvent. From Eq. (31), it follows that Kf is a characteristic constant of the solvent only.
Illustration If DfusHm(water) = 6.01 kg mol–1, determine freezing point depression constant of water
(0.018 kg mol -1 )(8.314 J K -1 mol -1 )(273.15 K )2
2
M1 RTf*
Kf = = -1
= 1.86 K kg mol–1
D fus H m ( 6 . 01 ¥ 103
J mol )
Determination of Molar Mass of Solute The molality m of the solution is
n2 (m2 / M 2 )
m= =
m1 m1
Ê m / M2 ˆ Kf m2
Hence – DTf μ Kf Á 2 or M2 =
Ë m1 ˜¯ ( -DTf ) m1
Illustrations
1. A solution contains 10.0 g of urea (H2NCONH2) dissolved in 100 g of water. What is its freezing point? Given : Kf
(water) = 1.86 K kg mol–1
m2 10.0 g 1
Amount of urea, n2 = = -1 = mol
M2 60 g mol 6
Solutions 5.19

n2 (1 / 6) mol
Molality of solution, m = = = 1.67 mol kg–1
m1 0.10 kg
Depression in freezing point, –DTf = Kf m = (1.86 K kg mol–1) (1.67 mol kg–1) = 3.11 K
Since DTf = –3.11 K, we have Dtf = –3.11 ºC
Freezing point of solution is tf = tf º + Dtf = 0 ºC – 3.11 ºC = – 3.11 ºC
2. The freezing point of solution containing 0.525 g camphor (solvent) and 0.04 g of an unknown compound is
–177 ºC. Determine molar mass of the compound. Given : Kf (camphor) = 37.7 K kg mol–1
Ê m ˆ Ê K ˆ Ê 0.04 g ˆ Ê 37.7 K kg mol -1 ˆ
M2 = Á 2 ˜ Á f ˜ = Á –1 –1
Ë m1 ¯ Ë - DTf ¯ Ë 0.525 g ˜¯ ÁË 17.70 K ˜¯ = 0.1623 kg mol = 162.3 g mol

Osmosis and Osmotic Pressure

solvent through a semipermeable membrane is known as osmosis. The membrane allows only the passage of solvent
molecules.
Consider a system shown in Fig. 11a. Due to osmosis, the level of liquid in the pure solvent side starts falling while
that in solution side start rising. Eventually, a stage is reached where osmosis seems to have stopped. This is the stage
of dynamic equilibrium where solvent molecules leaving and entering in either side from the other are identical. The
hydrostatic pressure created on the solution side exactly balances the excessive tendency of solvent to pass through the
membrane. This excessive pressure is known as the osmotic pressure of the solution. The osmosis in Fig. 11a can be
arrested by either increasing the external pressure on the solution side or decreasing the external pressure on the solvent
side by an amount equivalent to the osmotic pressure. The excessive pressure in Fig.11b is given as.
P = hrg (32)

(a) (b)
Fig. 11
where h is the excessive height in the tube on the solution side, r is the density of solution and g is the acceleration
due to gravity.
The van’t Hoff equation for osmotic pressure P of the solution is given as
P = cRT (33)
where c is the concentration of the solution, R is universal gas constant and T is the kelvin temperature.
Illustration Calculate the osmotic pressure of an aqueous solution containing 6.85 g sucrose (C12H22O11) in 500 mL
solution at 298 K.
m2 6.85 g
Amount of sucrose, n2 = = = 0.020 mol
M 2 342 g mol-1
5.20 Complete Chemistry—JEE Main

n2 0.020 mol
Concentration of the solution, c = = = 0.040 mol dm–3
V 0.500 dm3
Osmotic pressure, P = cRT = (0.040 mol dm–3) (8.314 kPa dm3 K–1 mol–1) (298 K) = 99.1 kPa

Determination of Molecular Mass of Solute


Since c = n/V and n = m/M, we get
n (m / M 2 )
P = cRT = 2 (RT ) = 2 RT
V V
m2 Ê m ˆ RT
or PV = RT or M2 = Á 2 ˜ (34)
M2 ËV ¯ P
The measurement of osmotic pressure is used to determine the molar masses of solutes having low solubility due
its large and heavy molecules. Examples include proteins, polymers of various types and colloids. This is due to the
fact that even a small concentration of the solution produces fairly large magnitude of osmotic pressure. The other
colligative properties such as elevation of boiling point and depression in freezing point of such solutions are too small
to be determined experimentally.
Illustration A solution of 0.20 g of polypeptide in 400 mL of an aqueous solution has an osmotic pressure of 0.50
kPa at 300 K. Calculate the molar mass of polypeptide.

Ê m ˆ RT ˆ 0.20 g È (8.314 kPa dm3 K -1 mol -1 )(300 K ) ˘


M = Á 2 ˜ ÊÁ Í ˙ = 2994.2 g mol–1
Ë V ¯ Ë P ˜¯ 0.40 dm3 Î
We have =
0.50 kPa ˚
Colligative Properties of Electrolytic Solution
Colligative properties of an electrolytic solution are found to be larger than those shown by a nonelectrolytic solution
of the same concentration. This is due to partial or complete ionization of electrolyte in the solution resulting in the
larger number of species as compared to the species available in a nonelectrolic solution of the same concentration.
Examples
1. Dilute Solution of a Strong Electrolyte If n is the number of species (i.e. cations plus anions) produced due to
the ionization of one molecule of the electrolyte, the colligative properties are found to be n times the corresponding
properties of a nonelectrolytic solution of the same concentration. For example, 0.1 M NaCl solution and 0.1 MgCl2
will exhibit, respectively, two and three times colligative properties as compared to 0.1 M glucose or sucrose solution.

2. Concentrated Solution of a Strong Electrolyte


Due to strong ionic interactions, a strong electrolyte, though completely ionized, is not completely dissociated. Thus,
such a solution exhibit colligative properties which are less than n times the corresponding properties of a nonelectrolytic
solution of the same concentration, where n is the number of species produced by complete ionization of a molecule
of the electrolyte.
3. Solution of a Weak Electrolyte A weak electrolyte is not completely ionized in the solution. For example, for
acetic acid we have


CH3COOH 
-
 CH3COO + H
+

1-a a a

The number of species in the solution is 1 – a + a + a = 1 + a, where a is the degree of dissociation of acetic acid.
Thus, the colligative properties of acetic acid solution is (1 + a) times those shown by a nonelectrolytic solution of the
same concentration.
Van’t Hoff Factor The colligative properties of an electrolytic solution can be expressed in terms of those of
nonelectrolytic solution of the same concentration by making use of van’t Hoff factor, i
Colligative effect produced by a given concentration of an electrolytic solution
i=
Colligative effect produced by the same concentration of a nonelectrolytic solution
Solutions 5.21

- DTf DTb P Dp
Thus, we can write i= = = = (35)
(- DTf )0 ( DTb )0 (P )0 ( Dp)0
where the quantities without subscript refer to electrolytic solution and those with subscript zero refer to nonelectrolytic
solution of the same concentration.
Degree of dissociation of a Weak Electrolyte and van’t Hoff Factor Let AxBy be the electrolyte and let a be its
degree of dissociation. If n0 is the initial amount of Ax By, we will have

Ax B y 
y+
 xA + yB
x-

n0(1– a) n0(xa) n0( ya)


Total amount of species = n0(1 – a) + n0xa + n0 ya = n0[1 + (x + y – 1)a] = n0[1 + (n – 1)a]
where n = x + y = sum of stoichiometric numbers of cation and anion in a molecule of electrolyte. In terms of van’t
Hoff factor, we have
n0 [1 + (n - 1)a ] i -1
i= = 1 + (n - 1)a or a= (36)
n0 n -1
Illustration A 0.126 molal solution of K3Fe(CN)6 freezes at –0.65 °C. Calculate the values of (a) van’t Hoff factor,
and (b) degree of dissociation of the salt. Give : Kf (water) = 1.86 K kg mol–1.
- DTf 0.65 K
Van’t Hoff factor, i= = = 2.77
K f m (1.86 K kg mol-1 ) (0.126 mol kg -1 )
i - 1 2.77 - 1
Degree of dissociation, a = = = 0.59
n -1 4 -1
Isotonic Solutions Two solutions having the same osmotic pressure are known as isotonic solutions.

Illustration A 10% by mass of cane sugar (C12H2O11) in water is isotonic with another solution containing 1.75 mass
percent of a solute. Assuming identical densities of both the solution, calculate the molar mass of solute
Two isotonic solutions will have the same molar concentrations. Since densities of both the solutions are identical, their
molar concentrations will also be identical. Hence,
1.75 g 10 g 1.75 ¥ 342
= or M2 = g mol-1 = 59.85 g mol-1
M2 342 g mol-1 10
Molar Mass of an Electrolyte
as shown in the following expression

m2 ( M 2 )0
Relative lowering of vapour pressure M2 = = (Eq. 26)
(m1 / M1 )[i (- Dp1 )0 / p1 ]
* i

Kb m2 ( M 2 )0
Elevation of boiling point M2 = = (Eq. 29)
[i ( DTb )0 ] m1 i

Kf m2 ( M 2 )0
Depression in freezing point M2 = = (Eq. 31)
[i ( DTf )0 ] m1 i

M2 = ÊÁ 2 ˆ˜
m RT ( M 2 )0
Osmotic pressure = (Eq. 34)
Ë V ¯ [i (P )0 ] i
where the quantity (M2)0 refers to the molar mass of electrolyte assuming it to be nonelectrolyte and i is the van’t Hoff
factor. Since i > 1 for an electrolyte, the molar mass of an electrolyte is reduced by a factor i of the molar mass of the
corresponding nonelectrolyte. For example, for a dilute solution of sodium chloride, we will have
5.22 Complete Chemistry—JEE Main

( M 2 )0 (23 + 35.5) g mol-1


M= = = 29.25 g mol-1
2 2
Molecules of ethanoic acid and benzoic acid dimerise in benzene due to hydrogen bondings.
O ... H O
2 CH3COOH   CH3 - C C CH3
O H ... O

( M 2 )0 60 g mol-1
The van’t Hoff factor in this case is 1/2. Hence M2 = = = 120 g mol-1
i 1/2
Number Average Molar Mass If a solution contains more than one solute species, the molar mass determined by
substituting the value of total molality /concentration in the appropriate expression of colligative property gives the

N1M1 + N 2 M 2 +  Âi Ni M i
Mn = =
N1 + N 2 +  Âi N i
where Ns represent the number of species and Ms represent the respective molar masses.

MULTIPLE CHOICE QUESTIONS ON SECTION 3

1. Relative lowering of vapour pressure of solvent when a nonvolatile solute is added is


(a) equal to the mole fraction of solvent in solution (b) equal to the mole fraction of solute in solution
(c) independent of mole fraction of solvent (d) dependent on the nature of solute
2. A solution of 30 g of an unknown compound in 116 g of solvent (molar mass = 58 g mol–1) has vapour pressure
of 0.475 atm. If the vapour pressure of pure solvent is 0.5 atm, the molar mass of unknown compound is
(a) 100 g mol–1 (b) 200 g mol–1 (c) 250 g mol–1 (d) 300 g mol–1
3. A solution contains 20 g glucose (C6H12O6) and 34.2 g of sucrose (C12H22O11) in 108 g water. If the vapour
pressure of pure water is 35 mmHg, the decrease in vapour pressure of the solvent will be
(a) 0.85 mmHg (b) 0.98 mmHg (c) 1.19 mmHg (d) 2.25 mmHg
–1) is 400 K. If its D H = 40.0 kJ mol–1, the value of its
4. The boiling point of a solvent (molar mass = 50 g mol vap
boiling point elevation constant is
(a) 0.85 K kg mol–1 (b) 1.66 K kg mol–1 (c) 1.25 K kg mol–1 (d) 1.82 K kg mol–1
5. A solution of 0.5 g of a solute (molar mass = 150 g mol–1) in 50 g of a solvent yields a boiling point elevation
of 0.40 K. Another solution of 0.60 g of an unknown solute in the same mass of solvent exhibits a boiling point
elevation of 0.8 K. The molar mass of unknown solute is
(a) 60 g mol–1 (b) 90 g mol–1 (c) 120 g mol–1 (d) 180 g mol–1
6. Given is the mass fraction, w2, of solute in a solution. If r is the density of solution, the expression of osmotic
pressure is
w r RT w RT w RT
(a) P = w2rRT (b) P = 2 (c) P = 2 (d) P = 2
M2 r M 2r
–3
7. The density of 9.0% by mass solution of glucose (C6H12O6) is 1.05 g cm at 300 K. The osmotic pressure of the
solution is
(a) 9.22 atm (b) 10.40 atm (c) 11.2 atm (d) 12.92 atm
8. A 0.120 molal solution of CsCl (with ionizes in the solution as CsCl  Cs + + Cl- ) freezes at –0.4 °C. The van’t
Hoff factor and degree of dissociation of CsCl in the solution, respectively, are (Given Kf(water) = 1.86 K kg
mol–1)
(a) 1.79, 0.79 (b) 1.5, 0.5 (c) 1.92, 0.92 (d) 1.2, 0.2
Solutions 5.23

9. The osmotic pressure of a solution containing 0.10 g of haemoglobin in 10.0 cm3 of solution is 30.0 Torr at
300 K. The molar mass of haemoglobin is
(a) 4500 g mol–1 (b) 5452 g mol–1 (c) 6232 g mol–1 (d) 6932 g mol–1
10. A 0.101 M solution of Ba3(PO4)2 is isotonic with 0.004 M glucose solution. The degree of dissociation and van’t
Hoff factor of Ba3(PO4)2, respectively, are
(a) 4, 0.5 (b) 4, 0.65 (c) 4, 0.75 (d) 4, 0.85
11. If potassium ferrocyanide is 85% ionized in a solution, its van’t Hoff factor will be
(a) 2.4 (b) 3.4 (c) 3.9 (d) 4.4
12. Which of the following pairs of solutions will be isotonic to each other?
(a) 0.01 M Glucose + 0.01 M Sodium chloride
(b) 0.01 M Potassium ferrocyanide + 0.02 M Calcium nitrate
(c) 0.01 M Potassium ferricyanide + 0.015 M Potassium nitrate
(d) 0.01 M Magnesium chloride + 0.01 M Mercurous nitrate
13. The van’t Hoff factor of solutes A, B and C in aqueous solutions are 0.8, 1.6 and 1.2, respectively. The freezing
point of equimolar solutions follow the order
(a) A > B > C (b) A > C > B (c) B > A > C (d) B > C > A
14. Arrange the aqueous solutions of A(0.01 M sodium chloride), B(0.01 M glucose) and C(0.01 M acetic acid) in
increasing order of boiling points.
(a) A > B > C (b) A > C > B (c) B > A > C (d) B > C > A
15. Which of the following solute will have minimum value of van’t Hoff factor?
(a) 0.1 molal solution of acetic acid in water
(b) 0.1 molal solution of benzoic acid in benzene
(c) 0.1 molal solution of glucose in water
(d) 0.1 molal solution of sodium chloride in water

ANSWERS
1. (b) 2. (d) 3. (c) 4. (b) 5. (b) 6. (b)
7. (d) 8. (a) 9. (c) 10. (c) 11. (d) 12. (d)
13. (b) 14. (b) 15. (b)

HINTS AND SOLUTIONS


1. Relative lowering of vapour pressure is equal to the mole fraction of solute in the solution.
m2 30 g
2. M2 = = -1
= 300 g mol–1
(m1 / M1 )(- Dp1 / p1 ) (116 g /58 g mol )(0.025 atm / 0.5 atm)
*

m2 20 g 1 m3 34.2 g 1
3. Amount of glucose, n2 = = = mol ; Amount of sucrose, n3 = = -1
= mol
M 2 180 g mol-1 9 M 3 342 g mol 10
m1 108 g
Amount of water, n1 = = = 6 mol
M1 18 g mol-1
Mole fraction of solutes in solution,
n2 + n3 [(1 / 9) + (1 / 10)] mol 0.211 mol
x= = = = 0.034
n1 + n2 + n3 6 mol + [(1 / 9) + (1 / 10) mol] 6.211 mol
Vapour pressure of solution, p1 = x2p*1 = (0.034) (35 mmHg) = 1.19 mmHg
5.24 Complete Chemistry—JEE Main

M1 RTb*2 (0.05 kg)(8.314 J K -1 mol -1 )(400 K ) 2


4. Kb = = = 1.66 K kg mol–1
D vap H (40 ¥ 103 J mol -1 )
5. The boiling point elevation constant of solvent is
DTb DTb 0.40 K
Kb = = =
m (m2 / M 2 ) / m1 (0.5 g/150 g mol-1 ) /(0.05 kg)
Molar mass of unknown solute is
Ê K ˆ Ê m ˆ È (0.40 K )(0.05 kg) 1 ˘ È 0.6 g ˘
M2 = Á b ˜ Á 2 ˜ = Í = 90 g mol–1
Ë DTb ¯ Ë m1 ¯ Î (0.5 g /150 g mol-1 ) 0.8 K ˙˚ ÍÎ 0.05 kg ˙˚

Ê n2 ˆ (m2 / M 2 ) RT Ê m2 ˆ RT r w2 RT r
6. P = cRT = ÁË ˜¯ RT = = =
V (m1 + m2 ) / r ÁË m1 + m2 ˜¯ M 2 M2

w2 RT r (0.09)(0.082 L atm K -1 mol)(300 K )(1.05 ¥ 103 g L-1 )


7. P = = = 12.92 atm
M2 (180 g mol-1 )
8. The depression in freezing point assuming solute as nonelectrolyte is
(–DTf )0 = Kf m = (1.86 K kg–1 mol–1)(0.12 mol kg–1) = 0.223 K
- DTf 0.4 K
The van’t Hoff factor is i= = = 1.79
( - DTf )0 0.223 K
i - 1 1.79 - 1
The degree of dissociation is a= = = 0.79
n -1 2 -1
RT ˆ È 0.10 g ˘ È (0.082 L atm K -1 mol -1 )(300 K ) ˘
9. M2 = ÊÁ 2 ˆ˜ ÊÁ
m
= ˙ = 8.2 ¥ 760 g mol = 6232 g mol
–1 –1
Ë V ¯ Ë P ˜¯ ÍÎ10.0 ¥ 10-3 L ˙˚ ÍÎ (30 / 760) atm ˚
P (0.004 M ) RT i -1 4 -1 3
10. i = = = 4; a= = = = 0.75
(P )0 (0.001 M ) RT n -1 5 -1 4
i -1
11. a = 0.85; 0.85 = fi i = 4.4
5 -1
12. The total concentration of species, assuming complete ionization wherever needed, are as follows.
0.01 M Glucose 0.01 M ˘
NaCl Æ Na + + Cl- 0.02 M ˙˚
not isotonic
0.01 M 0.01 M

K 4 [Fe(CN)6 ] Æ 4K + + [Fe(CN)6 ]4- 0.05 M ˘


0.04 M 0.01 M ˙ not isotonic
Ca(NO3 )2 Æ Ca 2+ + 2 NO3- ˙
0.02 M 0.04 M
0.06 M ˙˚

K 3 [Fe(CN)6 ] Æ 3K + + [Fe(CN)6 ]3- 0.04 M ˘


0.03 M 0.01 M ˙ not isotonic
KNO3 Æ K + + NO3- ˙
0.015 M 0.015 M
0.03 M ˙˚

MgCl2 Æ Mg 2+ + 2Cl- 0.003 M ˘


0.02 M
0.01 M ˙ isotonic
Hg 2 (NO3 )2 Æ Hg 2+ - ˙
2 + 2 NO3 0.03 M ˙˚
0.01 M 0.02 M
Solutions 5.25

13. (–DTf ) = i (–DTf )0. For equimolar solutions, large the van’t Hoff factor, larger the decrease in freezing point and
hence minimum will be the freezing point.
14. DTb = Kb m, larger the molality (or concentration for dilute solution), larger the value of DTb. Hence, larger will be
the boiling point. Though, all the three solutions are equimolar, but the number of species available (or the total
concentration of species) will follow the order A > C > B. The same order holds good for boiling points.
15. Benzoic acid in benzene dimerises and hence its van’t Hoff factor is 1/2 which will be minimum amongs other
values.

MULTIPLE CHOICE QUESTIONS FOR THE ENTIRE CHAPTER

Composition of a Solution
1. Which of the following is true?
(a) Molarity of a solution is independent of temperature.
(b) Molality of a solution is independent of temperature.
(c) Mole fraction of a solute in a solution is dependent on temperature.
(d) The unit of molality is mol dm–3.
2. The molality of a solute in the solution is the
(a) mass of solute per dm3 of solution (b) amount of solute per dm3 of solution
(c) amount of solute per kg of solution (d) amount of solute per kg of solvent
3. The unit of molality is
(a) mol (b) mol dm–3 (c) mol kg–1 (d) mol–1
4. The unit of molarity is
(a) mol (b) mol–1 (c) mol dm–3 (d) mol kg–1
5. The expression relating molarity of a solution with its molality is
1 + mM 2 mr 1 + mr mM 2
(a) M = (b) M = (c) M = (d) M =
mr 1 + mM 2 mM 2 1 + mr
where the various symbols have their usual meanings.
6. The expression relating mole fraction of solute in a solution with its molality is
1 + mM1 1 - mM1 mM1 mM1
(a) x2 = (b) x2 = (c) x2 = (d) x2 =
mr mM1 1 + mM1 1 - mM1
where the various symbols have their usual meanings.
7. The expression relating mole fraction of solute in a solution with its molarity is
MM1 MM1 MM1 MM1
(a) x2 = (b) x2 = (c) x2 = (d)
M ( M1 + M 2 ) + r M ( M1 - M 2 ) + r M ( M1 + M 2 ) - r M ( M1 - M 2 ) - r
where the various symbols have their usual meanings.
8. The density of a 10.0% by mass KC1 solution in water is 1.06 g cm–3. Its molarity is
(a) 1.489 M (b) 1.420 M (c) 1.420 mol kg–1 (d) 1.489 mol kg–1
9. What mass of ethanol be added to 1.0 kg water to have the mole fraction of ethanol equal to 0.20?
(a) 63.89 g (b) 6.39 g (c) 638.89 g (d) 683.89 g
10. The volume of 96% H2SO4 (density 1.83 g mL–1) required to prepare 2.0 L of 3.0 M H2SO4 solution is
(a) 33.47 mL (b) 3.347 mL (c) 334.7 mL (d) 343.7 mL
–3
11. The density of 95.2 mass % H2SO4 is 1.53 g cm . The molarity of this solution is
(a) 12.5 mol dm–3 (b) 13.6 mol dm–3 (c) 14.8 mol dm–3 (d) 16.2 mol dm–3
5.26 Complete Chemistry—JEE Main

12. The mole fraction of a solute in carbon tetrachloride is 0.235. The molality of the solution is about
(a) 2.0 mol kg–1 (b) 1.5 mol kg–1 (c) 1.0 mol kg–1 (d) 0.5 mol kg–1
Ideal and Nonideal Solutions
13. Which of the following is true?
(a) The ideal behaviour of a liquid solution is due to the fact that the different molecules present in it do not interact
with one another.
(b) Henry’s law deals with the variation of solubility of gas with temperature.
(c) The constituents of an ideal solution follow Raoult’s law under all conditions.
(d) The addition of a nonvolatile solute to a volatile solvent decreases the boiling point of the latter.
14. For an ideal binary liquid solution with pA > pB, which of the following is true?
(a) (xA)liquid = (xA)vapour
(b) (xA)liquid > (xA)vapour
(c) (xA)liquid < (xA)vapour
(d) (xA)liquid and (xA)vapour do not bear any relationship with each other
15. Which of the following behaviours is not true for an ideal binary liquid solution?
(a) Plot of pA versus xA (mole fraction of A in liquid phase) is linear
(b) Plot of pB versus xB is linear
(c) Plot of ptotal versus xA (or xB) is linear
(d) Plot of ptotal versus xA (or xB) is nonlinear
16. For a dilute solution, Raoult’s law states that
(a) the lowering of vapour pressure is equal to the mole fraction of solute
(b) the relative lowering of vapour pressure is equal to the mole fraction of solute
(c) the relative lowering of vapour pressure is proportional to the amount of solute in solution
(d) the vapour pressure of the solution is equal to the mole fraction of solvent
17. Raoult’s law is obeyed by a binary liquid solution when
(a) the forces of attractions between like molecules are greater than those between unlike molecules
(b) the forces of attractions between like molecules are smaller than those between unlike molecules
(c) the forces of attractions between like molecules are more or less identical with those between unlike molecules
(d) the volume occupied by unlike molecules are different
18. An azeotropic solution of two liquids has a boiling point lower than either of the boiling points of the two liquids
when it
(a) shows negative deviation from Raoult’s law (b) shows positive deviation from Raoult’s law
(c) shows no deviation from Raoult’s law (d) is saturated
19. An azeotropic solution of two liquids has a boiling point higher than either of the boiling points of the two liquids
when it
(a) shows negative deviation from Raoult’s law (b) shows positive deviation from Raoult’s law
(c) shows no deviation from Raoult’s law (d) is saturated
20. Which of the following behaviours is true for an ideal binary liquid solution?
(a) Plot of ptotal versus yA (mole fraction of A in vapour phase) is linear
(b) Plot of ptotal versus yB is linear
(c) Plot of l/ptotal versus yA (or yB) is linear
(d) Plot of l/ptotal versus yA (or yB) is nonlinear
21. A binary solution of ethanol and n-heptane is an example of
(a) an ideal solution
(b) a nonideal solution with positive deviations from Raoult’s law
(c) a nonideal solution with negative deviations from Raoult’s law.
(d) a solution exhibiting positive deviations at low concentrations and negative deviations at higher concentration
Solutions 5.27

22. An azeotropic solution of two liquids has boiling point higher than either of them when it
(a) shows negative deviations from Raoult’s law (b) shows positive deviations from Raoult’s law
(c) shows no deviations from Raoult’s law (d) follows Henry’s law
23. Which of the following solutions is expected to show positive deviations from Raoult’s law?
(a) Pyridine-formic acid (b) Ethanol-hexane
(c) Acetone-chloroform (d) Hydrochloric acid-water
24. Which of the following solutions is expected to show negative deviations from Raoult’s law?
(a) Ethanol-hexane (b) Ethanol-water (c) Acetone-chloroform (d) Ethyl ether-acetone
25. The dissolution of sulphuric acid in water will exhibit
(a) negative deviations from ideal behaviour
(b) positive deviations from ideal behaviour
(c) ideal behaviour
(d) negative or positive deviation depending upon the mass of H2SO4 dissolved.
26. The formation of an ideal solution involves
(a) increase in enthalpy (b) increase in entropy (c) increase in free energy (d) increase in energy
Vapour Pressure
27. The vapour-pressure lowering of a solvent is proportional to
(a) the mole fraction of the solute (b) the mole fraction of the solvent
(c) the molality of the solvent (d) the normality of the solution
28. The vapour pressure of a solvent in a solution is proportional to
(a) the mole fraction of the solute (b) the mole fraction of the solvent
(c) the molality of the solvent (d) the normality of the solute
29. The vapour pressure of pure liquid is 70 Torr at 27 °C. The vapour pressure of a solution of this liquid and another
liquid (mole fraction 0.2) is 84 Torr at 27 °C. The vapour pressure of pure liquid B at 27 °C is
(a) 140 Torr (b) 280 Torr (c) 160 Torr (d) 200 Torr
30. The vapour pressure of pure benzene and toluene are 160 Torr and 60 Torr, respectively. The mole fraction of
benzene in vapour phase in contact with equimolar solution of benzene and toluene would be
(a) 0.50 (b) 0.84 (c) 0.73 (d) 0.27
31. The mass of sucrose to be added to 300 g of water to lower its vapour pressure by 1.0 mmHg at 25°C is (Given:
*(water) = 23.8 mmHg)
(a) 249.9 g (b) 329.4 g (c) 215.2 g (d) 342.2 g
32. A solution contains 1 mol of pentane (p* = 450 mmHg) and 4 mol of hexane (p* = 150 mmHg), the mole fraction
of pentane in vapour phase will be
(a) 0.454 (b) 0.429 (c) 0.641 (d) 0.75
Depression of Freezing Point

(a) ice (b) solid solution of sugar and ice


(c) sugar (d) a compound formed from sugar and water
34. In the phenomenon of osmosis through the semipermeable membrane
(a) solvent molecules pass from solution to solvent (b) solvent molecules pass from solvent to solution
(c) solute molecules pass from solution to solvent (d) solute molecules pass from solvent to solution
35. The freezing point of a 0.05 molal solution of a nonelectrolyte in water Kf = 1.86 K kg mol–1 is
(a) –1.86 °C (b) –0.93 °C (c) –0.093 °C (d) 0.93 °C
36. Which of the following exhibits the greatest freezing-point lowering?
(a) 0.1 mol kg–1 NaCl (b) 0.1 mol kg–1 CaCl2 (c) 0.1 mol kg–1 HC2H3O2 (d) 0.1 mol kg–1 NaC2H3O2
5.28 Complete Chemistry—JEE Main

37. The freezing point depression constant is given as


(a) Kf = M1RTf*2/DfusHm (b) Kf = M1R/Tf*2 DfusHm
*2
(c) Kf = MTf /RDfusHm (d) Kf = DfusHm/MRTf*2
where the various symbols have their usual meanings.
38. The unit of freezing point depression constant is
(a) K mol–1 (b) K kg–1 mol–1 (c) K kg mol–1 (d) K kg–1
39. Arrange the following (0.1 molal) solutions in the increasing order of freezing point.
Soln of Ca3(PO4)2; Soln of Na2SO4; Soln of glucose
(I) (II) (III)
(a) I > II > III (b) I > III > II (c) III > II > I (d) III > I > II
Elevation of Boiling Point
40. The boiling point elevation constant is given as
(a) Kb = M1R T*b 2/DvapHm (b) Kb = M1R/Tb*2 DvapHm (c) Kb = M1Tb*2/R D vav Hm (d) Kb = DvapHm/M1RTb*2
where the various symbols have their usual meanings.
41. The boiling point of water at 735 Torr is 99.07 °C. The mass of NaCl added in 100 g water (Kb = 0.51 K kg mol–1)
to make its boiling point 100 °C is
(a) 10.68 g (b) 5.34 g (c) 2.67 g (d) 26.7 g
42. The elevation of boiling point of water produced by dissolving 1.17 g sodium chloride in 100 g water (Kb = 0.512
K kg mol–1) is
(a) 0.103 K (b) 0.205 K (c) 0.309 K (d) 0.410 K
43. The normal boiling point of pure ethyl acetate is 77.06 °C. A solution of 50.0 g of naphthalene (C10H8) dissolved
in 150 g of ethyl acetate boils at 84.27 °C. The boiling point elevation constant of ethyl acetate is
(a) 2.77 K kg mol–1 (b) 1.88 K kg mol–1 (c) 2.12 K kg mol–1 (d) 1.54 K kg mol–1
44. The boiling point of a solution at a height of 10 km above sea level is
(a) found to be greater than that at sea level (b) found to be lesser than that at sea level
(c) found to be the same as that at sea level (d) dependent upon the air dissolved in solution
45. A solution of sucrose in 100 g of water boils at 100.25 °C. If this solution is boiled till its temperature is 101 °C,
the mass of water evaporated from the solution is
(a) 25 g (b) 50 g (c) 75 g (d) 85 g
46. Molal elevation boiling point constant Kb depends
(a) only on the nature of solvent
(b) only on the nature of solute
(c) on the nature of both solute and solvent
(d) remains constant at a place at different heights from the sea level.
Osmotic Pressure
47. The osmotic pressure of a solution (density = 1.02 g cm–3) containing 50 g glucose (C6H12O6) in 1 kg of water
at 300 K is
(a) 67.39 kPa (b) 673.43 kPa (c) 6.74 kPa (d) 673.85 Pa
48. The osmotic pressure of 0.1 M sodium chloride solution at 27 °C is
(a) 4.0 atm (b) 2.46 atm (c) 4.92 atm (d) 1.23 atm
49. Which of the following pairs of solutions are expected to be isotonic?
(a) 0.1 M urea and 0.1 M NaCl (b) 0.1 M urea and 0.1 M MgCl2
(c) 0.1 M Ca(NO3)2 and 0.1 M Na2SO4 (d) 0.1 M NaCl and 0.1 M glucose
50. 250 mL of a solution containing 10.0 g of sodium chloride and glucose produces 15 atm of osmotic pressure at
27 °C. The mass percentage of sodium chloride in the mixture is
(a) 25% (b) 33.8% (c) 52.4% (d) 66.2%
Solutions 5.29

51. Isotonic solutions have the same


(a) freezing point (b) boiling point (c) osmotic pressure (d) vapour pressure
52. Through the semipermeable membrate, there can occur

Van’t Hoff Factor


53. The van’t Hoff factor of the compound K3Fe(CN)6 in dilute solution is
(a) 1 (b) 2 (c) 3 (d) 4
54. The expression relating degree of dissociation of the weak electrolyte AxBy with its van’t Hoff factor is
i -1 i -1 x + y -1 x + y -1
(a) a = (b) a = (c) a = (d) a =
x + y -1 x + y -1 i -1 i +1
55. Assuming 100% ionization, the increasing order of the freezing point of the solution will be
(a) 0.10 mol kg–1 Ba3(PO4)2 < 0.10 mol kg–1 Na2SO4 < 0.10 mol kg–1 KCl
(b) 0.10 mol kg–1 KCl < 0.10 mol kg–1 Na2SO4 < 0.10 mol kg–1 Ba3(PO4)2
(c) 0.10 mol kg–1 Na2SO4 < 0.10 mol kg1 Ba3(PO4)2 < 0.10 mol kg1 KCl
(d) 0.10 mol kg–1 KCl < 0.10 mol kg–1 Ba3(PO4)2 < 0.10 mol kg–1 Na2SO4
56. When mercuric iodide is added to an aqueous solution of potassium iodide, the
(a) freezing point is lowered (b) freezing point is elevated
(c) freezing point does not change (d) effect on freezing point cannot be predicted
57. The freezing point of equimolal solution will be lowest for
(a) anilinium chloride (b) calcium nitrate (c) glucose (d) sodium phosphate
58. The van’t Hoff factor of benzoic acid in benzene is found to be
(a) 1 (b) 2 (c) 0.5 (d) 1.5
59. Which of the following electrolytes would have the same van’t Hoff factor as that of potassium ferricyanide?
(a) NaCl (b) Na2SO4 (c) A12(SO4)3 (d) A1(NO3)3
60. The van’t Hoff factor for 0.1 molal Ba(NO3)2 solution is 2.74. Its degree of dissociation is
(a) 0.74 (b) 0.87 (c) 0.91 (d) 1.0
61. The van’t Hoff factor of acetic acid in water will be
(a) equal to one (b) equal to two (c) in between one and two (d) more than two
62. The molar mass of calcium nitrate as determined from the colligative properties is found to be 65.6 g mol–1. The
degree of dissociation of the salt is
(a) 0.25 (b) 0.50 (c) 0.75 (d) 0.85
63. 5 g of a nonvolatile solute in 100 g of water has a vapour pressure of 2980 Pa. If the vapour pressure of pure water
is 3000 Pa, the molar mass of solute is
(a) 120 g mol–1 (b) 125 g mol–1 (c) 130 g mol–1 (d) 134 g mol–1
64. The molar mass of benzoic acid in benzene as determined by colligative properties is found to be
(a) 122 g mol–1 (b) 183 g mol–1 (c) 244 g mol–1 (d) 305 g mol–1
Determination of Molar Mass
65. The expression to compute molar mass of a solute from the relative lowering of vapour pressure of a solvent is
(a) M2 = m2/{(m1/M1) (–Dp1)} (b) M2 = m2/{(m1/M1) (–Dp1/p*1)}
(c) M2 = m2(–Dp1/p*1)/(m1/M1) (d) M2 = (m1/M1) (–Dp1/p*1)/m2
where the various symbols have their usual meanings.
66. The expression to compute molar mass of a solute from the elevation of boiling point of a solvent is
K b m1 DTb m2 K b m2 DTb m1
(a) M2 = (b) M2 = (c) M2 = (d) M2 =
DTb m2 K b m1 DTb m1 K b m2
where the various symbols have their usual meanings.
5.30 Complete Chemistry—JEE Main

67. The expression to compute molar mass of a solute from the depression in freezing point of a solvent is
K f m1 (-DTf ) m2 (-DTf ) m1 K f m2
(a) M2 = (-DT ) m (b) M2 = (c) M2 = (d) M 2 =
f 2 K f m1 K f m2 (- DTf ) m1
where the various symbols have their usual meanings.
68. The molar mass of a solute from the osmotic pressure which it produces when a mass m of solute is dissolved in
volume V is given by the expression
Ê mˆ P m PR
(a) M = ÊÁ ˆ˜ (c) M = ÊÁ ˆ˜ P RT (d) M = ÊÁ ˆ˜
m RT m
(b) M = ÁË ˜¯
ËV ¯ P V RT ËV ¯ ËV ¯ T
69. The vapour pressure of a solution having 2.0 g of a solute X (molar mass 32 g mol–1) in 100 g of CS2 (vapour
pressure 854 Torr) is 848.9 Torr. The molecular formula of the solute is
(a) X (b) X2 (c) X4 (d) X8
70. Pure Benzene freezes at 5.4 °C. A solution of 0.223 g of phenyl acetic acid (C6H5CH2COOH) in 4.4 g of benzene
(Kf = 5.12 K kg mol–1) freezes at 4.47 °C. From this observation, one can conclude that
(a) phenyl acetic acid exists as such in benzene
(b) phenyl acetic acid undergoes partial ionization in benzene
(c) phenyl acetic acid undergoes complete ionization in benzene
(d) phenyl acetic acid dimerizes in benzene
71. The vapour pressure of a solution of 5 g of a nonelectrolyte in 100 g water at a particular temperature is 2950 Pa
and that of pure water at the same temperature is 3000 Pa. The molar mass of the solute is
(a) 54 g mol–1 (b) 119 g mol–1 (c) 179 g mol–1 (d) 229 g mol–1
72. The vapour pressure of a pure liquid A is 10.0 Torr. at 27 °C. One gram of B is dissolved in 20 g of A, the vapour
pressure is lowered to 9.0 Torr. If the molar mass of A is 200 g mol –1, the molar mass of B is
(a) 75 g mol–1 (b) 85 g mol–1 (c) 100 g mol–1 (d) 115 g mol–1
73. A solution contains 10 g of a solute and 116 g of acetone at 30 °C. Its vapour pressure is 290 Torr. If the vapour
pressure of pure acetone is 300 Torr at 30 °C, the molar mass of solute is
(a) 75 g mol–1 (b) 100 g mol–1 (c) 125 g mol–1 (d) 150 g mol–1
74. The molar mass of acetic acid dissolved in benzene determined from the depression in freezing point is found to
be
(a) 30 g mol–1 (b) 60 g mol–1 (c) 120 g mol–1 (d) 240 g mol–1
75. The freezing point of a solution containing 36 g of a compound (empirical formula : CH2O) in 1.20 kg of water
is found to be – 0.93 °C. The molecular formula of the solute is
(a) CH2O (b) C2H4O2 (c) C3H6O3 (d) C4H8O4
76. The molar mass of sodium chloride determined by osmotic pressure would be
(a) 58.5 g mol–1 (b) more than 58.5 g mol–1 (c) less than 58.5 g mol–1 (d) 117.0 g mol–1
77. A solution containing 8.6 g L–1 of urea (molar mass 60 g mol–1) is isotonic with a 5% solution of unknown solute.
The molar mass of the solute will be
(a) 348.9 g mol–1 (b) 174.5 g mol–1 (c) 87.3 g mol–1 (d) 34.89 g mol–1
78. Which of the following physical properties is used to determine the molar mass of a polymer solution?
(a) Relative lowering of vapour pressure (b) Elevation of boiling point
(c) Depression of freezing point (d) Osmotic pressure
79. The molar mass of sodium chloride determined by osmotic pressure measurement will come out to be
(a) 58.5 g mol–1 (b) 117.0 g mol–1
(c) 29.25 g mol–1 (d) Anywhere in between 29.25 and 58.5 g mol–1
Solutions 5.31

ANSWERS
1. (b) 2. (d) 3. (c) 4. (c) 5. (b) 6. (c)
7. (b) 8. (b) 9. (c) 10. (c) 11. (c) 12. (a)
13. (c) 14. (b) 15. (d) 16. (b) 17. (c) 18. (b)
19. (a) 20. (c) 21. (b) 22. (a) 23. (b) 24. (c)
25. (a) 26. (b) 27. (a) 28. (b) 29. (a) 30. (c)
31. (a) 32. (b) 33. (a) 34. (b) 35. (c) 36. (b)
37. (a) 38. (c) 39. (c) 40. (a) 41. (b) 42. (b)
43. (a) 44. (b) 45. (c) 46. (a) 47. (b) 48. (c)
49. (c) 50. (b) 51. (c) 52. (b) 53. (d) 54. (a)
55. (a) 56. (b) 57. (d) 58. (c) 59. (d) 60. (b)
61. (c) 62. (c) 63. (d) 64. (c) 65. (b) 66. (c)
67. (d) 68. (a) 69. (d) 70. (d) 71. (a) 72. (c)
73. (d) 74. (c) 75. (b) 76. (c) 77. (a) 78. (d)
79. (c)

HINTS AND SOLUTIONS


(10 g/74.5 g mol-1 )
8. M = = 1.42 ¥ 10–3 mol cm–3 = 1.42 mol dm–3
(100 g / 1.06 g cm -3 )
(m / 46 g mol-1 ) m 0.2 ¥ 1000 1
9. 0.2 = -1 -1
; = ¥ = 13.89 i.e. m = 13.89 ¥ 46 g = 638.89 g
(m / 46 g mol ) + (1000 g / 18 g mol ) 46 g 18 0.8
10. Volume of 96% H2SO4 = (VM) Mm (100/98)/r
Ê 100 ˆ Ê 1 ˆ
= (2.0 L)(3.0 mol L–1)(98 g mol–1) ÁË ˜¯ Á -1 ˜ = 334.7 mL
96 Ë 1.83 g mL ¯
100 ˆ 3 0.10
= ÊÁ
m 100 g
11. Let there be 100 g of solution. Its volume will be V= = ˜ cm = dm3
r 1.53 g cm -3 Ë 1.53 ¯ 1.53
m 95.2 g
This solution contains 95.2 g of H2SO4. Its amount is n = =
M m 98 g mol-1

n (95.2 / 98)
Hence, the molarity of solution is M= = = 14.86 mol dm -3
V (0.1 / 1.53) dm3
12. Let
Amount of solute = 0.235 mol; Amount of solvent = (1 – 0.235) mol = 0.765 mol
Mass of solvent, m = nMm = (0.765 mol) (154 g mol–1) = 117.81 g
n 0.235 mol
Molality of solution = =  2.0 mol kg -1
m 117.81 ¥ 10-3 kg
25. The forces of attraction between solute and solvent are larger than those existing in solute-solute and solvent-
solvent. The solution thus exhibits negative deviation from ideal behaviour.
29. p = xA pA* + xB pB* fi 84 Torr = (0.8) (70 Torr) + (0.2) (p*B) fi p*B = 140 Torr
30. pb = xb pb = (0.5) (160 Torr) = 80 Torr; ptotal = xb p*b + xt p*t = (0.5) (160 Torr) + (0.5 Torr) (60 Torr) = 110 Torr
pb 80 Torr
yb = = = 0.73
ptotal 110 Torr
5.32 Complete Chemistry—JEE Main

31. Since p = x1 p1, we get


p 23.8 - 1.0 n1
=
(300 / 18) mol
x1 = * = = 0.958. Now x1 =
p1 23.8 n1 + n2 (300 / 18) mol + (m/342 g mol-1 )
Equating these two, we get
(300 / 18) {(300 / 18)(1 - 0.958)}(342 g )
= 0.958 or m = = 249.89 g
(300 / 18) + (m / 342 g) 0.958
32. In vapour phase, we will have
ppentane = {1/(1 + 4)} (450 mmHg) = 90 mmHg; ppentane = {4/(1 + 4)} (150 mmHg) = 120 mmHg
ppentane 90
ypentane = = = 0.429
ppentane + phexane 90 + 120
39. Larger the sum of stoichiometric number of ions of the molecule, larger ions it produces in solution, and thus
larger decrease in freezing point. Consequently, the freezing point is the lowest.
DT (100 - 99.07)
41. m = b = mol kg -1
Kb 0.51
Assuming NaCl to be completely dissociated, we will have
1 Ê 100 - 99.07
Molality of NaCl = Á mol kg -1 ˆ˜
2Ë 0.51 ¯
1 1 0.93 ˆ
Mass of NaCl in 100 g water = ÊÁ ˆ˜ ÊÁ (58.5) g = 5.33 g
Ë 10 ¯ 2 Ë 0.51˜¯
(1.17 / 58.5) mol
42. Molality of sodium chloride = = 0.2 mol kg -1
0.1 kg
Molality of ions in solution = 2 ¥ 0.2 mol kg–1 DTb = Kbm = (0.512 K kg mol–1) (2 ¥ 0.2 mol kg–1) = 0.205 K
43. Molality of solution is
m2 / M 2 (50 g ) /(128 g mol-1 )
m= = = 2.604 mol kg -1
m1 0.150 kg
DTb (84.27 - 77.06) K
Kb = = -1
= 2.77 K kg mol -1
m 2.604 mol kg
44. At 10 km above sea level, the atmospheric pressure will be less than that existing at sea level. Hence, the boiling
point of the solution will be less.
45. We have from DTb = m Kb,
( DTb ) 2 m2 n2 /(mwater ) 2 (mwater )1
= = =
( DTb )1 m1 n2 (mwater )1 (mwater ) 2
Hence (mwater)2 = (mwater)1 {DTb)1/(DTb)2} = (100 g) (0.25K/1K) = 25 g.
Mass of water evaporated = 100 g – 25 g = 75 g
(50 / 180) mol
47. Concentration of glucose = = 0.27 mol L-1
(1.05 kg/1.02 kg L-1 )
Osmotic pressure, P = cRT = (0.27 mol L–1) (8.314 kPa L–1 K–1 mol–1) (300 K) = 673.43 kPa
48. Osmotic pressure, P = cRT= (2 ¥ 0.1 mol L–1) (0.082 atm L K–1 mol–1) (300 K) = 4.92 atm
50. Let m be the mass of NaCl in the mixture.
mNaCl m 10.0 g - m
Amount of NaCl, nNaCl = = -1
; Amount of glucose, nglucose =
M NaCl 58.5 g mol 180 g mol-1
Total amount of species in the mixture
Solutions 5.33

2m 10.0 g - m ˆ
ntotal = {nNa + + nCl- } + nglucose = ÊÁ
1
+ ˜¯
Ë 58.5 180 g mol -1
{(2m / 58.5) + (10.0 g - m) / 180}/ g mol-1
Total concentration of species in the solution, c =
0.250 L
Now P = cRT, we have
15 atm {(2m / 58.5) + (10.0 g - m) / 180}/ g mol-1
=
(0.082 L atm K -1 mol -1 )(300 K) 0.250 L

Ê 2m + 10.0 g - m ˆ g = 15 ¥ 0.250 = 0.1524


or Ë 58.5 180 ¯ (0.082)(300)
(180 ¥ 0.1524 - 10.0)(58.5)
or m= g = 3.38 g
(360 - 58.5)
3.38
Mass percentage of NaCl = ¥ 100 = 33.8 %
10
y+ x-
54. A x B y  xA + yB
1-a ax ay
Total amount of species = 1 + (x + y – 1)a
1 + ( x + y - 1)a i -1
Van’t Hoff factor, i = or a =
1 x + y -1
55. Assuming complete dissociation (–DTf )Ba3(PO4)2 > (–DTf)Na2SO4 > (–DTf ) KCl
Hence (Tf)Ba3(PO4)2 < (Tf)Na2SO4 < (Tf)KCl
58. Benzoic acid dimerizes in benzene. Its van’t Hoff factor will be 1/2.
i -1 2.74 - 1
60. a = = = 0.87
( x + y) - 1 3 -1
62. We have
Ca ( NO3 ) 2  Ca 2+ + 2 NO3-
c(1 – a) ca c(2a)
Total concentration = c(l – a) + ca + c(2a) = c(l + 2a)
Theoretical molecular mass (1 + 2a ) 164 1 + 2a
= fi = fi a = 0.75
Observed molecular mass 1 65.6 1
63. Mole fraction of water in solution is
(100 / 18) mol
x=
(5 g / M ) + (100 / 18) mol
Hence, p = x1 p*1
(100 / 18) mol
2980 Pa = (3000 Pa ). This gives M = 134.1 g mol-1
(5 g / M ) + (100 / 18) mol
64. Benzoic acid dimerizes in benzene. So, its molar mass is twice as that of monomer.
(100 g / 76 g mol-1 )
69. Mole fraction of solvent =
(100 g/76 g mol-1 ) + (2 g / M )
(100 / 76 mol-1 )
Hence, 848.9 Torr = (854 Torr )
(100 / 76 mol-1 ) + (2 g / M )
Solving for M, we get M= 253 g mol–1
5.34 Complete Chemistry—JEE Main

253 amu
Number of S atoms in a molecule of solute = =8
32 amu
( - DTf ) (5.4 - 4.47) 0.223 g/M 0.933
70. m = = mol kg -1 i.e. = mol kg -1
Kf 5.12 (4.4 / 1000) kg 5.12
Solving for M, we get M = 279 g mol–1
279
Number of molecules of phenyl acetic acid = =2
136
Dp (3000 - 2950) Torr 1 (m / M 2 )
71. x2 = – = = . Now x2  2 .
*
p1 3000 Torr 60 m1 / M1
1 (5 g / M )
Hence, = . This gives M = 54 g mol -1
60 (100 g / 18 g mol -1 )
Dp 1 Torr 1 (m / M 2 )
72. x2 = - *
= = Now x2  2
p1 10 Torr 10 (m1 / M1 )
1 (1g / M 2 )
Hence, = . This gives M 2 = 100 g mol -1
10 (20 g/200 g mol -1 )
Dp 10 Torr 1 (m2 / M 2 )
73. x2 = - = = . Now x2  .
p* 300 Torr 30 (m1 / M1 )
1
1 (10 g / M 2 )
Hence, = . This gives M 2 = 150 g mol-1
30 (116 g / 58 g mol-1 )
74. Acetic acid dimerizes in benzene. Its molar mass would be 2 ¥ 60 g mol–1, i.e. 120 g mol–1.
Ê 36 g/M ˆ
75. –DTf = Kf m fi 0.93 K = (1.86 K kg mol–1) Á
Ë 1.20 kg ˜¯
1.86 ¥ 36
This gives M= g mol–1 = 60 g mol–1.
0.93 ¥ 1.20
Molar mass 60 g mol-1
n= = =2
Molar Empirical formula mass 30 g mol-1
Hence, molecular formula is C2H4O2
77. Isotonic solution has the same osmotic pressure and hence the same concentration in the solution.
(8.6 g/60 g mol -1 ) (5 g/M ) 5 ¥ 60
= . Hence, M = g mol -1 = 348.8 g mol -1
1L 0.1 L 8.6 ¥ 0.1
79. The dissolution of sodium chloride produces equal amount of Na+ and Cl– ions. Since the molar mass is the number
average, the molar mass will be 29.25 g mol–1
n1M1 + n2 M 2 M1 + M 2 (23 + 35.5) g mol-1
Mn = = = = 29.25 g mol-1
(n1 + n2 ) 2 2

MULTIPLE CHOICE QUESTIONS FROM AIEEE AND JEE MAIN

1. Which one of the following aqueous solutions will exhibit highest boiling point?
(a) 0 01 M Na2SO4 (b) 0.01 M KNO3 (c) 0.015 M urea (d) 0.015 M glucose
[2004]
2. 6.02 ¥ 10 molecules of urea are present in 100 mL of its solution. The concentration of urea solution is
20

(a) 0.001 M (b) 0.01 M (c) 0.02 M (d) 0.1 M [2004]


(Avogadro constant, NA = 6.02 ¥ 10 mol )
23 –1
Solutions 5.35

3. Which of the following liquid pairs shows a positive deviation from Raoult’s law?
(a) water-hydrochloric acid (b) benzene-methanol
(c) water-nitric acid (d) acetone-chloroform [2004]
4. Which one of the following statements is false?
(a) Raoult’s law states that the vapour pressure of a component over a solution is proportional to its mole fraction.
(b) The osmotic pressure (P) of a solution is given by the equation P = MRT, where M is the molarity of the
solution.
(c) The correct order of osmotic pressure for 0.01 M aqueous solution of each compound is
BaCl2 > KCl > CH3COOH > sucrose.
(d) Two sucrose solutions of same molality prepared in different solvents will have the same freezing point
depression. [2004]
5. If a is the degree of dissociation of Na2SO4, the van’t Hoff factor used tor the calculation of molecular mass is
(a) 1 + 2a (b) 1 – 2a (c) 1 + a (d) 1 – a [2005]
6. Benzene and toluene form nearly ideal solutions. At 20 °C, the vapour pressure of benzene is 75 Torr and that of
toluene is 22 Torr. The partial vapour pressure of benzene at 20 °C for a solution containing 78 g of benzene and
46 g toluene in Torr is
(a) 37.5 (b) 53.5 (c) 50 (d) 25 [2005]
7. Equimolar solutions in the same solvent have
(a) same boiling point and same freezing point (b) different boiling point and different freezing point
(c) same boiling point but different freezing point (d) same freezing point but different boiling point
[2005]
8. Among the following mixtures, dipole-dipole as the major interaction, is present in
(a) benzene and carbon tetrachloride (b) benzene and ethanol
(c) acetonitrile and acetone (d) KCl and water [2006]
9. 18 g of glucose (C6H12O6) is added to 178.2 g of water. The vapour pressure of water for this aqueous solution at
100 °C is
(a) 752.40 Torr (b) 759.00 Torr (c) 7.60 Torr (d) 76.00 Torr [2006]
–1) in the same
10. A 5.25% solution of a substance is isotonic with a 1.5% solution of urea (molar mass = 60 g mol
solvent. If the densities of both the solutions are assumed to be equal to 1.0 g cm–3, molar mass of the substance
will be
(a) 90.0 g mol–1 (b) 115.0 g mol–1 (c) 105.0 g mol–1 (d) 210.0 g mol–1 [2007]
11. In a saturated solution of the sparingly soluble electrolyte AgIO3 (relative molecular mass = 283) the
equilibrium which sets in is

AgIO3 (s)  Ag + (aq ) + IO3- (aq )

If the solubility product constant Ksp of AgIO3 at a given temperature is 1.0 ¥ 10–8 M, what is the mass of AgIO3
contained in 100 mL of its saturated solution?
(a) 28.3 ¥ 10–2 g (b) 2.83 ¥ 10–3 g (c) 1.0 ¥ 10–7 g (d) 1.0 ¥ 10–4 g [2007]
12. A mixture of ethyl alcohol and propyl alcohol has a vapour pressure of 290 mmHg at 300 K. The vapour pressure
of propyl alcohol is 200 mmHg. If the mole fraction of ethyl alcohol is 0.6, its vapour pressure (in mmHg) at the
same temperature will be
(a) 350 (b) 300 (c) 700 (d) 360 [2007]
13. At 80 °C, the vapour pressure of pure liquid A is 520 mmHg and that of pure liquid B is 1000 mmHg. If a solution
of A and B boils at 80 °C and 1 atm pressure, the amount per cent of A in the mixture is
(a) 50 mol per cent (b) 52 mol per cent (c) 34 mol per cent (d) 48 mol per cent
[2008]
5.36 Complete Chemistry—JEE Main

14. The vapour pressure of water at 20 °C is 17.5 mmHg. If 18 g of glucose (C6H12O6) is added to 178.2 g of water
at 20 °C, the vapour of the resulting solution will be
(a) 17.325 mmHg (b) 17.675 mmHg (c) 15.750 mmHg (d) 16.500 mmHg
[2008]
15. Two liquids X and Y form an ideal solution. At 300 K, vapour pressure of the solution containing 1 mol of X
and 3 mol of Y is 550 mmHg. At the same temperature, if 1 mol of Y is further added to this solution, vapour
pressure of the solution increases by 10 mmHg. The vapour pressures (in mmHg) of X and Y in their pure states,
respectively, will be
(a) 400 and 600 (b) 500 and 600 (c) 200 and 300 (d) 300 and 400 [2009]
16. A binary liquid solution is prepared by mixing n-heptane and ethanol. Which one of the following statements is
correct regarding the behaviour of the solution?
(a) The solution is nonideal showing negative deviation from Raoult’s law
(b) n-Heptane shows positive deviation while enthanol shows negative deviation from Raoult’s law
(c) The solution formed is an ideal solution
(d) The solution is nonideal showing positive deviation from Raoult’s law. [2009]
17. If sodium sulphate is considered to be completely dissociated into cations and anions in aqueous solution,
the decrease in freezing point of water (DTf) when 0.01 mol of sodium sulphate is dissolved in 1 kg of water,
(Kf = 1.86 K kg mol–1) is
(a) 0.0186 K (b) 0.0372 K (c) 0.0558 K (d) 0.0744 K [2010]
18. On mixing, heptane and octane form an ideal solution. At 373 K, the vapour pressures of the two liquid components
(heptane and octane) are 105 kPa and 45 kPa, respectively. Vapour pressure of the solution obtained by mixing
25.0 g of heptane (molar mass = 100 g mol–1) and 35.0 g of octane (molar mass of octane = 114 g mol–1) will be
(a) 144.5 kPa (b) 72.0 kPa (c) 36.1 kPa (d) 96.2 kPa [2010]
19. Ethylene glycol is used as an antifreeze in a cold climate. Mass of ethylene glycol which should be added to 4 kg
of water to prevent it from freezing at –6 °C will be (Kf for water = 1.86 K kg mol–1 and molar mass of ethylene
glycol = 62 g mol–1)
(a) 304.60 g (b) 804.32 g (c) 204.30 g (d) 400.00 g
[2011 (Cancelled)]
20. The degree of dissociation (a) of a weak electrolyte AxBy is related to van’t Hoff factor (i) by the expression
x + y +1 i -1 i -1 x + y -1
(a) a = (b) a = (c) a = (d)
i -1 x + y -1 x - y +1 i -1
[2011 (Cancelled)]

21. A 5.2 molal aqueous solution of methyl alcohol, CH3OH is supplied. What is the mole fraction of methyl alcohol
in the solution?
(a) 0.050 (b) 1.100 (c) 0.190 (d) 0.086
[2011 (Cancelled)]
–1
22. A 5 % solution of cane sugar (molar mass: 342 g mol ) is isotonic with 1% of a solution of an unknown solute.
The molar mass of unknown solute in g/mol is
(a) 136.2 (b) 171.2 (c) 68.4 (d) 34.2 [2011]
23. The molality of a urea solution in which 0.0100 g of urea, [(NH2)2CO], is added to 0.3000 dm3 of water at STP is
(a) 0.555 mol kg–1 (b) 5.55 ¥ 10–4 mol kg–1 (c) 33.3 mol kg–1 (d) 3.33 ¥ 10–2 mol kg–1
[2011]
24. Kf for water is 1.86 K kg mol–1. If your automobile radiator holds 1.0 kg of water, how many grams of ethylene
glycol (C2H6O2) must you add to get the freezing point of hte solution lowered to – 2.8 °C?
(a) 72 g (b) 93 g (c) 39 g (d) 27 g [2012]
Solutions 5.37

25. Consider separate solutions of 0.500 M C2H5OH(aq), 0.100 M Mg3(PO4)2 (aq), 0.250 M KBr (aq) and 0.125 M
Na3PO4(aq) at 50 °C. Which statement is true about these solutions, assuming all salts to be strong electrolytes?
(a) 0.500 M C2H5OH(aq) has the highest osmotic pressure.
(b) They all have the same osmotic pressure
(c) 0.100 M Mg3(PO4)2 (aq) has the highest osmotic pressure
(d) 0.125 M Na3 PO4(aq) has the highest osmotic pressure [2014, online]
26. Dissolving 120 g of a compound of molar mass 60 g mol–1 in 1000 g water gave a solution of density 1.12 g mL–1.
The molarity of the solution is
(a) 1.00 M (b) 2.00 M (c) 2.50 M (d) 4.00 M [2014]
27. The observed osmotic pressure for a 0.10 M solution of Fe(NH4)2(SO4) at 25 °C is 10.8 atm. The expected and
experimental (observed) values of van’t Hoff factor (i) will be respectively (R = 0.082 L atm K–1 mol–1)
(a) 5 and 4.42 (b) 4 and 4.00
(c) 5 and 3.42 (d) 3 and 5.42 [2014, online]
28. For an ideal solution of two components A and B, which of the following is true?
(a) Dmix H < 0
(b) Dmix H > 0
(c) A· · ·B interactions is stronger than A· · ·A and B· · · B interactions.
(d) A· · ·A, A· · ·B and B· · ·B interactions are identical
29. A solution at 20 °C is composed of 1.5 mol of benzene and 3.5 mol of toluene. If the vapour pressures of pure
benzene and toluene are 74.7 Torr and 22.3 Torr, respectively, then the total vapour pressure of the solution and
benzene mole fraction in equilibrium with it will be respectively,
(a) 35.0 Torr and 0.480 (b) 38.0 Torr and 0.589
(c) 30.5 Torr and 0.389 (d) 35.8 Torr and 0.280 [2015, online]
30. Determination of molar mass of acetic acid in benzene using freezing point depression is affected by
(a) dissociation (b) association
(c) partial ionization (d) complex formation [2015, online]
31. The vapour pressure of acetone at 20 ºC is 185 Torr. When 1.2 g of a non-volatile substance was dissolved in 100
g of acetone at 20 ºC, its vapour pressure was 193 Torr. The molar mass of the substance is
(a) 32 g mol–1 (b) 64 g mol–1 (c) 128 g mol–1 (d) 488 g mol–1 [2015]
32. The solubility of N2 in water at 300 K and 500 Torr partial pressure is 0.01 g L . The solubility (in g L–1) at 750
–1

Torr partial pressure is


(a) 0.0075 (b) 0.005 (c) 0.02 (d) 0.015 [2016, online]
33. An aqueous solution of a salt MX2 at certain temperature has a van’t Hoff factor of 2. The degree of dissociation
for this solution of the salt is
(a) 0.50 (b) 0.33 (c) 0.67 (d) 0.80 [2016, online]
34. 18 g glucose (C6H12O6) is added to 178.2 g water. The vapour pressure of water (in Torr) for the aqueous solution
is
(a) 76.0 (b) 752.4 (c) 759.0 (d) 7.6 [2016]

ANSWERS
1. (a) 2. (b) 3. (b) 4. (d) 5. (a) 6. (c)
7. (a) 8. (c) 9. (a) 10. (d) 11. (b) 12. (a)
13. (a) 14. (a) 15. (a) 16. (d) 16. (c) 17. (c)
18. (b) 19. (b) 20. (b) 21. (b) 22. (c) 23. (b)
24. (b) 25. (b) 26. (b) 27. (a) 28. (d) 29. (b)
30. (all) 31. (b) 32. (d) 33. (a) 34. (b)
5.38 Complete Chemistry—JEE Main

HINTS AND SOLUTIONS


1. Larger the number of species in solution, larger the boiling point. 0.01 M Na2SO4 solution has 0.03 M of species
(= 0.02 M Na+ + 0.01 M SO42– ).
N 6.02 ¥ 1020
2. Amount of urea, n = = -1
= 10-3 mol
N A 6.02 ¥ 10 mol
23

n 10-3 mol
Concentration of urea solution, c = = = 10-2 mol L-1
V 0.1 L
3. The molecular interactions between benzene and methanol are weaker than those existing between benzene and
benzene, and methanol and methanol. Consequently, the solution will exhibit positive deviations from Raoult’s
law.
4. The freezing point depression is given by –DTf = Kf m. Different solvents have different values of Kf and hence
show different freezing point depression.
5. Na2SO4 ionizes as
Na2SO4 Æ 2Na+ + SO 2– 4
n(1 – a) n(2a) na

Total amount of species in solution will be n¢ = n(1– a) + 2na + na = n(1+ 2a)



The van’t Hoff factor is i = = 1 + 2a
n
m1 78 g
6. Amount of benzene, n1 = = = 1 mol
M1 78 g mol-1
m2 46 g
Amount of toluene, n2 = = = 0.5 mol
M 2 92 g mol-1
n1 1
Amount fraction of benzene, x1 = =
n1 + n2 1.5
Ê 1ˆ
Partial pressure of benzene, p1 = x1 p1* = Á ˜ (75 Torr ) = 50 Torr
Ë 1.5 ¯
7. The solutions will have same boiling point and freezing point provided the solutes dissolved are nonelectrolytes.
8. Both acetonitrile and acetone are polar and have permanent dipole moment.
18 g 178.2 g
9. Amount of glucose = -1
= 0.1 mol; Amount of water = = 9.9 mol
180 g mol 18 g mol-1
9.9 g
Mole fraction of water = = 0.99
0.1 + 9.9
Vapour pressure of water in solution at 100 °C is p = xp* = (0.99) (760 Torr) = 752.40 Torr
10. Isotonic solutions have the same osmotic pressure. Hence, from P = cRT, we get
c1 = c2 fi n1 = n2
5.25 g 1.5 g 5.25 ¥ 60
i.e. = -1
fi M= g mol-1 i.e. M = 210 g mol–1
M 60 g mol 1.5
11. For AgIO3 (s)  Ag + (aq ) + IO3- (aq ), we have
Ksp = [Ag+] [IO–3 ] = s2 Hence s = Ksp = 1.0 ¥ 10 -8 M 2 = 1.0 ¥ 10 -4 M
Mass of AgIO3 in 100 mL solution is
m = (1.0 ¥ 10–4 mol L–1) (283 g mol–1) (0.1 L) = 2.83 ¥ 103 g
12. Using the expression p = xA pA + xB pB; (A is ethyl alcohol and B is propyl alcohol)
Solutions 5.39

we get 290 mmHg = 0.6 pA + 0.4 ¥ 200 mmHg


(290 - 0.4 ¥ 200) mmHg
This give pA = = 350 mmHg
0.6
13. Let xA be the amount fraction of A in the liquid solution. We will have
ptotal = xA p*A + xB p*B = xA p*A + (1 - xA ) p*B = xA ( p*A - p*B ) + p*B
ptotal - p*B 760 mmHg - 1000 mmHg 240 1
This gives xA = = = =
p*A - p*B 520 mmHg - 1000 mmHg 4880 2
nA 1 mol
Hence, the amount per cent of A is ¥ 100 = ¥ 100 = 50
nA + nB 2 mol
m2 18 g
14. Amount of glucose, n2 = = = 0.1 mol
M 2 180 g mol -1
m1 178.2
Amount of water, n1 = = = 9.9 mol
M1 18 g mol-1
n1 9.9 g
Mole fraction of water, x1 = = = 0.99
n1 + n2 (9.9 + 0.1) mol
Vapour pressure of solution, p = x1 p *1 = (0.99)(17.5 mmHg) = 17.325 mmHg.

15. (a) For an ideal solution p = xX p*X + xY p*Y

Ê 1 ˆ p* + Ê 3 ˆ p* fi p* + 3 p* = 2200 mmHg
550 mmHg = ÁË ˜ X ÁË ˜ Y
1 + 3¯ 1 + 3¯
X Y

Ê 1 ˆ p* + Ê 4 ˆ p* fi p* + 4 p* = 2800 mmHg
560 mmHg = ÁË ˜ X ÁË ˜ Y
1 + 4¯ 1 + 4¯
X Y

Solving for p X* and p*Y, we get p*X = 400 mmHg and p*Y = 600 mmHg
16. The interaction between unlike molecules is weaker than those involved between like molecules. This results into
positive deviations from Raoult’s law
17. Na 2SO 4 (aq) Æ 2 Na + (aq) + SO24- (aq); van't Hoff factor = 3
Hence, – DTf = i Kf m = (3) (1.86 K kg mol–1) (0.01 mol kg–1) = 0.0558 K
m1 25.0 g
18. Amount of heptane, n1 = = = 0.25 mol
M1 100 g mol-1
m2 35.0 g
Amount of octane, n2 = = = 0.307 mol
M 2 114 g mol-1
n1 0.25
Mole fraction of heptane, x1 = = = 0.45
n1 + n2 0.25 + 0.307
Mole fraction of octane, x2 = 1 – xx = 0.55
According to Raoult’s law
p = x1 p1* + x2 p*2 = (0.45) (105 kPa) + (0.55) (45 kPa) = 72.0 kPa
m2 / M 2
19. Since - DTf = K f m = K f , we have
m1

(- DTf )m1M 2 (6 K )(4 kg)(62 g mol-1 )


m2 = = = 800 g
Kf (1.86 K kg mol-1 )
5.40 Complete Chemistry—JEE Main

20. We have
A x B y  xA y + + yB x -
m(1 – a) m(xa) m(ya)

Total molality of the solution = m(1 – a) + m(xa) + m(ya) = m{(x + y – l) a + 1}


DTb K m{( x + y - 1)a + 1}
The van’t Hoff factor is i= = b
( DTb )0 Kb m
i +1
or i = (x + y – l)a + 1 i.e. a=
x + y -1
21. In 5.2 molal aqueous solution, we have
n2 = 5.2 mol and n1 = (1000 g/18 g mol–1) = 55.56 mol
The mole fraction of methyl alcohol in the solution is
n2 5.2 mol 5.2
x2 = = = = 0.086
n1 + n2 (55.56 + 5.2) mol 60.76
22. Isotonic solutions have identical osmotic pressure and hence identical concentrations, i.e. the same amounts in the

5g 1g 342 ˆ
= M = ÊÁ g mol -1 = 68.4 g mol -1
Ë 5 ˜¯
-1
; This gives
342 g mol M
23. Assuming density of water equal to 1 kg dm–3, we have
n2 m2 / M 2 (0.0100 g/60 g mol-1 )
m= = = = 5.55 ¥ 10-4 mol kg -1
m1 m1 (0.3000 kg )
24. Since – DTf = Kf m, we get.
- DTf 2.8 K
m= = = 1.505 mol
Kf 1.86 K kg mol-1
Since the molality, m = n/m1 , we get
n = mm1 = (1.505 mol kg–1) (1 kg) = 1.505 mol
Finally, the mass of ethylene glycol, C2H6O2 (the molar mass = 62 g mol–1) required will be
m = n Mm = (1.505 mol)(62 g mol–1) = 93.3 g
25. All the four solutions have the same amount of the species present in the solution.
C2H5OH 0.500 M
0.100 M Mg3(PO4)2 0.300 M Mg2+ + 0.200 PO3– 4
0.250 M KBr 0.250 M K+ + 0.250 M Br–
0.125 M Na3PO4 0.375 M Na+ + 0.125 M PO3– 4
Since all of them have the concentration of total species, their osmotic pressure will have the same value.
26. Total mass of the solution = 120g + 1000g = 1120g
m 1120 g
Volume of the solution, V= = = 1000 mL = 1.0 L
r 1.12 g mL-1

m 120 g
Amount of compound, n= = = 2.0 mol
M 60 g mol -1

n 2.0 mol
Molarity of solution, M= = = 2.0 mol L-1
V 1L
Solutions 5.41

27. Fe(NH4)2(SO4)2 ionizes as


Fe(NH4)2 (SO4)2 Æ Fe2+ + 2NH +4 + 2SO2– 4
The number of species produced is 5 times than that of Fe(NH4)2 (SO4)2. Therefore, the expected van’t Hoff factor
is 5.
The expected osmotic pressure assuming no ionization is
p = cRT = (0.10 mol L–1) (0.082 L atm K–1 mol–1) (298.15 K) = 2.45 atm
The observed van’t Hoff factor is
10.8 atm
i= = 4.4
2.45 atm
28. For an ideal solution, interactions among A· · ·A, B· · ·B and A· · ·B are identical.
nb 1.5
29. Mole fraction of benzene, xb = = = 0.3
nb + nt 1.5 + 3.5
Mole fraction of toluene, xt = 1 – xb = 0.7
The vapour pressure of the solution is
p = xb p*b + xt p*t = (0.3 ¥ 74.7 + 0.7 ¥ 22.3) Torr
= 38.02 Torr
Mole fraction of benzene in vapour phase is
pb (0.3 ¥ 4.7) Torr
yb = = = 0.589
p (38.02 Torr )
30. All are correct.
31. According to Raoult’s law,
p1 183 Torr 2
p1= x1 p*1 = (1 – x2)p*1 i.e. x2 = 1 – *
= 1− =
p1 185 Torr 185
n2 (1.2 g/M )
Also, x2 = =
n1 + n2 (100 g / 58 g mol−1 ) + (1.2 g/M )
(1.2 g/M ) 2
Hence, −1
=
(100 g / 58 g mol ) + (1.2 g/M ) 185
This gives
1.2 g ⎛ 2 ⎞ ⎛ 2 ⎞ ⎛ 100 g ⎞ ⎛ 183 ⎞ ⎛ 185 ⎞ ⎛ 58 g mol ⎞
−1
⎜1 − ⎟=⎜ ⎟⎜ ⎟ or M = (1.2 g) ⎜ ⎟⎜ ⎟ ⎜ ⎟ = 63.68 g mol–1
M ⎝ 185 ⎠ ⎝ 185 ⎠ ⎝ 58 g mol−1 ⎠ ⎝ 185 ⎠ ⎝ 2 ⎠ ⎝ 100 g ⎠
2 1.2 g/M
Approixmate value Writing x2 n2/n1,we get = .
185 (100 g / 58 g mol−1 )
⎛ 58 185 ⎞ −1
This gives M = ⎜1.2 × × ⎟ g mol = 64.38 g mol–1
⎝ 100 2 ⎠
32. According to Henry’s law, s = kH p. Hence
s2 p Êp ˆ Ê 750 Torr ˆ
= 2 i.e. s2 = Á 2 ˜ s1 = Á ˜ (0.01 g L-1 ) = 0.015 g L–1
s1 p1 Ë p1 ¯ Ë 500 Torr ¯
33. We have
MX2 

 M + 2X
2+ –
m(1 – a) ma 2ma

Total molality in the solution = m(1 + 2a)


m(1 + 2a )
Hence =2
m
This gives a = 0.50
5.42 Complete Chemistry—JEE Main

m2 18 g
35. Amount of glucose, n2 = = = 0.1 mol
M 2 180 g mol-1
m1 178.2 g
Amount of water, n1 = = = 9.9 mol
M1 18 g mol-1
n2 0.1
Amount fraction of glucose, x2 = = = 0.01
n1 + n2 0.1 + 9.9
The relative lowering of vapour pressure of water is
Dp
- *1 = x2
p1
p1 = x2 p*1 = (0.01) (760 Torr) = 7.6 Torr
p1 = p*1 p1 = (760 – 7.6) Torr = 752.4 Torr
Note: In the above calculations, temperature of aqueous solution is assumed to be 100 °C.
6
Chemical Thermodynamics

Section 1 Basic Definitions and First Law of Thermodynamics

The subject of thermodynamics deals basically with the intersection of one body with another in terms of the quantities
of heat and work.

1.

Neither energy nor matter can be transferred to or from it.


2.
the system.
3.
The boundary may be imaginary or real; it may be rigid or nonrigid; it may be a conductor of heat (diathermic

4.

distinguished based on the size of system.

5. A process is the path along which a change of state of a system takes place. The following ways of
carrying a process may be considered.

state.
6.2 Complete Chemistry—JEE Main

6. If the change in property of a system does not depend upon the path adopted in going form
one state to another, the property is said to be state function.
Energy of a system can be changed in

The internal energy (symbol: U q


the form of work (symbol: w
dU = dq + dw

DU = q + w
DU, whereas heat and work are simple written as q
and w. This follows from the fact that U is a state function whereas q and w are path dependant.
For an isolated system, both q and w
DU = 0, i.e. U

constant.

system against an external pressure is known as mechanical work.


Consider a gaseous system as shown in Fig. 1 If the piston is pushed
through a distance dl
Work = Force ¥ Distance
= (Pressure ¥
ie. dw = (pext A l pext (Adl pextdV
where dV
recommendations, the expression of w is
w = –pextdV
For expansion of the gas, dV w
work is done by the system. For compression of the gas, dV
and dw

dU = dq + dw

dU = dq – pextdV

dU = dqV or DU = qV
that is, q is not state function
but qV is a state function.
For pext = p
dU = dq – pdV
Adding and subtracting Vdp on the right side, we get
Chemical Thermodynamics 6.3

dU = dq – pdV + (Vdp – Vdp


= dq – (pdV + Vdp + Vdp = dq – d(pV + Vdp
or dU + d(pV = dq + Vdp or d(U + pV = dq + Vdp or dH = dq + Vdp
where H = U + pV and is known as enthalpy function.
If heat is added at constant p
dH = dqp or DH = qp
q is not a state function but
qp is a state function.

T is the increase in temperature by adding


heat dq, then the heat capacity is
dq
C=
dT
For gaseous system, two types of heat capacities may be considered. These are
CV = (∂q/∂T V
Heat capacity at constant pressure: Cp = (∂q/∂T p

dU = dqV = CV dT
dH = dqp = Cp dT
DH DU
H = U + pV Hence, DH = DU + D(pV
D(pV  0
Hence, DH  DU
For an ideal gas D(pV  D(nRT nR(DT
Hence, DH  DU + nR DT
Cp CV
H = U + pV
Hence dH = dU + d(pV H = dU + nR dT
Cp dT = CV dT + nRdT or Cp – CV = nR
For one mole of an ideal gas, Cp,m – CV,m = R

compression of a system is
dw = –pext dV

Let an ideal gas undergo expansion from V1 to V2 (or compression from V2 to V1

For expansion, the external pressure must


satisfy the expression. pext £ p2. Let pext = p2
w = –pext (DV p2(V2 – V1 †

V 2 > V 1, w
For compression, the external pressure must satisfy the expression pext ≥ p1. Let pext = p1
w = –pext DV = –p1(V1 – V2 †

V1<V2, w
wexp < wcomp

† If pext < p2 in expansion or pext > p1


6.4 Complete Chemistry—JEE Main

or compression is carried out in more than one step, the pext has to be adjusted in each step. Let pext
pressure in each step.

compression. This trend is continued as the number of steps is increased.

For expansion pext = p – dp For compression pext = p + dp


The expression of work is
– dp V  – p dV
dw = – pext dV = – (p +
(The product dpdV
For an ideal gas, p = nRT/V.
dV
Hence dw = –nRT
V

Vf
dV Vf
w = - nRT Ú V
= - nRT ln
Vi
Vi
For expansion Vf (=V2 Vi(=V1 w
system
For compression, Vf (=V1 Vi (=V2 w
Chemical Thermodynamics

under the isotherm.

compression.
For adiabatic expansion or compressure, heat is neither enters
q
dU = dw or nCV dT = –pextdV
or, w = –pext (V2 – V1 nCV(T2 – T1
For expansion V2 > V1, w < 0, hence T1 > T2
For compression, V2 < V1, w > 0 and hence, T2 > T1

from the expression


w = DU = nCV, m (T2 – T1
T2
pext = p ± dp.
Thus dU = dw or nCV, mdT = –pext dV
or nCV, mdT = – (p ± dp V  –pdV
Replacing p = nRT/V, we get
nRT ˆ
nCV, mdT = - ÊÁ
dT dV
Ë V ˜¯
dV or CV, m = –R
T V

T2 2 V
dT dV T2 V
CV , m Ú = - R Ú or CV , m ln = - R ln 2
T1
T V
V T1 V1
1

CV , m/ R CV , m / R
ÊT ˆ ÊV ˆ Ê T2 ˆ V1
or ln Á 2 ˜ = ln Á 1 ˜ or ÁË T ˜¯ =
Ë T1 ¯ Ë V2 ¯ 1 V2

or T2CV, m / RV2 = T1CV, m / RV1 or T


CV, m / R
V = constant

CV , m CV ,m 1 1
= C -C = =
R p ,m V,m (C / C
p, m V , m) - 1 g -1
T1/(g V = constant or TV g – 1
g -1
Ê RT ˆ
TÁ T g p 1– g = constant T g p 1– g
Ë p ˜¯
= constant or or

Ê pV ˆ (V g -1 ) = constant pV g
The relation between p and V is ÁË ˜ or
R ¯

pext
6.6 Complete Chemistry—JEE Main

dw = –pextdV = 0 or w
For such an expansion, DV = 0, DH = 0 and Dq = 0

3
Ek = nRT
2
Ê ∂Ek ˆ 3 3
Hence, CV = ÁË ˜¯ = nR and CV , m = R
∂T V 2 2

3 3 5 C p, m 5
CV, m = R; C p, m = R + R = R; g = = = 1.667
2 2 2 CV , m 3

motions.

RT towards molar energy of the molecule.


N–5 and 3N N is the
RT towards molar energy of the molecule.
The following is the summary of molar energy contributions by each motion towards the total molar energy of the
molecules. It also includes, their heat capacities and the ratio of heat capacities.

Molecules Translational Rotational Vibrational Total CV, m Cp,m g =Cp, m/CV, m


Diatomic 3 1 7 7 9 9
RT 2 ¥ RT RT RT R R = 1.286
2 2 2 2 7 7
Triatomic
linear 3 1 13 13 15 15
RT 2 ¥ RT 4RT RT R R = 1.154
2 2 2 2 2 13
nonlinear 3 1 7
RT 3 ¥ RT 3RT 6RT 6R 7R = 1.167
2 2 6

CV, m.

MULTIPLE CHOICE QUESTIONS ON SECTION 1


Chemical Thermodynamics 6.7

5. The work done on a gaseous system is 12 J and also 20 J of heat is added to it. The change in internal energy of
the gaseous system is

6. The heat capacity of 12.0 g of helium is about


–1 –1 –1 –1

7. 10 J of heat is supplied to 36.0 g water at 373.15 K and 1 atm pressure. The heat capacity of water system will be
–1 –1 –1
3 to 20 dm3 against an external pressure

9. The difference between Cp and CV of 0.2 mole of an ideal gas is about


–1 –1 –1 –1
3 to 5 dm3 at 300 K. The work

11. A gas undergoes an isothermal expansion against a constant external pressure in stepwise manner such that the

be largest when the expansion is carried out in

12. A gas undergoes an isothermal compression against a constant external pressure in stepwise manner such that the

compression is carried out in

13. 28.0 g of nitrogen undergoes expansion from 1 atm, 10 dm3 to 0.5 atm, 20 dm3 against a constant external pressure

V1 to V2 against a constant external pressure. It is

DU = 0 and w DU = 0 and w DU = 0 and w DU > 0 and w > 0


V1 to V2 against a constant external pressure. If T1
to T2
T2 = T1 T2 > T1 T2 < T1 V1/T1 = V2/T2
16. If T2 and T2¢
V1 to V2, which of the following facts holds good:
g g
T2 > T2¢ T2 < T2¢ T2 = T2¢ V1 T1 = V1 T2
where g is the ratio Cp, m/CV, m of the gas.
p 1, V 1, T 1 p 2, V 2, T 2

g g
p 1V 1 = p 2V 2 p1 V1 = p2 V2 p1/T1 = p2/T2 V1/T1 = V2/T2
where g is the ratio Cp,m/Cv,m of the gas.
V1 to V2. If 298 K and 278

R R R R
V1 to V2 V2 to V1,
then in this process
DU = 0 and w DU = 0 and w DU = 0 and w DU > 0 and w > 0
6.8 Complete Chemistry—JEE Main

R R R R
21. The ratio g = Cp, m/CV, m for a diatomic gaseous molecules is

g = Cp, m/CV, m for diatomic gaseous


molecules will be

23. The molecule of a gas has three independent translational motions along with each of the three Cartesian axis. The
contribution of each translation motion of a molecule towards total energy is
kT kT kT kT

25. Which of the following statements is

qp and qV are state functions

ANSWERS

HINTS AND SOLUTIONS


1. In an isolated system, neither heat nor matter can transferred to or from the system.

5. DU = q + w = 20 J + 12 J = 32 J
m 12 g
6. Amount of helium, n = = = 3 mol . Helium is monatomic gas. Its CV, m R
M 4 g mol-1

R = ÊÁ molˆ˜ (8.314 J K -1 mol -1 ) = 37.41 J K -1


9
Heat capacity, CV = nCV, m
Ë2 ¯
7. At 1 atm pressure, 373.15 K is the normal boiling point of water. Heat supplied at this temperature is utilized in
q q
C= = =•
Dt 0
8. w = –pext (V2 – V1 ¥ 3 – 10 dm3
–1 mol–1 –1
9. Cp – CV = nR
V2 Ê 5 dm3 ˆ
–1
10. w = –nRT ln mol–1 ÁË 20 dm3 ˜¯ = 6916.6 J = 6.92 kJ
V1
Chemical Thermodynamics 6.9

11. Larger the number of steps, larger the magnitude of work done by the gas.
12. Lesser the number of steps, larger the work done on the gas.
13. For expression, pext £ p . The magnitude of work will be maximum when pext = p .
14. DU
compared to the magnitude of work done by the system during expansion, w = wexp + wcomp > 0

T 2 < T 1.
T2 < T2¢.
g
pV = constant holds good.
Ê 3ˆ R
18. w = nCV, m (T2 – T1 ÁË ˜¯ R
2
19. DU = 0 as U a state function. w

20. For a monatomic gas, CV, m R.


21. For a diatomic gas, CV, m R and Cp, m R. Thus g =Cp, m/CV, m = 9/7 =1.286
22. In this case, CV, m R and Cp, m R. . Thus g =Cp, m/CV, m = 7/5 =1.4
kT.
V1
24. w = wexp + wcomp = –pext (V2 –V1 nRT ln
V2
3 –10 dm3 –1 mol–1

= –101.325 J + 1729. 0 J = 715.74 J

Section 2 Thermochemistry

chemical reaction, and the surroundings.


x, pronounced as

amount of a reactant consumed or a product produced


x=
stoichioometric number of the reactant or product
The unit of x is mol.
Let the decomposition of N2 5 is studied with the initial
amount of N2 5 equal to n0.
To start with, extent of reaction x = 0. The enthalpy of the system to start with is
Hinitial = n0 Hm( N2 5
When the reaction has proceeded to the extent x, then
2 N 2 O5 (g)  4 N 2 O5 (g) + O 2 (g)
n0 - 2x 4x x
H = (n0 – 2x Hm( N2 5 xHm( 2 xHm( 2
The enthalpy change is
DH = H – Hinitial = x[–2Hm( N2 5 Hm 2 Hm 2
Complete Chemistry—JEE Main

DH
D rH = = –2Hm( N2 Hm Hm
x 5 2 2

Unit of DH J kJ
D rH = = or
Unit of x mol mol
In general, enthalpy of reaction may be expressed as

D rH = Â B n B H m (B) - Â B n B H m (B)

For exothermic reactions, heat is released and thus DrH


requires that the enthalpy of reactants is larger than that of products.
For endothermic reactions, heat is absorbed and DrH
than that of reactants.

follows that

For example
2 may be formed in two ways
2 æÆ 2 D r H = –393.5 kJ mol–1
1
2 2 æÆ Dr H = –110.3 kJ mol–1
1
2 2 æÆ 2 Dr H = –283.2 kJ mol–1
Add
2 æÆ 2 D r H = –393.5 kJ mol–1
The chemical equations may be treated as algebraic expressions which can be added
or subtracted to yield the required chemical equation. The corresponding enthalpy changes are also manipulated in the

required to completely separate one mole of solid ionic compound into gaseous ions. For example
æÆ Na+ –

æÆ Dr H1
æÆ Na+ – ; Dr H2
1 1
Dissociation of 2 Cl2 2 Cl2 æÆ Dr H3

Formation of Cl– – æÆ Cl– Dr H4


Condensation of Na+ – + – æÆ Dr H5
Add
1
2 Cl2 æÆ Dr H6
According to Hess’s Law DrH6 = DrH1 + DrH2 + DrH3 + DrH4 + DrH5
Except DrH5, all other DrHs can be determined experimentally.
Hence –DrH5 = DrH1 + DrH2 + DrH3 + DrH4 – DrH6
The term –DrH5 represents lattice energy.
Chemical Thermodynamics

enthalpy of reaction is known as standard enthalpy of reaction, if the external pressure on the chemical system is 1 bar
(1 atm = 1.01325 bar = 1.03125 ¥ 105

DrH° = ÂPnP H°m, P – ÂRnR H°m, R

H°m = U°m + pVm, we get DrH° = ÂPnP(U°m + pVm P – ÂRnR(U°m + pVm R


= (ÂPnPU°m, P – ÂRnRU°m, R + (ÂPnP(pVm P – ÂRnR(pVm R
= DrU° + ÂPnP(pVm P – ÂRnR(pVm R

pVm  0 for one mole of solid or liquid


pVm  RT for one mole of gaseous species
DrH°  DrU°

DrH° = DrU° + [ ÂPnP(RT – ÂRnR(RT = DrU° + (RT Â Pn P – Â Rn R


= DrU° + (Dng RT
where Dng is the change in stoichiometric number of gaseous species in going from reactants to products.

2 2 Æ 2NH3 DrH° = –92.38 kJ mol–1. Calculate the energy of reaction at


298 K.
Dng
DrU = DrH – (Dng RT = –92.38 kJ mol–1 –1
mol–1
= –92.38 kJ mol–1 + 4955 J mol–1 = –92.38 kJ mol–1 + 4.96 kJ mol–1
= –87.42 kJ mol–1

formation of substance can be built.

For examples,
D fH 2 D fH DfH°(Br2 D fH D fH
The standard enthalpy of formation (symbol: Df H

the requisite amounts of elements in their stable state of aggregation.


The formation of one mole of a substance implies that the compound appears as product with stoichiometric number
equal to one.
For examples
1
H (g)
2 2
+ 12 Br2 (l) æÆ HBr (g) Df H° = –36.40 kJ mol–1

C(graphite) + 2H 2 (g) æÆ CH 4 (g) Df H° = –74.80 kJ mol–1


3
2 2 æÆ 3 Df H° = –393.72 kJ mol–1

The standard enthalpy of a reaction may be computed by using the


Complete Chemistry—JEE Main

DrH° = Â B n B D f H ∞(B) - Â B n B D f H ∞(B)

Æ 2
DrH° = DfH D fH 2 D fH D fH
–1

= 65.3 kJ mol–1

D cH

CH4 2æÆ 2 2
DcH° = DfH 2 D f H°(H2 DfH°(CH4 D fH 2
–1

= –694.46 kJ mol–1

Measurement of Enthalpy of Combustion


a substance. A known mass of the substance in a closed steel

The calorimeter is surrounded by a known mass of water. The


sample is ignited electricity to bring about the combustion

qcomb = –(mwater cwater + mbomb c bomb DT


where c

of substance is equal to energy of combustion.


qcomb qcomb
D cU = =
nsubs msubs / M subs
The enthalpy of combustion will be
DcH = DcU + (Dng RT

of neutralization is the enthalpy change when one mole of H+


H+ – æÆ H2 DneutH° = –55.84 kJ mol–1

acid or base is made to ionize completely in the solution.


2 æÆ H3 + – DionzH° = 43.71 kJ mol–1
– æÆ H + + CN–
3
DrH° = DneutH° + DionzH –1 =12.13 kJ mol–1
Chemical Thermodynamics

2 æÆ HCl(10H2 DsolH° = –69.49 kJ mol–1


2 æÆ HCl(200H2 DsolH° = –74.20 kJ mol–1

æÆ DsolH° = –75.15 kJ mol–1


The integral enthalpy of dilution is the change in enthalpy when a solution containing
1 mole of solute is diluted from one concentration to another.
2 æÆ HCl(200H2 DdilH
HCl(10H2 –1 = –4.71 mol–1

of a solution of known concentration so that there occurs no appreciable change in concentration of the solution.

of a solution of known concentration so that there occurs no appreciable change in concentration of the solution.

It is the enthalpy change when one mole of one allotropic form changes to another.
æÆ DtrsH° = 1.90 kJ mol–1
It is the enthalpy change (symbo: DatH
atoms in the ground state in the gaseous phase.
For example H2 æÆ DatH° = 625 kJ mol–1
The bond enthalpy (symbol: e
dissociate the said bond present in different gaseous molecules into free atoms in the gaseous state.
For example
H2 æÆ DrH°1 = 501.87 kJ mol–1
æÆ DrH°2 = 423.38 kJ mol–1
1
Hence e 2 (DrH°1 + DrH°2 = 12 kJ mol–1 = 462.62 kJ mol–1
DrH°1 and DrH°2
of all bond dissociation enthalpies.
The data on the bond enthapies may be employed to estimate the approximate

formation of a compound.

Enthalpy of formation of ethane from the following data:


e = 348 kJ mol–1, e = 412 kJ mol–1 e = 436 kJ mol–1 and DsubH = 716 kJ mol–1
2 æÆ C2H6 D rH
We may proceed as follows:
DfH°
2 ææÆ 2H 6
Make bond
Æææ
ææÆ

ææÆ

Break bond –eC–C – 6eC–H


carbon DsubH° 3eH – H
H H

¨æÆ H C· · · C H

H H
According to Hess’s law
DfH° = 2DsubH eH–H – 3eC–C – 6eC–H –1

= – 80 kJ mol–1
Complete Chemistry—JEE Main

MULTIPLE CHOICE QUESTIONS ON SECTION 2

DcH°(H2 –1
; D cH –1
; DcH°(CH4 –1

DfH°(CH4 will be
kJ mol–1 kJ mol–1 kJ mol–1 kJ mol–1

DcH°(C2H4 –1; DcH°(C2H6 –1; DcH°(H2 –1

DrH° of the reaction C 2H 4 2 æÆ C2H6


kJ mol–1 kJ mol–1 kJ mol–1 kJ mol–1

DfH(Mg3N2 –1
DfH°(NH3 –1

D rH H3 æÆ Mg3N2 2
kJ mol–1 kJ mol –1 kJ mol–1 kJ mol–1

Df H°(H2 –1; Df H 3
–1; Df H°(N2 5
–1

The Dr H° for the reaction N2 5 H 2O æÆ 3


kJ mol–1 mol–1 kJ mol–1 kJ mol–1

–1

heat capacity and density of both the solution are 4.18 J K–1 g–1 and 1 g mL–1

Df H 2
–1
and Df H 3
–1

1
DrU° 2 2 2 æÆ 3

kJ mol–1 mol–1 kJ mol–1 kJ mol–1


7. For the reaction. 2A2 5B2 æÆ 2A2B5
DrH° > DrU° DrH° < DrU° DrH° = DrU° DrH° = –3DrU°
–1
8. A 0.1215 g sample of solid magnesium (molar mass: 24.30 g mol
= 1.7448 kJ °C–1
combustion of magnesium at 298 K is about.
kJ mol–1 kJ mol–1 kJ mol–1 kJ mol–1
e = 374 kJ mol ; eC = C = 632 kJ mol ; e
–1 –1
= 411 kJ mol–1
If the enthalpy formation of benzene from gaseous atoms is 5536 kJ mol–1, the resonance energy benzene is
kJ mol–1 kJ mol–1 kJ mol–1 kJ mol–1
e = 388 kJ mol ; e
–1 = 436 kJ mol ; DfH (NH3
–1 –1

The bond enthalpy of N∫∫N is about


kJ mol–1 kJ mol–1 kJ mol–1 kJ mol–1
H2 4 5H2O æÆ H2 4◊5H2 DrH° = –58.0 kJ mol–1
–1 g–1 is obtained when 1 mol of H
2 4 is added to 5 mol of H2
Chemical Thermodynamics

12. If DcH° –1
, DfH° 2
–1
and Df H°(H2 –1

of DfH°
kJ mol–1 kJ mol–1 kJ mol–1 kJ mol–1
–1
4 and –51.3 kJ mol–1
4
kJ mol–1 kJ mol–1 kJ mol–1 kJ mol–1
14. If Df H°(C2H6 = –84 kJ mol , DsubH°
–1 –1, D (H = 435 kJ mol–1 and the bond enthalpy
at 2
e = 414 kJ mol–1, then the bond enthalpy e will be about
kJ mol–1 kJ mol–1 kJ mol–1 kJ mol–1
DfH°m
+ – – –
3

ANSWERS

HINTS AND SOLUTIONS


1
2 2 æÆ H2 2 DcH° = –286.1 kJ mol–1
2 æÆ 2 DcH° = –394.9 kJ mol–1
4 2 æÆ 2 2 DcH° = –882.0 kJ mol–1
The required chemical equation for which Df H 2 æÆ CH4

Hence, DfH°(CH4 –1
= –85.1 kJ mol–1
2H 4 2 æÆ 2 2 DcH° = –1410.9 kJ mol–1
7
2H 6 2 2 æÆ 2 2 DcH° = –1559.8 kJ mol–1

2
1
2 2 æÆ H2 DcH° = –285.8 kJ mol–1

The required chemical equation for which DrH° is required is C2H 4 2 æÆ C2H6

Hence, D rH –1 = –136.9 kJ mol–1


2 æÆMg3N2 ; DfH° = –463 kJ mol–1
1
2 N2 3
2 H2 æÆ NH3 DfH° = –46 kJ mol–1

3 æÆ Mg3N2 2
Hence, DrH° = (– 463 + 2 ¥ –1 =
–371 kJ mol–1
1
2 2 2 æÆ H2 DfH° = –286 kJ mol–1
1 1
2 N2 3
2 2 2 H2 æÆ 3 DfH° = –174 kJ mol–1

2
5
2 2 æÆ N2 5 DfH° = –14.5 kJ mol–1

The required reaction N2 5 2 æÆ 3


Hence, D rH –1 = –76.5 kJ mol–1
Complete Chemistry—JEE Main

–1
n1 = VM
–1
Amount of HCl, n2 = VM
+ –
, heat liberated in this process is
–1
q = nDneutH
q 7320 J
Increase in temperature of the solution will be DT = = = 5.0 ∞C
mc (350 g)(4.18 J g -1 ∞C -1 )
Hence, temperature of solution is 25.0 °C + 5.0 °C = 30.0 °C

2 æÆ 2 ; DfH° = –296.8 kJ mol–1


3
2 2 æÆ 3 DfH° = –395.7 kJ mol–1

2
1
2 2 æÆ 3

Hence, D rH –1 = –98.9 kJ mol–1


Dng = –1/2. Hence
DrU° = DrH° – (Dng RT = –98.9 kJ mol–1 ¥ 10–3 kJ K–1mol–1
–1 = –97.66 kJ mol–1

Dng = –3. From the expression DrH° = DrU° + (Dng RT, it follows that DrH° < DrU°

qwater = mwatercwaterDT –1
°C–1
qbomb = (mbombcbomb DT = CbombDT = (1744.8 J °C–1
qtoal = qwater + qbomb
q 3000 J
DcU = total = -1
= 600 ¥ 103 J mol–1
m / M (0.1215 g / 24.30 g mol )
9. Enthalpy of formation of benzene from the gaseous atoms using bond enthalpies data is
DH = –3e –3eC == C – 6e –1 = –5484 kJ mol–1

Resonance energy = DH DH –1
= –52 kJ mol–1
10. 1
N2 3
H2 D fH
2 2 ææÆ 3
Æææ
ææÆ

ææÆ

1
2 eN ∫∫ N 3
e –3e
2

H
¨æÆ H N
H

According to Hess’s law Df H = 1


2
eN ∫∫ N + 3
2
e – 3e

or eN ∫∫ N = 2DfH – 3e + 6e = (–2 ¥ 46.11 – 3 ¥ 436 + 6 ¥ –1 = 927.8 kJ mol–1


mcp DT = – D f H°
–1 –1
where m = m H2 4
+ mH2 = (nM H2 4
+ (nM water

-Df H 58 ¥ 103 J
where DT = = = 73.74 ∞C
mc p (188 g)(4.184 J º C-1 g -1 )
15
12. C6H6 2 2 æÆ 2 2

DcH°(C6H6 = 6 Df H 2, Df H°(H2 Df H(C6H6


Hence Df H°(C6H6 = (–3260 + 6 ¥ 393 + 3 ¥ –1
= – 47 kJ mol–1
Chemical Thermodynamics

13. H+ –
æÆ H2 DrH°1 = –57.3 kJ mol–1
NH4 + H+ æÆ NH4+ 2 DrH2° = –51.3 kJ mol–1
NH4 æÆ NH4+ – DrH° =
DrH° = DrH2° – DrH°1 = 6.0 kJ mol–1
DfH°
3H2 ææÆ 2H 6
14.

Æææ
ææÆ
ææÆ

–3eC–C –6eC–H
2DsubH 3DatH°(H2
H H

¨æÆ H C C H

H H
According to Hess’s law
DfH° = 2DsubH DatH°(H2 eC–C – 6eC–H
eC–C = 2DsubH DatH°(H2 eC–H – DfH° = [ 2 ¥ 720 + 3 ¥ 435 – 6 ¥ –1

= 345 kJ mol–1
Df H°(H+ =0

Section 3 Criteria of Spontaneity

Many such examples can be cited. All these processes are said to be spontaneous. In these processes, the spontaneous

First law of thermodynamics holds good for both spontaneous and nonspontaneous processes.

1
2 2 æÆ D f H° = 6.3 kJ mol–1

Thus DrH°
criterion of spontaneity is obtained from the phenomenon of mixing of two gases as shown in Fig. 8.

Before mixing After mixing


Complete Chemistry—JEE Main

The mixing of two gases shown in Fig. 8 is accompanied with increase in disorderlines of the two gases. This
. For an
.

DSsystem + DSsurroundings
When this isolated system attains an equilibrium state where the net interactions between system and surroundings
is zero, there will occur in
.
Mathematically, the criteria of spontaneity and equilibrium of an isolated system may be stated as
For spontaneity DS > 0
For equilibrium DS = 0

‘‘ ’’

of entropy is
dqrev
dS =
T
where dq T.
The characteristics of entropy function may be understood from the example
V1 to V2
dU = dq + dw
U = f(T T, dU = 0.
dV
dq = – dw = pdV = nRT
V
V2
or q = nRT ln
V1
qrev V
DS = = nR ln 2
T V1

V1 and V2 are state functions, DS S is a state function.

states that.

–1.

æÆ Tm* æÆ liquid (Tm* æÆ liquid (Tb* æÆ gas (Tb* æÆ gas (T


heat heat heat heat heat
where T *m and T *B
Chemical Thermodynamics

D fus H ∞ D vap H ∞
DSsÆl = and DSlÆ v =
T*
m Tb*

The molar enthalpy of fusion of solid water is 6008.2 J mol–1

D fus H m∞ 6008.2 J mol-1


D fus Sm∞ = *
= = 22.0 J K -1 mol-1
Tm 273.15 K
–1

D vap H m∞ 40585 J mol-1


D vap Sm∞ = = = 108.8 J K -1 mol-1
Tb* 373.15 K

DfusH°m and D H°m at temperatures T*m and T*b


The change in entropy of the surrounding will be same but of opposite sign and thus DsysS°m + DsurrS°m = 0.
mations take place at temperatures other than T*m and T*b
in nature. In this case, DsysS°m + DsurrS°m > 0.
Entropy change in a chemical reaction is obtained by using the expression

DrS° = Â Bn B Sm∞ , B - Â Bn B Sm∞ ,B

CH4 2 æÆ 2 2

DrS° = Sm 2 Sm° (H2 Sm(CH4 Sm 2


= (213.74 + 2 ¥ 69.91 – 186.26 – 2 ¥ –1
mol–1
= –242.70 J K–1 mol–1

equation.

DtotalS = DsysS + DsurrS > 0

At constant p, heat is exchanged between system and surroundings at the temperature of transformation. Hence, we
can write
q q D H
DsurrS = p ,surr = - p ,sys = - sys
T T T
D sys H
Hence, DtotalS = DsurrS + DsysS = - + DsysS
T
or TDtotalS = – (DsysH – TDsysS
At constant T
or TDtotalS = –Dsys(H – TS DsysG
where G H and S are state functions. Dropping the subscript ‘sys’
as DH and DS
DG = – TDtotalS
Complete Chemistry—JEE Main

Hence, For spontaneous process, DtotalS > 0 and thus DG < 0.


For equilibrium process, DtotalS = 0 and thus DG = 0.
For nonspontaneous process, DtotalS < 0 and thus DG > 0.
Four cases may be distinguished
D rH and DrS In this case, DG°(= DH° – TDS
is always spontaneous. Example is
2H2 2 æÆ 2H2 2 DrH° = –196.1 kJ mol–1, DrS° = 125.8 J K–1 mol–1,
At 298 K, DrG° = –233.6 kJ mol –1

DrH° and DrS° In this case, DG


nonspontaneous. Example is
N2 2 æÆ 2 DrH° = 66.4 kJ mol–1, DrS° = –121.8 J K–1 mol–1,
At 298 K, DrG° = 102.7 kJ mol , –1

D rH and DrS
DrG° = DrH° – TDrS T
DrH° > TDrS° i.e. T < DrH°/ DrS°
DrH° < TDrS° i.e. T > DrH°/ DrS°
= zero if DrH° = TDrS° i.e. T = DrH°/ DrS°
For example,
2N2 4 æÆ 2 DrH° = 57.2 kJ mol–1, DrS° = 175.8 J K–1 mol–1
D r H ∞ 57.12 ¥ 103 J mol-1
= = 325.4 K
Dr S∞ 175.8 J k -1mol-1
Thus if T > 325.4 K; spontaneous reaction
T < 325.4 K; nonspontaneous reaction
T = 325.4 K; equilibrium reaction
D rH and DrS
DrG = DrH – TDrS T
ÁDH Á< ÁTD S Á i.e. T > ÁDrH Á/ ÁDrS Á
ÁDH Á> ÁT D S Á i.e. T < ÁDrH Á/ ÁDrS Á
= zero if ÁDH Á= ÁTD S Á i.e. T = ÁDrH Á/ ÁDrS Á
For example,
2H2 2 æÆ 2NH3 DrH° = –92.2 kJ mol–1, DrS° = –198.8 J K–1 mol–1
-1
D r H ∞ 92.2 ¥ 10 J mol
3
= = 463.8 K
Dr S∞ 198.8 J k -1mol-1
Thus if T > 463.8 K; nonspontaneous reaction
T < 463.8 K spontaneous reaction
T = 463.8 K; equilibrium reaction
DrH° and DrS°are assumed to be independent of temperature.
DG
G = H – TS = (U + pV TS

dG = dU + pdV + VdP – TdS – SdT = (dq + dw pdV + Vdp – TdS – SdT


w
wmech = – pdV
wnonmech
q = TdS
dG = dwnonmech + Vdp – SdT
At constant T and p
Chemical Thermodynamics

dGT, p = dwnonmech or DGT, p = wnonmech

Ecell
cell, then the electrical work produced is
welec = –nFEcell
where n
spontaneous cell reaction and F is Faraday constant (=96487 C mol–1
DGT,p = dwnonmech
DGT,p = –nFEcell
For a spontaneous reaction, DGT,p < 0, hence Ecell > 0 For a Daniell cell (E
æÆ Zn2+ –

Cathode Cu2+ –
æÆ
2+
æÆ Zn2+
The decrease in free energy is DG –1 –1
–1
Thus, 212300 J mol of electrical work is produced per unit extent of cell reaction.

The standard free energy of formation of elements in their stable form aggregations at 1 bar pressure is taken to be
zero.

of the substance is formed from the stable form of elements at their stable states of aggregation.

The reaction to be referred is 12 H2 1


2 Cl2 æÆ DG° = –95.3 kJ mol–1
DG° of this reaction is said to be the standard free energy of formation (symbol: DfG
The free energy change in a chemical reaction may be computed
by the expression

DrG° = Â B n B D f G°B - Â B n B D f G°B


For example,
CH4 2 æÆ 2 2
DrG° = DfG 2 DfG°(H2 DfG°(CH4 D fG 2
¥ –1

= –817.7 kJ mol–1

Ê pˆ
For a gaseous phase DfG = DfG° + RT ln Á ˜
Ë p∞ ¯

DfG = DfG° + RT ln ÊÁ ˆ˜
c
For a substance in solution
Ë c∞ ¯
For a condensed phase DfG = DfG°
where p c° (= 1 mol dm–3
6.22 Complete Chemistry—JEE Main

Free energy of a reaction may be expressed in terms of standard free energy

Expression of free energy change of the reaction N2(g, pN2 2(g, pH2 æÆ 2NH3(g, pNH3

DrG = 2DfG(NH3 DfG(N2 DfG(H2


Expressing DfG in terms of DfG°,we get
È Ê pNH3 ˆ ˘ È Ê pN 2 ˆ ˘ È Ê pH 2 ˆ ˘
DrG = 2 Í D f G∞( NH3 , g) + RT ln Á ˜ ˙ - Í D f G∞( N 2 , g) + RT ln Á ˜ ˙ - 3 Í D f G∞(H 2 , g) + RT ln Á ˙
Î Ë p∞ ¯ ˚ Î Ë p∞ ¯ ˚ Î Ë p∞ ˜¯ ˚

È ( pNH3 / p∞) 2 ˘
= [2D f G∞( NH3 , g) - D f G∞( N 2 , g) - 3D f G∞(H 2 , g)] + RT ln Í 3˙
ÍÎ ( pN 2 / p∞)( pH 2 / p∞) ˙˚
= DrG° + RT ln Q°p
where Q°p is known as standard reaction quotient.

D rG
becomes
0 = DrG° + RT ln K°p
or DrG° = – RT ln K°p = –2.303 RT log K°p
where K°p (= Q°p
DfG°(NH3 –1
at 298 K. Hence,
DrG° = 2DfG°(NH3 DfG°(N2 DfG°(H2
–1 = –32.90 kJ mol–1

(-32.90 ¥ 103 J mol-1 )


Hence log K°p = – = 5.77
(2.303)(8.314 J K -1 mol-1 ) (298 K)
5.
K°p

MULTIPLE CHOICE QUESTIONS ON SECTION 3

1. In the mixing of two ideal gases at the same temperature, the entropy of mixing is not zero because

V1 to V2 is
V1 V2 V1 V2
DS = nR ln DS = nR ln DS = nRT ln DS = nRT ln
V2 V1 V2 V1
3. The transformation H2 æÆ H2 DsysS°= –20.5
J K–1. Which of the following facts will hold good for the change in entropy of surroundings (DsurS
DsurS° = ÁDsysS°Á DsurrS° > ÁDsysS° Á DsurrS° < ÁDsysS° Á DsurrS° = 0
4. A body of Cu at 100 °C is in contact with another body of Cu at 0 °C. There occurs transfer of heat from a hot
body to a cold body till both acquires the same temperature. If DhS° and DcS° are the changes in entropy of hot

DhS° + DcS DhS° + DcS DhS° + DcS DhS° = DcS°


Chemical Thermodynamics 6.23

6. For a reaction A2 æÆ DrH° = 50.0 kJ mol–1 and DrS° = 125 J K–1 mol–1. The reaction will be
spontaneous
T T > 400 K
T p = 1 atm
7. For a reaction A2 2 æÆ 2AB2, DrH° = –100 kJ mol–1 and DrS° = –250.0 J K–1 mol–1. The
reaction will be spontaneous
T T > 400 K
T T > 500 K
8. The reaction H2  H2 DrH° = 40.6 kJ mol–1 DrS°
will be about
–1
mol–1 –1
mol–1 –1
mol–1 –1
mol–1
æÆ Df H –1,
–1 mol–1 and –1 mol–1
S S
transformation is in equilibrium is

Á Zn2+ Mg2+ Á

–1 –1 –1 –1

11. The standard free energy change of reaction 2Fe2+ 2 æÆ 2Fe3+ – –1.

The standard emf of the cell producing this reaction will be about

2+ æÆ A2+ 9.65 at 300 K. The standard

13. The standard free energy change of the reaction 4NH3 2 æÆ 2


DfG°(NH3 –1, D G°
f
–1 amd DfG(H2 –1.
–1 –1 –1 –1

1
2 æÆ 2 2 æÆ 2
2
3 æÆ N2 2 3 æÆ 2 2
15. If DfH° –1, S° –1 mol–1 and S° –1 mol–1,
æÆ
T > 500K
T

– + + –
3
DrG – DrG° of the reaction N2 2 æÆ 2

–1 –1 –1 –1
6.24 Complete Chemistry—JEE Main

ANSWERS

HINTS AND SOLUTIONS


1. In the mixing of two gases, disorderliness or entrorpy of both the gases increases.
nRT
2. dU = 0 at constant temperature. Hence dq = -dw = + pdV = dV
V
dq dV V2
Thus dS = = nR or DS = nR ln
T V V1
2 æÆ H2 DtotalS° = DsysS° +
DsurrS° > 0. From this, it follows that DsurrS° > ÁDsysS° Á.
DtotalS° = DhS° + DcS° > 0.
DG < 0 and
thus w
6. For spontaneous process DrG = DrH – TDrS < 0.
Hence T > DrH/DrS = (50 ¥ 103 J mol–1 –1 mol–1

DrG° = DrH° – TDrS DrH – (–TDrS DrH > (–TDrS or


T < (–DrH D rS ¥ 103 J mol–1 –1
mol–1
D H ∞ 40.6 ¥ 103 J mol-1
DrG° = DrH° – TDrS DrS° = r = = 108.8 J K -1 mol-1
T 373 K
9. DrH° = DfH D fH = 0.33 kJ mol–1 – 0 = 0.33 kJ mol–1
DrS° = S S –1 mol–1 = 0.8 J K–1 mol–1

D r H ∞ 0.33 ¥ 103 J mol -1


T= = = 412.5 K
Dr S ∞ 0.8 J K -1 mol -1
10. DrG° = -nFE –1 –1 = – 50.18 kJ mol–1
n = 2.
D r G∞ 114.0 ¥ 103 J mol-1
Hence, E° = - = = 0.59 V
nF (2)(96500 C mol-1 )
RT
12. DG° = -nFE° and DG° = -RT ln K°. Hence nFE° = RT ln Kº or E° = ln K°.
-1 -1 nF
(8.314 J K mol )(300 K )
E° = [2.303 log 109.65 ]
(2)(96500 C mol -1 )
13. DrG° = 4DfG° DfG°(H2 DfG°(NH3 DfG° 2
–1

= –1226.2 kJ mol–1
14. Larger the change in gaseous molecules, larger the entropy change.
æÆ
DrH° = DfH° DfH° –1 = – 1.89 kJ mol–1

DrS° = S° S° –1 mol–1 = 3.36 J K–1 mol–1

DrH° < 0 and DrS° > 0, the reaction is spontaneous at all temperatures as DrG°
Chemical Thermodynamics

17. H+
2
( pNO / p∞)
DrG = DrG° + RT ln 2

( pN 2 / p∞)( pO2 / p∞) 2

(0.2) 2
Hence DrG – DrG° = RT ln = –RT
(0.2)(0.2)2
–1
mol–1
= 4015.2 J mol–1

MULTIPLE CHOICE QUESTIONS ON THE ENTIRE CHAPTER

Identify the correct choice in the following questions.

q w q /T qw
4. Which of the following is
qp qV q/T q + dw

2 at room temperature and pressure


2 2

H + pV U + pV U – pV H – pV
9. The relation DsubH°= DfusH°+ D H° is true at all

10. Which of the following expressions is


U = d q + dw DU = Dq + Dw DH = qp DS = q /T
11. If one gram of kerosene liberates 46.0 kJ of heat when it is burned, to what temperature can 0.25 g of kerosene
raise the temperature of 75 cm3
6.26 Complete Chemistry—JEE Main

14. A gas expands from 1.5 L to 6.5 L against a constant pressure of 0.5 atm and during this process the gas also
absorbs 100 J of heat. The change in the internal energy of the gas is

15. If a gas expands adiabatically from 1.0 L to 12.0 L against a constant pressure of 0.75 atm. The DU of the gas is

16. Molar heat capacity of water in equilibrium with ice at constant pressure is
–1 mol–1 –1 mol–1

The change in internal energy of the gas is

2æÆ 2 4 æÆ
2 2 + 2H2
3 æÆ æÆ H2
– + H+
2

pV pV g pg V pCp, m V CV, m = constant


20. An ideal gas undergoes expansion as shown in the following.

p2 > p3 p2 < p3 p2 = p3 p2 = p3
DH DU
21. For which of the following reactions, will DH be equal to DU
1
2 2 H2 2 2
2
2 N2 4 3 2 2
1
2 2 DH = 30.66 kJ mol –1
2
1
DH = DU DH > DU DH < DU DH = DU
2
23.
2 CH4 DH1
CH4 DH2

DH1 = DH2 DH1 > DH2 DH1 < DH2 DH1 = 2DH2
24. The enthalpy change for the reaction 3 2 at 1000 K is 176 kJ mol–1
DU for the reaction will be
–1 –1 –1 –1

25. The reaction of cyanamide, NH2 DU at 300 K was found


to be – 743 kJ mol–1 DH at 300 K for the combustion reaction
3
NH2 2 N2 2 2
2
would be
–1 –1 –1 –1

26. An ideal gas is heated from temperature T1 to T2


DH = DU DH = DU + pDV DH = DU + V DP DH = DU + nR (T2 – T1
Chemical Thermodynamics 6.27

27. The difference between D r H and D rU of the reaction 2C6H6 2 æÆ 2 2


–1 –1 –1 –1

28. Identify the reaction in which D r H < D rU.


2 4 2 æÆ 3 2 2 æÆ
2 2 æÆ H2 3 æÆ 2 2
29. Identify the reaction in which D r H > D rU.
6H12 6 2 æÆ 2 2 2 2 æÆ 2H2
3 æÆ 2 2 2 2 æÆ

D cU D cH D cA D cS
31. If D f H°(NH3 –1
D f U°(NH3
–1 –1 –1 –1

1
2 2 H2 2 2 2H2
2
1 1
2 2 H2 2 2 H2
2 2
of enthalpy of combustion at 25 °
4 2 2 2 4 2 2 2
4 2 2 2 4 2 2 2

2 2 2 2
2 2 2 2
35. The standard enthalpy of combustion at 25 °C of H2
and – 3920 kJ mol–1
–1 –1 –1 –1

36. The molar enthalpies of combustion of C2H2 –1,


2
2H 2
–1 –1 –1 –1

37. The enthalpy change of the reaction H+ – H2 –1. If the enthalpies of formation
of H+ 2
–1 –
–1 –1 –1 –1

38. The enthalpy change for the reaction H2 2 æÆ H2 2


–1. If the enthalpy of
–1
formation of H2 2 , the enthalpy of formation of H2
–1 –1 –1 –1
–1
2 2 . If the enthalpy
3 2 æÆ H2 4
–1
, the enthalpy of formation of H2 4
–1 –1 –1 –1

DionizH –1
and DionizH°(CH3 –1
. Predict which one of the

Ka Ka(CH3 Ka Ka(CH3
Ka Ka(CH3 Ka Ka(CH3
41. Enthalpy of neutralization of a strong acid and a strong base is
–1 –1 –1 –1
6.28 Complete Chemistry—JEE Main

42. In an endothermic reaction

DH
DU
43. In an exothermic reaction

DH
DU
2 2

45. The standard enthalpies of formation at 300 K for CCl4 2 2


–93 kJ mol–1 DU° at 300 K for the reaction CCl4 2 2
–1 –1 –1 –1


3
– H2 2–
3 DH1 = –41.84 kJ mol–1
+ – H2 DH2 = – 57.32 kJ mol–1
– H+ 2–
3 3
–1 –1
–1 –1
–1 and – 395.3 kJ mol–1

–1 –1 –1 –1

1 1
Df H Df H 2 Df H Df H Df H 2
2 2
1
Df H Df H 2 Df H 2 Df H Dc H°(C, graph Dc H
2 –1 and – 834
49. The enthalpies of formation of Al2 3 2 3 kJ mol–1 The
enthalpy change of the reaction Fe2 3 æÆ Al2 3
–1 –1 –1 –1
–1,
2 2
2 2 æÆ 2
–1 –1 –1 –1

51. The bond dissociation enthalpies of H2 –1


2

–1 –1 –1 –1
–1

–1 –1 –1 –1

53. The bond enthalpies of H H and Cl Cl are 430 and 242 kJ mol–1, DfH –1
,
the bond enthalpy of HCl would be
–1 –1 –1 –1
Chemical Thermodynamics 6.29

54. The enthalpy of combustion of H2 –1 and bond enthalpies of H


2
are 433 kJ mol–1 and 492 kJ mol–1 H is
–1 –1 –1 –1

e (N∫ –1, e –1 and e –1.

The enthalpy change of the reaction N2 2 æÆ 2NH3


–1 –1 –1 –1

Df H –1 and D f H –1 –1, then

–1 –1 –1 –1

Answer Q.57 to Q.60 based on the following informations. Enthalpies of formation of ethane, ethyene and benzene from
the gaseous atoms are – 2839.2, –2275.2 and –5536 kJ mol–1 –1.

–1 –1 –1 –1

58. The bond enthalpy of C=C is


–1 –1 –1 –1

59. Enthalpy of formation of kekule structure of benzene from gaseous atoms is


–1 –1 –1 –1

60. Resonance energy of benzene is


–1 –1 –1 –1

61. The unit of entropy is


–1 –1 –1

62. For a chemical reaction at equilibrium at temperature T


T > DH/DS T < DH/DS T = DH/DS T = DS/DH
2 2

–1
–1
mol–1 –1
mol–1
–1 mol–1 –1 mol–1

DS DH DS DH
DS DH DS DH

DS DH DS DH
DS DH DS DH

DSsys DSsurr DStotal DStotal

2 2 2NH3 2 2 2H2
2 2 2 2
Complete Chemistry—JEE Main

DSsys = DSsurr DSsys = – DSsurr


DSsys = DSsurr DSsys = DSsurr = 0

DSsys = DSsur DSsys = – DSsur DSsys DSsurr DSsys DSsurr

G = H – TS G = H + TS G = U – pV – TS G = U + pV + TS

75. The expression connecting DG° and K°eq of a reaction at equilibrium is


DG° = RT ln K eq
o
DG° = – RT ln K eqo
DG° = RT log K eq
o
DG° = – RT log K eqo

DU°300K = – 10.0 kJ mol–1 and DS°300K = – 45 J K–1 mol–1. The reaction

DS DS
DS T of the gas
78. Which of the following statements regarding the entropy is

2 2 3 2

RT
DG° = – nFE DG° = – RT ln K°p E° = log K°p DG = DG° + RT ln Qp
nF
 sure
DG° of the reaction A B is
RT RT RT RT log 4
2 H2 DS1
H2 H2 DS2
Predict which of the following is true.
DS1 = DS2 DS1 > DS2
DS1 < DS2 DS1 may be greater or smaller than DS2

DS DG DU Dp = 0
83. The term R ln Kp° is equal to
DG DG DG/T DG°/T

4NH3 2 2
Df H°/kJ mol–1 – 46.2 90.4 – 241.8
S°/J K–1 mol–1 192.5 205.0 210.6 188.7
Df G°/kJ mol–1 – 16.6 – 228.6

–1 –1 –1 –1

–1 –1 –1 –1

86. The standard entropy change of the reaction is


–1
mol–1 –1
mol–1 –1
mol–1 –1
mol–1
Chemical Thermodynamics

87. The standard free energy change of the reaction is


–1 –1 –1 –1

–1 –1 –1 –1

89. If the standard free energy change for a reaction is 1.546 kJ mol–1
constant for the reaction is

90. Which of the following species is


2 2
–1
–1 –1 mol–1 –1 mol–1 –1 mol–1

2 2
2 2 at constant temperature and pressure. The entropy change of the
system

94. The combustion reaction occurring in an automobile is 2C8H18 2 æÆ 2 2


reaction is accompanied with
DH DS DG DH DS DG
DH DS DG DH DS DG
95. For the transformation H2 Æ H2 DStotal (= DSsys + DSsurr

96. For a spontaneous endothermic reaction


DG DH DG DS > DH/T
–1
and 40 J K–1 mol–1, the boiling point of the
liquid will be

 DH° and DS° at temperature T


by the expression
DH° – TDS° = + RT ln K°p DH° – TDS° = – RT ln K°p
DH° = – RT ln K°p DH° + TDS° = – RT ln K°p

ANSWERS
6.32 Complete Chemistry—JEE Main

HINTS AND SOLUTIONS

5. Isolated system cannot exchange heat and matter.


6. Closed system can exchange heat and not matter.

q and w Dq and Dw
but simply as q and w.
1
11. Heat liberated =
4
Hence
–1 11500 11500
g–1 t cal i.e. t – 25 °C = °C = 36.65 °C
4184
. 4184
. ¥ 75
t = 36.65°C + 25°C = 61.65 °C
Ê 8.314 J ˆ
12. w = – pext DV ÁË 0.082 atm L ˜¯ = – 243.3 J

8.314
13. 0.082 atm L ∫ 8.314 J. 1 atm L = J = 101.3 J
0.082
Ê 8.314 J ˆ
14. DU = q + w ÁË 0.082 atm L ˜¯ = – 153.3 J

Ê 8.314 J ˆ
15. DU = w = – pext DV ÁË 0.082 atm L ˜¯ = – 835.9 J

Cp = (dq/dT p
temperature. Hence Cp = dq /zero = •.
17. DU = q + w = 100 J – 25 J = 75 J
3 requires energy.
pV g = constant where g = Cp, m /CV, m
20. In adiabatic expansion, temperature of the gas decreases where in isothermal expansion, temperature remains
p μ T, p2 > p3.
21. DH = DU only when Dvg = 0.
22. Dng DH = DU + (Dng RT suggests that DH > DU.
Chemical Thermodynamics 6.33

2 CH4
DHa
2H2 DHb
CH4 DH2
Hence DH1 =DHa + DHb + DH2
DHa and DHb DH1 > DH2.
24. Dng = 1. Hence
DU = DH – (Dng RT = 176 kJ mol–1 10–3 kJ K–1 mol–1
–1
= 167.7 kJ mol–1

25. DH = DU + (Dng RT = – 743 kJ mol–1 + ÊÁ ˆ˜ (8.314 ¥ 10–3 kJ K–1 mol–1


1 –1
Ë 2¯
26. H = U + pV. Hence DH = DU + D(pV DU + nRDT
27. D rH – DrU = (Dng RT –1
mol–1 –1

Dc U°
of the reaction.
31. The reaction is
1 3
N2 H2 Æ NH3 Dng = – 1
2 2
Hence Df U° = DfH° – (Dng RT ¥ 10–3 –1
= – 47.71 kJ mol–1

33. In enthalpy of combustion, the stochiometric number of the species being combusted is equal to one. The products
2 2

35. C6H10 2 C6H12


6H12 2 2 2
17
6H10 2 2 2
2
1
2 2 H2
2
DH = DH + DH – DH –1
= – 121 kJ mol–1

5
C 2H 2 2 2 2 DH1 = – 1300 kJ mol–1
2
2 2 DH2 = – 394 kJ mol–1
1
H2 2 H2 DH3 = – 286 kJ mol–1
2
Hence, for the equation
2 C 2H 2
DH = 2DH2 + DH3 – DH1 = [– 2 –1
= 226 kJ mol–1
37. DH = Df H (H2 Df H(H+ Df H –

– 57.3 kJ mol–1 = – 285.84 kJ mol–1 – Df H –

Df H° – –1
= – 228.54 kJ mol–1
6.34 Complete Chemistry—JEE Main

38. The reaction to be considered is H2 1


2 2 æÆ H2

H2 2 æÆ H2 2 DH = – 98.3 kJ mol–1
H2 2 æÆ H2 2 DH = – 187.4 kJ mol–1

DH –1
= – 285.7 kJ mol–1

2 æÆ 2 DH = – 298.2 kJ mol–1

2 + 1
2 2 æÆ 3 DH = – 98.7 kJ mol–1

H2 1
2 2 æÆ H2 DH = – 287.3 kJ mol–1

3 2 æÆ H2 4 DH = – 130.2 kJ mol–1
The formation of H2 4 2 2 æÆ H2 4

2 4
DH –1
= – 814.4 kJ mol–1
DionzH, weaker the acid.
42. In an endothermic reaction, heat is added to the system, Hproducts > Hreactants and thus DH
Hproducts < Hreactants and DH (or DU

DH = Df H 2 Df H Df H(CCl4 Df H(H2
–1

= –175 kJ mol–1
DU = DH – (Dng RT = – 175 kJ mol–1 10–3 kJ K–1 mol–1
= – 175 kJ mol–1 – 7.48 kJ mol–1 = – 182.5 kJ mol–1

DH = DH1 – DH2 –1

2 2 DHa = – 393.5 kJ mol–1


2 2 DHb = – 395.3 kJ mol–1

Hence
DH = DHb – DHa –1
= 1.80 kJ mol–1
2 2 D cH
1
2 2 D cH
2
Hence, for the equation
1
2
2
DH = DcH D cH
49. D r H = Df H(Al2 3 D f H(Fe2 3
–1
= –836 kJ mol–1

2 æÆ 2 DH = – 393.5 kJ mol–1
2 æÆ DH = – 110.5 kJ mol–1
2 2 æÆ H2 DH = – 241.8 kJ mol–1
Chemical Thermodynamics

2 2 æÆ 2

DH –1
= 41.2 kJ mol–1
51. H2 2
1 1
Df H [e e e [(435 + 243 – 2 –1
2 2
= – 92 kJ mol–1
DH for the equation

DH = Df H(H, D fH DHf –1
= 424 kJ mol–1

1 1
H2 DH1 = 430 kJ mol–1
2 2
1 1
Cl2 DH2 = 242 kJ mol–1
2 2
1 1
H2 Cl2 HCl DH3 = – 91 kJ mol–1
2 2

DH = DH1 + DH2 – DH3 –1 = 427 kJ mol–1


54. For the equation
1
H2 2 H2 DH = – 249 kJ mol–1
2
1
e e e –1
2
È 1 ˘
ÍÎ433 + 2 (492) ˙˚ kJ mol – 2e
–1 –1

e
1 È 1 ˘
ÍÎ+ 249 + 433 + 2 (492) ˙˚ kJ mol = 464 kJ mol
–1 –1
2

55.

Hence, DH = e (N ∫∫ e e ¥ 436 – 6 ¥ –1

= – 93 kJ mol–1

1
F2 Æ D f H° = 92.0 kJ mol–1
2
Æ D f H° = 536 kJ mol–1
1
F2 Æ DH –1
2
DH for the reaction
Æ
This reaction can be obtained by the following manipulations
6.36 Complete Chemistry—JEE Main

Hence e –1
= 521.5 kJ mol–1
57. e C–C = Enthalpy to break C2H6 into gaseous atoms –6 ¥ e C–H
= (2839.2 – 6 ¥ –1
= 373.98 kJ mol–1
58. e C=C = Enthalpy to break C2H4 into gaseous atoms –4 ¥ e C–H
= (2275.2 – 4 ¥ –1
= 631.72 kJ mol–1
59. e( e C–C – 3e C=C – 6e C–H –1

–1
= –53.68 kJ mol–1
63. Dng
DH 40.63 ¥ 103 J mol-1
64. DS = = = 108.9 J K–1 mol–1
T 373 K
65. For a spontaneous process, DG DG = DH – TDS
DG DH DS
66. DG DH DS
69. Increase in entropy will occur if Dng
reactants.
70. Decrease in disorderliness would result into the decrease in entropy.

heat absorbed by the system is equal to the heat lost by the surroundings, DSsys and DSsurr
but carry opposite sign.
DStotal = DSsys + DSsurr DSsys DSsurr
74. Enthalpy of an ideal gas depends only on the temperature.
76. DH° = DU° + (Dng RT = – 10.0 kJ mol–1 10–3 kJ K–1 mol–1
= – 12.49 kJ mol–1
DG° = DH° – TDS° = – 12.49 kJ mol–1 10–3 –1 mol–1 –1

DS
in entropy during decrease in temperature.

DG° = – RT ln ÊÁ ˆ˜ = RT ln 4
1 1
80. K°eq = . Hence,
4 Ë 4¯

82. Adiabatically implies q w = 0. Hence DU = q + w = 0


84. DH° = 4Df H DfH°(H2 DfH°(NH3 –1

= – 904.4 kJ mol–1
85. DU° = DH° – (Dng RT = – 904.4 kJ mol–1 10–3 kJ mol–1 –1

86. DS° = 4S S°(H2 S°(NH3 S 2


–1 mol–1
= [4
= 179.6 J K–1 mol–1
87. DG° = DH° – TDS° = [– 904.4 – 298 (179.6 10–3 –1 = – 957.9 kJ mol–1
88. DG° = 4 Df G DfG°(H2 DfG°(NH3
–1 = 4 D G
f
1
D fG (347.3 kJ mol–1 –1
4
Chemical Thermodynamics 6.37

89. DG° = – RT ln K°
1.546 103 J mol–1 = – (8.314 J K–1 mol–1 K°
1546
. ¥ 10
3
i.e. log K° =– i.e. K
2.303 ¥ 8.314 ¥ 773

91. DS = DH/T = (37300 J mol–1 –1


mol–1
92. H2
95. The reaction H2 Æ H2 DStotal
96. For a spontaneous reaction, DG < 0. Hence DH – TDS < 0 or DH < TD S.
DH 25 ¥ 103 J mol-1
97. Tb = = = 625 K
DS 40 J K -1 mol-1
98. For a reaction at equilibrium DG° = – RT ln K°p where DG° = DH° – TDS°.

MULTIPLE CHOICE QUESTIONS FROM AIEEE AND JEE MAIN

1. The correct relationship between free energy change in a reaction and the corresponding equilibrium constant Kc
is
DG° = RT ln K°c DG = RT ln K°c DG = RT ln K°c DG° = RT ln K°c
2. The enthalpy change for a reaction does not depend upon

T and p in which on p – V work in being done, the change in


G S
SV,E = 0, dGT,p SV,E < 0, dGT, p SV,E > 0, dGT,p SV,E = 0, dGT,p = 0

kJ mol–1
H2C==CH2 + H2 æÆH3 3
at 298 will be
–1 –1 –1 –1

5. The internal energy change when a system goes from state A to B is 40 kJ mol–1. If the system goes from A to B

–1 –1 –1

6. An ideal gas expands from 1 ¥ 10 –3


m to 1.0 ¥ 10
3 –2 3
m at 300 K against a constant pressure of
1 ¥ 105 N m–2. The work done is

7. The enthalpies of combustion of carbon and carbon monoxide are –393.5 and –283 kJ mol–1
enthalpy of formation of carbon monoxide is
–1 –1 –1 –1

8. For a spontaneous reaction the DG, equilibrium constant (K E°cell


6.38 Complete Chemistry—JEE Main

9. Consider the reaction N2 + 3H2 æÆ 2NH3 carried out at constant temperature and pressure. If DH and DU
are the enthalpy and internal energy changes for the reaction, which of the following expression is true.
DH < DU DH > DU DH DH = DU
10. If the bond dissociation energies of XY, X2 and Y2 D fH
for the formation of XY is –200 kJ mol–1. The bond dissociation energy of X2, will be
–1 –1 –1 –1
–1
11. The standard enthalpy of formation (DfH 4 . The additional

2
2 and enthalpy of sublimation of carbon

Ti, is the initial


temperature and Tf
Tf =Ti
Tf > (Tf
Tf >Ti Tf =Ti
Tf = (Tf
13. (DH – DU R = 8.314 J K–1 mol–1
–1 –1 –1 –1

14. The enthalpy changes for the following processes are listed below:
Cl2 Æ –1 Æ –1

I2 Æ –1 I2 Æ I2 –1

2 2

–1 –1 –1 –1

DU
–1 and
–1 K–1
R = 8.3 J mol
–1 –1 –1 –1

16. Identify the correct statement regarding a spontaneous process:

3 æÆ 2 DH° and DS° are + 179.1


kJ mol–1 and 160.2 J K–1 mol–1 DH° and DS° do not change with

1 Cl DdissH° DcgH° DhydH°


2 æææÆ æææÆ Cl– æææÆ Cl–
2

2
–1 –1 –1 –1

DdissH°(Cl2 –1; DcgH –1; and DhydH°(Cl– –1;


Chemical Thermodynamics 6.39

2, Y2 and XY3 are 60, 40 and 50 J–1 K–1 mol–1


1 3
2 X2 + 2 Y2 æÆ XY3, DH = –30 kJ mol–1. This reaction will be at equilibrium at temperature equal to

H2 æÆ H+ –
DH = 57.32 kJ mol–1
H2 + 1
2 2 æÆ H2 ; DH = –286.20 kJ mol–1
DfG°(H+

ions at 25 °C is
–1 –1 –1 –1

21. In a fuel cell, methanol is used as fuel and oxygen gas is used as an oxidizer. The reaction is
CH3 3
2 2 æÆ 2 H2
3 2 2
–394.4 kJ mol–1 –1

the fuel cell will be

22. The standard enthalpy of formation of NH3 is – 46.0 kJ mol–1. If the enthalpy of formation of H2 from its atom is
–436 kJ mol–1 and that of N2 is –712 kJ mol–1 3 is
–1 –1 –1 –1

2 3 at 500 °C is as follows.
2 4
Al2 3 æÆ 2; DrG = –966 kJ mol–1
3 3

T, DH and DS Te is the
temperature at equilibrium, the reaction would be spontaneous when
T = Te Te > T T > Te Te is 5 times T

dm3 3 at 27 °C
–1 –1 –1 –1

y change (DH 2H 5 2 æÆ 2 2 2
1366.5 kJ mol–1
–1 –1 –1 –1

27. The incorrect expression among the following is


DGsystem Vf
= -T w = –nRT ln
DS total Vi
DH ∞ - T DS ∞
K∞ = K° = exp(–DG°/RT
RT

q and w for the process will be


–1 mol–1
(R = 83.14 J K
q = +208 J, w q = –208 J, w q = –208 J, w q = +208 J, w = +208 J
Complete Chemistry—JEE Main

29. For complete combustion of ethanol, C2H5 2 æÆ 2 2 the heat produced as measured
in bomb calorimeter, is 1364.47 kJ mol–1 at 25°C. Assuming ideality the enthlapy of combustion, DcH, for the
reaction will be (R = 8.314 J K–1 mol–1
–1 –1 –1 –1

30. The entropy (S


–1
CH4 mol–1 2
–1
mol–1
–1
2 mol–1 H2 –1
mol–1
The entropy change (DS CH4 2 æÆ 2 2
–1 mol–1 –1 mol–1 –1 mol–1 –1 mol–1

–1
31. The standard enthalpy of formation (DfH°298K 4 is –74.9 kJ mol

2 molecule

32. Consider the reaction 2 2 æÆ 2N2 5 DrH = –111 kJ mol–1


If N2 5 2 5 D rH DH of sublimation
for N2 5 is 54 kJ mol–1
–1 –1 –1 –1

¥ 1023 mol–1
¥ 10 3
¥ 103 nm
¥ 104 ¥ 10 nm
4

34. The following reaction is performed at 298 K.


2 2

2 Kºp = 1.6 ¥ 1012


R ¥ 1012 –1
–1 +R ¥ 1012
12
–1 ln (1.6 10 )

R (298 K )
¥ 86600 J mol–1 – R ¥ 1012
Kp
1 1
2 Æ 2 Æ
2 2 M Æ MO
Identify the correct statement:
T

T Æ is spontaneous. C Æ CO
20
T
Æ H S 1200
–1 –1 –1
– 29.8 kJ mol and – 0.100 kJ K mol at 298 K. The equilibrium constant for
T/K
the reaction at 298 K is:
–10 10
Chemical Thermodynamics

the correct statement about the reaction among the following.


H S H S
H S H S
38. If 100 mol of H2 2 2
1 bar pressure is: (R = 8.3 J K–1 mol–1
2H2 2 2H2 2

39. The heats of combustion of carbon and carbon monoxide are – 393.5 and – 283.5 kJ mol–1
of formation (in kJ mol–1

ANSWERS

HINTS AND SOLUTIONS


1. The correct expression is DG° = –RT ln K°c
2. The enthalpy change of a reaction does not depend upon the nature of intermediate path.
sysS + dsurrS > 0 or dsysS – dsysH/T > 0
H = U + pV TdS – d(U +pV TdS – dU – pdV –Vdp > 0
p (i.e. dp TdS – dU – pdV > 0
At constant U and V, this reduces to dSU,V > 0.
G = H – TS TdS – d(G +TS TdS – dG – TdS – SdT > 0 or – dG – SdT > 0.
At constant T T, p > 0 or dG T, p < 0
4. H2C==CH2 2 H3 3

DH = 4e + eC==C + e –e – 6e ¥ –1 = 125 kJ mol–1


U=0
6. The work done is w = –pDV = – (10 N 5 m–2 –2 m3 – 10–3 m3 ¥ 102 N m = –900 J
2 æÆ 2 DH = –393.5 kJ mol–1
2 æÆ 2 DH = –283 kJ mol–1

2 æÆ DH –1 = –110.5 kJ mol–1
8. For a spontaneous reaction, DG E°cell producing the
6.42 Complete Chemistry—JEE Main

DH = DU + (Dng RT Dng = –2. Hence, DH < DU.


1 1
2 X2 2 Y2 æÆ

1
DfH° = 1
2
e X2 + 1
2
eY2 – eXY = 1
2
e X 2+ 1
2
( 12 e X 2 ) – eX = – eX
2
4 2
Hence eX2 = – 4(DfH – 4(–200 kJ mol–1 –1

2 æÆ CH4 DfH° = –74.84 kJ mol–1


4 æÆ
Hence, in addition to DfH°(CH4 2 and enthalpy of sublimation
of carbon.

Tf > (Tf
1
2 2 
For this reaction, Dng = 1/2. Hence
DrH° – DrU° = (Dng RT –1 mol–1 –1

1I 1 DfH°
ææÆ
14. 2 2 2
Cl2
Æææ

Æææææææ

ææææææÆ
1
2

1I 1 –211.3 kJ
2 2 2
Æææ

1
2

ææÆ
Cl ◊◊◊ Cl

Hence, Df H° = ÎÈ 12 (62.76) + 12 (151.0) + 12 (242.3) - 211.3˘˚ kJ mol-1 = 16.73 kJ moll-1

2 Æ H2
For this process, Dng = +1. Hence
DrU = DrH – (Dng RT = (41 ¥ 103 mol–1 –1 mol–1
= 41 ¥ 103 J mol–1 – 3095.9 J mol–1 = 37904 J mol–1 = 37.904 kJ mol–1
16. For a spontaneous process DStotal (= DSsystem + DSsurroundings DSsurroundings = 0.
Hence, DSsystem

DH ∞ 179.1 ¥ 103 mol-1


Teq = = = 1118 K
DS ∞ 160.2 J K -1mol-1
At T > Teq, DG
18. According to Hess’s law
-1
DH = 1
2 DdissH° + DegH° + DhydH° = ÎÈ 12 (240) - 349 - 381˘˚ kJ mol = –610 kJ mol–1

DS = S(XY3 1
2 S(X2 3
2 S(Y2 (50 - 12 ¥ 60 - 32 ¥ 40) J K -1 mol -1 = – 40 J K–1 mol–1

DG = 0 = DH – T DS = –30 ¥ 103 J mol–1 –T (–40 J K–1 mol–1


30 ¥ 103
T= K = 750 K
40
Chemical Thermodynamics 6.43

20. For the reaction, H2 æÆ H+ –

DrH = Df H (H+ Df H –
Df H (H2
57.32 kJ mol–1 = 0 + Df H – –1

Df H – –1

21. For the reaction CH3 3


2 2 æÆ 2 2
Dr H° = Df H 2 Df H°(H2 Df H°(CH3
= (–394.4 – 2 ¥ –1

= –702.6 kJ mol–1
D r H ∞ -702.6 kJ mol -1
h= = = 0.9678
Dc H ∞ -726 kJ mol -1

1
2 N2 H2 æÆ NH3
3
2
DH° = –46.0 kJ mol–1
æÆ H2 DH° = –436 kJ mol–1
æÆ N2 DH° = –712 kJ mol–1
We require to calculate
3 æÆ DH°1
3 1
This is obtained by the manipulation 2 2

-1 -1
Hence, DH°1 = ( + 46.0 + 32 ¥ 436 + 12 ¥ 712) kJ mol = 1056 kJ mol

e = 1
3 (1056 kJ mol–1 –1

23. The basic reactions area:


2
3
(2Al3+ ) + 4e- æÆ 34 Al and 2
3
(3O 2- ) æÆ O2 + 4e-
2
3
Al2 O3 æÆ 43 Al + O 2
Thus, using the expression DG= –nFE, we get
DG 966 ¥ 103 J mol-1
E=– =- = -2.50 V
nF (4)(96500 C mol-1 )
Thus,
DG = DH – TDG
Reactions at equilibrium: DG = DH – TeDS = 0
For a reaction to be spontaneous: DG = DH – TDS < 0
DG T > Te as both DH and DS
100 ˆ
= (2 mol)(8.314 J K -1 mol -1 ) ÊÁ 2.303 ¥ log
V2 -1
25. DS = nR ln ˜ = (2) (8.314) (2.303) J K = 38.3 J K
–1
V1 Ë 10 ¯
DrH = DrU + (Dng RT DrU = DrH – (Dng RT
Dng = –1. Hence
DrU = –1366.5 kJ mol–1 ¥ 10–3 kJ K–1 mol–1
–1
 1364.0 kJ mol–1
27. The correct thermodynamic expressions are
Vf
DGsys = –TDStotal; w = –nRT ln ; DG° = DH°–TDS° = –RT ln K°
Vi
DH ∞ - T DS ∞
is incorrect as ln K° = -
RT
6.44 Complete Chemistry—JEE Main

28. For an isothermal expansion of an ideal gas, DU q = –w


absorbed, q = 208 J and w = – 208 J.
Dng = –1. Hence
DH° = DU° + (Dng RT = (–1364. 47 kJ mol–1 ¥ 10–3 kJ K–1mol–1
= –1366.95 kJ mol–1
4 2 æÆ 2 2
DS° = S 2 S°(H2 S°(CH4 S 2
= (213.6 + 2 ¥ 69.9 –186.2 – 2 ¥ –1 mol–1
= –242.8 J K–1 mol–1

2 æÆ CH4 DfH°298K= –74.9 kJ mol–1

CH4 æÆ
for which we must require the enthalpy change for the reactions
æÆ DsubH
H2 æÆ DatH°(H2

2 æÆ 2N2
2 5 DrH = –111 kJ mol–1
2N2 5 æÆ 2N2 5 DrH = –2 ¥ 54 kJ mol–1
Add
2 2 æÆ 2N2 5 DrH = – (111 + 2 ¥ –1 = –219 kJ mol–1

CH4 Æ DH1 = 4e
C 2H 6 Æ DH2 = e + 6e
From DH1, we get eC–H –1 = 90 kJ mol –1

From DH2, we get eC–H = DH2 – 6eC–H = (620 – 6 ¥ –1 = 80 kJ mol–1

e C -- H (80 kJ mol-1 )
E= = = 1.33 ¥ 10–22 kJ = 1.33 ¥ 10–14 J
NA (6.02 ¥ 1023 mol-1 )

hc (6.626 ¥ 10-34 J s) (3 ¥ 108 m s -1 )


l= = = 1.495 ¥ 10–6 m
E (1.33 ¥ 10-19 J )
= 1.495 ¥ 103 nm
Dr RT ln Kºp = –R 12

Also, DrGº = 2DfG 2 D fG


Hence,

D fG 2 [DrGº + 2 Df
1
2 = 1
2 [–R 12 –1

DG° = – RT ln Kp
1
2 2 Æ DG°1
M (s) + 12 O 2 (g) Æ MO(s); DG∞2 = postive
C(s) + MO(s) Æ CO(g) + M (s) DG∞3 = DG∞1 - DG∞2
Chemical Thermodynamics

DG°2 < DG°1 at T > 1200 K, DG°3 T < 1200 K, DG°2 > DG°1, and thus DG°3
and hence the reaction will be spontaneous.
G H°– T S° = (– 29.8 kJ mol–1 –1 mol–1
G° = – RT ln K°p, we get K°p = 1
G H–T S
H S G H > T S. At high temperature,
G H<T S
2

nRT (1 mol)(8.3 kPa dm3 K -1 mol-1 )(300 K )


V= = = 24.9 dm3
p (102 kPa )
Work of expansion of 1 mol of gas against 1 bar pressure is
w = –p( V 2 3 3 ∫ – 2490 J = – 2.49 kJ
w = – p V = –( n RT –1
mol–1 ∫ – 2.49 kJ

2 liberated from the decomposition of 100 mol of H2 2, which is 50 mol, expands against
1 bar pressure, then the work done is
w –1
mol–1 ∫ 124.5 kJ

2 Æ 2 cH = – 393.5 kJ mol–1
1
2 Æ cH = –283.5 kJ mol–1
2
1
2 Æ fH
2

–1
fH cH cH

= –110.0 kJ mol–1
7
Chemical and Ionic
Equilibria

UNIT 1 Chemical Equilibrium

Irreversible Reactions

+ – (aq) æÆ 2O(l)
+(aq) + Cl– (aq) æÆ

4(aq) + 5Fe2+ +(aq) æÆ 2+(aq) + 5Fe3+ 2O(l)
Reversible Reactions

2 2
N2 2 3
N 2O 4 2NO2

dynamic equilibrium
Equilibrium Constant Kc

The rate of reaction at constant temperature is directly proportional to the product of molar concentrations of

chemical equation.
2O 4 2NO2
r μ[N2O4] or r = k [N2O4]
rb μ[NO2] 2 or rb = kb [NO2]2
2
kf [ NO 2 ]eq
k [N2O4]eq = kb [NO]2eq or Kc = k = [ N O ]
b 2 4 eq
The constant Kc
7.2 Complete Chemistry—JEE Main

Notes:

Product of concentrations of products each raised to its sttoichiometric number


Kc = (1)
Product of concentrations of reactantss each raised to its stoichiometric number

Kc = (mol dm–3)Dn (2)


where Dn
Dn = S Pn P - S Rn R (3)

Standard Equilibrium Constant Kc° Kc


–3
the species and the standard concentration (symbol c
For the reaction N2O 4 2NO2

([ NO 2 ]/c°) 2 [ NO 2 ]2 1 K c
Kc° = Kc° = ◊ =
[ N 2 O 4 ]/c° [ N 2 O 4 ] c° c°
Kc and Kc° is Kc = Kc° (c°)Dng (4)
Equilibrium Constant Kp
2O 4 2NO2
r = k ¢ p N 2O 4 rb = kb¢ (pNO2)2
r = rb and hence
k¢ ( pNO2 ) 2
k ¢ pN2O4= kb¢ (pNO2)2 or Kp = =
kb¢ ( pN 2 O 4 )
Kp is (atm)Dn or (bar) Dn where Dn

Dn = S P (g )n P (g ) - S R (g )n R (g ) (5)

Illustrations

3 4NO2 2 2 3 P p is

1024( pO2 )7
Kp = ( p - 7 p ) 4
O2

where pO2 2

3 4NO2 2 2
P – 4pO 4pO 2pO pO
2 2 2 2

p=p 3
+ pNO2+ p 2O
+ p O2
= (P – 4pO2) + (4pO2) + (2pO2) + (pO2) = P + 3pO2
Chemical and Ionic Equilibria 7.3

( pNO2 ) 4 ( pH 2O )2 ( pO2 ) (4 pO2 )4 (2 pO2 )2 ( pO2 ) 1024( pO2 )7 1024( pO2 )7


Kp = = = =
( pHNO3 )4 ( P - 4 pO2 )4 [( p - 3 pO2 ) - 4 pO2 ]4 ( p - 7 pO2 )4

2O 4 2NO2 Kp 2O 4

n 2O 4 a
2O 4
N 2O 4 2NO2
n(1 – a) n(2a)
ntotal = nN2O4+ nNO2= n(1 – a) + n(2a) = n(1 + a)
2O4 and NO2
Ê nN 2O4 ˆ n(1 - a ) (1 - a )
p N 2O 4 = x N 2O 4 p = Á ˜ p= p= p
Ë n ¯
totaln(1 + a ) (1 + a )
Ê nNO2 ˆ n(2a ) (2a )
pNO2 = xNO2 p = Á ˜ p= p= p
Ë ntotal ¯ n(1 + a ) (1 + a )
2
pNO [{2a /(1 + a } p]2 Ê 4a 2 ˆ
= 2
=Á p
Kp is Kp = p
N 2O4 {(1 - a ) /(1 + a )} p Ë 1 - a 2 ˜¯
Standard Equilibrium Constant K°p Kp°
1
p
reaction N 2O 4 2NO2
( pNO2 / p∞) 2 ( pNO2 ) 2 1 K p
Kp° = ( p = =
N 2O4 / p∞) ( pN O ) 2 p∞ p∞
2 4

Kp and Kp° is
Kp = Kp° (p°)Dn (6)

Relation Between Kp and Kc


pV = nRT or p = (n/V)RT = cRT
For the reaction N 2O 4 2NO2
2
( pNO2 ) ([ NO 2 ]RT ) 2
Kp = p = = K c ( RT )
([ N O ]RT )
N 2O4 2 4

Kp = Kc (RT) Dn

Illustration For the reaction 2SO3 2SO2 2 Kp


Kc
Dn = (2 + 1) – 2 = 1

Since Kp = Kc(RT)Dn

Kp (1.80 Pa)
Kc = Dn g
= ¥ –4
Pa J–1 ¥ –4
m–3 mol
( RT ) (8.314 J K -1 mol-1 ) (700K)

¥ mol dm–3

po
7.4 Complete Chemistry—JEE Main

Relation Between Kp° and Kc° 2O 4 2NO2


2 2
Ê [ NO 2 ]ˆ Ê c∞ RT ˆ
Ë c° ¯ ÁË p° ˜¯
2
( pNO2 /p°) o Ê c∞ RT ˆ
= = Kc Á
Ë p∞ ˜¯
Kp° =
( pN 2O4 /p°)2 Ê [ N 2 O 4 ]ˆ 2 Ê c∞ RT ˆ
Ë c° ¯ ÁË p° ˜¯
Dn
Ê c∞ RT ˆ
Kp° = Kc° Á
Ë p∞ ˜¯
(8)

Illustration Kc° 3 2SO2 2

Kp° ¥ –5 and p

K ∞p K ∞p p∞
Kc° = Dn g
= (since Dn = 1)
(c∞ RT / p∞) c∞ RT
(1.80 ¥ 10-5 )(1 bar )
=
(1 mol dm -3 )(8.314 ¥ 10-2 bar dm3 mol-1 K -1 )(700 K)
¥

Equilibrium Constant Kx Kx

For the reaction N2O 4 2NO2

( xNO2 ) 2
Kx = ( x
N 2O4 )

Relation Between Kp and Kx Since xB = pB /ptotal


( pNO2 / ptotal )2 Ê 1 ˆ
= Kp Á
Kx = ( p
N 2O4 / p total ) Ë ptotal ˜¯
Dn g
Ê 1 ˆ
Kx = K p Á
Ë ptotal ˜¯

Equilibrium Constant Kn Kn

For the reaction N2O 4 2NO2


(nNO2 ) 2
Kn = ( n
N 2O4 )
Ê nB ˆ
Relation Between Kp and Kn pB = xB ptotal = Á p
Ë ntotal ˜¯ total
Since

( pNO2 ) 2 [(nNO2 / ntotal ) ptotal ]2 Ê ptotal ˆ


= = Kn Á
Ë ntotal ˜¯
Kp =
( pN ) (nN 2O4 / ntotal ) ptotal
2 O4

Dn g
Ê ptotal ˆ
Kp = K n Á
Ë ntotal ˜¯
(11)
Chemical and Ionic Equilibria 7.5

Note: Dn
Kp = Kc = Kx = Kn (12)

Manipulation of Chemical Equations and the Associated Equilibrium Constants


Reversing a Chemical Equation
2O 4 2NO2 2 N 2O 4
( pNO2 ) 2 ( pN 2 O 4 )
KP = ( p KP¢ = ( p ) 2
N 2O4 ) NO 2

KP¢ =1/Kp
2. Multiplying a Chemical Equation by a Constant
1 1
2O 4 2NO2 2 [N2O4 2NO2 2 N 2O 4 NO2
( pNO2 ) 2 pNO2
KP = ( p KP¢ = ( p 1/ 2
N 2O4 ) N 2O4 )

KP¢ =Kp(1/2)
n
n
(Kp)new = (Kp)n
3. Addition of Two Chemical Equations

( pNO )2
N2 K p1 =
2 ( pN 2 )( pO2 )
( pNO2 ) 2
2NO2 K p2 =
2 ( pNO ) 2 ( pO2 )

( pNO2 ) 2
N2 2NO2 Kp =
2 3 ( pN 2 )( pO2 )2
Kp3 = Kp1 Kp2

4. Subtraction of Two Chemical Equations

( pNO2 ) 2
N2 2NO2 K p1 =
2 ( pN 2 )( pO2 ) 2

( pNO ) 2
N2 + O2 K p2 =
( pN 2 )( pO2 )

( pNO2 ) 2
2NO Æ 2NO2 K p3 =
2 ( pNO ) 2 ( pO2 )

K p2
Kp3 = K ;
p1
7.6 Complete Chemistry—JEE Main

Illustration
(i) Br2 2 KP1 ¥ 4

KP2 –2
(ii) Br2 2 2BrF3
Kp 3 2

1 1
2 2

K 1p/12 (2.25 ¥ 104 )1 / 2


Kp = = = (3600 bar 2 )1 / 2 = 60 bar
K 1p/22 (6.25 bar -2 )1 / 2

Equilibrium Constant of a Reaction Involving Condensed Phase(s)

( pHI ) 2
2 2(s) Kp =
pH2
Reaction Quotient and Equilibrium Constant
a p )+b pB) æÆ £ c p C) + d p D)
( pC )c ( pD ) d
Qp =
( pA ) a ( pB )b

( pC )ceq ( pD )eq
d
Kp = a
( pA )eq ( pB )beq
p )eq

Qp > Kp

Æ reactants)
Qp < Kp
Æ
Qp = Kp
Illustration 2 2 3
The reaction sets in is N2 2 3 Kc ¥ 2 (mol/L)–2

[ NH3 ]2 (8.0 mol/ 20 L) 2


Qc = = ¥ 3
(mol/L)–2
[ N 2 ][H 2 ]3 (1.5 mol/ 20 L)(2.0 mol/ 20 L)3
Since Qc > Kc

Le Chatelier’s Principle

If a system at equilibrium is subjected to a change, the system adjusts to a new equilibrium stage in such a
way so as to oppose or reduce the said change.
Chemical and Ionic Equilibria 7.7

is added

2 2 3 D rH –1

(i) Effect of Changing Pressure

Note:
2 2 3

(ii) Effect of Changing Temperature Since DrH


æÆ
æÆ

Note:


d ln K eq Dr H ∞
= (13)
dT RT 2

∞ = - D r H ∞ + constant
ln K eq (14)
RT
K°eq T DrH°/R

Fig. 1
7.8 Complete Chemistry—JEE Main

D rH T

D rH T

(iii) Effect of Adding One of the Components 2

3
(iv) Effect of Adding Inert Gas

Inert Gas Added at Constant Volume

Ê nB ˆ Ê ntotal RT ˆ nB RT
pB = xB ptotal = Á Á ˜=
Ë ntotal ˜¯ Ë V ¯ V
V xB is decreased while ptotal
nB
Inert Gas Added at Constant Pressure

Ê nB ˆ
pB = xB ptotal = Á p = (decreases) (constant) = decrease
Ë ntotal ˜¯ total

Dn Qp Kp
Qp becomes
Kp
Dn Qp Kp
Qp becomes
Kp

Note: Dn
2 2

MULTIPLE CHOICE QUESTIONS ON UNIT 1

2 2

2 2 æÆ 2SO3 Kp ¥ bar–1 Kc
will be
¥ 13
dm3 mol–1 ¥ 11
dm3 mol–1
¥ dm3 mol–1 ¥ dm3 mol–1
2 2 2 2
Chemical and Ionic Equilibria 7.9

K p= K c
(a) 2SO3 2SO2 2 2 2
(c) N2 2 3 2

(a) is endothermic (b) is exothermic

T
T

Kºp
T
T
T
T
2 2 Kc

–1
2 2 Kc
–1 –1
Ls
–2
L2 s–1 –1
L s–1 L–1 –2
L2 s–1
–1
2 Kc 2

3(s) CaO(s) + CO2 Kp 3


3 dissociated is

2 2

Kp V and T
ptotal/Kp
(a) 2 (b) 4 (c) 6 (d) 8

(i) BrF3 2 BrF5 Kp ¥ 35 atm–1


2 ClF3 Kp ¥ 12 atm–1

2 BrF3 Kp ¥ atm–1
The Kp 3 5
¥ 38 ¥ 38 ¥ 38 ¥ 16
Kc 2O(l) 2

¥ –3 mol L–1 ¥ –3 mol L–1 ¥ –3 mol L–1 ¥ –3 mol L–1

2N2 2 æÆ
2
2
7.10 Complete Chemistry—JEE Main

3(s) in the reaction


CaCO3(s) CaO(s) + CO2
2

+ – Kc1
3 2O(l) 4
(ii) CO2 2O(l) 2CO3 Kc2
CO (aq) +(aq) + CO2– Kc3
2 3 3
O(l) + – Kc4
2
Kc + (aq)
+ CO32–
3(aq)
+ CO2 2O(l) 4
2 K /K2 2 K K /K2
(a) Kc1Kc2Kc3/Kc4 (b) Kc1Kc2 c3 c4 (c) Kc1Kc2/Kc3Kc4 (d) Kc1 c2 c3 c4
2 2 Kc 2 2
Qc
(a) Qc > Kc
(b) Qc > Kc
(c) Qc < Kc
(d) Qc < Kc
5 Br2 2 Kp ¥ –14
bar4 2] = 4 ¥ –5
bar and
[F2] = 4 ¥ –4
5 will be
¥ –4
¥ –4
¥ –8
¥ –8
bar

2 2 2 Kp1
(ii) N2 2 3 Kp2
3 2 2 Kp3
Kp 2 2
(a) K 1p/22 K p 3 / K 3p1/ 2 (b) K p1 K 1p/22 / K 3p 3/ 2 (c) K p 2 K 1p/32 / K 3p1/ 2 (d) K p 2 K 1p/32 / K 1p/12

2(s) Ba2+(aq) + 2F– Ks ¥ –6 3


+(aq) + F–
Ka ¥ –4
+
Kc 2 (aq) Ba2+
¥ –3

ANSWERS

HINTS AND SOLUTIONS

2 2 p p 2
=p2
pH 2 pI2 (0.08 atm)(0.08 atm)
Kp = 2
= = 4.0
( pHI ) (0.04 atm) 2
Chemical and Ionic Equilibria 7.11

2 2 2SO3 Dn
Dn
Since Kp = Kc (RT)
Kc = Kp (RT)–Dn ¥ bar–1 ¥ –2 bar dm3 mol–1 –1

¥ 11 dm mol–1
3

Ê 40 g ˆ 7.2 g
ÁË 100 g ¥ 18 g˜¯ = 7.2 g; = 0.4 mol
18 g mol-1
H 2 O(g) + CO(g)  H 2 (g) + CO 2 (g)
(1- 0.4) mol (1- 0.4) mol 0.4 mol 0.4 mol
Since Dn Kc = Kn
(0.4 mol)(0.4 mol)
Kc = = 0.44
(0.6 mol)(0.6 mol)
Dn Kp = Kc

Dr H ∞
K°eq is ln K°eq = - + constant or K°eq = Ae - D r H ∞ / T
T
D rH K°eq
positive
D rH K°eq = + constant K°eq will increase linearly
T
with increase in 1/T

2ICl(g)  I2 (s) + Cl2 (g)


0.75 M -x x x

[I2 ][Cl2 ] x2
K c= = = 16
[ICl2 ] (0.75 - x)2
x
= 4 or 5x x
0.75 M - x
0.6 M
¥ 100 = 80%
0.75 M
kf 1.54 mol-1 L s -1
Keq = k /kb kb = = = 3.08 mol-2 L2 s -1
K eq 0.50 mol L-1

[I]2 (0.5 mol/ V )2


I 2 (g)  2I(g) Kc = = = 0.01 mol L-1
1.0 mol 0.5 mol [I 2 ] (1.0 mol/ V )
V = 25 L
Kp pCO2
pV (1.16 atm)(10 L) 1
is n= = = mol
2 RT (0.082 L atm K -1 mol-1 )(1160 K ) 8.2
Ê 1 molˆ
3 m = nM = ÁË ˜¯ –1
8.2
12.2 g
3 dissociated =
20 g
7.12 Complete Chemistry—JEE Main

2
p –p 2p p +p
p +p p fi p
( pI ) (2 p) 2
(2 ¥ 0.05 atm) 2
2
Kp = ( p ) = p - p = (0.07 - 0.05) atm = 0.5 atm
I2 0
p

p(1 – 1/3) p/3 p/3


Ê
ptotal = p 1 +

=
4
p
Ë 3¯ 3
pA pB ( p / 3)( p / 3) p (4 / 3) p
Kp = = = The ratio ptotal/Kp = =8
pA B p(1 - 1 / 3) 6 ( p / 6)

K p (iii ) K p (i ) (7.8 ¥ 1027 atm -1 )(8.6 ¥ 1035 atm -1 )


Kp = = ¥ 38
K 2p ( ii ) (7.8 ¥ 1012 atm -1 ) 2

Ê 1 atm ˆ
K p = pwater = 31.2 Torr = (31.2 Torr) Á = 0.041 atm
Ë 760 Torr ˜¯
Dn 2O(l) 2

(0.041 atm)
Kc =Kp (RT)–1 = ¥ –3
mol L–1
(0.082 L atm K -1 mol-1 )(300 K )

2
3

Kc = K2c1Kc2Kc3/K2c4
[ClF]2 (3.65 M) 2
Qc = = = 666.1 Since Qc > Kc
[Cl2 ][F2 ] (0.2 M)(0.1 M)

[Br2 ][F2 ]5 (4 ¥ 10-5 bar)(4.0 ¥ 10-4 bar)5 4.096 ¥ 10-25 bar 6


2
5] = = = = 4.0 ¥ 10-8 bar 2
Kp (1.024 ¥ 10-14 bar 4 ) 1.024 ¥ 10-14 bar 4

[BrF5] ¥ –4 bar
3 1
2 2
K p 2 K 1p/32
Kp =
K 3p1/ 2

BaF2(s) Ba2+(aq) + 2F– Ks


+ –
(aq) + 2F K2a

BaF2 + (aq) Ba2+


Chemical and Ionic Equilibria 7.13

Ks 1.0 ¥ 10-6 M 3
K= = = 2.37 M
K a2 (6.5 ¥ 10-4 M) 2

MULTIPLE CHOICE QUESTIONS ON THE ENTIRE UNIT

Equilibrium Constants Kp and Kc

3(s) CaO(s) + CO2 2


3
3
3(s) CaO(s) + CO2
2 to

Kc
not
Keq
direction
Keq
direction
Keq
Keq
direction
N 2O 4 2NO2 a 2O 4
Kp is
1/ 2 1/ 2
Kp p Kp Ê Kp p ˆ Ê Kp ˆ
(a) a = (b) a = (c) a = Á ˜ (d) a = Á ˜
4 + Kp p 4 + Kp Ë 4 + K p p¯ Ë 4 + Kp ¯

Calculations of Equilibrium Constant


2 2 2

2 2 2
will be
2.27
7.14 Complete Chemistry—JEE Main

1 1
2 2 2 2
2 2 will be
7.4
Kc SO2 2 SO3
3

Kc 2 2 2 2 are

–1
Kc

–1 –1 –1 –1

2 2

Kp
–1 –1 –1 –1

2 2
–2
2 2 Kp
¥ –2
¥ 2)
the Kp –5 atm3
2 4(s) 3 2
2 4

CaCO3(s) CaO(s) + CO2 the Kp


CaCO3 2

C2 6 C2 4 2 the Kp DG
pº = 1 atm)
–1 –1 –1 –1

–2 mol–1 L s–1 4 mol–1 L s–1

¥ 6 (b) 4 ¥ 4 ¥ ¥ –2

(c) 2 1 2
Chemical and Ionic Equilibria 7.15

(c) 2 2 12 2
2 2 2 2
and CO2 2

¥ –4
mol–1 dm3 s–1
¥ –5
mol–1 dm3 s–1

K°p 5 PCl3 2
K°p
(a) 8 (b) 16 (c) 32 (d) 64
2 2 DH –1
–1
–1 –1 –1

2O 4
N2O4 decomposed to NO2

Relation Between Kp and Kc


2 2 Kp and Kc
(a) Kp > Kc (b) Kp < Kc (c) Kp = Kc (d) Kp = K2c
2 4 2 C2 6 Kp and Kc

(a) Kp > Kc (b) Kp < Kc (c) Kp = Kc (d) cannot be predicted


2O 5 4NO2 2
(a) Kp > Kc (b) Kp < Kc (c) Kp = Kc (d) Kp = Kc
Kp and Kc
(a) Kp (RT)D = Kc (b) Kp = Kc (RT)D (c) Kp = Kc R (d) Kc = R Kp
Kc N 2O 4 2 NO2 –3 Kp
¥ –6 –1
–1
Kp 2SO2 2 2SO3 Kc
be
¥ 2
mol–1 cm3 ¥ 2 –1
¥ –3
¥ 2 –1

Kp < Kc
2 2 2 2 3
(c) 2SO3 2SO2 2 2 CO2 2
Le-Chatelier Principle
2 2 DH –1
not
7.16 Complete Chemistry—JEE Main

2 2 3 DH –1 not

3
3

2O(s) 2

2O(s) 2 2
N2 2 3 DH –1
3 is expected to
increase at

1
SO2 2
O2 SO3

3 3
3 3
1
SO2 2
O2 SO3

3
3
3

3 2
CaO(s) + CO2 CaCO3

3 2

N2 2 3 2
3 is added to the system
2 is added to the system
Chemical and Ionic Equilibria 7.17

N2 2 3 3

2
N2 2 3 2

(c) N2 3
N2 2 3 2 in the system at

(c) O2 3
2 2 DH
increased when

2 2 2SO3 DH 3

3 2SO2 2
3 is
(a) increased (b) decreased

Miscellaneous Problems

N 2O 4 2 NO2
D H –1

D G –1

2O 4

–1 –1 –1 –1

–1 –1 –1 –1

–1
mol–1 –1
mol–1 –1
mol–1 –1
mol–1
K°p

K°c pº = 1 atm)
¥ –2 ¥ –3

K°p p is
2a Ê p ˆ 4a Ê p ˆ
2
4a Ê p ˆ
2
2a Ê p ˆ
(a) K°p = (b) K°p = Á ˜ (c) K°p = (d) K°p =
1 - a ÁË p∞ ˜¯ 1 + a 2 Ë p∞ ¯ Á ˜
1 - a 2 Ë p∞ ¯ 1 + a ÁË p∞ ˜¯
7.18 Complete Chemistry—JEE Main

K°p p is
Ê 2a ˆ Ê pˆ Ê 4a ˆ Ê p ˆ 2
Ê 2a ˆ Ê pˆ Ê 4a 2 ˆ Ê p ˆ
(a) K°p = Á ÁË p∞ ˜¯ (b) K°p = Á Á ˜ (c) K°p = Á ÁË p∞ ˜¯ (d) K°p = Á Á ˜
Ë 1 - a ˜¯ Ë 1 - a 2 ˜¯ Ë p∞ ¯ Ë 1 + a ˜¯ Ë 1 - a ˜¯ Ë p∞ ¯
2O 4 2O 4

(a) remains constant (b) increases (c) decreases (d) cannot be predicted
2O 4 K°p
(a) remains constant (b) increases (c) decreases (d) cannot be predicted
2O 4 2O 4
(a) remains constant (b) increases (c) decreases (d) cannot be predicted
2O 4 2
2O 4 2O4 at 1 atm
(a) remains constant (b) increases (c) decreases (d) cannot be predicted
2O 4 2
2O 4 2O 4
(a) remains constant (b) increases (c) decreases (d) cannot be predicted
1
K °p 2 N 2O 4
2

2O 4 will be

2O 4

–1
–1 –1 –1 –1
–1

–1 –1 –1 –1

2O 4 2
N 2O 4 2

ANSWERS
Chemical and Ionic Equilibria 7.19

HINTS AND SOLUTIONS

Keq = p(CO2
N 2O 4 2 NO2
1–a 2a ntotal = 1 + a
2
1- a 2a pNO 2a ˆ 2 Ê 1 + a 1 ˆ 4a2
p N 2O 4= p and pNO2 = p Kp = 2
= ÊÁ p˜ Á ˜ = p
1+ a 1+ a pN 2 O 4 Ë1+ a ¯ Ë1- a p¯ 1- a2
1/ 2
Ê Kp p ˆ
a= Á ˜
Ë 4 + K p p¯
Knew = 1/Kold
Knew = (Kold)2 2

2 2 SO3
1 mol – x 1 mol – x 1 mol + x 1 mol + x
2
(1 mol + x) 2
Ê 1 mol + x ˆ
Kc = ÁË 1 mol - x ˜¯
(1 mol - x)2
or (1 mol + x) = 4 (1 mol – x) or x
x
2 2
1 mol – x 2 mol – x 2x
Since D Kc = Kp = Kn
(2 x)2 (2 x)2
Kn = that is
(1 mol - x) (2 mol - x) (1 mol - x) (2 mol - x)
x x x

1 mol – x x 2x
2x –1
x
2L
2
[C]2 (0.70 mol 2 L )
Kc = = –1
L
[A] [B]2 (0.65 mol 2 L ) (0.8 mol 2 L )2

2
p p 2p
ptotal p p) + 2p p
p ptotal
7.20 Complete Chemistry—JEE Main

( pNOCI )2 (2 ¥ 0.139 atm) 2 –1


Kp = =
( pNO )2 ( pCl2 ) (0.373 - 2 ¥ 0.139)2 (0.310 - 0.139) atm3
2 2
x x
nH 2S x
Since D Kp = Kn =
nH 2 0.2 mol - x
0.0127 ¥ 0.082 ¥ 363 ˆ
x x p = ÊÁ ˜¯ atm
Ë 1

2 4(s) 3 2
x 2p p
Kp = p2 3
pCO2 –5 atm3 = (2p)2 p
1/ 3
Ê 2.9 ¥ 10-5 atm ˆ
p = Á ˜¯ ptotal = 2p + p = 3p = 3
Ë 4
Kp = pCO 2
DG° = – RT ln Kºp –1
mol–1 –1

a –x a – 2x

kf 1.62 ¥ 10-2 mol-1 L s -1


Keq = =
kb 4.0 ¥ 104 mol-1 L s -1
Keq = p pB
p pB p pB
2
1 [C] 1 = [B]2 [C]22
[C]1
2 = 2[B]1 2 =
2
1 [C]31 = [B]2 [C]32
[C]1 [C]1
2 = 8[B]1 2 = 1/3
=
2
8 2
1 [C]31 = [B]22 [C]32
[B]1
2 = 2 [C1 2 =
8
2 2 2
x x x x
2
x
x
(0.4 mol - x) 2
Keq = k /kb ¥ -4 mol–1 dm3 s–1 ¥ -5 mol–1 dm3 s–1
Kp
Ea(b) -1

2 2

Since Dn Kp = Kn 2 2

2O 4 2NO2
¥ ntotal
p
Kp = Kc (RT)D D Kp = Kc
Chemical and Ionic Equilibria 7.21

D Kp = Kc/RT RT –1 mol–1 –1

Kp < Kc .
D Kp = Kc (RT)3 Kp > Kc RT
Kp = Kc (RT)D D =+1
–3 mol L–1 –1 mol–1
Kp = Kc (RT)D D =–1
–1 –1 –1 2
Kc = Kp RT mol–1 L
DH

D
DV 2O(s) 2

D r H° = 2D H°(NO2) – D H°(N2O4) = (2 ¥ - –1 –1

D r G° = 2D G°(NO2) – D G°(N2O4) = (2 ¥ - –1 –1

D r S° = (D r H°– Dr G°)/T –1 –1
mol–1 –1
mol–1
K°p = exp(– D G°/RT) = ¥ 3 J mol–1 –1
mol–1
Kc° = K°p (c°RT/p°)–Dn Dn = + 1
–3 3 –1
mol–1 –1

¥ –3

2O 4 2NO2
n(1–a) n(2a n(1 + a)

Ê n (1 - a ) ˆ Ê n (2a ) ˆ
p N 2O 4 = Á p pNO2 = Á p
Ë n (1 + a ) ˜¯ total Ë n (1 + a ) ˜¯ total
( pNO /p∞)2 2
Ê 2a ˆ Ê 1 + a p∞ ˆ 4a 2 ptotal
ptotal / p∞˜ Á
2
K°p = = Á =
( pN 2O4 /p∞) Ë1 + a ¯ Ë 1 - a ptotal ˜¯ 1 - a 2 p∞
Ê 4a 2 ˆ Ê (1 + a ) pinitial ˆ 4a 2 Ê pinitial ˆ
peq = (1 + a) pinitial K°p = Á ÁË ˜¯ =
Ë 1 - a 2 ˜¯ p∞ 1 - a ÁË p∞ ˜¯
Ê 4a 2 ˆ Ê pinitial ˆ 4a 2
K°p = Á Á ˜ fi a a
Ë 1 - a ˜¯ Ë p° ¯ 1-a
ptotal = (1 + a) pinitial
K°p
K°p

2O 4
7.22 Complete Chemistry—JEE Main

K°p (new) = 1/ K ºp (old) = 1/ 0.129


ptotal = (1 + a) pinitial
pinitial

E = D r H + Ea(b) –1 –1

–1

2O 4 2NO2
1 mol + x 1 mol – 2x x

1 mol + x Ê 1 mol - 2 x ˆ
p N 2O 4 = pNO2 = Á (1 atm)
2 mol - x Ë 2 mol - x ˜¯
2
pNO 2 (1 mol - 2x)2
Kp = =
pN 2 O 4 (2 mol - x) (1 mol + x)
x x

MULTIPLE CHOICE QUESTIONS FROM AIEEE AND JEE MAIN

N 2O 4 2NO2 2O 4 2
¥ –2
¥ –2
mol L–1 Kc
(a) 3 ¥ 3
¥ 2
¥ –1
¥ –3
[2003]

2SO2 2 2SO3 DH –1

[2003]
4(s) + 5O2 P 4O
[P4 O10 ] [P4 O10 ] 1
(a) K c = 5 (b) K c = (c) Kc = [O2]5 (d) K c = [2004]
[P4 ][O 2 ] 5[P4 ][O 2 ] [O 2 ]5

2 COCl2 Kp/Kc
(a) 1/RT (b) RT (c) RT [2004]
2 2 T is 4 ¥ –4

1 1
Kc 2 N2 2 O2
¥ 2 ¥ –4 [2004]
Keq

(c) exothermic
(d) endothermic [2005]
Chemical and Ionic Equilibria 7.23

2 2 Kp and Kc
–1 –1
that R
(a) Kp = Kc
Kp Kc
(c) Kp Kc
(d) Kp is less than Kc [2005]
4
3 2

4
2 2 2 2 [2005]

PCl5 PCl3 2
P 5 is x
3 will be
x ˆ
(a) ÊÁ
x ˆ 2x ˆ x ˆ
(b) ÊÁ (c) ÊÁ (d) ÊÁ
Ë 1 - x ˜¯ Ë x + 1˜¯ Ë 1 - x ˜¯ Ë x - 1˜¯
P P P P [2006]

1
3 SO2 2 O2 K°c ¥ –2

K°c the reaction 2SO2 2 2SO3 will be


¥ –2 ¥ –3 ¥ –2 [2006]
Kp1 and Kp2 2Y and Z

[2008]

2 CO2 2 K1
4 2 2 K2
4 2 CO2 2 K3

(a) K21 = K3K32 (b) K3 = K1 K 2 (c) K1 = K2K3 (d) K3 = K1K2 [2008]


2 2
K is

[2011, Cancelled]
Kc 2 2 T is 4 ¥ –4
Kc (1/2)N2 2
¥ 2
(c) 4 ¥ –4
[2012]
1
2 + 2 O2 SO3 Kp = Kc(RT)x
x
(a) 1 (b) –1 (c) –1/2 (d) 1/2 [2014]
2 4(s) 3 2
Kp ¥ –5 atm3
¥ –2
¥ –2
¥ –2
¥ –2
atm
[2014, online]
2 and 2
[2014, online]
7.24 Complete Chemistry—JEE Main

[2014, online]
2O 4 2 2O 4 2NO2
2O 4 2O 4
[2015, online]

[2015, online]
–1

–1 mol–1
R
Q > Kc Q > Kc
Q < Kc Q < Kc [2015]

–1

[2016]

T Kp
[2016 online]

ANSWERS

HINTS AND SOLUTIONS


[ NO 2 ]2 (1.2 ¥ 10-2 M) 2
Kc = = = 3.0 ¥ 10-3 M
[ N 2 O 4 ] (4.8 ¥ 10-2 M)

4(s) + 5O2 P 4O Kc = 1/[O2]5 as only

2 COCl2 Dn Kp = Kc (RT)Dn = Kc/RT or Kp /Kc = 1/RT


2 2 Kc = 4 ¥ –4
1 1
(1/2)N2 2 K¢c = = = 50
Kc 4 ¥ 10-4


d ln K eq DH ∞ DH ∞
= which on integeration gives ∞ =-
ln K eq + constant
dT RT 2 RT
K°eq and 1/T
DH/R DH
Dn
Kp and Kc is Kp = Kc(RT) where Dn

Dn
Chemical and Ionic Equilibria 7.25

(RT) Dn –1 mol–1 –1

Since Kp = Kc –1 Kp > KC
4 3 2
t
teq p p
peq p) + p p
2
Kp
5 PCl3 + Cl2
n(1 – x) nx nx ntotal = n(1 + x)
Ï nx ¸ x
pPCl = Ì ˝P =
3 Ó n(1 + x ) ˛
P
1+ x
1
2 2 2SO3 3 SO3 2 O2

1 1
K c∞ = = = 416

( Kc ) 2
( 4 .9 ¥ 10-2 ) 2

2Y
p (1 – a) p (2a) peq1 = p (1 – a) + p (2a) = p (1 + a)
Ê pY2 ˆ
{ pX (2a )}2 4 pXa 2
K p1 = Á = =
Ë pX ˜¯ pX (1 - a ) 1- a
Z P + Q
pZ(1 – a) p Za p Za peq2 = pZ(1 – a) + pZa + pZa = pZ(1 + a)
Ê pP pQ ˆ ( pZa )( pZa ) pZa 2 K p1 4 pXa 2 /(1 - a ) 4 pX
K p2 = Á = = ; Now = =
Ë pZ ˜¯ pZ (1 - a ) 1- a K p2 pZa /(1 - a )
2
pZ
pX 1 peq1 pX 1
Kp1/Kp2 = ; Also = =
pZ 36 peq2 pZ 36

DG(iii) = DG(i) + DG(ii) or –RT ln K3° = –RT ln K1° – RT ln K2°


This leads to K3 = K1 K2
2
p) 2p
p) + 2p p
p p
pCO2 pCO = 2 ¥
2 2
pCO (0.6 atm)
Kp = = = 1.8 atm
pCO2 0.2 atm

[ NO]2
N2 2 K c1 =
[ N 2 ][O 2 ]
[ N ]1 / 2 [O 2 ]1 / 2
(1/2)N2 2 K c2 = 2
[ NO]
1 1 1
Kc 2 = = = = 50.0
K c1 4 ¥ 10-4 2 ¥ 10-2
x
x = S Pn P - S Rn R

x = 1 - ÊÁ1 + ˆ˜ = -
1 1
Ë 2¯ 2
7.26 Complete Chemistry—JEE Main

2 4(s) 3 2
2p 2p
Kp = (p )2 (pCO2) = (2p)2 (p) = 4p3
3
1/ 3 1/ 3
Ê Kp ˆ Ê 2.9 ¥ 10-5 atm3 ˆ
p= Á
Ë 4 ¯˜ =Á
Ë 4 ˜¯ = 0.01935 atm
ptotal = p + pCO = 2p + p = 3p = 3 ¥ ¥ –2 atm
3 2

Æ 2 2 Dn Kp = Kc = Kn

Æ 2 2
t n
teq n –n / 2 n /4 n /4

(nH )(nI ) (n0 / 4)(n0 / 4)


kn = 2
2
2
=
(nHI ) (n0 / 2) 2

N 2O 4 2NO2
n (1 – a) n (2a)
n = n (1 – a) + n (2a) = n (1 + a

mass of 1 mol of N 2 O 4 (92 g mol-1 )


r= =
V (1.2 mol) RT /p
(92 g mol-1 ) p (92 g mol-1 ) (101.325 kPa )
= = –3
(1.2 mol) (RT ) (1.2 mol) (8.314 kPa dm3 K -1 mol-1 ) (300 K )

DrGº = –RT ln Kºc


ΔrGº (2492.2 J mol−1 )
ln Kºc = – =− = −1 Kºc = e–1 –1
RT (8.314 J K −1mol−1 )(300 K )
[B][C] (2 mol)(1 mol / 2)
Now Qc = 4
[A]2 (1 mol / 2)2
Since Qc > Kºc
Keq
x x x x
[C][D] (1 M + x)2
Keq = =
[A][B] (1 M - x)2
1M + x
x x)
1M - x
or 11x x
eq x
p +pY
Since p = pY p = pY Kp = p pY = (5 bar)2 = 25 bar2
Chemical and Ionic Equilibria 7.27

UNIT 2 Ionic Equilibrium

SECTION 1 Concepts of Acids and Bases

Arrhenius Concept

+

+ –
+ –

Brönsted-Lowry Concept
+
+
+

accepts proton

2O Æ Cl– 3O
+

acid1 base2 1 2

loses proton

2O æÆ Cl– 3O
+

acid1 1 2

+
3O
3 O+

2

as compared to Cl

2
+
3O
7.28 Complete Chemistry—JEE Main

Table 1 Common Conjugate Acid-Base Pairs


Acid Conjugate base
Name Formula Formula Name
Perchloric acid 4 ClO–4 Perchlorate ion

2SO4 4

Br– Bromide ion


Cl– Chloride ion
Nitric acid NO–3 Nitrate ion
æÆ

3
+
3O 2O
– –
4 SO42
Phosphoric acid –
3PO4 2PO 4

2 NO 2 Nitrite ion

3COO

æÆ
3

Cabonic acid 2CO3 3 Bicarbonate ion

2S
+
4 3
CN– Cyanide ion

2O

3 2

2O æÆ Cl– 3O
+

3 2O NH +4 –

base1 acid2 acid1 stron 2

General Criterion on the Extent of a Reaction

Levelling Effect
+
3O
3 O+

4 2O æÆ 3O + ClO–4
2O æÆ 3O
+ + Cl–
2O æÆ 3O
+ + Br–
+
3O

+ –
2 2O 3O

3PO4 2O 3O
+ + H 2 PO 4-
2 2O 3O
+
+ NO -2
Chemical and Ionic Equilibria 7.29




2O Æ 2

NH -2 2O Æ 3

CN– 2O

– –
2O 2

Phenomenon of Hydrolysis + –
3O

+ –
(i) Complete Hydrolysis 3O
2
(a) PH +4 2O æÆ 3O
+
3
acid1 base2 acid2 base1

PH +4 3O
+
3 2

2O æÆ 2

base1 acid2 acid1 base2

+ –
2 2

+ –
(ii) No hydrolysis 3O

2 3 O+
(a) Na+ 2O ¨æ 3O
+

acid1 base2 base1 acid2

Na+ 3O
+
2

(b) Cl– 2O ¨æ –

base1 acid2 acid1 base2

Cl– –
2 3O
+
+ –
(iii) Hydrolysis to a Limited Extent 3O
) and
– +
2 3O ) exhibits hydrolysis to a limited

(a) NH +4 2O 4 3O
+

acid1 base2 base1 acid2

NH +4 3O
+
and 4 2

– –
3COO 2O 3
base1 acid2 acid1 base2

– –
3COO 3 2
+
3O

Lux-Flood Concept

CaO + SiO2 Æ CaSiO3


7.30 Complete Chemistry—JEE Main

Lewis Concept

3 3 Æ 3 ÆBF3
base acid

MULTIPLE CHOICE QUESTIONS ON SECTION 1

HSO 4- + CN -  SO 42- + HCN


(a) HSO -4 SO 2-
4 (b) HSO -4 –
SO 2-
4 (d) CN–
– ClO - ClO - and ClO -
2 3 4
-
(a) ClO– (b) ClO -2 (c) ClO3 (d) ClO -4

(a) NH +4 > HSO -4 > H3O + (b) NH +4 > H3O + > HSO 4-
(c) H3O + > HSO -4 > NH +4 (d) H3O + > NH +4 > HSO -4

(a) NO -2 > CH3COO - > HCO3- (b) NO -2 > HCO3- > CH3COO -
- - -
(c) HCO3- > NO 2- > CH3COO - (d) HCO3 > CH3COO > NO 2

(a) NH -2 > OH - > CN - (b) NH -2 > CN - > OH -


(c) CN - > NH 2- > OH - (d) CN - > OH - > NH -
2

2SO4 4 3PO4

(a) NH -2 – (c) NO -2 3

+
3O 2O
+
3O 2O
+
3O 2O
+
3O 2O
+ –
3O 2

– + CN–
2O
3PO4 + CN– H 2 PO 4-

2 3PO4 2 3PO4
3PO4 2 3PO4 2

- -
(a) HN3 (b) N 2 (c) N3- (d) N3–
Chemical and Ionic Equilibria 7.31

not
(a) BF3 3 3 (d) FeCl3

2+
(a) BF3 4 3 5 5 2O (c) BF3 3 2O (d) BF3 3 2


(b) CN– –
(d) NO -2

ANSWERS

HINTS AND SOLUTIONS


HSO -4

3O
+
HSO -4 + as compared
to NH +4
2 3 2CO3
HCO3- > CH3COO - > NO 2-
2 3
NH -2 > OH - > CN -
+
3O

2O æÆ 3O
+

+
2O 3O

2 3PO4
3PO4 2
3 N3-
3
2+
3 and SnCl4

7.32 Complete Chemistry—JEE Main

SECTION 2 The pH Scale and pH of Acid and Base Solutions

The pH Scale
+ –14
3O

+
3O ] /mol dm–3}
+ –3
Note: 3O ] by mol dm

Equilibrium Constant of Water


+ –
2 2O 3O (1)

[H3O + ][OH - ]
Kc = (2)
[H 2 O]2

3O
+ – ¥
–1

n m/M (1000 g / 18 g mol-1 ) –1


2O] = = =
V V (1 L)
Kc(water) is
(10-7 M )(10-7 M )
Kc = ¥ –18
(55.56 M )2
Ionization Constant of Water
+ –
2O
+ +
3O
+ -
[H ][OH ]
Ki = Kc 2O] = (3)
[ H 2 O]
(10-7 M )2
Ki = ¥ –16
(55.56 M )

Ionic Product of Water 2


+ –
Kw = Ki 2 3O ] (4)
Kw is Kw ¥ ¥ ¥ –14 2

Kw [H3O + ] [OH - ] –3
= )
M2 M M

ÊK ˆ Ê H O+ ˆ Ê OH - ˆ
log Á w2 ˜ = log Á 3 2 ˜ + log Á
ËM ¯ Ë M ¯ Ë M ˜¯

pKºw
Chemical and Ionic Equilibria 7.33

ÊK ˆ Ê [OH - ] ˆ
pKºw = - log Á w2 ˜ - log Á (6)
ËM ¯ Ë M ˜¯

Nature of Aqueous Solution


+ –
3O
+ –
3O
K w)

+ –
3O
+ –
3O
Kw
1 1
[H3O + ] = K w 2 pK°w or [OH - ] = K w 2 pK°w

1 1
[H3O + ] > K w 2 pK°w or [OH - ] < K w 2 pK°w

1 1
[H3O + ] < K w 2 pK°w or [OH - ] > K w 2 pK°w

+
3O
+ –
3O
+ –
3O

Variation of the pH of Water with Temperature

Table 1
qC 25 ºC
Kw ¥ –15 2 ¥ –15 2 ¥ –14 2 ¥ –14 ¥ –14 2

pKºw

(pKºw/2)

pH of Solution of a Monoprotic strong Acid


+ +
3O 3O
£ –6
3O
+
+] +] +]
total acid water
+] –] c
water water
+] =c –] = c + Kw
total water total
+] 2 –c +] – Kw
total total
7.34 Complete Chemistry—JEE Main

+] c + c2 + 4K w
total = (8)
2
Illustration –6

+] = 10-6 M + (10-6 M ) 2 + 4 (10-14 M 2 ) = 10-6 M + 3.22 ¥ 10-6 M –6


2 2
+ –6

Note:
pH of a Solution of a Monoprotic Weak Acid

+ –

c a
+ –
[H + ][A - ]
Ka =
[HA]
c (1– a) ca ca

(c a )(c a ) ca 2
Ka = =
c(1 - a ) 1- a
or ca2 + Kaa – Ka a = (–Ka + K a2 + 4 K a c) ) / 2c (11)
a 3O
+]
(= ca
Ka a

ca 2
Ka =  ca 2 or a = Ka / c (12)
1- a
Ostwald Dilution Law cÆ a

–5
Illustration Ka
1/ 2
Ka Ê 1.8 ¥ 10-5 M ˆ
a= = ÁË
–2
c 0.1 M ˜¯
+
]=ca –2 –3 +

pH of a Solution of a Strong or Weak Base


– –2
–]/mol dm–3 –2

–] –] –] –] +] –] + Kw –]
total base water base 3O water base total
–] 2 –] –]
total base total – Kw

–]
[OH - ]base + [OH - ]base
2
+ 4K w 10-7 M + (10-7 M )2 + 4(10-14 M 2 )
total= =
2 2
10-7 M + 2.24 ¥ 10-7 M
=
2
Chemical and Ionic Equilibria 7.35


¥ and
–3
4 Kb ¥ –5

NH 4 OH  NH +4 + OH - [ NH +4 ][OH - ]
Kb =
c (1- a ) ca ca [ NH 4 OH]
(c a )(c a ) - K b + K b2 + 4 K b c
Kb = or ca2 + Kba – Kb a=
c(1 - a ) 2c
(-1.8 ¥ 10-5 M ) + [(1.8 ¥ 10-5 M )2 + 4(1.8 ¥ 10-5 M )(10-3 M )]1 / 2
a=
2(10-3 M )

-1.8 ¥ 10-5 M + 2.69 ¥ 10-4 M


=
2(10-3 M )
–] = ca ¥ –3 ¥ –3

pH of a Solution of Diprotic Acid


2

– + [HA - ][H3O + ]
2 2O 3O Ka1 = (13)
[H 2 A]
– 2– +
[A 2- ][H3O + ]
2O 3O Ka2 = (14)
[HA - ]
Ka1>>Ka2
+
3O

Computation of [H3O+] 3O
+

+ O +] O +] +]
3O ]total 3O
+ –
3 2 3 water
+
3O

Since Ka1>>Ka2

– <<
+] +]
3O 3O 2

+ +
3O ] 3O ] 2
–] +
3O
+ 2
[H3O ]
Ka1 = (15)
[H 2 A]0 - [H3O + ]
+] 2 + Ka1 +] – Ka1
3O 3O 2

3 O+
Ka1
+
3O 2
+] K a1[H 2 A]0
3O =
Computation of [HA–] – +]
3O (18)
Computation of [A2–]
[A 2- ][H3O + ] 2–
Ka2 = -
[HA ]
7.36 Complete Chemistry—JEE Main

Illustration Ka1 2C 2O 4 ¥ –2
K a 2 (HC2 O -4 ) = 6.4 ¥ 10-5 M
From
+] - K + K 2
+ 4 K [ H C O ] -0.059 M + [(0.059 M ) 2 + 4(0.059 M )(0.1 M )]1 / 2
= a1 a1 a1 2 2 4 0 =
2 2
-0.059 M + 0.0165 M
=
2
[HC2 O -4 ] + [C2 O 2-
4 ] = Ka2 ¥ –5

+
2C 2O 4 2C 2O 4]

pH of Sulphuric Acid Solution

2SO4 Æ + –
4 Ka1

– [H +][SO 24- ]

+
+ SO2– Ka2 =
[HSO 4- ]
4

Illustration 2SO4 Ka2 ¥ –2

H 2SO 4 Æ H + + HSO 4- and HSO -4  H+ + SO 24-


0.2 M 0.2 M 0.2M - x 0.2 M + x x

[H + ][SO 24- ] (0.2M + x)( x)


Ka2 = =
[HSO -4 ] (0.2 M - x)
x x
+

[HSO -4 ] 2–
4

2SO4

pH of a Mixture of Two Weak Acids


+ –
[H + ][A - ] ( x + y )( x)
K 1= =
[HA] c1 - x
c1 – x x+y x

+ [H + ][B- ] ( x + y )( y )
+ B– K 2= =
[HB] (c2 - y )
c2 – y x+y y

x  c1 and y  c2

( x + y )( x) K1c1 ( x + y )( y ) K 2 c2
K1 = or x= and K2 = or y=
c1 ( x + y) c2 ( x + y)

x and y
K1c1 K 2 c2
x+y= + or (x + y)2 = K1c1 + K2c2
( x + y) ( x + y)

+
]=x+y= K1c1 + K 2 c2
Chemical and Ionic Equilibria 7.37

+ 1 2
K 1c 1 + K 2c 2 }
2
[A - ] x K1c1 /( x + y ) Kc
The ratio - = = = 11 (21)
[B ] y K 2 c2 /( x + y ) K 2 c2

Illustration K1 ¥ –4
2 (K2 ¥ –4

3O
+
] = (K1c1 + K2c2)1/2 ¥ –4
¥ –4 1/2

¥ –4 2)1/2

MULTIPLE CHOICE QUESTIONS ON SECTION 2

+ –
Kc 2 2O 3O at 25 °C
¥ –16
¥ ¥ –18
¥ –18

¥ –15
¥ –16
¥ –14
¥ –16

Kºw

¥ –14 2

2SO4
3 2SO4

2SO4

Kºa

Ka1 ¥ –2 Ka2 ¥ –5

¥ –2 ¥ –2 ¥ –4 ¥ –5
+
2SO4 3O at 25 ºC will
7.38 Complete Chemistry—JEE Main

c1 K1 c2
–]/[B–
K2
(a) K1c1/K2c2 (b) K1c1 / K 2 c2 (c) K2c2/K1c1 (d) K1c2 / K 2 c1
Kºi

Ka ¥ –4 Ka ¥ –4

2SO4
(b) pKa

ANSWERS

HINTS AND SOLUTIONS


2
[H3O + ][OH - ] È (10-7 M ) ˘
Kc = = Í ˙ ¥ –18
[H 2 O]2 Î (55.56 M ) ˚
-7
[H3O + ][OH - ] (10 M )
2
Ki = = ¥ –16
[ H 2 O] 55.56 M
Kºa Ka

+
3O will also increase and

+
3 3O

+
3O
(20 mL)(0.1 M ) + (30 mL)(0.2 M ) 8
3O
+] = = M
(20 mL + 30 mL) 50
a = Ka / c Ka
Ka Kºa
2–] = Ka2
2
– Ka +
4 3O

+ – [H + ][A - ] +
[H + ][B- ]
K1 = + B– K2 =
[HA] [HB]
Chemical and Ionic Equilibria 7.39

È A - ˘ K1[HA]/[H + ] K1c1
Í -˙= +
=
Î B ˚ K 2 [HB]/[H ] K 2 c2
Ki 3O
+ –
2 ¥ –16

K°i Ki ¥ –16
)
3 O +
] = (K c
1 1 + K c
2 2 ) 1/2
¥ –4
¥ –4 1/2
¥ –2

¥ –2
+
2SO4 3O 2SO4
wise manner and HSO -4
Ka and their pKºa
+
3O

SECTION 3 Hydrolysis of Salts

– +
[CH3COO - ][H3O + ]
3 2O 3COO 3O Ka = (22)
[CH3COOH ]

– –
[CH3COOH ][OH - ]
3COO 2O Kb = (23)
[CH3COO - ]
3

[CH3COO - ][H3O + ] [CH3COOH ][OH - ] + –]=K


K aK b = (24)
[CH3COO - ]
w
[CH3COOH ]

Ka Kb

Kh Ka or Kb

+
Salt Formed from a Strong Acid and Strong Base Cl– +
and Cl–

Kºw

Salt Formed from a Weak Acid and Strong Base 3COO Na+

– –
3COO 2O 3 (25)

[CH3COOH ][OH - ]
Kb = (26)
[CH3COO - ]
7.40 Complete Chemistry—JEE Main

[H + ][OH - ] K
Kb = = w
([CH3COO - ][H + ]/[CH3COOH ]) Ka

– – –] –] –]
3 3COO 3COO 3COO

[OH - ]2
Kb = –] = Kbc (28)
[CH3COO - ]0
where c –
3COO 3

Kw Kw Kw K w Ka
+] = -
= = 1/ 2
=
[OH ] Kbc [( K w / K a )c] c

1È º Ê c ˆ˘
ÍpK w + pK aº + log Á -3 ˜ ˙
2Î Ë mol dm ¯ ˚

Illustration Ka ¥ –5

-5
pKºa = - log {K a / M} = - log (1.8 ¥ 10 ) = 4.74
1 1
[14 + 4.74 + log (0.1)] = (17.74) = 8.87
2 2
Salt formed from a Strong Acid and Weak Base (NH +4 Cl- )

NH +4 + H 2 O  NH 4 OH + H + (31)
+

[ NH 4 OH][H + ] [H + ][OH - ] Kw
Ka = ∫ = (32)
[ NH 4+ ] + -
[ NH 4 ][OH ]/[ NH 4 OH] Kb

4
+]
and [ NH +4 ] = [ NH +4 ]0 - [H + ]  [ NH +4 ]0

[ NH 4 OH][H + ] [H + ]2
Ka =  +
]= Ka c (33)
[ NH +4 ] [ NH 4+ ]0
where c [ NH +4 ] 4
1/ 2
ÈÊ K ˆ ˘ 1 ÈpK ∞ - pK ∞- log Ê c ˆ ˘
+
] = ÍÁ w ˜ c ˙ or ÍÎ w b ÁË ˜¯ ˙ (34)
ÎË K b ¯ ˚ 2 M ˚

–5
Illustration Kb 4 ¥
pKºb Kb ¥ –5

1 10.26
[14 - 4.74 - log (0.1)] = = 5.13
2 2
Salts Formed from Weak Acid and Weak Base

+ +
4 2O 4 (35)
– –
2O (36)
Chemical and Ionic Equilibria 7.41

+ –
4
+ – – +
4 4

4 Ka(acid) π Kb
+ –
4 2O 4
The hydrolysis constant is
[ NH 4 OH][HAc] [H + ][OH - ] Kw
Kh = = + - - +
=
[ NH +4 ] [Ac- ] Ê [ NH 4 ][OH ] ˆ Ê [Ac ][H ] ˆ K b K a
ÁË [ NH OH] ˜¯ ÁË [HAc] ˜¯
4
Expression of Degree of Hydrolysis
+ –
4 2O 4
c(1 – a) c(1 – a) ca ca

[ NH 4 OH][HAc] (ca )(ca ) a2


Kh = = =
[ NH +4 ][Ac- ] {c (1 - a )}{c (1 - a )} (1 - a )2

Kh
a= (38)
1 + Kh
+] –]
Expression of pH of Solution 4 4 we write

[ NH 4 OH][HAc] [HAc]2
Kh = 
[ NH 4+ ][Ac- ] [Ac- ]2

[H + ][Ac- ] [HAc] [H + ]
Ka = fi =
[HAc] [Ac- ] Ka
Kh becomes
2
Ê [H + ]ˆ
Kh = Á (41)
Ë K a ˜¯
1/ 2 1/ 2
Ê K ˆ ÊK K ˆ
+] = Ka K h = Ka Á w ˜ =Á a w˜ (42)
Ë Ka K b ¯ Ë Kb ¯

1
[pKºw + pKºa – pKºb] (43)
2

Ka > Kb then pKºa < pKºb


Ka < Kb then pKºa > pKºb
Ka = Kb then pKºa = pKºb

Illustration 4 Ka ¥
Kb 4 ¥ –5

Kw (1.0 ¥ 10-14 M 2 )
Kh = = = 1.389
K a K b (4.0 ¥ 10-10 M )(1.8 ¥ 10-5 M )
7.42 Complete Chemistry—JEE Main

Kh 1.389
a= =
1 + Kh 1 + 1.389
pKºa ¥ Kºb ¥ –5

1 1
[pKºw + pKºa – pKºb] =
2 2

MULTIPLE CHOICE QUESTIONS ON SECTION 3

¥ –5 Kb
¥ –8 ¥ ¥ ¥

(a) a = K w K a / c (b) a = K w / K a c (c) a = K w c / K a (d) a = K a c / K w


where Ka
¥ –4

Ka 3 ¥ –5

Ka(C6 5 ¥ –5

Ka ¥ –4

Ka ¥ –4

Ka1 2CO3 ¥ Ka –
3 ¥ –11
2CO3
be

(a) Na2CO3 4 3 4CN


Kb 4
= Ka ¥ –5

¥ –2 ¥ –3 ¥ –4

3
1 1
K°a1 + K°a2 2 (pK°a1 + pK°a2 K°a1 – K°a2 2 (pK°a1 – K°a2)

¥ ¥ –8
¥ ¥

4 Ka 2+
¥ –8

3 2CO3 4 3 4Cl 2CO3


2CO3 3 4 4 2 CO 3 3
Chemical and Ionic Equilibria 7.43

ANSWERS

HINTS AND SOLUTIONS


Kb = Kw /Ka ¥ –14 2
¥ –5
¥
– –
2O
c(1 – a) ca ca
-
[HAc][OH ] (ca )(ca ) ca 2
Kb = = =
[Ac- ] c(1 - a ) 1 - a
a Kb = ca2 or a = K b / c Kb = Kw /Ka where Ka
a = K w / Ka c
– –
2O
c–x x x
[HF][OH - ] x2 x2
Kb = =  x c)
[ F- ] c-x c
–]
=x= K b c = ( K w / K a )c ¥ –14 2
¥ –4 1/2
¥ –6

¥ –6

Ka K°a
1
2 [pK°w + pK°a c
2– – –
3 2O 3
1 1 1
2 [pK°w + pK°a

3 c 2 [14 ¥ –11
2 [14

Kw (1.0 ¥ 10-14 M 2 ) (1.0 ¥ 10-7 M )


Kh = = Kh = = 5.56 ¥ 10-3
K a K b (1.8 ¥ 10-5 M 2 ) 2 (1.8 ¥ 10-5 M )

Kh 5.56 ¥ 10-3
a= = -3
= 5.52 ¥ 10-3
1 + Kh 1 + 5.56 ¥ 10
+ –
3

[H3O + ][CO32- ]
– K a2 =
3 2O 3 O+ + CO32– [HCO3- ]

[H 2 CO3 ][OH - ]

2O 2CO3
– Kb =
3
[HCO3- ]
[OH - ][H + ] K
Kb = - +
= w
[HCO3 ][H ]/[H 2 CO3 ] K a1
7.44 Complete Chemistry—JEE Main

Ka2
K a2 [HCO3- ] K a2 {[H 2 CO3- ][OH - ]/ K b } K a2 [H 2 CO3 ] K w
+
3O ] =
= =
[CO32- ] [CO32- ] [CO32- ] ( K w / K a1 ) [H3O + ]
[H 2 CO3 ]
+ 2
3O ] = K a1 K a2
[CO32- ]
Since both Ka2 and Kb 2CO3] [CO23–
+] 2 1
3O = Ka1Ka2 2 (pK°a1+ pK°a2)
+ –3 +] = {K b (A - )}c or Kb – +]2/c
– –3 2 –5
that is Kb
– –14 2 –5
Finally Ka Kw / Kb

Kh a
a= Kh =
1 + Kh 1- a
a ˆ 2 Ê 0.5 ˆ 2
Kh = ÊÁ =Á = 1.0
Ë 1 - a ˜¯ Ë 1 - 0.5 ˜¯
2+ + +
2O
[Cu(OH)+ ][H + ] [H + ]2 [H + ]2
Kh = = 
[Cu 2+ ] [Cu 2+ ]0 - [H + ] [Cu 2+ ]0
+] = (Kh 2+] )1/2 ¥ –8 1/2 ¥ –5

¥ –5
+
4 4
NaNO3 2–
2CO3 3

SECTION 4 Buffer Solutions

Common-Ion Effect

c c1

+ –
2O 3O (44)
co – x’ x’ x’

+ –
2O 3O
c –x x c1 + x
+ -
[H3O ][Ac ] (c1 + x)( x)
Kt = = (45)
[HAc] c0 - x
2 + x (c + K ) – K c
x 1 a a o

x +
3O

c c1
Chemical and Ionic Equilibria 7.45

(c1 )( x) Êc ˆ [acid]
Ka = or x = Ka Á 0 ˜ 3O
+] = Ka
c0 Ë c1 ¯ [salt]

+ Ê [acid] ˆ Ê [salt] ˆ
log Á K°a + log Á
Ë [salt] ˜¯ Ë [acid] ˜¯
3O Ka

4 4

Ê [salt] ˆ
pK°b + log Á
Ë [base] ˜¯
(48)

Illustration Ka ¥ –5

+ –
-
+
[H ][Ac ] [H + ][Ac- ] [H + ]2
Ka = =  or [H + ] = K a [HAc]0
[HAc] [HAc]0 - [H + ] [HAc]0
+
¥ –5 1/2
¥ –3 +
¥ –3

Ê [salt] ˆ Ê 0.05 ˆ
pK°a + log Á ¥ –5
Ë [acid] ˜¯ ÁË ˜
0.10 ¯

Acidic Buffer Solution


æÆ 2

(acid) (added) (salt)

– +
æÆ
(salt) (added) (acid)

– – + –

Illustration 3
3
Ê [salt] ˆ Ê 0.05 M - 0.01 M ˆ
K°a + log Á log Á
Ë [acid] ˜¯ Ë 0.10 M + 0.01 M ˜¯
Basic Buffer Solution

4
+ æÆ 2
+
4
(base) (salt)

+
4
+ æÆ 4
(salt) (base)

Buffer capacity
7.46 Complete Chemistry—JEE Main

[salt] + ∂cacid/∂ ∂cbase/∂


∂(salt)/∂

a = [salt] + [acid] and b = [salt]


Ê [salt] ˆ b ˆ
ln ÊÁ
1
Then K°a + log Á ˜ = pK°a + ˜
Ë [acid] ¯ 2.303 Ë a - b ¯
∂(pH) ∂b b( a - b) ˆ ∂[salt] Ê [salt][acid] ˆ
= 2.303 ÊÁ
1 a
= or ˜ i.e. = 2.303 Á
∂b 2.303 b(a - b) ∂(pH) Ë a ¯ ∂(pH) Ë [salt] + [acid] ˜¯

∂b/∂ b/a – b) = 1 or b = a

Buffer Range

pKa – 1 to pKa + 1 (or pKb – 1 to pKb

MULTIPLE CHOICE QUESTIONS ON SECTION 4

Ka –
2PO4 ¥ –8
2PO4 and Na2 4

Kb ¥ –4
3) 2 3) 2 2

Ka ¥ –5
–1

3PO4 2 4
2– is
4
(a) 1 ¥ ¥ ¥ –11 ¥ –12

a and b

b(a – b)/a a(a – b)/b ab/(a + b a + b)/ab


4 Kb ¥ –5
4
25° C is
Chemical and Ionic Equilibria 7.47

K°a

ANSWERS

HINTS AND SOLUTIONS


K°a ¥ –8
K°a – 1 to pK°a
K°b ¥ –4
Kb – 1 to 14 – pKb
K°a ¥ –5

K°a
Ê [acid] ˆ [acid]
log Á =2
Ë [salt] ˜¯ [salt]
–1

∂b b( a - b) ˘
= 2.303 ÈÍ
∂(pH) Î a ˙˚
where a = [acid] + [salt] and b

∂b È (0.05 M)(0.15 M - 0.05 M) ˘


= 2.303 Í ˙
∂(pH) Î (0.15 M) ˚
[salt ] Ê 0.05 M ˆ
Ka + log = pKa + log Á
Ë 0.10 M ˜¯
pKa pKa
[acid]
Ka ¥ –12

ba ˆ
= 2.303 ÊÁ
[base][acid]
2.303 ˜
[base] + [acid] Ë b + a¯

[salt ] Ê 0.2 M ˆ
pK°b + log ¥ –5
) + log Á
[base] Ë 0.1 M ˜¯

n1 = VM1 ¥ –3 –1 –3
mol
n2 = VM2 ¥ –3 –1 –3 mol
m 0.04 g
n3 = = -1
= 10-3 mol
M 40 g mol
7.48 Complete Chemistry—JEE Main

–3 –3
= 2¥ –3
mol

1 1
2 [ pK°w + pK°a c/ mol dm–3)] = 2 ¥ –3)]

1
=2

SECTION 5 Solubility Product

Expression of Solubility Product

+(aq)
+ Cl–(aq)
CaF2(s) Ca2+(aq) + 2F–(aq)
Ca3(PO4)2 3Ca2+(aq) + 2PO3–
4 (aq)
Ks

+] [Cl–]
Ks
Ks(CaF2) = [Ca2+] [F–]2
Ks(Ca3(PO4)2) = [Ca2+]3 [PO43–]2
General Expression of Solubility Product =x y+ + yBx–
xB y
y+]x
Ks xB y) = [Bx–]y
s
x y+ + yBx–
xB y
(xs) (ys)
Ks = (xs)x (ys)y = xx yy sx+y or s = (Ks/xxyy)1/(x+y)
Illustrations
3–
4 ¥

s
Ca3(PO4)2 3Ca2+ + 2 [PO43–]
(3s) (2s)
s ¥ s ¥
Ks = [Ca2+]3[ PO42–]2 = (3s)3 (2s)2 s5 ¥ 5 ¥ –32 5

2CrO4 ¥ –12 3
2CrO4
+(aq) CrO42– (aq)
2CrO4(s) +
(2s) (s s
+ 2
Ks ] [ CrO42–] = (2s)2 (s) = 4s3
s = (Ks / 4)1/3 ¥ –12 3
/4)1/3 ¥ –4
Chemical and Ionic Equilibria 7.49

Ionic Product

Illustrations
–4
3) 2 2
Ks 2 ¥ –12 3
+ – –5
2+ –] 2 –4 –5 2 –14 3
2
Ks 2
–4
3) 2 2
–] ions need to initiate precipitation is

1/ 2 1/ 2
Ê Ks (Mg(OH) 2 ) ˆ Ê 8.9 ¥ 10-12 M 3 ˆ
–]
= Á ˜¯ =Á = 2.988 ¥ 10-4 M
Ë [Mg 2+ ] Ë (10-4 M ) ˜¯

¥ –4

Preferential precipitation of salts

1. Salts having the same stoichiometry

Illustration – – – +

Ks ¥ 2 Ks ¥ –13 2 and K
s ¥ 2

Ks Ks Ks
– – –

Quantitative Predictions + – – –

Ê Ks (AgCl) ˆ 1.7 ¥ 10-10 M 2


Cl– +] = Á = = 1.7 ¥ 10-9 M
Ë [Cl- ] ˜¯
For
(0.1 M )

Ê Ks (AgBr) ˆ 5.0 ¥ 10-13 M 2


For Br– +] = Á = = 5.0 ¥ 10-12 M
Ë [Br - ] ˜¯ (0.1 M )

Ê Ks (AgI) ˆ 8.5 ¥ 10-19 M 2


– +
]= Á = = 8.5 ¥ 10-18 M
Ë [I - ] ˜¯ (0.1 M )

+ + –

– + ¥ –12 – ions

7.50 Complete Chemistry—JEE Main

Ê Ks (AgI) ˆ (8.5 ¥ 10-19 M 2 ) -7


–] = Á [Ag + ] ˜ = (5.0 ¥ 10-12 M ) = 1.7 ¥ 10 M
Ë Br - ¯

Br– + ¥ – ions

Ê Ks (AgBr) ˆ (5.0 ¥ 10-13 M 2 )


[Br–] = Á = = 2.94 ¥ 10-4 M
Ë [Ag + ]Cl ˜¯ (1.7 ¥ 10-9 M )
2. Salts having different stoichiometry

Illustration +
2CrO4
Ks ¥ 2 and Ks 2CrO4 ¥ –12 3

+ – ions is
-10
Ks (AgCl) 1.7 ¥ 10 M 2
[Ag + ] = -
= = 1.7 ¥ 10-9 M
[Cl ] (0.1 M )
+ 2–
4 ions is
1/ 2 1/ 2
Ê K (Ag 2 CrO 4 ) ˆ Ê 1.9 ¥ 10-12 M3 ˆ
[Ag + ]= Á s =Á ˜¯ = 4.36 ¥ 10-6 M
Ë [CrO 42- ] ˜¯ Ë 0.1 M
+

The CrO42– +
¥ –6

Ks (AgCl) 1.7 ¥ 10-10 M 2


[Cl- ]= = = 3.90 ¥ 10-5 M
[Ag + ]CrO2 - 4.36 ¥ 10-6 M
4
– +
2CrO4
2– +
4

2CrO4

Qualitative Analysis of Cations

+ 2+ Pb2+
Group I 2 and
Group II 2+ 3+ 2+ 2+ 2+ 3+ 3+ and Sn2+ are

3+ 3+ and Cr3+
Group III
Group IV 2+ 2 2+
and Zn2+
2+ 2+ and Sr2+
Group V
2+ +
Group VI

MULTIPLE CHOICE QUESTIONS ON SECTION 5

4
–1
¥ –4

Na2SO4
¥ ¥ –8
¥ –8
¥ –8
Chemical and Ionic Equilibria 7.51

3(PO4)2 ¥ mol L–1


¥ –14 5 ¥ –32 5 ¥ –32 5 ¥ –32 5

2 Ka1 2 ¥ Ka2 2 ¥ –14


2–

+ 2+ and Fe2+ Ks ¥ –22 2 Ks(FeS) = 4 ¥ 2 and Ka1 Ka2


2 ¥ –21 2
2S in

Ks ¥ 2
2(Ks ¥ –11 3
3(Ks ¥ 2
2(Ks
¥ –15 3

2 (c) SrCO3 2

2 3) 2 Ks 2) =
¥ –15 3)

2 Ks ¥ 3

– – 2– 3– +
3 4

Ks ¥ 2
Ks ¥ 2)

2CO3 (Ks ¥ –12 3


3 4 (Ks ¥ –22 4)

s =(Ks 1/5
Ks
2CO3 3 (c) Ca3(PO4)2 2
¥
oxalate is
¥ 3
¥ 3
¥ 3
¥ –11 3

Ks(PbF2 ¥ –8 3
and Ks 2 ¥ 3
2 2

¥ –3
¥ –3
¥ –3
¥ –3

¥ –3
¥ –3
¥ –3
¥ –3

3
3+ – 3+ + –
]
3+ + – 3+ + –
]
+
2 (aq) Ba2+ Kc 2

not
2+
2–
and S are also considered

2
7.52 Complete Chemistry—JEE Main

ANSWERS
1. (b) 2. (c) 3. (d) 4. (a) 5. (b) 6. (a)
7. (b) 8. (b) 9. (c) 10. (d) 11. (a) 12. (b)
13. (d) 14. (a)

HINTS AND SOLUTIONS


Ê 9.32 ¥ 10-4 g ˆ Ê 1000 mL L-1 ˆ
1. Solubility (in mol/L) is s= Á = 4 ¥ 10–5 mol L–1
Ë 233 g mol-1 ˜¯ ÁË 100 mL ˜¯

For the equilibrium BaSO 4 (s)  Ba 2+ (aq ) + SO 24- (aq ), Ks (BaSO 4 ) = [Ba 2+ ][SO 24- ] = s2 = (4 ¥ 10–5 mol L–1)2

Ks (BaSO 4 ) (4.0 ¥ 10-5 mol L-1 )2


Solubility of BaSO4 in 0.1 M Na2SO4 is [Ba 2+ ] = = = 1.6 ¥ 10-8 M
[SO 24- ] -1
(0.1 mol L )
2+
2. Ca 3 (PO 4 ) 2  3 Ca + 2 PO34
(3 s ) (2s)

K s = [Ca 2+ ]3 [PO34- ]2 = (3s )3 (2s )2 = 108 s 5 = (108)(1.6 ¥ 10–7 M)5 = 1.13 ¥ 10–32 M5

3. H 2S  H + + HS- ; Ka1 = [H+][HS–]/[H2S]

HS-  H + + S2- ; Ka2 = [H+][S2–]/[HS–]

H 2S  2H + + S2- ; Ka = [H+]2[S2–]/[H2S]
Obviously, Ka = Ka1 Ka2 = (10–7 M) (10–14 M) = 10–21 M2

K a [H 2S] (10-21 M 2 )(0.1 M )


[S2–] = = = 2.5 ¥ 10-21 M
[H + ]2 (0.2 M ) 2
Number of S2– ions = [S2–]NA = (2.5 ¥ 10–21 mol L–1) (6.022 ¥ 1023 mol–1) = 1506 L–1

4. H 2S  2H + + S2- ; Ka1 Ka2 = 1.0 ¥ 10–21 M2

Since Ka1 Ka2 = [H+]2 [S2–]/[H2S], we have


K a1 K a 2 [H 2S] (1.0 ¥ 10-21 M 2 )(0.1 M )
[S2–] = = = 1.0 ¥ 10–20 M
[H + ]2 (0.1 M) 2
Ionic products of given ions are
For ZnS Qc = [Zn2+] [S2–] = (0.1 M) (10–20 M) = 10–21 M2
For FeS Qc = [Fe2+] [S2–] = (0.1 M) (10–20 M) = 10–21 M2
Since only for Qc(ZnS) > Ks(ZnS), only ZnS is precipitated.
5. Since Ks(SrCO3) > Ks(AgCl) and Ks(SrF2) > Ks(Fe(OH)2), we compute solubility of SrCO3 and SrF2 only.
SrCO3  Sr 2+ + CO32- ; K s = [Sr 2+ ][CO32- ] = s 2 s = Ks = (7 ¥ 10-10 M 2 )1/ 2 = 2.65 ¥ 10-5 M
s s
SrF2  Sr 2+ + 2F- ; Ks = [Sr 2+ ][F- ]2 = 4s 3 s = ( K s / 4)1 / 3 = (8.0 ¥ 10-11 M 3 / 4)1/ 2 = 2.71 ¥ 10-4 M
s 2s

Thus, SrF2 has largest solubility.


1/ 2 1/ 2
Ê Ks ˆ Ê 9 ¥ 10-15 M 3 ˆ
2+ – 2 [OH - ] = Á =Á = 3.0 ¥ 10-7 M
Ë [Mg 2+ ]˜¯
6. Ks (Mg(OH)2) = [Mg ] [OH ] . Hence
Ë 0.1 M ˜¯
Chemical and Ionic Equilibria 7.53

pOH = –log(3.0 ¥ 10–7) = 6.52 pH = 14 – 6.52 = 7.48


7. M (OH) 2  M 2+ + 2OH -
s 2s
Ks = [M2+] [OH–]2 = (s) (2s)2 = 4s3; s = (Ks /4)1/3 = (3.2 ¥ 10–17 M3/4)1/3 = (8 ¥ 10–18 M3)1/3 = 2 ¥ 10–6 M
[OH–] = 2s = 4 ¥ 10–6 M; pOH = –log(4 ¥ 10–6) = 5.4 pH = 14 – 5.4 = 8.6
+
8. The concentration of Ag ions needed to precipitate given ions are as follows.
Ks (AgCl) 1.8 ¥ 10-10 M 2
AgCl : [Ag + ] = = = 1.8 ¥ 10-9 M
[Cl- ] 0.1 M
Ks (AgI) 8.0 ¥ 10-17 M 2
AgI : [Ag + ] = = = 8.0 ¥ 10-16 M
[I- ] 0.1 M
1/ 2 1/ 2
+ Ê K (Ag 2 CO3 ) ˆ Ê 8.0 ¥ 10-12 M3 ˆ
Ag2CO3 : [Ag ] = Á s =Á ˜¯ = 8.94 ¥ 10-6 M
Ë [CO 2- ] ˜¯ 3 Ë 0.1 M
1/ 3 1/ 3
+ Ê K (Ag3 AsO 4 ) ˆ Ê 1.0 ¥ 10-22 M 4 ˆ
Ag3AsO4 : [Ag ] = Á s =Á ˜¯ = 1.0 ¥ 10-7 M
Ë [AsO3- ] ˜¯ 4 Ë 0.1 M
Ag+
9. The exponent 1/5 implies that ions are obtained from the ionization of salt, the salt is Ca3(PO4)2.
10. Ks = [Ag+]2 [Ox2–] = (0.1 M)2 (1.1 ¥ 10–9 M) = 1.1 ¥ 10–11 M3
11. Let x and y be the respective solubilities of PbF2 and MgF2. Then ([Pb2+] = x, [Mg2+] = y and [F–] = 2(x + y)
For PbF2 : Ks = [Pb2+][F–]2 = 4x(x + y)2 = 7.2 ¥ 10–8 M
For MgF2 : Ks = [Mg2+][F–] = 4y(x + y)2 = 14.4 ¥ 10–8 M
Solving for x and y, we get x = 1.26 ¥ 10–3 M and y = 2.52 ¥ 10–3 M
(Note:
12. Since the solution is electrically neutral, sum of total positive charge will be equal to total negative charge. Each
Al3+ carries 3+ charge, hence 3 [Al3+] + [H+] = [OH–].
13. Let s be the solubility of BaF2 in a buffer solution of pH = 2 (i.e. [H+] = 10–2 M). Then
BaF2 (s) + 2H + (aq) Æ Ba 2+ (aq) + 2HF(aq)
s 2s
2+ 2 2
[Ba ][HF] ( s )(2 s )
Kc = + 2
= ; 4s3 = (10–2 M)2 Kc = (10–2 M)2 (2.56 M)
[H ] (10-2 M ) 2
1/ 3
È (10-4 M 2 )(2.56 M ) ˘ -6 -2
˙ = (64 ¥ 10 M ) = 4 ¥ 10 M
1/ 3
s= Í
Î 4 ˚
14. The solubility of PbS is increased if the hydrolysis of Pb2+ and S2– ions are also considered. The solubility of Zn(OH)2
is pH dependent due to the reaction between OH– from Zn(OH)2 and H+ ions (i.e. pH) present in the solution.

SECTION 6 Acid-Base Indicators

Indicators, in general, are either organic weak acids or weak bases with a characteristic of having different colours in
the ionized and unionized form. For example,
7.54 Complete Chemistry—JEE Main

Phenolphthalein

Methyl orange

Indicator Constant Representing a weak-acid indicator as HIn, we write


HIn  H + + In -

[H + ][In - ] [HIn ]
K HIn = or [H + ] = K HIn
HIn [In - ]

[In - ]
or pH = pK°HIn + log (50)
[HIn]
The constant KHIn is known as indicator constant.
Indicator Range we have
The solution gets characteristic colour of In– if the ratio [In–]/[HIn] ≥ 10
The solution gets characteristic colour of HIn if the ratio [HIn] / [In–] ≥ 10.
With these parameters in Eq. (50), we have
For colour of In– pH = pK°HIn + 1 and For colour of HIn pH = pK°HIn – 1
Diagrammatically, we have

[In -1 ] 1
At pH = pK°HIn – 1, -
¥ 100 = ¥ 100 = 9.1 that is only 9.1% of indicator is present in the ionized
form. [In ] [HIn ] (1 + 10)
[In -1 ] 10
At pH = pK ∞HIn + 1, - ¥ 100 = ¥ 100 = 91.0 that is 91.0% of indicator is present in the ionized form.
[In ] + [HIn ] 10 +1
Chemical and Ionic Equilibria 7.55

Selection of Indicators In the titration of an acid solution (say, 0.1 M HCl) versus a base solution (say, 0.1 M
NaOH) or vice versa, the pH of the solution near the equivalence point changes steeply. The centre of this steep change
lies at pH = 7 only when a strong acid is titrated wita a 14.0
strong base. If the titration involves a weak acid or a weak
base, the pH is either greater than or lesser than 7 due to the 12.0
hydrolysis of the salt formed. 10.0
The selection of acid-base indicator to locate end point phenolphthalein
8.0
of a titration is decided by the following factors:
bromothymol
1. The steep portion of the titration curve at the 6.0 blue
equivalence point must encompass an interval
4.0

pH
of pH values at least as large as the pH transition methyl orange
range of an indicator. 2.0
2. The pH transition range of the indicator must
coincide with the steep portion of the titration 0.0
0 10 20 30 40 50 60
curve. Vh/cm3
The following is the description of selection of indicators Fig. 1
in different types of titrations.
14.0
1. Titration of a Strong Acid Versus Strong Base Figure
1 displays the typical titration curve depecting pH of the 12.0
solution versus volume of base added. The steep change of
pH at the equivalence point depends upon the concentrations 10.0
of acid and base being titrated. A typical range lies from
pH = 3 to pH = 11. Therefore, any indicator whose pH range 8.0
is within this interval can be used.
pH

6.0
The commonly used indicators are: methyl red
1. phenolphthalein (pH range : 8.3 to 11, colour change 4.0
is from colourless to pink).
2. methyl orange (pH range : 3.1 to 4.4, colour change 2.0
is from red to yellow).
3. methyl red (pH range : 4.2 to 6.3, colour change is 0.0 10 15 20 25 30 35 40
0 5
from red to yellow). Va/cm3
2. Titration of a Weak Base Versus a Strong Acid Fig. 2
Figure 2 displays the typical titration curve depicting pH of 14.0
the solution versus volume of acid added. A typical range of
pH at the equivalence point is from pH = 6.5 to pH = 4 The 12.0
commonly used indicators are methyl red and methyl orange.
10.0
3. Titration of a Weak Acid Versus a Strong Base phenolphthalein
8.0
Figure 3 displays the typical titration curve depicting pH of
the solution versus volume of base added. A typical range of bromothymol
pH

6.0 blue
pH at the equivalence point is from pH = 6 to pH = 10.5. The
commonly used indicator is phenolphthalein. 4.0
4. Titration of a Weak Acid Versus a Weak Base
2.0
In this titration, the steep rise in pH near the equivalence
point does not occur. Also, the rise of pH does not encompass
0.0 10 15 20 25 30 35 40
an interval equal to the pH transition range of any of the 0 5
indicators. Hence, no suitable indicator can be chosen to Vb/cm3
locate the end point of the titration. Fig. 3
7.56 Complete Chemistry—JEE Main

MULTIPLE CHOICE QUESTIONS ON SECTION 6

1. At pH = pK HIn
∞ - 1, an indicator is present in
(a) 80% ionized form (b) 85% ionized form (c) 9.1% ionized form (d) 50% ionized form
2. At pH = pK HIn
∞ + 1, an indicator is present in
(a) 80% ionized form (b) 91% ionized form (c) 25% ionized form (d) 50% ionized form
3. When 50% of an indicator is present in ionized form, the pH of the solution will be equal to
(a) pK°Hin (b) pK°Hin – 1 (c) pK°Hin ∞ - pK In
(d) pK HIn ∞-
4. Which of the following indicators (buffer range provided in brackets) cannot be used to detect end point of titration
between strong acid and strong base?
(a) Trinitrobezene (12.0 to 14.0) (b) Phenol red (6.0 to 7.6)
(c) p-Nitrophenol (5.0 to 7.0) (d) Phenolphthalein (8.3 to 10.0)
5. An indicator is 20% ionized in a solution of pH = 2.4. The percentage ionization in a solution of pH = 3.3 will be about
(a) 33.3% (b) 45.0% (c) 50.0% (d) 66.67%
6. The titration of a weak base versus weak acid cannot be carried out by using acid-base indicator because
(a) there is no variation in pH of the solution
(b) the variation in pH is gradual with no steep change in pH near the equivalence point.
(c) the change in pH near the equivalence point does not encompasses an interval equal to the pH transition range
of the indicator
(d) no indicator can be found which changes colour at pH of equivalence point.
7. 50 mL of 0.1 M solution of a weak acid is titrated against 0.1 M NaOH solution. After the addition of 10 mL of
NaOH, the pH of the solution is found to be 4.14. After the addition of 40 mL of NaOH solution, the pH of the
solution will be
(a) 4.74 (b) 5.10 (c) 5.34 (d) 6.20
8. Which of the following indicators can be used to detect end point in the titration of a weak acid versus strong base?
(a) Methyl red (4.2 to 6.3) 14.0
(b) Methyl violet (0.1 to 3.2)
12.0
(c) Methyl orange (3.1 to 4.4)
(d) Thymolphthalein (9.3 to 10.5) 10.0

9. The pH titration curve of 0.1 M Na2CO3 solution versus 0.1 M HCl is 8.0
pH

points are 8.34 and 3.97, respectively. The steep portion at these two 6.0

4.0

(a) Methyl violet (0.1 to 3.2) and phenolphthalein (8.3 to 10.0) 2.0
(b) Phenolphthalein (8.3 to 10.0) and methyl orange (3.1 to 4.4)
(c) Trinitrobenzene (12.0 to 14.0) and phenol red (6.8 to 8.4) 0.0
0 10 20 30 40 50 60
(d) Methyl orange and methyl violet Va/cm3
10. The colour of phenolphthalein in acid and base ranges of the indicator respectively, are
(a) pink, colourless (b) pink, red (c) colourless, pink (d) red, pink
11. The colour of methyl orange in acid and base ranges of the indicator, respectively, are
(a) yellow, red (b) red, yellow (c) yellow, blue (d) blue, red
Chemical and Ionic Equilibria 7.57

ANSWERS
1. (c) 2. (b) 3. (a) 4. (a) 5. (d) 6. (c)
7. (c) 8. (d) 9. (b) 10. (c) 11. (b)

HINTS AND SOLUTIONS


1. Since pH = pK°HIn + log ([In–]/[HIn]), we will have
log([In–]/[HIn]) = –1 or [In–]/[HIn] = 1/10. Percentage ionization = [1/(1 + 10)] ¥ 100 = 9.1%
2. Here log([In–]/[HIn]) = 1 or [In–]/[HIn] = 10/1. Percentage ionization = [10/(10 + 1)] ¥ 100 91%
3. At 50% ionization, [In– ]/[HIn] = 1. Hence, pH = pK°HIn
4. Indicator range of trinitrobenzene is beyond the steep portion of pH near the equivalence point.
5. We have
0.2 ˆ
2.4 = pK°HIn + log ÊÁ or pK°HIn = 3.0
Ë 0.8 ˜¯
3.3 = 3.0 + log ([In–/HIn]) i.e. log ([In–]/[HIn]) = 0.3 This gives [In–]/[HIn] = 2.
The percentage ionization = [2/(2 + 1)] ¥ 100 = 66.67%
6. During the titration, a buffer solution of weak acid and salt of its cojugate base is formed. Its pH is given by
pH = pK °a
4.14 = pK °a + log[10/(50 – 10)]. This gives pK°a = 4.14 – log(1/4) = 4.14 + 0.60 = 4.74
After the addition of 40 mL of NaOH solution, we will have
pH = 4.74 + log[40/(50 – 40)] = 4.74 + 0.60 = 5.34
8. In the titration of a weak acid (say, acetic acid) versus strong base (say, sodium hydroxide), the pH at the
equivalence point is more that 7 due to hydrolysis of congugate base of the weak acid. The steep rise in pH around
equivalence point encompasses the range 7 to 10. Thus, thymolphthalein can be used as the indicator to locate end
point.

10. Phenolphthalein is colourless in acid range and pink in base range of the indicator.
11. Methyl orange is red in acid range and yellow in base range of the indicator

MULTIPLE CHOICE QUESTIONS ON THE ENTIRE UNIT

pH of a Solution

(a) pH = – ln ([H+]/mol dm–3) (b) pH = – log ([H+]/mol dm–3)


(c) pH = – log [mol dm–3/[H+]) (d) pH = – ln (mol/dm–3/[H+])
2. Which of the following expressions is true?
(a) [H+] + [OH–] of water is a constant and is independent of temperature
(b) [H+] [OH–] of water is a constant and is independent of temperature
(c) pH + pOH = 14 and is independent of temperature.
(d) Irrespective of whatever may be the temperature, pH + pOH = pK°w
7.58 Complete Chemistry—JEE Main

3. What will be the pH of water at 50 °C? Given: pK°w = 13.26 at 50°C.


(a) 6.0 (b) 7.0 (c) 6.63 (d) 13.26
4. What will be the change in the pH of water if 10–3 mol of NaOH is added to 1.0 dm3 of water at 25°C?
(a) Increased by 2 (b) Increased by 4 (c) Decreased by 2 (d) Decreased by 4
+ ions present in 1 mL of a solution having pH = 13 is
5. The number of H
(a) 1013 (b) 6.022 ¥ 1013 (c) 6.022 ¥ 107 (d) 6.022 ¥ 1010
6. Give pK°w(H2O) = 13.54 at 40 °C. Predict the nature of solution having pH = 7 at 40 °C.
(a) acidic (b) alkaline (c) neutral (d) cannot be predicted
7. Which of the following expressions is not true?
(a) For a neutral solution [H+] = [OH–] = Kw
(b) For an acidic solution [H+] > Kw and [OH–] < Kw
(c) For an alkaline solution [H+] < K w and [OH–] > K w
(d) For a neutral solution [H+] = [OH–] = 10–7 M at all temperatures
–5
8. 10 M HCl solution at 25 °C is diluted 1000 times. The pH of the solution will
(a) be equal to 8 (b) lie between 7 and 8 (c) lie between 6 and 7 (d) remain unchanged
9. 10–5 M NaOH solution at 25 °C is diluted 100 times, the pH of the solution will
(a) be equal to 7 (b) lie between 7 and 8 (c) lie between 6 and 7 (d) remain unchanged
10. At 90 °C, pure water has [H3O+] = 10–6 mol dm–3. The value of pKw at 90 °C is
(a) 6 (b) 12 (c) – 6 (d) – 12
11. Which of the following statements is correct?
(a) pK°w increases with increase in temperature
(b) pK°w decreases with increase in temperature
(c) pK°w = 14 and is independent of temperature
(d) pK°w increases at low temperatures but decreases at high temperatures
12. For pure water,
(a) pH increases with increase in temperature
(b) pH decreases with increase in temperature
(c) pH = 7 and is independent of temperature
(d) pH increases at low temperatures but decreases at high temperatures
13. For pure water, the product [H+] [OH–]
(a) increases with increase in temperature
(b) decreases with increase in temperature
(c) is constant and is independent of temperature
(d) increases in the low temperature range but decreases in the high temperature range
14. The pH of a solution having [H+] = 10–8 M
(a) lies between 6 and 7 (b) lies between 7 and 8
(c) is 8 (d) is 7.5
15. The pH of a neutral water is 6.5. The temperature of water
(a) is 25 °C (b) is more than 25 °C (c) is less than 25 °C (d) cannot be predicted
16. 3 –4
One dm solution of 10 M HCl solution is diluted 1000 times. Its pH will be
(a) 7 (b) Less than 7 (c) More than 7 (d) eqaul to 8
17. 100 mL of 0.1 M HCl solution is mixed with 50 mL of 0.05 M NaOH. The resultant solution will have pH
(a) 1.5 (b) 1.8 (c) 2.12 (d) 3.42
18. Which of the following has the minimum [H+]?
(a) 0.1 M NaOH (b) 0.1 M NaC2H3O2 (c) 0.1 M NH3 (d) 0.1 M C2H5OH
Chemical and Ionic Equilibria 7.59

19. The pH of HCl solution is 1.0. If 900 mL of water is added to 100 mL of this solution, the pH of the resultant
solution will be
(a) 0.1 (b) 2.0 (c) 4.0 (d) 7.0
20. Which of the following solutions will have pH close to 1.0?
(a) 100 mL of 0.1 M HCl + 100 mL of 0.1 M NaOH
(b) 55 mL of 0.1 M HCl + 45 mL of 0.1 M NaOH
(c) 10 mL of 0.1 M HCl + 90 mL of 0.1 M NaOH
(d) 75 mL of 0.2 M HCl + 25 mL of 0.1 M NaOH
21. The number of H+ ions in 1 cm3 of a solution of pOH =13 is
(a) 6.022 ¥ 104 (b) 6.022 ¥ 1019 (c) 6.022 ¥ 1010 (d) 6.022 ¥ 1013
22. The pH of a solution is 3. If its pH is to be raised to 6, then the [H+] of the original solution has to be
(a) doubled (b) halved (c) increased 1000 times (d) decreased 1000 time
Weak Acids
23. The pH of a solution of 10–1 M acetic acid (Ka = 1.8 ¥ 10–5 M) is
(a) 2.87 (b) 1.87 (c) 3.87 (d) 1.34
24. The expression of Ostwald dilution law is
(a) a = K a c (b) a = Ka c (c) a = K a c (d) a = c K a
where the various symbols have their usual meanings.
25. For a fairly concentrated solution of a weak electrolyte AxBy, the degree of dissociation is given as
( )
1 ( x+ y)
(a) a = K eq c ( x + y ) (b) a = K eq c x + y -1 x x y y

(c) a = K eq c xy (d) a = K eq xy c
26. The pH of 0.1 M NH4OH (Kb = 1.8 ¥ 10–5
M) at 25 °C is
(a) 2.87 (b) 1.8 (c) 11.13 (d) 12.2
27. Which of the following shows the highest percentage dissociation?
(a) 1.0 M HCN (Kºdiss = 4.0 ¥ 10–10) (b) 0.1 M HCN
(c) 1.0 M HNO2 (Kºdiss = 4.5 ¥ 10 )–4
(d) 0.1 M HNO2
28. The pH of a 0.02 M Ca(OH)2 at 25°C solution is
(a) 12.0 (b) 12.3 (c) 12.6 (d) 13.6
29. A monoprotic acid is 0.001% ionized in 0.1 M of its solution. The ionization constant of the acid is
(a) 10–5 M (b) 10–8 M (c) 10–11 M (d) 10–13 M
30. Which one of the following will have the largest value of pH?
(a) 1.0 M acetic acid (b) 0.1 M acetic acid (c) 0.01 M acetic acid (d) 0.001 M acetic acid
31. The pH of 1 dm solution of 0.5 M acetic acid (K°a = 1.8 ¥ 10 ) is 2.52. If this solution is diluted 4 times, its pH
3 –5

will change by
(a) + 0.30 (b) + 0.60 (c) + 0.90 (d) – 1.2
32. The degree of dissociation of 10 M ammonium hydroxide (K°b = 1.8 ¥ 10 ) solution is about
–3 –5

(a) 0.25 (b) 0.20 (c) 0.13 (d) 0.09


Hydrolysis of Salts
33. A solution of 0.1 M CuSO4 solution is
(a) acidic in nature
(b) alkaline in nature
(c) neutral in nature
(d) acidic at low temperature, neutral at room temperature and alkaline at high temperature
7.60 Complete Chemistry—JEE Main

34. A solution of NaCN is


(a) acidic in nature
(b) alkaline in nature
(c) neutral in nature
(d) acidic at low temperature, neutral at room temperature and alkaline at high temperature
35. A solution of NaCl is
(a) acidic in nature
(b) alkaline in nature
(c) neutral in nature
(d) acidic at low temperature, neutral at room temperature and alkaline at high temperature
36. A solution of ammonium chloride is
(a) acidic in nature
(b) alkaline in nature
(c) neutral in nature
(d) acidic at low temperature, neutral at room temperature and alkaline at high temperature
37. A solution of ammonium acetate (Ka(acetic acid) = Kb(ammonium hydroxide)) is
(a) acidic in nature
(b) alkaline in nature
(c) neutral in nature
(d) acidic at low temperature, neutral at room temperature and alkaline at high temperature
38. A solution of ammonium cyanide (Ka(HCN) < Kb(NH4OH)) is
(a) acidic in nature
(b) alkaline in nature
(c) neutral in nature
(d) acidic at low temperature, neutral at room temperature and alkaline at high temperature
39. The expression to compute pH of NH4Cl solution is
(a) pH = 12 pK°w – 12 pK°b(NH4OH) – 12 log ([salt]/M)
1 1 1
(b) pH = 2
pK°w + 2
pK°b(NH4OH) – 2
log ([salt]/M)
1 1 1
(c) pH = 2
pK°w + 2
pK°b(NH4OH) + 2
log ([salt]/M)
1 1 1
(d) pH = 2
pK°b(NH4OH) – 2
pK°w + 2
log ([salt]/M)
40. The expression to compute pH of sodium acetate solution is
1 1 1
(a) pH = 2
pK°w – 2
pK°a(CH3COOH) – 2
log ([salt]/M)
1 1 1
(b) pH = 2
pK°w + 2
pK°a(CH3COOH) – 2
log ([salt]/M)
1 1 1
(c) pH = 2
pK°w + 2
pK°a(CH3COOH) + 2
log ([salt]/M)
1 1 1
(d) pH = 2
pK°a (CH3COOH) – 2
pK°w + 2
log ([salt]/M)
41. Mixing 0.1 mol of NaOH, 0.1 mol of HC2H3O2, and one litre of water, yields a solution which
(a) is acidic
(b) is neutral
(c) is basic
(d) may be acidic, neutral or basic depending upon the temperature
42. Which of the following solutions has the maximum pH value?
(a) 0.2 M HNO3 (b) 0.2 M HCl (c) 0.2 M HC2H3O2 (d) 0.2 M NaC2H3O2
Chemical and Ionic Equilibria 7.61

43. Which of the following is acidic?


(a) KCN solution (b) NaHCO3 solution (c) NH4Cl solution (d) CH3COONH4 solution
44. Which of the following is alkaline?
(a) KCl solution (b) CH3COONH4 solution
(c) FeCl3 solution (d) KCN solution
45. The pH of 0.1 M solution of the following salts increases in the order
(a) NaCl < NH4Cl < NaCN < HCl (b) HCl < NH4Cl < NaCl < NaCN
(c) NaCN < NH4Cl < NaCl < HCl (d) HCl < NaCl < NaCN < NH4Cl
46. Which of the following salt solutions will have lowest pH value?
(a) CaCO3 (b) CaCl2 (c) Ca(OH)2 (d) CH3COONa
47. The expression to be used to calculate the pH of ammonium acetate solution is
(a) pH = 12 (pK°w + pK°a – pK°b) (b) pH = 12 (pK°w + pK°a + pK°b)
1 1
(c) pH = 2
(pK°w – pK°a + pK°b) (d) pH = 2
(pK°w – pK°a – pK°b)
where the various symbols have their usual units.
48. The expression to be used to calculate the pH of sodium bicarbonate solution is
(a) pH = pK°a1 / pK°a2 (b) pH = pK°a1 + pK°a2
1 1
(c) pH = 2
(pK°a1 + pK°a2 ) (d) pH = 2
(pK°a1 – pK°a2)
where K°a1 and K°a2 stand for the standard ionization constants of H2CO3– and HCO3–, respectively.
Solubility Product
49.
ÈCa 2+ ˘˚ ÈÎPO34- ˘˚
(a) Ksp = [Ca2+] [PO3–
4 ] (b) Ksp = Î
[ Ca 3 (PO 4 )2 ]

(c) Ksp = [Ca2+]3 [PO3–


4 ]
2 (d) Ksp = [Ca2+]2 [PO3–
4 ]
3

50. The solubility of mercurous chloride in water will be given as


(a) Ksp(Hg2Cl2) (b) Ksp(Hg 2 Cl2 ) 4 (c) [Ksp(Hg2Cl2)/4]1/3 (d) [Ksp(Hg2Cl2)]1/3
51. If s0, s1, s2 and s3 are the solubilities of AgCl in water, 0.01 M CaCl2, 0.01 M NaCl and 0.05 M AgNO3 solutions,
respectively, then
(a) s0 > s1 > s2 > s3 (b) s0 > s2 > s1 > s3 (c) s0 > s2 > s3 > s1 (d) s0 > s1 = s2 > s3
52. Given: Ksp(AgCl) = 1.7 ¥ 10–10 M2, Ksp(Ag2CrO4) = 1.9 ¥ 10–12 M3
AgNO3 is added to a solution containing 0.1 M each of NaCl and Na2CrO4?
(a) chloride ion (b) chromate ion
(c) Both Cl– and CrO2–
4 precipitate together (d) cannot be predicted
53. The solubility of magnesium hydroxide in pure water is 8.1 ¥ 10–5 mol per 500 mL of water. The solubility product
of magnesium hydroxide is
(a) 6.72 ¥ 10–9 M2 (b) 5.51 ¥ 10–13 M3 (c) 8.84 ¥ 10–12 M3 (d) 1.77 ¥ 10–11 M3
54. Given: Ksp(AgI) = 8.5 ¥ 10 M . The solubility of AgI in 0.1 M KI solution is
–17 2

(a) 8.5 ¥ 10–15 M (b) 8.5 ¥ 10–16 M (c) 8.5 ¥ 10–17 M (d) 8.5 ¥ 10–18 M
55. The concentration of S2– in a 0.1 M H2S (Ka1Ka2 = 1.1 ¥ 10–21 M2) which is also 0.1 M in H+ is
(a) 1.1 ¥ 10–19 M (b) 1.1 ¥ 10–20 M (c) 1.1 ¥ 10–21 M (d) 1.1 ¥ 10–22 M
56. The pH of Mg(OH)2 solution is 10.45 at 25 °C. The solubility product of magnesium hydroxide will be
(a) 2.24 ¥ 10–11 M3 (b) 1.12 ¥ 10–11 M3 (c) 3.36 ¥ 10–11 M3 (d) 5.60 ¥ 10–12 M3
57. The solubility product of A2X3 is 1.08 ¥ 10 M . Its solubility is
–23 5

(a) 1.0 ¥ 10–3 M (b) 1.0 ¥ 10–4 M (c) 1.0 ¥ 10–5 M (d) 1.0 ¥ 10–6 M
7.62 Complete Chemistry—JEE Main

58. The molarity of Mg 2+ ions in a saturated solution of Mg 3(PO 4) 2 whose solubility product is
1.08 ¥ 10–13 M5 is
(a) 1.0 ¥ 10–3 M (b) 2.0 ¥ 10–3 M (c) 3.0 ¥ 10–3 M (d) 4.0 ¥ 10–3 M
2+, Zn2+, Mn2+ and Co2+ is saturated with
59.
H2S (which provides 1.0 ¥ 10 M of S in the solution) is
–14 2–

(a) Pb2+ (b) Zn2+ (c) Mn2+ (d) Co2+


(Given: Ksp(PbS) = 2.3 ¥ 10 M , Ksp(ZnS) = 1.0 ¥ 10 M , Ksp(MnS) = 7.9 ¥ 10 M2 and Ksp(CoS) = 7 ¥
–27 2 –21 2 –13

10–23 M2.)
3 ] was found to be 1.2 ¥ 10 M. The concentration
60. A solution is saturated with respect to SrCO3 and SrF2. The [CO2– –3
– in the solution would be
of F
(a) 1.3 ¥ 10–3 M (b) 3.7 ¥ 10–2 M (c) 5.8 ¥ 10–7 M (d) 2.6 ¥ 10–2 M
(Given: Ksp(SrCO3) = 7.0 ¥ 10–10 M2 and Ksp(SrF2) = 7.9 ¥ 10–10 M3.)
61. The maximum pH of a solution which have 0.10 M of Mg2+ from which Mg(OH)2 is not precipitated is
(a) 4.96 (b) 6.96 (c) 7.04 (d) 9.04
(Given: Ksp(Mg(OH)2) = 1.2 ¥ 10 M .)–11 3

62. Which of the following is most soluble?


(a) CuS (K°sp = 8 ¥ 10–37) (b) Ag2S (K°sp = 6 ¥ 10–51)
(c) Bi2S3 (K°sp = 1 ¥ 10–70) (d) MnS (K°sp = 7 ¥ 10–16)
Buffer Solution
63. The pH of a solution obtained by mixing 20 mL of 0.02 M sodium acetate and 10 mL of 0.01 M acetic acid
(Ka = 1.8 ¥ 10–5 M) is
(a) 4.75 (b) 5.05 (c) 4.45 (d) 5.34
64. Which of the following constitutes a buffer solution when dissolved in 500 mL water?
(a) 0.05 mol NaOH and 0.05 mol HCl
(b) 0.05 mol NaCl and 0.05 mol HCl
(c) 0.05 mol propanoic acid and 0.05 mol sodium propanoate
(d) 0.05 mol acetic acid and 0.05 mol sodium hydroxide
65. Which of the following can be used as a buffer solution?
(a) Sodium chloride solution (b) Ammonium chloride solution
(c) Sodium acetate solution (d) Ammonium acetate solution
66. The solution containing weak acid and salt of its conjugate base has maximum buffer capacity when
(a) [salt] = [acid] (b) [salt] > [acid]
(c) [salt] < [acid] (d) [salt] + [acid] is minimum
67. Adding sodium hydroxide to a solution of acetic acid
(a) increases [C2H3O–2] (b) increases [H3O+] (c) increases [HC2H3O2] (d) decreases the pH
68. In a buffer solution, pH = pKa when the solution contains
(a) [salt] = [acid] (b) [salt] > [acid]
(c) [salt] < [acid] (d) [salt] + [acid] has a maximum value
69. Which of the following buffer solutions containing the indicated ratio of [salt]/[acid] has a maximum buffer
capacity?
(a) 1/2 (b) 1 (c) 2/1 (d) 4/1
70. A buffer solution is prepared by using a weak acid having pKa = 4. The approximate range it can be used effectively
is
(a) 2 to 4 (b) 3 to 4 (c) 3 to 5 (d) 4 to 5
71. A buffer solution of a weak acid and salt of its conjugate base is diluted to twice its volume. Its pH
(a) increases (b) decreases
(c) remains the same (d) does change but cannot be predicted
Chemical and Ionic Equilibria 7.63

Indicator
72. An acid-base indicator has Kºa = 3.0 ¥ 10–5. The acid form of the indicator is red and the basic form is blue. The
change in [H+] required to change the indicator from 75% red to 75% blue is
(a) 8 ¥ 10–5 M (b) 9 ¥ 10–5 M (c) 1 ¥ 10–5 M (d) 4 ¥ 10–5 M
73. Indicator range of phenolphthalein is
(a) 4.2 to 6.3 (b) 5.0 to 7.0 (c) 8.3 to 10.0 (d) 12.0 to 14.0
74. Indicator range of methyl red is
(a) 4.2 to 6.3 (b) 5.0 to 7.0 (c) 8.3 to 10.0 (d) 12.0 to 14.0
75. The indicator thymol blue has pH = 2.0 when half of the indicator is present in the ionized form. The pK°Indicator is
(a) 1.0 (b) 2.0 (c) 3.0 (d) 4.0
76. The indicator constant of an indicator is 3.0. The percentage of the indicator in the unionized form in a solution
of pH = 2.6 is
(a) 40% (b) 55% (c) 62% (d) 71%
77. The indicator constant of an indicator is 3.0. The percentage of the indicator in the unionized form in a solution
of 3.4 is
(a) 28.6% (b) 35% (c) 42% (d) 51%
78. The indicator constant of an indicator is 10–4. If pH of solution in which this indicator is present is 3.5, which of
the following facts will be correct?
(a) Ionized indicator is present in the larger amount
(b) Unionized indicator is present in the larger amount
(c) Both ionized and unionized indicator are present in equal amounts
(d) Percentage of indicator present in ionized form cannot be predicted
Acids and Bases
79. The conjugate acid of NH2– is
(a) NH3 (b) NH2OH (c) NH+4 (d) N2H4
80. The compound that is not a Lewis acid is
(a) BF3 (b) AlCl3 (c) BeCl3 (d) SnCl4
81. Which of the following acids is expected to be strongest?
(a) H3PO3 (b) H3PO4 (c) H3AsO3 (d) H3AsO4
82. In the given anions, the weakest Brönsted base is
(a) ClO–4 (b) ClO2– (c) ClO–3 (d) ClO–
83. Which of the following acts as a Brönsted acid as well as a Brönsted base?
(a) Na2CO3 (b) OH – (c) NH3 (d) HCO3–
84. In the reaction HCN + H2O H3O+ + CN–, the conjugate acid-base pair is
(a) HCN, H2O (b) H3O+, CN – (c) HCN, H3O+ (d) HCN, CN–
85. Which of the following statements is not correct?
(a) A substance which supplies OH– in aqueous medium is a base.
(b) A substance which can accept a proton in aqueous medium is a base.
(c) A substance which can donate a pair of electrons is a base.
(d) A substance which can accept a pair of electrons is a base.
86. Which of the following species is not a Lewis acid?
(a) BF3 (b) Cu2+ (c) NH3 (d) AlCl3
87. In aqueous solution, which of the following orders regarding acid strength is correct?
(a) HCN < CH3COOH < HCl < HClO4 (b) HCN < CH3COOH < HClO4 < HCl
(c) HCN < HClO4 < HCl < CH3COOH (d) HClO4 < CH3COOH < HCl < HCN
7.64 Complete Chemistry—JEE Main

88. In aqueous solution, which of the following orders regarding base strength is correct?
(a) CH3COO– < HS– < NH3 < CO32– (b) CH3COO– > HS– > NH3 > CO2– 3

(c) CH3COO– > HS– < NH3 < CO32– (d) CH3COO– > HS– > CO32– > NH3
89. The decreasing base strength of OH–, NH2–, HC∫∫C– and CH3CH2– is
(a) CH3CH2– > NH2– > HC∫∫C– > OH– (b) HC∫∫C– > CH3CH2– > NH2– > OH–
– – –
(c) OH > NH2 > HC∫∫C > CH3CH2 – (d) NH2– > HC∫∫ C– > OH– > CH3CH2–
90. The correct order of increasing base strength of Cl–, ClO4– and CH3COO– is
(a) ClO4– < CH3COO– < Cl– (b) CH3COO– < Cl– < ClO4–
– –
(c) ClO4 < Cl < CH3COO – (d) Cl– < ClO4– < CH3COO–
91. The strongest base amongst OH–, F–, NH2– and CH3– is
(a) NH2– (b) CH3– (c) F– (d) OH–

Problems for Revision


Based on acetic acid (K°a = 1.8 ¥ 10–5) solution, answer the following questions.
92. The degree of dissociation of 0.05 M acetic acid solution is
(a) 0.0019 (b) 0.019 (c) 0.19 (d) 0.45
93. The concentration of H+ in 0.05 M acetic acid solution is
(a) 4.5 ¥ 10–3 M (b) 9.5 ¥ 10–3 M (c) 4.5 ¥ 10–4 M (d) 9.5 ¥ 10–4 M
94. The pH of 0.05 M acetic acid solution is
(a) 3.02 (b) 3.52 (c) 4.02 (d) 4.52
95. If 0.05 M acetic acid solution is diluted 10 times, its degree of dissociation changes to
(a) 0.02 (b) 0.04 (c) 0.06 (d) 0.08
96. If 0.05 M acetic acid solution is diluted 10 times, its pH becomes
(a) 3.53 (b) 2.22 (c) 3.22 (d) 4.22
97. It is desired to change [H+] of 0.05 M acetic acid solution to its half value, the dilution factor of the solution is
(a) 4.0 (b) 5 (c) 6.0 (d) 6.5
98. It is desired to change pH of 0.05 M acetic acid solution to 1.5 times pH value, the solution should be diluted to
(a) 540.5 (b) 870.5 (c) 1033.3 (d) 1150.0
99. If to 100 mL of 0.1 M acetic acid solution, 0.82 g of sodium acetate is added, the pH of the solution (assume no
dilution factor) becomes
(a) 3.75 (b) 4.75 (c) 5.25 (d) 5.75
100. If the solution obtained in Q.99 is diluted 10 times, it pH becomes
(a) 4.75 (b) 3.75 (c) 5.75 (d) 6.25
101. In going from solution of Q.99 to the solution of Q.100, its buffer capacity
(a) increases
(b) decreases
(c) remains constant
(d) may increase or decrease depending upon temperature
102. The buffer range of the acidic buffer obtained from acetic acid and sodium acetate is
(a) 3.0 to 5.0 (b) 3.2 to 5.2 (c) 3.75 to 5.75 (d) 4.2 to 6.2
103. If to a solution of Q. 99, 0.002 mol of HCl is added, its pH changes to
(a) 2.0 (b) 3.0 (c) 3.5 (d) 4.6
104. If to a 100 mL of solution of Q. 100, 0.002 mol of HCl is added, its pH changes to
(a) 1.5 M (b) 2.0 (c) 3.0 (d) 4.0
Chemical and Ionic Equilibria 7.65

105. The pH of a solution of a salt increases with increase in the concentration of the salt. The salt may be
(a) ammonium chloride (b) sodium acetate (c) ammonium acetate (d) sodium bicarbonate
106. The pH of a solution of a salt decreases with increase in the concentration of the salt. The salt may be
(a) ammonium chloride (b) sodium acetate (c) ammonium acetate (d) sodium bicarbonate
107. The pH of a solution of a salt remains more or less constant with change in the concentration of the salt. The salt
may be
(a) ammonium chloride (b) sodium acetate (c) ammonium acetate (d) sodium sulphate
108. The pH of sodium bicarbonate solution depends on
(a) the ionization constant of H2CO3
(b) the ionization constant of HCO 3–
(c) the ionization constants of both H2CO3 and HCO3–
(d) the concentration of sodium bicarbonate
109. The pH of a solution of salt formed from a weak acid and a weak base decreases with
(a) increase in pKa of acid (b) increase in pKb of base
(c) increase in concentration of the salt (d) decrease in concentration of the salt.
110. The solubility of Zn(OH)2 in water with increase in pH of the solution (from acidic to neutral to alkaline conditions)
shows the pattern
(a) initially decreases followed by an increase
(b) initially increases followed by a decrease
(c) remains constant

111. In the titration of H2 Vbase are observed provided


(a) Ka1/Ka2 £ 1 (b) Ka1/Ka2 £ 10 (c) Ka1/Ka2 £ 100 (d) Ka1/Ka2 ≥ 1000
112. The solubility of sparingly soluble salt AxBy in water is given by the expression
1/( x + y ) x + y)
(a) s = Ksp (b) s = xx yy K 1/(
sp (c) s = (Ksp/xxyy)1/(x + y) (d) s = (Ksp/xyyx)1/(x + y)

ANSWERS
1. (b) 2. (d) 3. (c) 4. (b) 5. (c) 6. (b)
7. (d) 8. (c) 9. (b) 10. (b) 11. (b) 12. (b)
13. (b) 14. (c) 15. (b) 16. (b) 17. (c) 18. (a)
19. (b) 20. (d) 21. (b) 22. (d) 23. (a) 24. (a)
25. (b) 26. (c) 27. (d) 28. (c) 29. (c) 30. (d)
31. (a) 32. (c) 33. (a) 34. (b) 35. (c) 36. (a)
37. (c) 38. (b) 39. (a) 40. (c) 41. (c) 42. (d)
43. (c) 44. (d) 45. (b) 46. (b) 47. (a) 48. (c)
49. (c) 50. (c) 51. (b) 52. (a) 53. (d) 54. (b)
55. (b) 56. (b) 57. (c) 58. (c) 59. (a) 60. (b)
61. (d) 62. (d) 63. (d) 64. (c) 65. (d) 66. (a)
67. (a) 68. (a) 69. (b) 70. (c) 71. (c) 72. (a)
73. (c) 74. (a) 75. (b) 76. (d) 77. (a) 78. (b)
79. (a) 80. (c) 81. (b) 82. (a) 83. (d) 84. (d)
85. (d) 86. (c) 87. (a) 88. (a) 89. (a) 90. (c)
7.66 Complete Chemistry—JEE Main

91. (b) 92. (b) 93. (d) 94. (a) 95. (c) 96. (a)
97. (b) 98. (c) 99. (b) 100. (a) 101. (b) 102. (c)
103. (d) 104. (b) 105. (b) 106. (a) 107. (c) 108. (c)
109. (b) 110. (a) 111. (d) 112. (c)

HINTS AND SOLUTIONS

3. pK°w = pH + pOH For water pH = pOH. Hence, pH = 12 pK°w = 6.63


4. pH changes from 7 to 11. Hence, change is 4.
5. pH equal to 13 implies that [H+] = 10–13 mol L–1
Amount of H+ in 1 mL solution = 10–16 mol
Number of H+ in 1 mL solution = (10–16 mol) (6.022 ¥ 1023 mol–1) = 6.022 ¥ 107
6. For neutral water pH = 12 pKw = 6.77. Since given pH (= 7) is greater than 6.77, it is an alkaline solution.
7. The statement (d) is applicable only at 25°C and is not independent of temperature.
8. The solution will remain acidic as it represents 10–8 M HCl. Hence, its pH will be less than 7 at 25 °C.
9. Solution will become 10–7 M in NaOH. Since the solution remains alkaline, its pH will be more than 7.
10. For pure water, [H3O+] = [OH–]
Kw= [H3O+] [OH–] = 10–12 (mol dm–3)2; pK°w = – log K°w = 12
11. Kw increases with increase in temperature. pK°w (= – log K°w) will decrease with increase in Kw.
12. For a pure water, [H+] increases with increase in temperature due to more ionization of water. pH (= – log [H+])
will decrease.
15. pH equal to 6.5 means [H+] > 10–7 M, i.e. there occurs more ionization of water as compared to that occurring at
25 °C. More ionization means temperature is greater than 25 °C
19. [H+] in pH = 1.0 is 10–1 M. Solution becomes diluted ten times. Hence, its [H+] becomes 10–2 M. Its pH would
become 2.
20. The pH of choice a will be 7 as the acid completely neutralizes the base. The choice b will contain 10 mL of excess
0.1 M HCl. The total volume of solution becomes 100 mL. Hence, its effective molarity will be 0.01 M and thus
its pH = 2. The choice c will contain 80 mL of excess 0.1 M NaOH. Hence, its pH > 7. The choice d will contain 50 mL
of excess 0.2 M HCl. Because of dilution to 100 mL, its effective molarity will be 0.1 M and hence pH = 1.
21. For pOH = 13, [OH–] = 10–13 M and [H+] = 10–1 M
Ê 10-1 mol ˆ
Number of H+ in 1 cm3 solution = Á 3 3 ˜ (6.022 ¥ 1023 mol–1) = 6.022 ¥ 109 cm–3
Ë 10 cm ¯

22. For pH = 3, [H+] = 10–3 M. For pH = 6, [H+] = 10–6 M. Hence, [H+] has to be decreased 1000 times.
23. CH3COOH CH3COO– + H+
[CH3COO - ] [H + ] [H + ]2
Ka = 
[CH3COOH] [CH3COOH]0

[H+] = (Ka[CH3COOH]0]1/2 = (1.8 ¥ 10–5 ¥ 0.1)1/2 M = 1.34 ¥ 10–3 M


pH = – log (1.34 ¥ 10–3) = 2.87
25. Ax By x Ay+ + yBx–
c(1 – a) c(xa) c(ya)
1/( x + y )
(c x a ) x (c y a ) y Ê K eq ˆ
Keq=  c x+y–1 x xyy ax+y or a = Á x + y -1 x y ˜
c (1 - a ) Ëc x y ¯
Chemical and Ionic Equilibria 7.67

26. NH4OH NH+4 + OH–


[NH +4 ] [OH - ] [OH - ]2
Keq =  (since [NH+4] = [OH–])
[NH 4 OH] [NH 4 OH]0

or [OH–] = K eq [NH 4 OH ]0 = 1.8 ¥ 10-5 ¥ 0.1 M 2 = 1.34 ¥ 10–3 M


pOH = – log {[OH–]/M} = 2.87; pH = 14 – pOH = 11.13
27. Ostwald dilution law; more dilute the solution, more the dissociation.
28. Assuming complete dissociation
[OH–] = 2 ¥ 0.02 M; pOH = 1.40; Thus pH = 14 – pOH = 12.6
[H + ] [A - ] (ca ) 2
29. HA H+ + A– Ka = =  ca 2
[HA] c (1 - a )
c(1 – a) ca ca
Hence, Ka = (0.1 M) (10–5)2 = 10–11 M
30. The less concentrated solution will produce lesser H+ and hence larger pH.
31. Solution concentration becomes 0.125 M

K ∞a 1.8 ¥ 10-5 M
a= = = 0.012 ; [H+] = ca = (0.125 M) (0.012) = 0.0015 M
c 0.125 M
pH= –log (0.0015) = 2.82; D(pH) = 2.82 – 2.52 = 0.30

Ka 1.8 ¥ 10-5 M
32. a = = 0.134
c 10 -3 M
33. Solution is acidic due to the hydrolysis reaction Cu2+ + H2O Cu(OH)+ + H+
34. Solution is alkaline due to the hydrolysis reaction CN– + H2O HCN + OH–
+ –
35. Solution is neutral because no hydrolysis of Na and Cl take place.
36. Solution is acidic due to the hydrolysis reaction NH4+ + H2O NH4OH + H+
37. A solution of ammonium acetate is neutral because both NH4 and Ac– ions are hydrolysed to equal extent.
+

38. Solution is alkaline as CN– is hydrolysed more than NH4+ ions.


39. NH+4 + H2O NH4OH + H+
2
K w [ NH 4 OH ] ÈÎH ˘˚ ÈÎH ˘˚
+ +
Kw
K h= =  or [H+]2 = [NH4]0
Kb ÈÎ NH 4+ ˘˚ ÈÎ NH 4+ ˘˚ Kb
0
1 1 1
2 log ([H+]/M) = log K°w – log K°b + log ([NH4+]/M); or – log ([H+]/M)= – 2
log K°w + 2
log K°b – 2
log ([NH4+]/M)
1 1 1
pH = 2
pK°w – 2
pK°b – 2
log ([salt]/M)

40. Ac– + H2O HAc + OH–


2
K w [ HAc] ÈÎHO ˘˚ ÈÎOH ˘˚
- - 2
K Ê Kw ˆ Kw
Kh = =  ; [OH ] = w [Ac–]0 ;
– 2 –
ÁË + 2 ˜¯ = K [Ac ]0
Ka ÈÎAc- ˘˚ ÈÎAc- ˘˚ Kb [H ] a
0
1 1 1
which on taking log and multiplying by – 1 gives pH = 2
pK°w + 2
pK°a + 2
log ([salt]/M)
42. Sodium acetate solution will be alkaline and thus will have maximum value of pH.
45. HCl is the strongest acid. Its pH will be minimum. NaCl does not hydrolyse in solution. Its pH = 7
NH +4 in NH4Cl on hydrolysis produces H+. Hence, its pH will be slightly less than 7.
CN– in NaCN on hydrolysis produces OH–. Hence, its pH will be slightly greater than 7.
7.68 Complete Chemistry—JEE Main

46. In CaCO3, both ions show hydrolysis. In CaCl2, only Ca2+ is hydrolysed. Both Ca(OH)2 and CH3COONa will be
alkaline in nature. Hence, the lowest pH will be that of CaCl2 solution.
47. NH +4 + Ac– + H2O NH4OH + HAc
[ NH 4 OH][HAc] [HAc]2
Kh =
[ NH +4 ][Ac- ] [Ac- ]2

[NH 4 OH ] [HAc] Kw
Also Kh = ◊ [H3O + ][OH - ] =
[NH +4 ][OH - ] -
[Ac ][H3O ] +
Kb Ka

[HAc] Kw [H3O + ] Kw K w Ka
Hence = or = or H 3O + =
[Ac- ] Ka K b Ka Ka K b Kb

1
pH = (pK°w + pK°a – pK° )
2 b

[H 2 CO3 ][OH - ] Kw
48. HCO–3 + H2O H2CO3 + OH– ; Kh = =
[HCO3- ] K a1

[H3O + ][CO32- ]
HCO–3 + H2O H3O+ + CO2–
3; Ka2 =
[HCO3- ]

Kh Kw [H CO ] [OH - ] Kw
= = 2 -3  (since [H2CO3] ; [CO32–])
Ka 2 K a1 K a 2 [CO3 ] [H3O ] [H3O + ]2
+

1
Also Kh/Ka2 = Kw/Ka1Ka2. Hence, [H3O+]2 = Ka1 Ka2 or pH = (pK a∞1 + pK a∞2 )
2
x = ( Ksp / 4)
13
50. Hg2Cl2 Hg2+
2 + 2 Cl

Ksp = [Hg2+ – 2 2
2 ] [Cl ] = (x) (2x) ;
x 2x
51. s0 = Ksp ; s1 = Ksp/0.02 M; s2 = Ksp/0.01 M; s3 = Ksp/0.05 M. Obviously s0 > s2 > s1 > s3.
-10
52. [Ag+] needed to start precipitation of Cl– = 1.7 ¥ 10 M
0.1

[Ag+] needed to start precipitation of CrO2– 1.9 ¥ 10-12


4 = M
0.1
– to precipitate.
53. Solubility per litre of water = 16.2 ¥ 10–5 mol L–1
Ksp = [Mg2+] [OH–]2 = (16.2 ¥ 10–5 M) (2 ¥ 16.2 ¥ 10–5 M)2 = 1.77 ¥ 10–11 M3
Ksp 8.5 ¥ 10-17 M 2
54. [Ag+] = = = 8.5 ¥ 10-16 M
[I- ] 0.1 M

K a1 K a2 (1.1 ¥ 10-21 M3 )(0.1 M )


55. [S2–] = [H 2S] = = 1.1 ¥ 10–20 M
[H + ]2 [0.1 M ]2

56. pH = 10.45; pOH = 14 – 10.45 = 3.55; [OH–]/M = antilog (– 3.55) = 2.82 ¥ 10–4

Ê 2.82 ¥ 10-4 ˆ
Ksp = [Mg2+] [OH–]2 = Á M˜ (2.82 ¥ 10-4 M ) 2 = 1.12 ¥ 10–11 M
Ë 2 ¯
Chemical and Ionic Equilibria 7.69

57. A2X3 2A3+ + 3X2– Ksp = [A3+]2 [X2–]3 = (2x)2 (3x)3 = 108x5
(2x) (3x)
1/ 5 1/ 5
Ê Ksp ˆ Ê 1.08 ¥ 10-23 5 ˆ
x= Á ˜ =Á M ˜ = 1.0 ¥ 10-5 M
Ë 108 ¯ Ë 108 ¯
58. Mg3(PO4)2 3Mg2+ + 2 PO3–4 ; Ksp = [Mg2+]3 [PO3–
4 ]
2

If x is the solubility of Mg3(PO4)2, then


1/ 5
Ê 1.08 ¥ 10-13 5 ˆ
(3x) (2x) = 1.08 ¥ 10
3 2 –13 5
M; x= Á M ˜ = 0.001 M
Ë 108 ¯
[Mg2+] = 3x = 3 ¥ 10–3 M
59. Lesser the value of Ksp, earlier the salt precipitates. Hence, Pb2+

60. [Sr2+] =
Ksp (SrCO3 ) 7.0 ¥ 10-10 M 2 = 5.8 ¥ 10–7 M
=
[CO32- ] 1.2 ¥ 10-3 M
1/ 2 1/ 2
È Ksp (SrF2 ) ˘ Ê 7.9 ¥ 10-10 M 3 ˆ
[F–] = Í ˙ =Á = 3.7 ¥ 10–2 M
2+
Î [Sr ] ˚ Ë 5.8 ¥ 10-7 M ˜¯
61. For Mg(OH)2 not to be precipitated
1/ 2 1/ 2
È Ksp (Mg (OH)2 ) ˘ Ê 1.2 ¥ 10-11 M 3 ˆ
[OH–] < Í ˙ ; [OH–] < Á ˜¯ = 1.095 ¥ 10–5 M
Î [Mg 2+ ] ˚ Ë 0.10 M

pOH > 4.96; pH < 14 – 4.96 = 9.04


[salt] (0.02 ¥ 2 3) M
63. pH = pK°a + log = – log (1.8 ¥10–5) + log = 4.74 + 0.60 = 5.34
[acid] (0.01 ¥ 1 3) M
[salt ]
68. For an acidic buffer solution pH = pK∞a + log
[acid]
Obviously, pH = pK°a if [salt] = [acid].
69. The buffer capacity is maximum when the ratio of concentrations of salt to acid is one. It decreases on either sides
of this ratio.
70. The approximate buffer range is in the range pK°a – 1 to pK°a + 1.
71. The pH of a buffer solution is not changed on dilution as the ratio of concentrations of salt to acid remains
unchanged.
72. For 75% red, we have
[salt]
= – log (3 ¥ 10–5) + log ÊÁ ˆ˜ = 4.52 – 0.48 = 4.04
25
pH = pK°a + log
[acid] Ë 75 ¯
[H+] = 10–pH = 10–4.04 = 9.12 ¥ 10–5 M
For 75% blue, we have
pH = – log (3 ¥ 10–5) + log ÊÁ ˆ˜ = 4.52 + 0.48 = 5.00;
75
[H+] = 10–5 M
Ë 25 ¯
The change in [H+] is
D [H+]= 9.12 ¥ 10–5 M – 1.0 ¥ 10–5 M = 8.12 ¥ 10–5 M
[ In - ]
75. pH = pK°HIn + log , since [In–] = [HIn], pK°HIn = pH
[ HIn ]
[ In - ] [ In - ] [ In - ] [ In - ] + [HIn ]
76. 2.6 = 3.0 + log . Hence = antilog (–0.4) or = 0.40 or = 1.40
[ HIn ] [ HIn ] [ HIn ] [ HIn ]

Per cent of HIn = (1/1.40) (100) = 71.4%


7.70 Complete Chemistry—JEE Main

[ In - ] [ In - ] [ In - ] [ In - ] + [HIn ]
77. 3.4 = 3.0 + log . Hence, = antilog (0.4) or = 2.5 or = 3.5
[ HIn ] [ HIn ] [ HIn ] [ HIn ]
Per cent of HIn = (1/3.5) (100) = 28.6%
78. HIn H+ + In–; Since pH < pK°HIn, the solution contains larger H+ than that obtained from ionization of HIn
alone. Hence, the above equilibrium is shift to left, thus more of unionized indicator is present.
81. Phosphorus is more electronegative than arsenic.
82. Strongest acid amongst HClO, HClO2, HClO3 and HClO4 is HClO4. Hence, the conjugate base ClO–4 will be the
weakest.
86. :NH3 contains a lone pair of electrons. It can be donated and hence it acts as a Lewis base.
87. HCN is the weakest acid. The next stronger acid is CH3COOH. Out of HCl and HClO4, the former is a weaker
acid.
88. Weaker the acid, stronger the conjugate base. The order of acid strength is CH3COOH > H2S > NH +4 > HCO3–.
Hence, the conjugate base follows the reverse order.
89. The weaker the acid, stronger the conjugate base. The order of acid strength is CH3CH3 < NH3 < HC ∫ CH < H2O.
Hence, the conjugate base follows the order CH3CH2– > NH2– > HC∫C– > OH–.
90. The correct order of acid strength is CH3COOH < HCl < HClO4. Hence, the conjugate base follows the order
CH3COO– > Cl– > ClO 4–.
91. The weakest acid amongst H2O, HF, NH3 and CH4 is CH4. Hence, the strongest base will be CH 3–.
92. CH3COOH CH3COO– + H+
c(1 – a) ca ca
[CH3COO - ] [H + ] (ca ) (ca )
K a= =  ca 2; a= Ka / c = 1.8 ¥ 10-5 / 0.05 = 0.019
[CH3COOH] c(1 - a )
93. [H+] = ca = (0.05 M) (0.019) = 9.5 ¥ 10–4 M
94. pH = – log {[H+]/M} = – log (9.5 ¥ 10–4) 3.02
95. c = 0.005 M; a= K a / c = ( 1.8 ¥ 10-5 / 0.005 ) = 0.06
96. pH = – log {(0.06) (0.005)} = 3.53
97. [H+] = 0.00095 M/2 = 0.000425 M
CH3COOH CH3COO– + H+
[CH3COO - ] [H + ] [H + ]2
K a= 
[CH3COOH] [CH3COOH ]0
[CH3COOH]0= [H+]2/Ka = (0.000425 M)2/(1.8 ¥ 10–5 M) = 0.01 M
0.05 M
Dilution factor= =5
0.01 M
98. pH = 1.5 ¥ 3.02 = 4.53
[H+]= (10–pH) M = 2.95 ¥ 10–5 M; c = [H+]2/Ka = (2.95 ¥ 10–5 M)2/(1.8 ¥ 10–5 M) = 4.84 ¥ 10–5 M
0.05 M
Dilution factor = = 1033.3
4.84 ¥ 10-5 M
99. M(CH3COONa) = 82 g mol–1
n(CH3COONa)= m/M = 0.82 g / 82 g mol–1 = 0.01 mol
c(CH3COONa)= n/V = 0.01 mol/0.1 L = 0.1 M
pH = pKa + log [salt]/[acid] = –log (1.8 ¥ 10–5) + log (0.1/0.1) = 4.75
100. pH of buffer solution does not change on dilution.
101. Buffer capacity decreases on diluting the solution.
Chemical and Ionic Equilibria 7.71

102. Buffer range, pH = pKa ± 1. Thus, buffer range is from 3.75 to 5.75.
103. On adding 0.002 mol HCl, 0.002 mol of sodium acetate is changed into acetic acid.
n(CH3COONa) = (0.1 M) (0.1 dm3) – 0.002 mol = 0.008 mol
n(CH3COOH) = (0.1 M) (0.1 dm3) + 0.002 mol = 0.012 mol
0.008 ˆ
= 4.75 + log ÊÁ
n (salt)
pH = pK°a + log ˜ = 4.75 – 0.176 = 4.57
n (acid) Ë 0.012 ¯
104. In the solution of Q.100, [CH3COOH] = [CH3COONa] = 0.01 M
n(CH3COOH) = n(CH3COONa) = (0.01 M) (0.1 dm3) = 0.001 mol
On adding 0.002 mol HCl, the entire CH3COONa is replaced by CH3COOH. Thus, solution contains 0.002 mol
acetic acid and 0.001 mol HCl. The solution concentrations are
[CH3COOH] = 0.02 M and [HCl] = 0.01 M
The major source of H+ in the solution will be HCl.
Hence, pH = – log (0.01) = 2
105. For a salt formed from strong base and weak acid, pH of the solution is given by
1
pH = [pK°w + pK°a + log (c/c°)] (where c° = 1 M)
2
106. For a salt formed from weak base and strong acid, pH of the solution is given by
1
pH = [pK °w – pK °b – log (c/c°)] (where c° = 1 M)
2
107. For a salt formed from weak acid and weak base, pH of the solution is given by
1
pH = (pK °w + pK °a – pK °b)
2
108. For a salt such as NaHCO3, pH of the solution is given by
1
pH = (pK°a1 + pK °a2)
2
where Ka1 and Ka2 are the ionization constants of H2CO3 and HCO3–, respectively.
109. For a salt formed from a weak acid and a weak base, pH of the solution is given by
1
pH = (pK °w + pK °a – pK °b)
2
110. The solubility of an amphoteric salt is larger in acidic and alkaline media as compared to the neutral medium.
Ka1/Ka2 ≥ 103
112. A xB y y+ x–
xA + yB . If s is the solubility of the salt, then in solution, we will have
A xB y xAy+ + yB x–; Ksp = [Ay+ ]x [Bx– ] y = (xs)x (ys)y; This gives s = (Ksp /xx yy )1/(x + y).
(xs) (ys)

MULTIPLE CHOICE QUESTIONS FROM AIEEE AND JEE MAIN

(a) PH3 (b) H2O (c) H2S (d) NH3 [2003]


2. When rain is accompanied by a thunderstorm the collected rain water will have a pH value
(a) which depends on the amount of dust in air
(b) slightly lower than that of rain water without thunderstorm
(c) slightly higher than that when the thunderstorm is not there
[2003]
7.72 Complete Chemistry—JEE Main

3. The solubility of a sparingly soluble salt AB2 in water is 1.0 ¥ 10–5 M. Its solubility product will be
(a) 1.0 ¥ 10–10 M3 (b) 4 ¥ 10–15 M3 (c) 4 ¥ 10–10 M3 (d) 1 ¥ 10–15 M3 [2003]
4. The conjugate base of H2PO4– is
(a) PO3–4 (b) P2O5 (c) H3PO4 (d) HPO2–4 [2004]
–1
5. The molar solubility (in mol L ) of a sparingly soluble salt MX4 is s. The corresponding solubility product is Ksp.
The solubility s is given in terms of Ksp by the relation
(a) s = (Ksp/128)1/4 (b) s = (128 Ksp)1/4 (c) s = (256 Ksp)1/5 (d) s = (Ksp/256)1/5 [2004]
6. Hydrogen-ion concentration in mol/L in a solution of pH = 5.4 will be
(a) 3.68 ¥ 10–6 (b) 3.98 ¥ 10–6 (c) 3.98 ¥ 108 (d) 3.88 ¥ 106 [2005]
7. The standard solubility product of a salt having general formula MX2 in water 4 ¥ 10 . The concentration of M2+
–12

ions in the aqueous solution of the salt is


(a) 1.6 ¥ 10–4 M (b) 4.0 ¥ 10–6 M (c) 2.0 ¥ 10–6 M (d) 1.0 ¥ 10–4 M [2005]
8. What is the conjugate base of OH ? –

(a) O2 (b) H2O (c) O (d) O2– [2005]


9. The pK°a of a weak acid (HA) is 4.5. The pOH of an aqueous buffered solution of HA in which 50% of the acid is
ionized is
(a) 4.5 (b) 2.5 (c) 9.5 (d) 7.0 [2007]
10. In a saturated solution of the sparingly soluble electrolyte AgIO3 (relative molecular mass = 283) the equilibrium
which sets in is
AgIO3 (s)  Ag + (aq ) + IO3- (aq )
If the standard solubility product constant K°sp of AgIO3 at a given temperature is 1.0 ¥ 10–8, what is the mass of
AgIO3 contained in 100 mL of its saturated solution?
(a) 28.3 ¥ 10–2 g (b) 2.83 ¥ 10–3 g (c) 1.0 ¥ 10–7 g (d) 1.0 ¥ 10–4 g [2007]
2A are 1.0 ¥ 10 and 5.0 ¥ 10
–5 –10 respectively,
overall standard dissociation constant of acid will be
(a) 5.0 ¥ 10–5 (b) 5.0 ¥ 1015 (c) 5.0 ¥ 10–15 (d) 0.2 ¥ 1015 [2007]
12. The pK°a of a weak acid, HA, is 4.80. The pK°b of a weak base, BOH, is 4.78. The pH of an aqueous solution of
the corresponding salt, BA, will be
(a) 9.22 (b) 9.58 (c) 4.79 (d) 7.01 [2008]
13. Four species are listed below.
(i) HCO–3, (ii) H3O+, (iii) HSO–4 , (iv) HSO3F
Which one of the following is the correct sequence of their acid strength?
(a) (iii) < (i) < (iv) < (ii) (b) (iv) < (ii) < (iii) < (i)
(c) (ii) < (iii) < (i) < (iv) (d) (i) < (iii) < (ii) < (iv) [2008]
14. Solid Ba(NO3)2 is gradually dissolved in a 1.0 ¥ 10 M Na2CO3. solution. At what concentration of Ba will a
–4 2+

precipitate begin to form? (Given: Ksp = 5.1 ¥ 10–9 M2 for BaCO3.)


(a) 8.1 ¥ 10–8 M (b) 8.1 ¥ 10–7 M (c) 4.1 ¥ 10–5 M (d) 5.1 ¥ 10–5 M [2009]
15. In aqueous solution the ionization constants for carbonic acid are K °1 = 4.2 ¥ 10 and K °2 = 4.8 ¥ 10–11. Select the
–7

correct statement for a saturated 0.34 M solution of the carbonic acid.


(a) The concentration of H+ is double that of CO2– 3
(b) The concentration of CO23– is 0.034 M
(c) The concentration of CO23– is greater than that of HCO–3
(d) The concentrations of H+ and HCO–3 are approximately equal.
16. Solubility product of silver bromide is 5.0 ¥ 10–13 M2. The quantity of potassium bromide (molar mass taken as
120 g mol–1) to be added to 1 litre of 0.05 M solution of silver nitrate to start the precipitation of AgBr is
(a) 5.0 ¥ 10–8 g (b) 1.2 ¥ 10–10 g (c) 1.2 ¥ 10–9 g (d) 6.2 ¥ 10–5 g [2010]
Chemical and Ionic Equilibria 7.73

17. At 25 °C, the solubility product of Mg(OH)2 is 1.0 ¥ 10–11 M3. At which pH will Mg2+ ions start precipitating in
the form of Mg(OH)2 from a solution of 0.001 M Mg2+ ions?
(a) 8 (b) 9 (c) 10 (d) 11 [2010]
18. The correct order of increasing basicity of the given conjugate bases (R ∫ CH3) is
(a) RCOO– < HC ∫ C– < NH2– < R– (b) RCOO– < HC ∫ C– < R– < NH2–
(c) R– < HC ∫ C– < RCOO– < NH2– (d) RCOO– < NH 2– < HC ∫ C– < R– [2010]

19. Three reactions involving H2PO 4 are given below?
(i) H3PO4 + H2O Æ H3O+ + H2PO–4
(ii) H2PO–4 + H2O Æ HPO2–
4 + H 3O
+

(iii) H2PO4– + OH– Æ H3PO4 + O2–


In which of the above reactions does H2PO4– act as an acid?
(a) (i) only (b) (ii) only (c) (i) and (ii) only (d) (iii) only [2010]
20. The K°sp for Cr(OH)3 is 1.6 ¥ 10 –30
. The molar solubility of this compound in water is

(a) ( 2 1.6 ¥ 10-30 )M (b) ( 4 1.6 ¥ 10-30 )M (c) ( 4 1.6 ¥ 10-30 / 27)M (d) (1.6 ¥ 10–30/27)M
21. An acid HA ionizes as HA H+ + A–. The pH of 1.0 M solution is 5. Its dissociation constant would be
(a) 1 ¥ 10–10 M (b) 5 M (c) 5 ¥ 10–8 M (d) 1 ¥ 10–7 M
22. The pH of a 0.1 molar solution of the acid HQ is 3. The value of the ionization constant Ka of this acid is
(a) 3 ¥ 10–1 M (b) 1 ¥ 10–3 M (c) 1 ¥ 10–5 M (d) 1 ¥ 10–7 M [2013]
23. How many liters of water must be added to 1 L of an aqueous solution of HCl with a pH of 1 to create an aqueous
solution with pH of 2?
(a) 0.1 L (b) 0.9 L (c) 2.0 L (d) 9.0 L [2013]
24. Among the following oxoacids, the correct order of acid strength is
(a) HClO2 > HClO4 > HClO3 > HOCl (b) HOCl > HClO2 > HClO3 > HClO4
(c) HClO4 > HOCl > HClO2 > HClO3 (d) HClO4 > HClO3 > HClO2 > HClO [2014]
25. Assuming that the degree of hydrolysis is small, the pH of 0.1 M solution of sodium acetate (K°a = 1.0 ¥ 10–5)
will be
(a) 5.0 (b) 6.0 (c) 8.0 (d) 9.0 [2014, online]
26. Zirconium phosphate [Zr3(PO4)4] dissociates into three zirconium cations of charge +4 and four phosphate anions
of charge –3. If molar solubility of zirconium phosphate is denoted by s and the solubility product by Ksp then
which of the following relationship between s and Ksp is correct?
(a) s = Ksp/(6912)1/7 (b) s = (Ksp/144)1/7 (c) s = [Ksp/6912]1/7 (d) s = (Ksp/6912)7
[2014, online]

ANSWERS
1. (b) 2. (b) 3. (b) 4. (d) 5. (d) 6. (b)
7. (d) 8. (d) 9. (c) 10. (b) 11. (c) 12. (d)
13. (d) 14. (d) 15. (d) 16. (c) 17. (c) 18. (a)
19. (b) 20. (c) 21. (a) 22. (c) 23. (d) 24. (d)
25. (d) 26. (c)
7.74 Complete Chemistry—JEE Main

HINTS AND SOLUTIONS


3. Ksp = [A2+][B–]2 = (1.0 ¥ 10–5 M) (2 ¥ 1.0 ¥ 10–5 M)2 = 4 ¥ 10–15 M3
4. We have to consider the reaction H2PO–4 HPO2–4 + H+
acid base
5. We have MX4 M4+ + 4X–
s 4s
Ksp = [M4+] [X–]4 = (s) (4s)4 = 256 s5; Hence s = (Ksp/256)1/5
6. We have pH = –log{[H+]/M}. Hence [H+] = 10–pH M = 10–5.4 M = 3.98 ¥ 10–6 M
7. For the MX2 salt, we have
MX2 M2+ + 2X– Ksp = (s) (2s)2 = 4s3
s 2s
1/ 3 1/ 3
Ê Ksp ˆ Ê 4 ¥ 10-12 ˆ
Hence, s= Á = M˜ = 10-4 M
Ë 4 ˜¯ ÁË 4 ¯

8. The reaction to be considered is OH– Æ O2– + H+


acid base
[salt ]
9. From Henderson’s equation, we get pH = pKa + log
[acid]
50% of acid ionization implies that [salt] = [acid]
Hence, pH = pKa, and pOH = pKw – pH = 14 – 4.5 = 9.5
10. For AgIO3(s) Ag+(aq) + IO–3 (aq), we have Ksp = [Ag+] [IO–3 ] = s2
Hence s= Ksp = 1.0 ¥ 10-8 M 2 = 1.0 ¥ 10-4 M
Mass of AgIO3 in 100 mL solution is m = (1.0 ¥ 10–4 mol L–1) (283 g mol–1) (1.0 L) = 2.83 ¥ 10–3 g
11. The overall standard dissociation constant of the acid is
K°a = K°a1 K°a2 = (1.0 ¥ 10–5) (5.0 ¥ 10–10) = 5.0 ¥ 10–15
12. For the salt formed from a weak acid and a weak base, the pH of the solution is given by
1
pH = (pK°w + pK°a – pK°b)
2
1
Hence, at 25 ° C, we will have pH = (14 + 4.80 – 4.78) = 7.01
2
13. HCO–3 is the weakest acid. This is followed by HSO–4. The strongest acid in aqueous solution is H3O+. The choice
d indicates that the given acids follow the correct order.
14. We have
BaCO3(s) Ba2+(aq) + CO32– (aq) ;Ksp = [Ba2+] [CO2–3 ]
2+
The minimum [Ba ] required to start precipitation of BaCO3 is
Ksp 5.1 ¥ 10-9 M 2
[Ba2+] = = = 5.1 ¥ 10-5 M
[CO32- ] 1.0 ¥ 10-4 M
15. Carbonic acid ionizes as
(i) H2CO3 H+ + HCO3– K°1 = 4.2 ¥ 10–7
(ii) HCO3– +
H + CO3 2– K°2 = 4.8 ¥ 10–11
Since K°2 << K°1, the concentrations of H+ and HCO–3 will be apporoximately equal as governed by Eq. (i).
16. AgBr(s) Ag+(aq) + Br–(aq); Ksp(AgBr) = [Ag+][Br–] = 5.0 ¥ 10–13 M2
It is given that [Ag+] = 0.05 M (from AgNO3 solution);
Ksp (AgBr ) 5.0 ¥ 10-13 M 2
Hence, [Br–] = = = 1 ¥ 10-11 M
[Ag + ] 0.05 M
Mass of KBr to be added m = [KBr] MKBr = (1 ¥ 10–11 M) (120 g mol–1) = 1.2 ¥ 10–9 g L–1
Chemical and Ionic Equilibria 7.75

1/ 2
Ê Ksp ˆ Ê 1.0 ¥ 10-11 M 3 ˆ -4
17. [OH–] = Á
Ë [Mg 2+ ]˜¯

Ë 10-3 M ˜¯ = 1.0 ¥ 10 M pOH = –log(10–4) = 4; pH = 14 – 4 = 10

18. The conjugate acids of the given bases are RCOOH, HC∫CH, NH3 and CH4. The order of acidity of these acids
is RCOOH > CH∫CH > NH3 > CH4. Their conjugate bases will follow the reverse order, i.e.
RCOO– < HC ∫ C– < NH2– < CH–3.
19. The reaction (ii) provides H3O+. Hence, in this reaction H2PO4– acts as an acid.
20. We have Cr(OH)3 æÆ Cr3+ + 3OH–
s 3s
Hence, Ksp = [Cr ] [OH ] = (s) (3s)3 = 27s4
3+ – 3

Ksp / 27 = 1.6 ¥ 10-30 M 4 / 27


4
Thus s= 4

[H + ][A - ] [H + ]2
21. HA H+ + A– Ka = 
[HA] [HA]0
(10-5 M ) 2
For pH = 5, [H+] = 10–5 M Hence, Ka = = 10-10 M
1.0 M
22. For a weak acid, we have
[H + ][Q - ]
HQ H + + Q –; Ka =
[HQ]
[H + ]2 (1 ¥ 10-3 M )2
Also [H+] [Q–] and [HQ] = [HQ]0. Thus Ka = = = 1 ¥ 10-5 M
[HQ]0 0.1 M
23. pH = 1 fi [H+] = 10–1 M
pH = 2 fi [H+] = 10–2 M
To change the solution having 10–1 M H+ ions to 10–2 M H+, the solution will have to be diluted 10 times. Thus
9 L of water will have to be added to 1 L of the original solution.
24. The larger the number of oxygen atoms attached to chlorine, greater the electron pull towards oxygen, hence, more
easy to remove hydrogen from the acid. The given acids are
Cl – OH; OCl – OH; O2Cl – OH; O3Cl – OH
(HOCl) (HClO2) (HClO3) HClO4
25. The acetate ion undergoes hydrolysis as
CH3COO– + H2O CH3COOH + OH–
[CH3COOH ][OH - ] [H + ][OH - ] Kw 1.0 ¥ 10-14 M 2
Kh = ∫ = = = 1.0 ¥ 10-9 M
[CH3COO - ] [CH3COO - ][H + ]/[CH3COOH] K a 1.0 ¥ 10-5 M
Also [CH3COOH] = [OH–].
[OH - ]2
Hence, Kh = or [OH - ] = (K h [CH3COOH]0 )1/2
[CH3COOH ]0
or [OH ] = [(1.0 ¥ 10–9 M) (0.1M)]1/2 = 1.0 ¥ 10–5 M

[H+] = Kw / [OH–] = 1.0 ¥ 10–14 M2/(1.0 ¥ 10–5 M) = 1.0 ¥ 10–9 M


pH = – log ([H+]/M) = 9
26. Zr3(PO4)4 3Zr4+ + 4 PO3–4
(3s) (4s)
Ksp = [Zr4+]3 [PO43–]4 = (3s)3 (4s)4 = 6912 s7
Hence, s = (Ksp/6912)1/7
8
Redox Reactions and
Electrochemistry

UNIT 1 REDOX REACTIONS AND ELECTROLYSIS

SECTION 1 Redox Reactions

For example

æÆ Zn
Oxidised Reduced

Concept of Oxidation Number

4 2
4
8.2 Complete Chemistry—JEE Main

2 4 8

4 3

2 2
2O 2 2O 2 2 compound OF2.

2O 3 x x
2 4 x x
2 2 x x
2 4O 6 x x
2 x x
Explaining Redox Reaction in Terms of Concept of Oxidation Number

Zn

oxidizing agent
decrease
reducing agent increase

Balancing Redox Reactions

Oxidation-State Change Method

2
3 2O 3 2
Redox Reactions and Electrochemistry 8.3

Step 3 2O 3 2
Step 3 2O 2 3 2
Step 3

Step 3 2O 3 2 2O

3 2O 3 2 2O
Ion-Electron or Half-Equation Method

reduction
Step 3ææææ Æ
oxidation
2O ææææ Æ
Step 3

2O 2
Step 3 2 2O
2 2O
Step 3 3e 2O

2 2 2e
Step 3 2O] × 2

2 2 ]¥3
Step 3 2 2O

Comment on Fractional Oxidation Number 2 4O 6 is 2.5. The


2 4O 6

O O

Equivalent Mass of a substance† O O


expressed as Fig. 1

( 1n substance) that is Molar mass of ÊÁË substanceˆ˜¯ =


1
n
Molar mass of substance
n
.
8.4 Complete Chemistry—JEE Main

Molar mass of a substance


n eq mol-1
n
Unit of molar mass g mol-1
eq mol-1 eq mol-1
Illustrations 2 2O 7 in its oxidation reaction in acidic medium.

2O 7 Æ 2
Molar mass of K 2 Cr2 O7 294 g mol-1
2O 7
2
6 eq mol-1 6 eq mol-1
4 in its oxidation reaction.
In acidic medium
4 Æ 2
Molar mass of KMnO 4
4
5 eq mol-1
In weakly acidic, alkaline or neutral medium
4 Æ 2 2O

Molar mass of KMnO 4


4
3 eq mol-1
In alkaline medium
4 Æ 4
Molar mass of KMnO 4
4
1 eq mol-1

MULTIPLE CHOICE QUESTIONS ON SECTION 1

3 3 3 2 2 2O 4 2

2O 3 Æ 3 2 4 2O 7

2O 4 Æ 2 2 3 Æ 2 4 2O
2 is

2 5 x 2 Æ 3 2 y 2O x and y

4 Æ 4 2 2O
are

8 2O 3 4O 6 2O 8 8 2O 3 4O 8 4O 6
8 2O 8 2O 3 4O 6 8 2O 8 4O 6 2O 3
Redox Reactions and Electrochemistry 8.5

2 2O 3 ◊ 2 2 2O 3 2 Æ 2 4O 6

2 2O 5

O O

3 3O

6] 6] 6] 4
2 2 2 4
4 2 6 2 4

4 2 6 2 4 4 2 4 2 6]
2 6 2 6 4 2 6 4 2 6]

ANSWERS

HINTS AND SOLUTIONS


3

x x
2 and

4 Æ 4 2 2O
8 2O 3 4O 6
O O

2O 8 O O O O

O O
8.6 Complete Chemistry—JEE Main

2 ¥ Molar mass of Na 2S2 O3 ◊ 5H 2 O


2O 3 Æ 4O 6 2 2O 3◊ 2
2 eq mol-1
8. The superoxide is O 2
O O

2 2O 5 is

O
2O2 is O O

x x
2 2 4
4 2 4 2 6

MULTIPLE CHOICE QUESTIONS FROM AIEEE AND JEE MAIN

dichromate solution is
[2005]
2 4

5 2 4 Æ 3 2 2 2 4 Æ l2 2 2O

2 2 4 Æ 4 2 2 4 Æ 4 [2006]
3

[2011]
2

[2011]

2O 2
4 2O 7 3 [2014]
2 3 2 Æ 4
statements

2 3
2 3 [2014]

2O 7 2O 4 Æ 2
[2014]

4 6] 2O 3 2 2 [2014]
Redox Reactions and Electrochemistry 8.7

3 4 2] is
[2005]
D rG
2 Æ D rG
2 Æ D rG

[2011]

ANSWERS

HINTS AND SOLUTIONS

2O 7 Æ 2 2
2 4 2 4 2 2 4 and
4 2
2O 7 Æ 2
2O 7 ∫ 2 2O 7 ∫
n VM
2 2O 7 n n/
Ê 0.45 molˆ (294 g mol-1 ) = 22.05 g
2 2O 7 m n Mm ÁË ˜¯
6
2

2O æÆ 2 O H

4 2O 2
4 2O 2 Æ 2 2

2O 2 3 2O 7

3 2O 2 Æ 4 2O

2O 2 Æ 2O 2

2O 7 2O 2 Æ 5 2O
5
2.
5 Æ 2 2

2 3 2 æÆ 4
8.8 Complete Chemistry—JEE Main

2O 7

2O 7 Æ 2O

4 6] 2O 3 2 2

x x x
D rG
D rG

SECTION 2 Electrolytic Cell

Fig. 2 Electrolytic cell


Table 1
Cathode Anode

Anions
Reduction Oxidation

Working of an Electrolytic cell


Redox Reactions and Electrochemistry 8.9

Electrode processes
Cathodic Process

cathodic processes take place in the order decreasing reduction potential.


Anodic Process

anodic processes take place in the order of decreasing oxidation potential (i.e. increasing reduction potential).
Illustration Electrolysis of acidic solution of CuSO4

Cathodic Process
Æ E
Æ 2 E

Anodic Process
Æ ; Eoxid
4 Æ 2O 8 ; Eoxid
Æ O2 ; Eoxid

Comment
2

A Few Examples of Electrolysis

Table 2 Electrolysis of Some of the Aqueous Solutions of Salts


Solution Electrodes Cathodic reaction Anodic reaction

2 Æ 2 Æ 2

2 4 Æ Æ O2
2O
2 2
2
4 Æ Æ O2
2O 2
2 Æ Æ 2

4 Æ Æ

Faraday’s Laws of Electrolysis

The mass of a chemical substance involved at an electrode reaction is directly proportional to the amount of
current passed through the cell.
2. The masses of different substances produced by a given amount of current are proportional to the equivalent
masses of the substances.†


8.10 Complete Chemistry—JEE Main

It M B
mB
F ne / nB
mB MB I t and n e
and nB
reaction at the electrode.
Value of Electronic Charge

Æ
¥ 23
mol

ions
2
1
Cl Æ Cl2 (g) + e-
-
2

(96 487 C mol-1 )


e= = 1.602 ¥ 10-19 C
( 6.022 ¥ 1023 mol-1 )
Illustrations
4
F .
Æ Thus nB ne
It M (2.5 A)(50 ¥ 60 s) (63.5 g mol-1 )
m =
F ne (96500 C mol-1 ) 2

F .
Q It ne nB
Ê 96500 C mol-1 ˆ
m ÊÁ ˆ˜ n e = (12.5 g) Á
F
19300 C ˜¯
M Ë It ¯ Ë

M F .
nB
-1
Ê It ˆ Ê M ˆ (0.01 A)(3 ¥ 60 ¥ 60 s) Ê 192 g mol ˆ
ne ÁË ˜¯ ÁË ˜¯ =
F m (96500 C mol-1 ) ÁË 0.0715 g ˜¯

MULTIPLE CHOICE QUESTIONS ON SECTION 2

† F.
F
Redox Reactions and Electrochemistry 8.11

4 F
mol M

2
F F F F

3 4
M
and M

Æ 2 ; E°oxd
1
Æ 2 ; E°oxd
2
Æ E°oxd

>

Æ E
Au Æ Au; E
Æ E

> Au > Au
2

3 solution is

ANSWERS

HINTS AND SOLUTIONS


Æ 2 2

m 1 È It M ˘ It 1 (1 A) (30 ¥ 60 s ) 1 9
n = Í ˙ = = mol
M M Î F n e ˚ F n e (96500 C mol-1 ) 2 965
8.12 Complete Chemistry—JEE Main

nVm = ÊÁ molˆ˜ (22.414 mol-1 ) = 0.21 L


9
V
Ë 965 ¯
m It M F ne
F Ên ˆ Ê 96500 C mol-1 ˆ Ê ˆ
m ÊÁ ˆ˜ Á e ˜ = (0.635 g) Á
2
t Ë I ¯ Ë M¯ Ë 10 A ˜ Á -1 ˜
¯ Ë 63.5 g mol ¯
m 20.0 g
n= = = 0.5 mol ion r to produce
M 40 g mol-1
the ¥ F, F.
m It M Fne
mFn e (1 g)(96500 C mol-1 )(2)
M = = 64.33 g mol-1
It (10 A)(300 s)
m It M Fne
ItM (10 A)(160.83 s)(135 g mol-1 )
ne = =3
Fm (96500 C mol-1 )(0.75 g)
m It F M ne
mAg Ê M Ag ˆ Ê n Cu ˆ Ê 108 g mol-1 ˆ Ê 2 ˆ
Á n ˜ ÁË M ˜¯ ; m ÁË ˜ Á -1 ˜
(0.2 g)
mCu Ë Ag ¯ Cu 1 ¯ Ë 63.5 g mol ¯

n e Fm n e F ( rV ) (1)(96500 C mol-1 )(10.5 g cm -3 )(100 ¥ 10-2 cm3 )


Q It = =
M M (108 g mol-1 )

2 Æ 2 2
2
2 mol H 2 O(l) 4
2 ¥ 2 mol gases = mol H 2 O
3 mol gases 3

2 Æ 2
Æ 2
2F H–
2 mol OH - Ê 2 mol OH - ˆ
n ¥Q =Á -1 ˜
[(0.257 A)(25 ¥ 60 s)] ¥ mol
2F Ë 2 ¥ 96500 C mol ¯

MULTIPLE CHOICE QUESTIONS ON THE ENTIRE UNIT

Electrolyte Cells and Electrolysis

I Qt I Q/t I Qt I t/Q
Redox Reactions and Electrochemistry 8.13

4
4 2O 2 4 2

2O O2 2

2O O2 2

4 2O 2 4 2

2O O2 2
not

not

not

and anode are


2 2 2 2 2 2 2

2 at anode
4 2 at anode
3
3 Æ 2 2 3 Æ 2

2O Æ 2 Æ
2 4
2O 8 4 at anode
Faraday Laws of Electrolysis

QM ItF | n e | It | n e |
m m m m ItMF/ | e|
F | ne | M MF
8.14 Complete Chemistry—JEE Main

F NA F F¥ NA NA F

4 solution is

3
3 2.

2 2

3 4

Redox Reactions

2O 2 2 2 2O 7 4 2O

2 2 3 2 3 2 2O

3 2O 3 3 2 2 2F 2

3 2 4 2 3 2 2 3

4 4 4 2 2O 7

ANSWERS
Redox Reactions and Electrochemistry 8.15

HINTS AND SOLUTIONS

2O.

2 2 4 .
2 4 4

2O Æ 2 .
Æ
4 Æ 2O 8

m F | n e | (V r ) F | n e | (1 cm3 ) (8.94 g cm -3 ) (96500 C mol-1 ) 2


Q =
M M 63.5 g mol-1
m Q 9.65 ¥ 3 ¥ 60 ¥ 60
= = mol
M F | ne | 96500 ¥ 2
(0.5 dm3 ) (2 mol dm -3 ) - 0.54 mol
0.5 dm3
m F | n e | (V r ) F | n e | (80 ¥ 0.005 ¥ 10.5 g) (96500 C mol-1 ) (1)
t =
IM IM (3 A) (108 g mol-1 )

1Ê 1 mol ˆ 1 1

Ë 96500 C ˜¯ 2 2

1
2 2 2O 2 O2
2 2.

Æ
¥
Q It ¥5¥
Q 0.75 ¥ 5 ¥ 60
n mol
F 96500
2 Æ 2
2 n
2
1 Ê 0.75 ¥ 5 ¥ 60
V Á molˆ˜
4Ë 96500 ¯
1.08 g
= 0.01 mol
108 g mol-1
Ê 63.5 g mol- 1 ˆ
ÁË 2 ˜¯
8.16 Complete Chemistry—JEE Main

MULTIPLE CHOICE QUESTIONS FROM AIEEE AND JEE MAIN

[2003]
not
4
2 4 4

¥ [2003]

are
[2005]

cathodic reaction is Al Æ

¥ 4 ¥ ¥ 7 ¥ 7 [2005]

[2014]
2O 2
2O 2 Æ2 2O 2O 2 Æ O2 2
2O 2 Æ 2O 2O 2 Æ O2 2 2O
[2014]

F
[2014]

NA
NA NA NA NA [2014]

[2016, online]

[2015]

ANSWERS
Redox Reactions and Electrochemistry 8.17

HINTS AND SOLUTIONS

m Q F M/ n e
mF n e (5.12 ¥ 103 g)(96500 C mol-1 )(3) 5.12 ¥ 9.65
Q = = ¥ 107 C = 5.49 ¥ 107 C
M (27 g mol-1 ) 9

2O 2

It (10.0 A)(2 ¥ 60 ¥ 60 s)
=
F (96500 C mol-1 )

0.746 mol
3
0.250 mol
m 6.35 g
n = = 0.10 mol
M 63.5 g mol -1

NA NA

Anode
CH 2 COO - CH 2
-
Ω Æ ΩΩ (g ) + 2 CO 2 (g ) + 2e
ææ
CH2COO - CH2

2 2
2 4 2 2

Æ
8.18 Complete Chemistry—JEE Main

UNIT 2 Electrolytic Conduction

Ohm’s Law
I ER
I E and R

Fig. 1
Resistivity of the Solution l
t A

R μ ÊÁ ˆ˜ R = r ÊÁ ˆ˜
l l
or
Ë A¯ Ë A¯
r
unit of A ˆ
r R ÊÁ Wm
Ë unit of l ˜¯
W R.
Conductance and Conductivity G
G R
k

k
r
k is W m . The unit W . k

= Ê l ˆ = GÊ l ˆ
k Á ˜ ÁË ˜¯
r R Ë A¯ A
lA Kcell
k G Kcell

Equivalent Conductivity


V


Redox Reactions and Electrochemistry 8.19

A l V Al.

Ê Aˆ Ê Veq / eq -1 ˆ
G k Á ˜ =k Á ˜ or Gl 2 = k (Veq / eq -1 )
Ël¯ Ë l2 ¯
Gl2 †
L
L Gl 2
kV
L k V 3 2

c V c
L k c

of the substance, when placed between two parallel electrodes which are at a unit distance apart and large enough
to contain between them the whole volume.
Molar Conductivity Vm
d
Lm k Vm k c
c Lm is 2 mol . The name molar conductivity follows

from the fact that it is conductivity divided by molar concentration.


Relationship Between Lm and Leq n z n ˙z

c n z cm
k k k / cm Lm
L = -1
= -1
=
ceq (n + z+ eq mol )cm n + z+ eq mol n + z+ eq mol-1

Variation of Conductivity of a Solution

Variation of Molar Conductivity of a Solution

Vm c
on dilution.
Variation for Strong Electrolyte

Lm L•m - b c
b L•m
c= .
Kohlrausch equation.

Variation for Weak Electrolyte

Fig. 2

† zB zB is
2 mol .
8.20 Complete Chemistry—JEE Main

Lc
a
L•
Lc L•
Kohlrausch’s Law of Independent Migration of Ions

aÆ cÆ

L•m = n + l+• + n - l-•


n and n l+• and l-• are the

Application of Kohlrausch’s Law Lm


L•m
Illustration
L•m L•m L•m L•m
l•m l•m l•m l•m l•m l•m
l•m l•m L•m
L•m 2
mol ; L•m 2
mol L•m 2
mol
Thus L•m 2 mol 2 mol
Limiting Molar Conductivities of Ions

n + l+• n - l -•
t• and t- =
L •m L •m
t and t l• and l•.

Illustration Molar conductivities of H+ and Cl– Ions


t t t L•m 2
mol
l•m t L•m ¥ 2
mol 2
mol
l•m t L•m ¥ mol 2 2 mol

l •m ( 1x A x+ ), l •m ( 1y B y - ) and so on†

l•m x xlm ( 1x A x+ ) and lm• (B y - ) = ylm ( 1y B y - )

For Example
lm• ( 13 Al3+ ) 2 mol then
l•m 3lm• ( 13 Al3+ ) = 3 ¥ 63 2
mol 2
mol

† •
(
l m 1x A
x+
)
• 1 y-
and l m B
y ( )
1 A x + and 1 B y -
x y
Redox Reactions and Electrochemistry 8.21

Illustration Molar and Equivalent Conductivities of Al2 (SO4)3

L•m 2 4 3 lm• lm• 4

2 ÈÎ3lm• ( 13 Al3+ )˘˚ + 3 ÈÎ2lm• ( 12 SO 42- )˘˚

6 ÈÎlm• ( 13 Al3+ ) + lm• ( 12 SO 42- )˘˚ 2 mol 2 mol ¥ 2 mol


2 mol

∫ L•m ÊÁ Al2 (SO 4 )3 ˆ˜ = Lm (Al2 (SO 4 )3 )


1 1
L• 2 mol
2 4 3 Ë6 ¯ 6
Comments on Some Values of Molar Ionic Conductivities
1. Abnormal High Molar Ionic Conductivities of H+ and OH– :
lm• 2 •
mol and lm 2 mol

Fig. 3
2. The Molar Conductivities of Alkali-Metal Ions

lm• 2
mol

Conductometric Titrations

Illustration

Na + OH -
ææææ
-H O
Æ
2
8.22 Complete Chemistry—JEE Main

k k

and

Fig. 4

Fig. 5 Conductometric titration curve of a weak acid Fig. 6 Conductometric titration curve of a mixture of
(e.g., ) versus a strong base (e.g., ) a strong acid (e.g., ) and a weak acid (e.g. )
with a strong base (e.g. )

ion.

MULTIPLE CHOICE QUESTIONS ON UNIT 2

W
the solution is
W cm W cm
¥

2
mol W cm mol
¥ cm2 mol 2
mol

2
¥ 3
W
¥ 2
mol ¥ 2
mol 2
mol 2
mol
W
2
mol
Redox Reactions and Electrochemistry 8.23


lm 2 mol and lm• ( 14 Fe(CN)64- ) = 111.0 2 mol L•m 4 6
2 2 2 2
mol mol mol mol
lm• ( 12 Zn 2+ ) = 52.8 2 mol L•m 3 6] 2
2 mol L•m 6
2 mol 2 mol 2 mol 2 mol

¥ 2 mol
a 2 mol
¥ ¥ ¥ ¥
2 ¥ .
2 mol is
2 2
¥ 3 ¥ 3 ¥ 3 ¥ 3

2 mol
4
m 4
2 mol 2 mol 2 mol 2 mol
Lc and L• c
i
cLc cL2c cL2c cL2•
Ka = Ka = Ka = Ka =
L• - Lc L• - Lc L• (L• - Lc ) Lc (L• - Lc )
represented

L Lc bc L = Lc - b c L Lc bc L = Lc + b c
b is a constant

l• > l• > l• l• > l• > l• l• > l• > l• l• > l• > l•

solution is
2 mol 2 mol 2 mol 2 mol
not

not

4 2
8.24 Complete Chemistry—JEE Main

ANSWERS

HINTS AND SOLUTIONS


k Kcell R W W cm
2. Lm k c ¥ W cm ¥ mol cm3 2 mol
1Ê lˆ Ê 1 ˆ Ê 1.0 cm ˆ
3. k ÁË ˜¯ = ÁË 3 ˜ ¥
R A 7 ¥ 10 W ¯ ÁË 3.0 cm 2 ˜¯
k 4.76 ¥ 10-5 S cm -1
L = ¥ 3 cm mol ¥ 3 cm mol 2 mol
c 0.1 mol dm -3
K cell 0.4 cm -1
4. k = ¥
R 200 W
k 2 ¥ 10 -3 S cm -1
c = ¥ mol cm ¥
Lm 20.0 S cm 2 mol -1
5. L•m 4 6 l•m l•m ( 14 Fe (CN)64- ) ¥ ¥ 2
mol 2
mol
6. Lm 3 6] 2 lm Zn lm 6 3 ¥ 2lm ( 12 Zn ) + 2+
2lm (Fe(CN)36- )
2+
or lm 6
1
2 ÎÈ Lm ( Zn 3 [Fe(CN)6 ]2 ) - 6lm ( 2 Zn ) ˘˚
1 1
2
[922.8 - 6 ¥ 52.8] Scm 2 mol-1 2
mol
7. c
k 5.56 ¥ 10-8 S cm -1 5.56 ¥ 10-8 S cm -1
L = = ¥ 2
mol
c 55.56 mol dm -3 m -3
55.56 ¥ 10-3 mol cm
Lwater 1.0 ¥ 10-6 S cm 2 mol-1
a = ¥
lH• + lOH

- (349.8 + 197.6) S cm 2 mol-1
2 is
k 4.0 ¥ 10 S cm -1 -5
c = ¥ mol cm ¥ ¥ mol dm
L 200 S cm 2 mol -1
2
c 2c
Ksp ] [F ]2 c3 ¥ 3
¥ 3

L•m 4 L•m 4 L•m L•m 2


mol 2
mol
Lc
a
L•

c a a ca

[H + ][A - ] (ca )(ca ) c a 2 c( Lc / L• )2 c L2e


Ka = = =
[HA] c(1 - a ) 1 - a 1 - ( Lc / L• ) L• ( L• - Lc )
L = Lc - b c .

.
Redox Reactions and Electrochemistry 8.25

k 0.00632 S cm -1
L mol dm3 mol 3 2 mol
c 0.05 mol dm -3

4 2
conductance.

MULTIPLE CHOICE QUESTIONS ON THE ENTIRE UNIT

Ohm’s Law

W W W cm W cm2 mol

W W cm W cm2 W cm2 mol


¥ W cm
W

lA Al lA l2 A

2 ¥
cm2
W W W W
W

Equivalent and Molar Conductivites

W W cm W cm2 W cm2 mol


Lm
m k m k m k m k
k c is the molar concentration.

m L n z m n z L m n L z m z L n
n z
8.26 Complete Chemistry—JEE Main

4 W cm2

¥ W cm ¥ W cm ¥ W cm2 ¥ W cm
L• L• L• 2 4 W cm2 L• 2 4

W cm2 W cm2 W cm2 W cm2


Lc 4 W cm2 mol

l• 4 W cm2 mol and l• W cm2 mol


• 2-
L• ( 13 Al3+ )
W cm2 mol and l ( 12 SO 4 ) W cm2 mol L• 2 4 3
W cm2 mol W cm2 mol W cm2 mol W cm2 mol
W . The molar

2 2 2 2
mol mol mol mol
L•m 2
mol L•m 2

mol and L•m 2 mol

4 ¥ 2

mol 4 is
¥ 2 ¥ 2 ¥ 2 ¥ 2

¥ l• 2 mol and l• 2

mol
¥ ¥ ¥ ¥
i
ti li Âi li ti ci li Âi ci li ti ci ki Âi ci ki ti Âi li li

Lm and c Lm and c is linear


Lm and c Lm c is linear

L 2 4 3 lm lm 4 L 2 4 3 lm lm 4
L 2 4 3 lm lm 4 L 2 4 3 lm lm 4

2
L•m 2 L•m 2 L•m 2 4 L•m 2 4
L•m 2 L•m 2 L•m 2 4 L•m 2 4
L•m 2 L•m 2 L•m 2 4 L•m 2 4
L•m 2 L•m 2 L•m 2 4 L•m 2 4
Redox Reactions and Electrochemistry 8.27

ANSWERS

HINTS AND SOLUTIONS


3. Kcell kR ¥ W cm W

Aˆ Ê 1.50 cm 2 ˆ
k ÊÁ
1 1
G ˜ ¥ ÁË 0.5 cm ˜¯ ¥ 2 R =
Ë l¯ G 2.91 ¥ 102 S
K cell 0.25 cm -1
k =
R 25.0 W
È 2.54 -3 ˘
k L c W cm2 Í (159 / 2) ¥ 1000 eq cm ˙ ¥ W cm
Î ˚
L• 2 4 L• L• 2 4 L• W cm2 W cm2
L 11.5 11.5
a •
= =
L ( 73. 4 + 197 .6 ) 271 .0
3+
l• l• ( 13 Al ) ¥ 63 W cm2 mol W cm2 mol
2-
l• 4 l• ( 12 SO 4 ) ¥ W cm2 mol W cm2 mol
l• 2 4 3 l• l• 4 ¥ ¥ W cm2 mol W cm2 mol
K cell 2.0 cm -1
k = W cm
R 200 W
k 0.01 W -1 cm -1
L = W dm3 W cm2 mol
c 0.1 mol dm -3
L•m L•m L•m L•m 2
mol 2
mol
L(HC) 3.83
a = ¥
L•(HC) 383
[H + ][C- ] (ca )(ca )
Ka = ca 2 2
[HC] c(1 - a )
c a ca ca
L kc
k 3.06 ¥ 10-6 S cm -1
c = ¥ cm ¥ ¥ mol dm
L 1.53 S cm 2 mol -1
Ksp 4 4 ¥ mol dm 2
¥ mol2 dm
k 5.54 ¥ 10 -8 S cm -1
a =
cL•m (H 2 O) (55.56 mol dm -3 ) {(349.8 + 197.8) S cm 2 mol -1}

5.54 ¥ 10-8 S cm -1
¥
(55.56 ¥ 10 -3 mol cm -3 ) (547.6 S cm 2 mol -1 )
8.28 Complete Chemistry—JEE Main

Conductivity of ith ion c l


ti = i i
Total conductivity of the solution S i ci li

MULTIPLE CHOICE QUESTIONS FROM AIEEE AND JEE MAINS

Lμ 2
mol
μ
L
2 2 2 2
mol mol mol mol
[2004]
2
3
mol
2 2 2 2
mol mol mol mol
[2005]

[2005]
μ
L and Lμ 2
mol

Lμ L μ 2O Lμ Lμ [2006]

[2006]
W. The

W
¥ 2
mol ¥ 2
mol
¥ 2 mol
¥ 2
mol [2006]
2

L•CH3COONa 2
L• 2

L•
L•C L• 3
L• 2 [2007]
W
W
2 mol ¥ 2 mol ¥ 2 mol 2 mol
[2011]
c Lc and L•
Lc and L•
Lc = L• + B c Lc L• Bc Lc L• Bc Lc = L• - B c
[2014]
W . The resistance
W
2
mol is
¥ 2 ¥ ¥ ¥ 3
[2014]
Redox Reactions and Electrochemistry 8.29

ANSWERS

HINTS AND SOLUTIONS

Lμ Lμ Lμ Lμ 2 mol 2 mol

L•m L•m L•m L•m 2 mol 2 mol

L• L• L• L•
L• L• L•

ion.
R W and k
Kcell kR W

K 129 m -1 k (129 / 520) S m -1


k and L ¥ 2
mol
R 520 W c 0.02 ¥ 103 mol m -3
L• 3 L• 3 L• L•
L•
R r l r Kcell
R
Kcell Rk W
r

K cell 65 m -1
k = 0.25 S m -1
R 260 W
k 0.25 S m -1 0.25 S m -1
L = = ¥ 2
mol
c 0.4 mol dm -3 0.4 mol (10-1 m) -3
Lc and L•
Lc = L• - B c

increases on dilution.
Kcell Rk W

k 70 m -1 1 k (1 / 40) S m -1 (1 / 4) S m -1
k = = S m -1; Lm = = ¥ 2
mol
R 280 W 4 c (0.5 mol dm -3 ) (0.5 ¥ 103 mol m -3 )
8.30 Complete Chemistry—JEE Main

UNIT 3 GALVANIC CELLS

Zn is oxidised to Zn ion at one electrode.


ion is

Fig. 1 Daniell cell

Principle of Working of a Galvanic Cell

ion.
æÆ Zn
Resume of working of a Galvanic Cell

two half-cells
salt bridge
4 3.
negatively charged

positively charged
Redox Reactions and Electrochemistry 8.31

to anode.
Cell Diagram

eliminated.

c c2
Cell Reaction

Illustration
c c2
Æ Zn2+ + 2e
+ 2e Æ
Add
+ 2+
Æ Zn +

c2 Zn c
Æ + 2e
Zn + 2e Æ
Add
+ Zn2+ Æ +

Reduction Potentials and Cell Potential


Oxidation and reduction Potentials
oxidation potential

reduction potential

.
Correlation Between Oxidation and Reduction Potentials

An atom having larger oxidation potential produces an ion with the lesser reduction potential and vice versa.
8.32 Complete Chemistry—JEE Main

Cell Potential

Ecell †
Ecell

Ecell
Ecell

or Ecell = E E
E E

c c

Ecell = E u u EZn
Spontaneity of Cell Reaction E E

Ecell E E E > EL if the cell diagram has a positive cell potential,


the cell reaction will be spontaneous in nature.
E E

Ecell
E E E <E if a cell diagram has a negative cell potential,
the cell reaction will be nonspontaneous in nature.
Standard Half-Cell (or Electrode) Potentials
E

||

Eºcell = Eº Eº
hydrogen-hydrogen ion half-cell is chosen as the standard half-cell and its
standard half-cell potential is assigned a value of zero at

Eºcell = Eº

Redox Reactions and Electrochemistry 8.33

Table 1
Electrode reaction E/ Half-cell representation
Æ

Zn Æ Zn Zn

Fe Æ Fe Fe

Fe Æ Fe Fe

Æ 2 2

Æ Br

2 2 Æ 2 2

1 _
2 2 Æ 2

Fe Æ Fe Fe

1 Br
2 Br2 Æ Br 2

1
2 2 Æ 2

Notes
less

Illustration 2
Ecell E 2
E
Ecell
more

Illustration 2
Ecell E E 2 t
Ecell
8.34 Complete Chemistry—JEE Main

Illustration
E E
E E

Reduction reaction

Æ ¥2 ER E
+ 2e Æ E E

Æ Ecell ER E
i.e. Ecell E E

Nernst Equation potential

c c2
c2 Æ c
DG DG RT ln Q
2+
[ Ni ]/ c º
Q Q
([Ag + ]/ c º ) 2
c .
DG nFE
RT 2.303RT
nFEcell nFEcell RT ln Q or Ecell Ecell ln Q Ecell Ecell log Q º
nF nF
Illustration
RT
Ecell Ecell ln Q Ecell E E
nF
[ Ni 2+ ] / c o 0.5
Q + o 2
=
([Ag ] / c ] (0.1) 2

(2.303)(8.314 J K -1 mol-1 )(298.15 K )


2.303RT
F (96500 C)
(0.059 V)
Ecell log (50)
2
Standard Equilibrium Constant of a Reaction
DG RT ln K DG nFEcell
nFEcell
º nFEcell
º
RT ln K nFEcell or ln K K
RT (2.303) RT
Redox Reactions and Electrochemistry 8.35

Illustration Equilibrium constant at 298 K for the reaction 2Fe3+(aq) + Sn2+(aq) 2Fe2+(aq) + Sn4+(aq)
In this reaction, Fe3+ is reduced to Fe2+ and Sn2+ is oxidised to Sn4+. Thus, the cell Fe2+, Fe3+ | Pt will constitute the
right-hand half-cell and Sn2+, Sn4+ |Pt will constitute the left-hand half-cell. Thus, the galvanic cell producing the given
reaction is
Pt|Sn2+, Sn4+ Fe2+, Fe3+ | Pt
Its standard cell potential is Eºcell = EºR – EºL = EºFe2+,Fe3+| Pt – EºSn2+, Sn4+ | Pt = 0.771 V – 0.150 V = 0.621
nEcell
º 2(0.621 V)
Hence log Kºc = = = 21.0. Thus Kºc = 1.0 ¥ 1021
(2.303RT /F ) (0.059 V)
Inter-Relationship Amongst Some Reduction Potentials It is possible to determine inter-relationship amongst
some reduction potentials. A few examples are described in the following.
1. Let we have
Fe3+(aq) + 3e– Æ Fe(s) Eº1 = – 0.036 V
Fe2+(aq) + 2e– Æ Fe(s) Eº2 = – 0.440 V
3+(aq) + e– Æ Fe2+(aq) Eº = ?
Fe 3
The value of Eº3 can be determined as follows.
Fe3+(aq) + 3e–Æ Fe(s) DGº1 = –3FEº1
Fe (aq) + 2e Æ Fe(s)
2+ –
DGº2 = –2FEº2
Subtract
Fe3+(aq) + e–Æ Fe2+(aq) DGº3 = –F(3Eº1 –2Eº2)
But DGº3 = –(1) FEº3. Hence,
Eº3 = 3Eº1 –2Eº2 = 3(–0.036V) – 2(–0.440 V) = 0.772 V
2. Let we have
AgCl(s) + e–ÆAg(s) + Cl–(aq) DGº1 = – (1)FEº1
Ag+(aq) + e–ÆAg(s) DGº2 = – (1)FEº2
Subtract
AgCl(s) Æ Ag+(aq) + Cl–(aq) DGº3 = F(Eº2 –Eº1)
But DGº3 = –RT ln Kºsp. Hence,
RT
F(Eº2 –Eº1 ) = –RT ln Kºsp or Eº1 = Eº2 +
ln Kºsp
F
Knowing Eº2 and Kºsp, the value of Eº1 can be determined or alternatively if Eº1 and Eº2 are known, the value of
Kºsp(AgCl) can be determined.
Reducing and Oxidising Ability of Species
Consider the displacement reaction A(s) + B+(aq) Æ A+(aq) + B(s)
The cell producing this reaction is A(s)|A+(aq) B+(aq)|B(s) with cell potential Eºcell = EºB+|B – EºA+|A
The reaction will be spontaneous provided EB+ | B > EA+ | A so that Eºcell is positive.
In the given reaction, the reducing tendency of ions is B+ > A+ and the oxidation tendency of atoms is A > B.
Hence, A (which acts as reducing agent) reduces B+ (which acts as oxidising agent). Thus
Larger the reduction potential of a species, better the oxidising agent, and the reduced species is poor
reducing agent.
In the given reaction B+ is better oxidising agent than A+ and A is better reducing agent than B
Illustration Predicting displacement ability of Atoms
Given : Eº(Al3+ |Al) = –1.66 V; Eº(Cu2+|Cu) = 0.34 V
Eº(Fe2+ |Fe) = –0.44 V; Eº(Mg2+ |Mg) = –2.37 V
and Eº(Zn2+ |Zn) = –0.76 V
8.36 Complete Chemistry—JEE Main

The decreasing order of reduction potentials is Eº(Cu2+ |Cu) > Eº(Fe2+ |Fe) >Eº(Zn2+ | Zn) > Eº(Al3+ | Al) > Eº(Mg2+ |Mg)
Thus, the decreasing order of reducing ability and hence the decreasing order of oxidising agent is Cu2+ > Fe2+ >Zn2+
>Al3+ >Mg2+
The increasing order of oxidising ability and hence the increasing order of reducing agent is Cu < Fe < Zn <Al < Mg
If E(B+ |B) > E(A+ |A), then the reaction B+ + A Æ B + A+ will be spontaneous and hence. A can displace the metal B
from its salt. Thus
Mg can displace Cu, Fe, Zn and Al Al can displace Cu, Fe, and Zn
Zn can displace Cu and Fe Fe can displace Cu
Electrode Potentials and Electrolysis
In general, the reduction process at cathode occurs in the order of decreasing reduction potential, i.e. the species
with higher reduction potential is reduced in preference to the other species.
For example, In the electrolysis of sodium chloride, we have
Na+ (aq) + e–Æ Na(s); Eº = – 2.71
1
H2O(l) + e–Æ H2(g) + OH–(aq); Eº= – 0.83 V
2
Since EºH2 | OH– |Pt > EºNa+ |Na, it is water which is reduced in preference to Na+.
On the other hand, the oxidation process at anode occurs in the order of increasing reduction potential (i.e.
decreasing oxidation potential) i.e. the species with lower reduction potential (i.e. higher oxidation potential) is
oxidised in preference to the other species.
For example, In the electrolysis of sodium bromide, we have
1
2 Br2(l) + e Æ 2Br (aq);
– –
Eº = 1.06 V
O2(g) + 4H+ + 4e– Æ 2H2O(l); Eº = 1.23 V
Since EºBr– | Br2 < EºO2 | H+ | pt, . Br– ion is oxidised in preference to H2O.
Note: If the two potentials are close to each other, it is not possible to predict with certainty the species that will
undergo oxidation or reduction. Sometimes, a higher potential is required for the desired change due to
imperfection at the electrode reaction.
Extraction of an Element from its oxide T/K
2HgO
Figure 2 displays the variation of DG of oxidation reactions per mole 773 1273 1773 2273
of oxygen consumed for a few elements with increasing temperature. 0
2Cu2O
Ellingham diagram. This diagram assumes 2ZnO
2
that DrH and DrS of the oxidation reactions are independent of tem- Cr O
3 2 3
DfG/4.184 kJ (mol O2)–1

2FeO
perature. Figure 2 may be used to predict the temperature conditions CO2
–100
at which an element is obtainable by the reduction of its oxide. Tak-
ing an example of the reaction 2 Al O
2 3
3
FeO + C Æ Fe + CO
we write the following two reactions to obtain this reaction. –200 2CO
aO
(i) 2Fe + O2 Æ 2FeO DG1 TiO
2
2C
(ii) 2C + O2 Æ 2CO DG2
gO
Subtracting Eq. (i) from Eq. (ii), we get 2M
2C + 2FeO Æ 2CO + 2Fe DG3 = DG2 – DG1
–300
For this reaction to be feasible, we must have DG1 > DG2. We can
Fig. 2 Ellingham diagram
Ellingham diagram. The requirement is that the plot of DG versus
T for the oxidation of Fe must lie above that of C. The temperature (Teq) at which these two plots intersect each other
represent the reaction at equilibrium, since DG of the reaction is zero. At T > Teq, DG1 > DG2 indicating that the reduction
of FeO with C is feasible whereas if T < Teq, DG1 < DG2, the reaction is not feasible.
Redox Reactions and Electrochemistry 8.37

Some Commercial Cells

Examples of Primary cells


Leclanche Dry Cell

2
4 2
4 2 2
Reactions
Æ 2+
+ 2e–
2 2

Æ 2 3

(aq)
2 2 Æ 2+
2 3

4 Æ–
3 + Cl

2
3 + 2Cl Æ
2+ –
3)2Cl2

Comments
4
Zinc-Mercury Cell

Reactions
– Æ 2

2 Æ – –

Comment

Alkaline Cell

2
Reactions

Æ 2 + 2e–
2 2

Æ –

2 2 Æ 2

Silver Oxide Cell

2
Reactions

Æ 2 + 2e–
2 2

Æ –

2 2 Æ 2
8.38 Complete Chemistry—JEE Main

Examples of Secondary Cells


Lead Storage Cell

Reactions
2–
4 Æ 4 + 2e

+ 2– + 2e– Æ
2 4 4 2
2
+ 2– (aq)
4 Æ 4 2

Comment 2 4 2 4

Nickel-Cadmium Cell
2
2 2


2 2 2
Reactions
– Æ 2 + 2e–
2 2
– Æ 2

2 2 Æ 2 2(s)

Comment

Hydrogen-Oxygen Fuel Cell

Reactions
2
– Æ 2

Air
Rust Fe2O3 O2
2 2
–Æ –

2 2 Æ 2 water
Comment 2 2 H+
2 2 cathode anode

Phenomenon of Corrosion e–

O2 + 4H+ + 4e– Fe Fe2+ + 2e–


Corrosion of Iron 2H2O

2+ + Iron
2
Reactions
Æ 2+(aq) + 2e–] ¥ 2 Fig. 3
O 2 (g) + 4H + (aq ) + 4e- Æ 2H 2 O(l)
Add
2Fe(s) + O 2 (g) + 4H + (aq ) Æ 2Fe2+ (aq ) + 2H 2 O(l)
2+
Side reaction
2+
2 2 Æ 2 3
+(aq)

Cell potential E°cell = E°R – E°


Factors Helping Corrosion Some of the factors are :
Redox Reactions and Electrochemistry 8.39

+ 2+
2 3

Prevention from Corrosion

2+(aq) +(aq)
2

MULTIPLE CHOICE QUESTIONS ON UNIT 3


4
+ – Æ 2+
2 E
2
+ + 2e– Æ E2
4
(a) Ecell = E – E2 (b) Ecell = E2 – E (c) Ecell E – 2E2 Ecell E2 E

2+ 2+ 3+ 2+
2

4 n 2+ +

2 c 2

2+ 2+ 2+
E E E

– – – – –
E 3 E 2 E
– –
3 2


E 2Cl2

1
2
HgO(s) + 12 H 2 O(l) + e - Æ 12 Hg (l) + OH - (aq ) E
+
(aq) + e– Æ 1
2
2
(a) E° + (RT/F K° (b) E° – (RT/F K° (b) E° + (2RT/F K° K°
+ – –
E E 2
– + –
2
¥ ¥ –20 ¥ –22 ¥ –23

Eº (Al3+ 2+ E 2+

2+ 3+ 2+ 3+
E E E
8.40 Complete Chemistry—JEE Main

n+
+ ne– Æ E° m+
+ me– Æ E °2
If n > m E °3 + (n –m)e– Æ m+
nE1∞ + mE ∞2 nE1∞ - mE ∞2 mE1∞ + nE ∞2 mE1∞ - nE ∞2
(a) E°3 = (b) E°3 = (c) E°3 = E°3 =
n+m n-m n+m n-m
1
2
– Æ 2 2

K K K E°Cl– (=E°),
E° – (=E°2 E°I– (=E°3) is
(a) E° > E°2 > E°3 (b) E° > E°2 > E°3 (c) E°3 > E°2 > E° E°3 > E° > E°2

2+ 2–
2

3+ 2+
E
3+ 2+
E

2+

2 4 2 4 2
2+
 2+
Eºcell K°c 2+
Æ 2+

¥ 20
¥ –20
¥

ANSWERS

HINTS AND SOLUTIONS


4 Æ+ 2+ –
2
Æ 2
+ + 2e–
Ecell = ER – E = E – E2
2+ + 2+ 3+ 2+ 2+
2


4
+ – Æ 2+
2
RT Ê [Mn 2+ ] ˆ
E = E° – ln
5F ÁË [MnO 4- ][H + ]5 ˜¯
+

E
Redox Reactions and Electrochemistry 8.41

+(aq) + 2e– Æ 2 R
(H 2 (g )) L Æ 2H (aq) + 2e- +

(H 2 (g)) L Æ (H 2 (g)) R

RT Ê ( pH 2 ) R ˆ 0.5 ˆ
log ÊÁ
0.059 V 0.059 V
Ecell = - 2 F ln Á ( p ) ˜ = - Ë ˜
0.25 ¯ = –
Ë H2 L ¯ 2 2


3 2

Æ – –
E

2
– Æ 1
2 2
– E


3 2
– Æ 1
2 2
– E

-5FE ∞ = -4 F (0.54 V) - 1F (0.45 V). E


1
2
Hg 2 Cl2 (s) + e- Æ Hg (l) + Cl (aq ) -

RT
E = E° – ln ([Cl- ] / c∞)
F
RT
1
2
1
2 2
– Æ 1
2
– E = E° – –
F
+

RT Ê [H + ][OH - ]/ M 2 ˆ RT + – 2+ +
E = E° – Á + ˜ = E° –
F Ë [H ]/ M ¯ F
Ê RT
ln K w∞ ˆ˜ +
RT
= Á E∞ - +
Ë F ¯ F
+
(aq) + e– Æ 1
2 2

E° – (RT/F K°

2 + e– Æ –
E
Ag Æ Ag + + e- E ∞ = - 0.8 V
Ag (CN) -2 Æ Ag + + 2CN - ∞ = - 1.18 V
Ecell
DG° = –nFEcell
° = –RT K RT K°

nEcell (1)(-1.18 V)
K° = = = -20 K ¥ –20
( RT /F ) (0.059 V)

3+ + e– Æ 2+ DG° F
2+ + 2e– Æ DG2 = – (2)F

Cr 3+ + 3e- Æ Cr DG3∞ = - F ( - 0.42 - 2 ¥ 0.90) V


DG 3 = –3FEº 3+ Eº 3+ E
8.42 Complete Chemistry—JEE Main

n+
+ ne– Æ DGº = –nFEº
m+ + me– Æ DGº2= –mFEº2
n+ + (n – m)e– Æ m+ DGº3= –F(nEº – mEº2)
DG°3 = –(n – m) FE°3
–(n – m) FE°3 = –F(nE° – mE°2 E°3 = (nE° – mE°2)/(n – m)

RT
E = E° – ln{( pH 2 /bar )1/ 2 ([OH - ]/ M )}
F
+

RT ÏÔ ([H ][OH ]/ M )( pH 2 /bar ) ¸Ô Ê E ∞ - RT ln K ∞ ˆ - RT ln ÏÔ ( pH 2 /bar ) ¸Ô


+ - 2 1/ 2 1/ 2

E = E° – ln Ì ˝ = ÁË w˜¯ F Ì + ˝
F ÔÓ [H + ]/ M Ô˛
F ÔÓ [H ]/M Ô˛
+
2
+ + e– Æ 12 2

RT ÏÔ (pH 2 /bar ) ¸Ô
1/ 2
E=– ln Ì ˝ (2)
F Ô [H + ]/ M Ô
Ó ˛

RT 2.303 RT
E° = K° = K°
F F
RT ∞ (AgX)}
E°X– = E° + + ln {Ksp
F
K° E°X–

3+

E 3+ 2+

RT Ê [Fe2+ ] ˆ
E = E° – ln
nF ÁË [Fe3+ ] ˜¯
3+ 2+ 3+ 2+

4
2+
Æ 2+

E°cell
nE ∞ (2)( -0.59 V)
K°c = = = -20. K°c ¥ –20
RT/F (0.059 V)

MULTIPLE CHOICE QUESTIONS ON THE ENTIRE UNIT

Characteristics of Galvanic Cells


not
Redox Reactions and Electrochemistry 8.43

(b)
(c)
not

2 4 2(s)
4(s)

2 4
2 4
2 4

Oxidation and Reduction Abilities


2+ 2+
2+ 2+
E E
2+ 2+

2+ 2+ 2+ 2+

E + E 2+
+ +
2 2
2+ 2+
2

2+ Æ 2+(aq); E
2+
2
8.44 Complete Chemistry—JEE Main

+ 2+
E E
+ +
2
2+ 2+
2
not
n+ n+
(a) If E 2
n+ +
(b) If E 2
2
2

2+ + 2+ 2+ +
E
2+ 2+ 2+
E E E

2X– + Y2 2Y– + X2
–+Y
2
–+X 2X–
2 2

(a) E° – 2
> E°Y – 2
> E°X – 2
> E° – 2
(b) E° – 2
E°Y– 2
E°X– 2
E° – 2

(c) E° – 2
E°Y– 2
> E°X– 2
> E° – 2
E° – 2
> E°Y – 2
E°X– 2
E° – 2

E + + 2+ 3+

+ 3+ 2+

E + E 3+ 2+ E 2+ E 3+

E 2+ + E 2+
+ 2+ 2+ 2+ 2+

E 2+ E 3+

E + E 3+ 2+
2

2+(aq)
2

2+ 2+

2+ 2+
2+ 2+
2+ 2+
E E
Redox Reactions and Electrochemistry 8.45

2+ 2+
E E
2+ +
2
2+ 2+
2
3) 2 3 3) 2
+ 2+ 2+
E E E

2+

2+ 4+ 2+ 2+ 2+ +
E E 2 E 2

4+ 4+ 2+ 2+
2
2+ 4+ 2+ 2+

Cell Representation, Reactions and Potentials


2+ 4+

Ce4+(aq) + e– Æ Ce3+(aq); E
3+(aq) + e– Æ 2+(aq); E
E 3+ 2+ 4+

E 3+ 2+

E(Ce4+, Ce3+ 4+

2
–(aq) Æ 2(l) + 2Cl–(aq);
DG is

+ –
2 2

– 2+ + + +
4

+ 3+ 3+ +
E E E
8.46 Complete Chemistry—JEE Main

2+ 2+ + +
E E E


(a) E 2 E°(Cl– 2 E –
2 E°(I– 2)

+ +
(a) p 2 p 2
+ +
(c) p 2 p 2
4

E 2 2 2

E 2 2

E° 3+ E° 2+ E° 3+ 2+

1
2 2 Æ +(aq) + Cl–
3 2 3
2 2
2+ +
2+ +

3+ + e– æÆ 2+ is
(a) E = E° + (RT/nF 2+ 3+]) (b) E = E° – (RT/nF 2+ 3+])

(c) E = E 2+ 3+ E=E 2+ 3+])

Applications of Galvanic cells


E + E –

¥ ¥ ¥ ¥

+(aq) 2+

+ 2+
E E
¥ ¥ ¥ ¥ 8

E°cell K°
eq

eq = nFEcell
° /RT K°eq = – nFEcell
° /RT
K°eq = nFEcell
° /RT K°eq = – nFEcell
° /RT
Redox Reactions and Electrochemistry 8.47

2+ 2+ E°cell Kºeq 2+
2+

K°eq K°eq K°eq K°eq


Ecell 2+ 2+

8 4 20
2+ 2+
Ecell

æÆ 2+ + 2e– 2+ æÆ 3+
+ e–
– æÆ – – æÆ –
2 2 2 2
2+ 2–
2
2+
3+ 2+
E
3+ 2+
E

Miscellaneous Problems


4
+ 2+
Æ 2+ 3+
2
– 2+ + 3+ 2+
E 4 E

2+ – 3+
4

– 2+ 2+ 2+ + 3+ – 2+
4 3 4

– 2+]
4
8.48 Complete Chemistry—JEE Main

3+

3+ by

+
3 by

– 2+ 2+ 3+ +
4 3

2+
–3

ANSWERS

HINTS AND SOLUTIONS

9.65 ¥ 60 ¥ 60
96500

2+
60 ¥ 60
2 ¥ 10000
65.4 ¥ 60 ¥ 60 63.5 ¥ 60 ¥ 60
g g
2 ¥ 10000 2 ¥ 10000

E 2+ E 2+ 2+

Cell 2+ 2+ Cell reaction 2+ Æ 2+

2+

+ +

x 2+

2+ Æ 2+(aq)

x x
Redox Reactions and Electrochemistry 8.49

E
RT [Fe2 + ] 0.059 V Ê x ˆ
E° = ÁË 0.1 M - x ˜¯
2F [Cd 2 + ] 2
Ê x ˆ 2 ¥ 0.04 x
ÁË 0.1 M - x ˜¯ = 0.059 0.1 M - x
22.7 ¥ 0.1 M
x=
23.7
2+]
eq
+

2
2+ + 2+ 2+ + 3+
2+ + 3+ 2+ + 2+
2
– + Y2 2Y– + X2 E°Y– > E X–
2 2

– + Y2 E° – > E°Y–
2 2


+ X2 2X– 2 E°X – 2
> E° – 2

E° – 2
> E°Y– 2
> E°X – 2
> E° – 2

3+ 2+ 2+ + 2+
E E E E
3+ 2+ 2+ +

˘ Reducing tendency
X + + e– = X; E
˙ X + > Y + > Z+
Y+ + e– = Y; E ˙
+
˙ Oxidizing ability
+ e– E ˙
˚ X + > Y + > Z+
˘ Oxidation tendency
X X + + e –; E ˙ Z>Y>X
Y Y + + e –; E ˙
+
˙ reducing ability
+ e –; E ˙
˚ Z>Y>X

2+ 2+ E 2+ Æ
2+ 2+

2+ + 2+ +
2+
2

4+ 2+ 4+

2+ 2+ 2+
8.50 Complete Chemistry—JEE Main

+ Ce4+ Æ 2+ 3+
+ Ce3+
2+] = 40 mL 3+] 10 mL
=
(50 + 10) mL (50 + 10) mL
RT [Fe2+ ]
E 3+ 2+ = E° 3+ 2+ –
F [Fe3+ ]

E 3+ 2+ = ECe4+, Ce3+
RT [Fe2+ ]
E 3+ 2+ = E° 3+ 2+ –
F [Fe3+ ]

RT [Ce3+ ]
ECe4+, Ce3+ = E°Ce4+, Ce3+ –
F [Ce4+ ]

RT [Fe2+ ] [Ce3+ ]
E 3+ 2+ + ECe4+, Ce3+ = E° 3+ 2+ + E°Ce4+, Ce3+ –
F [Fe3+ ] [Ce4+ ]
3+ 3+ 2+ Ce4+
1 1
E=E 3+ 2+ = E° 3+ 2+ + E°Ce4+, Ce3+ ] =
2 2
4+

3+] 50 mL 4+] 10 mL
= =
(50 + 60) mL (50 + 60) mL
RT [Ce3+ ]
ECe4+, Ce3+ = E°Ce4+, Ce3+ –
F [Ce4+ ]

D G° = – nFEcell
°; ° = – D G°/nF;
Ecell Ecell
° ¥ 3

1 1
Right half-cell +
R + e– Æ 2 Left half-cell 2 Æ + + e–
2 2
Cell reaction +
R Æ +

Cell potential
RT [H + ]L Ê 0.01 ˆ
Ecell = – + ÁË -6 ˜¯
F [H ]R 10

4
+ – Æ 2+
2
2+ 4
RT [Mn ] [H 2 O]
E = E° –
F [MnO -4 ] [H + ]8

RT [Mn 2 + ] [H 2 O]4 RT 1
E = E° – -
F [MnO -4 ] 5F [H + ]8
Redox Reactions and Electrochemistry 8.51

RT [Mn 2 + ] [H 2 O]4 RT 1
E2 = E° – -
F [MnO 4- ] 5F ([H + ] / 100)8
RT ([H + ]/ 100)8 RT - 8 (0.059 V)
E2 – E = + 8
=– 8
= ¥2
F [H ] F 5

+
+ e– Æ E DG° = – F
3+
+ 3e Æ –
E DG° = – 3F
3+
+ 2e Æ– +
; DG° = – 2F (E° 3+ + )

– 2F (E° 3+ + ) = – 3F F E° 3+ +

2+
DG° = – 2F E° 2+
2+
+eÆ +
DG°2 = – F E° 2+ +

+
+ e– DG°3 = – F E° +

DG°3 = DG° – DG°2


– F E° + = – 2 F E° 2+ + F E° 2+ +

E° + =2¥

RT p1H/ 22
E +
2
= - ln +
F [H ]
+
p 2 E +
2

RT 1
E = E° – ln
2 F [Cu 2+ ]1
RT 1 RT 1
E2 = E° – ln 2+
= E∞ - ln
2 F [Cu ]2 2F 0.1[Cu 2+ ]1
RT 1 RT 1 RT RT 0.059 V
E2 – E = - ln 2+
+ ln 2+
= = -
2F 0.1[Cu ]1 2 F [Cu ]1 2F 2F 2

1 RT p1H/ 22
+ + e– 2; E= - ln +
2 F [H ]
E=0
12
Ê pH 2 ˆ [H + ]
ÁË atm ˜¯ = = 10-7 p
mol dm -3 2

RT ( pH 2 /atm)1/ 2 RT 1
E= - ln +
=- ln
F [H ]/M F (10-7 )
3+ + 3e– E° DG° = – 3FE°
2+
+ 2e– E°2 DG°2 = – 2FE°
2
3+
+ e– 2+
E°3 DG°3 = – FE°
3
8.52 Complete Chemistry—JEE Main

DG°3 = DG° – DG°2 FE°3 = – 3FE° + 2FE°2


E°3 = 3E° – 2E°2
+
2 2

2+ + 2e– Æ DG° = – 2F
+ + e– Æ DG°2 = – F
2+ + e– Æ + DG°3 = DG° – DG°2 = – F F
+
E
2+ + 2e– æÆ 2+
2
RT [Hg 2+
2 ]
E = E° –
2F [Hg 2+ ]2
2Cl2
RT (0.1) [Hg 2+
2 ]
E2 = E° –
2F [Hg 2+ ]2
RT RT
E2 – E = –
2F 2F
2Cl2 + 2e– æÆ –

RT –] 2
E = E° –
2F

RT –] 2}
E2 = E° –
2F
RT RT
E2 – E = –
2F F
+

RHC – Æ –

LHS Æ + + e–

Æ + –

E°cell = E° – – E° +

nFE°cell = – RT K °

nEcell (1) (- 0.7 V)
K° = = K° ¥
( RT / F ) (0.059 V)

2+ +

RHC ++ e– Æ
LHC Æ 2+ + 2e–
Cell reaction +Æ 2+

Cell potential E°cell = E° + – E° 2+

E°cell = E° + – E° 2+
Redox Reactions and Electrochemistry 8.53

nEcell
° (2) (0.19 V)
nFE°cell = – RT K°eq K °eq = K°eq =
2.303 (RT /F ) (0.059 V)
K °eq ¥
E°cell
2+ 2+

E°cell


n F Ecell ∞
n Ecell 2 ¥ (-0.6264)
K°eq = = =
2.303 RT (2.303 RT F ) 0.059

Ê n Ecell ˆ Ê 2 ¥ 0.48 ˆ
o
Keq ÁË 0.059 V ˜¯ ÁË ˜
0.059 ¯
DG° = – nFE°cell
3+
(aq) + e– æÆ 2+

RT [Fe2+ ]
E = E° –
F [Fe3+ ]

3+ 2+ + 2+ –
4
E°cell = E° + 2+ –
4
– E° 3+ 2+

E°cell
RT Ê [Mn 2+ ] [Fe3+ ]5 ˆ
Ecell = E°cell – ln Á - + 8 2+ 5 ˜
5 F Ë [MnO 4 ] [H ] [Fe ] ¯
2+ 3+ –
4
+
Ecell
Ecell
RT
DE = -
F
RT
DE = -
F
RT Ê 1 ˆ 0.059 V ˆ
DE = - ln Á 8 ˜ = + ÊÁ ˜ (8
5F Ë 2 ¯ Ë 5 ¯

RT È (0.5) (2.0)5 ˘ Ê 0.059 V ˆ log È 1 ˘


Ecell ln Í ˙ ÁË ˜¯ Í 4˙
5F Î (1.0) (2.0)8 (0.5)5 ˚ Î ( 2.0) (0.5) ˚
3
5

Ê 0.059 V ˆ 0.059 ¥ 0.301


ÁË ˜ V
5 ¯ 5
0.059 V ˆ
DE = - ÊÁ ˜
–3
Ë 5 ¯
8.54 Complete Chemistry—JEE Main

MULTIPLE CHOICE QUESTIONS FROM AIEEE AND JEE MAIN

[2003]

[2003]
2+
Æ 2+
E°cell

[2003]

¥ ¥ ¥ –2
[2003]

[2004]
E E° 3+ 2+ E° 2+

3+(aq) Æ 2+ 2+(aq) is
[2004]

F ,R )
¥ ¥ ¥ ¥ 30 [2004]
+(aq) Æ 2+
2 2 4
E
E
E
E [2004]
E° 3+ 2+

[2004]

– Æ – E Æ + + e– E
K° RT/F
[2006]
2+ 2+ E°cell
2+ 2+ 2+ 2+]) is
¥ 4 [2007]
3+ 2+
E E
3+ 2+

[2008]
Redox Reactions and Electrochemistry 8.55

E° 3+ E° 2+
3+
(aq) + e– Æ 2+

[2009]
2 3
2 4
Al2O3 Æ Al + O 2 ; D r G = 966 kJ mol-1
3 3
[2010]

E° 2+

[2010]

(a) p + p +
2 2
+ +
(c) p 2 p 2
[2011, (Cancelled)]
2+ 2+ 2+
2+
Æ X2+
[2012]

E° 3+ E° –
4
2+ E° 2
2– 3+ E°Cl2 –

[2013]
– 3+ 2+ –
(a) Cl 4

2+
+ 2e– Æ E
3+ + e– Æ 2+; E
Ecell 2+ Æ 3+

[2014]
E° +

[2014]

3+(aq) + e– Æ 2+(aq) E
Al3+(aq) + 3e– Æ Al(s) E
2(aq) + 2e Æ
– –
(aq) E

2+ – – 2+ – 2+ 2+ –

[2014, online]
E
2+ 2+
E E

[2015, online]
– 2+ – –
4 2 2
– RT/F
2
– – – Cl– –
(a) Cl (b)
– – – [2015, online]
8.56 Complete Chemistry—JEE Main

[2016, online]

[2016, online]

[2016]

ANSWERS

HINTS AND SOLUTIONS

RT [ Zn 2+ ] 0.059 V ˆ Ê 1 ˆ
Ecell = Ecell
∞ - ln = (1.10 V) - ÊÁ ˜¯ log ÁË ˜¯ = 1.07V
2+
2 F [Cu ] Ë 2 0.1
DG° = –RT K° = –nFE°cell

nFEcell ∞
nEcell 2(0.295 V)
K∞ = = = K ¥
(2.303) RT (2.303 RT ) /F (0.059 V)

2
–(aq) Æ 2

2 2

Æ –
(aq)

2 2 Æ 2

3(aq) Æ 2+ 2+ 2+ 3+ 2+

E°cell = E° 3+ 2+ – E° 2+

DG = –nFE°cell = – RT K°eq
n 1
K°eq = ∞ =
Ecell (0.591 V) = 10 Keq
2.303( RT /F ) (0.059 V)
+
(aq) Æ 2+
2
2+
RT ([ Zn ] / c∞)( pH 2 / p∞)]
Ecell = E°cell – ln
nF ([H + ]/ c∞)2
Redox Reactions and Electrochemistry 8.57

+
2 4 Ecell

3+
+ e– 2+
E° 3+
+ e– 2+
E
3+
+ e– 2+
E° 3+
+ e– 2+
E
3+ 2+ 2+
3–

E Æ +
+ I– is E°cell

∞ = Ecell -0.952 V
log Ksp = = -16.13
RT /F 0.052 V

2+ 2+

2+(aq) 2+

[ Zn 2+ (aq)]
Keq =
[Cu 2+ (aq )]

E°cell

nEcell
DG° = –nFE°cell = –RT K°eq K °eq =
(2.303RT /F )
RT/F
(2)(1.10 V)
K °eq = = 37.3 K °eq
(0.059 V)
Ecell = ER – E
Right half-cell + 2e– Æ 2+

= - 0.42 V + ÊÁ
RT 1 0.059 V ˆ
ER = E°R – ln
Ë ˜ log (0.01) = - 0.42 V - 0.06 V = - 0.48 V
2+
2 F [Fe ]/ c∞ 2 ¯
Left half-cell 3+
+ 3e– Æ
0.059 V ˆ
= - 0.72 + ÊÁ
RT 1
E = E° – ln ˜ log (0.1) = - 0.72 V - 0.02 V = - 0.74 V
3F [Cr 3+ ]/ c∞ Ë 3 ¯
Ecell

3+
+ 3e– Æ DG°(i) = – 3F
2+
+ 2e– Æ DG°(ii) = – 2F

3+ + e– Æ 2+ DG° = DG°(i) – DG°(ii) F F F


DG∞ ( -0.770 V) F
E ∞Fe3+ , Fe2 + 1Pt =- = = 0.770 V
nF (1) F

2
3
(2Al3+ ) + 4e- Æ 34 Al and 2
3
(3O 2- ) Æ O 2 + 4e-
2
3
Al2 O3 Æ 34 Al + O 2 .
DG = –nFEcell
DG 966 ¥ 103 J mol-1
Ecell = – =- = -2.50 V
nF (4)(96500 C mol-1 )
8.58 Complete Chemistry—JEE Main

E
2+ 2+ 2+ 2+
E E E E

+(aq) + e– Æ 2
1/ 2
RT ( pH 2 /atm)
E= - ln
F ([H + ]/M )
+
E p 2

2+ Æ X2+ 2+
2+ 2+ 2+ 2+ 2+


4
2+ 2+ 3+ 2+

2+
+ 2e– Æ E°
3+
+eÆ 2+]
¥2 E°2
2+
Æ 3+
E°cell = E – E2
E
n+
+ ne– Æ
n+

2+ – – 2+


4
+ – Æ 2+
2

RT Ê [Mn 2+ ] ˆ
E = E∞ - ln Á
5F Ë [MnO -4 ] [H + ]8 ˜¯
+ –3

RT RT Ê [Mn 2+ ] ˆ
E = E∞ - ln (1024 ) - ln
5F 5F ÁË [MnO -4 ] ˜¯
2+ –
4

Ê 0.059 V ˆ
E ÁË ˜
5 ¯
2 2
9
Chemical Kinetics

The subject of chemical kinetics deals with the rates of chemical reactions including their dependence on the concentration
of reacting species and the experimental conditions.
General Appearance of Changes in the Concentration
of Species in a Reaction In general, the concentration of
a reactant decreases while that of a product increases with the
progress of a reaction. For example, for the reaction
2N2O5(g) Æ 4NO2(g) + O2(g)
the changes in the concentrations of N2O5, NO2 and O2 are
shown in Fig. 1

Average Rates of Changes in Concentration of Species in


a Reaction The average rate of change in the concentration
of the species B of a reaction over a time interval Dt
D[B]/Dt. The changes D[B]/Dt of various species in a balanced
chemical equation can be correlated with each other based on
the principle described in the following.
The magnitude of average rates of change in the concentration
of species in a balance chemical equation become equal to
each other when divided by the corresponding stoichiometric Fig. 1
numbers.
For example, for the chemical equation
2N2O5(g) Æ 4NO2(g) + O2(g),
1 D[ N 2 O5 ] 1 D[ NO 2 ] D[O 2 ]
we have - = = (1)
2 Dt 4 Dt Dt
Note: D[N2O5]/Dt has a negative value as the concentration of N2O5 decreases with time while D[NO2]/Dt and
D[O2]/Dt have positive values. The negative sign before D[N2O5]/Dt makes this term also positive.
Instantaneous Changes in Concentration of Species in a Reaction In chemical kinetics, the instantaneous
changes in concentration of species of a reaction play important role in determining the rate of reaction.

D[B]ˆ
= lim ÊÁ
d[B]
˜ (2)
dt Dt Æ 0 Ë Dt ¯ t
9.2 Complete Chemistry—JEE Main

The instantaneous rate (or simply the rate) of decrease (or increase) in the concentration of a reactant (or a product)
may be determined by determining the slope of concentration curve versus time of the reactant (or product) at the given
time (Figs 2 and 3)
Concentration of reactant

[A]¢1

[A]¢2 [A]

t1¢ t t2¢
Time

Fig. 2 Fig. 3
The instantaneous changes in concentration at a given time, like average, are related to each other through their
stoichiometric number. For example, for the reaction 2N2O5(g) Æ 4NO2 (g) + O2(g), we have
1 d[ N 2 O5 ] 1 d[ NO 2 ] d[O 2 ] (3)
= - =
2 dt 4 dt dt
Illustration The rate of formation of N2 in the reaction 4NH3 + 3O2 Æ 2N2 + 6H2O is 0.68 mol L–1 s–1. Determine
the rate of formation of H2O and the rates of consumptions of NH3 and O2.
The identity of changes in concentrations of the given reaction is
1 d[ NH3 ] 1 d[O 2 ] 1 d[ N 2 ] 1 d[H 2 O]
- =- = =
4 dt 3 dt 2 dt 6 dt
d[H 2 O] Ê 1 d[ N 2 ] ˆ
= 6Á = (3) (0.68 mol L–1 s–1) = 2.04 mol L–1 s–1
Ë 2 dt ˜¯
Now
dt

d[ NH3 ] Ê 1 d[ N 2 ] ˆ
– = 4Á = (2) (0.68 mol L–1 s–1) = 1.36 mol L–1 s–1
dt Ë 2 dt ˜¯

d[O 2 ] Ê 1 d[ N 2 ] ˆ
= 3Á = (1.5) (0.68 mol L–1 s–1) = 1.02 mol L–1 s–1
Ë 2 dt ˜¯

dt

Rate of a reaction
The rate of a reaction is the rate of decrease (or increase) in the concentration of a reactant (or a product) divided by
the corresponding stoichiometric number in the balanced chemical equation of the reaction.
For example, for the reaction
4NH3(g) + 3O2(g) Æ 2N2(g) + 6H2O(g)

1 d[ NH3 ] 1 d[O 2 ] 1 d[ N 2 ] 1 d[H 2 O]


r= - =- = =
4 dt 3 dt 2 dt 6 dt
Notes:
There is a negative sign before the rate of change in concentration of a reactant. This results in a positive value
of rate of reaction since d[B]/dt of a reactant is itself negative
It is mandatory to state the chemical equation to which the rate of reaction is referred to. For example, for the
decomposition of N2O5
Chemical Kinetics 9.3

1 d[ N 2 O5 ] 1 d[ NO 2 ] d[O 2 ]
2N2O5 Æ 4NO2 + O2; r= - = =
2 dt 4 dt dt
1 d[ N 2 O5 ] 1 d[ NO 2 ] 1 d[O 2 ]
N2O5 Æ 2NO2 + O2; r ¢ = – = =
2 dt 2 dt (1/ 2) dt
Obviously, r¢ = 2r.
IUPAC Recommendations about Rate of Reaction If a reaction involves change in volume, the concentrations of
its reactant or product changes not only due to the reaction in progress but also due to the volume change. To avoid such a

Illustration The rate expression for the reaction is


2N2O(g) Æ 4NO2(g) + O2(g)

1 dn( N 2 O5 ) 1 dn( NO 2 ) dn(O 2 )
x=- = =
2 dt 4 dt dt
Rate of Reaction in terms of Extent of Reaction Let the decomposition of N2O5 is initiated with the amount n0 of
N2O5. If x is the extent of reaction at time t, we will have
2N2O5(g) Æ 4NO2(g) + O2
t=0 n0 0 0
t n0 – 2x 4x x
∑ 1 dn( N 2 O5 ) 1 d(n0 - 2x ) dx
Hence x=- =- =
2 dt 2 dt dt
1 dn( NO 2 ) 1 d(4x ) dx
= = =
4 dt 4 dt dt
dn(O 2 ) dx
= =
dt dt (4)
Thus, the rate of reaction is equal to the change in extent of reaction with time.
Rate of Reaction at Constant Volume Condition Dividing Eq. (4) by constant volume, we have

x 1 d(nN 2O5 /V ) 1 d{n0 - 2x / V } d (x / V ) dx
r= =- =- = =
V 2 dt 2 dt dt dt
1 d(nNO2 /V ) 1 d(4x / V } dx
= =- =
4 dt 4 dt dt
d(x / V } dx
d(nO2 /V )
= = = (5)
dt dt dt
where x is the extent of reaction divided by constant volume.
Differential Rate Law Differential rate law (or simply rate law) expresses the dependence of rate of reaction on the
concentration of reacting species involved in a chemical reaction.
A early generalization in this regard is due to Gulber and Waage. This generalization, known as law of mass action,
is stated as follows.
The rate of reaction is proportional to the product of effective concentration of the reacting species, each raised to a
power which is equal to the corresponding stoichiometric number of the substance appearing in the chemical equation.
The law of mass action is strictly applicable to elementary reactions†.

† An elementary reaction involves one step for the conversion of reactants into products. The reactants collide at one place and are
simultaneously converted into products. The number of molecules appearing in the reactant side is known as molecularity of elementary
reaction. Elementary reactions with molecularity greater than three are very rare.
9.4 Complete Chemistry—JEE Main

d[A]
For example, A Æ products – = k[A]
dt
1 d[A] d[B]
2A + B Æ products – =- = k[A]2[B]
2 dt dt
Order of Reaction For a general chemical reaction n 1A 1 + n 2A 2 Æ n 3A 3 + n 4A 4 the experimental data on the

r = k[A1]a[A2]b
where the dimensionless exponents a, b,. . .may or may not be equal to the corresponding stoichiometric numbers n1,
n2,.... These exponents may have positive or negative integral values, fractional values or zero values.
The exponent a is known as partial order of the reaction with respect to A1 and so on.
The overall order of the reaction is
n=a+b+... (6)
The constant k is known as rate constant. Its unit is
unit of rate of reaction mol L-1 s -1 ...
unit of k = a +b +
= -1 a + b +
= (mol L–1)1 – (a + b + ) s–1 (7)
(unit of concentration) (mol L )
It may be emphasized that the rate equation with its rate constant and order of various reacting species is an

Integrated Rate Laws


The integrated rate law expresses the concentration of reacting species with time. This information is obtained by
integrating the corresponding differential rate law.
Zero-order Reaction For a zero-order reaction
[A]0
A Æ product
d[A]
the rate of reaction is r = – =k[A]0 = k (8)
dt
The unit of k is that of rate reaction r, that is mol L–1 s–1. Slope = –k
If [A]0 and [A]t are the concentrations at t = 0 and t = t, respectively, we
[A]

have
[A]t t
[A]0/k
Ú d[A] = - k Ú dt
[A]0 0

This gives [A]t = [A]0 – kt (9) t


The variation of [A] with time is linear with slope equal to –k (Fig. 4). Fig 4
Half-Life: The half-life period of a reaction is the time required for the concentration of reactant to decrease to half
of its original concentration, i.e. at t = t1/2,
[A]t = [A]0 /2. From Eq. (9), we have
[A]0/2 = [A]0 – k t1/2 i.e. t1/2 = [A]0/2k (10)
that is, half-life period is directly proportional to the initial concentration of the reactant.
Time of completion of the reaction Here[A]t = 0. Hence
tcompletion = [A]0/k

Example Enzyme-catalysed recations substrate æenzyme


æææ Æ product
r = k[enzyme][subtrate]0
to the substrate.
Chemical Kinetics 9.5

First-Order Reaction
A Æ product
d[A]
the rate of reaction is r=- = k[A] (12)
dt
The unit of k is (unit of r/unit of [A]), i.e. s–1. If [A]0 and [A]t are the concentrations at t = 0 and t = t, respectively, then
[A]t t
d[A]
Ú [A]
= - k Ú dt
[ A ]0 0

Ê [A]t ˆ
ln Á = - kt
Ë [A]0 ˜¯
This gives (13)

log {[A]/mol dm–3}


Slope – k /2.303
or ln ( [A]t/cº ) = ln ( [A]0/cº ) = –kt
where cº = 1 M. The variation of ln ( [A]t /cº )1/2 or ( log ( [A]t /cº )) with time
is linear with slope equal to –k (or –k/2.303), (Fig. 5).
Half-Life At t = t1/2, [A]t = [A]0/2. Hence
Ê [A]0 / 2 ˆ
ln Á
Ë [A]0 ˜¯
= –k t1/2 t/s

ln 2 2.303 log 2 0.693 Fig. 5


or t1/2 = = = (14)
k k k
Thus, half-life period is independent of the initial concentration of the reactant (Fig. 6) and has a constant value
dependent only on the rate constant.
Note
etc.) to occur is independent of the initial concentration of the reactant.
Illustration
to reduce the initial concentration to 12.5% is
Concentration

t t t
100% æææ
1/ 2
Æ50% æææ
1/ 2
Æ 25% æææ
1/ 2
Æ12.5%

3t1/2 = 3 ¥ 2.0 min = 6.0 min


Examples
N 2 O5 (g ) Æ 2 NO 2 (g) + 12 O 2 (g)
0 1 2 3 4
Number of half-life
H 2 O 2 (aq ) Æ H 2 O(l) + 12 O 2 (g)
Fig. 6
All radioactive decays.

equation in terms of physical quantities which are directly proportional to the concentrations.

For a reaction involving gaseous species, concentration terms may be replaced by partial pressures. Let the reaction
AÆ products be started with the pressure p0 of A. If Dng is the change in stoichiometric number of gaseous species,
then
A Æ products
p0 – p (Dng +1)p
with ptotal = (p0 – p) + (Dng + 1)p = p0 + (Dng)p.
This gives p = (ptotal – p0)/ Dng,
9.6 Complete Chemistry—JEE Main

ptotal - p0 ( Dn g + 1) p0 - ptotal
and p0 – p = p0 - =
Dn g Dn g
The integrated rate law is
Ê p - pˆ Ê ( Dn g + 1) p0 - ptotal ˆ
ln Á 0 = –kt i.e. ln Á ˜ = –kt
Ë p0 ˜¯ Ë Dn g p0 ¯
If p•
p• = (Dng + 1)p0
with this, the rate law becomes
Ê p - ptotal ˆ
ln Á • = –kt
Ë p• - p0 ˜¯

Illustration For the reaction (CH3)3COOC(CH3)3 (g) Æ 2 CH3COCH3(g) + C2H6(g), we have


di-tertiarybutyl peroxide
Ê 3 p0 - ptotal ˆ
Dng = 2 + 1 – 1 = 2; p• = (Dng + 1)p0 = 3p0. The rate law becomes ln Á ˜¯ = –kt
Ë 2 p0
The acid hydrolysis of an ester
H+
RCOOR¢ + H2O æææ Æ RCOOH + R¢OH
may be studied by titrating the product RCOOH against an alkali solution. It Vt and V• are the volume of alkali solution
required for the neutralization of RCOOH at time t
[Ester]0 = V• and [Ester]t = V• – Vt
Hence, the rate law becomes
Ê [Ester ]t ˆ Ê V - Vt ˆ
ln Á ˜ ∫ ln Á • = - kt
Ë [Ester ]• ¯ Ë V• ˜¯

order kinetics, is
C12H2O11(aq) + H2O(s) æÆ C6H12O6 + C6H12O6
sucrose glucose fructose
(dextrorotatory) (leavorotatory)
The reaction can be followed by measuring the rotation of plane polarized light. If qt and q• are the respective angles
of rotation at time t and at the end of reaction, then
[sucrose]0 μ q•– q0 and [sucrose] μ q•– qt
Hence, the rate law is
Ê [sucrose]0 ˆ Ê q - q0 ˆ
ln Á ˜ ∫ ln Á • = - kt
Ë [sucrose]t ¯ Ë q • - q t ˜¯

Second-Order Reactions For a second-order reactions A + B æÆ products, the rate law is


d[A] d[B]
- =- = k[A][B]
dt dt
Let a0 and b0 be the initial concentrations of A and B, respectively, at let x be the extent of reaction divided by constant
volume, at time t, we will have
A + B æÆ products
t = 0 a0 b0
t a0 – x b0 – x
d[A] d(a0 - x) dx
Since, - =- = , we write the rate law as
dt dt dt
dx
= k (a0 - x)(b0 - x)
dt
Chemical Kinetics 9.7

The integrated expression is


Ê b - xˆ Ê b0 ˆ Ê b0 - a0 ˆ
log Á 0 ˜ = log Á ˜ + Á ˜ kt
Ë a0 - x ¯ Ë a0 ¯ Ë 2.303 ¯
Special Case If [A]0 = [B]0 then the rate expression becomes
d[A]
- = k [A]2 (15)
dt
1 1
The integrated expression is - = kt (16)
[A]t [A]0
Half- Life Period Equation (16) gives
1 1 1
- = kt1/2 i.e. t1/2 = (17)
[A]0 / 2 [A]0 k[A]0
that is, half-life period is inversely proportional to the initial concentration of the reactant.
Note: Equations (15) to (17) are applicable for a reaction A Æ products provided the reaction is second order
with respect to A.
Examples of Second-Order Reactions
2CH3CHO Æ CH4 + 2CO
CH3COOC2H5 + OH– Æ CH3COO– + C2H5OH
S2O82– + 2I– Æ 2SO2–
4 + I2

Pseudo First-Order Reaction Consider the hydrolysis of an ester. Its reaction is


+
CH3COOC2H5 + H2O æH
ææÆ CH3COOH + C2H5OH
This reaction is of third order. Its rate law is
d(ester )
- = k’ [ester][H2O][H+]
dt
The concentration of H+ remains constant as it acts as a catalyst (it is consumed in the reaction in one of its elementary
step and is regenerated in another step).
The concentration of H2O remains constant as it is present in large amount.
These two concentration terms can be merged into rate constant to give
d(ester )
- =k ¢ [ester]; where k ¢ = k[H2O][H+]
dt

Note The rate of reaction increases with increase in the concentration of H+ as the rate constant is dependent on
this term.
Fractional-order Reactions For a reaction following the differential rate equation
d[A]
= k[A]n +1 / 2 ; (where n is integer)
dt
the integer rate law is
1 Ï 1 1 ¸
Ì n -1 / 2
- n -1 / 2 ˝ = kt
n - 1 / 2 Ó ([A]0 - x) [A]0 ˛

1 2n -1 / 2 - 1
with t1/2 =
k (n - 1 / 2) [A]0n -1 / 2
As mentioned above, the rate constant and order of a reaction are determined experimentally based on the kinetic
data observed during the progress of the reactions.
9.8 Complete Chemistry—JEE Main

Methods to Determine Order and Rate Constant

Integration Method In this method, the data are substituted in the integrated rate equations for different-order
reactions. The equation which gives almost a constant value of rate constant decides the order of the reaction.

Graphical Method In this method, the data are plotted according to the different integrated rate equations to yield
a straight line. The rate constant of the reaction is obtained from the slope of the resultant straight-line plot.

Half-Life Method This method can be employed only when the rate law involves only one concentration term. For
t0.5 = 0.693/k and it is independent of the initial concentration of the reactant. For the second
order t0.5 μ1/[A]0 and so on.

Van’t Hoff’s Differential Method In this method, the order of a reaction with respect to each of its reactants can
be determined. A number of kinetic experiments are carried out by varying the initial concentration of only one species
keeping all others constant. From the variation in the rate of reaction, the order of the reaction with respect to the species
whose concentration is varied is determined. The experiment is repeated this way with the variation of concentration
of other species one by one.

Ostwald Isolation Method In this method, the concentrations of all species except one are taken in large amounts.
During the course of the reaction, the variation in these excess concentration terms is negligible and thus remain virtually
constant. These constant terms can be merged with the rate constant and, thus, the rate of reaction depends only on one
term which is not taken in the excess amount. In this manner the experiment is repeated taking one term each time not
in excess and all other terms in excess amounts and the rate of reaction with respect to the species not taken in large
amounts is determined as in the case of the van’t Hoff differential method.

Illustration The following data were obtained at 303 K for the reaction 2A + BÆC + D.
[ A ]0 [ B]0 Rate of reaction
Experiment Number
mol dm -3
mol dm -3 mol dm -3 min -1
1 0.1 0.1 6.03 ¥ 10–3
2 0.3 0.2 7.2 ¥ 10–2
3 0.3 0.4 2.88 ¥ 10–1
4 0.4 0.1 2.4 ¥ 10–2
Derive the expression of rate law. What is the value of rate constant?

rate in experiment 3 is four times than that in experiment 2. This means, doubling the concentration of B (and keeping
concentration of A constant) makes rate of reaction four times indicating that the reaction is second order with respect
B. Similarly, comparing experiments 1 and 4 (where the concentration of B remains constant whereas that of A in

of reaction also becomes four times. This indicates that the order of the reaction with respect to A is one. Hence, the
differential rate law is given by
rate = k[A][B]2

Temperature Dependence of Rate constant The rate constant, hence the rate of reaction, increases with increase
in temperature. It is found that this increase is about 2 to 3 times with an increase of 10 ºC of the reaction.
The Arrhenius equation relating the rate constant, k with kelvin temperature, T, is
k = Ae–Ea/RT (18)
where A is known as pre-exponential factor. Its unit is the same as that of rate constant k, i.e. (mol–1 dm3)n –1 s–1 where
n is the order of reaction, R is a gas constant (= 8.314 J K–1 mol–1) and Ea is known as energy of activation.
Writing Eq. (18) in the logarithm form, we have
Chemical Kinetics 9.9

ln ÊÁ ˆ˜ = ln ÊÁ ˆ˜ - a
k A E 1
Ë kº¯ (19)
Ë kº¯ R T

log ÊÁ ˆ˜ = log ÊÁ ˆ˜ -
k A Ea 1
or Ë kº¯ Ë k º ¯ 2.303R T (20)

where kº represents the unit of rate constant k, so that the quantities k/kº and
A/kº become unitless.
According to Eq. (20), log (k/kº) varies linearly with l/T with slope equal to
–Ea/(2.303 R) and intercept equal to log (A/kº). This is shown in Fig. 7.
For the two sets of data, we have
Êk ˆ Ea Ê 1 1 ˆ
log Á 2 ˜ = - -
2.303R ÁË T2 T1 ˜¯
(21)
Ë k1 ¯ Fig. 7

The conversion of reactants into products requires reacting molecules to


come close and collide together. During the collision, the molecular rearrangement takes place involving breaking of
some bonds and making others. This rearrangement takes place only when the colliding molecules have energy equal
to or greater than some minimum energy. If the energy of colliding molecules is less than this energy, no rearrangement
takes place and the reactant molecules do not pass over to the products. The difference between the minimum energy

activation (Fig. 8). It can be shown on the basis of kinetic theory of gases that the number of molecules having energy Ea
or more than this is proportional to the Boltzmann factor exp(–Ea/RT) and thus the rate constant is directly proportional
to this factor. On increasing temperature, this factor increases (Fig. 9) so is the rate constant.

Fig. 8 Fig. 9

Illustration ¥ 10–2 s–1 at 300 K. Calculate the rate constant at


350 K if Ea = 50.0 kJ mol–1
Êk ˆ Ê Aˆ
log Á 1 ˜ = log ÊÁ ˆ˜ -
A Ea 1 Êk ˆ Ea 1
and log Á 2 ˜ = log ÁË ˜¯ -
Ë kº¯ Ë k º ¯ 2.303R T1 Ë kº¯ kº 2.303R T2
From these, we get
Êk ˆ Ea Ê 1 1 ˆ Ea (T2 - T1 ) (50.0 ¥ 103 J mol-1 ) È 350 K - 300 K ˘
log Á 2 ˜ = - Á - ˜= = -1 -1 Í ˙
Ë k1 ¯ 2.303R Ë T2 T1 ¯ 2.303R T2 T1 (2.303)(8.314 J K mol ) Î (350 K ) (300 K) ˚
= 1.244
log (k2/kº) = log (k1/kº) + 1.244 = log (4.00 ¥10–2) + 1.244 = –1.398 + 1.244 = – 0.154
k2 = 0.70 s–1
9.10 Complete Chemistry—JEE Main

The collision theory of gaseous molecules provides the


rationalization of rate law based on the collisions between the reacting molecules.
A reaction involves the rearrangement of bonds present in the reacting molecules to those present in the products.
This is possible provided
The reacting molecules collide with each other to provide the close proximity for the bond rearrangement.
Reacting molecules must have some minimum energy to initiate the process of bond rearrangement.

Rate of reaction = (Number of collisions per litre per second) (Fraction of molecules having minimum energy)
= ZAB e–Ea/RT (22)
where ZAB = p s AB
2
(8kBT / pm )1 / 2 N A* N B* . Here
s AB
2
is collision diameter (=(sA+ sB)/2, where sA and sB are radii of A and B, respectively).
kB is the Boltzmann constant, m is reduced mass (=mAmB /(mA+ mB)).
NA* and NB* are the respective numbers of molecules of A and B per unit volume
Equation (22) predicts rate law which is much larger than the observed rate law for most of reactions. This fact is
taken into account by introducing steric factor which requires that the reacting molecule not only colloide but also have

2AB(g) Æ A2(g) + B2(g)

Fig. 10
With steric factor p
r = p ZABe–Ea/RT (22)
The steric factor is usually less than 1 and consequently predicts the reduced rate of reaction.

Reaction Mechanism The pathway by which the reaction occurs is called its mechanism. The determination of rate
law of reaction helps proposing its mechanism.
The overall balanced equation for a reaction represents the net chemical change that occurs as the reaction proceeds
to completion.
The overall equation may proceed involving one elementary reaction or more than one elementary reactions. In the
latter case, the net reaction is obtained by the algebraic sum of elementary reactions.
Chemical Kinetics 9.11

Illustration The reaction 2NO + 2H2 Æ 2H2O + N2 proceeds through the following steps.
2NO Æ N2O2
N 2O 2 Æ N 2O + H 2O
N 2O + H 2 Æ N 2+ H 2O
Rate-Determining Step If a reaction involves more than one elementary step, the slowest-step is the rate-determining
step. The products obtained in the slowest step quickly combine with other substances to give products of the overall
reaction.
Illustration
k
Over-all reaction H 2 (g ) + I 2 (g ) æ æ
Æ 2HI(g)
K
Mechanism 
I 2 (g ) 
eq 1
 
 2 I(g ) (fast equilibrium)
Keq 2

I(g) + H 2 (g )  
 H 2 I(g ) (fast equilibrium)


H 2 I(g) + I(g) ææÆ 2HI(g) (slow)
From the slow step, we write
r =k¢ [H2I] [I] (1)
From the second fast-equilibrium reaction, we have
[H 2 I(g)]
Keq2 = fi [H2I(g)] = Keq2 [I(g)][H2(g)]
[I(g)][H 2 (g)]
Substituting [H2I(g)] in Eq. (1), we get
r = k’Keq2 [H2(g)] [I(g)]2 (2)

[I(g)]2
Keq1 = fi [I(g)]2 =Keq1[I2(g)]
[I2 (g)]
Substituting [I(g)]2 in Eq. (2), We get
r = k¢Keq1Keq2[H2(g)][I2(g)] = k[H2(g)] [I2(g)]
The main characteristics of a catalyst are as follows;
A catalyst is a substance that increases the rate of a reaction.
In a reaction, catalyst goes through a cycle in which it is used up and regenerated so that it is used over and over
again.
A catalyst, though involved in a reaction, does not appear in the over-all chemical equation. Its presence is shown
by writing it over the arrow connecting reactants and products.
A catalyst operates by providing an alternate path that has a lower energy of activation for the reaction (Fig. 11).
A catalyst lowers the energies of activation of both the forward and reverse reactions without affecting the enthalpy
of reaction. This leads to increase in the rate of both forward and backward reactions and thereby helps attaining
the equilibrium position of the reaction more rapidly.
From Fig. 11, it follows that
DrH = Ea(f) – Ea(b) (24)
where Ea(f) and Ea(b) are the activation energies of the forward and backward reactions, respectively.
For exothermic reactions Ea(f) < Ea(b)
For enothermic reactions Ea(f) > Ea(b)
These characteristics are shown in Fig. 12
9.12 Complete Chemistry—JEE Main

A catalyst does not change the equilibrium constant of a reaction. Thus, the relative concentrations of reactants
and products at equilibrium are not affected by the use of a catalyst.
A catalyst is said to be inhibitor if it slows down the rate of reaction.

Fig. 11

Fig. 12

MULTIPLE CHOICE QUESTIONS FOR REVISION

1. Ammonia reacts with oxygen giving nitrogen and water. If the rate of formation of N2 is 0.70 mol L–1 s–1, the rate
at which O2 is consumed is
(a) 1.05 mol L–1 s–1 (b) 0.70 mol L–1 s–1 (c) 2.10 mol L–1 s–1 (d) 0.35 mol L–1 s–1
2. The rate constant of a reaction is 1.5 mol–3/2 L3/2 s–1, the order of the reaction is
(a) 1 (b) 1.5 (c) 2.5 (d) 3.0
3. The half-life of a reaction A Æ product is found to be inversely proportional to [A]0 1/2. The order of the reaction is

(a) 1 (b) 1.5 (c) 2.5 (d) 3.0


4. The reaction (CH3)2O(g) Æ CH4(g) + H2
constant volume with p0 as the initial pressure of (CH3)2O. If p is the pressure at time t, then a linear plot will be
observed between
(a) ln(p/p0) versus t (b) ln{(p – p0)/ p0}versus t
(c) ln{(2p – p0)/ p0}versus t (d) ln{(3p – p0)/ (2p0)}versus t
Chemical Kinetics 9.13

5. As per IUPAC (International Union of Pure and Applied Chemistry) the rate of reaction is expressed as
(a) d[reactant]/dt (b) d[product]/dt (c) dx/dt (d) (dx/dt)/V
where x is the extent of reaction.

(a) s–1 (b) mol L–1 s–1 (c) mol L–1 (d) mol–1 L s–1
7. The half-life for the thermal decomposition of acetone is 80 s and is independent of initial concentration of

(a) 186.1 s (b) 206.1 s (c) 150.1 s (d) 226.1 s


8. A living plant disintegrates at the rate of 15 disintegrates per minute per gram of the carbon. After 1.94 ¥ 103 y. the

= –0.097)
(a) 5270 y (b) 5730 y (c) 6026 y (d) 6750 y
9. The rate of reaction becomes double when its temperature is raised from 300 K to 330 K. The activation energy

(a) 18.96 kJ mol–1 (b) 23.96 kJ mol–1 (c) 28.96 kJ mol–1 (d) 33.96 kJ mol–1
10. The reaction NH+4 + OCN– OC(NH2)2, proceeds through the following mechanism.
Keq

NH +4 + OCN -  
 NH 4 OCN (fast)
k
NH +4 OCN ææ
Æ OC(NH 2 ) 2 (slow)

The rate constant of the reaction is


(a) k (b) kKeq (c) k/Keq (d) Keq/k
OH -
11. The kinetic data for the reaction OCl– + I– æææÆ OI– + Cl–

[OCl - ] [I- ] [OH - ] 10-4 ¥ d[IO - ]/ dt


mol dm -3 mol dm -3 mol dm -3 mol dm -3 s -1
0.0017 0.0017 1.0 1.75
0.0034 0.0017 1.0 3.50
0.0017 0.0034 1.0 3.50
0.0017 0.0017 0.5 3.50
The rate low for the formation of OI– is
(a) r = k [OCl–][I–] (b) r = k [OCl–]2
(c) r = k [I–]2[OCl–] (d) r = k [OCl–][I–]/[OH–]

(a) kT2/kT1 (b) kT + 10K/kT (c) kT/kT + 20K (d) kT + 20K /kT
13. Which of the following statements is not correct?
(a) Larger the activation energy, lesser the value of rate constant of a reaction
(b) Larger the temperature, larger the value of rate constant of a reaction
(c) Larger the activation energy, larger the effect of a given temperature increase on the rate constant
(d) At the lower temperature, increase in temperature causes lesser change in the value of rate constant than at
higher temperature

For a reaction A(g) Æ products, the following data are available.


Initial pressure of A, pA/Torr 429 273 150
Half-life period, t1/2/s 350 550 1001
9.14 Complete Chemistry—JEE Main

14. The order of the reaction is


(a) 0 (b) 1 (c) 2 (d) 3
15. The rate constant of the reaction is
(a) 6.66 ¥ 10–2 Torr–1 s–1 (b) 6.66 ¥ 10–4 Torr–1 s–1 (c) 6.66 ¥ 10–6 Torr–1 s–1 (d) 3.33 ¥ 10–6 Torr–1 s–1
16. The rate of reaction at initial pressure of 500 Torr of A is
(a) 1.665 Torr s–1 (b) 2.665 Torr s–1 (c) 5.656 Torr s–1 (d) 9.25 Torr s–1

(a) 5 min (b) 10 min (c) 20 min (d) 12 min


18. The half-life of a reaction A Æ product is increased two times when the concentration of A is double. The
order of the reaction is
(a) 0 (b) 1 (c) 2 (d) 3
19. For a reaction A + B 2 C + D, if the activation energy of the forward and backward reactions are 25 kJ
mol–1 and 35 kJ mol–1 respectively. The enthalpy change of the reaction is
(a) 10 kJ mol–1 (b) –10 kJ mol–1 (c) 60 kJ mol–1 (d) –60 kJ mol–1
20. A catalyst lowers the activation energy from 20 kJ mol–1 to 15 kJ mol–1 of the forward direction of the reaction
A B. Which of the following statement regarding the reaction is correct?
(a) Activation energy of the backward direction is increased by 5 kJ mol–1.
(b) Activation energy of the backward direction is decreased by 5 kJ mol–1.
(c) Enthalpy of reaction is increased by 5 kJ mol–1.
(d) Equilibrium constant of the reaction is decreased.
k
21. The reaction NO 2 + CO ææ
Æ NO + CO 2 follows the mechanism
k
NO 2 + NO 2 ææ
1
Æ NO3 + NO (slow)
k
NO3 + CO ææ 2
Æ NO 2 + CO 2 (fast)
The rate low of the reaction will be
(a) –d[NO2]/dt = k [NO2] [CO] (b) –d[NO2]/dt = k [NO3] [CO]
(c) –d[NO2]/dt = k1[NO2]2 (d) –d[NO2]/dt = k[NO2]2 [CO]
k
22. The reaction 2NO + O 2 ææ
Æ 2NO 2 follows the mechanism
Keq

NO + NO  
 N 2 O2 (in rapid equilibrium)
k2

N 2O2 + O2 
 2NO 2 (slow)
The rate law of the reaction will be
(a) r = k [NO]2[O2] with k = k2Keq (b) r = k [NO2]2[O2] with k = k2/Keq
(c) r = k [NO2]2[O2] with k = Keq/k2 (d) r = k [NO]2[O2] with k = 2k2Keq

ANSWERS
1. (a) 2. (c) 3. (b) 4. (d) 5. (c) 6. (b)
7. (a) 8. (c) 9. (a) 10. (b) 11. (d) 12. (b)
13. (d) 14. (c) 15. (c) 16. (a) 17. (b) 18. (a)
19. (b) 20. (b) 21. (c) 22. (a)
Chemical Kinetics 9.15

HINTS AND SOLUTIONS


1. The reaction is 4NH3 + 3O2 Æ 2N2 + 6H2O
1 d[ N 2 ] 1 d[O 2 ] d[O 2 ] 3 d[ N 2 ] Ê 3 ˆ
=- . Hence, - = = Á ˜ (0.7 mol L-1 s -1 ) = 1.05 mol L-1 s -1
2 dt 3 dt dt 2 dt Ë 2¯
–3 1–n –1
2. The unit of rate constant is (mol dm ) s , where n is the order of reaction. Hence, 1– n = –3/2. This gives
n =1 + 3/2 = 5/2 i.e 2.5.
3. The half-life is t1/2 μ 1/ [A]n–1
0 , where n is the order of reaction. Hence, n –1 = 1/2 i.e. n = 3/2.
4. For a gaseous reaction at constant volume the rate constant is ln {(Dng + 1) p0 – p}/ (Dng p0)} versus t is a straight
Dng = 2. Hence the plot of ln{3(p0 – p)/2p0} versus t will a
straight line

6. The rate law of any reaction is expressed as –d[reactant]/dt or d[product]/dt divided by corresponding
stoichiometric number. Its unit will be mol L–1 s–1.
k = 0.693/t1/2 = 0.693/ (80 s). For the reaction to be 80%,
[A]t/[A]0 = 0.2. Hence

2.303 log{[A]t /[A]0 } (2.303)(log 2 ¥ 10-1 ) (2.303)(0.301 - 1)


t= - =- = - = 186.1 s
k (0.693 / 80 s) (0.693 / 80 s)

2.303 log ( [A]t /[A]0 ) 2.303 log (12 / 15)


8. k = - =- = (2.303)(0.097) / (1.94 ¥ 103 y) = 1.15 ¥ 10-4 y -1
t 1.94 ¥ 103 y
0.693 0.693
t1/ 2 = = = 5026 y
k 1.15 ¥ 10-4 y -1

{log(k2 / k1 )}(2.303R)(T1T2 ) (log 2)(2.303)(8.314 J K -1 mol-1 )(300 K )(330 K )


9. We have Ea = =
T2 - T1 (30 K)
(0.30)(2.303)(8.314)(300)(330)
= J mol-1 = 18956 J mol–1 = 18.96 kJ mol–1
(30)
10. From slow step, we write r = k[NH4OCN]
where [NH4OCN] from the fast equilibrium step is [NH4OCN] = Keq[NH+4][CN–]. Hence, the rate law is
r = (kKeq)[NH+4][CN–]
The rate constant is kKeq.
11. On doubling the [OCl–] keeping the [I–] and [OH–] constant, the rate of formation of [OI–] is doubled, hence the
order of reaction with respect to [OCl–] is one.
On doubling the [I–] keeping the [OCl–] and [OH–] constant, the rate of formation of [OI–] is doubled, hence the
order of reaction with respect to I– is one.
On changing the [OH–] to a half-value, the rate of formation of [OI–] is doubled. Hence, the order of reaction with
respect to [OH–] is –1.
Hence the rate law is r = k[OCl–] [I–][OH–]–1 = k[OCl–] [I–]/[OH–]

by 10 K.
Ê kT ˆ E Ê 1 1 ˆ E T -T E DT
13. We have ln Á 2 ˜ = - a Á - ˜ = a 2 1 = a
Ë kT1 ¯ R Ë T2 T1 ¯ R T2T1 R (T1 + DT )T1
9.16 Complete Chemistry—JEE Main

Since the lower temperature occurs in the denomination, the value kT2/kT1 will be larger on increasing the temperature
by DT.
14. For zero-order reaction t1/2 μ [A]0 t1/2 = constant
For second-order reaction t1/2 μ 1/[A]0 For third-order reaction t1/2 μ 1/[A]20
For the given data, t1/2[A]0
(429 Torr) (350 s) = 150 150 Torr s (273 Torr) (550 s) = 150 150 Torr s
(150 Torr) (1001 s) = 150 150 Torr s
Thus, the reaction is second order.
1 1
15. For the second-order reaction, k = = = 6.66 ¥ 10–6 Torr–1 s–1
t1 / 2 ( pA )0 (429 s)(350 Torr )
16. The rate of reaction at 500 Torr is
r = k (pA)20 = (6.66 ¥ 10–6 Torr–1 s–1) (500 Torr)2 = 1.665 Torr s–1

t t t
[A]0 æææ
1/ 2
Æ [A]0 / 2 æææ
1/ 2
Æ [A]0 / 4 æææ
1/ 2
Æ [A]0 / 8
50% 75% 87.5%
completed completed completed
Hence, 3t1/2 =15 min or t1/2 = 5 min. For 75% completion, t = 2t1/2 = 10 min
18. For a zero-order reaction, the half-life is directly proportional to the initial concentration of A. Hence, the reaction
is of zero order.
19. DH = Ea(f) – Ea(b) = (25 – 35) kJ mol–1 = –10 kJ mol–1.
20. Energy of backward reaction is also decreased by 5 kJ mol–1 without affecting the value of DH and Keq of the
reaction.
2
2]/dt = k [NO2] . Note that the rate law is not divided
by 2 as one molecule of NO2 is recovered in the second step.
1 d[ NO 2 ] k [ N O ][O ]
22. Since the second step is slow, the rate law is = 2 2 2 2
2 dt
[ N 2O2 ]
From the fast equilibrium, we have Keq = . This gives [N2O2] = Keq [NO]2
[ NO]2
1 d[ NO 2 ]
Hence, = k2 Keq [NO]2 [O2]
2 dt

MULTIPLE CHOICE QUESTIONS ON THE ENTIRE CHAPTER

General Characteristics
1. The rate constant does not depend upon
(a) temperature (b) concentration of reactants and products
(c) activation energy (d) catalyst
2. For the reaction 2A Æ 3B, the rate of reaction may be represented as
(a) r = – d[A]/dt = d[B]/dt (b) r = – d[A]/dt = (1/3) d[B]/dt
(c) r = – (1/2) d[A]/dt = (1/3) d[B]/dt (d) r = (1/2) d[A]/dt = – (1/3) d[B]/dt
3. The order of a reaction
(a) is never equal to zero or fraction
(b) is a theoretical parameter and can be predicted from the expression of the over-all chemical equation
(c) is always equal to the total stoichiometric number of the chemical equation
(d) is always determined experimentally
Chemical Kinetics 9.17

4. The units of rate of reaction and its rate constant


(a) are identical
(b) are quite independent of each other
(c) depend on the experimental conditions of the reaction

5. In a reaction 2X + Y Æ X2Y, the reactant X disappears at


(a) half the rate as that of disappearance of Y (b) the same rate as that of disappearance of Y
(c) the same rate as that of appearance of X2Y (d) twice the rate as that of appearance of X2Y
6. The units of rate constant and rate of reaction are identical for

(c) second-order reaction (d) reversible reaction


7. The order of an elementary reaction
(a) is equal to its molecularity (b) cannot be predicted
(c) depends upon temperature (d) depends on the experimental condition
8. The radioactive decay follows

(c) second-order kinetics (d) fractional-order kinetics


9. During the course of a chemical reaction, the rate of a reaction
(a) remains constant throughout (b) increases as the reaction proceeds

10. The reaction 2A + B Æ D + E involves the following mechanism


AÆB (fast)
BÆC (slow)
A+CÆD+E
The rate expression would be
(a) k[A]2[B] (b) k[B] (c) k[A] (d) k[A] [B]
11. The hydrolysis of an ester was carried out separately with 0.05 M HCl and 0.05 M H2SO4. Which of the following
will be true?
(a) kHCl > kH2SO4 (b) kHCl < kH2SO4 (c) kHCl = kH2SO4 (d) kHCl = (1/2) kH2SO4
12. The hydrolysis of an ester was carried out separately with 0.1 N HCl and 0.1 N H2SO4. Which of the following
will be true?
(a) kHCl > kH2SO4 (b) kHCl < kH2SO4 (c) kHCl = kH2SO4 (d) kHCl = (1/2)kH2SO4s
kf f¢
13. The equilibrium reaction A B is started with A such that initially one can write A ækæ ÆB
kb
In such a case, which of the following will be true?
(a) kf = k ¢f
(b) kf > k¢f
(c) kf < k¢f
(d) kf may be greater or smaller than k¢f depending upon the concentration of A
k
14. The equilibrium reaction A f B is started with B such that initially one can write B ææ
b k¢
ÆA
kb
In such a case, which of the following will be true?
(a) kb = k ¢b
(b) kb > k¢b
9.18 Complete Chemistry—JEE Main

(c) kb < k¢b


(d) kb may be greater or smaller than kb depending upon the concentration of B
15. For a reaction N2 + 3H2 Æ 2NH3, the rate of formation of ammonia was found to be 2.0 ¥ 10–4
mol dm–3 s–1. The rate of consumption of H2 will be
(a) 1.0 ¥ 10–4 mol dm–3 s–1 (b) 2.0 ¥ 10–4 mol dm–3 s–1
(c) 3.0 ¥ 10–4 mol dm–3 s–1 (d) 4.0 ¥ 10–4 mol dm–3 s–1
16. For a chemical reaction X Æ Y, the rate of reaction increases by a factor of 1.837 when the concentration of
X is increased by 1.5 times. The order of the reaction with respect to X is
(a) 1 (b) 1.5 (c) 2 (d) 2.5
17. In a reaction 2A + B æÆ A2B, the reactant A will disappear at
(a) half the rate at which B disappears (b) the same rate as that of B
(c) the same rate as that of appearance of A2B (d) twice the rate at which B decreases
18. The terms rate of reaction and rate of appearance (or disappearance) of reactant (or product)
(a) represent one and the same physical quantity
(b) differ by a constant factor
(c) are positive parameters and have same value

balanced chemical equation


19. For the reaction 2A + 3B æÆ 4C, the rate of reaction may be represented as
d [A] d [B] d [ C] d [A] d [B] d [ C]
(a) r = - 2 =-3 =4 (b) r = - 6 =-4 =3
dt dt dt dt dt dt
1 d [A] 1 d [B] 1 d [C] 1 d [A] 1 d [B] 1 d [C]
(c) r = - = = (d) r = - =- =
2 dt 3 dt 4 dt 2 dt 3 dt 4 dt
20. The unit of rate of reaction is
(a) s–1 (b) mol s–1 (c) mol L–1 s–1 (d) mol–1 L s–1
21. The half-life of a reaction A æÆ B varies as the inverse of concentration of A. The order of the reaction would be

22. If kf and kb are the rate constants of forward and backward reactions in an equilibrium reaction, the equilibrium
constant of the reaction is given by
(a) Keq = kf /kb (b) Keq = kb/kf (c) Keq = kf kb (d) Keq = 1/kf kb
23. The reaction A + 2B + C Æ D occurs by the following mechanism
k1
A+B E (rapid equilibrium)
k2

E+C k3 F (slow)
k4
F+B D (very fast)
The rate law for this reaction is
(a) r = k [C] (b) r = k [A] [B]2 [C] (c) r = k [D] (d) r = k [A] [B] [C]
24. The rate equation for 2N2O5 Æ 4NO2 + O2 is r = (6.3 ¥ 10 s ) [N2O5]. The initial rate of decomposition of
–4 –1

0.1 M N2O5 will be


(a) 6.3 ¥ 10–6 mol dm–3 s–1 (b) 6.3 ¥ 10–5 mol dm–3 s–1
(c) 6.2 ¥ 10–4 mol dm–3 s–1 (d) 6.3 ¥ 10–3 mol dm–3 s–1
k1 k2
25. For the reaction A k –1
B and A C, the rate of change of concentration of B is given as
d[B]
(a) d[B]/dt = – k2 [B] (b) = k [C]
dt
(c) d[B]/dt = k1[A] – k–1[B] (d) d[B]/dt = k1[A] + k–1[B] – k2[B]
Chemical Kinetics 9.19

k k
26. For the reactions A ææ
1
Æ B and A ææ 2
Æ C, the ratio of [B]/[C] is given by
(a) r = k1/k2 (b) r = k2/k1 (c) r = k1k2 (d) r = k1/k 22
27. A reaction A Æ B is of the order 2.5 with respect to A. The half-life of the reaction will be given
(a) 0.693/ k (b) 1/([A]0 k)
(c) k/[A] 0 (d) [1/(1.5 k [A]1.5 1.5
0 )] (2 –1)

28. For the reaction 5Br– +BrO3– + 6H+ Æ 3Br2(aq) + 3H2O(l), if the rate of disappearance of Br– is 2.5 ¥ 10–2
mol L–1 s–1, the rate of appearance of Br2 will be
(a) 2.5 ¥ 10–2 mol L–1 s–1 (b) 1.5 ¥ 10–2 mol L–1 s–1
(c) 1.25 ¥ 10–2 mol L–1 s–1 (d) 3.0 mol L–1 s–1
Zero-order and Fractional-order Reactions
29. A substance (initial concentration a) reacts according to zero order kinetics. The time it takes for the completion
of the reaction is
(a) a/k (b) a/2k (c) k/a (d) 2k/a
30. The half-life of a zero order reaction A æÆ B is
(a) directly proportional to the concentration of A (b) independent of the concentration of A
(c) inversely proportional to the concentration of A (d) determined by the concentration of B
31. A zero order reaction A æÆ B is half completed in time equal to
0/2k (d) 2k/[A]0
32. In a kinetic study, the plot of [reactant] versus time is a straight line with a negative slope. The reaction follows

(c) second-order kinetics (d) fractional-order kinetics


1 2 -1 1/ 2
33. For a reaction half-life period is given by t1 / 2 = . The reaction follows
k (1 / 2) [A]01 / 2

(c) fractional order kinetic with order = 3/2 (d) fractional order kinetics with order = 1/2
34. A reaction A ææ k
Æ product follows half-order kinetics with respect to A. A linear plot is observed between
1/2 versus t with slope equal to –k
(a) [A] (b) [A]1/2 versus t with slope equal to –k/2
(c) [A]1/2 versus t with slope equal to –2k (d) [A]3/2 versus t with slope equal to –k
35. A reaction A ææ k
Æ product follows half-order kinetics with respect to A. Its half-life period will be

[A]01 / 2 2[A]01 / 2 2[A]01 / 2 ( 2 - 1) 2 2[A]01 / 2 ( 2 - 1)


(a) ( 2 - 1) (b) ( 2 - 1) (c) (d)
k k k k
First-order Reactions
36. The reaction 2N2O5 (g) Æ 4 NO2(g) + O2(g) provides a linear plot when ln pN2O5 is plotted against t with a
negative slope. The decomposition of N2O5 follows

ÆB is given as
(a) t1/2 = 0.693 k (b) t1/2 = 0.693 ln k (c) t1/2 = 0.693/k (d) t1/2 = log 2/k
38. The unit of rate constant of the decomposition reaction 2N2O5 Æ 2NO2 + O2
to N2O5 is
(a) s–1 (b) mol–1 dm3 s–1 (c) mol dm–3 s–1 (d) mol–2 dm6 s–1

(a) 0.223 min–1 (b) 0.0223 min–1 (c) 2.23 min–1 (d) 22.3 min–1
9.20 Complete Chemistry—JEE Main

40. The half-life of a radioactive isotope is 3 h. What mass out of 100 g is left after 15 h?
(a) 12.5 g (b) 6.25 g (c) 3.125 g (d) 1.562 g

(a) 8 (b) 9 (c) 10 (d) 12

required to reduce initial concentration by a factor of 1/16 will be


(a) 10 min (b) 20 min (c) 30 min (d) 40 min
k = 5.0 ¥ 10 –5 s–1. The rate of the reaction after 3.85 h

when the reaction is started with 1.0 M of A will be


(a) 4.0 ¥ 10–5 mol dm–3 s–1 (b) 3.0 ¥ 10–5 mol dm–3 s–1
(c) 2.5 ¥ 10–5 mol dm–3 s–1 (d) 2.0 ¥ 10–5 mol dm–3 s–1
44. The gaseous reaction A(g) Æ pA = 90
mmHg, the total pressure after 10 min is found to be 180 mmHg. The rate constant of the reaction is
(a) 1.15 ¥ 10–3 s–1 (b) 2.30 ¥ 10–3 s–1 (c) 3.45 ¥ 10–3 s–1 (d) 4.60 ¥ 10–3 s–1
2O5, it is found that
d[ N 2 O5 ]
2N2O5(g) Æ 4NO2(g) + O2(g) – = k [N2O5]
dt
1 d[ N 2 O5 ]
N2O5(g) Æ 2NO2(g) + O2(g) – k¢ [N2O5]
2 dt
Which of the following is true?
(a) k = k¢ (b) k < 2k¢ (c) k = 3k¢ (d) k = 2k¢
O
2 5 , the following information is available.
2N2O5(g) Æ 4NO2(g) + O2(g) rate = k[N2O5]
N2O5(g) Æ 2NO2(g) + 1 O2(g) rate = k¢ [N2O5]
2
Which of the following expressions is true?
(a) k = k¢ (b) k > k¢ (c) k = 3k¢ (d) k¢ = 2k
k
ææ
2O 5 Æ 4NO2 + O2, the half-life period is given by
(a) 2.303/k (b) 0.693/k (c) 2.303/2k (d) 0.693/2k
48. The reaction n1A + n2B æÆ
reaction is started with [A]0 and [B]0, the integrated rate expression of this reaction would be
[A]0 [A]0 [A]0 [A]0
(a) ln = k1t (b) ln = k1t (c) ln = n1k1t (d) ln = n1k1t
[A]0 - x [A]0 - n1 x [A]0 - n1 x [A]0 - n1 x
where x is the extent of reaction divided by constant volume.
æÆ products
(a) the degree of dissociation of reactant is equal to 1 – exp (– kt).
(b) the pre-exponential factor in the Arrhenius equation has the dimension of time.
(c) the time taken for the completion of 75% reaction is thrice of half-life time
(d) a plot of reciprocal of concentration of the reactant versus time is a straight line.

128 mmol L–1 is


(a) (128 mmol L–1)/24 (b) (128 mmol L–1)/25 (c) (128 mmol L–1)/26 (d) (128 mmol L–1)/27
æÆ products?
(a) [A] versus t (b) log {[A]/M} versus t (c) 1/[A] versus t (d) [A]2 versus t
Chemical Kinetics 9.21

52. For a reaction A Æ B, it is found that [A]0 t1/ 2 is constant. The order of the reaction with respect to A is
(a) 0 (b) 1 (c) 2 (d) 3
53. The gaseous phase reaction A(g) Æ
pressure p0 of A, then the pressure pt of the system at time t is given by the expression
3 p0 - pt 3 p0 - pt pt - p0 pt - p0
(a) ln = – kt (b) ln = – kt (c) = – kt (d) = – kt
2 p0 2 p0 p0 p0
54. The rate constant of acid hydrolysis of an ester with pH = 3 is 1.1 ¥ 10–4 s–1. Its rate constant at pH = 2 will be
(a) 1.1 ¥ 10–4 s–1 (b) 11 ¥ 10–4 s–1 (c) 1.1 ¥ 10–2 s–1 (d) 1.1 ¥ 10–1 s–1
55. The rate law for the reaction 2NO(g) + H2 (g) Æ N2O(g) + H2O(g) is given by dp(N2O)/dt = k (pNO)2 (pH2).
If (pNO)0 is very much larger than (pH2)0, then the reaction follows

Second-order Reactions
56. The half-life of a second-order reaction A Æ B is given as
(a) t1/2 = 0.693/k (b) t1/2 = k/[A0] (c) t1/2 = [A]0/k (d) t1/2 = 1/k[A]0
57. How will the rate of reaction 2NO(g) + O2(g) Æ 2NO2(g) get affected if the volume of the reacting system is
2.
(a) Diminishes to one-fourth of its initial value (b) Diminishes to one-eighth of its initial value
(c) Increases four times (d) Increases eight times
58. The half-life period and the initial concentration for a reaction are as follows.
t1/2/s 420 294 735
a/ mmHg 350 500 200
The order of the reaction is
(a) zero (b) one (c) two (d) three
59. For a reaction A Æ products follows a linear plot of 1/[A]t verus t. The reaction follows

60. For a reaction 2NO(g) + H2(g) Æ N2O(g) + H2O(g) the rate law is dp(N2O)/dt = k (pNO)2 (pH2). The half-
life of the reaction when (pNO)0 = 10 mmHg and (pH2)0 = 1200 mmHg is found to be 830 s. The half-life when
(pNO)0 = 20 mmHg and (pH2)0 = 1200 mmHg will be
(a) 830 s (b) 415 s (c) 1245 s (d) 208 s
61. For a second order reaction A Æ product, the plot of
(a) [A]t versus time is linear with slope = k (b) 1/[A]t versus time is linear with slope = – k
2 versus time is linear with slope = k
(c) 1/[A]t (d) 1/[A]t versus time is nonlinear
Temperature Dependence
62. The rate constant of a reaction follows
(a) exponential increase with increase in temperature (b) exponential decrease with increase in temperature
(c) linear increase with increase in temperature (d) linear decrease with increase in temperature
63. The plot of log (k /k°) versus 1/T is linear with a slope of
(a) – Ea/R (b) Ea/R (c) – Ea/2.303 R (d) Ea/2.303 R

(a) the difference in energies of reactants and products


(b) the sum of energies of reactants and products
(c) the difference in energy of intermediate complex with the average energy of reactants and products
(d) the difference in energy of intermediate complex and the average energy of reactants
9.22 Complete Chemistry—JEE Main

65. The logarithm of rate constant of a reaction


(a) increases linearly with increase in inverse of temperature
(b) decreases linearly with increase in inverse of temperature
(c) increases linearly with increase in temperature
(d) decreases linearly with increase in temperature
66. The activation energy for a reaction which doubles the rate when the temperature is raised from 300 K to 310 K is
(a) 50.6 kJ mol–1 (b) 53.6 kJ mol–1 (c) 56.6 kJ mol–1 (d) 59.6 kJ mol–1
67. For the decomposition of N2O5(g), it is given that
2N2O5(g) Æ 4NO2 (g) + O2(g) activation energy Ea
N2O5(g) Æ 2NO2(g) + 1
2
O2(g) activation energy E a¢
then
(a) Ea = E a¢ (b) Ea > E a¢ (c) Ea < E a¢ (d) Ea = 2E a¢
68. By increasing the temperature by 10 °C, the rate of forward reaction at equilibrium is increased by a factor of 2.
The rate of backward reaction by this increase in temperature
(a) remains unaffected (b) increases by a factor greater than two
(c) decreases by a factor lesser than two (d) is also increased by a factor of two
69. For an exothermic reaction A æÆ B, the activation energy is 65 kJ mol–1 and enthalpy of reaction is 42 kJ
mol–1. The activation energy for the reaction B æÆ A would be
(a) 23 kJ mol–1 (b) 107 kJ mol–1 (c) 65 kJ mol–1 (d) 42 kJ mol–1
70. The rate constant, the activation energy and the Arrhenius parameters of a chemical reaction at 25 °C are 2.0 ¥ 10–5
s–1, 100 kJ mol–1 and 6.0 ¥ 1014 s–1, respectively. The value of rate constant at very high temperature approaches
(a) 2.0 ¥ 10–5 s–1 ¥ 1014 s–1 (d) 12 ¥ 10–9 s–1
71. The rate constant of a reaction (activation energy E1) is twice as the rate constant of a second reaction (activation
energy E2). Which of the following conditions will hold good if the Arrhenius parameters A1 =A2?
(a) E1 = E2 (b) E1 < E2 (c) E1 > E2 (d) E1 = (1/2) E2
–1
72. The activation energy of a reaction is 65.8 kJ mol , on changing the temperature from 2 °C to 27 °C, its rate
constant changes by

73. The order of a reaction with respect to OH– is –1. The OH– species
(a) act as a catalyst (b) act as an inhibitor
(c) will always appeared in the chemical equation (d) help in increasing the rate of reaction
74. The use of a catalyst helps in
(a) increasing the rate of forward reaction only
(b) increasing the rate of backward reaction only
(c) increasing the rates of both forward and backward reactions
(d) increasing the relative amounts of products
75. A catalyst lowers the activation energy of the forward reaction by 10 kJ mol–1. What effect it has on the activation
energy of the backward reaction?
(a) Increase by 10 kJ mol–1 (b) Decrease by 10 kJ mol–1
(c) Remains unaffected (d) Cannot be predicted
76. A catalyst is a substance which
(a) increases the equilibrium concentrations of the products
(b) decreases the energy of activation
(c) does not alter reaction mechanism
(d) increases the frequency of collision of reacting species
Chemical Kinetics 9.23

77. In the Haber’s process of the synthesis of ammonia, the use of catalyst helps
(a) increasing rate constant without changing the equilibrium amount of NH3
(b) increasing rate constant with increasing the equilibrium amount of NH3
(c) decreasing rate constant with increasing the equilibrium amount of NH3
(d) decreasing rate constant with decreasing the equilibrium amount of NH3

ANSWERS
1. (b) 2. (c) 3. (d) 4. (d) 5. (d) 6. (a)
7. (a) 8. (b) 9. (c) 10. (b) 11. (b) 12. (a)
13. (a) 14. (a) 15. (c) 16. (b) 17. (d) 18. (d)
19. (d) 20. (c) 21. (c) 22. (a) 23. (d) 24. (b)
25. (c) 26. (a) 27. (d) 28. (b) 29. (a) 30. (a)
31. (c) 32. (a) 33. (c) 34. (b) 35. (b) 36. (b)
37. (c) 38. (a) 39. (b) 40. (c) 41. (c) 42. (b)
43. (c) 44. (a) 45. (a) 46. (d) 47. (d) 48. (c)
49. (a) 50. (c) 51. (b) 52. (d) 53. (a) 54. (b)
55. (a) 56. (d) 57. (b) 58. (c) 59. (c) 60. (b)
61. (b) 62. (a) 63. (c) 64. (d) 65. (b) 66. (b)
67. (a) 68. (d) 69. (b) 70. (c) 71. (b) 72. (d)
73. (b) 74. (c) 75. (b) 76. (b) 77. (a)

HINTS AND SOLUTIONS


1 d[X] d[X 2 Y] d[X] Ê d[X 2 Y] ˆ
5. rate = – = or - = 2 ÁË ˜
2 dt dt dt dt ¯
10. Slow step is the rate determining step.
11. For hydrolysis of an ester kapp = ktrue [H+]
Since [H+] in HCl is smaller than in H2SO4, it follows that (kapp)HCl < (kapp)H2SO4
12. In 0.1 N HCl and 0.1 N H2SO4 acids, [H+] in the former is larger since HCl ionizes in a single step whereas H2SO4
ionizes in two steps.
13. kf does not depend upon the concentration of A.
d[ N 2 ] 1 d[H 2 ] 1 d[ NH3 ]
15. We will have – =– =
dt 3 dt 2 dt
d[ NH3 ] = 2 ¥ 10–4 mol dm–3 s–1. Hence, d [H 2 ] 3 d [ NH3 ]
It is given that - = = 3 ¥ 10-4 mol dm -3 s -1
dt dt 2 dt
log 1.837
16. (1.5)n = 1.837; n= = 1.5
log 1.5
23. From the slow step, we write rate = k3[E][C]
k1 [E]
Keq = = fi [E] = Keq [A][B]
k2 [A][B]

Hence, rate = k3 Keq [A][B][C] = k[A][B][C]

24. Initial rate = (6.3 ¥ 10–4 s–1) [N2O5]0 = (6.3 ¥ 10–4 s–1) (0.1 M) = 6.3 ¥ 10–5 M s–1
9.24 Complete Chemistry—JEE Main

d [B] d [ C] d [B]/ dt k
26. = k1[A] and = k2[A]. Hence, = 1
dt dt d[C]/dt k2

d[A] d[A] 1 1
27. - = k[A]2.5. Hence, - = k dt. Thus - = (1.5)kt
dt [A]2.5 [A]1.5
[A] 1.5
0

For t0.5, [A] = [A]0 /2. Hence

1 1 1
1.5
- = 1.5kt0.5 fi t0.5 = (21.5 – 1)
([A]0 /2) [A]1.5
0
1.5
1.5 k [A] 0

1 d[Br - ] 1 d[Br2 ]
28. We have - =
5 dt 3 dt

d[Br2 ] 3 d[Br - ] 3
Hence, = - = - (– 2.5 ¥ 10–2 mol L–1 s–1) = 1.5 ¥ 10–2 mol L–1 s–1
dt 5 dt 5
[A]
d [A]
29. –
dt
=k i.e. – Ú d [A] = k Ú dt
0
i.e. [A]0 – [A] = kt
[ A ]0

At tcompletion, [A] = 0. Hence, t = a/k


d [A ]
32. For a zero-order reaction - = k . Thus [A]t – [A]0 = –kt
dt
33. We have
[A ]t
d [A ] d [A ]
- = k [ A ]3 2 or Ú 3 2 = - kt
[A ] [A ]
dt
0

1 1 1 2 -1
Thus ÈÎ[A]t-1 2 - [A]0-1 2 ˘˚ = kt and t1/2 = {([A]0 / 2) -1 2 - [A ]0-1 2 } =
(1 2) k (1 2) k (1 2) [A ]10 2

d [A ] k
34. - = k [ A ]1 2 . This gives [A ]1t 2 - [A ]10 2 = - t
dt 2

[ A ]0 2 - [ A ]1 2 = -
k
or [ A ] 12Ê ˆ1 k
- 1˜ = - t1 2 or t1/2 =
2 [ A ]10 2 ( )
2 -1
35. t1 2 0 Á
2 ¯
Ë
0
2 2 k
0 – ln [A]t = kt i.e. ln [A]t = ln[A]0 – kt

[A]t ln 2 0.693
ln = – kt. At t0.5, [A]t = [A]0/2. Hence, t0.5 = =
[A]0 k k
[A]t k k
39. log =– t i.e. log 0.8 =– (10 min)
[A]0 2.303 2.303
2.303
or k=– log (0.8) = 0.0223 min–1
10 min
[A]0 100 g
5
= = 3.125 g
2 32
0.1
= – kt99.9
100
1 t0.99 log (10-3 ) -3
= – kt0.5. Hence = = = 10
2 t0.5 log (0.5) -0.3
Chemical Kinetics 9.25

kt kt (5.0 ¥ 10-5 s -1 ) (3.85 ¥ 60 ¥ 60 s)


43. log ([A]t /M) = log ([A]0/M) – =– =– = – 0.300
2.303 2.303 2.303
[A]t = 0.500 mol dm–3
Rate = k[A]t = (5 ¥ 10–5 s–1) (0.5 mol dm–3) = 2.5 ¥ 10–5 mol dm–3 s–1
44. A(g) Æ 2B(g) + C(g)
p0 – p 2p p
Total pressure = (p0 – p) + (2p) + p = p0 + 2p
i.e.180 mmHg = 90 mmHg + 2p or p = 45 mmHg
p -p k
Now log 0 =– t
p0 2.303
45 k 0.301 ¥ 2.303
log =– (10 ¥ 60 s). Thus k= = 1.155 ¥ 10–3 s–1
90 2.303 10 ¥ 60 s
45. For the reaction 2N2O5(g) Æ 4NO2(g) + O2(g), the rate expression is
1 d[ N 2 O5 ] d [N 2 O5 ]
– = k¢¢ [N2O5] i.e. – = 2k¢¢ [N2O5] = k [N2O5]
2 dt dt
1 d[ N 2 O5 ]
and for the reaction N2O5(g) Æ 2NO2(g) + O2(g), the rate expression is – = k¢ [N2O5]
2 dt
Since –d[N2O5]/dt does not depend upon the way the chemical reaction is formulated, we will have
k = k¢
2N2O5(g) Æ 4NO2(g) + O2(g); 1 d[N 2 O5 ]
46. For - = k[ N 2 O5 ]
2 dt
d[N 2 O5 ]
For N2O5(g) Æ 2NO2(g) + 1
2
O2(g); - = k¢ [ N 2 O5 ]
dt
Since –d[N2O5]/dt does not depend upon the way the chemical reaction is formulated, we will have
k¢ = 2k.
47. For 2N2O5 Æ 4NO2 + O2, we have
1 d[N 2 O5 ] d[N 2 O5 ]
- = k[ N 2 O5 ] or - = 2k dt
2 dt [N 2 O5 ]
On integrating, we get
[N 2 O5 ]0 ln 2 0.693
ln = 2kt . For a half-life, [N2O5]t = (1/2) [N2O5]0. Hence t1/ 2 = =
[N 2 O5 ]t 2k 2k
48. We have
[A] t
1 d[A] d[A]
-
n1 d t
= k [A] i.e. Ú [A]
= – n1 k Ú dt
[A]0 0

[A] [A]0
i.e. ln = – n1 kt or ln = n1 kt
[A]0 [A]0 - n1 x

[A] [A] [A]0 (1 - a )


ln = – kt or = e– kt i.e = e- kt i.e a = 1 – e– kt
[A]0 [A]0 [A]0

50. 30 min is equal to six times the half-life of 5 min. Hence, the concentration will be reduced by 26 times.

0/M} – kt
9.26 Complete Chemistry—JEE Main

52. t1/2 μ 1/[A] 0n–1 where n is the order of the reaction. If n = 3, then t1/2 μ 1/[A] 02 or (t1/2) ([A]0)2 = constant
53. A(g) 2B(g) + C(g)
p0 – p 2p p
pt = (p0 – p) + 2p + p = p0 + 2p
[A] p0 - p p0 - ( pt - p0 )/ 2 3 p0 - pt
ln = – kt fi ln = – kt fi ln = – kt fi ln = – kt
[A]0 p0 p0 2 p0
+]. Hence
[H + ]pH = 2 1.1 ¥ 10-4 s -1 -2
kpH = 2 = (kpH =3 ) = ÈÎ10 ˘˚ = 11 ¥ 10–4 s
[H + ]pH =3 ÈÎ10-3 ˘˚
55. When (pNO)0 is very much larger than (pH2)0 2.
56. For a second order reaction
1 1 1
– = kt At t0.5, [A]t = [A]0/2. Hence, t0.5 =
[A]t [A]0 k [A]0
57. On increasing the volume to a twice value, the concentrations of each species is reduced by a factor of 2. Hence
[ NO] ˆ 2 Ê [O 2 ]ˆ
rate2 = k ÊÁ
rate2 1
rate1= k [NO]2 [O2] and ˜ Á ˜ i.e. =
Ë 2 ¯ Ë 2 ¯ rate1 8
58. For a second-order kinetics, [A]0 t0.5 is constant. Hence, a t0.5/mmHg s = [420 ¥ 350 = 294 ¥ 500 = 200 ¥ 735]
1 1
59. For a second-order kinetics - = kt
[A ]t [A ]0
( )
60. Since pH 2 >> ( pNO )0 the reaction follows second-order kinetics with respect to NO. Thus, if the pressure of NO
is doubled, it half-life becomes half.
62. According to the Arrhenius equation k = A exp(– Ea/RT) ,that is, k increases exponentially with increase in
temperature.
63. Arrhenius equation in the logarithm form is
Êkˆ Ê Aˆ Êkˆ
log Á ˜ = log ÊÁ ˆ˜ –
E A Ea 1
ln Á ˜ = ln Á ˜ – a or
Ë kº¯ Ë kº¯ RT Ë k º ¯ Ë kº ¯ 2.303 R T
2.303 ¥ 8.314 ¥ 300 ¥ 310
log 2 = ÊÁ log 2ˆ˜ J mol-1 = 53600 J mol–1 = 53.6 kJ mol–1
2.303RT1T2 k
66. Ea =
DT k1 Ë 10 ¯
68. At equilibrium, Rate of forward reaction = Rate of backward reaction

Obviously, Ea (b) = Ea(f) + DH = (65 + 42) kJ mol–1


70. We have k = Ae–Ea/RT. When T Æ •, k Æ A.
E Ê 1 1ˆ 65.8 ¥ 103 J mol-1 Ê 1 1 ˆ 65.8 ¥ 103 ¥ 25
72. ln (k2 /k1) = – a Á - ˜ = - Á - ˜ = = 2.40
R Ë T2 T1 ¯ (8.314 J K -1 mol-1 ) Ë 300 K 275 K ¯ 8.314 ¥ 300 ¥ 275
k2/k1 = exp(2.40) = 11.0
Chemical Kinetics 9.27

MULTIPLE CHOICE QUESTIONS FROM AIEEE AND JEE MAIN

1. The rate law for a reaction between the substances A and B is given by
rate = k[A]n [B]m
On doubling the concentration of A and reducing the concentration of B to half, the ratio of the new rate to the
earlier rate of the reaction will be as
(a) 2(n – m) (b) 1/2(m + n) (c) (m + n) (d) (n – m) [2003]
2. For the reaction system 2NO(g) + O2(g) Æ 2NO2(g) volume is suddenly reduced to half its volume by
2 and second order with respect to NO,
the rate of reaction will
(a) increase to four times of its initial value (b) diminish to one-fourth of its initial value
(c) diminish to one-eighth of its initial value (d) increase to eight times of its initial value [2003]
3. In respect of the equation k = A exp(–Ea/RT) in chemical kinetics, which one of the following statements is correct?
(a) R is Rydberg’s constant (b) k is equilibrium constant
(c) A is adsorption factor (d) Ea is the energy of activation [2003]
4. The half-life of a radioactive isotope is three hours. If the initial mass of the isotope were 256 g, the mass of it
remaining undecayed after 18 hours would be
(a) 16.0 g (b) 4.0 g (c) 8.0 g (d) 12.0 g [2003]

taken for the concentration to change from 0.1 M to 0.025 M is


(a) 30 minutes (b) 15 minutes (c) 7.5 minutes (d) 60 minutes [2004]
6. The rate equation for the reaction 2A + B Æ k[A][B]. The correct statement in
relation to this reaction is that the
(a) unit of k must be s–1
(b) t1/2 is a constant
(c) rate of formation of C is twice the rate of disappearance of A
(d) value of k is independent of the initial concentrations of A and B. [2004]
7. The half-life of a radioisotope is four hours. If the initial mass of the isotope was 200 g, the mass remaining after
24 hours undecayed is
(a) 1.042 g (b) 2.084 g (c) 3.125 g (d) 4.167 g [2004]
8. Consider an endothermic reaction X ÆY with the activation energies Eb and Ef for the backward and forward
reactions, respectively. In general
(a) Eb = Ef Eb and Ef
(c) Eb < Ef (d) Eb > Ef [2005]
9. A reaction involving two different reactants can never be
(a) second order reaction (b) bimolecular reaction
[2005]
10. t1/4 can be taken as the time taken for the concentration of a reactant to drop to 3/4 of its value. If the rate constant
k, the t1/4 can be written as
(a) 0.69/k (b) 0.75/k (c) 0.10/k (d) 0.29/k [2005]
11. A reaction was found to be second order with respect to the concentration of carbon monoxide. If the concentration
of carbon monoxide is doubled, with everything else kept the same, the rate of reaction will
(a) double (b) remain unchanged
(c) triple (d) increase by a factor of 4 [2006]
9.28 Complete Chemistry—JEE Main

k = A e–E/RT
In this equation, E represents
(a) the fraction of molecules with energy greater than the activation energy of the reaction
(b) the energy above which all the colliding molecules will react
(c) the energy below which colliding molecules will not react
(d) the total energy of the molecules at a temperature, T [2006]
13. The following mechanism has been proposed for the reaction of NO with Br2 to form NOBr.
NO(g) +Br2 (g) NOBr2(g)
NOBr2(g) + NO (g) 2NOBr(g)
If the second step is the rate determining step, the order of the reaction with respect to NO(g) is
(a) 2 (b) 1 (c) 0 (d) 3 [2006]
14. The energies of activation for forward and reverse reactions for A2 + B2 2AB are 180 kJ mol–1 and
200 kJ mol–1, respectively. The presence of a catalyst lowers the activation energy of both (forward and reverse)
reactions by 100 kJ mol–1. The enthalpy change of the reaction (A2 + B2 Æ 2AB) in the presence of catalyst will be
(a) 300 kJ mol–1 (b) 120 kJ mol–1 (c) 280 kJ mol–1 (d) 20 kJ mol–1 [2007]
15. Consider the reaction, 2A +B Æ Products
When concentrations of B alone was doubled, the half-life did not charge. When the concentrations of A alone
was doubled, the rate increased by two times. The unit of rate constant for this reaction is
(a) L mol–1 s–1 (b) no unit (c) mol L–1 s–1 (d) s–1 [2007]
16. For a reaction 1 A Æ 2B, the rate of disappearance of A is related to the rate of appearance of B by the expression
2
(a) –d[A] / dt = 4d[B] / dt (b) –d[A]/ dt = (1/2)d[B] / dt
(c) –d[A] / dt = (1/4)d[B] / dt (d) –d[A]/ dt = d[B] / dt [2008]

(a) 46.06 minutes (b) 460.6 minutes (c) 230.6 minutes (d) 23.03 minutes [2009]
18. Consider the reaction Cl2(aq) + H2S(aq) Æ S(s) + 2H (aq) + 2Cl (aq)
+ –

The rate equation for the reaction is rate = k[Cl2][H2S]


Which of the following mechanisms is/are consistent with this rate equation?
(A) Cl2 + H2S Æ H+ + Cl– + Cl+ + HS– (slow)
Cl + HS Æ H + Cl + S
+ + + –
(fast)
(B) H2S H+ + HS– (fast equilibrium)
Cl2 + HS– Æ 2Cl– + H+ + S (slow)
(a) A only (b) B only
(c) Both (A) and (B) (d) Neither (A) nor (B) [2010]
19. The time for half period of a certain reaction A Æ Products is 1 h when the initial concentration of the reactant
A is 2.0 mol L–1. How much time does it take for its concentration to come from 0.50 to 0.25 mol L–1 if it is a zero
order reaction?
(a) 1 h (b) 4 h (c) 0.5 h (d) 0.25 h [2010]
20. The rate of a chemical reaction doubles for every 10 °C rise of temperature. If the temperature is raised by 50 °C,
the rate of reaction increases by about
(a) 64 times (b) 10 times (c) 24 times (d) 32 times
[2011, (Cancelled)]

1 k
A ææ Æ B; Activation energy Ea1
2 k
A ææ Æ C; Activation energy Ea2
Chemical Kinetics 9.29

If Ea/2 = 2 Ea1, then k1 and k2 are related as


(a) k1= 2k2 eEa2/RT (b) k1= k2 eEa1/RT (c) k2= k1 eEa2/RT (d) k1= Ak2 eEa1/RT [2011]
Æ products, the concentration of A changes from 0.1 M to 0.025 M in 40 min.
The rate of reaction when the concentration of A is 0.01 M is
(a) 1.73 ¥ 10–5 mol dm–3 min–1 (b) 3.47 ¥ 10–4 mol dm–3 min–1
(c) 3.47 ¥ 10–5 mol dm–3 min–1 (d) 1.73 ¥ 10–4 mol dm–3 min–1 [2012]
23. The rate of a reaction doubles when its temperatures changes from 300 K to 310 K. The activation energy of such
a reaction will be (R = 8.314 J K–1 mol–1 and log 2 = 0.301)
(a) 53.6 kJ mol–1 (b) 48.6 kJ mol–1 (c) 58.5 kJ mol–1 (d) 60.5 kJ mol–1 [2013]
24. For the non-stoichiometric reaction 2A + B Æ C + D, the following kinetic data were obtained in three separate
experiments, all at 298 K.
Initial concentration of A Initial concentration of B Initial rate of formation of C in
mol L–1 s–1
0.1 M 0.1 M 1.2 ¥ 10–3
0.1 M 0.2 M 1.2 ¥ 10–3
0.2 M 0.1 M 2.4 ¥ 10–3
The rate law for the formation of C is
(a) d[C]/ dt = k[A] (b) d[C]/ dt = k[A] [B] (c) d[C]/ dt = k[A]2 [B]
(d) d[C]/ dt = k[A] [B]2
[2014]
25. In the reaction of formation of SO3 by contact process 2SO2 + O2 2SO3, the rate of change of [O2] was
measured as d[O2]/dt = –2.5 ¥ 10 mol L s . The rate of change of [SO2] in mol L–1 s–1 will be
–4 –1 –1

(a) –1.25 ¥ 10–4 (b) –2.50 ¥ 10–4 (c) –3.75 ¥ 10–4 (d) –5.00 ¥ 10–4
[2014, online]
26. For the reaction, 2N2O5Æ 4NO2 + O2, the rate equation can be expressed in term of
d[ N 2 O5 ] d[ NO 2 ]
- = k [ N 2 O5 ] and = k ¢[ N 2 O5 ]
dt dt
The constant k and k¢ are related as
(a) k = k¢ (b) 2k = k¢ (c) k = 2k¢ (d) k = 4k¢
[2014, online]
–4 M–1 s–1 10–3 M–1 s–1 at
27. The constant (k) for a particular reaction is 1.3¥ 10 at 100 °C and 1.3¥ 150 °C. What is
the energy of activation (Ea) in kJ mol–1 for the reaction? (R = gas constant = 8.314 J K–1 mol–1).
[2014, online]
28. For the reaction 3A + 2B Æ C + D, the differential rate law can be written as
1 d[A] d[C] d[A] d[C]
(a) = = k[A]n [B]m (b) - = = k[A]n [B]m
3 dt dt dt dt
1 d[A] d[C] 1 d[A] d[C]
(c) + =- = k[A]n [B]m (d) - = = k[A]n [B]m [2014, online]
3 dt dt 3 dt dt
29. The reaction 2N2O5(g) Æ 4NO2(g) + O2
N2O5 was found to increase from 50 mmHg to 87.5 mmHg in 30 min. The pressure exerted by the gases after 60
min will be (Assume temperature remains constant)
(a) 106.25 mmHg (b) 116.25 mmHg
(c) 125 mmHg (d) 150 mmHg [2015, online]
30. For the equilibrium A(g) B(g), DH is – 40 kJ/mol. If the ratio of activation energies of the forward (Ef) and
reverse (Eb) is 2/3, then
(a) Ef = 60 kJ/mol ; Eb = 100 kJ/mol (b) Ef = 30 kJ/mol ; Eb = 70 kJ/mol
(c) Ef = 80 kJ/mol ; Eb = 120 kJ/mol (d) Ef Eb = 30 kJ/mol [2015, online]
9.30 Complete Chemistry—JEE Main

31. A + 2B Æ C, the rate equation for the reaction is given as Rate = k [A] [B]
If the concentration of A is kept the same but that of B is doubled, what will happen to the rate itself?
(a) halved (b) the same (c) doubled (d) quadrupled
[2015, online]

(a) low probability of simultaneous collision of all the reacting species


(b) increase in entropy and activation energy as more molecules are involved.
(c) shifting of equilibrium towards reactants due to elastic collisions
(d) loss of active species on collisions [2015]
33. Match the catalysts to the correct processes.
Catalyst Process
(A) TiCl3 (i) Wacker process
(B) PdCl2 (ii) Ziegles-Natta polymerization
(C) CuCl2 (iii) Contact process
(D) V2O5 (iv) Deacon’s process
(a) (A) – (iii), (B) – (ii), (C) – (iv), (D) – (i) (b) (A) – (ii), (B) – (i), (C) – (iv), (D) – (iii)
(c) (A) – (ii), (B) – (iii), (C) – (iv), (D) – (i) (d) (A) – (iii), (B) – (i), (C) – (ii), (D) – (iv) [2015]
34. The reaction of ozone with oxygen atoms in the presence of chlorine atoms can occur by a two step process shown

(i) O3 (g) + Cl∑ (g ) Æ O 2 (g ) + ClO∑ (g ) kI = 5.2 ¥ 109 L mol–1 s–1


∑ ∑ ∑
(ii) ClO (g ) + O (g ) Æ O 2 (g ) + Cl (g) kII = 2.6 ¥ 1010 L mol–1 s–1 [2016, online]
The closest rate constant for the overall reaction O3(g) + O (g) Æ 2O2(g) is
(a) 1.4 ¥ 1020 L mol–1 s–1 (b) 3.1 ¥ 1010 L mol–1 s–1 (c) 5.2 ¥ 109 L mol–1 s–1 (d) 2.6 ¥ 1010 L mol–1 s–1
35. The rate law for the reaction below is given by the expression k[A][B]
A+B Product
If the concentration of B is increased from 0.1 to 0.3 mole, keeping the value of A at 0.1 mole, the rate constant
will be
(a) 3 k (b) 9 k (c) k/3 (d) k [2016, online]
36. Decomposition of H2O2 2O2 decreases
from 0.5 M to 0.125 M in one such decomposition. When the concentration of H2O2 reached 0.05 M, the rate of
formation of O2 will be
(a) 6.93 ¥ 10–4 mol L–1 min–1 (b) 2.66 L min–1 at STP
(c) 1.34 ¥ 10 –2 mol L–1 min–1 (d) 6.93 ¥ 10–2 mol L–1 min–1 [2016]

ANSWERS
1. (a) 2. (d) 3. (d) 4. (b) 5. (a) 6. (d)
7. (c) 8. (c) 9. (c) 10. (d) 11. (d) 12. (c)
13. (a) 14. (d) 15. (a) 16. (c) 17. (a) 18. (a)
19. (d) 20. (d) 21. (d) 22. (b) 23. (a) 24. (a)
25. (d) 26. (b) 27. (b) 28. (d) 29. (a) 30. (c)
31. (c) 32. (a) 33. (b) 34. (c) 35. (a) 36. (a)

HINTS AND SOLUTIONS


m
Ê [B] ˆ r2
1. r1 = k [A]n [B]m and r2 = k [2A]n ÁË ˜¯ = 2(n – m) k [A]n [B]m. Hence, = 2(n – m)
2 r1
r2
2. r1 = k [O2] [NO]; r2 = k (2[O2])(2 [NO])2 = 8k [O2] [NO]2. Hence, =8
r1
Chemical Kinetics 9.31

4. 18 hours implies six half-lives. Hence, the mass remaining undecayed will be m = 256/26 = 4.0 g
5. The decrease of concentration from 0.8 M to 0.4 M implies that t1/2 = 15 min. The change in concentration from
0.1 M to 0.025 M will require two half-lives. Hence, t = 2t1/2 = 30 min.
6. For the reaction 2A + B Æ C, the rate law is r = k [A] [B]
The unit of k will be mol–1 L s–1. The half-life period, t1/2 is not constant.
The rate of formation of C will be half of the rate of disappearance of A. The rate constant will be independent of
initial concentrations of A and B.
7. Twenty four hours is equivalent to six half-lives. Mass of remaining radioisotope = (1/26) (200 g) = 3.125 g.
8. Endothermic reaction implies the absorption of heat when reactants are converted into products. This implies that
the enthalpy of products (EP) is more that of reactants (ER) i.e. EP > ER.
Since Ef = Etransition state– ER and Eb = Etransition state– EP , it follows that Eb < Ef
9. Unimolecular reaction involves one molecule of reactants.
[A]t 1 3 0.29
10. We have ln = - kt. Thus t1 / 4 = - ln 
[A]0 k 4 k
2 2
11. r1 = k[CO] and r2 = k(2[CO]) , Hence, r2 / r1 = 4
12. The choice (c) is correct.
13. The steps are
 [ NOBr2 ]
NO(g) + Br2(g)    NOBr2(g); Keq =
[ NO][Br2 ]
k
NOBr2 + NO(g) ææ Æ 2NOBr(g); (slow step)
The rate expression is
rate = k[NOBr2][NO] = k{Keq [NO] [Br2]}[NO] = k Keq [NO]2 [Br2]
Hence, the order of the reaction with respect to NO is 2.
14. The enthalpy of reaction is not changed in the presence of a catalyst. Hence
DrH = Ea(b) – Ea(f) = 200 kJ mol–1 – 180 kJ mol–1 = 20 kJ mol–1

the concentration of A. Thus, the reaction is second order. Hence, the unit of rate constant is
unit of rate of reaction mol L-1 s -1
unit of rate constant = = = mol–1 L s–1
(unit of concentration)2 (mol L-1 s -1 )
16. For the given chemical equation, we have
1 d[A] 1 d[B] 1 d[A] 1 d[B] d[A] 1 d[B]
- = i.e. - = . Hence, - =
VA dt VB dt (1 / 2) dt 2 dt dt 4 dt
0.693 0.693
17. k = = = 0.1 min -1
t1 / 2 6.93 min
1 ˆ 2 ¥ 2.303
log ÊÁ
2.303 [A]t 2.303
t =- =- = = 46.06 min
Ë 100 ˜¯ 0.11 min -1
log
k [A]0 0.1 min -1
18. For the mechanism A, the rate equation is r = k[Cl2] [H2S]
For the mechanism B, the rate equation is r = k[Cl2] [HS–]
From the fast equilibrium, we have
[H + ][HS- ] K eq [H 2S]
Keq = fi [HS- ] =
[H 2S] [H + ]
Thus r = k Keq [Cl2] [H2S]/[H+]

19. For zero-order reaction [A]0 – [A]t = kt


Its half-life time as [A]0 – [A]0 /2 = kt1/2 i.e. t1/2= [A]0 /2k
For given concentration, we have
[A]0 – 0.25 M = kt2 and [A]0 – 0.5 M = kt1
9.32 Complete Chemistry—JEE Main

These give 0.25 M = k(t2 – t1)


0.25 M 0.25 M 0.25 M
or Dt = t2 – t1 = = = = 0.25 h
k [A]0 / 2t1 / 2 2.0 M/(2 ¥ 1 h )
Since initial concentration is decreased four times, the half-life period will also decrease four times, i.e. from 1 h
to 0.25 h.
20. We will have
r1 ææ Æ 2r1 ææ Æ 4r1 ææ Æ 8r1 ææ Æ 16r1 ææ Æ 32r1
(t ) (t + 10 ∞ C) (t + 20 ∞ C) (t + 30 ∞ C) (t + 40 ∞ C) (t + 50 ∞ C)
1 1 1 1 1 1
In short, r= 25 r1 = 32 r1
21. We have k1 = A1e- Ea1 / RT and k2 = A2 e- Ea 2 / RT = A2 e-2 Ea1 / RT
k1 A1 e- Ea1 / RT A ÊAˆ
Hence = = 1 e Ea1 / RT or k1 = Á 1 ˜ k2 e Ea1 / RT = Ak2 e Ea1 / RT
k2 A2 e-2 Ea1 / RT A2 Ë A2 ¯
1/ 2 t 1/ 2 t
22. For M of A æææ Æ 0.05 M of A æææ Æ 0.025 M of A
Hence, 2t1/2 = 40 min or t1/2= 20 min. The rate constant of the reaction would be
0.693 0.693
k= = = 0.03465 min -1
t1 / 2 20 min
The rate of rejection when [A] = 0.01 M would be
r = k [A] = (0.03465 min–1) (0.01 mol dm–3) = 3.465 ¥ 10–4 mol dm–3 min–1
23. We have k = A e- Ea / RT fi ln (k / k ∞) = ln ( A / k ∞) - Ea / RT
Ea Ê 1 1 ˆ Ea Ê T2 - T1 ˆ RT1T2 È k ˘
fi ln (k2/k1) =
R ÁË T - T ˜¯ = R ÁË T T ˜¯ fi Ea =
T2T1 ÎÍ 2.303 log 2 ˙
k1 ˚
1 2 1 2

Substituting the given values we get


È (8.314 J K -1 mol-1 )(300 K )(310 K ) ˘
Ea = Í ˙ (2.303 log 2) = 53.599 J mol–1 = 53.6 kJ mol–1
Î (310 K - 300 K ) ˚
24. On changing the concentration of B to twice without changing the concentration of A, there is no change in the
rate of reaction. Hence, the order of the reaction with respect to B is zero.
On changing the concentration of A to twice without changing the concentration of B, the rate of reaction becomes
twice. Hence, the order of the reaction with respect to A is one.
25. For the reaction 2SO2 + O2 Æ 2SO3, the rate of reaction is
1 d[SO 2 ] d[O 2 ] d[SO 2 ]
- =– = 2.5¥ 10–4 mol L–1 s–1. Hence, - = 5.0 ¥ 10–4 mol L–1 s–1
2 dt dt dt
26. For the reaction 2N2O5 Æ 4NO2 + O2, the rate of expression is
1 d[ N 2 O5 ] 1 d[ NO 2 ]
- = = k ¢¢[ N 2 O5 ] (1)
2 dt 4 dt
d[ N 2 O5 ] 1 d[ N 2 O5 ] k
It is given that - = k[ N 2 O5 ]; Hence, - = [ N 2 O5 ]
dt 2 dt 2
d[ NO 2 ] 1 d[ NO 2 ] k ¢
= k ¢[ N 2 O5 ]; Hence, = [ N 2 O5 ]
dt 4 dt 4
k k¢
Equation (1) requires that = i.e. 2k =k ¢
2 4
Ê k2 ˆ Ea Ê 1 1 ˆ
27. We have ln Á ˜ = - Á - ˜
Ë k1 ¯ R Ë T2 T1 ¯
RT2 T1{2.303 log(k2 / k1 )} (8.314 J K -1 mol-1 )(423 K)(373 K)(2.303) log (1.3 ¥ 10-3 / 1.3 ¥ 10-4 )
Hence, Ea = =
(T2 - T1 ) (423 K - 373 K)
Chemical Kinetics 9.33

8.314 ¥ 423 ¥ 373 ¥ 2.303


= J mol-1 = 60420 J mol-1 = 60.42 kJ mol–1
50

the corresponding stoichiometric number appeared in the balanced chemical equation. For a reactant, it carries
negative sign as its concentration decreases with time.
29. We have
2N2O5(g) æÆ 4NO2(g) + O2(g)
p0 – 2p 4p p
Total pressure, ptotal = (p0 – 2p) + 4p + p = p0 + 3p
After 30 min, ptotal = 87.5 mmHg. Hence
p - p0 (87.5 - 50) mmHg
p = total = = 12.5 mmHg.
3 3
Partial pressure of N2O5 at 30 min will be
pN2O5 = p0 – 2p = (50 – 2 ¥ 12.5) mmHg = 25.0 mmHg.
Since the initial pressure of 50 mmHg of N2O5 is reduced to 25 mmHg, the half-life of the reaction will be
30 min.
After 60 min (which is equal to two half-lives), the partial pressure of N2O5 will be
pN2O5 = 12.5 mmHg.
For this value, the value of p will be
p0 - pN 2O5 (50 - 12.5) mmHg
p= = = 18.75 mmHg.
2 2
Finally, the pressure of the gas will be
p = p0 + 3p (50 + 3 ¥ 18.75) mmHg = 106.25 mmHg
30. Only for the choice (c), Ef/Eb = 2/3
31. The rate of reaction will also be doubled.
32. There is low probability of simultaneous collision of all the reacting species. Thus, the reactions with order more
than three are not known.
33. TiCl3 is used in Ziegler-Natta polymerization (A-ii)
V2O5 is used in Contact process (D-iii)
PdCl2 is used in Wacker process (B-i)
CuCl2 is used in Deacon’s process (C-iv)
34. Slow step is the rate-determining step. Hence, the rate constant will be 5.2 ¥ 109 L mol–1 s–1
35. The rate constant remains constant irrespective of the concentrations of A and B.
1
36. The decomposition reaction is H 2O2 Æ H 2O + O2
2

t t
0.5 M ææ
1/ 2
Æ 0.25 M ææ
1/ 2
Æ 0.125 M
Hence, 2t1/2 = 50 min, i.e. t1/2 = 25 min
The rate constant of the reaction is
0.693 0.693
k= = 2.772 ¥ 10–2 min–1
t1/ 2 25 min
1 d[O 2 ]
The rate expression is = k [H2O2]
(1 / 2) dt
When [H2O2] = 0.05 M, the rate of formation of O2 will be
d[O 2 ] 1
= (2.772 ¥ 10–2 min–1) (0.05 mol L–1) = 6.93 ¥ 10–4 mol L–1 min–1
dt 2
10
Surface
Chemistry

The term ‘adsorption’ implies the presence of excess concentration of any particular component (known as adsorbate)
at the surface of liquid or solid phase (known as adsorbent) as compared to that present in the bulk of the material. This
is due to the presence of residual forces at the surface of the body. The process of adsorption is an exothermic process
and is associated with a decrease in entropy of the system, such that |DH | < T DS as DG of the process is negative.
On the basis of the forces of attraction between adsorbent and adsorbate, two types of adsorption, namely, physisorption

The extent of adsorption of gases increases with increase in the pressure of the gases and it decreases with increase in
temperature of the gas. The variation in the mass (x) of gas adsorbed by a given mass (m) of an adsorbent (say, charcoal)
with change in the pressure of the gas at a constant temperature may be expressed by the following equations (known
as adsorption isotherms).
Freundlich equation (x/m) = kp n; (k is constant and n
Langmuir equation (x/m) = k k2 p k p); (k and k2 are constants)

Table 1 Characteristics of Physisorption and Chemisorption

Physisorption Chemisorption
The forces of attractions are of van der Waals The forces of attraction are of a chemical nature
type (weak forces) (strong forces)
2. Predominates at low temperature Predominates at high temperature
3. All gases show this adsorption at low temperatures
4. Heat of adsorption is low, about 40 kJ mol Heat of adsorption is large ( 80 to 420 kJ mol )
5. Reversible in nature Usually irreversible
6. Low activation energy ( 5 kJ) Large activation energy
7. Adsorption is multilayer Adsorption is monolayer
According to Freundich equation, log (x/m) varies linearly with log p. The actual plots show a slight curvature,
especially at low temperatures. Langmuir equation predicts the linear variation between p/(x/m) and p. At low pressure,
the Langmuir equation is reduced to the form x/m = kp and at high pressures, it is reduced to x/m = k; hence at intermidiate
pressures, an equation of the type x/m = kp n (Freundlich equation) may be applicable for the adsorption. The above
expressions are also applicable for the adsorption of acetic acid or oxalic acid from its solution by activated charcoal.
Colloids or sols are the substances whose sizes lie in between the solutes present in a true solution (e.g. salt, sugar,

but not through perchment paper or animal membrane.


10.2 Complete Chemistry—JEE Main

Table 2 Types of colloidal systems

Dispersed Dispersed Colloidal Examples


phase medium system
Solid Solid Solid sol Coloured gems and glasses, some
alloys, minerals
Solid Liquid Sol Starch or proteins in water, paints,
gold sol
Solid Gas Solid aerosol Smoke, dust, storm
Liquid Solid Gel Jellies, Cheese, butter, boot polish
Liquid Liquid Emulsion
medicines
Liquid Gas Liquid aerosol Mist, fog, cloud, insecticide sprays
Gas Solid Solid foam gases Styrene foam, rubber, occluded
Gas Liquid Foam or Froth Whipped cream, lemonade froth,
soap suds

former is less stable and gets coagulated by adding electrolytes, heating or agitation. They carry charges. On the other
hand, lyophilic sols are quite stable and are not easily coagulated. They do not carry charges.

nature of the particle. A multimolecular colloid consists of an aggregate of small particles held together by van der Waals
forces. A macromolecular particle is itself a large molecule (e.g. starch, cellulose and proteins). An associated colloidal
system behaves as normal solution at low concentration but becomes colloidal at higher concentrations. Examples
include soap and synthetic detergents. The long chain RCOO– of these molecules associate at higher concentrations
and form micelles.
A mixture of colloidal particles and true electrolytes may be separated by using dialysis or electrodialysis method.
The mixture is taken in a bag made from parchment paper or a cellophane membrane. This bag is dipped in warm
water. Through the parchment paper or cellophane membrane only true particles are able to pass through. The colloidal
particles remain within the bag.
The colloidal particles in a solution is a two-phase system. These exhibit Brownian movement (zig-zag motion in
all possible directions), Tyndall effect (scattering of light), electrophoresis (preferential movement in the presence of
electrical potential), osmotic pressure, diffusion and sedimentation.
The colloidal particles in solution acquire charges due to preferential adsorption of ions. For example, ferric hydroxide
sol is positively charged due to adsorption of Fe

ions and becomes positively charged in a solution containing silver nitrate due to the
adsorption of Ag ions. The colloidal particles get coagulated with the addition of electrolytes due to the neutralization
of charges. According to Hardy-Schulze rule, larger the charge on the ion, larger its coagulation ability.
Certain lyophilic sols can protect the lyophobic sols from coagulation by electrolytes. Zsigmondy introduced the term

solution.
Soap (or detergent) molecules involve a long chain fatty acid terminating in a carboxylate anion (or sulphonate

acid points outwardly. The hydrocarbon chain is easily miscible with the grease on the clothes and encapsulates it along
with the dirt to form a micelle. These micelles are removed by rinsing with water.
Emulsions are sols of liquid in liquid. Two types of emulsions may be distinguished, namely, oil-in-water and water-
in-oil. To make emulsions stable, emulsifying agent such as soaps and detergents are added.
Any substance which can decrease the surface tension of water to a large extent is known as surfactant. Examples
are soap and detergents. Such substances have larger concentrations at the surface of water as compared to the bulk of
the solution.
Surface Chemistry 10.3

MULTIPLE CHOICE QUESTIONS

Adsorption and Catalysis


Physical adsorption
(a) involves the weak attractive interactions between the adsorbent and adsorbate
(b) involves the chemical interactions between the adsorbent and adsorbate
(c) is irreversible in nature
(d) increases with increase in temperature
2. Chemisorption
(a) involves the weak attractive interactions between the adsorbent and adsorbate
(b) is irreversible in nature
(c) decreases with increase in temperature
(d) involves multilayer adsorption
3. Which of the following is not correct?
(a) The extent of adsorption depends on the nature of the adsorbent and adsorbate
(b) The extent of adsorption depends on the pressure of the gas
(c) The extent of adsorption depends on the temperature
(d) The extent of adsorption has no upper limit
4. Which of the following statements regarding adsorption is not true?
(a) The phenomenon of adsorption implies the presence of excess concentration of adsorbate at the surface of
adsorbent
(b) The phenomenon of adsorption is due to the presence of residual forces at the surface of the body
(c) During adsorption, there occurs a decrease in free energy of the system
(d) During adsorption, there occurs an increase in entropy of the system
5. Which of the following statements regarding adsorption is not correct?
(a) The process of adsorption is an exothermic process
(b) The substances being adsorbed is known as adsorbate
(c) The substance on which adsorption occurs is known as adsorbent
(d) The activation energy in chemisorption is smaller as compared to that of physisorption
6. Which of the following statements is not correct?
(a) Physical adsorption is monolayer
(b) Physical adsorption is reversible in nature
(c) Physical adsorption involves low activation energy
(d) The extent of physical adsorption decreases with increase in temperature
7. Which of the following statements is not correct?
(a) Chemisorption is monolayer
(b) Chemisorption is irreversible in nature
(c) Chemisorption involves larger activation energy
(d) The extent of chemisorption initially decreases with increase in temperature

x
=kp n
m
the value of n is
(a) always greater than one
(b) always smaller than one
(c) always equal to one
(d) greater than one at low temperature and is smaller than one at high temperature
10.4 Complete Chemistry—JEE Main

9. Which of the following graphs would yield a straight line plot?


(a) x/m versus p (b) log x/m versus p (c) log x/m versus log p (d) x/m versus log p

(a) The extent of physical adsorption increases linearly with increase in pressure in the low pressure region
(b) The extent of physical adsorption attains a limiting value at the high pressure region

(d) Physical adsorption involves the reversible process


adsorption
GS
desorption
where G, S and GS represent, respectively, the unabsorbed gaseous molecules, adsorption sites and adsorbed
gaseous molecules.

(a) Catalyst is not involved in the reaction


(b) The concentration of a catalyst remains constant throughout the progress of chemical reaction
(c) The mechanism of catalytic reactions may vary from reaction to reaction
(d) NO acts as a homogeneous catalyst in the oxidation of SO2 into SO3

(b) Catalyst operates by providing alternate path for the reaction that involves a lower energy of activation
(c) Catalyst lowers the energy of activation of the forward reaction without affecting the energy of activation of
the backward reaction
(d) Catalyst does not affect the overall enthalpy change of the reaction

(a) The catalyst changes not only the rate of forward reaction but also that of the backward reaction
(b) The catalyst changes the value of equilibrium constant of the reaction
(c) The mechanism of a catalytic reaction depends on the type of the catalyst, i.e. whether it is homogeneous or
heterogeneous
(d) Enzymes are essentially proteins which are responsible for the catalysing reactions occurring in living matter

(a) Most heterogeneous catalytic reactions involve the solid surface of the catalyst
(b) Heterogeneous catalysts primarily function by lowering the activation energy of the reaction
(c) A solid catalyst present in the powder form is more effective as it has larger surface area
(d) The catalyst may be deactivated by heating it to a high temperature in vacuum

(a) plot of p/(x/m p is linear (b) plot of p/(x/m) versus p is linear


x/m) versus p x/m p is linear
Colloidal Solution
Which of the following statements is not true for a lyophobic sol?

20. Which of the following statements is not true for a lyophilic sol?
Surface Chemistry 10.5

(a) A colloidal solution is a heterogeneous two-phase system


(b) Liquid-liquid colloidal system is known as emulsion
(c) Silver sol in water is an example of lyophilic sol
(d) Metal hydroxides in water are examples of lyophobic sol
22. The diameters of colloidal particles may range from

23. Which of the following colloidal system represents a gel?


(a) Solid in liquid (b) Solid in gas (c) Liquid in solid (d) Liquid in gas
24. Which of the following colloidal system represents a sol?
(a) Solid in liquid (b) Solid in gas (c) Liquid in solid (d) Liquid in gas
25. Which of the following represents a multimolecular colloidal particles?
(a) Sol of sulphur (b) Starch (c) Soaps (d) Proteins
26. Which of the following represents a macromolecular colloidal particles?
(a) Sol of gold (b) Cellulose (c) Soaps (d) Synthetic detergents
27. Which of the following represents an associated colloids?
(a) Sol of gold (b) Starch (c) Proteins (d) Soaps
28. Which of the following does not represent macromolecular colloidal particles?
(a) Nylon (b) Plastics (c) Rubber (d) Soaps
29. Which of the following statements is not correct?
(a) Peptization is the process by which certain substances are converted into the colloidal state when shaken in
water containing a minute amount of an electrolyte
(b) Metal sols of gold, silver, platinum, etc. can be prepared by Bredig’s arc method

(d) Dialysis is a process with the help of which impurities (made up of ions and molecules) present in a sol can
be conveniently removed
30. Which of the following ions is most effective in the coagulation of a ferric hydroxide sol?
(a) Cl– (b) Br– (c) NO–3 (d) SO2–
4

(a) dialysis (b) peptization


(c) mechanical dispersion (d) oxidation
32. Which of the following sols is negatively charged?
(a) Arsenius sulphide (b) Aluminium hydroxide
(c) Ferric hydroxide (d) Silver iodide in AgNO3 solution
33. Which of the following sols is positively charged?
(a) Silver iodide in potassium iodide solution (b) Ferric hydroxide
(c) Gold (d) Silver

(a) Na (b) Mg (c) Ca (d) Al

(a) Na (b) Mg (c) Ca (d) Al

(a) Cl– (b) Br– (c) SO2–


4 (d) [Fe(CN)6]3–
37. The addition of soap in water
(a) increases its surface tension
(b) decreases its surface tension
(c) increases its surface tension at low concentration and decreases at high concentration.
(d) decreases its surface tension at low concentration and increases at high concentration.
10.6 Complete Chemistry—JEE Main

38. The presence of electric charge on colloidal particles can be illustrated by the technique of

40. Which of the following solutions changes the colour from red to blue of a colloidal gold solution?

(a) lyophilic colloid (b) lyophobic colloid


(c) gel (d) emulsion
42. At isoelectric point,
(a) a colloidal particle moves towards cathode during electrophoresis
(b) a colloidal particle moves towards anode during electrophoresis
(c) a colloidal particle does not move either towards cathode or towards anode during electrophoresis
(d) pH of medium becomes 7.
43. The Brownian motion is due to the

(c) convection current (d) impact of solvent molecules on the colloidal particles
44. Tyndall effect is due to

(c) refraction of light by colloidal particles (d) absorption of light by colloidal particles
45. An As2S3 sol carries a negative charge. The maximum precipitating power for this sol is shown by
(a) K2SO4 (b) CaCl2 (c) Na3PO4 (d) AlCl3
46. Milk is
(a) fat dispersed in water (b) water dispersed in fat
(c) fat and water dispersed in an oil (d) a homogeneous solution of fat and water
47. Which of the following statements is correct?

48. Peptization process involves

ANSWERS

25. (a) 26. (b) 27. (d) 28. (d) 29. (c) 30. (d)

43. (d) 44. (a) 45. (d) 46. (a) 47. (b) 48. (c)
Surface Chemistry 10.7

MULTIPLE CHOICE QUESTIONS FROM AIEEE AND JEE MAIN

(a) Both enthalpy and entropy of adsorption are negative


(b) Adsorption on solids is reversible
(c) Adsorption increases with increase in temperature
(d) Adsorption is spontaneous [2003]

respectively. Which of the following statements is not correct?


(a) Mixing of the sols has no effect
(b) Coagulation in both the sols can be brought about by electrophoresis

(d) Sodium sulphate solution causes coagulation in both the sols. [2005]
3. The volume of a colloidal particle, Vc, as compared to the volume of a solute particle in a true solution, Vs, could
be
–3 3 23
(a) Vc/Vs (b) Vc/Vs (c) Vc/Vs Vc/Vs [2005]

(a) the mass of gas striking a given area of surface is independent of the pressure of the gas
(b) the rate of dissociation of adsorbed molecules from the surface does not depend on the surface covered
(c) the adsorption at a single site on the surface may involve multiple molecules at the same time
(d) the mass of gas striking a given area of surface is proportional to the pressure of the gas [2006]

of their protective powers is


(a) B < D < A < C (b) D < A < C < B (c) C < B < D < A (d) A < C < B < D [2008]
6. Which of the following statements is incorrect regarding physisorptions?
(a) Under high pressure, it results into multimolecular layer on adsorbent surface
(b) Enthalpy of adsorption (DadsH) is low and positive

[2009]
7. According to Freundlich adsorption isotherm, which of the following is correct?
(a) (x/m) μ p0
(b) (x/m) μ p
(c) (x/m) μ p n
(d) All the above are correct for different ranges of pressure [2012]
8. The coagulating powers of electrolytes having ions Na , Al and Ba for arsentic sulphide sol increases in the
order:
(a) Al < Ba < Na (b) Na < Ba < Al
(c) Ba < Na < Al (d) Al < Na < Ba [2013]
9. The following statements relate to the adsorption of gases on a solid surface.
incorrect statement among them:
(a) Enthalpy of adsorption is negative
(b) Entropy of adsorption is negative
(c) On adsorption, the residual forces on the surface is increased
(d) On adsorption decrease in surface energy appears as heat [2015, online]
10.8 Complete Chemistry—JEE Main

molar concentration?
(a) CH3 – (CH2) N (CH3) Br– (b) CH3 – (CH2) – OSO–3 Na
(c) CH3 – (CH2)8 – COO– Na (d) CH3 – (CH2) N (CH3) Br– [2015, online]

charcoal is
[2015]

(a) Adsorption is monolayer.


(b) Adsorption increases with increase in temperature.
.
(d) Energy of activation is low. [2016, online]

and stir well.

and stir well. [2016, online]

(a) Gelatin (b) Starch (c) Oleate (d) Gum Arabic


[2016, online]
x/m) versus log p in a Freundlich adsorption isotherm, which of the following statements
is correct? (k and n are constants)
n n appears as the slope
n) appears as the intercept (d) Both k n) appear in the slope term [2016]

ANSWERS

HINTS AND SOLUTIONS


3
3. The probable answer could be Vc/Vs as the size of the colloidal particle will be greater than that of a particle
in a true solution.
4. Langmuir adsorption involves monolayer formation
5. Larger the gold number, lesser the protective power.
6. Adsorption is an exothermic process and thus DadsH has a negative value.
Surface Chemistry 10.9

7. Freundlich adsorption isotherm is generally represented as (x/m) = kp n with n


imited range of pressure. At low pressure, x/m is generally vary linearly with pressure. Hence, (x/m) μ p.
At high pressure, (x/m) attains a constant value. Hence, (x/m) μ p0 x/m) does not
increase as fast as pressure. Hence (x/m) μ p n with n
of pressure.
8. The larger the charge, larger the coagulating power. Thus the order is Na < Ba < Al .
9. On adsorption, the residual forces on the surface is decreased.

character increases.
–4
n = V(N – N2) = (0.05 L)[(0.06–0.042) mol L mol
Mass of acetic acid adsorbed is
–4 –3
m = nM mol) (60 g mol

chloride solution.
A better protective colloid requires minimum gold number. Hence, gelatin is the better protective colloid.
x
= kp n
m
Ê xˆ
Taking logarithm, we get log Á ˜ = log k log p
Ë m¯ n
n.
11
Chemical Families–Periodic
Properties

Dobereiner’s Triads (1829)

Newland Law of Octaves (1864)

Mendeleev Law

the properties of elements are periodic function of their atomic masses

Modern Periodic Table

the properties of elements are periodic


function of their atomic numbers

Periods
11.2 Complete Chemistry—JEE Main

Table 1
Chemical Families–Periodic Properties 11.3

ns np n

Table 2 Number of elements in the different periods of modern periodic table

Period Number* Orbitals being Number of


elements

Groups

Nobel Gases

n n

n
n

Group
n n n n n n n n n n
representative elements

n ns transition elements n

Z
Z
11.4 Complete Chemistry—JEE Main

n n– ns
inner transition elements

Periodicity in Properties

Van der Waals radius


Metallic radius
Covalent bond radius

ns
Chemical Families–Periodic Properties 11.5

| – | Ed (AB) - [ Ed (AA) + Ed (BB) / 2

Ed
H=

IE + EA
=
2

Oxidation States
11.6 Complete Chemistry—JEE Main

MULTIPLE CHOICE QUESTIONS


Chemical Families–Periodic Properties 11.7
11.8 Complete Chemistry—JEE Main

Questions 41 to 44 are based on the following passage.


The long form of the periodic table is outlined below.

General Characteristics
Chemical Families–Periodic Properties 11.9

n n
11.10 Complete Chemistry—JEE Main
Chemical Families–Periodic Properties 11.11

– –

– – – – – – – –

– –
– –

– –
– –

– –
– –
11.12 Complete Chemistry—JEE Main

ANSWERS
Chemical Families–Periodic Properties 11.13

HINTS AND SOLUTIONS


11.14 Complete Chemistry—JEE Main


n
Chemical Families–Periodic Properties 11.15

MULTIPLE CHOICE QUESTIONS FROM AIEEE AND JEE MAIN

[2003]

[2003]

[2003]

H3
[2003]


[2004]

[2004]

– –
DH
– –
DH
11.16 Complete Chemistry—JEE Main



[2004]

[2004]

[2004]

[2004]

[2006]

correct

[2006]

[2007]

[2009]

[2009]

– – –
– –
[2010]

[2011 Cancelled]

[2011]
Chemical Families–Periodic Properties 11.17

– – – –

[2012]

[2013]

[2013]

Æ 19
Æ
Æ
Æ [2014, online]

[2014, online]

- - - -
- - - - - - - -
- - - - - - - -
[2014, online]

-
[2014, online]

[2015, online]

[2015]

[2016]

[2016, online]

ANSWERS
11.18 Complete Chemistry—JEE Main

HINTS AND SOLUTIONS


¢

.
Chemical Families–Periodic Properties 11.19

- - - -

n n
n
12
General Principles and
Processes of Isolation of
Metals

Only a few metals such as gold, silver, copper and platinum occur in nature in the free or native state. Most of the metals
occur in the form of their compounds such as oxides, carbonates, sulphides and silicates. Such naturally occurring
sources are called minerals ore. Thus, an ore is
a mineral but a mineral may not be an ore. For example, iron pyrites, FeS2, is a mineral but not an ore.
In this unit, we shall describe some commercially important ores of Fe, Cu, Al and Zn and the principles involved in
the extraction of the metals from their important ores.

Iron
Iron occurs in the free state as meterorites which also contain 20 to 30% nickel.
In the combined state, iron occurs in the following minerals.
Magnetite, Haematite, Limonite,
Fe3O4 Fe2O3 3Fe2O3 ◊ 3H2O
Spathic iron ore, Iron pyrites, Copper pyrites,
FeCO3 FeS2 CuFeS2
Iron is extracted by the carbon reduction method.
Extraction of Iron Iron is extracted from its principal ore, haematite. After the preliminary washing, concentration
and roasting, the ore is smelted in the presence of coke and limestone in a blast furnace (Fig. 1).
Roasted ore (8 parts) with desulphurized coke (4 parts) and limestone pieces (1 part) is fed into the blast furnace from
the top. P
is a temperature gradient as we move from the bottom (temperature about 2000 K) to the top (temperature about 500 K)
of the blast furnace. The blast furnace may be broadly divided into three main parts as described in the following.
1. Zone of Fusion The lower portion where coke burns and produce carbon dioxide and a lot of heating is known
as zone of fusion:
C + O2 Æ CO2 DH = - 406 kJ mol -1
Here the temperature is about 1775 K. A little above this, where temperature is about 1475 K - 1575K, iron coming
from above melts.
2. Zone of Heat Absorption The middle portion (temperature 1075 K - 1275 K), CO2 rising up is reduced to CO
with the absorption of heat:
CO2 + C Æ 2CO DH = 163 kJ mol -1

combines with silica (present as impurity-gangue) to form calcium silicate (fusible slag):
CaCO3 Æ CaO + CO2
CaO + SiO2 Æ CaSiO3
12.2 Complete Chemistry—JEE Main

Fig. 1 Blast furnace

3. Zone of Reduction The upper portion (675 K-975 K) where iron oxide is reduced to spongy iron by carbon
monoxide rising up the furnace:
Fe2O3 + 3CO Æ 2Fe + 3CO2
The reduction is believed to take place in stages:
3Fe2O3 + CO Æ 2 Fe3O4 + CO2
Fe3O4 + CO Æ 3FeO + CO2
FeO + CO Æ Fe + CO2

molten iron from oxidation. These two can be removed from different holes (Fig. 12.1). Waste gases escaping at the top
consists of about 30% CO, 10% CO2 and the rest nitrogen.
Iron obtained from the blast furnace is known as pig iron.
Pig iron contains about 2-5% carbon as well as other impurities (usually Si, Mn, S and P). Pig iron is converted into

various articles. Wrought iron, which is the purest form of iron, can be obtained by heating cast iron in a reverberatory
furnace lined with iron oxide. Wrought iron contains about 0.2% carbon.
Zinc
The chief ore of zinc are (i) zinc blende (ZnS), (ii) calamine (ZnCO3), zincite (ZnO) and franklinits (ZnO ◊ Fe2O3).
The extraction of ore is carried out by the carbon reduction of ZnO which is obtained by roasting the ore and concentrating
4 which is obtained by roasting
the ore at moderate temperature and then dissolving it in sulphuric acid.
Copper
Copper occurs in the native state as well as in the compound form. The natural ores of copper are
Copper pyrites, CuFeS2 Malachite, Cu(OH)2 ◊ CuCO3 Copper glance, Cu2S
Cuprite or ruby copper, Cu2O Azurite, Cu(OH)2 ◊ 2CuCO3
Copper may be extracted by self-reduction method.
General Principles and Processes of Isolation of Metals 12.3

Extraction of Copper The ores of Copper are: copper pyrites (CuFeS2), malachite (CuCO3 2), cuprite
(Cu2O) and copper glance (Cu2S). Copper is mainly extracted from copper pyrites. After the concentration of its ore

reactions occurring are


(i) 2CuFeS2 + O2 Æ Cu2S + 2FeS + SO2 (major reaction)
(ii) 2Cu2S + 3O2 Æ 2Cu2O + 2SO2
(iii) 2FeS + 3O2 Æ 2FeO + 2SO2 (minor reactions)
The ore is then mixed with a little coke and sand and smelted in a water-jacketed blast furnace. The minor reactions
that occurred during roasting continue here. Ferrous oxide combines with sand to form a fusible slag. Cuprous oxide

for oxygen than copper.


(iv) FeO + SiO2 Æ FeSiO3
(v) Cu2O + FeS Æ Cu2S + FeO
Molten mass collected from the bottom of furnace contains largely cuprous sulphide and a little ferrous sulphide. This
molten mass is known as matte.

A blast of sand and air is blown in the converter through tuyeres which
are situated a little above the bottom. This causes removal of S and As as
oxides and ferrous oxide as slag (reaction iv). At the same time Cu2S is
oxidized mostly into Cu2O (reaction ii) and partly into CuO and CuSO4.
All these react with Cu2S giving copper. The reactions are
(vi) 2Cu2S + 3O2 Æ 2Cu2O + 2SO2≠
2Cu2S + 5O2 Æ 2CuSO4 + 2CuO
2Cu2O + Cu2S Æ 6 Cu + SO2≠
CuSO4 + Cu2S Æ 3Cu + 2SO2≠
Cu2S + 2CuO Æ 4Cu + SO2≠ Fig. 2 Bessemer converter

Aluminium
Aluminium does not occur free in nature. In the combined state, it occurs in the following forms.
Oxides: Corundum, Al2O3 2O3 ◊ H2O and bauxite, Al2O3 ◊ 2H2O.
Fluorides: Cryolite, Na3AlF6
Silicates: Feldspar, KAlSi3O8, mica (KAlSi3O10(OH)2) and kaolinite (Al(OH)4, Si2O5)
Basic Sulphates: Alunite or alumstone, K2SO4 ◊ Al2(SO4)3 ◊ 4Al(OH)3
Basic Phosphates: Turquoise, AlPO4 ◊ Al(OH)3 ◊ H2O.
Aluminates: Aluminates of Mg, Fe and Mn.
Aluminum is the third most abumdant element of earth’s crust.
Extraction of aluminium Aluminium is isolated from the electrolysis of bauxite, Al2O3 ◊ 2H2

upon the impurity present in the ore.


If the bauxite contains iron oxide as the impurity, one can use Baeyer’s or Hall’s process as described below.
Baeyer’s Process Finally ground ore is roasted to convert ferrous oxide to ferric oxide and then digested with
concentrated caustic soda solution at 423 K. Al2O3 dissolves while Fe2O3
and from the solution Al(OH)3 is precipitated by adding a weak acid. The ignition of Al(OH)3 gives Al2O3.
Al2O3 + 2OH - + 3H2O Æ 2Al(OH)–4
aluminate ion dissolves
Al(OH)–4 + H Æ Al(OH)3 + H2O
+

precipitates
heat
2Al(OH)3 ææÆ Al2O3 + 3H2O
12.4 Complete Chemistry—JEE Main

Hall’s process In this process the ore is fused with sodium carbonate when soluble metaaluminate (NaAlO2) is
produced. This is extracted with water leaving behind iron oxide. Carbon dioxide at 323-333 K is passed through water
extract to get Al(OH)3 which on heating gives Al2O3.
fused
Al2O3 + Na2CO3 ææÆ 2NaAlO2 + CO2
extracted with water
2NaAlO2 + 3H2O + CO2 Æ 2Al(OH)3 + Na2CO3
heat
2Al(OH)3 ææÆ Al2O3 + 3H2O.
If the impurity is silica, the Serpek’s process is used to purify bauxite.
Serpek’s Process The powdered ore is mixed with coke and heated to 2075 K in a current of nitrogen. Silica present is
reduced to silicon which volatilizes off and alumina gives aluminium nitride. The hydrolysis of the latter gives Al(OH)3,
heating of which gives Al2O3.
SiO2 + 2C Æ Si ≠ + 2CO2≠
Al2O3 + 3C + N2 Æ 2AlN + 3CO
AlN + 3H2O Æ Al(OH)3 + NH3
2Al(OH)3 ææÆ Al2O3 + 3H2O
heat
After obtaining pure Al2O3, it is dissolved in fused cryolite, Na3AlF6 2, and is electrolysed
in an iron tank lined with blocks of carbon which serve as the cathode. The anode consists of a number of graphite rods
suspended vertically inside the tank (Fig. 3).
Aluminium gets settled at the bottom of the tank and can be removed. The reactions occurring at the electrodes are
Cathode Al3+ + 3e– Æ Al
2 Æ O2 + 4e
2O2– –
Anode
C + O2 Æ CO2
Anode is replaced periodically because of its consumption.

Fig. 3 Electrolytic cell for the production of aluminium

MULTIPLE CHOICE QUESTIONS

Iron
1. In the metallurgy of iron, when limestone is added to the blast furnace, the calcium ions ends up in
(a) slag (b) gangue (c) metallic Ca (d) calcium carbonate
2. Which of the following minerals does not contain iron?
(a) Magnetite (b) Magnesite (c) Haematite (d) Limonite
General Principles and Processes of Isolation of Metals 12.5

3. The principal ore of iron is


(a) haematite (b) iron pyrites (c) copper pyrites (d) spathic iron
4. The principal reducing agent in the metallurgy of iron is
(a) carbon (b) carbon monoxide (c) carbnon dioxide (d) aluminium
5. The principal reaction in the zone of fusion of blast furnace employed in the metallurgy of iron is
(a) C + O2 Æ CO2 (b) 2C + O2 Æ 2CO
(c) CO2 + C Æ CO (d) Fe2O3 + 3CO Æ 2Fe + 3CO2
6. The principal reaction(s) in the zone of heat absorption of blast furnace employed in the metallurgy of iron is/are
(a) C + O2 Æ CO2 (b) 2C + O2 Æ CO
– CO2 SiO2
(c) 2CO + O2 Æ 2CO2 (d) CaCO3 ææÆ CaO ææÆ CaSiO3
7. Iron is mainly extracted by
(a) carbon reduction method
(b) self-reduction method
(c) the method of electrolysis
(d) leaching with aqueous solution of NaCN followed by reduction
8. Which of the following sequences of carbon content is correct?
(a) steel < cast iron < wrought iron (b) steel < pig iron < wrought iron
(c) steel < wrought iron < cast iron (d) wrought iron < steel < cast iron
9. In the metallurgy of iron, the material obtained from the bottom of blast furnace is
(a) slag (b) pig iron (c) cast iron (d) wrought iron
10. The chemical processes in the production of steel from haematite ore involve
(a) reduction (b) oxidation
(c) reduction followed by oxidation (d) oxidation followed by reduction
Aluminium
11. In the electrolysis of alumina, cryolite is added to
(a) lower the melting point of alumina (b) decrease the electrical conductivity
(c) minimise the anode effect (d) remove impurities from alumina
2) which is added in small quantities in the electrolytic reduction of alumina
dissolved in fused cryolite (Na3AlF6) is
(a) that of a catalyst
(b) to make the fused mixture very conducting
(c) to lower the melting temperature of the mixture
(d) to decrease the rate of oxidation of carbon at the anode.
13. In the electrolytic extraction of aluminium, the solvent is
(a) molten Al2O3 (b) water (c) Fe2O3 + Al (d) molten cryolite
14. Mineral that does not contain Al is

15. The main ore of aluminium is


(a) bauxite (b) alumina (c) potash alum (d) cryolite
16. Which of the following statements regarding the metallurgy of aluminium by electrolytic method is not correct?
(a) Electrolyte is Al2O3 dissolved in Na3AlF6 containing a little of CaF2
(b) Anode consists of a number of graphite rods which are periodically replaced

17. In the commercial electrochemical process for aluminium extraction the electrolyte used is
(a) Al(OH)3 in NaOH solution (b) an aqueous solution of Al2(SO4)3
(c) a molten mixture of AlO(OH) and Al(OH)3 (d) a molten mixture of Al2O3 and Na3AlF6
12.6 Complete Chemistry—JEE Main

18. Electrolytic reduction of alumina to aluminium by Hall-Heroult process is carried out


(a) in the presence of NaCl

(c) in the presence of cryolite which forms a melt with lower melting temperature
(d) in the presence of cryolite which forms a melt with higher melting point
Copper and Silver
19. Copper is mainly extracted from
(a) cuprite (b) azurite (c) copper pyrites (d) malachite
20. The composition of cuprite is
(a) Cu2S (b) CuFeS2 (c) Cu(OH)2 · CuCO3 (d) Cu2O
21. Copper is mainly extracted by
(a) carbon reduction method
(b) self-reduction method
(c) the method of electrolysis
(d) leaching with aqueous solution of NaCN followed by reduction

(a) silver (b) lead (c) copper (d) iron


23. In the metallurgy of copper, the material obtained from the bottom of blast furnace is
(a) slag (b) copper (c) matte (d) cuprous oxide

(a) Zn, Cu, Ag, Au (b) Zn, Ag, Au (c) Cu, Ag, Au (d) Au
Additional Problems
25. Of the following, the metals that cannot be obtained by electrolysis of the aqueous solution of their salts are
(a) Ag (b) Mg (c) Cu (d) Au
26. The principal reducing agent in the metallurgy of tin is
(a) carbon (b) carbon monoxide (c) carbon dioxide (d) aluminium
27. Calamine is
(a) ZnS (b) ZnCO3 (c) ZnO (d) ZnSO4
28. Zinc is obtained from ZnO by
(a) carbon reduction method (b) reduction by H2
(c) boiling giving Zn and O2 (d) treating of KCN followed by electrolysis
29. The impure zinc contains impurities such as cadmium, arsenic, iron and lead. From this, zinc may be recovered
by
(a) distillation at 500 K
(b) distillation at 1170 K – 1270 K followed by passing a column at 1070 K
(c) distillation at 3100 K
(d) solvent extraction method

ANSWERS
1. (a) 2. (b) 3. (a) 4. (b) 5. (a) 6. (d)
7. (a) 8. (d) 9. (b) 10. (d) 11. (a) 12. (c)
13. (d) 14. (b) 15. (a) 16. (c) 17. (d) 18. (c)
19. (c) 20. (d) 21. (b) 22. (a) 23. (c) 24. (d)
25. (b) 26. (a) 27. (b) 28. (a) 29. (b)
General Principles and Processes of Isolation of Metals 12.7

MULTIPLE CHOICE QUESTIONS FROM AIEEE AND JEE MAIN

1. The substance not likely to contain CaCO3 is


(a) dolomite (b) a marble statue
(c) calcined gypsum (d) sea shells [2003]

(a) magnetite (b) cassiterite (c) galena (d) malachite [2004]


3. Aluminium oxide may be electrolysed at 1000 °C to furnish aluminium metal (atomic mass : 27 amu).
The cathodic reaction is Al3+ + 3e– –1
, the electricity required to prepare
5.12 kg of aluminium is
(a) 5.49 × 104 C (b) 5.49 × 101 C (c) 5.49 × 107 C (d) 1.83 × 107 C [2005]
4. Which of the following factors is of no for roasting sulphide ores to the oxides and not subjecting
the sulphide ores to carbon reduction directly?
(a) CO2 is more volatile than CS2
(b) Metal sulphides are thermodynamically more stable than CS2.
(c) CO2 is thermodynamically more stable than CS2
(d) Metal sulphides are less stable than the corresponding oxides. [2008]

Ti(s) + 2I2(g) æ523 K 1700 K


ææÆ TiI4 (g) æææÆ Ti(s) + 2I2(g)
[2012]
6. The metal that cannot be obtained by electrolysis of an aqueous solution of its salts is
(a) Cr (b) Ag (c) Ca (d) Cu [2014]
7. The form of iron obtained from blast furnace is:
(a) Steel (b) Cast Iron
(c) Pig Iron (d) Wrought Iron [2014, online]
8. In the isolation of metals, calcination process usually results in:
(a) metal carbonate (b) metal oxide
(c) metal sulphide (d) metal hydroxide [2015, online]
9. Calamine is an ore of:
(a) Aluminium (b) Copper (c) Iron (d) Zinc [2015, online]
10. In the context of the Hall-Heroult process for the extraction of Al, which of the following statements is false?
(a) CO and CO2 are produced in this process
(b) Al2O3 is mixed with CaF2 which lowers the melting point of the mixture and brings conductivity
(c) Al3+ is reduced at the cathode to from Al
(d) Na3AlF6 serves as the electrolyte [2015]
11. Extraction of copper by smelting uses silica as an additive to remove
(a) Cu2O (b) FeS (c) FeO (d) Cu2S [2016, online]

(a) Siderite (b) Galena (c) Malachite (d) Magnetite [2016]

ANSWERS
1. (c) 2. (c) 3. (c) 4. (d) 5. (d) 6. (c)
7. (c) 8. (a) 9. (d) 10. (d) 11. (c) 12. (b)
12.8 Complete Chemistry—JEE Main

HINTS AND SOLUTIONS

3. Using the expression m = (Q/F) (M/|ne|), we get


. × 103 g) ( 96500 C mol −1 ) ( 3)
m F |ν e | (512 5.12 × 9.65
Q= = = ¥ 107 C= 5.49 × 107 C
M ( 27 g mol −1 ) 9

reverberatory furnace and blowing a current of air over the molten surface. The impurity is oxidized which is
swept away by the air current.

Impurities are oxidized and escape as gases. To reduce any formed oxide of the metal, a pole of green wood is
added which liberates hydrocarbon gases at high temperature (mainly CH4) helping the reduction of oxide.

metal forms volatile metallic iodide and impurities do not react with iodine. The metallic iodide is heated to a high
choice d).
6. The alkaline-earth metal Ca cannot be obtained by the electrolysis of an aqueous solution of its salt. In the
electrolysis, water is electrolysed.
7. Iron obtained from the blast furnace is known as pig iron.

9. Calamine is the ore of Zn


10. Na3AlF6 also decreases melting point and brings conductivity. The electrolyte is molten Al2O3. The reactions
occurring at electrodes are :
Cathode Al3+(melt) + 3e– Æ Al(l)
Anode C(s) + O2–(melt) Æ CO(g) + 2e–
C(s) + 2O2–(melt) Æ CO2 (g) + 4e–
11. Silica is added to remove FeO as FeSiO3.
12. Siderite FeCO3 Galena PbS
Malachite Cu(OH)2 ◊ CuCO3 Magnetite Fe3O4
13
Hydrogen

Hydrogen
1
) is the

+
+ –

Similarities to Alkali Metals

Table 1
Element
1
H
2 1

2 2 6 1
Na
2 2 6 2 6 1
K
2 2 6 2 6 10 2 6 1

2 2 6 2 6 10 2 6 10 2 6 1
Cs
+
+

H æÆ H+ + e–
æÆ + + e–

2O, H2
2 2

HCl æÆ H+ + Cl– NaCl æÆ Na+ + Cl–


2H+ + 2e– æÆ H2 +
+ e– æÆ
Similarities to Halogens
13.2

Table 2
Element
H 1
2, 7
2, 8, 7
Br 2, 8, 18, 7

– –

H + e æÆ H
– – –
æÆ –

+
H–, Na+H–, Ca2+ –
)2 +
Cl–, Na+Cl– , Ca2+ –
)2
2
and
H2

,
ZrH

2) n 2) n )n

2500
Ionization energy/kJ mol-1

2000
F
1500 Cl
H
1000

500
Li Na K

0
1 5 10 15 20
Atomic number

Fig. 1

2
2
, SiH
, SiCl and PH are similar to
NCl and PCl
13.3

+ Cl2 , - HCl + Cl , - HCl


æææææ
CH 4 ¨ ææææ æÆ CH3Cl æææææ
2
Æ CH 2 Cl2
Methane + H , - HCl Methyl chloride Methylene chloride
+ Cl2
Ø- HCl
+ Cl , - HCl
CCl4 ¨ææææ
2
æ CHCl3
Carbon tetrachloride Chloroform
Occurrence

Isotopes of Hydrogen

Table 3 H 2, D 2 2

H2 D2 2

T/K
T/K
D –1

D –1

e –1

Nuclear Isomers

Ortho

ortho ortho

MULTIPLE CHOICE QUESTIONS


13.4

2 + H2 OH

2 and H2 2 2

2O > H 2S > H 2 2O > H2S < H2Se


2O < H 2S > H 2 2O < H2S < H2Se

K°w

2 2O
2 2 2O

2 2

2
2– – –
2 2
13.5

2O with

2O 2

+ 6H+ + H2O2 2+
+ 8H2 2 + 2OH– + H2O2 2+
+ 2H2O + O2

+ 2H+ + H2O2 2 + 2H2 + H 2O 2 + H 2O + O 2
O
2 2
6+ 2H+ + H2O2 6 + 2H2 + H 2O 2 2O2 + H2O
2+
+ H 2O 2 + 2OH– O
2 2 2O

—O—

ortho
ortho-
ortho
ortho
2O 2

2O 2 2O 2

2O 2 2
2O 2 2O 2 2O 2 2O 2

2O 2 2 + 2H2 2O 2 +O 2O2 + H2O

2O 2 +O 2H2O + 2O2 2
13.6

2
2 2O

ANSWERS

HINTS AND SOLUTIONS

2 2 2S is
smaller than H2

K°w = – log K°w K°w K°w

2O is 2H2O Æ + –
+ O2
2O 2
2O 2
2O 2

2O 2 2 2
2H2O2 Æ H 2O + O 2
2 2O 2
H 2O 2 –1
¥ –1
–1 –1
2 )
–1

2O 2
2O 2 2 2O 2 ¥ 2
2O 2

2
2O 2

H2O2 Æ H2O + O and O + O Æ 2O2

O Æ O2 Æ
13.7

MULTIPLE CHOICE QUESTIONS FROM AIEEE AND JEE MAIN

2 2
2
2Cl2
2 [2008]

[2012]
2O 2
2O2 + 2H + 2e Æ 2H2 – 2e– Æ O2 + 2H+
+ –
2O 2
2O2 + 2e Æ 2OH 2O2 + 2OH – 2e Æ O2 + 2H2O
– – – –

[2014]

2O 2

MnO -4 Cr2 O72- SO32- [2014 online]

[2015 online]
wrong

[2015 online]

ANSWERS

HINTS AND SOLUTIONS

2O 2

With MnO -4 2O 2

2MnO 4- + 6H + + 5H 2 O 2 Æ 2Mn 2+ +8H 2 O + 5O2

H 2 O 2 oxidizes SO2- 2- -
3 to SO 4 , I to I 2 and Cr2 O72- to CrO3

SO32- + H 2 O 2 Æ SO 24- + H 2 O

2 I - + 2H + + H 2 O 2 Æ 2 H 2 O + I 2

Cr2 O72- + 2H + 4H 2 O 2 Æ 2CrO5 + 5H 2 O


(perchromate)
14
s-Block Elements (Alkali and
Alkaline Earth Metals)

The Group 1 Elements – Alkali Metals


Group 1 of the periodic table is composed of the alkali metals lithium (Li), sodium (Na), potassium (K), rubidium (Rb),
caesium (Cs) and francium (Fr). Their physical properties are recorded in Table 1.
Table 1 General Physical Properties of Alkali Metals
Property Li Na K Rb Cs Fr
1 1 1 1 1
[He](2s) [Ne](3s) [Ar](4s) [Kr](5s) [Xe](6s) [Rn](7s)1
Atomic number 3 11 19 37 55 87
Relative atomic mass 6.941 22.990 39.098 85.468 132.905 223
Abundance in earth’s crust (ppm) 18 22700 18400 78 2.6 —
Atomic radius, r/pm 152 186 227 248 265 —
ionic radius, r/pm 76 102 138 152 167 —
T/K 453.5 370.8 336.2 312.0 301.5 —
T/K 1620.0 1154.4 1038.5 961.0 978.0 —
Density at 293 K, r cm –3
0.54 0.97 0.86 1.53 1.90 —
I/kJ mol–1 520 496 419 403 376 375
1.0 0.9 0.8 0.8 0.7
Standard electrode potential, E°/V at 298 K for
M+(aq) + e– Æ M(s) –3.03 –2.71 –2.93 –2.93 –2.92 —

Description of Physical Properties

number. The descriptions of these properties are as follows.


1. All are metals with one electron in their outermost orbitals and thus they form unipositive M+ ions(where M
is an alkali metal).
2. At normal temperature, all the metals adopt a body-centred cubic type of lattice with a coordination number of
8.
3. The elements are soft and low melting This is due to the contribution of only one electron per atom towards

4. Their atomic and ionic radii

5. Their melting and boiling points


14.2 Complete Chemistry—JEE Main

6. Their densities

than water.
Their ionization energies

Fig. 1 Metallic and Ionic Radii of Alkali Metals Fig. 2

550 Ionization Energies 550 Ionization Energies

500 500
IE/kJ mo–1

IE/kJ mo–1

450 450

450
450

350
Li Na K Rb Cs 350
Li Na K Rb Cs

Fig. 3 Densities of Alkali Metals Fig. 4


reducing agents.
9. The elements impart characteristic colours

10.
are called photoelectrons.
11. The electronegativity values

ionic bonds.
s-Block Elements (Alkali and Alkaline Earth Metals) 14.3

Table 2 Flame Colours of Alkali Metals


Element Colour Wavelength l/nm
Li crimson 670.8
Na yellow 589.2
K lilac 766.5
Rb red-violet 780.0
Cs blue 455.5

12. The standard electrode potentials, E°(M+

Li+ > Na+ > K+ > Rb+ > Cs+.


13. Alkali metals dissolve in liquid ammonia


(2e–   e2–
2 ).
The deep blue solution is moderately stable at temperatures where ammonia is still liquid. In the presence of

Na + NH3(1) Æ NaNH2 + 1 H2
2

14. The conducting ability of alkali ions follows the order


Li + < Na+ < K+ < Rb+ < Cs+

Isolation of Sodium and Potassium


Sodium occurs in rock salt (NaCl), chile salpeter (NaNO3 ◊
2 ◊ 6H2O), kainite (KSO4 ◊ 4 ◊ 2 ◊ 6H2O), salt peter (KNO3) and potassium feldspar (K2O ◊ Al2O3 ◊
6SiO2). Seawater is an important source of sodium chloride and potassium chloride.
Sodium and potassium are obtained by the electrolytic reduction method. Sodium is produced by the electrolysis of
2

+
(1) Na+

Chemical Properties of Alkali Metals

them from air.


2. Burning in Air or Oxygen
2Æ 2Li2O
2 Æ Na2O2
2 Æ MO2
(K,Rb,Cs)
14.4 Complete Chemistry—JEE Main

Explanation The stability of an ionic compound depends on the relative sizes of its ions. An ionic compound

3. Reaction with Water


2M + 2H2O Æ 2MOH + H2
(Li, Na, K, Rb, Cs)

4. Reaction with Nitrogen


6Li + N2 Æ 2Li3N

2 Li3N ⎯heat
⎯→ 6Li + N2
Li3N + 3H2O Æ 3LiOH + NH3

Compounds of Alkali Metals

1. Oxides

2–

Na2O2 + 2H2O Æ 2NaOH + H2O2



2]
2O2 and O2 are released.
1
KO2 + 2H2O Æ KOH + H2O2 + O2
2
2. Hydroxides

increases from LiOH to CsOH.


Explanation

is due to the reaction



C2H5OH + OH–   OC2H–5 + H2O

caustic soda.
3. Reaction with Halogens

2M + X2 Æ 2MX
s-Block Elements (Alkali and Alkaline Earth Metals) 14.5

MF > MCl > MBr > MI

lithium chloride is due to its covalent character.

Melting points

LICl NaCl KCl RbCl CsCl


Sodium halides Alkali chloride

Fig. 5 Fig. 6

The covalent character of LiX (where X = CI, Br, I) increases as the size of halide increases. This is due to the fact
that the smaller sized Li+
increases as LiCl < LiBr < LI.

Df H
For Chlorides, bromides and iodides, Df H
For the same alkali metal, Df H

low solubility due to smaller enthalpy of hydration Cs+ ion.


4. Carbonates and Bicarbonates

4HCO3. The solubilities of both

Manufacturing of Sodium Carbonate


This compound is manufactured by Solvay process
from limestone (CaCO3

CaCO3 Æ CaO + CO2 (i)


NH3 + H2O Æ NH+4 + OH– (ii)
NaCl + NH+4 + OH + CO2 Æ NH4Cl + NaHCO3

(iii)
14.6 Complete Chemistry—JEE Main

to obtain sodium carbonate


2NaHCO3 æheat
ææ Æ Na2CO3 + H2O + CO2 (iv)

from reaction (i) with ammonium chloride obtained in reaction (iii).


CaO + 2NH4Cl Æ 2NH3 + H2O + CaCl2
The by-product in the above process is calcium chloride.
Sodium carbonate crystallises as Na2CO3 ◊ 10H2O. It is
commonly known as washing soda

Na2CO3 ◊ 10H2O æ375 K


ææÆ Na2CO3 ◊ H2O + 9H2O
Na2CO3 ◊ H2O æ>373
æææ
K
Æ Na2CO3 + H2O
(white)
Na2CO3 is also known as soda ash.
Fig. 7
5. Nitrates Carbonates at 298 K

500 °C

2NaNO3  
 2NaNO2 + O2
800 °C
4NaNO3  
 2Na2O + 5O2 + 2N2
Nitrites produce brown fumes of NO2 when treated with dilute acids.
2NaNO2 + 2HCl Æ 2NaCl + H2O + NO2 + NO
2NO + O2 Æ 2NO2
2CO3 solution.
Na2CO3 + NO2 + NO Æ 2NaNO2 + CO2

KNO3 + Zn Æ KNO2 + ZnO


2KOH + 4NO Æ 2KNO2 + N2O + H2O
4KOH + 6NO Æ 4KNO2 + N2 + 2H2O.

6. Hydrides

3
4LiH + AlCl3 Æ Li[AlH4] + 3LiCl
7. Reaction with Compounds Containing Acidic Hydrogen

2M + C2H5OH Æ 2C2H5OM + H2
2M + 2HC ∫∫ CH Æ 2MC ∫∫ CH + H2.

Anomalous Behaviour of Litliium and Its Diagonal Relationship with Magnesium

diagonal relationship).
s-Block Elements (Alkali and Alkaline Earth Metals) 14.7

4Li + O2 Æ 2Li2O

2 Æ

6Li + N2 Æ 2Li3N

2 Æ 3N 2

Li2CO3 Æ Li2O + CO2 2LiOH Æ Li2O + H2O

3 Æ 2 2 Æ 2O

4LiNO3 Æ 2Li2O + 4NO2 + O2

3) 2 Æ 2 + O2

2NaNO3 Æ 2NaNO2 + O2
2 are soluble in alcohol. The other alkali metal chlorides remain insoluble.

MULTIPLE CHOICE QUESTIONS

1. The number of alkali metals known so far is


(a) 4 (b) 5 (c) 6 (d) 7

(b) Francium is a radioactive element

(d) Sodium is used in the photoelectric cells

(a) Alkali metals tarnish in air


(b) They are kept under kerosene

(a) LiCl < NaCl < KCl < RbCl (b) LiCl > NaCl > KCl > RbCl
(c) LiCl < NaCl > KCl > RbCl (d) LiCl > NaCl < KCl < RbCl

(b) Alkali metals form covalent hydrides with H2 when heated


14.8 Complete Chemistry—JEE Main

(c) Alkali metals in liquid ammonia imparts deep blue solution

2 are soluble in alcohol whereas chlorides of other alkali metals are insoluble

(a) RbO (b) Rb2O (c) Rb2O2 (d) RbO2

(a) Na+ ions (b) conduction electrons


(c) free protons (d) a body-centred cubic lattice
13. Ionic radii of alkali metal ions in water are in the order
(a) Li+ > Na+ > K+ > Rb+ (b) Li+ < Na+ < K+ < Rb+
(c) Li+ > Na+ > K+ < Rb+ (d) Li+ > Na+ < K+ < Rb+

(a) Li+ (b) Na+ (c) K+ (d) Rb+

(a) Li (b) Na (c) K (d) Rb

(a) Li (b) Na (c) K (d) Rb

(a) Li (b) Na (c) K (d) Rb


18. The number of water molecules of crystallization in sodium carbonate is
(a) 5 (b) 10 (c) 7 (d) 2
19. Chile salt petre is
(a) KNO3 (b) NaNO3 (c) NH4NO3 (d) LiNO3

(a) LiCN (b) NaCN (c) KCN (d) RbCN

(a) LiNO3 (b) NaNO3 (c) KNO3 (d) RbNO3

(c) Sodium thiosulphate (d) Sodium chloride


23. In the titration of sodium thiosulphate and iodine, the equivalent mass of the former is equal to
(a) molar mass (b) molar mass/2 (c) molar mass/3 (d) molar mass/4

(b) Francium is an radioactive element


(c) Atomic number of francium is 86
s-Block Elements (Alkali and Alkaline Earth Metals) 14.9

(c) The abundance of potassium in the earth’s crust is more than that of sodium

(a) Alkali metals cannot be obtained by the chemical reduction of their compounds
(b) Alkali metals are usually obtained by the electrolytic reduction

(a) Sodium and potassium are soft and silvery-white metals

(d) Sodium and potassium are kept under kerosene to avoid the contact with air and moisture

O2
(a) 4Na + O 2 2Na2O ædry
æÆ 2Na2O2
(dry)
O2
(b) 4K + O2 2K2O ædry
æÆ 2K2O2
(c) 2M + 2H2O 2MOH + H2; where M is Na or K
(d) 2M + 2NH3 2MNH2 + H2; where M is Na or K

2 · 6H2O (b) K2SO4 4 2 · 6H2O


(c) KNO3 (d) K2O · Al2O3 · 6SiO2

2 · 6H2O (b) K2SO4 4 · 6H2O


(c) K2SO4 2 · 6H2O (d) K2SO4 4 2 · 6H2O
32. Solvay process is used for the manufacture of
(a) Na2CO3 · 10H2O (b) K2CO3 (c) NaOH (d) Na2O2

(a) Na2CO3 · 10H2O (b) NaHCO3 (c) K2CO3 (d) NaOH


34. Castner-Kellner cell is used for the production of
(a) NaOH (b) Na2CO3 (c) NaHCO3 (d) NaCl

(a) The main raw material required in Solvay process are sodium chloride and ammonium chloride
(b) NaHCO3 is less soluble in water as compared to NH4Cl
(c) The byproduct in Solvay process is calcium chloride
(d) Na2CO3 · 10H2

(c) Sodium bicarbonate is used to cure acidity in the stomach

(a) Li (b) Na (c) Rb (d) Cs

(a) Na2CO3 and NaHCO3 (b) Na2CO3 and NaOH


(c) Na2CO3and NaOH (c) NaHCO3 and NaCI
14.10 Complete Chemistry—JEE Main

39. Sodium is manufactued


(a) in Nelson cell (b) by Downs process
(c) by Solvay process (d) by Hall’s process
40. The Glauber’s salt is
4 . 7H2O (b) ZnSO4 . 7H2O (c) FeSO4 . 7Hs2O (c) NaSO4 . 10H2O

(a) CaCO3 æheat


ææ Æ CaO + CO2

(b) NaCl + NH3 + H2O + CO2 NH4Cl + NaHCO3


(c) CaO + 2NH4Cl 2HN3 + H2O + CaCl2

(d) Na2CO3 + CO2 + H2O æheat


ææ Æ 2NaHCO3

(a) Li2CO3 (b) Na2CO3 (c) K2CO3 (d) CsCO3

(a) Li2CO3 (b) Na2CO3 (c) K2CO3 (d) CsCO3

(a) K2O (b) KO2 (c) K2O2 (d) KO


46. The compound KO2 is a

47. The useful product and by-product obtained in the Solvay's process are
(a) quick lime and CO2 (b) NaHCO3 and NH4Cl
(c) Na2CO3 and CaCl2 (d) NaHCO3 and CO2
48. Brine is
(a) concentrated solution of NaCl (b) concentrated solution of KCl
(c) concentrated solution of Na2CO3 (d) concentrated solution of NaHCO3
49. Metallic character of alkali metals
(a) increases with increase in atomic number
(b) decreases with increase in atomic number
(c) increases followed by a decrease with increase in atomic number
(d) decreases followed by an increase with increase in atomic number

(a) 1 (b) 2 (c) 13 (d) 16

(a) Li2O2 (b) Na2O2 (c) K2O2 (d) Rb2O2


52. In the reaction Na2S + O2 + H2O Æ A + NaOH the product A is
(a) Na2SO3 (b) Na2S2O3 (c) Na2S4O6 (d) Na2SO4
53. The correct order of density of alkali metal follows the order
(a) Li < Na < K (b) Li < K < Na (c) Na < Li < K (d) Na < K < Li
54. The molar ionic conductivity of alkali metal ions follows the order
(a) Li+ > Na+ > K+ (b) Li+ > K+ > Na+ (c) K+ > Na+ > Li+ (d) K+ > Li+ > Na+
s-Block Elements (Alkali and Alkaline Earth Metals) 14.11

ANSWERS
1. (c) 2. (d) 3. (c) 4. (b) 5. (c) 6. (c)
7. (b) 8. (a) 9. (d) 10. (b) 11. (d) 12. (b)
13. (a) 14. (a) 15. (a) 16. (a) 17. (d) 18. (b)
19. (b) 20. (c) 21. (c) 22. (c) 23. (a) 24. (c)
25. (c) 26. (a) 27. (d) 28. (c) 29. (b) 30. (a)
31. (d) 32. (a) 33. (b) 34. (a) 35. (a) 36. (a)
37. (a) 38. (c) 39. (b) 40. (d) 41. (d) 42. (d)
43. (a) 44. (a) 45. (b) 46. (b) 47. (c) 48. (a)
49. (a) 50. (a) 51. (a) 52. (b) 53. (b) 54. (c)

HINTS AND SOLUTIONS

2).
7. Alkali metals form ionic hydrides.

+
> Na+ > K+ > Rb+

14. Li+
23. The reaction is 2S2O32– + I2 S4O62– + 2I–. Equivalent mass = Molar mass
24. Atomic number of francium is 87.
25. The abundance of sodium is more than that of potassium.
26. Chile saltpeter is NaNO3.
27. Downs cell is used for the production of sodium.
2 2O 2) whereas potassium produces
2).
29. K + O2 KO2.
35. The main raw materials needed are limestone (CaCO3) and sodium chloride.
36. NaHCO3 is less soluble as compared to Na2CO3.
41. The reverse reaction is actually involved.

43. Li2CO3

44. Same as Q. 43.


45. Li forms Li2O, Na forms Na2O2 2).

2
47. The useful product is Na2CO3 and by-product is CaCl2
48. Brine is concentrated solution of NaCl.

50. The atomic numbers of alkali metal are 3,11,19,37,55,87,


14.12 Complete Chemistry—JEE Main

51. Li2O2
52. The reaction is 2Na2S + 2O2 + H2O Æ Na2S2O3 +2NaOH
–3 –3 –3
).
54. Because of hydration of ions in solution, the molar conductivity follows the order K+ > Na+ > Li+

MULTIPLE CHOICE QUESTIONS FROM AIEEE AND JEE MAIN

2O 2
2O 2.
3+
(c) Na2O2 to CrO 2-
4 [2014, online]

(a) LiO2, Na2O2 and K2O (b) Li2O2, Na2O2 and KO2
(c) Li2O, Na2O2 and KO2 (d) Li2O, Na2O and KO2 [2016]

ANSWERS
1. (d) 2. (c)

HINTS AND SOLUTIONS

O -2 . Na2O2 3+
in acidic medium to CrO 2-
4 and is a
derivative of H2O2.
Æ 2Li2O
2
2 Æ Na2O2

2 Æ KO2

The Group 2 Element – Alkaline Earth Metals

strontium (Sr), barium (Ba) and radium (Ra). Their physical properties are recorded in Table 3.
Table 3 General Physical Properties of Alkaline Earth Metals
Property Be Ca Sr Ba Ra
2 2 2 2 2
[He](2s) [Ne](3s) [Ar](4s) [Kr](5s) [Xe](6s) [Rn](7s)2
Atomic number 4 12 20 38 56 88
Relative atomic mass 9.012 24.305 40.08 87.67 137.33 226.025
Abundance in earth’s crust (ppm) 2 27640 46668 384 390 10–6
Atomic radius, r/pm 112 160 197 215 222 —
Ionic radius for 6-coordinate, r/pm 27* 72 100 118 135 148
T/K 1580 922 1112 1041 1000 (973)
T/K –2773 1378 1767 1654 (2123) (1973)
Density at 293 K, r/g cm–3 1.85 1.74 1.55 2.63 3.62 5.5
I/kJ mol–1
I 899 738 590 549 503 509
II 1757 1450 1145 1064 965 975
Standard electrode potential
E°/V at 298 K for
M2+(aq) +2e– Æ 2M(s) –1.70 –2.37 –2.87 –2.89 –2.90 –2.92
*
For 4-coordinate.
s-Block Elements (Alkali and Alkaline Earth Metals) 14.13

Description of Physical Properties


1. All are metals with two electrons in their outermost orbitals and thus they form bivalent M2+ ions.
2. The elements of Group 2 are soft

to one electron contributed by the elements of Group 1.


3. Their melting and boiling points

4. The atomic and ionic radii

5. Because of the smaller atomic radii, the elements are more dense

second
ionization energies

Fig. 8 Fig. 9 Atomic and Ionic Radii of Alkaline


I/kJ mol–1

Mg

Fig. 10 Densities of Alkaline Earth Metals Fig. 11


14.14 Complete Chemistry—JEE Main

7. Calcium, strontium and barium impart characteristic colours

Table 4 Flame Colours of Ca, Sr and Ba


Element Colour
Ca Brick red
Sr Crimson red
Ba Green
8. The standard electrode potential, E°(M2+

9. Alkaline-earth metals dissolve in liquid ammonia

which slowly decompose to amides.


M + 6NH3 ⎯evaporation
⎯⎯⎯→ M(NH3)6 —Æ M(NH2)2 + 4NH3 + H2
Concentrated solutions of metals in ammonia are bronze coloured, due to the formation of metal clusters.
10. The crystalline compounds of Group 2 contain more water of crystallization
2 ◊ 6H2O, CaCl2 ◊ 6H2O and BaCl2 ◊ 2H2O all
have water of crystallization. The number of molecules of water of crystallization decreases as the ions become

Chemical Properties

3. Burning in Air or Oxygen

2M + O2 Æ 2 MO M + O2 Æ MO2

Æ 3N 2
4. Reaction with Water

M + 2H2O Æ M(OH)2 + H2.


5. Reaction with Nitrogen
3M + N2 Æ M3N2.
The formation of nitride ion, N3–, from N2 2 is a very stable molecule.
3N 2
M2+ and N3–.
+
ion.

M3N2 + 6H2O Æ 3M(OH)2 + 2NH3.

Isolation of Magnesium and Calcium

3 ◊ CaCO3 3 4 ◊
7H2O), carnallite (KCI ◊ 2 ◊ 6H2 2 SiO4 2Si4O10(OH)2 Si O
3 2 5 (OH) 4]
s-Block Elements (Alkali and Alkaline Earth Metals) 14.15

Calcium is present in limestone, marble or chalk (CaCO3 4 ◊ 2H2 2),


3 ◊ CaCO 3 5 )
4 3 F]. Sea shells, corals and pearls are essentially CaCO 3 .

Their fused anhydrous chlorides are electrolysed to obtain them in the metallic form.

Compounds of Alkaline Earth Metals

1. Oxides
All the elements in Group 2 burn in O2

of the carbonates, MCO3.

electropositive character of the metals.

BaO2 and SrO2


2 and CaO2 2 with H2O2

2. Hydroxides
2
Be(OH)2

2 is a weak base.
Ca(OH)2 and Sr(OH)2
Ba(OH)2
Ca(OH)2 is known as slaked lime. Its solution in water Ca(OH)2 is known as lime water while that of Ba(OH)2 is known
as baryta solution
Excess CO
Ca(OH)2 + CO2 ⎯⎯
→ CaCO3 + H2O ⎯⎯⎯⎯⎯
2
→ Ca(HCO3)2
lime water milkiness disappears
turns milky

Fig. 12
14.16 Complete Chemistry—JEE Main

Explanation
1. Lattice Energy

its tendency to split into ions to pass over to the solution, i.e. less will be its solubility.
2. Hydration Energy

Hence,

2+
to Ba2+ ions. This factor is predominant as compared to the decrease in the hydration

3. Carbonates and Bicarbonates


Alkaline earth metals form solid carbonates. Bicarbonates are known only in solution because of the less basic character
of alkaline earth metals.

follows.
BeCO3 3 CaCO3 SrCO3 BaCO3
< 100 °C 540 °C 900 °C 1290 °C 1360 °C
Explanation As the size of M 2+ 2–
3 decreases

Explanation In case of carbonates, the size of CO32– 2+


to
Ba2+
2+
to Ba2+
4. Sulphates

BeSO4 4 are soluble. CaSO4


SrSO4, BaSO4 and RaSO4 are virtually insoluble.
Explanation In case of sulphates, the size of SO42– 2+
to Ba2+ do
2+
to Ba2+

MSO4 ⎯heat
⎯→ MO + SO3

BeSO4 4 CaSO4 SrSO4


500 °C 895 °C 1149 °C 1374 °C
Explanation As the size of M2+ 2–
4 decreases

5. Nitrates

2O 4

N O
2 4→ Be(NO ) .2N O ⎯warm
⎯⎯⎯⎯ to 50 °C
in vacuum →
BeCl2 ⎯⎯⎯ 3 2 2 4 Be(NO3)2 ⎯125
⎯⎯ °C
→ [Be4O(NO3)6]
s-Block Elements (Alkali and Alkaline Earth Metals) 14.17

Basic beryllium nitrate

Basic beryllium acetate, [Be4O(CH3COO)6], has a similar structure.


6. Halides
2 at the appropriate temperature.

700 °C
BeO + C + Cl2   
 BeCl2 + CO
It cannot be prepared in aqueous solution due to the formation of hydrated ion [Be(H2O)4]2+. The halides obtained has
the molecular formula [Be(H2O)4]Cl2, [Be(H2O)4]F2
[Be(H2O)4]Cl2 ⎯heat
⎯→ Be(OH)2 + 2HCl + 2H2O
Beryllium halides are covalent and fume in air due to hydrolysis:
BeCl2 + 2H2O Æ Be(OH)2 + 2HCl
In vapour phase it is present as BeCl2 and (BeCl2)2 and in solid phase it is polymerized.

Cl
CI—Be—CI Cl—Be — Be–Cl
Cl —
monomer dimer

2 ◊ 6H2 2 ◊ 6H2O. Anhydrous CaCl2


2

7. Complexes

2–
4 , [Be(H2O)4]2+, [Be(C2O4)2]2–, [Be4O(Ac)6], etc.
14.18 Complete Chemistry—JEE Main

hybridization of beryllium orbitals. These are the orbitals (2s and three 2p) which are available in the valence shell of

ethylenediamine tetraacetate (see unit 12).

Anomalous Behaviour of Beryllium

few points of differences are as follows.


1. Beryllium forms bonds with appreciable covalent character.
+
3O . This is due to the formation of a
bond.

4. Be(OH)2

Diagonal Relationship of Beryllium

1. Both beryllium and aluminium become passive by concentrated nitric acid due to the formation of an inert layer

2. Both beryllium and aluminium react with alkali to form beryllate, [Be(OH)4]2– and aluminate, [Al(OH)6]3–.
3. Both Be(OH)2 and Al(OH)3 are amphoteric.
4. Both beryllium and aluminium form polymeric halides.

5. Both the metals form nitrides which liberate ammonia when treated with water.
Be3N2 + 6H2O Æ 3Be(OH)2 + 2NH3
AlN + 3H2O Æ Al(OH)3 + NH3.

Be2C + 4H2O Æ 2Be(OH)2 + CH4


A14C3 + 12H2O Æ 4Al(OH)3 + 3CH4.
2+ 3+ 2–
7. Both Be and Al 4] , [Be(C2O4)2]2–, [AlF6]3– and [Al(C2O4)3]3–.
s-Block Elements (Alkali and Alkaline Earth Metals) 14.19

MULTIPLE CHOICE QUESTIONS

1. The number of alkaline earth metals is


(a) 4 (b) 5 (c) 6 (d) 7

(a) Alkaline earth metals are harder and denser than alkali metals

(c) Alkaline earth metals are more reactive than alkali metals

(b) Beryllium differs considerably from the rest of the alkaline earth metals
(c) Anhydrous salts of beryllium are covalent whereas those of Ca, Ba and Sr are ionic

(a) Be(OH)2 2 (c) Ca(OH)2 (d) Ba(OH)2

(a) BeSO4 4 (c) CaSO4 (d) BaSO4

(a) Beryllium does not react with water

(c) Calcium reacts even with cold water

(c) The sulphates of alkaline earth metal are stable to heat

(c) The halides of Ca, Sr and Ba are essentially ionic

(c) Anhydrous CaCl2

(b) Be and Al react with alkali to form beryllate, Be(OH)42–, and aluminate Al(OH)63–, respectively
14.20 Complete Chemistry—JEE Main

(c) Beryllium and aluminium nitrides liberate ammonia when treated with water
(d) Beryllium and aluminium carbides liberate acetylene when treated with water
13. If one mole of Ba(OH)2 is added to 10 litres of water at 25 °C, the pH of the resultant solution would be
(a) 13.3 (b) 13.0 (c) 0.7 (d) 1.7
14. Soap scum is often composed of
(a) sodium carbonate (b) calcium carbonate (c) sodium sterate (d) calcium sterate

(a) Be(OH)2 2 (c) Ca(OH)2 (d) Ba(OH)2

3 (b) CaF2 (c) CaCl2 2

18. Hardness in water is due to


(a) Ca2+ 2+
(b) HCO3– (c) Na+ (d) K+

(a) Be2+ 2+
(c) Ca2+ (d) Sr2+

(a) Be2+ 2+
(c) Ca2+ (d) Sr2+

from Be2+ to Ba2+


2+
to Ba2+.
(c) The salts of alkaline earth metals are less hydrated than those of alkali metals.
(d) Carbonates of alkaline earth metals are insoluble in water.

2 · 2H2 2 · 4H2 2 · 6H2 2 · 8H2O

2· 6H2 4 · 7H2 3)2 · 6H2 3 4) 2

(a) BaO (b) Ba(OH)2 (c) BaCO3 (d) Ba(NO3)2


26. The number of water of crystallization in barium chloride is
(a) 2 (b) 4 (c) 5 (d) 6

(a) Be2 (b) B2 (c) C2 (d) N2

(a) The alkaline earth metals are harder and denser than alkali metals

(c) Alkaline earth metals are too reactive to occur freely in nature

(a) Gypsum (b) Dolomite (c) Talc (d) Fluorspar


s-Block Elements (Alkali and Alkaline Earth Metals) 14.21

2Æ 2Æ
(c) 2Ca + CO2 æheat
ææ Æ 2CaO + C (d) Cr2O3 + 3Ca Æ 2Cr + 3CaO

(a) Ammonia soda process (b) Downs cell


(c) Dow process (d) Castner-Kellner cell
36. The composition of Sorel’s cement is
2 2 nH2O
3 3 nH2O

2 · 2H2 2 · 4H2 2 · 6H2 2 · 7H2O

3–
4 ions is due to the formation of
4 4 4 4 · 6H2 2 3 4) 2
40. Quick lime is
(a) CaCO3 (b) Ca(OH)2 (c) CaO (d) CaSO4
41. Limestone is
(a) CaCO3 (b) Ca(OH)2 (c) CaO (d) CaSO4
42. Slacked lime is
(a) CaCO3 (b) Ca(OH)2 (c) CaO (d) CaSO4
43. Milk of lime is
(a) solid Ca(OH)2 (b) suspension of Ca(OH)2 in water
(c) suspension of CaCO3 in water (d) suspension of CaO in water

(a) Limestone (b) Marble (c) Chalk (d) Talc

1
(a) CaSO4 · 2
H 2O (b) CaSO4 · H2O (c) CaSO4 · 2H2O (d) CaSO4

1
(a) CaSO4 (b) CaSO4 · 2
H 2O (c) CaSO4 · H2O (d) CaSO4 · 2H2O
47. The composition of dolomite is
3 (b) CaCO3 3 · CaCO3 3 · CaCO3 · 6H2O
14.22 Complete Chemistry—JEE Main

48. The composition of asbestos is


2O5(OH)2 2Si2O5 · 4H2 2O5)2(OH)4 2Si2O5(OH)4

51. Solubility of alkaline earth metal sulphates decreases down the Group 2 because

(c) enthalpy of hydration of bivalent metal ions decrease rapidly


(d) the compounds become more and more ionic

3 4 2
3 4)2 is treated with H2SO4, superphosphate of lime is obtained. The composition of this fertiliser is
(a) Ca3 4)2 + CaSO4 (b) Ca(H2 4)2
(c) Ca(H2 4)2 · H2O + 2CaSO4 · 2H2O (d) Ca(H2 4)2 + CaSO4

(a) CaO (b) CaCO3 (c) CaC2O4 (d) Ca(OH)2

(a) Dolomite (b) Gypsum (c) Epsomite (d) Talc


2+ +
ions because
2+ +
are insoluble.
2+ +
ions
+ 2+
2+ +
ions
57. The covalent halides are formed by

2+
3+
(a) Al ions (b) Na+ ions (c) Be2+ 3+
ions

(a) Ca > Ba > K (b) K > Ca > Ba (c) Ca > K> Ba (d) K > Ba > Ca

(a) 11 (b) 20 (c) 31 (d) 40

(a) Be(OH)2 2 (c) Ca(OH)2 (d) Ba(OH)2


2.6H2O, the residue contains
2

64. Thermal stability of carbonates of alkaline-earth metals follows the order


(a) CaCO3 3 > BaCO3 (b)CaCO3 > BaCO3 3
(c) BaCO3 > CaCO3 3 (d) BaCO 3 3 > CaCO3

(a) Be(OH)2 (b) Ca(OH)2 (c) Ba(OH)2 (d) NaOH


66. Identify the correct order of thermal stability of hydrides of Ca, Sr and Ba.
(a) CaH2 > SrH2 >BaH2 (b) CaH2 > BaH2 > SrH2 (c) BaH2 > CaH2 > SrH2 (d) BaH2 > SrH2 > CaH2
s-Block Elements (Alkali and Alkaline Earth Metals) 14.23

67. The formula of basic beryllium acetate is


(a) Be2O(CH3COO)2 (b) Be4O(CH3COO)4 (c) Be4O(CH3COO)6 (d) Be(CH3COO)2

ANSWERS
1. (c) 2. (c) 3. (a) 4. (d) 5. (d) 6. (a)
7. (b) 8. (d) 9. (b) 10. (b) 11. (a) 12. (d)
13. (a) 14. (d) 15. (a) 16. (c) 17. (d) 18. (a)
19. (a) 20. (d) 21. (d) 22. (c) 23. (c) 24. (b)
25. (b) 26. (a) 27. (a) 28. (d) 29. (b) 30. (c)
31. (d) 32. (c) 33. (c) 34. (a) 35. (c) 36. (b)
37. (c) 38. (c) 39. (b) 40. (c) 41. (a) 42. (b)
43. (b) 44. (d) 45. (a) 46. (d) 47. (c) 48. (d)
49. (d) 50. (a) 51. (c) 52. (d) 53. (c) 54. (c)
55. (a) 56. (b) 57. (a) 58. (b) 59. (b) 60. (b)
61. (d) 62. (a) 63. (d) 64. (c) 65. (a) 66. (a)
67. (c)

HINTS AND SOLUTIONS


2. Alkaline earth metals are less reactive than alkali metals.

10. Beryllium chloride vapour contains BeCl2 but the solid is a polymer with a chain structure.

12. Both beryllium and aluminium carbides form methane on hydrolysis:


Be2C + 4H2O 2Be (OH)2 + CH4
Al4C3 + 12H2O 4Al(OH)3 + 3CH4
13. Ba(OH)2 completely ionizes in water.
Hence, [OH –
15. Because of the small size of Be2+ ions, the Be2+ and OH –
19. Because of the small size of Be2+

27. Be2
2 is zero).

3O 4.
3Si4O10 (OH)2.
14.24 Complete Chemistry—JEE Main

32. 5Ca + 2CO2 æheat


ææ Æ 4CaO + CaC2.

2+

57. Beryllium halide is covalent, rest of alkaline-earth metals form ionic halides.

19K (3s)2 (3p)6 Æ (3s)2 (3p)5


20Ca (3s)2 (3p)6 (4s)1Æ (3s)2 (3p)6
56Ba (5s)2 (5p)6 (6s)1Æ (5s)2 (5p)6

stable (3p)6
60. Atomic numbers of elements of Group 2 are 4,12,20,38.....

2 due to the smaller size of Be. In fact, Be(OH)2

67. Basic beryllium acetate is Be4O(CH3COO)6. its structure is

MULTIPLE CHOICE QUESTIONS FROM AIEEE AND JEE MAIN

(d) inter-ionic attraction [2005]

(a) NaNO3 (b) KClO3 (c) CaCO3 (d) NH4NO3 [2012]

(a) Ba(OH)2 < Sr(OH)2 < Ca(OH)2 2


(b) Ba(OH)2 < Ca(OH)2 < Sr(OH)2 2
2 < Ca(OH)2 < Sr(OH)2 < Ba(OH)2
2 < Sr(OH)2 < Ca(OH)2 < Ba(OH)2 [2015 online]

2 (b) BeCl2 (c) SrCl2 (d) CaCl2 [2015 online]

(a) CaSO4 (b) BeSO4 (c) BaSO4 (d) SrSO4 [2015]


s-Block Elements (Alkali and Alkaline Earth Metals) 14.25

6. The correct order of the solubility of alkaline-earth metal sulphates in water is

[2016 online]

(a) Quick lime (b) Milk of lime (c) Slaked lime (d) Lime stone
[2016 online]

ANSWERS
1. (c) 2. (c) 3. (c) 4. (b) 5. (b) 6. (a)
7. (a)

HINTS AND SOLUTIONS


2. The thermal decomposition of CaCO3
CaCO3 ææ
Æ CaO + CO 2
basic acidic
oxide oxide

4. BeCl2 is essential covalent. This is due to the small size of Be. It can easily polarize Cl–
covalent bond.
2+
5.
2+
to Ba2+
4 to BaSO4.
2. The paste of lime
15
Study of the p-Block Elements
(Groups 13, 14 and 15)

The Group 13 Elements – Boron Family


Group 13 of the periodic table is composed of elements boron (B), aluminium (Al), gallium (Ga), indium (In) and
thallium (Tl). Their physical properties are recorded in Table 1.
Table 1 Atomic and Physical Properties of Group 13 Elements
Property B Al Ga In Tl
2 1 2 1
[He](2s) (2p) [Ne](3s) (3p) [ A r ] [Kr](4d) (5s) (5p) [Xe](4f)14(5d)10(6s)2(6p)1
10 2 1
10 2 1
(3d) (4s) (4p)
Atomic number 5 13 31 49 81
Relative atomic mass 10.81 26.982 69.72 114.82 204.383
Atomic radius, r/pm (80-90) 143 (125) 167 170
Ionic radius, r(M3+)/pm (27) 53.5 62.0 80.0 88.5
r(M+)/pm 120 140 150
Melting point, T/K 2453 933.4 302.8 249.6 576.5
Boiling point, T/K 3923 2740 2676 2353 1730
Density at 293 K, r/g cm–3 2.35 2.70 5.90 7.31 11.85
Ionization energy, l/kJ mol–1
I 800.5 577.4 578.4 558.2 589.1
II 2426.5 1816.1 1978.8 1820.2 1970.5
III 3658.7 2744.1 2962.3 2704.0 2877.4
Electronegativity 2.0 1.5 1.6 1.7 1.8
Enthalpy of fusion/kJ mol–1 23.60 10.50 5.59 3.26 4.31
Enthalpy of vaporization/kJ mol–1 504.5 290.8 270.3 231.8 166.1
Standard electrode
potential, E°/V at 298 K for
M3+(aq) + 3e– Æ M(s) (–0.87)* –1.66 –0.56 –0.34 +1.26
M+(aq) + e– Æ (M(s) + 0.55 (–0.79)† –0.18 –0.34
*
For H3BO3 + 3H+ + 3e– Æ B + 3H2O.

In acidic medium

Description of Physical Properties


ns)2(np)1, where n varies from 2 to 6. They are
expected to form compounds with +3 oxidation state.
15.2 Complete Chemistry—JEE Main

3. The metallic radii of atoms do not increase regularly on descending the group (Fig. 1).
A Few Comments
(i) Boron is not a metal, the reported radius is half the closest approach in its structure.
(ii) Gallium has an unusual structure, the reported radius is half the closet approach.
(iii) Ga, In and Tl follow immediately after the ten transition elements. Thus, the outer shell is preceded by
d10
near the nucleus as compared to their expected normal locations. Thus, their atomic radii are smaller than
the expected values. This contraction in size is sometimes called the d-block contraction.
(iv) The atomic radius of Tl is a little larger than In. This is due to the intervention of 4f electrons, which shield
the nuclear charge more poorly. The small value of atomic radii is a result of lanthanide contraction.
4. The ionic radii of M3+ increase down the group. The reported value of B3+ is an estimated value as B3+ is not
found to exist.
5. On descending the group, +1 oxidation state becomes more stable than +3 state due to the inert pair effect.
Explanation of Inert Pair Effect In inert pair effect, s electrons do not take part in bonding. The reason behind
this is the energy factor. The energy required to unpair them exceeds the energy evolved when they form bonds.
Thus, they remain intact.
6. The melting points of the Group 13 elements do not show a regular trend (Fig. 2).

Fig. 1 Atomic and Ionic Radii Fig. 2 Melting Points of the Group 13 Elements
The very high melting point of boron is due to its unusual crystal structure. The small size and high ionization energy
makes boron a nonmetallic element, thus metallic bondings do not exist. The structure of boron is icosahedral (20-faced)
with boron atoms at all 12 corners (Fig. 2).
Gallium again has an unusual structure. Each metal atom has one close neighbour at a distance of 243 pm and six
more distant neighbours at distances between 270 pm and 279 pm.
In this structure, more or less diatomic molecules exist which accounts for the low melting point (about 30°C).
7. The variation of boiling point corresponds to the expected pattern as no unusual structures exist in liquid phase
(Fig. 3).
8. The densities increase on descending the group (Fig. 4).
9. The ionization energies do not follow the expected trend of decreasing values on descending the group
(Fig. 5).
The sum of the three ionization energies for each element is very high. This explains their tendency to form
covalent compounds. Boron has no tendency to form ions, but the other elements have this tendence in solution.
10. The standard electrode potential, E°(M3+| M), becomes more negative in going from B to Al indicating the
increase in metallic nature from B to Al. After Al, the values become less and less negative and for Tl, it
becomes positive indicating the reduction M3+ + 3e– Æ M becomes more easy. This explains the +3 oxidation
state becomes less stable in aqueous solution on descending the group.
Study of the p-Block Elements (Groups 13, 14 and 15) 15.3

Fig. 3 Icosahedral Structure of Boron Fig. 4 Melting Points of the Group 14 Elements

Fig. 5 Densities of the Group 13 Elements Fig. 6 Ionization Energies of the Group 13 Elements
On the other hand, the more negative value of E°(M +|M) for Tl than In indicates the more stability of Tl+ than In+.
Isolation of Boron
The chief minerals of boron are borax (Na2[B4O5(OH)4] ◊ 8H2O), i.e. Na2B4O7 ◊ 10H2O, colemanite (Ca2[B3O4(OH)3]2
◊ 2H2O) i.e. Ca2B6O11 ◊ 5H2O and kernite (Na2[B4O5(OH)4] ◊ 2H2O) i.e. Na2B4O7 ◊ 4H2O.
Boron is isolated by converting its mineral into boron trioxide followed by is reduction with magnesium.
Na2[B4O5(OH4)] ◊ 8H2O + 2HCl Æ 2NaCl + 5H2O + 4H3BO3
borax orthoboric acid
red heat
2H3BO3 ææææ
Æ B2O3 + 3H2O
B2O3 + Mg Æ 2B + 3MgO
Crystalline boron is obtained by the reduction of boron trichloride with zinc or dihydrogen at high temperature.
1200 K
2BCl3 + 3Zn æææÆ 3ZnCl2 + 2B
1200 K
2BCl3 + 3H2 æææÆ 2B + 6HCl

Chemical Properties of Group 13 Elements


1. Reaction with Oxygen All elements burn in oxygen at high temperatures forming M2O3. The element Tl also forms
some T12O.
15.4 Complete Chemistry—JEE Main

The reaction of aluminium with oxygen is strongly exothermic.


3
2Al(s) + O2(g) Æ Al2O3(s) Df H = – 1670 kJ mol–1
2
This reaction is known as the thermite reaction.
metallurgical extraction of other metals from their oxides.
3Mn3O4 + 8Al Æ 9Mn + 4Al2O3
Cr2O3 + 2Al Æ Al2O3 + 2Cr.
2. Reaction with Water Aluminium, in principle, can react with water. But the reaction does not continue due to the
formation of thin protective layer of Al2O3.
Reactions with Acids and Alkalis Aluminium reacts with dilute mineral acids.
2Al + 6HCl Æ 2A13+ + 6C1– + 3H2.
With concentrated nitric acid, the metal becomes passive due to the formation of protective layer of oxide.
Aluminium also dissolves in aqeous NaOH indicating that the element is amphoteric.
2Al + 2NaOH + 6H2O Æ 2NaAlO2 ◊ 2H2O + 3H2
Sodium aluminate

Compounds of Group 13 Elements

1. Oxides
All the elements of Group 13 form oxides (M2O3). Their acidic character reduces on descending the group.
B2O3 is acidic
Al2O3 and Ga2O3 are amphoteric
ln2O3 and T12O3 are basic
The oxides M2O3 are known as sesquioxide (sesqui means one and a half) as they can be represented as MO3/2.
2. Hydroxides
All the elements of Group 13 form hydroxides, M(OH)3. Their acidic character decreases on descending the group.
Boric acid, B(OH)3 or H3BO3 is obtained by the action of hydrochloric acid or sulphuric acid on borax. On cooling

Na2[B4O5(OH)4] ◊ 8H2O + 2HC1 Æ 2NaCl + 4H3BO3 + 5H2O


Na2[B4O5(OH)4] ◊ 8H2O + H2SO4 Æ Na2SO4 + 4H3BO3 + 5H2O
It may also be obtained by the hydrolysis of most boron compounds like nitrides, sulphides, etc.
2BN + 6H2O Æ 2H3BO3 + 2NH3
B2S3 + 6H2O Æ 2H3BO3 + 3H2S
Boric acid is a white crystalline substance, soft and soapy to touch. It is moderately soluble in cold water. On heating
it decomposes to form metaboric acid at 375 K, tetraboric acid at 435 K and boron trioxide at red heat.
375 K
H3BO3 ⎯−⎯⎯
H O→ HBO2
2
Metaboric acid

4HBO2 ⎯−435
⎯⎯ K
H O→ H2B4O7
2
Tetraboric acid
Red heat
H2B4O7 ⎯⎯⎯⎯
−H O → 2B2O3
2
Boric oxide
Boric acid is a very weak monobasic acid. It does not liberate hydrogen ion but accepts a hydroxyl ion, i.e. it behaves
as Lewis acid.

B(OH)3 + 2H2O   H3O+ + [B(OH)4]– pKa = 9.25
(or H3BO3) Metaborate ion
Study of the p-Block Elements (Groups 13, 14 and 15) 15.5

The structures of B(OH)3 and [B(OH)4]– are as follows.

The boric acid cannot be titrated satisfactorily with NaOH as sharp end point is not obtained. However, in the
presence of a cis-diol (glycerol, mannitol or sugars), boric acid acts as a strong acid and can be titrated with NaOH in
the presence of phenolphthalein indicator. With a cis-diol, the product [B(OH)4]– in the above reaction forms a strong
complex, causing the reaction to move in the forward direction.

Boric acid contains triangular planar BO3–


3 units. In the solid the B(OH)3 units are hydrogen bonded gether into two
dimensional sheets with almost hexagonal symmetry (Fig. 7).

Fig. 7 Hydrogen Bonded Structure of Boric Acid

3. Halides
All the elements of Group 13 form trihalides.
The boron halides are covalent. BF3 is gaseous, BC13 liquid and BI3 is solid. They are all hydrolyzed by water. BF3
is hydrolyzed incompletely while BCl3 and BI3 are completely hydrolyzed.
Hydrolysis of BF3: In this, the HF produced reacts with the H3BO3
15.6 Complete Chemistry—JEE Main

4BF3 + 12H2O Æ 4H3BO3 + 12HF


12HF+ 3H3BO3 Æ 3H+ + 3[BF4]– + 9H2Q
4BF3 + 3H2O Æ H3BO3 + 3H+ + 3[BF4]–

Hydrolysis of BCl3 (and BI3):


BCl3 + 3H2O Æ H3BO3 + 3HCl
BF3 is useful organic catalyst for Friedel-Crafts reactions. It is also used extensively as a polymerization catalyst. It
can be obtained from B2O3 or borax.
B2O3 + 6HF + 3H2SO4 Æ 2BF3 + 3H2SO4 ◊ H2O
Na2B4O7 + 12HF ⎯−⎯⎯
6H 2 O
→ [Na2O(BF3)4] ⎯2H
⎯⎯ 2 SO 4
→ 4BF3 + 2NaHSO4 + H2O

attaining an octet of electrons. The structure of dimer (AlCl3)2 is

This dimeric structure is retained in non-polar solvents such as benzene. In water, the high enthalpy of hydration is
◊ 6H2O]3+ and 3X– ions.
Anhydrous aluminium chloride is used as a catalyst in the Friedel-Crafts reaction. It is also used in petroleum
cracking and in the manufacture of dyes, drugs and perfumes.
3
by p bonding. The other elements in the group have larger atoms and cannot get effective p overlap, so they polymerize

The sp2 hybridization leads to the triangular planar geometry. The observed bond length of 130 pm is shorter than the
sum of the covalent radii (B = 80 pm and F = 72 pm). This is explained by the formation of pp–pp bond in which the
lone pair on F is extended to the empty p orbital of B. This is known as back bonding. Due to the resonating structures

all the three B—F bond distances are identical.


Acid Character of Boron Trihalides
Due to back bonding, the electron density on boron is increased. The tendency to form pp–pp bond is maximum in BF3 and
falls rapidly on passing to BCl3 to BBr3 due to the increase in size of halogen 2p orbital. Thus, tendency to accept electron
pair is minimum for BF3 and BC13 and is maximum for BBr3. Hence, the increasing order of acid strength is
BF3 < BCl3 < BBr3
Study of the p-Block Elements (Groups 13, 14 and 15) 15.7

Borax Borax is usually written as Na2B4O7 ◊ 10H2O. Its structure is


OH

B–
O O
HO — B O B — OH
O O
B–

OH
It contains two tetrahedral units and two triangular units.
Its actual formula is Na2[B4O5(OH)4] ◊ 8H2O
Borax is used as a primary standard for titrating acids.
Na2B4O7 ◊ 10H2O + 2HCl Æ 2NaCl + 4H3BO3 + 5H2O
Since the products contains boric acid, methyl orange (pH range 3.1-4.4) is used as the indicator. One mole of borax
reacts with two moles of acid. The actual reactions are
[B4O5(OH)4]2– + 5H2O 2B(OH)3 + 2[B(OH)4]–
2[B(OH)4]– + 2H3O+ Æ 2B(OH)3 + 4H2O
Borax is also used for making buffer solution as its aqueous solution contains equal amounts of weak acid and its
salts.
Borax Bead Test In the borax bead test, B2O3
loop of platinum wire. The mixture fuses to give a glass-like metaborate bead.
CoO + B2O3 Æ Co(BO2)2
Cobtalt metaborate
(blue colour)
Metaborate beads of many transition metals have characteristic colours and thus borax bead test is used to identify
the metals.
4. Hydrides
Elements of group 13 do not react directly with hydrogen. Their hydrides are prepared by indirect methods.
Boron forms a large number of hydrides, called boranes, of the general formula BnHn+4 (known as nido-boranes) and
BnHn+6 (known as arachno-boranes). The most important of these boranes is B2H6, called diborane. It is prepared by the
following methods:
3Mg + 2B æDæ → Mixture of boranes ⎯Heat
Æ Mg3B2 + H3PO4 ⎯⎯ ⎯→ B2H6
Magnesium boride Diborane
2BCl3 + 6H2 ⎯⎯⎯⎯⎯
→ B2H6 + 6HC1
Silent electric
discharge

4BC13 + 3LiAlH4 ⎯⎯
→ 2B2H6 + 3LiCl + 3A1C13
2NaBH4 + I2 ⎯⎯
→ B2H6 + H2 + 2NaI
(in polyether solvent)
250 atm
B2O3 + 3H2 + 2Al ææææ
150∞C
Æ B2H6 + Al2O3
Structure of Diborane In diborane there are 12 valency electrons, three from each B atom and six from the H atoms.
The structure of diborane is as follows.
15.8 Complete Chemistry—JEE Main

Hb

tH Ht

B B Terminal B—H is 2c – 2e bond with bond length 119 pm


Bridged B—H is 3c – 2e bond with bond length 133 pm
tH Ht
Hp

Hts are terminal hydrogen atoms, they are in the same plane including B atoms. The other H atoms are in the
perpendicular plane, they lie centrally between two B atoms. The terminal B—H bonds are the normal two-centre two-
electron bonds. The bridging H—B—H bonds are the three-centre two-electron bonds.
The higher boranes have an open cage structure.

B2H6 + 2(CH3)3N: Æ 2[(CH3)3N Æ BH3]


Their reactions with ammonia depend on the experimental conditions.
low temperature
B2H6 + NH3 B2H6 ◊ 2NH3
excess

higher temperature
(BN)x
Boron nitride
If the ratio of B2H6 and NH3 is 1:2, the product obtained at higher temperature is borazine (B3N3H6).
The structure of boron nitride is very similar to graphite (as the sum of valence electrons of B and N is equal to that
of two C atoms). Borazine is known as inorganic benzene.
H H

B B B H B H H + B H
N N N N N N N N +

B B B B B B – B B
N N N H N H H N+ H
B B B H H
N N

Boron nitride Borazine


Diborane is a powerful electrophilic reducing agent for certain functional groups. For example
B H
RCN æææ
2 6
Æ RCH2NH2
B H
RNO2 æææ
2 6
Æ RNH2
B H
RCHO æææ 2 6
Æ RCH2OH
The other elements of the group 13 form only a few stable hydrides. Thus, (AlH3)n exists as a white polymer
containing aluminium atoms joined together by hydrogen bridges, Al—H—Al. Aluminium hydride can be prepared from
LiH and AlCl3 in ether solution.
3nLiH + nAlCl3 Æ (AlH3) + 3nLiCl
However, if excess of LiH is present, the product obtained is lithium aluminium hydride, LiAlH4.
Boron also forms the compounds called borohydrides containing the tetrahedral BH 4– unit. Borohydrides are obtained
by the action of diborane on alkali metal hydrides (in ether).
2MH + B2H6 Æ 2M+[BH4]–
(where M = Li or Na)
Study of the p-Block Elements (Groups 13, 14 and 15) 15.9

Sodium borohydride may be prepared as follows:


4NaH + B(OCH3)3 Æ 3NaOCH3 + Na[BH4]
Methyl borate
Alkali metal borohydrides are valuable reducing agents in inorganic chemistry.
Aluminium and gallium also form compounds analogous to borohydrides, e.g. Li[AlH4], Li[GaH4] LiAlH4 is used
as a reducing agent in organic chemistry for selectively reducing the functional groups.

Diagonal Relationship of Boron with Silicon


Boron shows anomalous behaviour in its groups because of its small size and nonavailability of d orbitals. It resembles
silicon, the second member of the next higher group.
1. Both boron and silicon are non-metals, and exist in allotropic forms. They have high melting points
and are semiconductors.
The other members of Group 13 are metals.

respectively. The hydrides are readily hydrolyzed. The lower hydrides can be obtained by the reduction of
chlorides with LiAlH4.
4BCl3 + 3LiAlH4 Æ 3AlCl3 + 3LiCl + 2B2H6
SiCl4 + LiAlH4 Æ AlCl3 + LiCl + SiH4
In contrast, aluminium hydride is a polymeric solid.
3. Both boron and silicon form halides which are readily hydrolyzed.
BCl3 + 3H2O Æ B(OH)3 + 3HCl
Boric acid
SiCl4 + 4H2O Æ Si(OH)4 + 4HCl
Silicic acid
The aluminium halides are only partly hydrolyzed in water.
4. B2O3 is an acidic oxide, like SiO2.
B2O3 + 6NaOH Æ 2Na3BO3 + 3H2O
SiO2 + 2NaOH Æ Na2SiO3 + H2O.
Al2O3 is an amphoteric oxide.
5. Both boron and silicon react with several metals to form metal borides and silicides.
3Mg + 2B Æ Mg3B2
Magnesium boride

2Mg + Si Æ Mg2Si
Magnesium silicide
Borides and silicides get decomposed by dilute acids to give volatile hydrides (boranes, silanes).
6. Both boron and silicon, as well as their oxides, react with alkalis to form borates and silicates containing BO4
and SiO4 tetrahedral units, respectively.

MULTIPLE CHOICE QUESTIONS

1. Which of the following elements does not belong to Group 13 of the periodic table?
(a) Boron (b) Aluminium (c) Gallium (d) Tin
2. The number of elements in Group 13 is
(a) 4 (b) 5 (c) 6 (d) 7
3. The atomic number of indium which belongs to 5th period is
(a) 47 (b) 48 (c) 49 (d) 50
15.10 Complete Chemistry—JEE Main

4. Which of the following statements is not true?


(a) Boron is a nonmetal while all other elements of Group 13 are metals
(b) Boron trichloride acts as a Lewis acid
(c) Boron does not form the hydrated B3+ ion
(d) The stability of the +1 oxidation state decreases down the group whereas that of +3 increases
5. Which of the following facts is not correct?
(a) Elements of Group 13 do not react directly with hydrogen
(b) The boron hydrides are known as boranes
(c) The structure of diborane is H3B – BH3
(d) Aluminium hydride exists as white polymer
6. The geometry of BH 4– unit is
(a) square planar (b) tetrahedral (c) octahedral (d) trigonal pyramidal
7. Which of the following statements is not correct?
(a) LiAlH4 and LiGaH4 are reducing agents in inorganic chemistry
(b) BF3 is a useful catalyst in Friedel-Crafts reactions
(c) LiAlH4 is used as a reducing agent in organic chemistry for selectively reducing the functional groups

8. Which of the following oxides is an acidic oxide?


(a) B2O3 (b) Al2O3 (c) Ga2O3 (d) In2O3
9. Which of the following oxides is an amphoteric oxide?
(a) B2O3 (b) Al2O3 (c) In2O3 (d) Tl2O3
10. Which of the following oxides is a basic oxide?
(a) B2O3 (b) Al2O3 (c) Ga2O3 (d) In2O3
11. Which of the following equations is not correctly formulated?
(a) Na2[B4O5 (OH)4] · 8H2O + 2HCl 2NaCl + 4H3BO3 + 5H2O
(borax)

(b) 2BN + 6H2O 2H3BO3 + 2NH3


375 K 435 K red heat
(c) H3BO3 æææÆ HBO2; 4HBO2 æææÆ H2B4O7; H2B4O7 æææÆ 2B2O3
- H2O - H2O - H 2O
(metaboric acid) (tetraboric acid) (Boric oxide)

(d) H3BO3 is a weak monobasic acid as it liberates hydrogen ions as


H3BO3 H + + H2BO3–
12. The diagonal relationship of boron is with
(a) carbon (b) silicon (c) magnesium (d) phosphorus
13. Which of the following facts regarding boron and silicon is not true?
(a) Boron and silicon are semiconductors
(b) Boron and silicon form halides which are not hydrolysed
(c) Boron and silicon react with magnesium to form magnesium boride and magnesium silicide which are
decomposed by acids to give volatile borane and silane, respectively
(d) Both boron and silicon react with alkalis to form borates and silicates containing BO4 and SiO4 tetrahedral
units, respectively

(a) B (b) Al (c) Ga (d) In


15. Which of the following ions is hydrolysed to the maximum extent?
(a) Al3+ (b) Ga3+ (c) Tl+ (d) Tl3+
16. The corrosion of aluminium is prevented by the formation of impervious surface coating of
(a) AlN (b) Al(OH)3 (c) Al2(CO3)3 (d) Al2O3
17. The precipitate of Al(OH)3 dissolves in NaOH solution. It is due to the formation of
(a) Al(H2O)4 (OH)+2 (b) Al(H2O)3 (OH)3 (c) Al(H2O)2 (OH)–4 (d) Al(OH)63–
Study of the p-Block Elements (Groups 13, 14 and 15) 15.11

18. Which of the following does not represent an alum?


(a) KAl(SO4)2 · 12H2O (b) NH4 Al(SO4)2 · 12H2O
(c) NH4 Cr(SO4)2 · 12H2O (d) Al2(SO4)3 · 18H2O
19. Which of the following ions shows the largest tendency to form complexes with Al3+?
(a) H2O (b) F – (c) Cl– (d) SO42–
20. The treatment of Mg3B2 with acids produces
(a) B2O3 (b) MgB2O4 (c) B4H10 (d) MgB4O7
21. B3+ cannot exist in aqueous solution because of its
(a) strong reducing ability (b) strong oxidizing ability
(c) small size and large charge (d)large size and small charge
22. The following solid exists in the form of ions
(a) BI3 (b) BCl3 (c) BF3 (d) CaB2O4

23. The geometry of BO3 ion is
(a) tetrahedral (b) octahedral (c) square planar (d) triangular planar
24. The reaction in which metal oxide is reduced by Al is known as
(a) Silberschmidt reaction (b) Bayer reaction (c) Goldschmidt reaction (d) Rosenmud reduction
25. Which of the following elements has the largest melting point?
(a) B (b) Al (c) Ga (d) In
26. Which of the following elements has the minimum melting point?
(a) B (b) Al (c) Ga (d) In
27. Which of the following statements regarding boron is not correct?
(a) Boron compounds act as Lewis acids (b) Boron is a good conductor of electricity
(c) Colemanite is one of the chief minerals of boron (d) Boron exists in two isotopic forms 10B and 11B
28. Which of the following statements regarding boron is not correct?
(a) Boron has high melting and boiling points
(b) The hydrides of boron are known as boranes
(c) The structure of B2H6 is H3B — BH3
(d) In Borax, two boron atoms are in triangular geometry and two boron atoms are in tetrahedral geometry
29. Which of the following is boric acid?
(a) HBO2 (b) H3BO3 (c) H2B4O7 (d) B2O3
30. Which of the following is metaboric acid?
(a) HBO2 (b) H3BO3 (c) H2B4O7 (d) B2O3
31. Which of the following is tetraboric acid?
(a) HBO2 (b) H3BO3 (c) H2B4O7 (d) B2O3
32. Which of the following statements is not correct?
(a) Boric acid is a very weak monobasic acid
(b) Boric acid contains planar BO3 units which are bonded together through hydrogen bonds forming a layer
structure
(c) Boric acid is used as a food preservative
(d) Boric acid is a tribasic acid
33. Which of the following statements is not correct?
(a) The aqueous solution of boric acid serves as mild antiseptic
(b) Boron-10 has a high ability to absorb neutrons

(d) Metaboric acid is formed by the dehydration of tetraboric acid

(a) NH3 (b) NH2OH (c) BCl3 (d) H3N Æ BCl3


35. Which of the following is the correct formula of borax?
(a) Na2[B4O5(OH)4] · 2H2O (b) Na2[B4O5(OH)4] · 4H2O
(c) Na2[B4O5(OH)4] · 6H2O (d) Na2[B4O5(OH)4] · 8H2O
15.12 Complete Chemistry—JEE Main

36. Orthoboric acid is


(a) monobasic (b) diabasic (c) tribasic (d) tetrabasic
37. Which of the following minerals does not contain aluminium?
(a) Cryolite (b) Feldspar (c) Beryl (d) Olivine
38. The composition of beryl is
(a) Be3Cr2Si6O18 (b) Be3Al2Si6O18 (c) KAlSi3O11(OH)2 (d) KAlSi3O6
39. Which of the following formulae represents mica?
(a) Be3Cr2Si6O18 (b) Be3Al2Si6O18 (c) KAlSi3O11(OH)2 (d) KAlSi3O6
40. Which of the following statements regarding aluminium is not correct?
(a) Aluminium is a light metal with considerable strength
(b) Aluminium is a good conductor of heat and electricity
(c) The corrosion of the aluminium metal is prevented by the formation of a coat of aluminium oxide on its layer
(d) Aluminium dissolves in hydrochloric acid but not in concentrated sodium hydroxide
41. Chemically, potash alum is
(a) K2SO4 · Al2(SO4)3 · 6H2O (b) K2SO4 · Al2(SO4)3 · 12H2O
(c) K2SO4 · Al2(SO4)3 · 18H2O (d)K2SO4 · Al2(SO4)3 · 24H2O
42. Which of the following metals has the largest abundance in the earth’s crust?
(a) Aluminium (b) Calcium (c) Magnesium (d) Sodium
43. The chief ore for the extraction of aluminium is
(a) bauxite (b) cryolite (c) beryl (d) mica
44. Aluminium chloride at higher temperatures exists as
(a) trigonal planar AlCl3 (b) dimer Al2Cl6 (c) Al3+(3Cl–) (d) (AlCl2)+ Cl–
45. In the gaseous phase, aluminium chloride at low temperatures (420 – 480 K) exists as
Cl Cl
(a) trigonal planar AlCl3 (b) dimer Al2Cl3, i.e. Cl Al ... Al Cl
Cl Cl Cl Cl Cl
3+ –
(c) dimer Al2Cl3, i.e. Al Al (d) Al (3Cl )
Cl Cl Cl
46. Which of the following statements regarding the structure aluminium chloride is correct?
(a) All the bond angles Cl—Al—Cl and Al—Cl—Al in Al2Cl3 are identical
(b) All the bond lengths Cl —Al in Al2Cl3 are identical
(c) All the bond lengths Cl—Al as well as all the bond angles Cl—Al—Cl and Al—Cl—Al are different
(d) The bond lengths of terminal Al —Cl and bridged Al—Cl bonds are different and also outer bond angle
Cl—Al—Cl and bridged bond angles Cl—Al—Cl and Al—Cl—Al have different values.
47. Which of the following alloys does not contain aluminium?
(a) Duralumin (b) Magnalium (c) Alnico (d) Constantan
48. Which of the following oxides has the largest percentage in a usual sample of Portland cement?
(a) MgO (b) CaO (c) SiO2 (d) Al2O3
49. Which of the following oxides has the lowest percentage in a usual sample of Portland cement?
(a) SO2 (b) MgO (c) SiO2 (d) Al2O3
50. The most common ore of aluminium is
(a) bauxite (b) alumina (c) potash alum (d) cryolite
51. In the aluminothermite process, aluminium acts as

52. Hydrogen gas will not reduce heated


(a) copper(II) oxide (b) iron(III) oxide (c) tin(IV) oxide (d) aluminium(III) oxide
53. Which of the following is an amphoteric oxide?
(a) B2O3 (b) Al2O3 (c) Ga2O3 (d) PbO
Study of the p-Block Elements (Groups 13, 14 and 15) 15.13

2)
is added to
(a) make solution conducting (b) make solution anhydrous
(c) lower the temperature of the melt (d) prevent hydrolysis of aluminium salt
55. Heating of Al2(SO4)3 gives
(a) Al2O3 and O2 (b) Al2O3 and SO2 (c) Al2O3 and SO3 (d) Al2O3 S and O2
56. Chrome alum is
(a) K2SO4 · Cr2 (SO4)3 · 6H2O (b) K2SO4 · Cr2 (SO4)3 · 12H2O
(c) K2SO4 · Cr2 (SO4)3 · 18H2O (d) K2SO4 · Cr2 (SO4)3 · 24H2O
57. Heating of diborane with excess of ammonia to a high temperature yields
(a) borane (b) diammoniate of borane
(c) borazine (d) borone nitride
58. Which of the following statements is incorrect?
(a) Boron does not form ionic compounds with the formation of B3+ ions
(b) Borane exhibits inert pair effect
(c) Aluminium carbide on reacting with water liberates methane
(d) Borane nitride on hydrolysis liberates NH3
59. Which of the following statements is not correct?
(a) Boron BF3 involves sp2 hybridization (b) BF3 acts as a Lews acid
(c) BF3 is a volatile liquid (d) forms an adduct with NH3
60. Which of the following statements is correct?
(a) Borax involves two boron atoms in sp2 hybridization and two boron atoms in sp hybridization
(b) Borax involves two boron atoms in sp2 hybridization and two boron atoms in sp3 hybridization
(c) Borax involves all the four atoms in sp2 hybridization
(d) Borax involves all the four atoms in sp3 hybridization
61. Borax has the molecular formula
(a) Na2[B4O3(OH)4].6H2O (b) Na2[B4O5(OH)4].6H2O
(c) Na2[B4O5(OH)4].8H2O (d) Na2[B4O6(OH)2].8H2O
62. In the reaction H3BO3 æ375
ææ K
Æ A æ435
ææ K
Æ B æred
æææheat
ÆC
The compounds A,B and C, respectively, are
(a) B2O3, H2B4O7, HBO2 (b) B2O3, HBO2, H2B4O7
(c) H2B4O7, HBO2, B2O3 (d) HBO2, H2B4O7, B2O3
63. In borax, the number of —OH group attached to boron atoms is
(a) 2 (b) 3 (c) 4 (d) 6
64. The number of water of crystallization in borax is
(a) 6 (b) 8 (c) 10 (d) 12

ANSWERS
1. (d) 2. (b) 3. (c) 4. (d) 5. (c) 6. (b)
7. (d) 8. (a) 9. (b) 10. (d) 11. (d) 12. (b)
13. (b) 14. (a) 15. (a) 16. (d) 17. (c) 18. (d)
19. (b) 20. (c) 21. (c) 22. (d) 23. (d) 24. (c)
25. (a) 26. (c) 27. (b) 28. (c) 29. (b) 30. (a)
31. (c) 32. (d) 33. (d) 34. (c) 35. (d) 36. (a)
37. (d) 38. (b) 39. (c) 40. (d) 41. (d) 42. (a)
43. (a) 44. (a) 45. (c) 46. (d) 47. (d) 48. (b)
49. (d) 50. (a) 51. (c) 52. (d) 53. (b) 54. (c)
55. (c) 56. (d) 57. (c) 58. (b) 59. (c) 60. (b)
61. (c) 62. (d) 63. (c) 64. (b)
15.14 Complete Chemistry—JEE Main

HINTS AND SOLUTIONS


3. The atomic number of indium is 2 + 8 + 8 + 18 + 13 = 49.
4. The stability of the +1 oxidation state increases down the group. This is because of the participation of only p
electron. The two s-electrons (s2) do not take part in the chemical reaction (inert pair effect).
5. The structure of diborane is

The two BH2 groups lie in the same plane while the two bridging hydrogen atoms lie in a plane perpendicular to
this plane.

dimers.
11. H3BO3 is a weak monobasic acid. It does not liberate H + but accepts OH–, i.e. it is Lewis acid.
13. The halides of B and Al are hydrolysed.
BCl3 + 3H2O B(OH)3 + 3HCl and SiCl4 + 4H2O Si(OH)4 + 4HCl
Boric acid Silicic acid
20. The treatment of borides with acids produces boranes.
27. Boron has a very low electrical conductivity.
28. The structure of B2H6 is
H H H
B B
H H H
The two BH2 groups lie in the same plane while the two bridging hydrogen atoms lie in a plane perpendicular to
this plane.
29. Boric acid is H3BO3. It is a monobasic acid. It does not liberate hydrogen ion but accepts a hydroxyl ion, i.e., it
behaves as a Lewis acid.
375 K
30. Metaboric acid is formed by heating of boric acid. H3BO3 æææÆ HBO2
- H2O
37. Olivine is (Mg, Fe)2 SiO4.
40. Aluminium dissolves both in HCl and conc. NaOH
2Al + 6HCl Al2Cl3 + 3H2 and 2Al + 2NaOH + 6H2O 2Na[Al(OH)4] + 3H2O
43. Bauxite is Al2O3 ·2H2O.
46. The structure of Al2Cl6 is
Cl Cl Cl
101°
118° Al Al 206 pm
79°

Cl Cl 221 pm
Cl

47. Constantan is an alloy of Cu (60%) and Ni (40%).


55. The reaction is Al2(SO4)3 Æ Al2O3 + 3SO3
56. The chrome alum involves 24 water of crystallization.
excess NH
57. B2H6 æææææ
high temp
3
Æ (BN)x

58. Inert pair effect operates on the elements at the end of the group.
Study of the p-Block Elements (Groups 13, 14 and 15) 15.15

59. BF3 is a gaseous substance.


60. The structure of borax is
OH


B
O O

HO B O B OH
O O
B–

OH
61. See Q.60
- H2O
62. The reactions are H3BO3 æ375
ææ K
Æ HBO2 æ435
ææ K
Æ H2B4O7 ææææ Æ B 2O 3
red heat

63. See Q.60 64. The formula of borax is Na2[B4O5(OH)4] .8H2O

MULTIPLE CHOICE QUESTIONS FROM AIEEE AND JEE MAIN

1. Aluminium chloride exists as dimer, Al2O6 in solid state as well as in solution of non-polar solvents such as
benzene. When dissolved in water, it gives
(a) Al3+ + 3Cl– (b) [Al(H2O)6]3+ + 3Cl– (c) [Al(OH)6]3– + 3HCl (d) Al2O3 + 6HCl [2004]
2. The structure of diborane is
(a) two 2c – 2e bonds and two 3c – 3e bonds (b) four 2c – 2e bonds and four 3c – 2e bonds
(c) four 2c – 2e bonds and two 3c – 2e bonds (d) two 2c – 2e bonds and four 3c – 2e bonds [2005]
3. Which one of the following is the correct statement?
(a) B2H6 . 2NH3 is known as inorganic benzene.
(b) Boric acid is a protonic acid
(c) Beryllium exhibits coordination number of six
(d) Chlorides of both beryllium and aluminium have bridged chloride structures in solid base. [2008]
4. Boron cannot form which one of the following anions?
(a) BO2– (b) BF63– (c) BH4– (d) B(OH)4–
[2011 cancelled]
5. Which of these statements is not true ?
(a) NO+ is isoelectronic with O2
(b) B is always covalent in its compounds
(c) In aqueous solution, the Tl+ ion is much more stable than Tl (III)
(d) LiAlH4 is a versatile reducing agent in organic synthesis. [2014, online]
6. In the following sets of reactants which two sets best exhibit the amphoteric character of Al2O3. xH2O ?
Set 1: Al2O3 . xH2O (s) and OH- (aq)
Set 2: Al2O3 . xH2O (s) and H2O (l)
Set 3: Al2O3 . xH2O (s) and H+ (aq)
Set 4: Al2O3 . xH2O (s) and NH3 (aq)
(a) 1 and 2 (b) 1 and 3 (c) 2 and 4 (d) 3 and 4 [2014, online]
15.16 Complete Chemistry—JEE Main

7. Identify the reaction which does not liberate hydrogen:


(a) Reaction of lithium hydride with B2H6

(c) Reaction of zinc with aqueous alkali


(d) Allowing a solution of sodium in liquid ammonia and stand [2016 online]

ANSWERS
1. (b) 2. (c) 3. (d) 4. (b) 5. (a) 6. (b)
7. (a)

HINTS AND SOLUTIONS


1. In water AlCl3 exists as [Al(H2O)6]3+ and 3Cl– ions. Therefore, the choice b is correct.

H H H
2. The structure of B2H6 is Be Be
H H H

Terminal Be—H is 2c–2e bond


Bridged Be—H—Be is 3c–2e bond
N B
3. Inorganic benzene is borazine B3N3H6 with the structure B N
N B
Boric acid is a Lewis acid. It does not liberate H+ but accepts a hydroxyl ion.
Beryllium exhibits coordination number of four.
Both BeCl2 and AlCl3 exist as bridged chlorides
Cl Cl Cl Cl
Cl Be Be — Cl ; Al Al
Cl Cl Cl Cl
2 2 2
(2s) (2p) . Its valence-shell has only four orbitals and thus it can show
a maximum of four valency.
Hence, the choice (b) is correct (because the formation of BF63– is not possible).
5. NO+ contains 14 electrons while O2 contains 16 electrons. These are not isoelectronic.
6. In acidic medium, aluminium exists as Al3+ and in alkaline medium, it exists as AlO2– (aluminate ion). Thus, Set
1 and Set 3 (choice b) represent amphoteric nature of aluminium.
7. The reaction of B2H6 with LiH is 2 LiH + B2H6 2Li(BH4)

The Group 14 Elements – Carbon Family


Group 14 of the periodic table consists of elements carbon (C), silicon (Si), germanium (Ge), tin (Sn) and lead (Pb).
The physical properties of these elements are recorded in Table 2.
Study of the p-Block Elements (Groups 13, 14 and 15) 15.17

Table 2 Atomic and Physical Properties of Group 14 Elements

Property C Si Ge Sn Pb
[He](2s) (2p) [Ne](3s) (3p) [Ar](3d) (4s) (4p) [Kr](4d) (5s) (5p) [Xe](4f)14(5d)l0(6s)2(6p)2
2 2 2 2 10 2 2 10 2 2

Atomic number 6 14 32 50 82
Relative atomic mass 12.011 28.086 72.59 118.69 207.2
a
Covalent radius, r/pm 77.2 117.6 122.3 140.5 146
Ionic radius M4+, r/pmb — 40 53 69 78
2+ b
Ionic radius M , r/pm — — 73 118 119
Melting point, T/K 4373 1693 1218 505 600
Boiling point, T/K — –3550 3123 2896 2024
d
Density at 293 K, r/g cm –3
3.51° 2.34 5.32 7.27 11.34
–1
Ionization energy, I/kJ mol
I 1086 786 761 708 715
II 2352 1577 1537 1411 1450
III 4619 3228 3301 2942 3081
IV 6221 4354 4409 3929 4082
Electronegativity 2.5 1.8 1.8 1.8 1.9
Electrical resistivity at 293 K,
r/ohm cm 1014 – 1016 ~48 ~47 1.1 ¥ 10–5 2 ¥ 10–5
(a) tetracovalent, (b) six-coordinate, (c) for diamond form, (d) (b-form stable at room temperature.

Description of Physical Properties


ns)2(np)2, where n varies from 2 to 6. The elements
exhibit +4 covalent oxidation state.

size, high electronegativity and nonavailability of d orbitals.


3. The group starts from a nonmetal and ends with a metal. There is increase in the metallic character on
descending the group.
C and Si are nonmetals.
Ge is a nonmetal but also has some metallic characteristics
Sn and Pb are metals.
4. The covalent and ionic radii increase on descending the group. (Fig. 8).
The small difference in sizes between Si and Ge is due the intervening 3d electrons in Ge which shields the
nuclear charge less effectively.
Similarly, the small difference in sizes between Sn and Pb may be explained on the basis of intervening 4f
electrons in case of Pb.
5. The melting points decrease on descending the group, with the exception of Pb whose melting point
is slightly higher than that of Sn (Fig. 9).
15.18 Complete Chemistry—JEE Main

Fig. 8 Covalent and Ionic Radii of the Group 15 Fig. 9 Melting Points of the Group 14 Elements Elements
Carbon has extremely high melting point. This is due to the stronger C—C bonds in the network of carbon
atoms which has face-centred cubic lattice (known as diamond lattice).
The melting points of Si and Ge are also high. Both these elements have diamond type crystal lattice. The
decrease in melting point from C to Si to Ge is due to the weakening of the M—M bond caused by the increase
in covalent radii of the element.
The melting points of Sn and Pb are low. They do not use all four outer electrons for metallic bonding.
6. The boiling points of the elements of Group 14 also decrease on descending the group.
7. The ionization energies decrease from C to Si, but then change in an irregular way because of the effects of
4+
ions is excluded.

oxidation state of these elements in the compounds is primarily +2.


8. The phenomenon of catenation (ability to like atoms to link one another through covalent bonds) decreases
down the group. This phenomenon is linked with the M—M bond enthalpy, which decreases down the group.
9. Besides sigma bond, carbon forms pp—pp multiple bonding with itself and with other nonmetals; especially
nitrogen and oxygen (e.g. C == C, C ∫∫ C, C ∫∫ N, C == O, C == S).
The compounds of Si with multiple bond are rare. However, the element can form double bond through dp—pp
bonding, in which the lone pair in p orbital of an atom is extended to an empty orbital of Si. This is known as
back bonding. One of the examples of trisilylamine N(SiH3)3.
10. Maximum covalency of carbon is four, whereas the other elements of group can exhibit maximum valence of
six due to the participation is d orbitals (e.g. [SiF6]2–, [Sn(OH)6]2–, [GeCl6]2– and [PbCl6]2–).

Trisilylamine

Fig. 10 The First Four Ionization Energies of the Group 14 Elements.


Study of the p-Block Elements (Groups 13, 14 and 15) 15.19

Chemical Properties of Group 14 Elements


The reactivity of elements decreases down the group. The inert pair effect becomes increasingly effective down the
group. The stability of the +4 oxidation state decreases while that of the +2 oxidation state increases on descending the
group. For example,
Sn2+ exists as simple ion and is strongly reducing.
Sn4+ is covalent.
Pb2+ is ionic, stable and more common than Pb4+.

Action of Water C, Si and Ge are unaffected by water. Sn reacts with steam to give SnO2 and H2. Pb is unaffected
by water due to the formation of protective oxide layer at the surface.
Action of Acids C, Si and Ge are unaffected by dilute acids. Sn and Pb reacts with dilute nitric acid.
4Sn + 10HNO3 Æ 4Sn(NO3)2 + NH4NO3 + 3H2O
(dilute)
3Pb + 8HNO3 Æ 3Pb(NO3)2 + 2NO + 4H2O.
(dilute)

Action of Alkalis
C is not affected by alkalis.
Si reacts with alkalis forming silicates.
Sn and Pb also react with alkalis forming stannate, [Sn(OH)6]2–, and plumbate [Pb(OH)6]2–.
These reactions show that Sn and Pb are amphoteric.
Action of Halogens
Graphite but not diamond is affected by F2 at higher temperatures giving (CF)n.
Si and Ge react with all halogens, forming volatile SiX4 and GeX4.
Sn and Pb are less reactive but do react giving SnX4 and PbX2.

Compounds of Group 14 Elements

1. Hydrides
All the elements of Group 14 form covalent hydrides. Carbon forms a very large number of compounds which include
alkanes (CnH2n+2), alkenes (CnH2n), alkynes (CnH2n–2) and aromatic compounds.
Silicon forms a limited number of saturated hydrides (SinH2n+2) called silanes, which are strong reducing agents. One
of the methods used in the preparation of silanes is by the reduction of silane halides by LiAlH4, LiH or NaH.
SiCl4 + LiAlH4 Æ SiH4 + AlCI3 + LiCl
Si2Cl6 + 6LiH Æ Si2H6 + 6LiCl
Si3Cl8 + 8NaH Æ Si3H8 + 8NaCl
Silanes are easily hydrolyzed in alkaline medium.
Si2H6 + (4 + n) H2O ⎯trace
⎯⎯⎯⎯of alkali
→ 2SiO2 ◊ nH2O + 7H2

Stannane (SnH4) and plumbane (PbH4) are also known but are less stable.
2. Halides
All the elements of Group 14 form tetrahalides with the exception of PbI4 which is not known. Probably, this is due to
the fact that PblV is a strong oxidizing agent and I– is a strong reducing agent, so both strong oxidizing and reducing
agents cannot exist together. They are all covalent and volatile with the exceptions of SnF4 and PbF4 which have three-
dimensional structures and are high melting.
The stability of the halides decreases down the group. CCl4 is stable while other halides are hydrolyzed due to the
availability of d orbitals. The hydrolysis of SiCl4 may be represented as follows.
15.20 Complete Chemistry—JEE Main

The hydrolysis of SiF4 besides producing SiO2 also produces [SiF6]2–


SiF4 + 2H2O Æ SiO2 + 4HF
SiF4 + 2F– Æ [SiF6]2–

3. Oxides
The elements of Group 14 form oxides of the type MO and MO2. The well-known oxides of carbon are CO and CO2.
Besides these, less stable C3O2, C5O2 and C12O9 are also known.
Carbon monoxide is extremely poisonous gas. If inhaled, it forms a complex with haemoglobin in the blood which
is much more stable than oxy-haemoglobin complex. This prevents the haemoglobin in the red blood corpuscles from

CO is a good reducing agent. It is also an important ligand which can donate or share a lone pair of electrons located
on carbon atom.
Carbon dioxide is a colourless, odourless gas, which play a vital role in the photosynthesis. Carbon dioxide is
represented as O == C == O. Each double bond involves one s bond and one p bond. The formation p bonds is possible
because of the small size of carbon and thus the overlapping between 2p(C) and 2p(O) are possible. Because of the double
bonds, CO2 exists as discrete molecule and is a gas.
Silicon also forms silicon dioxide which is solid and high melting. Silicon is not able to form pp-pp double bonds with
oxygen due to its large size which prevents effective overlap between 2p(Si) and 2p(O) orbitals. Instead, a continuous

tetrahedrally to four oxygen atoms and each oxygen atom is shared by two silicon atoms, as shown in Fig. 11.
Acidic nature of the dioxides decreases down the group.
CO2 and SiO2 are purely acidic.
GeO2 is weakly acidic.
SnO2 and PbO2 are amphoteric.

Fig. 11 Structure of Silicon Dioxide


The lower oxides GeO, SnO and PbO are also known. They are slightly more basic and ionic than the corresponding
higher oxides.
GeO is purely acidic
SnO and PbO are amphoteric.
Study of the p-Block Elements (Groups 13, 14 and 15) 15.21

The stability of lower oxide increases down the group.


Lead also forms a mixed oxide 2PbO ◊ PbO2 (i.e. Pb3O4). It is used as red pigment.
Silicates The earth’s crustal rocks and their breakdown products like clays, soils and sands, are mostly silicates
and silica. Mica, asbestos, quartz, feldspar, zeolites, etc. are some of the important silicate minerals. Basically, all the
silicates and silica contain SiO4–
4 tetrahedra. (Fig. 12).

4–
O –


Si = o
= Si
O O O

Fig. 12 The basic out of SiO4–


4

Depending on the number of corners (0, 1, 2, 3 or 4) of SiO4–


4 tetrahedron shared with other tetrahedra, a variety of
silicates are obtained. These are described below.
Orthosilicates
These contain individual discrete SiO4–
4 tetrahedra. Examples are phenacite (Be2SiO4), willemite (Zn2SiO4), and zircon
(ZnSiO4) minerals.
Pyrosilicates
These contain discrete Si2O6– 4–
7 ions formed by the sharing of one oxygen atom between be two SiO4 tetrahedra (Fig.
13). This structure is known as an island structure. Examples include thortveitite (Sc2Si2O7) and hemimorphite
(Zn4(OH)2Si2O7).

= Oxygen
= Silicon

Fig. 13 Island structure

Chain and Cyclic Silicates


These silicates are formed when two oxygen atoms of SiO4– 4 tetrahedron are shared with two other tetrahedra. The
resulting structure may be the chain type (Fig. 14) or the cyclic type (Fig. 15). The general formula of these silicates is
n, . Examples of cyclic silicates are benitoite (BaTiSi3O9), catapleite (Na2ZrSi3O9 ◊ 2H2O), dioptase (Cu6Si6O18
(SiO3)2n–
◊ 6H2O) and beryl (Be3Al2Si6O18).
– –

= Oxygen
– –
= Silicon

– – –

– –

Fig. 14 Chain structure


15.22 Complete Chemistry—JEE Main

– –

– –
– – –

– – = Oxygen
– – = Silicon
– –

=O – –
– = Si

Fig. 15 Cyclic structure


4–
4 tetrahedron
are shared with the adjacent tetrahedra. If
further sharing of oxygen atoms occurs by half of the silicon atom, a double chain or band structure is formed. Examples
of the former include enstatite MgSiO3 and diopside CaMg(SiO3)2. Asbestos also contains silicate chain. The band
structure contains the basic Si4O6–
11 repeating unit. An example is Ca2Mg5(OH)2(Si4O11)2.

Layer and Sheet Silicates

empirical formula (Si2O5)2n–


n . This is shown in Fig. 16. The minerals kaolin [Al2(OH)4Si2O5], talc [Mg3(OH)2Si4O10]
and the micas are common examples possessing layer and sheet structures.

= Oxygen
= Silicon

Fig. 16 Layer and Sheet Structure

Three-dimensional Silicates
In these silicates, all the four oxygen atoms of a SiO44 – tetrahedron are shared with other tetrahedra resulting in a three-
dimensional lattice. In case there is no replacement of silicon by other metals, the resulting formula is SiO2 (quartz).
General Structure of Silicates The number of oxygen atoms shared in the general formula of silicate may be
determined as follows.

Ê Number of ˆ
Ë shared ¯
( )(
Number of Charge on + Charge on
Á O atoms ˜ = Si atoms Si atom )(
silicate ion )
(Number of Si atoms)
Study of the p-Block Elements (Groups 13, 14 and 15) 15.23

For example,
Silicate anion Number of O atoms shared Expected structure
SiO4–
4 (1)(+ 4) + (-4) Discrete
=0
1
Si2O6–
7 (2)(+ 4) + (-6) Island structure
=1
2
(SiO3)2n–
n n(+4) + (-2n) Chain or cyclic type
=2
n
(Si2O5)2n–
n (2n)(+4) + (-2n) Two dimension sheet
=3
( 2n)
SiO2 1(+4) + 0 Three dimension lattice
=4
1

Silicones Silicones form a group of organosilicon polymers of wide commercial use. They have general formula
(R2SiO)n, where R may be methyl, ethyl or phenyl group.
For the straight-chain polymer, dimethyldichlorosilane (CH3)2SiCl2 is used. Its hydrolysis followed by polymerization
may be represented as follows

The above reaction continues because of active OH group at each end of the chain.

atoms.

Trimethylmonochlorosilane (CH3)3SiCl can be used to block the polymerization as it does not create the active OH
group at the end of the chain.
15.24 Complete Chemistry—JEE Main

The hydrolysis of methyltrichlorosilane RSiCl3 produces a very complex cross-linked polymer.


The durability and inertness of silicones is due to the silica-like arrangement Si—O—Si—O—Si which involves
very strong Si—O bonds with bond enthalpy 502 kJ mol–1. The Si—C bond is also very strong.

are excellent electrical insulators. Silicon resins are used in paints and varnishes.
4. Carbides
These are prepared by direct combination of metals with carbon at elevated temperatures or indirectly, the heating of

Ionic carbides are formed by metals of Group 1, 2 and 13. These compounds, in general occur as transparent crystals
and in the solid state they are nonconductor of electric current. They give hydrocarbons when treated with water or acids.
4–
), acetylides (C2– 4–
2 ) and allylides. (C3 ).
The examples are
Be2C + 4H2O Æ 2Be(OH)2 + CH4
Al4C3 + 12H2O Æ 4Al(OH)3 + 3CH4
CaC2 + 2H2O Æ Ca(OH)2 + C2H2
Al2C6 + 6H2O Æ 2Al(OH)3 + 3C2H2
Mg2C3 + 4H2O Æ 2Mg(OH)2 + CH3C ∫ CH
Covalent carbides formed by the elements of Group 16 and 17 are discrete molecules (CH4, CO2, CS2, etc.). Some
covalent carbides are giant molecules, e.g. carborundrum (SiC) and boron carbide (B4C3). These are characterized by
high decomposition temperature, chemical inertness and extreme hardness. Carborundum has a diamond like structure.
Interstitial (or metallic) carbides are formed by the transition elements such as TiC, HfC, W2C, MoC, Mo2C, Cr3C2
and Cr4C. In these the small carbon atoms occupy interstitial position in the crystal lattices of the metals. These
compounds are characterized by hardness, chemical inertness and high electric conductivity.
5. Zeolites
In the framework where all four oxygen atoms of SiO4 tetrahedron are shared, if some of the silicon atoms are
substituted by aluminium atoms, the aluminosilicate framework (known as zeolite with the general formula)
x/n [(AlO2)x (SiO2)y] ◊ 3H2O) is formed. Here each AlO2 unit acquires one negative charge as the aluminium atom
M n+ x–

is trivalent, whereas the silicon atoms is tetravalent. To balance the electrical charge some other cations (such as
Na+, Ca2+ and Mg2+) are absorbed in voids. Zeolites are characterized by their open structures that permit the exchange
of cations and water molecules. Zeolite structures also contain tunnels or systems of interconnected cavities which have

have been used as gaseous separator, cation exchanger, desiccants and catalysts.

Allotropes of Carbon
Carbon exists in two true allotropic forms, namely, diamond and graphite.
Diamond Diamond crystallizes in the face-centred cubic system. The diamond crystal is built up of a giant three
dimensional structure with tetrahedral arrangement of carbon atoms which are at equal distance of 154 pm from one
another (Fig. 17). The forces acting between atoms are very strong. The crystal of diamond is a nonconductor of
electricity as all the electrons of carbon are held in the bonds between carbon atoms. It is the densest and purest variety
of carbon. Also, it is the hardest natural substance known. Diamond has a high refractive index (= 2.45) because of which

almost all chemical reagents. It burns in air at 900° C in oxygen at 700 °C to form carbon dioxide.
Graphite Structurally graphite consists of a two-dimensional sheet-like network in which the carbon atoms are joined
together in hexagonal rings (Fig. 18).
Study of the p-Block Elements (Groups 13, 14 and 15) 15.25

141.5 pm

154 pm

340 pm

Fig. 17 The crystal structure of diamond Fig. 18 The layer structure of graphite
The two layer of carbon network in graphite are held together by weak van der Waals forces and are about 340 pm
apart. Within the layer, each carbon atoms covalently bonded by sp2 hybrid orbitals to three cabron atoms with C—C
distance equal to 141.5 pm. Thus, three of the four valence electrons are utilized in bonding with other carbons within
the layer and the fourth electron is in the pz atomic orbital which is perpendicular to the layer and is involved in p
bonding with such electrons in the same layer. Because of the highly delocalized nature of p electrons, graphite is a good
conductor of heat and electricity. It is less dense as compared to diamond. Since the different layers are held by weak
van der Waals forces, each layer can slide over the other and this leads to the softness and the lubrication properties of
graphite
Graphite is thermodynamically more stable than diamond.
Besides graphite and diamond, there are several amorphous forms of carbon which resemble graphite in character.
These include coal, coke, wood charcoal, carbon black, etc. Coal is believed to have been formed by the slow carbonization
of vegetable matter buried underneath the earth centuries ago, in limited supply of air under high temperature and
pressure prevailing there. The different varieties of coal available are peat (60% C), lignite (70 % C), bituminous (78 %
C), semibituminous (83% C) and anthracite (90% C). The common variety of coal is bituminous. When coal is subjected
to destructive distillation by heating in the absence of air, the residue left is known as coke. Wood charcoal is obtained
by heating wood in the absence of air.
Fullerenes
Heating of graphite is an electric arc in the presence of inert gases followed by condensation of sooty material results
in the formation of fullerenes. The latter mainly consists of C60 with small quantity of C70.
The molecule C60 is known as Bukminster fullerene

membered rings. All C atoms are sp2 hybridized and occupy 60 vertices of ball-type molecule.
Anomalous Behaviour of Carbon
Carbon differs from rest of the elements of the group because of its small size, high electronegativity and nonavailability
of d-orbitals. It has the unique characteristics of forming compounds with multiple bonds such as C == C, C == O,
C ∫∫ C, C ∫∫ N, etc.
It also exhibits the property of catenation, i.e. forming chains of identical atoms. This property is related to the
strength of the C—C bond. The higher the bond enthalpy, the greater will be the tendency to form chains (C—C bond
enthalpy 348 kJ mol–1 , Si—Si bond enthalpy 222 kJ mol–1).
The covalency of carbon is limited to four because of the nonavailability of d orbitals. The other elements of the
group are capable of forming compounds in which they attain a covalency higher than four.
The melting and boiling points of carbon are exceptionally high as compared to those of silicon and other elements
of the group.

Example Which of the bond in each of the following pairs is stronger?


(a) C—C and Si—Si
(b) C—O and Si—O
15.26 Complete Chemistry—JEE Main

Answer
(a) The C—C single bond is stronger than the Si—Si bond. This is due to the smaller size of C leading to the good
overlap between the orbitals involved in the bond formation.
(b) The Si—O bond is much stronger than the C—O bond. It is due to the formation of pp—dp bond between Si
and O atoms.
Example Describe the structural aspects of trimethylamine and trisilylamine.
Answer
Trimethylamine has a pyramidal structure whereas trisilylamine has a planar structure.
In trisilylamine, (H3Si)3N, nitrogen is sp2
a empty silicon 3dxz (or 3dyz) orbital. Thus, a dative pp—dp bond is established. This linkage provides additional bond
strength in each Si—N bond. Because of this linkage the skelton NSi3
In trimethylamine, the above type of pp—dp bond is not possible as carbon does not have low lying d orbitals.

MULTIPLE CHOICE QUESTIONS

1. Which of the following elements does not belong to Group 14?


(a) Carbon (b) Silicon (c) Germanium (d) Arsenic
2. The atomic number of the element tin which belongs to 5th period is
(a) 49 (b) 50 (c) 81 (d) 82
3. Which of the following elements may be regarded as nonmetal?
(a) Carbon (b) Silicon (c) Germanium (d) Tin
4. Which of the following elements may be regarded as semi-metals?
(a) Carbon (b) Germanium (c) Tin (d) Lead
5. Which of the following elements behaves like a metal?
(a) Carbon (b) Silicon (c) Germanium (d) Tin
6. Which of the following orders regarding the melting points is correct?
(a) C > Si > Ge > Sn (b) C < Si < Ge < Sn (c) C > Si < Ge < Sn (d) C < Si > Ge > Sn
7. Which of the following orders regarding the boiling point is correct?
(a) Si > Ge > Sn (b) Si < Ge < Sn (c) Si > Ge < Sn (d) Si < Ge > Sn
8. Which of the following statements is not correct?
(a) The stability of the +4 oxidation state of elements of Group 14 decreases down the group
(b) The stability of the +2 oxidation state of elements of Group 14 increases down the group

(d) A majority of the compounds of elements of Group 14 are four covalent and involve sp3 hybridization
9. Which of the following statements is not correct?
(a) All the elements of Group 14 form covalent hydrides
(b) Carbon forms a very large number of compounds with hydrogen
(c) Silicon like carbon forms a large number of compounds with hydrogen. These are collectively known as silanes
(d) The general formula of silanes is SinH2n+2 and these are strong reducing agents
10. Which of the following statements is not correct?
(a) All the elements of Group 14 form tetrahalides with the exception of PbI4
(b) All tetrahadies of elements of Group 14 are tetrahedral
(c) The stability of the halides of elements of Group 14 increases down the group
(d) SiCl4 exhibits the phenomenon of hydrolysis
11. Which of the following oxides is highly poisonous?
(a) CO (b) CO2 (c) C3O2 (d) C5O2
Study of the p-Block Elements (Groups 13, 14 and 15) 15.27

12. Which of the following statements is not true?


(a) Carbon produces carbon monoxide when it is burnt in a limited supply of air
(b) Carbon monoxide is a good reducing agent
(c) Carbon monoxide acts as a ligand through the donation of lone pair of oxygen
(d) Carbon monoxide acts as a ligand through the donation of lone pair of carbon
13. Which of the following statements is not correct?
(a) Carbon dioxide is a linear molecule with no dipole moment
(b) Carbon dioxide and silicon dioxide have the same structures
(c) Carbon dioxide is a gas whereas silicon dioxide is solid
(d) In carbon dioxide, a double bond exists in carbon and oxygen whereas in silicon dioxide each silicon-oxygen
bond is linked through a single sigma bond
14. Which of the following oxides is an acidic oxide?
(a) CO2 (b) Ge3O2 (c) SnO3 (d) PbO2
15. Which of the following oxides is an amphoteric oxide?
(a) CO2 (b) SiO2 (c) GeO2 (d) PbO2
16. Which of the following oxides is a basic oxide?
(a) CO2 (b) SiO2 (c) GeO2 (d) PbO2
17. Which of the following statements is not correct?
(a) Organosilicon polymers are known as silicones
(b) Silicones have the general formula (R2SiO)n where R = – CH3, – C2H5, – C6H5, etc.
(c) Hydrolysis of dialkyldichlorosilane produces cross-linked silicon polymer
(d) Hydrolysis of alkyltrichlorosilane produces cross-linked silicon polymer
18. Which of the following statements is not correct?
(a) The durability and inertness of silicones is due to the high bond enthalpy of Si —O bond

(c) Silicon rubbers are excellent electrical insulators


(d) The silicones always involve cross-linked between Si and O atoms
19. Carborundum is
(a) CaC2 (b) Fe3C (c) CaCO3 (d) SiC
20. Which of the following is expected to be conducting?
(a) SiO2 (b) Graphite (c) Diamond (d) Methane
21. Which of the following compounds of carbon may be considered a resonance hybrid?
(a) C2H5OH (b) C6H6 (c) C4H10 (d) C2H2
22. Iron may be rendered passive by application of
(a) PbO (b) Pb(OH)2 (c) PbCO3 (d) Pb3O4
23. The dissolution of stannous hydroxide in excess base produces
(a) Sn(OH)4 (b) Sn(OH)62– (c) SnO32– (d) Sn(OH)–3
24. Synthetic zeolites have been prepared for use as
(a) lubricants (b) molecular sieves (c) semiconductors (d) plastics
25. Which of the following statements is correct?
(a) Graphite is a bad conductor of electricity
(b) Dry graphite in vacuum is not slippery
(c) The adsorption of substances by graphite increases the friction as the layers slide past each other
(d) The graphite does not possess metallic properties
26. Which of the following statements is not correct regarding graphite crystal?
(a) The carbon atoms are arranged in planar layers
(b) Each carbon in graphite is sp2 hybridised
(c) The atoms in each layer is not tightly bonded together
(d) The binding force between layers is weak allowing the layers to slip over each other
15.28 Complete Chemistry—JEE Main

27. Which of the following carbides is an ionic carbide?


(a) SiC (b) B4C (c) TiC (d) CaC2
28. Which of the following is white lead?
(a) Pb(OH)2 · 2PbCO3 (b) Pb(OH)2 · Pb(CH3COO)2
(c) Pb(OH)2 (d) PbCO3
29. Which of the following is not hydrolysed?
(a) CCl4 (b) SiCl4 (c) GeCl4 (d) SnCl4
30. Which of the following order regarding bond energies is correct?
(a) e(C—H) > e(Si—H) > e(Ge—H) (b) e(C—H) < e(Si—H) < e(Ge—H)
(c) e(C—H) > e(Si—H) < e(Ge—H) (d) e(C—H) < e(Si—H) > e(Ge—H)
31. Tin exists in
(a) one allotropic form (b) two allotropic forms (c) three allotropic forms (d) four allotropic forms
32. Which of the following is red lead?
(a) PbO (b) Pb3O4 (c) Pb2O (d) PbO2
33. Which of the following represents producer gas?
(a) CO and N2 (b) CO2 and H2 (c) CO and H2 (d) CO2 and N2
34. Silicones contain
(a) C, O and Si (b) C and Si (c) O and Si (d) C, N, O and Si
35. Silanes are compounds which contain
(a) C, H and Si (b) C and Si (c) O and Si (d) H and Si
36. Which of the following statements is not correct?
(a) Silicon is the second member of Group 14
(b) Silicon tetrahalide exhibits the phenomenon of hydrolysis
(c) Because of large radius of silicon atom, the latter does not form the bonds of the type Si == Si, Si Si
(d) Silicon dioxide molecules in solid phase are held together by van der Waals forces
37. Which of the following statements is not correct?
(a) Silicon is extensively used as a semiconductor (b) Carborundum is SiC
(c) Silicon occurs in free state in nature (d) Mica contains the element silicon
38. Which of the following represents pyrosilicate ion?
(a) SiO4–4 (b) Si2O6–7 (c) (SiO8)21–
n (d) Si3O6–
9
39. Only one oxygen atom of two SiO4 tetrahedra is shared in
(a) orthosilicate (b) pyrosilicates (c) chain silicates (d) cyclic silicates
40. In chain and cyclic structures of silicates
(a) no oxygen atom is shared amongst SiO4 tetrahedra (b) one oxygen atom per tetrahedron is shared
(c) two oxygen atoms per tetrahedron are shared (d) three oxygen atoms per tetrahedron are shared
41. In the sheet type silicates
(a) no oxygen is shared amongst SiO4 tetrahedra (b) one oxygen atom per tetrahedron is shared
(c) two oxygen atoms per tetrahedron are shared (d) three oxygen atoms per tetrahedron are shared
42. In the structure of quartz,
(a) no oxygen atom is shared amongst SiO4 tetrahedra
(b) one oxygen atom is shared between two tetrahedra
(c) two oxygen atoms per tetrahedron are shared
(d) all the four oxygen atoms of a SiO4 tetrahedron are shared with other tetrahedra
43. Which of the following statements regarding glass is not true?

(b) Ordinary glass is a mixture of sodium and calcium silicates and may be represented as Na2SiO3 · CaSiO3 · 4SiO2
(c) Small amounts of Co(II), Cr(III), Fe(III) and Mn(IV) compounds impart respectively blue, green, brown and
violet colours to the glass
(d) Flint glass has a low refractive index
Study of the p-Block Elements (Groups 13, 14 and 15) 15.29

44. Which of the following statements is correct?


(a) All compounds of silicon are collectively known as silicones
(b) Silicon hydrides are known as silicones
(c) Silicon halides are known as silicones
(d) Silicones are polymeric organosilicon compounds
45. Which of the following statements regarding silicones is not correct?
(a) Silicones have good thermal stability
(b) Liquid silicones are lubricants
(c) Silicones are chemical inert substances
(d) Silicone rubber is not attack by ordinary chemical reagents except ozone
46. Lead dioxide is a/an
(a) neutral oxide (b) acidic oxide (c) basic oxide (d) amphoteric oxide
47. Plumbo-solvancy means dissolution of lead in
(a) water (b) acids (c) alkalis (d) organic solvents
48. Red lead has the chemical formula
(a) PbO (b) PbO2 (c) Pb2O3 (d) Pb3O4
49. Which of the following is not correct?
(a) Lead is a true metal with + 2 electrovalency
(b) Lead forms PbCl4 which is soluble in organic solvents
(c) Lead reacts with concentrated HCl to form PbCl2
(d) Lead reacts with NaOH solution to form Pb(OH)4
50. Which of the following is known as butter of tin?
(a) SnCl2·2H2O (b) SnCl2·3H2O (c) SnCl2· 4H2O (d) SnCl2·5H2O
51. Which of the following halides does not exist?
(a) PbF4 (b) PbCl4 (c) PbBr4 (d) PbI4
52. Which of the following halides is of yellow colour?
(a) PbCl2 (b) PbBr2 (c) PbI2 (d) PbF2
53. The element tin and lead belong to Group
(a) 11 (b) 12 (c) 13 (d) 14
54. Which is the chief ore of tin?
(a) Cerrusite (b) Anglesite (c) Cassiterite (d) Cinnabar
55. The chief ore of lead is
(a) galena (b) cinnabar (c) cassiterite (d) zincite

(a) Crude lead acts as anode (b) Pure lead acts as cathode
(c) Electrolytic solution contains PbSiF6 and H2SiF6 (d) Anode mud contains copper only
57. Heating lead in air at higher temperature (T 725 K), the oxide formed is
(a) PbO (b) PbO2 (c) Pb2O3 (d) Pb3O4
58. Which of the following equations is not actually observed?
(a) 4Sn + 10HNO3 4Sn(NO3)2 + NH4NO3 + 3H2O
(dilute)
(b) 3Sn + 8HNO3 3Sn(NO3)2 + 2NO + 4H2O
(dilute)
(c) Sn + 2HCl SnCl2 + H2
(conc.)
(d) Sn + 2H2SO4 SnSO4 + 2H2O + SO2
(conc.)
15.30 Complete Chemistry—JEE Main

59. Which of the following equations is not actually observed?


(a) 4Pb + 10HNO3 4Pb(NO3)2 + NH4NO3 + 3H2O
(dilute)
(b) 3Pb + 8HNO3 3Pb(NO3)2 + 2NO + 4H2O
(dilute)
(c) PbCl2 + 2HCl H2PbCl4
(conc.)
(d) Pb + 4HNO3 Pb(NO3)2 + 2NO2 + 2H2O
(conc.)
60. Which of the following statements is not correct?
(a) Divalent tin or lead compounds are essentially ionic in nature while tetravalent are generally covalent
(b) Tin(II) compounds are invariably reducing agents
(c) Lead(II) compounds are more stable than Pb(IV) compounds
(d) (C2H5)2Pb acts as an antiknocking compound in petrol
61. Tin dioxide is a/an
(a) neutral oxide (b) acidic oxide (c) basic oxide (d) amphoteric oxide
62. Which of the following is massicot?
(a) PbO (b) PbO2 (c) SnO (d) SnO2
63. Lead oxide is a/an
(a) neutral oxide (b) acidic oxide (c) basic oxide (d) amphoteric oxide
64. Which of the following halides will have minimum ionic character?
(a) PbF2 (b) PbCl2 (c) PbBr2 (d) PbI2
65. Which of the following statements is not correct?
(a) Lead(II) chloride and lead(II) iodide are soluble in hot water but crystallize out on cooling
(b) Lead(IV) chloride cannot be separated from the solution as it readily decomposes to lead(II) chloride and
chlorine
(c) Lead(IV) chloride is not readily hydrolysed by water
(d) Per cent of lead(II) halides decreases with the increase in atomic number of the halogen
66. Which of the following statements is not correct?
(a) Tin(IV) chloride is an ionic compound
(b) Tin(IV) chloride undergoes hydrolysis with water
(c) With excess of hydrochloric acid, tin(IV) chloride forms hexachlorostannic acid (H2SnCl6)
(d) Tin (II) chloride can reduce Fe(III) to Fe(II), Cu(II) to Cu(I), Hg(II) to Hg(0) and Au(III) to Au(0)
67. The only stable tetrahalide of lead is
(a) PbF4 (b) PbCl4 (c) PbBr4 (d) PbI4
68. Which of the following oxides is used in lead accumulators?
(a) PbO (b) Pb2O3 (c) PbO2 (d) Pb3O4
69. Which of the following elements does not show allotropy?
(a) Carbon (b) Silicon (c) Sulphur (d) Lead
70. Mosaic gold is
(a) SnS (b) PbS (c) SnS2 (d) CdS
71. Brown SnO is
(a) an acidic oxide (b) basic oxide (c) an amphoteric oxide (d) neutral oxide
72. Lead sesquioxide is
(a) PbO (b) Pb2O3 (c) PbO2 (d) Pb3O4
73. Galena is an ore of
(a) Fe (b) Cu (c) Zn (d) Pb
74. Desilverization of lead is known as
(a) Cyanide process (b) Parke’s process
(c) MacArthur-Forest process (d) Parting process
Study of the p-Block Elements (Groups 13, 14 and 15) 15.31

75. Which of the following statements is not correct?


(a) Zeolite contains aluminosilicate framework
(b) The general formula of zeolite is Mn+ x–
x/n [(AlO2)x (SiO2)y] · z H2O
(c) Zeolites are characterized by their open structures that permit the exchange of anions and water molecules
(d) Sodalite cage is formed by linking 24 SiO4 tetrahedra
76. Which of the following statements is not correct?
(a) Zeolite A is formed by linking sodalite cages through double four-membered rings
(b) Faujasite zeolite is formed by linking the sodalite cages through double six-membered rings

dimensions on the atomic scales


(d) Zeolites are anion exchanger
77. Bond energy is highest amongst
(a) C — C (b) Si — Si (c) Ge — Ge (d) Sn — Sn
78. Lead pencil contains
(a) PbS (b) PbO (c) FeS (d) graphite
79. Amongst the following element, which one has the highest melting point?
(a) Ge (b) Sn (c) Pb (d) Na
80. Hydrolysis of SiF4 in alkaline medium produces
(a) SiO2 (b) H2SiO4 and H2SiF6 (c) H2SiF6 (d) H2SiO4 and SiO2

(a) Water gas (b) Coal gas (c) Producer gas (d) Semi-water gas
82. Which of the following compounds of lead is the most stable?
(a) PbF4 (b) PbCl4 (c) PbBr4 (d) PbI4
83. Which of the following is a methanide?
(a) Be2C (b) CaC2 (c) Mg2C3 (d) TiC
84. Which of the following is an acetylide?
(a) Be2C (b) Al4C3 (c) CaC2 (d) Mg2C3
85. Which of the following is an allylide?
(a) Be2C (b) Al4C3 (c) CaC2 (d) Mg2C3
86. Which of the following bonds has the highest bond energy?
(a) Si — Si (b) Si = Si (c) Si – O (d) Si = O
87. Which of the following statements is correct?
(a) Si6Cl14 exists as a cyclic chlorosilane
+2
(b) Pyrolysis of SiCl4 at 1150 °C in the presence of Si produces members of the series SinCl2n
(c) Si — F bond is a weaker bond than C — F bond
(d) Si — Si bond is a stronger bond than C — C bond
88. Which of the following elements is not the constituent of zeolite A.
(a) Al (b) Si (c) O (d) Mg
89. Each silicon atom in the trimer cyclic silicate has
(a) one terminal oxygen and three bridging oxygens
(b) two terminal oxygens and two bridging oxygens
(c) three terminal oxygens and one bridging oxygen
(d) all the four oxygens as the bridging oxygen
90. Which one of the following is a covalent carbide?
(a) Mg2C (b) Al4C3 (c) Li3C2 (d) SiC
x
91. The charge x of Si6O 18 will be
(a) 4 – (b) 6 – (c) 8 – (d) 12 –
92. The oxidation state of Sc in Sc2Si2O7 is
(a) +2 (b) +3 (c) +4 (d) +6
15.32 Complete Chemistry—JEE Main

ANSWERS
1. (d) 2. (b) 3. (a) 4. (b) 5. (d) 6. (c)
7. (a) 8. (c) 9. (c) 10. (c) 11. (a) 12. (c)
13. (b) 14. (a) 15. (c) 16. (d) 17. (c) 18. (d)
19. (d) 20. (b) 21. (b) 22. (d) 23. (d) 24. (b)
25. (b) 26. (c) 27. (d) 28. (a) 29. (a) 30. (a)
31. (c) 32. (b) 33. (a) 34. (a) 35. (d) 36. (d)
37. (c) 38. (b) 39. (b) 40. (c) 41. (c) 42. (d)
43. (d) 44. (d) 45. (d) 46. (d) 47. (a) 48. (d)
49. (d) 50. (d) 51. (d) 52. (c) 53. (d) 54. (c)
55. (a) 56. (d) 57. (d) 58. (b) 59. (a) 60. (d)
61. (d) 62. (a) 63. (d) 64. (d) 65. (c) 66. (a)
67. (a) 68. (c) 69. (d) 70. (c) 71. (a) 72. (b)
73. (d) 74. (b) 75. (c) 76. (d) 77. (a) 78. (d)
79. (a) 80. (b) 81. (a) 82. (a) 83. (a) 84. (c)
85. (d) 86. (c) 87. (b) 88. (d) 89. (b) 90. (d)
91. (d) 92. (b)

HINTS AND SOLUTIONS


6. Because of catenation of carbon, its melting point is greater than silicon.
7. Boiling point decreases from Si to Pb.

9. Silicon forms a limited number of saturated hydrides.


10. The stability of the halides of elements of Group 14 decreases down the group.
12. Carbon monoxide donates a lone pair of carbon.
13. Carbon dioxide exists as individual molecule whereas silicon dioxide exists as giant molecule containing a
network of C and O bonds.
14. Acidic nature of the oxides decreases down the group.
17. The hydrolysis of dialkyldichlorosilane produces linear silicon.
R R R
H O
n R2SiCl2 æææ
2
Æ O Si O Si O Si O
- HCl
R R R

23. Sn(OH)2 + OH Sn(OH)3–
36. Each silicon atom in SiO2 is bounded to four oxygen atoms tetrahedrally. Each oxygen atom is shared by two
silicon atoms.

43. Flint glass is a lead-potash-lime glass and it has high refractive index.
49. Pb + 2OH – æÆ PbO22 – + H2
plumbate
54. Cerrusite is PbCO3. Anglesite is PbSO4. Cainnabar is HgS. Cassiterite is SnO2.
55. Galena is PbS.
56. Anode mud contains many other metals such as Ag, Au, Sn, As and Sb.
58. 4Sn + 10HNO3 4Sn(NO3)2 + NH4NO3 + 3H2O
(dilute)
59. Choice (b) is observed, i.e. 3Pb + 8HNO3 3Pb(NO3)2 + 2NO + 4H2O
(dilute)
Study of the p-Block Elements (Groups 13, 14 and 15) 15.33

60. (C2H5)4Pb acts as an antiknocking compound in petrol.


65. Lead(IV) chloride gets rapidly hydrolyzed by water. PbCl4 + 2H2O PbO2 + 4HCl
66. SnCl4 is a covalent compound. It is soluble in organic solvents.
67. PbF4 is yellow compound with melting point 873 K.
PbCl4 is yellow oil stable below 273 K and decomposes to PbCl2 and Cl2 above 323 K.
PbBr4 is even less stable. PbI4 does not exist.
77. C—C bond has the highest bond energy. As the size of atoms becomes larger, the bond energy decreases.
78. Lead pencil contains graphite
79. Melting point decreases on descending a group.
80. SiF4 + 4H2O Æ Si(OH)4 + 4HF and SiF4 + 2HF Æ2H+ + [SiF6]2–
81. Water gas is a mixture of CO and H2
both CO and H2 burn and evolve heat.
82. Stability decreases with increasing atomic number of halogen.
83. Be2C is a methanide as it produces CH4 on reacting with H2O.
84. CaC2 is acetylide as it produces actylene on reacting with H2O.
85. Mg2C3 is allylide as it produces propyne on reacting with H2O.
86. Amongst the given bonds, Si—O has the highest bond energy.
87. SiCl4 + SiÆSi2Cl6 + higher members of the series 5Si2Cl2+Si6Cl14+4SiCl4
88. Zeolites are the three-dimensional silicates and contain Na, Al and Si elements.
89. Trimeric cyclic silicate is

90. SiC is a covalent carbide, rest are ionic carbides.


x
91. Si6O 18 ∫ (SiO4–
4 )6 – 6O ∫ Si6O 18
2– 12 –

92. The charge of Si2O7x is Si2O 7x ∫ (SiO4–


4 )2 – O ∫ Si2O 7 . Hence,
2– 6–
the charge on Sc is (6+)/2 = 3+
Alternatively, 2x + 2(4+) + 7(–2) = 0 fi x = 3+

MULTIPLE CHOICE QUESTIONS FROM AIEEE AND JEE MAIN

graphite
(a) has carbon atoms arranged in a large plates of rings of strongly bound carbon atoms with weak interplate bond
(b) is a non-crystalline solid
(c) is a allotropic form of diamond
(d) has molecules of variable molecules masses like polymers [2003]
2. In curing cement plasters water is sprinkled from time to time. This helps in
(a) converting sand into silicic acid
(b) keeping it cool
(c) developing interlocking needle-like crystals of hydrated silicates
(d) hydrating sand and gravel mixed with cement [2003]
3. The soldiers of Napolean army while at Alps during freezing winter suffered a serious problem as regards to the tin
buttons of their uniforms. White metallic tin buttons got converted to grey powder. This transformation is related
to
15.34 Complete Chemistry—JEE Main

(a) an interaction with nitrogen of the air at very low temperatures


(b) a change in the crystalline structure of tin
(c) a change in the partial pressure of oxygen in the air
(d) an interaction with water vapour contained in the humid air. [2004]
4. In silicon dioxide
(a) each silicon atom is surrounded by four oxygen atoms and each oxygen atom is bonded to two silicon atoms
(b) each silicon atom is surrounded by two oxygen atoms and each oxygen atom is bonded to two silicon atoms
(c) silicon atoms are bonded to two oxygen atoms
(d) there are double bonds between silicon and oxygen atoms [2005]
5. A metal, M forms chlorides in its + 2 and + 4 oxidation states. Which of the following statements about these
chlorides is correct?
(a) MCl2 is more easily hydrolysed than MCl4
(b) MCl2 is more volatile than MCl4
(c) MCl2 is more soluble in anhydrous ethanol than MCl4
(d) MCl2 is more ionic than MCl4 [2006]
6. Among the following substituted silanes the one which will give rise to cross linked silicone polymer on hydrolysis
is
(a) R3SiCl (b) R4Si (c) RSiCl3 (d) R2SiCl2 [2008]
7. Which of the following exists as covalent crystals in the solid state?
(a) Iodine (b) Silicon (c) Sulphur (d) Phosphorus [2013]
8. Global warming is due to increase of:
(a) methane and nitrous oxide in atmosphere (b) methane and CO2 in atmosphere
(c) methane and O3 in atmosphere (d) methane and CO in atmosphere [2014, online]
9. Example of a three-dimensional silicate is:
(a) Zeolites (b) Ultramarines (c) Feldspars (d) Beryls [2014, online]
10. Match the items in column I with its main use listed in column II
Column I Column II
(A) silica gel (i) Transistor
(B) Silicon (ii) Ion-exchanger
(C) Silicone (iii) Drying agent
(D) Silicate (iv) Sealant
(a) (A)-(iii), (B)-(i), (C)-(iv), (D)-(ii) (b) (A)-(iv), (B)-(i), (C)-(ii), (D)-(iii)
(c) (A)-(ii), (B)-(i), (C)-(iv), (D)-(iii) (d) (A)-(ii), (B)-(iv), (C)-(i), (D)-(iii) [2016 online]

11. Assertion: Among the carbon allotropes, diamond is an insulator, whereas, graphite is a good conductor of
electricity
Reason: Hybridization of carbon in diamond and graphite are sp3 and sp2, respectively
(a) Both assertion and reason are correct, but the reason is not the correct explanation for the assertion.
(b) Both assertion and reason are correct, but the reason is the correct explanation for the assertion
(c) Both assertion and reason are incorrect
(d) Assertion is incorrect statement, but the reason is correct [2016 online]

ANSWERS
1. (c) 2. (c) 3. (b) 4. (a) 5. (d) 6. (c)
7. (b) 8. (b) 9. (d) 10. (a) 11. (b)
Study of the p-Block Elements (Groups 13, 14 and 15) 15.35

HINTS AND SOLUTIONS

3. At low temperature, tin undergoes a change in the crystalline structure; Grey Sn 291 K white Sn.
Grey tin is easily changed into a powder. Therefore, the choice b is correct.

O O
4. The structure of SiO2 is O Si O Si O
O O

5. The salt of lower oxidation state is ionic while that of higher oxidation state is covalent.
6. The hydrolysis of RSiCl3 produces a very complex cross-linked polymer.
The hydrolysis of R2SiCl2 produces a straight-chain polymer.
The compound R3SiCl is used to block the polymerization as it does not create the active OH group at the end of
the chain.
7. Iodine, sulphur and phosphorus exist as I2, S6 and P4, respectively. Silicon does exist as covalent crystal in the
solid state. Thus the choice (b) is correct.
8. Global warming is due to increase in the concentration of methane and carbon dioxide in atmosphere.
9. The replacement of Si4+ ions in the three-dimensional silicates (of general formula SiO2) by a combination of Al3+
and other cations (to maintain electrical neutrality) gives minerals known as feldspars, zeolites and ultramarines.
10. Silica gel is used as drying agent Silicon is used in transistor
Silicone is used as sealant Silicate is used in ion-exchanger
Therefore, the Choice (a) is the correct matching.
11. The choice (b) is correct.

The Group 15 Elements


Group 15 of the periodic table consists of elements nitrogen (N), phosphorus (P), arsenic (As), antimony (Sb) and
bismuth (Bi). The physical properties of these elements are recorded in Table 3.
Table 3 Atomic and Physical Properties of Group 15 Elements
Property N P As Sb Bi
[He](2s) (2p) [Ne](3s) (3p) [Ar](3d) (4s) (4p) [Kr](4d) (5s) (5p) [Xe](4f)14(5d)10(6s)2(6p)3
2 3 2 3 10 2 3 10 2 3

Atomic number 7 15 33 51 83
Relative atomic mass 14.01 30.97 74.92 121.75 208.98
Covalent radius, r/pma 74 110 120 140 152
Ionic radius M3+, r/pmb — 44 58 76 103
M5+, r/pmb — 38 46 60 76
Melting point, T/K 63 317.1c 1089d 903.7 544.4
Boiling point, T/K 77.2 553.5 888 1860 1837
e f
Density at 298 K, r/g cm–3 0.879 1.823 5.778 6.697 9.808
Ionization energy, I/kJ mol–1
I 1402 1012 947 834 703
II 2856 1903 1798 1595 1610
III 4577 2910 2736 2443 2466
–1
Sum (I + II + III)/kJ mol 8835 5825 5481 4872 4779
Sum (IV + V)/kJ mol–1 16920 11220 10880 9636 9776
Electronegativity 3.0 2.1 2.0 1.9 1.9

(a) tervalent, single bond, (b) 6-coordinate, (c) white form, (d) grey a-form at 38.6 atm, (e) at 63 K, (f) grey a-form.
15.36 Complete Chemistry—JEE Main

Description of Physical Properties


ns)2(np)3, where n varies from 2 to 6.

electronegativity and nonavailability of d orbitals.


3. The metallic character increases on descending the group.
N and P are nonmetals. As and Sb are metalloids. Bi is a true metal
4. The covalent radii increase on descending the group (Fig. 12). The variation is according to the expectation.
From phosphorus onwards, the difference becomes small due to poor shielding of nuclear charge by 3d and 4f
electrons.
The elements do not form M5+

5. The melting point increases from N to P (Fig. 20). This is attributed to their structures. Nitrogen
exists as diatomic molecule due to the multiple bond formation. Phosphorus exists in different allotropic forms,
with white phosphorus existing as P4.

participate in metallic bonding.

Fig. 19 Covalent Radii of the Group 15 Elements


6. The boiling points of elements increase down the group with the exception of the last member of the
group (Fig. 21).

Fig. 20 Melting Points of the Elements of Group 15 Elements Fig. 21 Boiling Points of the Group 15 Elements
Study of the p-Block Elements (Groups 13, 14 and 15) 15.37

7. Ionization energies of elements generally decrease down the group.


high and they
decrease down the group. The elements Sb and Bi comparatively have low values, so these elements form Sb3+
and Bi3+ in their salts with high electronegative element F.
5+
ions.
8. The densities of elements increase on descending the group.
Reactivity of Elements
Nitrogen exists as diatomic molecules. It is a gas at room temperature and is relatively unreactive.
The common allotropic forms of phosphorus are white and red. White phosphorus is soft, waxy and reactive.
It is stored under water as it ignites spontaneously in air at about 35 °C to give P4O10. It exists as tetrahedral P4
molecules.
The bond angle P—P—P is 60°, much smaller than the value expected for a tetrahedron
structure. This indicates the highly strained structure which leads to the instability and hence
reactivity of the molecule.
Red phosphorus is formed when white phosphorus is heated to a high temperature. It is a
polymeric solid which is less reactive then white phosphorus.
Heating white phosphorus under high pressure results into a highly polymerized form of P
called black phosphorus. It is the most stable form of phosphorus in which atoms are arranged
in corrugated planes.

The formation of diatomic nitrogen is due to the fact the bond enthalpy e(N ∫∫ N) is greater than the bond enthalpy
e(N—N) taken thrice. For phosphorus, the reverse is true, i.e. e(P ∫∫ P) < 3 e(P—P).
The reactivity of other elements decreases down the group. The tendency to form oxides with +3 oxidation state of
element decreases down the group.

Compounds of Group 15 Elements

Hydrogen Compounds
All the elements of Group 15 form volatile hydrogen compounds of the type MH3. The physical properties of these
hydrogen compounds are recorded in Table 4.
Table 4 Properties of Hydrogen Compounds of Group 15 Elements
Property NH3 PH3 AsH3 SbH3 BiH3
Melting point, T/K 195.4 139.5 156.7 185 —
Boiling point, T/K 239.7 185.5 210.6 254.6 289.8
Density, r/g cm –3
0.68 0.75 1.64 2.20
(239 K) (183 K) (209 K) (255 K)
(M—H) distance/pm 101.7 141.9 151.9 170.7 —
H—M—H angle 107.8° 93.6° 91.8° 91.3° —
Bond enthalpy/kJ mol–1 391 322 247 255 —
Enthalpy of formation,
DtH°/kJ mol–1 –46.1 –9.6 66.4 145.1 277.8
15.38 Complete Chemistry—JEE Main

Description of Properties
1. Ammonia, NH3, has exceptionally high melting 275
point. For other hydrides, the melting point of Bolling points
hydrides increases on descending the group (Fig. 250
15).
Explanation Because of the high electronegativity of 225
N, ammonia involves extensive hydrogen bondings.

Temperature /K
The other hydrides do not form hydrogen bonds. 200
2. The boiling point of NH3 is also higher than PH3.
Melting points
Beyond PH3, the boiling point increases down the 175
group (Fig. 15).
Explanation In liquid phase, ammonia is hydrogen 150
bonded due to the high electronegativity of nitrogen.
The other hydrides do not involve hydrogen bonding. 125
PH3 AsH3 SbH3
Their boiling point increases due to the increase
in the van der Waals forces which increases with
increase in size of molecule. Fig. 15 Melting and Boiling Points of Hydrides of Group 15
3. The densities recorded near the boiling point indicates
the increasing trend on descending the group.
4. The M—H bond distance increases on descending the group. This is due to increasing size and decreasing
electronegativity value of the element M.
5. The bond angle in NH3 is 107.8° indicating that sp3 hybrid orbitals of N are involved in the bondings with H
atoms.
N in ground state

N in NH3

The repulsion between a lone pair and bonded pairs is responsible for reducing bond angles from 109° 27¢ (the
expected value) to 107° 48¢.
For other hydrogen compounds, the angles are close to 90°, (and also decreases down the group) indicating the
involvement of more and more p orbitals in the bonding.
6. The bond enthalpy M—H decreases down the group indicating that the strength of bond decreases
on descending the group.
Besides the above characteristics, the following characteristics are observed on descending the group.

2. The thermal stability of hydrogen compounds decreases.


3. The reducing property of hydrogen compounds increases.
4. The replacement of H by other groups such as Cl or CH3
5. The ability to form complexes by donating the lone pair of electrons decreases.
6. The basic character decreases.
The above characteristics are explainable on the basis of bond enthalpy (which decreases down the group) and size
of the atom (which increases down the group). The basic character is due to lone pair which becomes more diffuse on
descending the group causing a decrease in basic nature.
Ammonia Ammonia is a colourless and pungent smelling gas. Its molecule is trigonal pyramidal
with nitrogen at the apex and is highly polar.
In laboratory NH3 may be produced on treating ammonium salt with NaOH or Ca(OH)2:
NH4Cl + NaOH æDæ
Æ NH3 + H2O + NaCl
In industry, NH3 is prepared by Haber’s process:

N2(g) + 3H2(g)   2NH3(g) DfH° = – 46. kJ mol–1
Study of the p-Block Elements (Groups 13, 14 and 15) 15.39

The condition used are about 200 atm and 700 K with a suitable catalyst (iron oxide with small amount of K2O and
Al2O3, earlier iron with promoter molybdenum).
Because of its ability to form hydrogen bond, it is highly soluble in water (53.1 g NH3 dissolves in 100 g water at
20 °C and 1 atm pressure).
Ammonia dissolved in water behaves as a weak base.

NH3 + H2O  +
 NH 4 + OH

Kb = 1.8 ¥ 10–5 mol dm–3
Ammonia forms salts with acids. In aqueous solution, ammonium salts are acidic due to the hydrolysis of NH+4 ions.
Ammonium salts decompose on heating. The decomposition reaction depends on the nature of anion.
(a) If the anion is not oxidizing such as CI– , CO2– 2–
3 or SO 4 .
heat
NH4Cl ⎯⎯→ NH3 + HC1 (NH4)2SO4 ⎯heat
⎯→ 2NH3 + H2SO4
(b) If the anion is oxidizing such as NO–2,NO–3 ClO–4 ,Cr2O2–
7
The salt decomposes to N2 or N2O.
NH4NO2 ⎯heat
⎯→ N2 + 2H2O NH4NO3 ⎯heat
⎯→ N2O + 2H2O (NH4)2Cr2O7 ⎯heat
⎯→ N2 + 4H2O + Cr2O3
Ammonia is used in the manufacture of nitric acid, various nitrogenous fertilizers and other compounds. It is also

nonaqueous solvent. Its self ionization is


2NH3   +
 NH 4 + NH 2

A substance producing NH+4 ion in liquid ammonia acts as an acid and that producing NH2– ion acts as a base.
Liquid ammonia is an extremely good solvent for the alkali metals and the heavier metals Ca, Sr and Ba of Group 2.
These solutions have conductivity comparable to that of pure metals. The solution of these metals are good reducing
agents due to the presence of free electrons.
Na ⎯liquid
⎯⎯⎯ NH3
→ [Na(NH3)n]+ + e–

Hydrazine and Hydroxylamine Nitrogen also forms hydrazine (N2H4) and hydroxylamine (NH2OH). The oxidation
state of nitrogen in these compounds are –2 and –1, respectively.
Hydrazine is manufactured by the Raschig process, in which ammonia is oxidized by sodium hypochlorite in dilute
aqueous solution.
NH3 + NaOCl Æ NH2Cl + NaOH (fast)
2NH3 + NH2C1 Æ NH2NH2 + NH4Cl (slow)
The side reactions occurring are
(i) N2H4 + 2NH2Cl Æ N2 + 2NH4Cl
(ii) 3NH2Cl + 2NH3 Æ N2 + 3NH4Cl
The reaction (i) is catalyzed by heavy metal ions in solution. For this reason, distilled water along with glue or gelatin
(which mask the metal) is used in this reaction. The reaction (ii) is minimized by using dilute solution.
The structure of hydrazine is

situated tetrahedrally with respect to N atom. The bond length N—N is 145 pm. The N—N bond is quite weak as
compared to the C—C bond in C2H6. This results due to the repulsion between nonbonding lone pairs. Due to this, nitrogen
has little tendency to catenation.
Hydrazine is a powerful reducing agent and is used to prevent the oxidation of the boiler and pipes. The compound and
its derivatives are also used as rockel fuel.
Hydroxylamine is manufactured by reducing nitrites or from nitromethane.
NH4NO2 + NH4HSO3 + SO2 + 2H2O Æ [NH3OH]+ HSO–4 + (NH4)2SO4
CH3NO2 + H2SO4 Æ [NH3OH]+ HSO–4 + CO.
15.40 Complete Chemistry—JEE Main

Hydroxylamine is a weaker base than ammonia or hydrazine. It can acts as ligands. It undergoes addition reaction
across a double bond.
Hydroxylamine is used to obtain cyclohexanone oxime, which is an intermediate in the manufacture of nylon 6.

O N–OH
(

Hydrogen Compounds of Other Elements Phosphine (PH3), arsine (AsH3), and stibine (SbH3) are obtained by acid
hydrolysis of phosphide, arsenide and antimonide, respectively.
Zn3M2 + 6HCl Æ 3ZnCl2 + 2MH3
(M = P, As or Sb)
In the laboratory, PH3 is obtained by boiling white phosphrous with concentrated NaOH or KOH solution in an inert
atmosphere.
P4 + 3KOH + 3H2O Æ PH3 + 3KH2PO2.
The compounds MH5 for the elements of Group 15 are not known. To exhibit pentavalency, the central atom should
exhibit sp3
participate in sp3d hybridization. Such a hybridization is possible with chlorine which is highly electronegative. Thus,
the pentahalides are known.
Halides
Trihalides 3 is stable,
NC13 is explosive, NBr3 and Nl3 are known only as their unstable ammoniates NBr3◊6NH3 and NI3◊6NH3).
All the trihalides are predominately covalent with the exception of BiF3 which is ionic. All of them have a tetrahedral
structure with one position occupied by a lone pair.
All the trihalides with the exception of NF3 hydrolyze in aqueous solution, the products depend on the element.
NCl3 + 3H2O Æ NH3 + 3HOCl
PCl3 + 3H2O Æ H3PO3 + 3HCl
AsCl3 + 3H2O Æ H3AsO3 + 3HCl
SbCl3 + H2O Æ SbO+ + 2H+ + 3Cl–
BiCl3 + H2O Æ BiO+ + 2H+ + 3Cl–
NF3 reacts with H2O vapour if exposed to a spark. The reaction occurring is
2NF3 + 3H2O Æ 6HF + N2O3.
NF3 has little tendency to form complexes while PF3 shows good tendency.
The dipole moment of NF3 is very low (0.23 D).
Explanation The high electronegative F atoms attract electrons, and the sum of the three N—F bond moments partly
cancel the moment from the lone pair, and thus reduces both the dipole moment and its donor characteristics

Pentahalides The following pentahalides are known.


PF5 AsF5 SbF5 BiF5
PC15 (AsCl5) SbCl5
PBr5
PI5
Study of the p-Block Elements (Groups 13, 14 and 15) 15.41

Arsenic pentachloride is highly reactive and unstable. Nitrogen does not form pentahalides as its valence shell has
only four orbitals (2s and 2ps) and these can accommodate a maximum of eight electrons.
All the pentahalides have a trigonal bipyramidal structure resulted from the sp3d hybridization of the
central atom.

PCl5 undergoes complete hydrolysis in water.


PCl5 + 4H2O Æ H3PO4 + 5HCl
Phosphoric acid
Structures of Phosphorus Pentahalides
PF5 is covalent both in gaseous, liquid and solid states.
PC15 is covalent in gaseous and liquid states, but in solid it exists as [PCl4]+ [PCl6]–, cation has tetrahedral structure
while anion has octahedral structure.
PBr5 and PI5 exist as [PBr4]+Br– and [PI4]+I– in solid states, respectively.
The compounds PHF4 and PH2F3 have been prepared.
Oxides of Nitrogen
Nitrogen forms many oxides, exhibiting all the oxidation states from +1 to +5. The negative oxidation state is not
possible as oxygen is more electronegative than nitrogen. Nitrogen could exhibit larger number of oxidation states in its
oxides due to the possibility of pp—pp multiple bonds between N and O. The other atoms of the Group 15 (i.e. P, As, Sb
and Bi) are too large to allow such type of multiple bonding due to poor overlap between the orbitals.
A brief description of oxides of nitrogen is in order.
Nitrous Oxide (N2O) N2O is a colourless gas. It is a neutral oxide. It can be prepared by heating a solution of NH4NO3

NH4NO3 ⎯heat
⎯→ N2O + 2H2O
HCl

The oxidation state of N in N2O is +1. Its structure is


≈ ≈
N N O N N O

The bond lengths N—N is 112.6 pm and N—O is 118.6 pm. These are shorter than the expected values.
N2O is used as an anaesthetic. It is also known as laughing gas, because small amounts cause euphoria.
Nitric Oxide (NO) NO is a colourless gas. It is a neutral oxide. It can be prepared by treating Cu with dilute HNO3.
3Cu + 8HNO3 Æ 2NO + 3Cu(NO3)2 + 4H2O
(dilute)
The oxidation state of N in NO is +2. Its structure is

N O N O
The bond length N—O is 115 pm which is intermediate between a double and a triple bond.
Being an odd electron molecule (valence electrons are 11), it is paramagnetic. However in the liquid and solid states,
nitric oxide becomes diamagnetic due to dimerization O—N—N—O. The asymmetrical dimer O—N—O—N has been
isolated in the presence a of Lewis acid (say, HCl).
15.42 Complete Chemistry—JEE Main

Nitric oxide can act as a neutral ligand. The brown ring test for nitrates is due to the complex [Fe(H2O)5NO]+ ion.
The other examples of complexes are sodium nitroprusside Na2[Fe(CN)5NO] ◊ 2H2O, [Fe(CO)2(NO)2] and [Cr(NO)4].
Problem Which species NO or NO+ has a stronger bond?
Answer
s 2s)2 (s*2s)2 (p2px)2 (p2py)2 (s 2pz)2(p*2p)1. The electron in
the p* orbital is relatively easily lost to give the nitrosonium ion (NO+). Since the electron is removed from antibonding
orbital, the bond is stronger in NO+ than in NO and the bond length decreases.
Nitrogen Sesquioxide (N2O3) Nitrogen sesquioxide is stable at temperatures below –30 °C. Above this temperature,
it decomposes as
N2O3 Æ NO + NO2.
It can be prepared by mixing equimolar amounts of NO and NO2 at low temperatures. N2O3 is an acidic oxide and is
the anhydride of nitrous acid (HNO2).
N2O3 + H2O Æ 2HNO2.
The average oxidation number of N in N2O3 is +3. Nitrogen sesquioxide exists in two form, which are interconvertible.
O O
N N O N N O
O O
(asymmetric form) (symmetric form)

The bond length N—N in asymmetric form is 186.4 pm, which is much longer than the expected value (147 pm).
This indicates that the N—N bond in asymmetric form of N2O3 is weak.
Nitrogen Dioxide (NO2) Nitrogen dioxide is a brown gas. It is an acidic oxide. It can be prepared by heating lead
nitrate.
2Pb(NO3)2 Æ 2PbO + 4NO2 + O2
The action of concentrated nitric acid on Cu also produces NO2.
Cu + 4HNO3 Æ 2NO2 + Cu(NO3)2 + 2H2O
(concentrated)
The oxidation number of N in NO2 is +4. Its structure is

Being an odd electron molecule, it is paramagnetic. On cooling, paramagnetism disappears due to the dimerization of
NO2. The structure of dimerized N02 (i.e. N2O4) is

The bond length N—N is 164 pm. It is much longer than the single bond length of 147 pm in hydrazine. This indicates
that N—N bond is weak.
The dimerization of NO2 is due to the fact that the unpaired electron is localized mainly on the N atom. This may
be compared with NO and ClO2 molecules which show little tendency to dimerize, where the electron is not localized
on the single atom.
N2O4 is a mixed anhydride.
N2O4 + H2O Æ HNO2 + HNO3
Study of the p-Block Elements (Groups 13, 14 and 15) 15.43

Liquid N2O4 serves as non-aqueous solvent. It ionizes as



N2O4  + –
 NO + NO 3
acid base

Dinitrogen Pentoxide (N2O5) N2O5 is a colourless ionic solid. It is an acidic oxide. It is an anhydride of nitric acid.
H2O + N2O5 Æ 2HNO3
It can be prepared by dehydrating nitric acid with P2O5 at low temperatures.
4HNO3 + P4O10 Æ 2N2O5 + 4HPO3
The oxidation state of N in N2O5 is +5. Its structure is

In solid state, N2O5 exists NO+2NO–3 (nitronium nitrate).


Oxides of Phosphorus
The most common oxides of phosphorus are P4O6 and P4O10 and less common oxides are P4O7, P4O8 and P4O9.
P4O6 is a dimeric of phosphorus trioxide, P2O3. It is obtained when phosphorus is burnt in a limited supply of air:
P4 + 3O2 Æ P4O6
P4O10 is a dimeric of phosphorus pentoxide, P2O5. It is obtained when phosphorus is burnt in excess of air.
P4 + 5O2 Æ P4O10
The oxidation number of phosphorus in P4O6 is +3. The structure of P4O6 is
P
165.6 pm

O O 127.0°
O P
O O
P P 99.5°

O O
P
The four P atoms are at the corners of a tetrahedron, and each of the six O atoms are along each of the six edges. The
bond angle POP is 127°. The bond length P—O is 165.6 pm. Each O atom is strictly above the edges, but it is more
convenient to draw them on the edges.
P4O6 is an acidic oxide. It is an anhydride of orthophosphorous (or simply phosphorous) acid.
P4O6 + 6H2O Æ 4H3PO3
The schematic representation of hydrolysis is as follows.
OH
OH

P P P OH
OH
O O
OH O OH
OH
OH
H OH
P O P P O P P O P
OH
O O O O O O
P O P

OH OH
15.44 Complete Chemistry—JEE Main

OH OH OH OH OH OH
2 OH
H
OH P O P OH OH P O P O P O P OH

O O O
H OH
2H P O P H 4H P OH

OH OH OH
Pyrophosphorous acid Orthophosphorous acid

P4O10 is a dimer of phosphorus pentoxide, P2O5. The oxidation state of phosphorus in P4O10 is +5. Its structure is
O
143 pm
P
102°
O O 123° P
160 pm
O O
O
O P P O
O O
P

O
In addition to the structure of P4O6, each phosphorus with its lone pair forms a coordinate bond with an oxygen
atom. The bond length P—O along the edges is 160 pm and the bond length PÆO on the corners is 143 pm. The bonds
on the corners are much shorter than a single bond. In fact, these are double bonds-one p bond (which is coordinate-
covalent bond) and one is p bond formed by the back bonding of 2p electrons on oxygen atom to the empty 3d orbital
on phosphorus (i.e. it represents pp—dp back bonding).
When P4O10
P4O10 + 6H2O æÆ 4H3PO4
Tetrapolyphosphoric acid
H4P4O12 + H2O æÆ H6P4O13
Tetrapolyphosphoric acid
H6P4O3 + H2O æÆ 2H4P2O7
Pyrophosphroric acid
[H4P2O13 + H2O æÆ 2H3PO4] ¥ 2
Pyrophosphroric acid
overall reaction P4O10 + 6H2O æÆ 4H3PO4
The above reactions are shown schematically in the following.
Study of the p-Block Elements (Groups 13, 14 and 15) 15.45

O
O OH O OH
P P P

O O O O
OH O O
H OH O OH
P O OH P
O P O P O P P
O O H
O O O O HO OH O
O
P P P
O O OH O OH
not isolated Tetrametaphosphoric acid

O O O O O O O
H OH H OH
P OH P O P OH OH P O P O P O P OH
OH OH OH OH OH OH OH
Orthophosphoric acid Pyrophosphoric acid Tetrapolyphosphoric acid

P4O10 is a strong dehydrating agent and removes water from many inorganic and organic compounds.
2H2SO4 + P4O10 Æ 4HPO3 + 2SO3; 4HNO3 + P4O10 Æ 4HPO3 + 2N2O5
Oxides of Other Elements
Arsenic forms As4O6 and As4O10. Antimony forms Sb4O6 and Sb4O10. Bismuth forms only Bi2O3. It is not dimerized.
The basicity of oxides (as well as hydroxides) increases on descending the group.
N2O3 and P4O6 are acidic. As4O6 and Sb4O6 are both amphoteric. Bi2O3 is basic.
The amphoteric reaction of As4O6 are
As4O6 + 12NaOH Æ 4Na3AsO3 + 6H2O; As4O6 + 12HCl Æ 4AsCl3 + 6H2O
The stability of pentoxide decreases on descending the group. This is illustrated from the fact that Bi does not form
pentoxide. The pentoxides are more acidic than trioxides.
Oxoacids of Nitrogen
Serval oxacids of nitrogen are known (Table 5).
Table 5 Oxoaicds of nitrogen
Oxo-acid Molecular formula Description
Hyponitrous acid H 2N 2O 2 Weak acid, salts known, HON = NOH
Hyponitric acid H 2N 2O 3 Known in solution
Nitroxylic acid H 2N 2O 4 Explosive
Nitrous acid HNO2 Weak acid, unstable, salts known (called nitrites)
Nitric acid HNO3 Most stable, strong acid, stable salts known (called nitrates)
Peroxonitrous acid HOONO Isomeric with nitric acid, unstable
Peroxonitric HNO4 Unstable, HOONO2

Nitrous acid is unstable except in dilute solution. With bases, it forms nitrite salts. Both nitrous acid and nitrites are
weak oxidizing agents. In their oxidizing actions (Fe2+ to Fe3+, I– to I2), they are reduced to either N2O or NO. By strong
oxidizing agents such as MnO –4, HNO2 and NO –2 are oxidized to NO–3. Nitrous acid is used to make diazo compounds.
Nitric acid is the only important oxoacids of nitrogen. Pure nitric acid, is colourless but on exposure to light it turns
slightly brown because of slight decomposition into NO2 and O2.
4HNO3 Æ 4NO2 + O2 + 2H2O
15.46 Complete Chemistry—JEE Main

Nitric acid forms nitrate salts (eg. NaNO3, known as Chile saltpetre, and KNO3 known as Indian saltpetre). It is a
strong oxidizing agent. Aqua regia contains 25% concentrated nitric acid and 75% concentrated HCl. Gold, which is
not soluble in nitric acid, dissolves in aqua regia due to the oxidizing power of HNO3 coupled with the ability of Cl– to
form complex with the metal ion. Laboratory grade nitric acid contains about 68% of HNO3
gravity of about 1.50.
Nitric acid attacks most of metals except noble metals (gold and platinum). The products depends on temperature
and the nature of metal. For example
3Cu + 8HNO3 ææ
Æ 3Cu(NO3)2 + 2NO + 4H2O Cu + 4HNO3 ææ
Æ Cu(NO3)2 + 2NO2 + 2H2O
(dilute) (conc.)
4Zn + 10HNO3 ææ
Æ 4Zn(NO3)2 + 5H2O + N2O Zn + 4HNO3 ææ
Æ Zn(NO3)2 + 2H2O + 2NO2
(dilute) (conc.)

Oxoacids of Phosphorus
Phosphorus forms two series of oxoacids. These are phosphorous and phosphoric series. Phosphorous acids contain
P in +3 (or +1) oxidation state and are reducing agents. Phosphoric acids contain P in + 5(4-4) oxidation state and are
oxidizing agents.
In all oxoacids, P is four-coordinate and tetrahedrally surrounded by other atoms. The oxoacid containing —OH
group produce H+ on ionization and thus is acidic. The H atom directly attached to P is not ionizable and has reducing
characteristic.
Table 6 records the oxo-acids formed by phosphorus.
Table 6 Oxoacids of Phosphorus
Oxoacid Molecular formula Oxidation, state of
phosphorus
Hypophosphorous acid H3PO2 +1
Orthophosphorous acid (or simply phosphorous acid) H3PO3 +3
Hypophosphoric acid H 4P 2O 6 +4
Orthophosphoric acid (or simply phosphoric acid) H3PO4 +5
Dipolyphosphoric acid (also called pyrophosphoric acid) H 4P 2O 7 +5
Cyclotrimetaphosphoric acid (HPO3)3 +5
Polymetaphosphoric acid (HPO3)n +5

The structure of the acids recorded in Table 6 are shown in the following.
The Phosphorous Acid Series The phosphorous acid series is obtained by the hydrolysis of phosphorus trioxide.

O O OH
-2H2O
P O P OH 4H P OH
O O O
Pyrophosphorous acid Orthophosphorous acid
Both pyrophosphorous acid and orthophosphorous acid contain P in +3 oxidation state and are reducing agents
since H is directly attached to P atom. Both are dibasic acids as each contains two —OH groups. Only H attached to O
ionizes in solution.
Two other acids belonging to the phosphorous acid series are metaphosphorous acid and hypophosphorous acid.
These are obtained as follows.
25 mmHg
PH3 + O 2 ææææÆ H 2 + HPO 2
Metaphosphorous acid

H+
P4 + 3OH - + 3H 2 O Æ PH3 + 3H 2 PO 2- ææÆ PH3 + 3H3 PO 2
Hypophophorous acid
Study of the p-Block Elements (Groups 13, 14 and 15) 15.47

The above reaction may be shown schematically as follows,


P
HO H OH OH
H H P P H
OH H OH
PH3 + HO H
OH H PH3 + 3 HO P OH
P P P OH

OH
P
H

PH3 + 3H P OH
O
The metaphosphorous acid is believed to exist as a polymer—similar to metaphosphoric acid.
The oxidation state of P in hypophosphorous acid is +1. It is a monobasic acid and is strongly reducing agent.
The Phosphoric Acid Series The phosphoric acid series is obtained by the hydrolysis of phosphorus pentoxide.
Orthophosphoric Acid (H3PO4) The oxidation state of P in H3PO4 is +5. Its structure is
O

P
HO OH

OH
It is a tribasic acid as it contains three replaceable H atoms. Its stepwise ionization is as follows.
H3PO4 

+
 H + H2PO 4

K°a1 = 7.5 ¥ 10–3

H2PO 4   +
 H + HPO 4
2–
K°a2 = 6.2 ¥ 10–8
HPO 42–  
  H + PO 4
+ 3–
K°a3 = 1 ¥ 10–12
Phosphoric acid forms three series of salts with alkalis, e.g. NaH2PO4, Na2HPO4 and Na3PO4.
Dihydrogen phosphate is slightly acidic in nature. Monohydrogen phosphate is slightly basic in nature. Phosphate
is appreciably basic in solution.
Phosphoric acid is prepared by ‘the wet process’ in which phosphate rock is treated with sulphuric acid.
Ca3(PO4)2 + 3H2SO4 Æ 2H3PO4 + 3CaSO4
[(3Ca3(PO4)2) ◊ CaF2] + 10H2SO4 Æ 6H3PO4 + 10CaSO4 + 2HF.
Pure phosphoric acid is prepared by the ‘furnace process’ in which molten P is burnt to give P4O10. This is followed
by the hydrolysis of P4O10.
P4 + 5O2 Æ P4O10; P4O10 + 6H2O Æ 4H3PO4.
Another method of preparing phosphoric acid is to treat phosphorus with concentrated nitric acid.
P4 + 20HNO3 Æ 4H3PO4 + 20NO2 + 4H2O.
Phosphoric acid loses water steadily on heating.
gentle heat strong heat
2H3 PO 4 ææææ
220 ∞C
Æ H 4 P2 O7 ææææ
320 ∞ C
Æ (HPO3 )n
Orthophosphoric acid Pyrohosphoric acid Polymetaphosphoric acid

The structure of polymetaphosphoric acid is


O O O

P P P
O OH O O O OH O
Polymetaphosphoric acid
15.48 Complete Chemistry—JEE Main

Polyphosphoric Acids A large number of polyphosphoric acids are known. These consist of chains of tetrahedra
[PO4] linked through the O atoms at one or two corners. A few such acids are as follows.

O O

P P
Phosphoric acid is
HO O OH tetrabasic acid

OH OH
Pyrophosphoric acid (H4P2O7)

O O O

P P P
Tripolyphosphoric acid
HO O O OH is a pentabasic acid
OH OH OH

O O O O

P P P P Tetraphosphoric acid
HO O O O OH is a hexabasic acid
OH OH OH OH

Tetrapolyphosphoric acid is one of the intermediate acids obtained in the hydrolysis of P4O10. The predecessor of
this acid is tetrametaphosphoric acid which has a cyclic structure.
O O

HO P O P OH
Tetrametaphosphoric
O O acid is tetrabasic acid
HO P O P OH
O O
Tetrametaphosphoric acid (H4P4O12)
The immediate hydrolysis product of tetrametaphosphoric acid is pyrophosphoric acid.
The various phosphates having cyclic structures have been isolated.
O- O-
O O O P O P O
P
O O O O O O O
P P
O O O P O P O
O O O P P
-
O O O- O- O-
Dimetaphosphate ion Trimetaphosphate ion Tetrametaphosphate ion
Study of the p-Block Elements (Groups 13, 14 and 15) 15.49

Hypophosphoric Acid (H4P2O6) The structure of hypophosphoric acid is

OH OH
O P P O
OH OH
It contains one atom less than pyrophosphoric acid and the oxidation state of P is +4. It is obtained by hydrolysis and
oxidation of red phosphorus by NaOCl or yellow phosphorus by water and air.
P

H H H OH OH
HO OH
4O2
H 2 P P 2O P P O
HO H
P P OH OH OH OH
H Hypophosphoric acid
OH
P
Since no H atom directly attached to P is present, hypophosphoric acid is not a reducing agent. It is a tetrabasic acid.
It is slowly hydrozysed as P—P bond is stronger than P—O—P.
OH OH OH OH

O P P O O P H + HO P O
OH OH OH OH
Orthophosphorous acid Orthophosphoric acid

Anomalous Behaviour of Nitrogen


Factors such as the small size, high electronegativity and nonavailability of d orbitals are responsible for the anomalous
behaviour of nitrogen as compared to the rest of the elements of the group. Some of the anomalies are as follows.
1. Nitrogen is a gas and exists as a stable diatomic molecule with a triple bond. The remaining elements are solids and
do not form stable diatomic molecules. The bond enthalpy of N2 is highest (941.4 kJ mol–1)
O
2. Nitrogen form compounds with multiple bonding (like N N , N
3. Nitrogen exhibits catenation as in H2N—NH2. O
4. Nitrogen covalency does not exceed beyond four as in the NH4+ ion.
5. The hydrogen compounds of nitrogen have hydrogen bonding.
6. Except nitrogen, all elements of the group can make use of d orbitals forming compounds of the type PC15,
AsF5, etc.
7. The single N–N bond is weaker than the single P–P bond because of high interelectronic repulsion of the non-
bonding electrons owing to the small bond length. This results in low tendency of catenation in nitrogen.

MULTIPLE CHOICE QUESTIONS

1. Which of the following elements does not belong to Group 15?


(a) Nitrogen (b) Phosphorus (c) Arsenic (d) Tin
2. Atomic number of bismuth which belongs to 6th period is
(a) 51 (b) 52 (c) 83 (d) 84
3. The number of elements in Group 15 is
(a) 3 (b) 4 (c) 5 (d) 6
4. Which of the following orders regarding the melting points of hydrides of Group 15 is
(a) NH3 > PH3 > AsH3 (b) NH3 < PH3 < AsH3 (c) NH3 > PH3 < AsH3 (d) NH3 < PH3 > AsH3
15.50 Complete Chemistry—JEE Main

5. Coordination number of nitrogen never exceeds


(a) 3 (b) 4 (c) 5 (d) 6
6. The ammonia molecule is
(a) triangular planar
(b) regular tetrahedron with bond angles equal to 109° 28 .
(c) trigonal bipyramidal
(d) trigonal pyramidal
7. Ammonia acts as a
(a) neutral species (b) Lewis acid (c) Lewis base (d) amphoteric hydride
8. Which of the following orders regarding the bond distance M—H of the hydrides of Group 15 elements is correct?
(a) N—H > P—H > As—H (b) N—H < P—H < As—H
(c) N—H > P—H < As—H (d) N—H < P—H > As—H
9. Which of the following orders regarding the bond angle H—M—H of the hydrides of Group 15 elements is
correct?
(a) H—N—H > H—P—H > H—As—H (b) H—N—H < H—P—H < H—As—H
(c) H—N—H > H—P—H < H—As—H (d) H—As—H < H—P—H > H—As—H
10. Which of the following orders regarding the bond enthalpy of M—H bond in the hydrides of Group 15 elements
is correct?
(a) N—H > P—H > As—H (b) N—H < P—H < As—H
(c) N—H > P—H < As—H (d) N—H < P—H > As—H
11. Which of the following orders regarding the enthalpy of formation of hydrides MH3 of Group 15 is correct?
(a) NH3 > PH3 > AsH3 (b) NH3 < PH3 < AsH3 (c) NH3 > PH3 < AsH3 (d) NH3 < PH3 > AsH3
12. Which of the following molecules includes nitrogen atom having oxidation state equal to – 2?
(a) N2 (b) NH2OH (c) N2H4 (d) NH3
13. The molecule in which the oxidation state of nitrogen is –1 is
(a) N2 (b) NH2OH (c) N2H4 (d) NH3
14. Which of the following trihalides is not known?
(a) NCl3 (b) PCl3 (c) NI3 (d) PI3
15. Which of the following halides is not known?
(a) NCl5 (b) PF5 (c) AsF5 (d) SbCl5
16. Which of the following chemical equations is correctly formulated?
(a) NCl3 + 3H2O Æ NH3 + 3HOCl (b) PCl3 + 3H2O Æ PH3 + 3HOCl
(c) AsCl3 + 3H2O Æ H3As + 3HOCl (d) SbCl3 + 3H2O Æ SbH3 + 3HOCl
17. Which of the following chemical equations is not correctly formulated?
(a) PCl3 + 3H2O H3PO3 + 3HCl (b) AsCl3 + 3H2O H3AsO3 + 3HCl
(c) SbCl3 + 3H2O H3SbO3 + 3HCl (d) BiCl3 + H2O BiO+ + 2H+ + 3Cl–
18. Which of the following chemical equations is not correctly formulated?
(a) NCl3 + 3H2O NH3 + 3HOCl (b) PCl3 + 3H2O H3PO3 + 3HCl
(c) AsCl3 + H2O AsO+ + 2H+ + 3Cl– (d) SbCl3 + H2O SbO+ + 2H+ + 3Cl–
19. Which of the following statements is not correct?
(a) With the exception of nitrogen, all other elements of Group 15 form pentahalides
(b) The pentahalides of elements of Group 15 are trigonal bipyramidal
(c) The pentahalides of elements of Group 15 undergo incomplete hydrolysis to give the corresponding acids
(d) The pentahalides of elements of Group 15 undergo complete hydrolysis to give the corresponding acids
20. Which of the nitrogen oxides is obtained when ammonium nitrate is heated?
(a) N2O (b) NO (c) NO2 (d) N2O5
21. The nitrogen oxide obtained on heating lead nitrate is
(a) N2O (b) NO (c) NO2 (d) N2O5
22. Which of the following is expected to be paramagnetic?
(a) NH2OH (b) N2H6Cl2 (c) N2O3 (d) NO2
Study of the p-Block Elements (Groups 13, 14 and 15) 15.51

23. The oxidation of NO in air produces


(a) N2O (b) N2O3 (c) N2O4 (d) N2O5
24. The dimerization of NO2 is accompanied with
(a) an increase in paramagnetism (b) a decrease in paramagnetism
(c) no change in paramagnetism (d) a darkening in colour
25. Lightning bolts in the atmosphere cause the formation of
(a) NO (b) NH3 (c) NH3·H2O (d) NH2OH
26. Which of the nitrogen oxide is released when P4O10 is treated with HNO3?
(a) N2O (b) NO (c) NO2 (d) N2O5
27. Given that 2NO2 N 2O 4 DH = negative
The conditions at which the conversion of NO2 to N2O4 becomes more and more are
(a) lower temperature and lower pressure (b) lower temperature and higher pressure
(c) higher temperature and higher pressure (d) higher temperature and lower pressure
28. Which of the following acids of phosphorus is a reducing acid?
(a) H3PO3 (b) H3PO4 (c) H4P2O7 (d) (HPO2)3
29. Which of the following acids is not an oxidizing agent?
(a) H3PO4 (b) H4P2O6 (c) H4P2O7 (d) H3PO2
30. The structure of (HPO3)3 is
(a) linear (b) tetrahedral (c) cyclic (d) octahedral
31. Orthophosphoric acid is a
(a) monobasic acid (b) dibasic acid (c) tribasic acid (d) tetrabasic acid
32. Orthophosphorous acid is a
(a) monobasic acid (b) dibasic acid (c) tribasic acid (d) tetrabasic acid
33. Which of the following acids is unstable?
(a) HNO2 (b) HNO3 (c) H2N2O2 (d) HOONO
34. Which of the following acids is explosive?
(a) Hyponitrous acid (b) Hyponitric acid (c) Nitroxylic acid (d) Peroxynitrous acid
35. Polymetaphosphoric acid has
(a) linear structure of HPO3 units (b) branched structure of HPO3 units
(c) cyclic structure of HPO3 units (d) discrete molecules of (HPO3)2, (HPO3)3, and so on
36. In which of the following acids, P—P bond is present?
(a) Hypophosphoric acid (b) Pyrophosphoric acid
(c) Orthophosphoric acid (d) Polymetaphosphoric acid
37. Which of the following formulae represents hypophosphoric acid?
(a) H3PO3 (b) H4P2O6 (c) H3PO4 (d) H4P2O7
38. Which of the following formulae represents pyrophosphoric acid?
(a) H3PO3 (b) H4P2O6 (c) H3PO4 (d) H4P2O7
39. With excess of water, PCl5 gives
(a) H3PO4 + HCl (b) H3PO3 + HCl (c) H3PO2 + HCl (d) H4P2O7 + HCl
40. The oxide which on dissolving in water turns blue litmus red is
(a) P2O5 (b) As2O3 (c) BaO (d) Sb2O3
41. Conc. HNO3 oxidizes iodine to
(a) HI (b) HIO3 (c) NH4I (d) HIO4
42. Which of the following acids contains phosphorus in the +4 oxidation state?
(a) Hypophosphorous acid (b) Orthophosphorous acid
(c) Phosphoric acid (d) Hypophosphoric acid
43. Phosphine may be produced by adding to water some
(a) Ca3P2 (b) P4O6 (c) P4O11 (d) HPO3
15.52 Complete Chemistry—JEE Main

44. In phosphorus oxide the number of oxygen atoms bonded to each phosphorus atom is
(a) 1 (b) 2 (c) 3 (d) 4
45. Which of the following statements is not correct?
(a) The molecule of N2O is linear (b) NO shows a strong tendency to form dimer N2O2

4 oxidizes NO to NO3 (d) NO can oxidize SO2 in water to H2SO4
46. Which of the following statements is not correct?
(a) The molecule of NO2 is angular
(b) Low temperatures favours the dimerization of NO2 to N2O4
(c) NO2 is soluble in water giving a mixture of HNO2 and HNO3
(d) The structure of N2O4 is nonplanar
47. Which of the following statements is not correct?

(c) Nitrite ion has a linear structure


(d) Thermal decomposition of NaNO3 gives NaNO2
48. Which of the following statements is not correct?
(a) Phosphorus trioxide exists as dimer P4O6 (b) Phosphorus pentoxide exists as dimer P4O10
(c) Orthophosphorous acid, H3PO3, is a tribasic acid (d) Orthophosphoric acid, H3PO4, is a tribasic acid
49. Which of the following statements is not correct?
(a) Orthophosphoric acid has approximately tetrahedrally shaped molecule
(b) Pyrophosphoric acid is represented as H4P2O7
(c) All the four sodium salts of pyrophosphoric acid are known
(d) Metaphosphoric acid is a glassy polymeric solid with the empirical formula HPO3
50. Which of the following represents superphosphate of lime?
(a) Ca(H2PO4)2 (b) Ca(H2PO4)2 + 2CaSO4
(c) Ca(H2PO4)2 + 2Ca(NO3)2 (d) Ca3(PO4)2
51. Which of the following represents triple superphosphate?
(a) 3Ca(H2PO4)2 (b) Ca3(PO4)2
(c) Ca(H2PO4)2 + 2CaSO4 (d) Ca(H2PO4)2 + 2Ca(NO3)2
52. Which of the following represents nitrophos?
(a) Ca(H2PO4)2 (b) Ca3(PO4)2
(c) Ca(H2PO4)2 + 2CaSO4 (d) Ca(H2PO4)2 + 2Ca(NO3)2
53. Which of the following halides does not exist?
(a) PI5 (b) PBr5 (c) PCl5 (d) PF5
54. Which of the following halides does not exist?
(a) SbF5 (b) AsF5 (c) PF5 (d) NF5
55. — —
The P P P bond angle in white phosphorus is
(a) 120° (b) 90° (c) 60° (d) 109° 28¢
56. Which of the following forms maximum P —H bonds?
(a) H3PO2 (b) H3PO3 (c) H3PO4 (d) H4P2O7
57. The solid phosphorus pentachloride exists as
(a) PCl5 (b) PCl+4 Cl– (c) PCl–6 (d) PCl+4 · PCl–6
58. The hydrolysis of PCl3 results into
(a) HPO3 (b) H3PO4 (c) H3PO2 (d) H3PO3
59. The most reactive form of phosphorus is
(a) red (b) yellow (c) violet (d) black
60. Poisonous form of phosphorus is
(a) white (b) red (c) black (d) violet
61. Which of the following hydrides does not exist?
(a) NH3 (b) PH5 (c) AsH3 (d) N2H4
Study of the p-Block Elements (Groups 13, 14 and 15) 15.53

62. Which of the following hydrides is thermally least stable?


(a) NH3 (b) PH3 (c) AsH3 (d) SbH3
63. Which of the following halides does not hydrolyse?
(a) NF3 (b) PCl3 (c) AsCl3 (d) SbCl3
64. Which of the following oxides does not contain N — N bond?
(a) N2O (b) N2O3 (c) N2O4 (d) N2O5
65. POCl3 is obtained when
(a) PCl5 is heated in air (b) PCl5 is completely hydrolysed
(c) PCl5 is hydrolysed to the limited extent (d) H2SO4 is added to PCl5
66. Which of the following hydrides has the highest melting point?
(a) NH3 (b) PH3 (c) AsH3 (d) SbH3
67. In the reaction N2 + 3H2 2NH3, the yield of ammonia is expected to be maximum at
(a) low temperature and low pressure (b) low temperature and high pressure
(c) high temperature and low pressure (d) high temperature and high pressure
68. Nitrogen is in the lowest oxidation state in
(a) nitrous oxide (b) nitric oxide (c) nitrogen dioxide (d) dinitrogen pentoxide
69. Highest oxidation state of nitrogen is achieved in
(a) nitrogen dioxide (b) dinitrogen trioxide (c) dinitrogen tetroxide (d) dinitrogen pentoxide
70. Which of the nitrogen oxides is obtained when copper reacts with conc. HNO3?
(a) NO (b) NO2 (c) H2O3 (d) N2O5
71. Which of the following orders is correct regarding the boiling points of elements of Group 15?
(a) NH3 > PH3 > AsH3 (b) NH3 > PH3 < AsH3 (c) NH3 < PH3 > AsH3 (d) NH3 < PH3 < AsH3
72. Which of the following statements regarding nitrogen molecule is not correct?
(a) Amongst the homonuclear diatomic molecules of second period, the bond dissociation enthalpy is maximum
in case of N2 molecule
(b) Amongst the homonuclear diatomic molecules of second period, the bond length is minimum in case of N2
molecule
(c) Nitrogen molecule is paramagnetic in nature
(d) Only at high temperatures, nitrogen molecule reacts with metals and nonmetals forming ionic and covalent
nitrides
73. Which of the following statements regarding ammonia is not correct?
(a) Ammonia is a colourless and pungent smelling gas

(c) Ammonia can act as Lewis acid


(d) Ammonium chloride dissolves in liquid ammonia has acidic properties
74. The gas liberated when copper reacts with dilute HNO3 is
(a) NO (b) NO2 (c) N2O3 (d) N2O5
75. In the brown-ring test of nitrate ion, the compound formed is
(a) [Fe(H2O)5NO]2+ (b) [Fe(H2O)5NO]3+ (c) [Fe(H2O)4(NO2)]2+ (d) [Fe(H2O)3(NO)3]2+
76. The treatment of glycerol with conc. HNO3 in the presence of conc. H2SO4 gives
(a) mononitroglycerol (b) dinitroglycerol (c) nitroglycerine (d) CO2 and H2O
77. Which of the following equations is not correctly formulated?
(a) Conc. HNO3 acquires a yellow colour due to the following photo-decomposition reaction
4HNO3 Æ 4NO2 + 2H2O + O2
(b) Cu + 4HNO3 Cu(NO3)2 + 2H2O + 2NO2
dilute
(c) 4Zn + 10HNO3 4Zn(NO3)2 + 5H2O + N2O
dilute
(d) P4 + 20HNO3 4H3PO4 + 4H2O + 20NO2
15.54 Complete Chemistry—JEE Main

78. The aqua regia is


(a) 3 parts conc. HNO3 + 1 part conc. HCl (b) 1 part conc. HNO3 + 3 parts conc. HCl
(c) 2 parts conc. HNO3 + 2 parts conc. HCl (d) 2.5 parts conc. HNO3 + 0.5 part conc. HCl
79. Ammonia can be dried by
(a) conc. H2SO4 (b) P4O10 (c) anhydrous CaCl2 (d) anhydrous CuSO4
80. Which of the following is the neutral oxide?
(a) NO (b) N2O (c) N2O3 (d) NO2
81. Nitrogen dioxide is not obtained on heating
(a) KNO3 (b) Pb(NO3)2 (c) Cu(NO3)2 (d) AgNO3
82. Which of the following halides of nitrogen is expected to be most stable?
(a) NF3 (b) NCl3 (c) NBr3 (d) NI3
83. Which of the following halides of nitrogen is expected to have least dipole moment?
(a) NF3 (b) NCl3 (c) NBr3 (d) NI3
84. Which of the following oxides of nitrogen occurs as a white solid?
(a) NO (b) NO2 (c) N2O3 (d) N2O5
85. The pseudohalogen amongst the following is
(a) NO (b) CO (c) Cl2 (d) (CN)2
86. Distillation of conc. HNO3 with P4O10 gives
(a) NO (b) N2O (c) NO2 (d) N2O5
87. Which of the following hydrides is acidic in nature?
(a) NH2OH (b) NH3 (c) N3H (d) N2H4
88. The order of stability of hydrides of Group 15 is
(a) NH3 > PH3 > AsH3 (b) NH3 < PH3 < AsH3 (c) NH3 > PH3 < AsH3 (d) NH3 < PH3 > AsH3
89. On heating ammonium dichromate, the products obtained are
(a) N2O and H2O (b) N2 and H2O (c) NO2 and H2O (d) NO and N2O
90. The laughing gas is
(a) N2O (b) NO (c) NO2 (d) N2O3
91. Which of the following statements regarding phosphorus is not true?
(a) Phosphorus belongs to the Group 15 of the periodic table
(b) The element phosphorus is obtained by heating the rock phosphate with coke and sand in an electric furnace
at about 1700 – 1800 K
(c) The formula of phosphorus is P4
(d) Black phosphorus is the least stable form of the allotropes of phosphorus
92. Pyrophosphorous acid is a
(a) monobasic acid (b) dibasic acid (c) tribasic acid (d) tetrabasic acid
93. Which of the following represents metaphosphoric acid?
(a) HPO3 (b) H3PO3 (c) H3PO4 (d) H4P2O7
94. Which of the following represents orthophosphoric acid?
(a) HPO3 (b) H3PO3 (c) H3PO4 (d) H4P2O7
95. Which of the following represents hypophosphorous acid?
(a) HPO3 (b) H3PO3 (c) H3PO4 (d) H3PO2
96. The white phosphorus is stored
(a) in air (b) under water (c) under kerosene (d) under CS2
97. Which of the following statements is true?
(a) Both white and red phosphorus are soluble in water
(b) Both white and red phosphorus are soluble in carbon disulphide
(c) White phosphorus is soluble in carbon disulphide whereas red phosphorus is insoluble
(d) White phosphorus is insoluble in carbon disulphide whereas red phosphorus is soluble
Study of the p-Block Elements (Groups 13, 14 and 15) 15.55

98. Which of the following statements is true?


(a) Both white and red phosphorus are reactive
(b) Both white and red phosphorus are inactive
(c) White phosphorus is reactive whereas red phosphorus is inactive
(d) White phosphorus is inactive whereas red phosphorus is reactive
99. Which of the following statements about phosphorus is not true?
(a) Phosphorus does not occur in free state
(b) Phosphorus is present in bones and teeth
(c) Phosphorus exists in several allotropic forms
(d) White phosphorus is much less active than red variety
100. When phosphorus is heated with conc. HNO3, it reduces the acid to
(a) NO (b) NO2 (c) N2O3 (d) N2O5
101. When phosphorus is heated with conc. HNO3, it is oxidized to
(a) H3PO2 (b) H3PO3 (c) H3PO4 (d) H4P2O7
102. P4O6 is the anhydride of
(a) H3PO2 (b) H3PO3 (c) H3PO4 (d) H4P2O7
103. P4O10 is the anhydride of
(a) H3PO2 (b) H3PO3 (c) H3PO4 (d) H4P2O7
104. Which of the following is orthophosphorous acid?
(a) H3PO2 (b) H3PO3 (c) H3PO4 (d) H4P2O7
105. When P4O6 is dissolved in cold water, the acid obtained is
(a) H3PO2 (b) H3PO3 (c) H3PO4 (d) H4P2O7
106. When P4O10
(a) H3PO2 (b) H3PO3 (c) H3PO4 (d) H4P2O7
107. Which of the following halogen compounds of nitrogen is not observed?
(a) NF3 (b) NF5 (c) N2F2 (d) NCl3
108. Which of the following allotropes of phosphorus is most reactive?
(a) Black (b) Brown (c) Red (d) White
109. Which of the following oxo acids of phosphorus is a reducing agent?
(a) H3PO4 (b) H3PO3 (c) H4P2O7 (d) HPO3
110. The number of P—O—P bonds in cyclic metaphosphate ion is
(a) 0 (b) 1 (c) 3 (d) 4
111. Which of the following is least basic?
(a) NF3 (b) NH3 (c) NCl3 (d) NI3
112. Which of the following is least basic?
(a) H3PO3 is diabasic and reducing acid (b) H3PO3 is diabasic and nonreducing acid
(c) H3PO4 is diabasic and reducing acid (d) H3PO4 is tribasic and reducing acid
113. Strong heating of orthophosphoric acid yields
(a) P2O5 (b) P4O10 (c) (HPO3)n (d) H3PO3+O2
114. Which of the following nitrogen oxide is a neutral oxide?
(a) N2O3 (b) N2O5 (c) NO2 (d) N2O
115. Which of the following order regarding second ionization energy of N, P and As is correct?
(a) N > P > As (b) N > As > P (c) P >N >As (d) P >As >N
116. Which of the following metaphosphate ion is not known to exist in free state?
(a) PO3– (b) (PO3)22– (c) (PO3)33– (d) (PO3)4–4
117. Which of the following facts regarding bond energy is not correct?
(a) EP – F > EN – F (b) EP – Cl > EN – Cl (c) EP – O < EN – O (d) EP – C < EN – C
118. Which of the following order regarding stability of phosphorus is correct?
(a) White > Red > Black (b) White > Black > Red
(c) Black > Red > White (d) Black > White > Red
15.56 Complete Chemistry—JEE Main

ANSWERS
1. (d) 2. (c) 3. (c) 4. (c) 5. (b) 6. (d)
7. (c) 8. (b) 9. (a) 10. (a) 11. (b) 12. (c)
13. (b) 14. (c) 15. (a) 16. (a) 17. (c) 18. (c)
19. (c) 20. (a) 21. (c) 22. (d) 23. (c) 24. (b)
25. (a) 26. (d) 27. (b) 28. (a) 29. (d) 30. (c)
31. (c) 32. (b) 33. (d) 34. (c) 35. (a) 36. (a)
37. (b) 38. (d) 39. (a) 40. (a) 41. (b) 42. (d)
43. (a) 44. (c) 45. (b) 46. (d) 47. (c) 48. (c)
49. (c) 50. (b) 51. (a) 52. (d) 53. (a) 54. (d)
55. (c) 56. (a) 57. (d) 58. (d) 59. (b) 60. (d)
61. (b) 62. (d) 63. (a) 64. (d) 65. (c) 66. (a)
67. (b) 68. (a) 69. (d) 70. (b) 71. (b) 72. (c)
73. (c) 74. (a) 75. (a) 76. (c) 77. (b) 78. (b)
79. (c) 80. (b) 81. (a) 82. (a) 83. (d) 84. (d)
85. (d) 86. (d) 87. (c) 88. (a) 89. (b) 90. (a)
91. (d) 92. (b) 93. (a) 94. (c) 95. (d) 96. (b)
97. (c) 98. (c) 99. (d) 100. (b) 101. (c) 102. (b)
103. (c) 104. (b) 105. (b) 106. (c) 107. (b) 108. (d)
109. (b) 110. (c) 111. (a) 112. (a) 113. (c) 114. (d)
115. (b) 116. (a) 117. (c) 118. (c)

HINTS AND SOLUTIONS


1. Tin belongs to Group 14.
2. Atomic number of bismuth will be 2 + 8 + 8 + 18 + 18 + 14 + 15 = 83
4. Because of hydrogen bonding, the melting point of NH3 is greater than PH3.
8. As the size of the atom of Group 15 increases, the bond length between the atom and hydrogen also increases.
9. Because of more positive charge on hydrogen and smaller size of nitrogen, the repulsion between H and H atoms
makes the bond angle in NH3 bigger than in PH3 and AsH3.
10. N — H bond is more stronger than P—H which, in turn, is stronger than As— H.
14. Because of the bigger size of iodine, it cannot be accommodated around the smaller size nitrogen atom.
15. The oxidation number of nitrogen does not exceed +3 because of the nonavailability of d orbitals.
16. NCl3 + 3H2O NH3 + 3HOCl AsCl3 + 3H2O H3AsO3 + 3HCl
PCl3 + 3H2O H3PO3 + 3HCl SbCl3 + H2O SbO+ + 2H + + 3Cl–
17. SbCl3 + H2O SbO+ + 2H + + 3Cl–
18. AsCl3 + 3H2O H3AsO3 + 3HCl
19. The petahalides of elements of Group 15 undergo complete hydrolysis to give corresponding acids, e.g.
PCl5 + 4H2O 5HCl + H3PO4
20. NH4NO3 N2O + 2H2O
21. 2PbNO3 2PbO + 4NO2 + O2
22. NO2 is an odd-electron molecule.
23. NO + O NO2 N 2O 4
24. N2O4 is diamagnetic whereas NO2 is paramagnetic.
26. P4O10 + 4HNO3 4H3PO3 + 2N2O5
Study of the p-Block Elements (Groups 13, 14 and 15) 15.57

27. DH negative means exothermic. Hence, lower temperature and higher pressure (as D g = –1) favours the formation
of dimer.
28. H3PO3 contains phosphorus in +3 oxidation state. It can be oxidized to +5 oxidation state (i.e. H3PO4). Hence, it
is a reducing agent.
29. H3PO2 contains phosphorus in +1 oxidation state. Its oxidation state cannot be reduced further. Hence, it is not an
oxidizing agent.
35. The structure of polymetaphosphoric acid is
O OH O
P P P
O O O O
OH OH OH

36. Hypophosphoric acid is H4P2O6. Its structure is


O O
HO P P OH
OH OH

38. Pyrophosphoric acid is H4P2O7.


39. PCl5 + 4H2O 5HCl + H3PO4
40. P2O5 is an acidic oxide.
42. Hypophosphoric acid is H4P2O6. The oxidation state of P is + 4.
43. Ca3P2 + 6H2O 2PH3 + 3Ca(OH)2
45. NO is dimerized to a very little extent.
46. The structure of N2O4 is planar.
47. The shape of NO2– is bent. It is due to the presence of a lone pair of electrons on the nitrogen atom.
N N
O O O O
48. H3PO3 is a diabasic acid.
49. Pyrophosphoric is tetrabasic acid, but it forms only two series of salts, e.g. Na2H2P2O7 and Na4P2O7.
55. White phosphorus has a molecular formula P4. The four atoms are present at the corners of a tetrahedron.
56. The structures of acids are
O O O O
H P OH, HO P OH HO P OH, H P O P OH
H OH OH OH OH
62. The stability of hydrides decreases down the group.
65. PCl5 + H2O POCl3 + 2HCl
calculated
quantity
66. Because of hydrogen bondings, NH3 has the highest melting point.
67. The reaction is exothermic and is attended to by the decrease in the gaseous molecules. Hence, to increase the
yield of NH3, the reaction should be carried out at low temperature and high pressure.
70. Reaction is Cu + 4HNO3 Cu(NO3)2 + 2H2O + 2NO2
(conc.)
72. Nitrogen is a diamagnetic as it contains no unpaired electrons.
73. Ammonia contains a lone pair. Hence, it acts as Lewis base.
15.58 Complete Chemistry—JEE Main

74. Reaction is 3Cu + 8NHO3 3Cu(NO3)2 + 4H2O + 2NO


(dil.)
77. Same as Q. 74.
81. Heating of KNO3 liberates O2.
82. Fluorine has the smallest size amongst halogens.
86. The reaction is 4HNO3 + P4O10 4HPO3 + 2N2O5
89. The reaction is (NH4)2Cr2O7 N2 + 4H2O + Cr2O3
91. Phosphorus exists in three allotropic forms—white, red and black. Black phosphorus exists in one amorphous and
three crystalline forms. Black phosphorus is the most stable form of the element.
96. White phosphorus is insoluble in water.
99. Chemically, red phosphorus is much less reactive than the white form.
102. P4O6 + 6H2O 4H3PO3
103. P4O10 + 6H2O 4H3PO4
107. The maximum valence of nitrogen is four as it has four valence orbitals. It has no d orbitals to show valence more
than four
108. White phosphorus is discrete P4 molecule.
109. H3PO3 is HOP(OH)2. The bond P—H is responsible for reducing agent.
110. The cyclic metaphosphate ion is

O O
P
O O
O O
P P
O
– O O

111. The highly electronegative F atoms make NF3 a poor electron donor.
112. See Q.109
113. The reactions are H3PO3 ægentle
æææ
220∞ C
heat
Æ H4P2O7 æstrong
æææ320∞ C
heat
Æ (HPO3)n

114. N2O3, N2O5 and NO2 are acidic oxide while N2O is a neutral oxide.
115. The effective nuclear charge of As is larger than P due to less shielding effect of 3d orbitals-making the second
ionization energy of As greater than that of P.
116. The free monometaphosphate ions does not exist. The metaphosphates form a family of ring compounds

MULTIPLE CHOICE QUESTIONS FROM AIEEE AND JEE MAIN

1. What may be expected to happen when phosphine gas is mixed with chlorine gas?
(a) PH3◊Cl2 is formed with warming up (b) The mixture only cools down
(c) PCl3 and HCl are formed and the mixture warms up (d) PCl5 and HCl are formed and the mixture cools down
[2003]
2. The number of hydrogen atom(s) attached to phosphorus atom in hypophosphorous acid is
(a) zero (b) one (c) two (d) three [2005]
3. Regular use of which of the following fertilizers increases the acidity of soil?
(a) Potassium nitrate (b) Urea
(c) Superphosphate of lime (d) Ammonium sulphate [2007]
4. The stability of dihalides of Si, Ge, Sn and Pb increases steadily in the sequence
(a) GeX2  SiX2  SnX2  PbX2 (b) SiX2  GeX2  PbX2  SnX2
Study of the p-Block Elements (Groups 13, 14 and 15) 15.59

(c) SiX2  GeX2  SnX2  PbX2 (d) PbX2  SnX2  GeX2  SiX2 [2007]
5. Identify the incorrect statement among the following.
(a) Ozone reacts with SO2 to give SO3
(b) Silicon reacts with NaOH(aq) in the presence of air to give Na2SiO3 and H2O
(c) Cl2 reacts with excess of NH3 to give N2 and HCl
(d) Br2 reacts with hot and strong NaOH solution to give NaBr, NaBrO4 and H2O [2007]
6. Three reactions involving H2PO 4– are given below.
(i) H3PO4 + H2O Æ H3O+ + H2PO 4–
(ii) H2PO4– + H2O Æ HPO 42– + H3O+
(iii) H2PO4– + OH– Æ H3PO4 + O2–
In which of the above reactions does H2PO4– acts as an acid?
(a) (i) only (b) (ii) only (c) (i) and (ii) only (d) (iii) only [2010]
7. Which of the following statements is wrong?
(a) N2O4 has two resonance structures.
(b) The stability of hydrides increases from NH3 to BiH3 in group 15 of the periodic table.
(c) Nitrogen cannot form dp-pp bond.
(d) Single N—N bond is weaker than the single P—P bond. [2011 cancelled]
8. The molecule having smallest bond angle is
(a) NCl3 (b) AsCl3 (c) SbCl3 (d) PCl3 [2012]
9. Which one of the following properties is not shown by NO?
(a) It’s bond order is 2.5 (b) It is diamagnetic in gaseous state
(c) It is a neutral oxide (c) It combines with oxygen to form nitrogen dioxide
[2014]
10. Which one of the following does not have a pyramidal shape?
(a) (CH3)3N (b) (SiH3)3N (c) P(CH3)3 (d) P(SiH3)3
[2014, online]
11. Which of the following statements is true?
(a) HNO3 is a stronger acid than HNO2 (b) H3PO3 is a stronger acid than H2SO3
(c) In aqueous medium HF is a stronger acid than HCl (d) HClO4 is a weaker acid than HClO3. [2006]
12. Which of the following compounds has a P—P bond?
(a) H4PO2O5 (b) H4P2O6 (c) H4P2O7 (d) (HPO3)3
[2015, online]
13. Addition of phosphate fertilisers to water bodies causes:
(a) enhanced growth of algae (b) increase in amount of dissolved oxygen in water
[2015, online]
14. Assertion: Nitrogen and oxygen are the main components in the atmosphere but these do not react to form oxides
of nitrogen
Reason: The reaction between nitrogen and oxygen requires high temperature.
(a) Both assertion and reason are correct, and the reason is the correct explanation for the assertion
(b) Both assertion and reason are correct, but the reason is not the correct explanation for the assertion
(c) The assertion is incorrect, but the reason is correct.
(d) Both the assertion and reason are incorrect. [2015]
15. The pair in which phosphorus atoms have a formal oxidation state of +3 is
(a) Pyrophosphorous and hypophosphoric acids
(b) Orthophosphorous and hypophosphoric acids
(c) Pyrophosphorous and pyrophosphoric acids
(d) Orthophosphorous and pyrophosphorous acids [2016]
15.60 Complete Chemistry—JEE Main

1000 ppb, 40 ppb, 100 ppm and 0.2 ppm. This is unsuitable for drinking respectively due to high concentration of
[2016]

ANSWERS
1. (c) 2. (c) 3. (d) 4. (c) 5. (d) 6. (b)
7. (b) 8. (c) 9. (b) 10. (b) 11. (a) 12. (b)
13. (a) 14. (a) 15. (d) 16. (b)

HINTS AND SOLUTIONS


O
2. The structure of hypophosphorous acid (H3PO2) is H P H
OH
There are two hydrogen atoms attached to P atom.
3. The acidity in soil increases due to H+ and SO42– ions released from ammonium sulphate.
4. The stability of +2 oxidation state of elements of Group 14 increases on descending the group.
5. The correct reaction is 3Br2 + 6NaOH æÆ 5NaBr + NaBrO3 + 3H2O
(hot)
+
6. The reaction (ii) provides H3O . Hence, in this reaction H2PO –4 acts as an acid.
7. The stability of hydrides decreases from NH3 to BiH3 in group 15 due to decrease in bond energy M—H.
8. The bond angle Cl —A—Cl (where A is N, P, As or Sb) decreases with the decrease in the electronegativity or
increase in the size of the central atom A. Thus, out of NCl3, PCl3, AsCl3 and SbCl3, the latter is expected to have
the smallest bond angle.
9.
(s 2s)2 (s*2s)2 (p 2px)2 (p 2py)2 (s 2p)2 (p* 2px)1
The molecule is paramagnetic as it contains one unpaired electron in p*2px orbital. Its bond order is 2.5 ( = (6
bonding electrons –1 antibonding electron)/2). It is a neutral oxide and combines with oxygen to give NO2.
10. Tetrasilylamine is planar due to back bonding of the lone pair in p orbital of N atom to an empty orbital of Si
thereby form double bond

12. Hypophosphoric acid has P—P linkage


OH OH
Ω Ω
O == P -- P == O
Ω Ω
OH OH
(H4P2O6)

14. Nitrogen and oxygen in atmosphere do not combine unless high temperature is employed. Atmospheric lightening
also helps in forming oxides of nitrogen.
15. Phosphorous series contains P in +3 state while phosphoric series contains P in +5 state.
16. The permissible limits are as follows.
Iron 0.2 ppm Lead 50 ppb
Fluoride 1.5 ppm Nitrate 50 ppm
16
Study of the p-Block Elements
(Groups 16, 17 and 18)

The Group 16 Elements


Group 16 of the periodic table consists of oxygen (O), sulphur (S), selenium (Se), tellurium (Te) and polonium (Po).
These are known as chalcogens or ore-forming elements. The physical properties of these elements are recorded in
Table 1.
Table 1 Atomic and Molecular Properties of Group 16 Elements

Property O S Se Te Po
[He](2s)2(2p)4 [Ne](3s)2(3p)4 [Ar](3d)10(4s)2(4p)4 [Kr](4d)10(5s)2(5p)4 [Xe](4f)14(5d)10(6s)2(6p)4

Atomic number 8 16 34 52 84
Relative atomic mass 16.00 32.08 78.96 127.60 210
Covalent radius, r/pma 74 104 114 137 168b
Ionic radius M2–, r/pm, 140 184 198 221 230c
d
Melting point, T/K 54 393 490 725 519-527
Boiling point, T/K 90 718 958 1263 1235
e f g
Density at 298 K, r/g cm –3
1.32 2.06 4.34 6.25 9.14b
–1
Ionization energy, I/kJ mol
I 1314 1000 941 869 813
II 3388 2251 2045 1790 –
Electronegativity 3.5 2.5 2.4 2.1 2.0
E/kJ mol–1 –142 –200 – – –
(a) two-coordinate covalent radius, (b) six-coordinate metallic radius, (c) approximate, (d) monoclinic form, (e) at 54 K, (f) rhombic form,
(g) a-monoclinic, (h) a-form

Description of Physical Properties


ns)2(np)4, where n varies from 2 to 6.

electronegativity and nonavailability of d orbitals.


3. The metallic character increase on descending the group.
O and S are non-metallic
Se and Te are weaker nonmetailic
Po is metallic
The above fact is supported by the chemical behaviour of these elements.
The tendency to from M2+ ions increases down the group.
The stability of M2– ions decreases down the group.
16.2 Complete Chemistry—JEE Main

4. The covalent radii increase on descending the group. The variation is as per expectation (Fig. 1). From S to Se,
the difference in atomic radii is small due to the poor shielding of nuclear charge by 3d electrons. The ionic
radii of M2– also increases on descending the group.
5. The melting and boiling points increase down the group with the exception of the last member for which a
decrease is observed (Fig. 2).
6. The ionization energy decreases on descending the group. The ionization energy of an element in Group 16 is

incoming electron from the smaller, more compact electronic cloud of oxygen atom.

1200
b.pt
Atomic Radii
1000

150
800

130
T/K

600
r/pm

m.pt
110
400

90 200

70 O2 S8 Se8 Te8 Po
O S Se Ta Po

Fig. 1 Atomic Radii of the Group 16 Elements Fig. 2 Melting and Boiling Points of Molecules of the Group 16
8. Polonium, the last member of he group, is a radioactive element.
9. Oxygen exists as diatomic molecules with two unpaired electrons and is
paramagnetic.
10. Sulphur exists in two allotropic forms—rhombic sulphur (or a sulphur),
monoclinic sulphur (or b g-monoclinic
sulphur, is obtained by chilling hot concentrated solution of S in CS2 (or toluene Structure of S8 molecule
or C2H5OH). All three forms contain puckered S8 rings.
Rhombic sulphur is stable at room temperature while monoclinic sulphur is stable at
temperature above 369 K. These two forms change reversibly with slow heating or slow
cooling.
In addition to S8, sulphur rings of 6, 7, 9–15, 18 and 20 atoms are known. In cyclo–S6, 205.7 pm
the ring adopts a chair form in the solid.
At elevated temperatures (~ 1000 K), S2 is the dominant species with paramagnetic 102.2°
property like O2.
Selenium exists in six allotropic forms. Four of these forms are red and two are grey.
Three of the four red forms have Se8 molecules differing in packing in crystals. The fourth
form is amorphous and contains polymeric chains. The stable form of grey selenium Structure of S6 molecules

Tellurium exists only in one crystalline form.


Polonium exists in two crystal forms—cubic and rhombohedral.
Study of the p-Block Elements (Groups 16, 17 and 18) 16.3

Reactivity of Elements
ns)2(np)4. They may accept 2 electrons to attain noble-
2–

2 electrons, thus forming two covalent bonds.


2. Oxygen is the most electronegative element in this group. Thus, its compounds with metals are ionic in nature.
Sulphur, selenium and tellurium also form sulphides, selenides and tellurides with the more electropositive
metals of Groups I and II. The ionic nature of these compounds decreases down the group.
3. With hydrogen, covalent compounds such as H2O and H2S are formed. The compounds with more electronegative
elements are also covalent with the oxidation state +II. Examples are F2O, Cl2O and SCI2.
4. S, Se and Te also exhibit +4 and +6 oxidation states due to the availability of d orbitals. These are more stable
than +2 oxidation state.

Formation of Two Bonds


s p s p

(i) (ii)

Two bonds, bond sp3 hybridization, bond angle near


angle near to 90° to 109° 28¢
Tetrahedral structure with two positions
occupied by two lone pairs
Formation of Four Bonds
s p d

sp3d hybridization
Trigonal bipyramid structure
with one position occupied by a lone pair
Formation of Six Bonds
s p d

sp3d2 hybridization octahedral structure


5. The compounds of S, Se and Te with O are tetravalent with oxidation state +4. They have both oxidizing and
reducing characteristics.
6. The compounds of S, Se and Te with F are hexavalent with oxidation state +6. They have oxidizing characteristics.
The stability of higher oxidation states becomes less on descending the group.
7. Oxygen exists as diatomic gaseous molecules while other elements of the group catenate to form chains or
rings. The tendency of catenation is maximum in sulphur and is displayed in its compounds like sulphanes
H—Sn—H, polysulphuric acids HO3S—Sn—SO3H, polysulphides S2–n and various
allotropic forms of elements.
8. The bonds between S and O, or Se and O, are much shorter than the expected single S O
bond length between these atoms. This is due to the possibility of forming a double
bond via pp — dp bonding in addition to the normal sigma bond. In pp — dp

Sulphur forms stronger p bonds than selenium due to similar sizes of orbitals. pp — dp sideways overlap
Comparison of Sideways Overlap Along a Period As one travels from Si to P to S to CI, the size of the 3d orbitals
decreases due to the increase in the nuclear charge. This decrease in size of the 3d orbitals is associated with the
progressively stronger pp — dp bonds, due to the achievement of comparable sizes of the 2p orbital and 3d orbitals. Thus
pp— dp bonding does not exist between O and Si, the SiO4 units polymerize involving Si—O—Si s bonds.
pp — dp bonding between O and P is stronger with large tendency of polymerization of oxoanions.
16.4 Complete Chemistry—JEE Main

pp — dp bonding between O and S is still more stronger with a small amount of polymerization of oxanions.
pp — dp bonding between O and Cl is strongest with no polymerization of oxoanions.
Ozone
through pure and dry oxygen or air. The silent discharge is carried out by packing insulating materials in the space
between electrodes through which discharge passes. This prevents any local rise in temperature, which would lead to
the decomposition of ozone. The reaction
3O2 2O3
is endothermic (DH = +284.5 kJ mol–1).
Ozone is pale-blue gas with a characteristic strong smell. When inhaled in small amount, it causes headache and
nausea. In larger quantities, it is poisonous. The main reactions of ozone are the oxidation reactions.
PbS + 4O3 Æ PbSO4 + 4O2; 2KI + H2O + O3 Æ 2KOH + I2 + O2
(neutral medium)
2HCl + O3 Æ Cl2 + H2O + O2; 2K2MnO4 + H2O + O3 Æ 2KMnO4 + 2KOH + O2
2I2 + 2H2O + 4O3 Æ 4HIO3 + O2; 2K4Fe(CN)6 + O3 + H2O Æ 2K3Fe(CN)6 + 2KOH + O2
(moist)
Moist sulphur, phosphours and arsenic are also oxidized to higher acids.
When ozone is passed through mercury, the latter loses its meniscus and sticks to glass due to the formation of Hg2O
which dissolves in mercury.
With peroxides, the reactions taking place are
BaO2 + O3 Æ BaO + 2O2; H2O2 + O3 Æ H2O + 2O2
With organic compounds containing double and triple carbon-carbon bonds, ozone forms ozonides which gets
decomposed by water or dilute acids.
Ozone is formed at the height of about 20 km from oxygen by the absorption of sunlilght. There is a formation of ozone

(such as CCl2F2, which is used as a refrigerant (Freon-12) and as a propellant in aerosol spray) contribute to the
decomposition of ozone layer. Sunlight causes the decomposition of these compounds producing active chlorine which
catalysed the decomposition of ozone. Emission of nitrogen oxides from supersonic jets also contribute in depleting
the ozone layer.
The structure of ozone is
O 128 pm

O 116° 49¢ O
The oxygen-oxygen bond length in ozone is 128 pm which is intermediate between a single bond (148 pm) and double
bond (121 pm). It has been explained on the base on the base of the resonating structures shown below.
- +
O O

O O O O -
-

The structure of ozone is best described as follows. The oxygen are sp2 hybridized. Two of these orbitals in the central
atom and one each of the terminal oxygens are involved in s bonds. The remaining sp2 orbitals contain lone pairs of

electrons (= 18 – 4 – 10) occupy molecular orbitals formed from the three pz atomic orbital from the three oxygen
atoms. Two of these electrons are in the bonding molecular orbital and the remaining two in the nonbonding molecular
orbital. Thus, double bonds involving three oxygen atom is best described as four-electron three-centre bond. The bond
order of each O —O bond is 1.5.

Compounds of Group 16 Elements


Hydrogen Compounds
All the elements of Group 16 form covalent compounds with hydrogen. These are water (H2O), hydrogen sulphide
(H2S), hydrogen selenide (H2Se) hydrogen telluride (H2Te) and hydrogen polonide (H2Po).
Study of the p-Block Elements (Groups 16, 17 and 18) 16.5

A convenient method to prepare H2S, H2Se and H2Te is to treat the metal sulphides with acids.
FeS + H2SO4 Æ H2S + FeSO4
FeSe + H2SO4 Æ H2Se + FeSO4
Al2Te3 + 3H2SO4 Æ Al2(SO4)3 + 3H2Te
In the laboratory, H2S is generated by the action of dilute H2SO4 on iron(II) sulphide in a Kipp’s apparatus.
Some of the physical properties of hydrogen compounds of Group 16 elements are recorded in Table 2.
Table 2 Properties of Hydrogen Compounds of Group 16
Characteristic H 2O H 2S H2Se H2Te
Melting point, T/K 273 187.5 207.5 221.9
Boiling point, T/K 373 212.3 231.5 271.2
Dissociation constant at 298 K 1.8 ¥ 10 –16
1.3 ¥ 10 –7
1.3 ¥ 10 –4
2.3 ¥ 10–3
Bond angle H—M—H 104.5° 92.5° 90° 89.5°
Bond length H—M/pm 96 134 146 169
Bond energy H—M, E/kJ mol–1 463 347 276 238
Enthalpy of formation DfH°/kJ mol –1
–242 –20.2 81 154.4

Water has exceptionally high melting and boiling points due to


hydrogen bondings (Fig. 3). Beyond H2S, the melting and boiling 360
points increase on descending the group.
The bond angle in H2O is 104.5° which is close to the tetrahedral
angle 109°28¢. This indicates the sp3 hybridization of oxygen orbitals. 320
Two positions of a tetrahedron are occupied by H atoms and two are
occupied by lone pair of electrons. The decrease of bond angle from
109°28¢ to 104.5° has been explained on the basis of lone pair-bonding 280
pair repulsion of electrons. b.pt
T/K

The bond angle in other hydrides are more near to 90° indicating the
involvement of p orbitals of element in bonding with hydrogen atoms. 240
The bond enthalpy H—M decreases down the group indicating the m.pt
weakening of bond. This may be attributed to the increasing size of
bonding atomic orbital which overlap with 1s orbital of hydrogen atom. 200
Also there occurs a decrease in ionic character of the bond which
results into the decrease of electrostatic attraction.
The dissociation constant (K = [H+]2 [M2–]/[H2M]) increases on 160
H2O H2S H2Se H2Te
descending the group. This fact is in agreement with the weakening of
bond strength on descending the group. Fig 3. Melting and Boiling Points of Hydrides of
All hydrogen compounds except water are poisonous and have the Group 16 Elements
unpleasant odour.
The stability of hydrogen compounds decreases on descending the group. This is also shown by the decrease in their
enthalpies of formation.
Solid water is less dense than the liquid. Liquid water has maximum density of 1 g cm–3 at 4 °C. Ice has a rather open
structure due to the hydrogen bondings (Fig. 4). There is a lot of unoccupied space in solid water. On melting, some free
water molecules occupy some of this space resulting into the decrease in volume and hence increase in density. This
effect is continued up to 4 °C, beyond which normal expansion occurs so the density decreases.
In ice, hydrogen atoms are located between oxygen atoms, and by forming hydrogen bonds with the latter bind the
whole structure together into a huge network of atoms. Throughout the network there are four hydrogen atoms arranged
tetrahedrally about each oxygen atom, two oxygen atoms on either side of each hydrogen atom
Hard and Soft Water
Hard Water If a sample of water contains bicarbonate, chloride and sulphate of calcium and magnesium, it is
known as hard water. Such a sample of water does not form lather with soap. The Ca2+ and Mg2+ ions present in water
precipitates out the soap. For examples,
16.6 Complete Chemistry—JEE Main

2C17H35COONa(aq) + M2+(aq) Æ (C17H35COO)2M Ø + 2Na+(aq)


(soap) (Ca2+ or Mg2+)
Soft water A sample of water free from soluble salts of calcium and
O H O H O
magnesium is known as soft water. O H
H O H
H H O H H O H H
Methods of Removing Hardness of Water O O H
O
H H H
H O H
and permanent. O H O
H H
Temporary Hardness The temporary hardness in water is due to
H
O H O H
the presence of bicarbonates of calcium and magnesium. These can be H
O HO H OH O H
removed by the following two methods H H O H O
(i) Boiling of Water On boiling temporary water, the soluble salts are O HO O H
H HO H
precipitated as shown in the following.
H H O H
Mg(HCO3)2 æDæ Æ Mg(OH)2Ø + 2 CO2 ≠ H H
O
Ca(HCO3)2 æDæ Æ CaCO3Ø + H2O + CO2≠ O H H H O H O
H O H H
O H O H O
O H H
H H
(ii) Clark’s Method To the temporary water, calculated quantity of O
lime is added. This precipitates out soluble calcium and magnesium H H
O O
bicarbonates:
Ca(HCO3)2 + Ca(OH)2 Æ 2CaCO3Ø + 2H2O
Mg (HCO3)2 + 2Ca(OH)2 Æ 2CaCO3Ø + Mg (OH)2Ø + 2H2O Fig. 4 Schematic Arrangement of Water
Molecules in Ice
Permanent Hardness The permanent hardness in water is due to
the presence of chlorides and sulphates of calcium and magnesium. These cannot be removed by boiling water. The
following methods are employed to remove these salts from water.
(i) Treatment with Washing soda The addition of washing soda (sodium carbonate) causes the precipitation of calcium
and magnesium.
MCl2 + NaCO3 Æ MCO3Ø + 2NaCl
(Mg, Ca)
MSO4 + Na2CO3 Æ MCO3Ø + Na2SO4
(ii) Calgon’s Method The addition of calgon (sodium hexametaphosphate, Na6P6O18) causes the following reactions
Na6P6O18 Æ 2Na+ + Na4P6O182–

M2+ 2–
+ Na4P6O18 Æ [Na2MP6O18]2– + 2Na+
(Ca2+ or Mg2+)
The Ca2+ and Mg2+ ions are retained in the complex ions and thus do not interfere the functioning of soda.
(iii) Ion-Exchange Method Both natural and synthetic zeolites (having aluminosilicate framework) have been used for
exchanging Ca2+ and Mg2+ ions in hard with Na+ ions. For example, for the synthetic zeolite permutit, we have
Na6(Al6Si6(OH)12O18) + 3CaSO4 = 3Na2SO4 + Ca3(Al6Si6(OH)12O18)
If the calcium zeolite is treated with a solution of sodium chloride, the action begins in reverse, and the original
sodium zeolite is regenerated in a form ready to be used again.
(iv) Using Ion Exchange Resins Giant organic molecules having acidic or basic groups are known as ion-exchage
resin. Acid resins contain the acid group – COOH. Such resins exchange their H+ ions with cations Ca2+ and Mg2+
present in hard water.
2 RCOO– H+ + Ca2+ Æ (RCOO)2 Ca + 2H+
acid resin or
base-exchange resin
Study of the p-Block Elements (Groups 16, 17 and 18) 16.7

Basic resins exchange their OH– ions with anions HCO–3, Cl–, and SO2–
4 present in hard water.

4 Æ (RNH3)2 SO4 + 2OH


RNH+3 OH– + SO2– –

base resin or
acid-exchange resin

Hard water is passed successively through a cation exchange and anion exchange resins. Cation-exchange resin
makes water acidic due to the release of H+ and anion-exchange makes water alkaline due to the release of OH–.
Liberated H+ and OH– combine to give neutral water.
The exhausted cation and anion exchange resins are regenerated by treating them with dilute acid and alkali solutions,
respectively.
Heavy Water Heavy water is D2O. It may be obtained by
exhaustive electrolysis of water. It is used as moderator in nuclear
H H
reactions and also in the preparation of deuterium compounds. 95.0 pm 98.8 pm
147.5 pm 145.8 pm 90.2°
Peroxides and Polysulphides Oxygen and to a greater extent 111.5°
sulphur form polyoxides and polysulphides, which are less stable 94.8° H 101.9° H
than the normal salts. Examples are H2O2, H2S2, H2S3, H2S4, etc.
The higher elements do not form such compounds. (a) (b)
H2O2 and H2S2 can be prepared by the addition of acid to a Fig. 5 Nonplanar Dihydroxyl Structure of Hydrogen
peroxide or a persulphide salt. Peroxide in (a) Gaseous Phase and (b)
BaO2 + H2SO4 Æ BaSO4 + H2O2 Crystalline Phase
Na2S2 + H2SO4 Æ Na2SO4 + H2S2
The structure of H2O2 molecules is as shown in Fig. 5.
H2O2 is a strong oxidizing agent. In acidic solutions, oxidizing action is slow while in alkaline medium it is fast.
Examples exhibiting oxidizing action are: Fe2+ to Fe3+, [Fe(CN)6]4– to [Fe(CN)6]3–, SO2– 2– –
3 to SO 4, I to I2 and NH2OH
to HNO3.
With a stronger oxidizing agent such KMnO4, KIO4, O3, H2O2 acts as a reducing agent. The reactions exhibiting
oxidizing and reducing agents are
H2O2 + 2H+ + 2e– Æ 2H2O E° = 1.77 V
H2O2 Æ O2 + 2H+ + 2e–
H2O2 is a more powerful oxidizing agent in acidic medium as compared to alkaline medium. On the other hand, H2O2
is a more powerful reducing agent in alkaline medium as compared to acidic medium. These facts may be explained
based on the above equations or on the basis of effects caused on the reduction potentials. For oxidizing action,
RT 1
E = E° - ln +
2F ([H ] / c∞) ([H 2 O 2 ] / c∞)
2

E becomes more positive on increasing the concentration of H+, hence, its reduction tendency or causing oxidation
of some other species is enhanced.
For reducing action, E becomes more negative on decreasing the concentration of H+, hence, its reduction tendency
or causing oxidation of some other species is lowered. Conversely, its oxidation tendency or causing reduction of some
other species in enhanced.
Kinetically, oxidation with H2O2 is slow in acidic medium but proceeds rapidly in alkaline medium.
Halides
All the elements of Group 16 react with halogens to form several types of binary compounds such as MX6, MX4, MX2,
M2X2 and M2X (Table 3).
16.8 Complete Chemistry—JEE Main

Table 3 Binary Compounds of Group 16 Elements with Halogens


MX6 MX4 MX2 M 2X 2 M 2X Others
O OF2 O2F2 ClO2 O3F2, O4F2
Cl2O Cl2O6
Cl2O7
Br2O BrO2 BrO3
I2O4, I2O5,

I4O9
S SF6 SF4 SF2 S 2F 2
SCl4 SCl2 S2Cl2
S2Br2
Se SeF6 SeF4
SeCl4 Se2Cl2
SeBr4 Se2Br2
Te TeF6 TeF4
TeCl4 TeCl2
TeBr4 TeBr2
TeI4
Po PoCl4 PoCl2
PoBr4 PoBr2
PoI4

Compounds of Oxygen

oxides.

3. In more common compounds with other halogens, oxygen has oxidation states of –2 and –1/2.
Compounds with Other Elements
1. Fluorine is the most electronegative element. It is able to bring out the maximum valency of six in S, Se and
Te.
2. Other halogens are able to bring out the maximum valency of four in S, Se, Te and Po.
3. On descending the group, the higher oxidation-state halides are more stable than those of the lower oxidation
states.
3 2
d hybridization of the central atom.
5. The structures of tetrahalides are trigonal bipyramidal with one of the equatorial
Cl
positions occupied by a lone pair of electrons.
6 is extremely Cl
stable compound. It is used as a gaseous dielectric (insulator) in high voltage
transformers. Te
SeF6 is slightly reactive whereas TeF6 is more reactive. The latter is easily
hydrolyzed as Cl
TeF6 + 6H2O Æ 6HF + H6TeO6 Cl
7. Tetrahalides are more reactive than hexahalides but are more stable than the Structure of tetrahalides
lower halides. Tetrahalides are very sensitive to water.
SF4 + 2H2O Æ SO2 + 4HF
Study of the p-Block Elements (Groups 16, 17 and 18) 16.9

8. The dihalides form angular molecules, based on a tetrahedron with two positions
occupied by lone pairs. The lone pairs distort the tetrahedral angle of 109°28¢ to
103° in SCl2, 101.5° in F2O and 98° in TeBr2. The tetrahedral structure indicates
the sp3 hybridization of the central atom.
9. The structure of monohalides (M2X2) is similar to that of H2O2. S

Oxides Cl Cl
Practically, all of the elements react with oxygen to form oxides. Based on their structures,

Normal Oxides In these oxides, the number of oxygen atoms in a molecule is in


accordance with the oxidation state of the other element. Examples are H2O, MgO, Al2O3, etc. Structure of dihalide

Peroxides In these oxides, the number of oxygen atoms is more than the expected number determined from the
oxidation number of other elements. All of them contain —O—O— linkage attached ionically or covalently to the
other element.
Ionically bound — Na2O2, K2O2, BaO2
Covalently bound — H2O2, H2SO5 (peroxomonosulphuric acid)
Peroxo compounds are strong oxidizing agents, and are hydrolyzed by water to give H2O2.
H2SO5 + H2O Æ H2SO4 + H2O2
Suboxides In these oxides, the number of oxygen atoms in a molecule is less than the expected number determined
from the oxidation number of other element. They involve M—M bonds in addition to M—O bonds.
Example O == C == C == C == O.

Basic Oxides The oxides of metals are generally basic Examples are Na2O, MgO, CrO, etc.
The ionic basic oxides have high lattice energy. So, their melting points are high.
If a metal can exist in more than one oxidation state, the oxide with the lowest oxidation state is the most ionic and
the most basic. The basicity decreases (or acidity increases) with increase in the oxidation state of element.
Examples CrO is basic, Cr2O3 is amphoteric and CrO3 is acidic.
Amphoteric Oxides Oxides which can react with both acids and bases are amphoteric oxides.
Examples BeO, Al2O3, GaO3, SnO, PbO and ZnO.
Acidic Oxides The oxides of nonmetals are generally acidic.
Examples CO2, NO, SO2, Cl2O, SO3, P2O5
Acidic oxides have low melting and boiling points.
In cases where the element can exist in more than one oxidation state, the oxide of the highest oxidation state is the
most acidic.
Examples N2O5 is more acidic than N2O3
SO3 is more acidic than SO2.
Neutral Oxides A few covalent oxides are neither acidic nor basic. These are said to be neutral oxides
Examples N2O, NO, CO
Oxides of Higher Elements of the Same Group The more common oxides of the higher elements of group 16 are
as follows.
Sulphur: SO2, SO3, S2O
Selenium: SeO2, SeO3
Tellurium: TeO, TeO2, TeO3
Polonium: PoO, PoO2
The structures of SO2 and SO3 are
16.10 Complete Chemistry—JEE Main

S S
O O O O
SO3 in gaseous phase
SO2 is a strong reducing agent: SO2 + 2H2O Æ SO42– + 4H+ + 2e–
Its reducing characteristics is enhanced in alkaline medium.
At room temperature, SO3 is solid and exists in three distinct forms, namely, a-SO3, b-SO3 and g-SO3. g-SO3 has a
cyclic structure, (SO3)3.
O O
O
S
O O
O O S
O O i.e. O
S S O
O
O S S
O O
O
O
O
b-SO3 4 ] units each sharing two corners.
O O
S
O O
O
O O O S O O
O S
S O S O S O i.e.
O O
O O O O
S
O
O

a-SO3 is made up of chaions of cross-linked sheets.


Oxoacids

1. Sulphurous Acid Series


HO
Sulphurous acid (H2SO3) S O
HO
O O

Di-or pyrosulphurous acid (H2S2O5) HO S S OH

O O

Dithionous acid (H2S2O4) HO S S OH


Study of the p-Block Elements (Groups 16, 17 and 18) 16.11

Comments
1. These acids contain one sulphur atom in +3 oxidation state.
2. Sulphurous acid is mostly present as SO2 2O.
3. Sulphurous acid form salts known as sulphites (SO32– ).
4. The structure of SO2–
3 ion is pyramidal, i.e. tetrahedral with one position occupied by a lone pair.

bond length S—O 151 pm


S bond angle O—S—O 106°
p bond is delocalized.
O O

O
5. Free di- or pyrosulphurous acid does not exist.
6. The salts of disulphrous acid are known as disulphites. These are also known as pyrosulphites and metasulphites.
These salts contain an S—S linkage.
7. Dithionous acid does not exist in nature.
8. The salts of dithionous acid are known as dithionites.
2. Sulphuric Acid Series
O

Sulphuric acid (H2SO4) HO S OH

O
O

Thiosulphuric acid (H2S2O3) HO S OH

S
O O

Di- or pyrosulphuric acid (H2S2O7) HO S O S OH

O O
Comments
1. The oxidation state of sulphur atom in H2SO4 is +6. In H2S2O3, the oxidation states are +6 and –2.
2. The salts of sulphuric acid are known as sulphates. The structure of sulphate is

tetrahedral structure, bond length S—O 149 pm.


S
The two p bonds are delocalized over the S and the four O atoms.
O O-

O-

3. The salts of thiosulphuric acid are known as thiosulphates.


4. The structure of thiosulphates is
16.12 Complete Chemistry—JEE Main

S
O O-

O-
5. Thiosulphuric acid cannot be formed by adding acid to thiosulphates because the free acid decompose in water
in a mixture of S, H2S, H2Sn, SO2 and H2SO4. It can be prepared in the absence of water at low temperatures.
Sulphuric Acid
Sulphuric acid is one of the most important industrial chemicals. Commercially, this is manufactured by the contact
process. Sulphur dioxide obtained from the burning of sulphur or sulphide ores are made to combine with O2 to give
SO3 where V2O5 is used as a catalyst. This is followed by the absorption of SO2 in H2SO4 to give oleum (H2S2O7).
S(s) + O2(g) æDæÆ SO2(g)
V2 O5
2SO2(g) + O2(g) æææ Æ 2SO3
SO3 + H2SO4 Æ H2S2O7
The conversion of SO2 to SO3 is an exothermic process and is accompanied with decrease in volume. According to
Le-Chatelier principle, low temperature and high pressure help increasing the production of SO3. A pressure of 2 bar
and a temperature of 720 K are used in contact process.
Dilution of oleum with water give sulphuric acid of desired concentration.
Sulphuric acid ionizes as
H2SO4 + H2O Æ H3O+ + HSO4– (a strong acid)
HSO4– + H2O Æ H3O+ + SO42– K°a2 = 1.2 ¥ 10–2
Concentrated H2SO4 is a strong dehydrating agent.
Hot concentrated H2SO4 acts as a moderately strong oxidizing agent:
Cu + 2H2SO4 Æ CuSO4 + SO2 + 2H2O
(conc.)
S + 2H2SO4 Æ 3SO2 + 2H2O
(conc.)
C + 2H2SO4 Æ CO2 + 2SO2 + 2H2O
(conc.)
3. Thionic Acid Series
O O

Dithionic acid (H2S2O6) HO S S OH

S S

O O

Polythionic acid (H2Sn+2O6) HO S (S)n S OH

O O

Comments
1. Dithionic acid is a dibasic acid. It does not exist in free state. However, its salts are stables and are known as
dithionates.
2. The average oxidation state of S in dithionic acid is +5.
Study of the p-Block Elements (Groups 16, 17 and 18) 16.13

3. The dithionate ion has a structure similar to that of ethane in eclipsed conformation.
The bond length S—S is 215 pm
The bond length S—O is 143 pm
The bond angle S—S—O is close to 103°.
4. The salts of polythionic acids are trithionate (S3O62–), tetrathionate (S4O62–), pentathionate (S5O62–) and
hexathionate (S6O62–).
4. Perosoacid Series

Peroxomonosulphuric acid (H2SO5) H O O S OH

O
O O

Peroxodisulphuric acid (H2S2O8) HO S O O S OH

O O

Comments
1. Peroxoacid contains —O—O— linkage.
2. The oxidation state of S in peroxoacid is +6.
3. Peroxodisulphuric acid is a obtained by electrolysis of sulphates at high current density.
4. Peroxodisulphuric acid is a powerful oxidizing agents. It converts Mn2+ to MnO4– and Cr3+ to Cr2O72–
5. Peroxomonosulphuric acid is known as Caro’s acid. It may be obtained by the hydrolysis of peroxodisulphuric
acid.
O O O O

HO S O O S OH HO S O OH + HO S OH

O O O O
Anomalous Behaviour of Oxygen
Oxygen behaves differently from other elements in many respects. This is due to its smaller size, higher electronegativity,
and the absence of d orbitals. Two of the anomalies may be mentioned here.
1. Oxygen forms strong hydrogen bonds in the compounds hydrogen attached to oxygen.
2. Oxygen has a tendency to form strong double bonds. The other elements also form double bonds. But they
become weaker on descending the group. Thus
O == C == O is a stable species
S == C == S is less stable
CSe2 polymerizes instead of forming double bonds
CTe2 is unknown.

MULTIPLE CHOICE QUESTIONS

1. Which of the following elements does not belong to Group 16?


(a) Oxygen (b) Sulphur (c) Selenium (d) Bismuth
2. Atomic number of tellurium which belongs to 5th period is
(a) 50 (b) 51 (c) 52 (d) 84
3. The number of elements in Group 16 is
(a) 3 (b) 4 (c) 5 (d) 6
16.14 Complete Chemistry—JEE Main

4. Which of the following orders of melting points of hydrides of Group 16 elements is true?
(a) H2S > H2Se > H2Te (b) H2S < H2Se < H2Te (c) H2S > H2Se < H2Te (d) H2S < H2Se > H2Te
5. Which of the elements of Group 16 has the maximum tendency of catenation?
(a) Oxygen (b) Sulphur (c) Selenium (d) Tellurium
6. Which of the following triatomic molecules does not exist?
(a) CO2 (b) CS2 (c) CSe2 (d) CTe2
7. Which of the allotropes of sulphur represents a-sulphur?
(a) Rhombic sulphur (b) Monoclinic sulphur (c) Plastic sulphur (d) Colloidal sulphur
8. Which of the following allotropes represents prismatic sulphur?
(a) Rhombic sulphur (b) Monoclinic sulphur (c) Plastic sulphur (d) Colloidal sulphur
9. Which of the following hydrides of Group 16 has the lowest boiling point?
(a) H2O (b) H2S (c) H2Se (d) H2Te
10. Which of the following statements is not correct?
(a) The stability of hydrides of elements of Group 16 increases down the group
(b) Oxygen cannot display oxidation states of +2, +4 and +6
(c) Oxygen exists as diatomic gaseous molecules while other elements of the Group 16 catenate to form chains
or rings
(d) Rhombic sulphur is stable at room temperature
11. Which of the following compounds undergoes hydrolysis?
(a) SF6 (b) SCl6 (c) SeF6 (d) TeF6
12. Which of the following substances is used as the gaseous dielectric in high voltage transformers?
(a) SF4 (b) SF6 (c) SCl6 (d) SeF6
13. Sulphur trioxide exists
(a) in two polymorphic forms (b) in three polymorphic forms
(c) in four polymorphic forms (d) only in one polymorphic form
14. The structure of SF4 is
(a) square planar (b) tetrahedral (c) trigonal bipyramidal (d) octahedral
15. Which of the following acids does not exist in the free form?
(a) H2SO2 (b) H2S2O3 (c) H2S2O7 (d) H2SO4
16. Which of the acids does not include S—S bond?
(a) Dithionous acid, H2S2O4 (b) Dithionic acid, H2S2O6
(c) Polythionic acid, H2Sn+2O6 (d)Pyrosulphuric acid, H2S2O7
17. Which of the following acids exists in free form?
(a) H2SO2 (b) H2SO3 (c) H2S2O3 (d) H2S2O4
18. Which of the following formulae represents peroxodisulphuric acid?
O O O O
(a) H O O S O S OH (b) H O O S O S O O H
O O O O
O O O O
(c) H O S O O S OH (d) H O O S S OH
O O O O
19. The molecular formula of pyrosulphuric acid is
(a) H2S2O3 (b) H2S2O4 (c) H2S2O6 (d) H2S2O7
20. The molecular formula of dithionous acid is
(a) H2S2O3 (b) H2S2O4 (c) H2S2O6 (d) H2S2O7
21. The molecular formula of dithionic acid is
(a) H2S2O3 (b) H2S2O4 (c) H2S2O6 (d) H2S2O7
Study of the p-Block Elements (Groups 16, 17 and 18) 16.15

22. The molecular formula of thiosulphuric acid is


(a) H2S2O3 (b) H2S2O4 (c) H2S2O6 (d) H2S2O7
23. The oxidation state of sulphur in peroxodisulphuric acid, H2S2O8, is
(a) + 5 (b) + 6 (c) + 7 (d) + 8
24. The oxidation state of sulphur in peroxomonosulphuric acid, H2SO5, is
(a) + 5 (b) + 6 (c) + 7 (d) + 8
25. Which of the following oxo-acids of sulphur has more than one oxidation state of sulphur?
(a) H2S2O3 (b) H2S2O4 (c) H2S2O6 (d) H2S2O8
26. Which of the following oxo-acids of sulphur has more than one oxidation state of oxygen?
(a) H2S2O3 (b) H2S2O4 (c) H2S2O6 (d) H2S2O8
27. When pure H2SO4 is electrolysed, the product at the anode is
(a) H2S2O7 (b) H2S2O8 (c) H2S2O3 (d) H2S4O6
28. When aqua regia dissolves HgS, the function of the HCl in the mixture is to
(a) oxidize the sulphur (b) oxidize the mercury (c) complex the sulphur (d) complex the mercury
29. In the manufacture of sulphuric acid by the contact process, SO3 is not added to water directly to form H2SO4
because
(a) the reaction does not go to completion (b) the reaction is exothermic
(c) the reaction is quite slow (d) SO3 is insoluble in water
30. The catalyst used in the contact process is
(a) Fe2O3 (b) V2O5 (c) SO3 (d) Ag2O
31. Of the following acids, the strongest is
(a) H2S (b) He2Se (c) H2Te (d) H2O
32. Pyrosulphuric acid may be synthesized by dissolving SO3 in
(a) H2O (b) H2SO3 (c) H2SO4 (d) H2S2O8
33. The stable form of sulphur at room temperature and normal atmospheric pressure is
(a) orthorhombic (b) monoclinic (c) hexagonal (d) amorphous
34. The hybridization of sulphur in SO2 is
(a) sp (b) sp2 (c) sp3 (d) dsp2
35. Earth is protected from UV radiations by a layer of
(a) N2 (b) O2 (c) O3 (d) CO2
36. H2S can be distinguished from SO2 by its action on
2Cr2O7 (d) lead acetate solution
37. Marshall’s acid is
(a) H2SO5 (b) H2S2O5 (c) H2S2O7 (d) H2S2O8
38. Caro’s acid is
(a) H2SO5 (b) H2S2O5 (c) H2S2O7 (d) H2S2O8
39. Oxygen atoms in H2O2 has
(a) sp hybrid orbitals (b) sp2 hybrid orbitals (c) sp3 hybrid orbitals (d) pure p orbitals
40. Which of the following is not true about H2O2?
(a) It is used as a bleaching agent (b) It is used as a mild antiseptic
(c) It is used as a rockel fuel (d) It is used for the manufacture of heavy water
41. The oxide that gives H2O2 on treating with a dilute acid is
(a) PbO (b) Na2O2 (c) MnO2 (d) TiO2
42. Which of the following catalysts is used in the lead chamber process?
(a) Pt (b) V2O5 (c) Oxides of nitrogen (d) Ni
43. Which of the following bonds has the highest bond energy?
(a) O—O (b) S—S (c) Se—Se (d) Te—Te
44. Which of the following is the weakest acid?
(a) H2S (b) H2Se (c) H2O (d) H2Te
16.16 Complete Chemistry—JEE Main

45. Which of the oxidation state(s) is/are shown by the element sulphur?
(a) –2 only (b) +2 only (c) –2, +2 and +4 (d) –2, +2, +4 and +6
46. Which of the following is not the mineral of sulphur?
(a) Chloroapatite (b) Fool’s gold (c) Sphalerite (d) Chalcopyrite
47. Which of the following statements regarding sulphur is not true?
(a) Rhombic sulphur is stable at room temperature
(b) Monoclinic sulphur is stable at room temperature
(c) Both rhombic and monoclinic sulphur are soluble in CS2
(d) Both rhombic and monoclinic sulphur have a puckered-ring structure of eight sulphur atoms
48. Which allotropic form of sulphur is obtained on passing H2S gas through nitric acid?
(a) Rhombic sulphur (b) Monoclinic sulphur (c) Plastic sulphur (d) Colloidal sulphur
49. Which of the following equations represents the oxidising action of sulphur dioxide?
(a) 2Fe3+ + SO2 + 2H2O 2Fe2+ + SO42– + 4H +
(b) 3Fe + SO2 2FeO + FeS

(c) 2MnO4 + 2H2O + 5SO2 2Mn2+ + 4H + + 5SO42–
(d) Cr2O72– + 2H+ + 3SO2 2Cr3+ + 3SO42– + H2O
50. The most favourable conditions for the formation of SO3 via the equation

SO2 + 1
2
O2 SO3 DH = – 95 kJ mol –1
are
(a) low temperature and low pressure (b) low temperature and high pressure
(c) high temperature and low pressure (d) high temperature and high pressure
51. Which of the following statements regarding sulphur dioxide is not true?
(a) SO2 is an angular molecule
(b) SO2 may be regarded as the resonance hybrid of two canonical structures
(c) SO2 is an anhydride of sulphuric acid
(d) SO2 is an anhydride of sulphrous acid
52. Which of the following statements regarding sulphur trioxide is not true?
(a) Sulphur trioxide exists in three polymorphic forms
(b) Sulphur trioxide is an anhydride of sulphuric acid
(c) Vanadium pentoxide is used as a catalyst in converting sulphur dioxide into sulphur trioxide
(d) sulphur trioxide has tetrahedral geometry
53. The elemental sulphur is written as
(a) S (b) S2 (c) S4 (d) S8
54. Which of the following statements is true?
(a) Both rhombic and monoclinic sulphur are soluble in water
(b) Both rhombic and monoclinic sulphur are soluble in carbon disulphide
(c) Both rhombic and monoclinic sulphur are insoluble in carbon disulphide
(d) Rhombic sulphur can be converted into monoclinic sulphur but the reverse is not possible
55. Colloidal sulphur is obtained when
(a) sulphur is heated in absence of air (b) sulphur is heated in presence of air
(c) sodium thiosulphate is treated with iodine (d) H2S is passed through water containing HNO3
56. Which of the following forms of sulphur is stable at room temperature and 1 atmospheric pressure?
(a) Amorphous (b) Hexagonal (c) Monoclinic (d) Orthorhombic
57. Burning sulphur in air produces
(a) H2S (b) H2S2 (c) SO2 (d) SO3
Study of the p-Block Elements (Groups 16, 17 and 18) 16.17

58. Sulphur dioxide is a strong reducing agent. However, it can also act as an oxidizing agent. Which of the following
reactions shows its oxidizing nature?
4 solution
(c) Reaction with H2
59. Which of the following statements is not true of sulphur?
(a) Sulphur belongs to Group VIA of the periodic table
(b) Sulphur exhibits allotropy
(c) Sulphur occurs both in the native form and combined form
(d) Sulphur is soluble in water
60. Which of the following gases is used for the qualitative analysis of metal ions?
(a) CO2 (b) H2S (c) SO2 (d) SO3
61. Which of the following is an anhydride of H2SO4?
(a) H2S (b) H2S2 (c) SO2 (d) SO3
62. Which of the following is an anhydride of H2SO3?
(a) H2S (b) H2S2 (c) SO2 (d) SO3
63. Which of the following is not true for SO2?
(a) SO2 is obtained by roasting metal sulphides (b) SO2 is acidic in nature
(c) SO2 is used as a disinfectant(d)SO2 is an anhydride of H2SO4
64. Which of the following statements regarding sulphuric acid is not true?
(a) Concentrated H2SO4 acts as a dehydrating agent
(b) Hot concentrated H2SO4 acts as a powerful oxidizing agent
(c) The reaction of PCl5 with sulphuric acid produces sulphuryl chloride
(d) The structure of sulphuric acid in vapour phase is square planar
65. The formula of sodium thiosulphate is
(a) Na2S2O3 · H2O (b) Na2S2O3 · 3H2O (c) Na2S2O3 · 5H2O (d) Na2S2O3 · 7H2O
66. Which of the following statements for sodium thiosulphate is not true?
(a) Sodium thiosulphate is used as an antichlor in the textile industry

(c) Sodium thiosulphate acts as an oxidizing agent in its reaction with iodine
(d) The structure of S2O 32– is tetrahedral
67. The oxidation number of sulphur in sodium thiosulphate is
(a) 0 (b) + 1 (c) + 2 (d) + 4
68. The dissolution of AgBr in S2O32– produces
(a) Ag(S2O3)– (b) Ag(S2O3)3–
2 (c) Ag(S2O3)5–3 (d) Ag(S2O3)33–
69. Which of the following statements regarding ozone is not true?
(a) Ozone is an allotrope of oxygen
(b) The ozone layer protects the earth’s surface from an excessive concentration of harmful ultraviolet radiation
(c) The conversion of oxygen into ozone is an exothermic process
(d) Ozone is much more powerful oxidizing agent than molecular oxygen
70. Which of the following statements regarding ozone is not correct?
(a) The ozone molecule is angular in shape
(b) The ozone is a resonance hybrid of the two structures
(c) The oxygen-oxygen bond length in ozone is identical with that of molecular oxygen

71. Which of the following oxidation state of oxygen is not observed in its compounds
(a) –1/2 (b) –1 (c) +1 (d) +2
72. Which of the following facts regarding bond energies is correct?
(a) eO=O < eS=S (b) eO – O < eS –S (c) eO–H < eS–H (d) eO – C < eS –C
16.18 Complete Chemistry—JEE Main

10
73. The structure of TeCl4 5s25p6)
Cl Cl
Cl Cl Cl Cl
Cl
Te Te Te
(a) Te (b) (c) (d)
Cl Cl Cl Cl Cl
Cl Cl

Cl Cl
74. The hybridization of S in SF4 is
(a) dsp2 (b) sp3 (c) dsp3 (d) sp3d
75. Silver bromide is soluble in sodium thiosulphate due to the formation of the complex
(a) Na[Ag(S2O3)] (b) Na3[Ag(S2O3)2] (c) Na5[Ag(S2O3)3] (d) Na7[Ag(S2O3)4]
76. The structure of SO23– is
2– 2– 2–
O O O O
S
(a) O S (b) S (c) (d)
O O
O O
77. Given are the standard reduction potentials of a some species
SO2 æ0ææ.51 V
Æ S4O2– 0.084 V
6 æææÆ S2O3 æææ
2– 0.44 V
Æ S8

The value of E° will be about
(a) 0.44 V (b) 0.54 V (c) 0.65 V (d) 0.76 V
78. Which of the following element will exhibit maximum tendency of forming pp–dp bonding in their oxoanions?
(a) Si (b) P (c) S (d) Cl
79. Which of the following elements has a strongest tendency to form double bonds?
(a) S (b) Se (c) P (d) Si
80. Which of the following oxide has acidic characteristics?
(a) N2O (b) NO (c) CO (d) N2O3

ANSWERS
1. (d) 2. (c) 3. (c) 4. (b) 5. (b) 6. (d)
7. (a) 8. (b) 9. (b) 10. (a) 11. (d) 12. (b)
13. (b) 14. (c) 15. (a) 16. (d) 17. (c) 18. (c)
19. (d) 20. (b) 21. (c) 22. (a) 23. (b) 24. (b)
25. (a) 26. (d) 27. (b) 28. (d) 29. (c) 30. (b)
31. (c) 32. (c) 33. (a) 34. (b) 35. (c) 36. (d)
37. (d) 38. (a) 39. (c) 40. (d) 41. (b) 42. (c)
43. (b) 44. (c) 45. (d) 46. (a) 47. (b) 48. (d)
49. (b) 50. (b) 51. (c) 52. (d) 53. (d) 54. (b)
55. (d) 56. (d) 57. (c) 58. (c) 59. (d) 60. (b)
61. (d) 62. (c) 63. (d) 64. (d) 65. (c) 66. (c)
67. (c) 68. (b) 69. (c) 70. (c) 71. (c) 72. (b)
73. (c) 74. (d) 75. (c) 76. (c) 77. (a) 78. (d)
79. (a) 80. (d)
Study of the p-Block Elements (Groups 16, 17 and 18) 16.19

HINTS AND SOLUTIONS


1. Bismuth belongs to Group 15.
2. Atomic number of tellurium is 2 + 8 + 8 + 18 + 16 = 52.
6. The tendency of the elements of Group 16 to form multiple bonds to C, N and O decreases down the group. CO2
is very stable, CS2 is moderately stable, CSe2 decomposes readily and CTe2 does not exist.
9. Because of hydrogen bondings, the boiling point of H2O is greater than H2S. H2Se and H2Te have higher boiling
points than H2S because of heavier atoms Se and Te.
10. The stability of hydrides of elements of Group 16 decreases down the group.
11. TeF6 is hydrolysed: TeF6 + 6H2O H6TeO6 + 6HF
13. SO3 exists in three forms: a-, b-, and g-sulphur trioxide.
14. The hybridization of S in SF4 is sp3d. Hence, it is trigonal bipyramidal.
O O O O
16. H2S2O4 : H O S S OH ; H2Sn+2O6 HO S (S)n S OH
O O
O O O O
H2S2O6 : HO S S OH ; H2S2O7 HO S O S OH
O O O O
–2 O –2 O
–1 +6 –1 –1 +6 –1
23. Peroxodisulphuric acid is HO S O O S OH
–2 –2 O
O
The oxidation state of S is + 6.
–2
O
–1 +6 –1
24. Peroxomonosulphuric acid is HO O S OH
–2
O
–2
O
–1 +4 –1
25. Thiosulphuric acid (H2S2O3) is HO S OH
0
S
26. See Q. 23.
27. 2H2SO4 H2S2O8 + 2H + + 2e–.
31. Electronegativity decreases down the group. Hence, the hold of H in the bond H—M decreases down the group
as the atom M becomes more and more metallic.
32. H2SO4 + SO3 H 2S 2O 7.
40. Heavy water is obtained by the electrolysis of alkaline solution of water.
44. See Q. 31.
46. Fool’s gold is Fe3S2. Sphlaerite is ZnS. Chalcopyrite is CuFeS2.
47. Rhombic sulphur is stable at room temperature.
51. SO3 is the anhydride of sulphuric acid.
52. SO3 has a plane triangular structure.
64. The structure of H2SO4 is tetrahedral.
66. Sodium thiosulphate acts a reducing agent.
2S2O32– S4O2–
6 + 2e

I2 + 2e– 2I –
16.20 Complete Chemistry—JEE Main

69. The conversion of O2 to O3 is an endothermic process.


70. The bond length in ozone is intermediate between those of single O —O and double O==O bonds. The bond angle
is O3 is 117° with O —O distance 127.8 pm.
71. The oxidation state +1 is not observed in the compounds of oxygen.
72. Due to lone pair-lone pair repulsions on the neighbouring atoms, the single bond energies of O—O is smaller than
that of S—S bond.
73. The structure of choice c represents the structure of TeCl4.
74. sp3d hybridization in S is involved in SF4.
75. Solubility of AgBr in Na2S2O3 is due to the formation of the complex Na5[Ag(S2O3)3].
76. Sulphur in SO32– involves sp3 hybridization. There are a total of four pairs of electrons around S.
77. We have
4SO2 + 4H+ + 6e– Æ S4O2– 6 + 2H2O] ¥ 2 DG°1 = – 12F (0.44 V)
S4O–6 + 2e– Æ 2S2O32–] ¥ 2 DG°2 = – 4F (0.08 V)
4S2O2–
3 + 24H +16e Æ S8 + 12H2O
+ –
DG°3 = – 16F (0.47 V)
Add
8SO2 + 32H+ + 32e– Æ S8 + 16H2O DG°4 = – 32F E°
Hence 32 E° = (12 ¥ 0.44 + 4 ¥ 0.08 + 16 ¥ 0.47) V
E° = (14.12/32) V = 0.44 V

MULTIPLE CHOICE QUESTIONS FROM AIEEE AND JEE MAIN

1. Which one of the following processes will produce hard water?


(a) Addition of Na2SO4 to water (b) Saturation of water with CaCO3
(c) Saturation of water with MgCO3 (d) Saturation of water with CaSO4 [2003]
2. Which of the following statements regarding sulphur is incorrect?
(a) The oxidation state of sulphur is never less than +4 in its compounds.
(b) S2 molecule is paramagnetic.
(c) The vapour at 200 °C consists mainly of S8 ring.
(d) At 600 °C, the gas mainly consists of S2. [2011 cancelled]
3. Identify the incorrect statement from the following.
(a) Oxides of nitrogen in the atmosphere can cause the depletion of ozone layer
(b) Ozone absorbs the intense ultraviolet radiation of the sun

(d) Ozone absorbs infrared radiation [2011]


4. Which of the following is the wrong statement?
(a) ONCl and ONO– are not isoelectronic (b) O3 molecule is bent
(c) Ozone is violet-black in solid state (d) Ozone is diamagnetic gas [2013]
5. Which of the following statements about the depletion of ozone layer is correct ?
(a) The problem of ozone depletion is less serious at poles because NO2
consuming ClO radicals.
(b) The problem of ozone depletion is more serious at poles because ice crystals in the clouds over poles act as
catalyst for photochemical reactions involving the decomposition of ozone by Cl and ClO radicals.
(c)
(d) Oxides of nitrogen also do not react with ozone in stratosphere. [2014, online]
6. Consider the reaction: H2SO3(aq) + Sn4+(aq) + H2 2+
(aq) + 3H+(aq) + HSO–4
Study of the p-Block Elements (Groups 16, 17 and 18) 16.21

Which of the following statements is correct ?


(a) Sn4+ is the oxidizing agent because it undergoes oxidation
(b) Sn4+ is the reducing agent because it undergoes oxidation
(c) H2SO3 is the reducing agent because it undergoes oxidation
(d) H2SO3 is the reducing agent because it undergoes reduction [2014, online]
7. Photochemical smog consists of excessive amount of X, in addition to aldehydes, ketones, peroxy-acetyl nitrile
(PAN), and so forth, X is:
(a) CH4 (b) CO (c) CO2 (d) O3 [2015, online]
8. Identify the incorrect statement regarding heavy water:
(a) It reacts with SO3 to form deuterated sulphuric acid (D2SO4)
(d) It is used as a coolant in nuclear reactors
(c) It reacts with CaC2 to produce C2D2 and Ca(OD)2
(d) It reacts with Al4C3 to produce CD4 and Al(OD)3. [2016, online]
9. Identify the incorrect statement:
(a) The S — S — S bond angles in S8 and S6 rings are the same
(b) Rhombic and monoclinic sulphur have S8 molecule
(c) S2 is paramagnetic like oxygen
(d) S8 ring has a crown shape. [2016, online]
10. Which of the following statements about water is false?
(a) Water can act both as an acid and as a base.
(b) There is extensive intramolecular hydrogen bonding in the condensed phase
(c) ice formed by heavy water sinks in normal water
(d) Water is oxidized to oxygen during photo synthesis [2016]

ANSWERS
1. (c) 2. (a) 3. (d) 4. none 5. (b) 6. (c)
7. (d) 8. (b) 9. (a) 10. (b)

HINTS AND SOLUTIONS


2. The oxidation state of S in H2S is – 2, which is less than +4.
Hence, the choice (a) is incorrect.
3. Ozone does not absorb infrared radiations. Hence, the choice (d) is correct.
4. The number of electrons in ONCl and ONO– are :
ONCl 8 + 7 + 17 = 32 ONO– 8 + 7 + 8 + 1=24
Thus, these two species are not isoelectronic. Ozone is bent molecule, violet-black in colour in solid phase and
diamagnetic in nature. Thus, all the four choices are correct and none of them is incorrect.
6. The reaction along with the oxidation number of species undergone oxidation and reduction is as follows.

7. Photochemical smong also contains O3.


16.22 Complete Chemistry—JEE Main

235
U.
9. The bond angle in S8 and S6 are different. S8 adopts a puckered ring or “Crown” structure. S6 adopts a chair form.
10. Water involves intermolecular hydrogen bondings.

The Group 17 Elements

collectively known as halogens which means sea salts producers (from Greek, halos = sea salts, genes = born). Astatine
is a short-lived radioactive element. The physical properties of these elements are recorded in Table 10.2.
Table 2. Atomic Properties of the Halogens

Property F CI Br I
2 5 2 5 10 2 5
[He](2s) (2p) [Ne](3s) (3p) [Ar](3d) (4s) (4p) [Kr](4d) (5s)2(5p)5
10

Atomic number 9 17 35 53
Relative atomic mass 18.998 35.453 79.904 126.905
Covalent radius r/pm 72 99 114 133
Ionic radius, r/pm 133 184 196 220
Electronegativity 4.0 3.0 2.8 2.5
–1
Ionization energy, l/kJ mol 1681 1256 1143 1009
–1
E/kJ mol –333 –349 –325 –296

Description of Physical Properties


ns)2(np)5, where n varies from 2 to 6. They have only one
electron less than the number present in the adjacent noble gas. Thus, they show a strong tendency to complete

metals to form ionic compounds or by sharing one electron with nonmetals to form covalent compounds.
2. The covalent and ionic radii increase down the group (Fig. 6).
3. The electronegativity and ionization energy decrease down the group (Fig. 7). The value of ionization energy

220 Radii ionic


Ionization energies

1600

180

1400
IE/kJ mol-1

140
colvalent
r/pm

1200

100
1000

60 800
F Cl Br I F Cl Br I

Fig. 6 Covalent and Ionic Radii of the Elements of Group 17 Fig. 7 Ionization Energies of the Elements of Group 17
Study of the p-Block Elements (Groups 16, 17 and 18) 16.23

350 Electron affinities

than Cl. This is due to the more repulsion experienced by the incoming 330

EA/kJ mol-1
atom.
Molecular Properties 310

All the halogens form diatomic molecules. Table 3 records some of the molecular
properties of halogens.
290
F CI Br I

Fig. 8
Elements of Group 17

Table 3 Molecular Properties of the Halogens


Property F2 Cl2 Br2 I2
Melting point, T/K 54 172 266 387
Boiling point, T/K 85 239 333 458
Density, liquid state, r/g cm –3
1.513 1.655 3.187 3.960
(85 K) (203 K) (273 K) (493 K)
Colour pale yellow greenish yellow reddish brown violet
Bond enthalpy, e/kJ mol –1
158.8 242.6 192.8 151.1
Bond distance, l/pm 143 199 228 266
Enthalpy of vaporization, DvapH/kJ mol –1
6.54 20.41 29.56 41.95
Enthalpy of fusion, DfusH/kJ mol –1
0.51 6.41 10.57 15.52
E°/V 2.87 1.36 1.09 0.54

Description of Molecular Properties


1. The halogens are coloured beacuase they abosrb radiations in the visible region resulting in the excitation of
outer electrons to higher energy levels. The smaller the atom, the more will be the energy needed for excitation.
Gaseous F2 appears pale yellow as it absorbs high energy violet light.
Gaseous Cl2 appears greenish yellow.
Gaseous Br2 apperars reddish-brown.
Gaseous I2 appears violet as it absorbs low energy yellow light
2. The density of halogens increases down the group.
3. The melting and boiling points increase down the group (Fig. 9). From the data on melting and boiling points,
it follows that at room temperature F2 and Cl2 are gases Br2 exists in liquid form and I2 exists in solid form.
4. The bond enthalpy increases from F—F to Cl—Cl followed by a decrease (Fig. 10).
Fluorine has exceptionally low bond enthalpy. This may be attributed to the lone pair repulsion which is
maximum due to small size of atoms.
Cl2 has a maximum value of bond enthalpy. This is because Cl—Cl bond acquires a double bond character due
to the back bonding of p electrons on one chlorine atom to the empty d orbital of the second atom.
5. The bond distance increases down the group (Fig. 11).
6. The nonmetallic character of halogens decreases down the group. Iodine is associated with some metallic
character. It has a metallic lusture and can exist as positive ion in ICl or ICN

ICl   I+ + Cl–

ICN  +
 I + CN

16.24 Complete Chemistry—JEE Main

Melting and boiling points Bond enthalpies


500 240
b.pt

400 200
m.pt
Bond distances

300 200 250

ex-x/kJ mol-1
200 180 200
T/K

r/pm
100 160 150

0 140 100
F2 Cl2 Br2 I2 F2 Cl2 Br2 I2 F2 Cl2 Br2 I2

Fig. 9 Melting and Boiling Fig. 10 Bond Enthalpies of Fig. 11 Bond Distances of
Points of the Halogens the Halogens Halogens
Reactivity of Halogens

occurs as insoluble CaF2 3AIF6(cryolite) and Ca2(PO4


occur as chlorides and bromides in seawater and underground brines. Sodium chloride also occurs as rock salt.
Some sea weeds and sponges contain iodine as iodides. Chile saltpeter (NaNO3) contains 0.02–1% iodine in
the form of sodium iodate, NaIO3. The position of abundance of halogens and their concentrations in earth’s
crust (in ppm) are given in Table 4.
Table 4. Distribution of Halogens in Earth’s Crust
Halogen Position of abundance in earth’s crust Abundance in earth’s crust (ppm)
Fluorine 13 544
Chlorine 20 126
Bromine 46 2.5
Iodine 60 0.46
2. All the halogens exhibit –1 oxidation state.
3. Fluorine is the most electronegative element, it exhibits only –1 oxidation state.
4. The other halogens, besides exhibiting –1 oxidation state, also exhibit +1, +3, +5 and +7 oxidation states. This

states are as follows.


s p d
+1 oxidation state ≠Ø ≠Ø ≠Ø ≠

+3 oxidation state ≠Ø ≠Ø ≠ ≠ ≠

sp3d hybridization

+5 oxidation state ≠Ø ≠ ≠ ≠ ≠ ≠

sp3d2 hybridization

+7 oxidation state ≠ ≠ ≠ ≠ ≠ ≠ ≠

sp3d3 hybridization
5. The oxidizing ability of halogens follows the order F2 > Cl2 > Br2 > I2.
Study of the p-Block Elements (Groups 16, 17 and 18) 16.25

Fluorine has maximum oxidizing ability due to its low enthalpy of dissociation and maximum enthalpy of
hydration. This fact also follows from their reduction potentials.
The consequence of the above order is that a halogen of lower atomic number will oxidize halide ions of higher
atomic number
F2 + 2Cl– Æ 2F– + Cl2
Cl2 + 2Br– Æ 2C1– + Br2
Br2 + 2I– Æ 2Br– + I2
6. F2, being strongly oxidizing agent, it oxidizes water as
2F2 + 6H2O Æ 4H3O+ + 4F– + O2
A similar reaction is expected for chlorine, but the reaction is slow due to the high value of energy of activation.
However, chlorine in water undergoes rapidly disproportionation reaction
Cl2 + H2O   HCl + HOCl
The extent of disproportionation is still less for Br2 and almost negligible for I2.
I2 is the weakest oxidizing agent. In this case, the reverse reaction is spontaneous, i.e. I– in acidic medium is
oxidized by atmospheric oxygen
4I– + O2 + 4H+ Æ 2I2 + 2H2O
Compounds of Group 17 Elements
1. Hydrogen Halides
All halogen react with hydrogen to give hydrogen halides. The reactivity of halogens towards hydrogen decreases down
the group. Table 5 records some physical properties of hydrogen halides.
Table 5. Some Properties of Hydrogen Halides
HF HCl HBr HI
Melting point, T/K 190 158.9 186.1 222.2
Boiling point, T/K 293 188.1 206.4 237.6
–3
Density/g cm 0.99 1.19 2.16 2.80
pKa 3.2 –7 –9 –11
Azeotropic composition (mass percent) 35.37 20.24 47.0 57.0
–1
Bond enthalpy/kJ mol 566 431 366 299
Bond distance, l/pm 100 170
Description
1. The melting and boiling points decrease from HF to HCl followed by an increase (Fig. 12).
HF has exceptionally high melting and boiling points. This is attributed to the hydrogen bonds due to the very

and the H atom of another molecule. The links the molecules together as (HF)n and they form zig-zag chains
in both the liquid and the solid (Fig. 13).
300

260

b.pt
F F F
220
T/K

H H H H H H
m.pt
180 F F F F

140
HF HCI HBr HI

Fig. 12 Melting and Boiling Points of Hydrogen Halides Fig. 13 Hydrogen Bonding in HF
16.26 Complete Chemistry—JEE Main

The hydrogen bonding even persists in gaseous HF (and not in HCl, HBr, Bond enthalpies
HI) forming cyclic (HF)6, dimeric (HF)2, and monomeric HF.
2. In the gaseous state, all halides are covalent. In aqueous solutions they 550
ionize as follows.
HF + H2O   +
 H 3O + F

450
HX + H2O æÆ H3O+ + X–

e/kJ mol-1
(X == Cl, Br, I)
Thus, HF acts as a weak acid and HC1, HBr and HI behave as strong
350
acids. The aqueous solution of HX is known as hydrohalic acid or simply
halogen acid.
3. In glacial acetic acid medium, the order of acid strength is 250
HI > HBr > HCl > HF HF HCl HBr HI
This order is in agreement with the bond enthalpy data (Fig. 14). Fig. 14 Bond Enthalpies of Hydrogen
4. All halogen acids form azeotropic solutions with water. An azotropic Halides
solution has a constant boiling point.
Liquid HF has been used as a nonaqueous solvent. The mineral acids HNO3, H2SO4 and HCl behave as bases
in HF solvent. The compounds SbF5, AsF5 and BF3 act as acids. The self ionization of HF is

2HF  +
 [H2F] + F

2. Other Halides
Halogens react with almost all elements of the periodic table, except helium, neon and argon, to form a vast variety of

Ionic Halides Metals with low ionization energies (like alkali and alkaline earth metals) form ionic halides. When
a metal exhibits more than one oxidation state, the halides in the lower oxidation states have more ionic character. For
example, PbCl2 is ionic while PbCl4 is covalent. The solubilities of ionic halides usually increase from F– to Br– to Cl–
to I–. This is due to the decrease in the value of lattice energy as the ionic radii increase.
Covalent (or Molecular) Halides Metals with high ionization energies form covalent halides. When a metal exhibits
more than one oxidation state, the halides in the higher oxidation states have more covalent character. For example,
UF6 is covalent while UF4 is ionic.
Most of electronegative elements also form covalent halides. Examples are BCl3, PCl3, PCl5, CCl4, etc. A large
number of covalent halides formed from nonmetals are hydrolyzed in aqueous solutions.
BCl3 + 3H2O Æ H3BO3 + 3H+ + 3Cl– PCl3 + 3H2O Æ H3PO3 + 3H+ + 3Cl–
SiCl4 + 4H2O Æ Si(OH)4 + 4H + 4Cl
+ –
PCl5 + 4H2O Æ H3PO4 + 5H+ + 5Cl–
Most of molecular halides are gases or volatile liquids. This is because of the strong covalent bonds within the
molecules and different molecules are held together by weak van der Waals forces.
When a nonmetal, E, reacts with halogen, the bond strength decreases in the order
E—F > E—Cl > E—Br > E—I.
Bridging Halides In these halides, halogen forms bridge between two atoms. Examples are AlCl3, BeF2 and BeCl2.
The bridge is formed due to one normal covalent bond with one of the atoms and one coordinate covalent bond with
another atom. This coordinate bond is formed by sharing a lone pair of electrons on halogen with another atom. Both
the bridging bonds are completely identical.

3. Halogen Oxides

while the compounds formed by other elements are called halogen oxides. Table 6 describes the various compounds
formed between halogens and oxygen.
Comments
1. Because of small difference in electronegativity between the halogens and oxygen, the bonds are largely
covalent with the exception of I2O4 and I4O9 which are ionic.
Study of the p-Block Elements (Groups 16, 17 and 18) 16.27

2. Most of the halogen oxides are unstable, and tend to explode when subjected to shock or sometimes even when
exposed to light.
3. The stability of oxides increases down the group. Thus, the iodine oxides are the most stable, then the chlorine
oxides and then bromine oxides (which decompose below room temperature).
4. The oxides with the higher oxidation states of halogens are more stable than the lower states.
Table 6. Compounds of Halogens with Oxygen
Oxidation state of Oxygen Chlorine Bromine Iodine
halogen oxides oxides oxides
–1 OF2, O2F2 — — —
+1 — Cl2O Br2O —
+3 — Cl2O3 —
+4 — ClO2 BrO2 I 2O 4
+5 — — — I 2O 5
+6 — Cl2O6 — —
+7 O 4F 2 Cl2O7 — I 4O 9

A Brief Description of Important Oxides

2F2 + 2NaOH Æ 2NaF + OF2 + H2O.


It is a colourless gas and is a strong oxidizing agent. It has been used as a rocket fuel.
Dichlorine Monoxide This is prepared by heating freshly precipitated (yellow) mercuric oxide with halogen gas
diluted with dry air.
575 K
2Cl2 + 2HgO æææÆ HgCl2 ◊ HgO + Cl2O
It is a yellow-brown gas which explodes in the presence of reducing agents or NH3, on heating.
3Cl2O + 10NH3 Æ 6NH4Cl + 2N2 +3H2O
C12O is acidic anhydride of hypochlorous acid.
Cl2O + H2O   2HOCl
Structures of F2O and Cl2O The Lewis dot structures of F2O and C12O (valence electrons: 20) are

There are four unpaired electrons around oxygen, so the geometry adopted by these electrons is tetrahedral as
predicted from VSEPR theory. The same structures are predicted from the valence-bond theory.

s p
Electronic structure of oxygen atom in free state ≠Ø ≠Ø ≠ ≠

Electronic structure of oxygen atom in F2O or Cl2O ≠Ø ≠Ø ≠ ≠



sp3 hybridization, tetrahedral geometry

Thus, in F2O and Cl2O two positions of a tetrahedron is occupied by halogens and the rest two
positions by lone pairs.
The expected bond angle X—O—X is 109°28¢. But, the observed values are O
F—O—F 103°
Cl—O—Cl 111° X
The decrease in bond angle in F2O has been attributed to the repulsion between the lone pairs.
X
The increase in bond angle in Cl2O has been attributed to the removal of steric crowding of the
larger chlorine atoms.
16.28 Complete Chemistry—JEE Main

Chlorine Dioxide This is most important oxide of chlorine. It is a yellow gas. Being highly reactive,
ClO2 is prepared in situ and is used after diluting with air or CO2.
Laboratory Preparation
H2O
2NaClO3 + 2(COOH)2 æ90 ææ∞C
Æ 2ClO2 + 2CO2 + (COONa)2 + 2H2O.
Commercial Preparation
trace of
2NaClO3 + SO2 + H2SO4 ææææ
NaCl
Æ 2ClO2 + 2NaHSO4

2HClO3 + 2HC1 Æ 2ClO2 + Cl2 + 2H2O.


ClO2 is a powerful oxidizing, bleaching and chlorinating agent. It is used for the

The ClO2 is an odd electron molecule and is thus paramagnetic. The odd electron
is delocalized over the entire molecule and thus ClO2 does not show any tendency for
dimerization.
The bond length Cl—O is 147 pm and it is shorter than for a single bond. This indicates
the delocalization of odd electron on the entire molecule.
Dichlorine Hexoxide This oxide exists in equilibrium with ClO3.
Cl2O6   2ClO3
Cl2O6 is made from ClO2 and O3. It is a strong oxidizing agent. It undergoes hydrolysis as shown in the following
reactions.
Cl2O6 + H2O Æ HClO3 + HClO4
Cl2O6 + 2NaOH Æ NaClO3 + NaClO4 + H2O
Its structure is not known. Probable structures are
O O O O O
O Cl Cl O or Cl Cl or ClO+2 ClO-4
O O O O O
Dichlorine Heptoxide It is obtained by dehydration of perchloric acid with P4O10.
PO
2HClO4 æ-ææ
2 5
H O
Æ Cl2O7.
2

O O 141 pm
O O 171 pm O
Cl 119° Cl 115°

O O
Oxides of Bromine Two oxides are Br2O and BrO2.
Br2O does not give HOBr on treating with water, but with NaOH it gives OBr–. It is a strong oxidizing agent, and
oxidizes I2 to I2O5.
BrO2 is hydrolyzed in alkaline solution giving Br– and BrO –3 ions.
Oxides of Iodine Iodine form stable oxides with oxygen. I2O5 is an anhydride of iodic acid.
I2O5 + H2O Æ 2HIO3.
It is an oxidizing agent. It is used to estimate carbon monoxide quantitatively based on the following reaction.
I2O5 + 5CO Æ 5CO2 + I2
The liberated iodine is determined by titrating against sodium thiosulphate solution.
I2O5 also oxidizes H2S to SO2 and NO to NO2 2, BrF3 or SF4 it forms IF5
2I2O5 + 10F2 Æ 4IF5 + 5O2
O O
The structure of I2O5 is I O I
O O
The other oxides I2O4 and I4O9 are less stable. Their probable structures are IO+◊IO3– and I3+◊(IO3– )3, respectively.
Study of the p-Block Elements (Groups 16, 17 and 18) 16.29

4. Oxoacids of Halogens
Halogens form four series of oxoacids of the type HOX (hypohalous acid), HXO2 (halous acid), HXO3 (halic acid) and
HXO4(perhalic acid). Table 7 records the oxoacids formed by halogens.
Table 7 Oxoacids Formed by Halogens
HOX HXO2 HXO3 HXO4
HOF
HOCI HClO2 HClO3 HClO4
HOBr HBrO3 HBrO4
HOI HIO3 HIO4
All oxoacids have tetrahedral structures as follows.
- - - -
O

X X X X
O O O O O O
O O O
OX -
OX -2 XO -3 XO -4

Hypohalous Acids Amongst the four hypohalous acids, HOCl is the most stable acid. The most unstable acid is HOF
which has been made using special techniques.
HOCl, HOBr and HOI can be obtained by shaking freshly precipitated HgO in water with appropriate halogen.
2HgO + H2O + 2X2 Æ HgO ◊ HgX2 + 2HOX
All hypohalous acids are very weak acids. They are good oxidizing agents especially in acidic medium. They are
known only in aqueous solution.
Sodium salt of hypochlorous acid (NaOCl) is well known, and is used for bleaching cotton fabrics. It is also used as
a disinfectant and sterilizing agent. The hypohalite ions can be generated by passing halogens in alkali solution:
X2 + 2NaOH Æ NaX + NaOX + H2O

However, the OX ions tend to disproportionate in basic medium, rate of disproportionation increases with temperature.
3OX– Æ 2X– + XO3–
Halous Acids Chlorous acid is the only halous acid known so far. This exists only in solution. Its salts are known as
chlorites. It is a weak acid, but stronger than hypochlorous acid.
Chlorus acid can be prepared by treating barium chlorite with H2SO4.
Ba(ClO2)2 + H2SO4 Æ 2HClO2 + BaSO4.
Sodium chlorite can be prepared by passing ClO2 through NaOH or Na2O2 solutions.
2ClO2 + 2NaOH Æ NaClO2 + NaClO3 + H2O
Chlorite Chlorate
2ClO2 + Na2O2 Æ 2NaClO2 + O2.
Chlorite ions are stable in basic solution but undergo disproportionation in acidic solution.
5HClO2 Æ 4ClO2 + HCl + 2H2O
Halic Acids Three halic acids are known: HClO3, HBrO3 and HIO3. The acids HClO3 and HBrO3 are known only in
solution. The acid HIO3 exists as a white solid. Both HClO3 and HBrO3 decompose if evaporated to dryness.
4HClO3 Æ 4ClO2(g) + 2H2O(g) + O2(g)
The halic acids are strong oxidizing agents.
HIO3 is made by oxidizing I2 with concentrated HNO3 or O3
I2 + 10HNO3 Æ 2HIO3+ 10NO2 + 4H2O
16.30 Complete Chemistry—JEE Main

HClO3 and HBrO3 are made by adding H2SO4 to barium halates.


Ba(ClO3)2 + H2SO4 Æ BaSO4 + 2HClO3
The salt of chloric acid can be made by passing Cl2 into a hot solution of NaOH.
∞C
6NaOH + 3Cl2 æ80 ææ Æ NaClO3 + 5NaCl + 3H2O.
Perhalic Acids Three perhalic acids are known: HClO4, HBrO4 and HIO4.
They are the strongest acids, very powerful oxidizing agents, and explods in contact with organic matter.
Perchloric acid can be obtained by the action of concentrated hydrochloric acid on sodium perchlorate. The latter is
obtained by the electrolytic oxidation of chlorates.
electrolysis
NaClO3 + H2O ææææÆ NaClO4 + H2.
Perbromates can be obtained by the action of powerful oxidizing agents such as F2 or XeF2 on bromates in aqueous
solution.
KBrO3 + F2 + 2KOH Æ KBrO4 + 2KF + H2O
RbBrO3 + XeF2 + H2O Æ RbBrO4 + 2HF + Xe
The common form of periodic acid is HIO4 ◊ 2H2O, i.e. H5IO6. This is prepared by oxidizing iodates by Cl2 in alkaline
medium.
IO3– + 6OH– + Cl2 Æ IO 5– 6 + 3H2O + 2Cl

This can also be prepared by electrolytic oxidation.


IO –3 + 6OH– Æ IO 5–
6 + 3H2O + 2e

Heating H5IO6 at 100 °C gives HIO4. On strong heating, HIO4 decomposes giving I2O5 and O2.
100 ∞C ∞C
2H5IO6 æ-æææ
4H O
Æ 2HIO4 æ200
ææÆ I2O5 + O2 + H2O.
2

Strengths of Oxoacids The strength of oxoacids depends on the number of oxygen atoms attached to the halogen.
The more oxygen atoms that are bonded to the halogen, the more the electrons will be pulled away from the O—H bond
and the more this bond will be weakend. Hence, the order of easiness with which the hydrogen ions can be removed
from acids is
O3Cl—OH > O2Cl—OH > OCl—OH > Cl—OH
i.e. HClO4 > HClO3 > HClO2 > HOCl.
Reason for not Existing HFO2, HFO3 and HFO4 All the oxoacids have tetrahedron structures. The sp3 hybrid
orbitals of halogens form weak s bonds with oxygen atoms, because the energies of s and p orbitals of halogens differ
appreciably. The ions of these acids are stabilized by strong pp—dp bonding between the full 2p orbitals on oxygen
with emptly d orbitals on the halogen atoms. Despite this, many of the oxoacids are known only in solution.
Fluorine does not have d orbitals and thus cannot form pp—dp
of HOF, does not form oxoacids.
5.7 Interhalogen Compounds
Each halogen reacts with every other halogen to form interhalogens or interhalogen compounds. Table 8 records the
types of interhalogens formed by halogens.
Table 8 Interhalogen Compounds†

F
ClF
1.0 ClF3 BrF
ClF5 BrF3
Cl 1.2 BrF5 (IF)*
1.5 IF3
IF5 0.2 BrCl
IF7 ICl
Br 0.5 (ICl3)2
0.3 IBr
I

The numbers on vertical lines are the difference in electronegativity values. * Unstable
Study of the p-Block Elements (Groups 16, 17 and 18) 16.31

Comments
1. There are four types of interhalogens, AX, AX3, AX5 and AX7.
2. The compounds AX and AX3 are formed where the electronegativity difference is not very large.
3. The compounds AX5 and AX7 are shown by large atoms Br and I surrounded by small atom F. Also, the
difference of electronegativity is large.
4. The compound IF is not stable. It involves the maximum difference in electronegativity and the least number
of F atom. On increasing F atom, the stability of IFx is increased.
5. There are never more than two different halogens in a molecule.
Preparations The interhalogens can be prepared by direct combination of the halogens. The type of interhalogen
formed depends on conditions.
200 ∞C
Cl2 + F2 æææÆ 2CIF I2 + Cl2 æÆ 2IC1
(equal volumes) (equimolar)
300 ∞C
Cl2 + 3F2 æææÆ 2ClF3 I2 + 3Cl2 æÆ (ICl3)2
(excess) (excess)
20 ∞C
Br2 + 3F2 æÆ 2BrF3 I2(s) + 5F2 æææ
Æ 2IF5
250 - 300 ∞C
(diluted with nitrogen) I2(s) + 7F2 æææææ
Æ 2IF7
Br2 + 5F2 æÆ 2BrF5
(excess)

General Characteristics
1. The bonds between interhalogens are covalent because of the small difference in electronegativity values.
2. The melting and boiling points increase as the difference in electronegativity increases.
3. The interhalogen are more reactive than the halogens (except F2). This is because the bond A—X is weaker
than A—A and X—X bonds.
4. Hydrolysis of interhalogen gives halide and oxohalide ions. The latter is formed from the larger halogen present
in the interhalogen.
5. Liquids ICl3, BrF3, and IF5 have appreciable conductivity. This has been explained on the basis self-ionization.
2ICl3  +
 [ICl2] + [ICl4]

2IF5  +
 [IF4] + [IF6]

2BrF3  +
 [BrF2] + [BrF4]

Structures of Interhalogens In AX3, the number of valence electrons is 4 ¥ 7 = 28. The Lewis structure of AX3 is
X—A—X

orientations. Experimental studies have shown that AX3 is T-shaped.


In general, in trigonal bipyramidal orientation, lone paris are always present in X
equatorial positions.
Gaseous state X
Bond angle 87°40¢ A
Equaterial A—X bonds (those in the triangle) = 159.8 pm
Apical A—X bonds (those pointing up and down) = 169.8 pm
The structure is distorted trigonal bipyramidal since bond angle is 87°40¢. The decrease of
bond angle from 90° to 87°40¢ is attributed to the lone pair-lone pair repulsion. Also note that X
the apical bond distance is larger than the equational bond distance.
e molecule AX3 are as follows.
A in free state ≠Ø ≠Ø ≠Ø ≠

A in the AX3 molecule ≠Ø ≠Ø ≠ ≠ ≠



sp3d hybridization, trigonal bipyramidal geometry
16.32 Complete Chemistry—JEE Main

In AX5 molecules, there are 6 ¥ 7 = 42 valence electrons. In the Lewis structure there are six electron pairs around
the central atom X. According to the VSEPR theory, these electrons will acquire octahedral orientation. Thus, the
structure of AX5 is
X

X X X X

A A
X X
X X X

Lewis structure
In general, in octahedral orientation, loan pairs are present in axial positions.

molecule are as follows.


s p d
A in free state ≠Ø ≠Ø ≠Ø ≠

A in AX5 ≠Ø ≠ ≠ ≠ ≠ ≠

sp3d2 hybridization, octahedral geometry

In AX7, there are 7 ¥ 8 = 56 valence electrons. In the Lewis structure of the molecule there are seven unpaired
electrons around the central atom A. These seven pairs will acquire pentagonal bipyramidal orientations. Thus, the
structure of IF7 is
F
X
F F
X X
A I
X X F F
F
X X F

follows
s p d
I in free state ≠Ø ≠Ø ≠Ø ≠

I in IF7 ≠ ≠ ≠ ≠ ≠ ≠ ≠

sp3d3 hybridization, pentagonal bipyramidal structure
In the gaseous phase, ICl3 is decomposed to IC1 and Cl2. So, its structure is not known. In solid state, 1Cl3 dimerizes
to (ICl3)2. Two T-shaped ICl3 molecules join together, forming a planar dimeric molecule (IC13)2.
Cl Cl Cl
I I
Cl Cl Cl
Anomalous Behaviour of Fluorine
Fluorine differs from rest of the members of its group because of its small size, high electronegativity and nonavailability
of d orbitals in the valence shell.
Study of the p-Block Elements (Groups 16, 17 and 18) 16.33

1. The oxidation state of F in its compound is always –1 while other elements exhibit a large number of oxidation
states (–1, +1. +3, +5, +6, +7).
2. Fluorine is the most reactive among halogens due to its low bond enthalpy. The latter arises due to the larger
repulsion between the nonbonding electrons in the compact molecule.

4. Fluoride exhibits a large tendency to form complex ions ([AlF6]3–, [FeF6]3–) due to its small size. The other
halides have much less tendency for complexation.
5. In its compound with hydrogen, it exhibits extensive hydrogen bondings.

7. HF is the weakest acid in aqueous solution, while HCl, HBr and HI are stronger acids.
8. In its salt with silver ions. AgF is sloble in water while other halides are insoluble.

MULTIPLE CHOICE QUESTIONS

1. The number of halogens known so far is


(a) 4 (b) 5 (c) 6 (d) 7
2. The atomic number of astatine which belongs to six period is
(a) 53 (b) 54 (c) 85 (d) 86
3. Which of the following is radioactive halogen?
(a) Cl (b) Br (c) I (d) At

(a) F < Cl < Br (b) F > Cl > Br (c) F > Cl < Br (d) F < Cl > Br
5. Which of the following orders of bond dissociation enthalpy of halogens is correct?
(a) F—F > Cl—Cl > Br—Br (b) F—F < Cl—Cl < Br—Br
(c) F—F > Cl—Cl < Br—Br (d) F—F < Cl—Cl > Br—Br
6. Which of the following halogens does not exhibit positive oxidation state?
(a) F (b) Cl (c) Br (d) I
7. Which of the following orders of melting point of hydrides of halogens is correct?
(a) HF > HCl > HBr (b) HF < HCl < HBr (c) HF > HCl < HBr (d) HF < HCl > HBr

(a) Fluorspar (b) Cryolite (c) Fluoroapatite (d) Chile salt peter
9. Which of the halogen is most abundant in the earth’s crust?
(a) F (b) Cl (c) Br (d) I
10. Which of the following orders of bond strengths between a nonmetal E and halogens is correct?
(a) e (E—F) > e (E—Cl) > e (E—Br) (b) e (E—F) > e (E—Cl) < e (E—Br)
(c) e (E—F) < e (E—Cl) < e (E—Br) (d) e (E—F) < e (E—Cl) > e (E—Br)
11. Which of the following reactions is quite violent in nature?

(c) Between hydrogen and bromine (d) Between hydrogen and iodine
12. The strength of halogen acids in water follows the order
(a) HF > HCl > HBr (b) HF < HCl < HBr (c) HF > HCl < HBr (d) HF < HCl > HBr
13. Which of the following statements is not correct?
(a) Halides formed with nonmetals are covalent in nature
(b) The reactivity of halogens decreases with increasing atomic number
(c) Hydrogen halides are ionic molecules in the gaseous phase
(d) Metals with low ionization energies form ionic halides while those of high ionization energies form covalent
molecules
16.34 Complete Chemistry—JEE Main

14. Which of the following statements is not correct?

(b) For a metal exhibiting more than one oxidation state, the halides in the lower oxidation state is more covalent
than the one in the higher state
(c) Halogens do not occur freely in nature
(d) Most of binary compounds between oxygen and halogens are unstable
15. Which of the following halogen oxides are ionic in nature?
(a) Cl2O (b) Br2O (c) BrO2 (d) I2O4
16. Which of the following interhalogens does not exist?
(a) ClF (b) ClF2 (c) ClF3 (d) ClF5

(a) XeF2 (b) XeF4 (c) XeF6 (d) XeF8


18. Which of the following represents hypohalous acid?
(a) HOX (b) HXO2 (c) HXO3 (d) HXO4
19. Which of the following represents halous acid?
(a) HOX (b) HXO2 (c) HXO3 (d) HXO4
20. Which of the following represents halic acid?
(a) HOX (b) HXO2 (c) HXO3 (d) HXO4
21. Which of the following represents perhalic acid?
(a) HOX (b) HXO2 (c) HXO3 (d) HXO4
21. Which of the following statements is not correct?

(b) Hypohalous acids HOCl, HOBr and HOI are all strong acids
(c) Hypochlorous acid is the most stable among hypohalous acids
(d) Chlorous acid is the only halous acid known
23. Which of the interhalogens is unstable?
(a) IF (b) IF3 (c) IF5 (d) IF7
24. Which of the following exhibits +7 oxidation state in interhalogen compounds?
(a) F (b) Cl (c) Br (d) I
25. The structure of IF7 is
(a) planar (b) distorted octahedral
(c) pentagonal bipyrimadal (d) distorted trigonal pyrimidal
26. Iodine in IF7 involves
(a) s2p3d2 hybridization (b) sp3d3 hybridization (c) p3d4 hybridization (d) p2d5 hybridization
27. The anhydride of HClO4 is
(a) ClO–4 (b) Cl2O7 (c) ClO2 (d) ClO3
28. Which of the following molecules is not paramagnetic?
(a) ClO2 (b) ClF3 (c) ClO3 (d) BrO2
29. Hypobromous acid is
(a) HOBr (b) HOBrO (c) HOBrO2 (d) HOBrO3

(a) is a weak acid (b) is a strong acid


(c) does not exist (d) is a good oxidizing agent
31. Which of the following exhibits the lowest bond energy?
(a) HF (b) HCl (c) HBr (d) HI
32. Which of the following has a pyramidal structure?
(a) ClO2 (b) ClO2– (c) ClO–3 (d) ClO–4
33. Which of the following forms the most basic 0.1 M solution?
(a) NaCl (b) NaOCl (c) NaClO2 (d) NaClO3
Study of the p-Block Elements (Groups 16, 17 and 18) 16.35

34. The shape of the XeF4 molecules is


(a) square planar (b) tetrahedral (c) square pyramidal (d) trigonal bipyramidal

(a) XePt F6 (b) Xe(RuF6)2 (c) XeOF4 (d) XeOF2


36. The shape of XeOF4 molecule is
(a) square pyramidal (b) tetrahedral (c) distorted tetrahedral (d) irregular
37. The F—Xe—F bond angle in XeF4 is
(a) 109° (b) 103° (c) 90° (d) 60°
38. Which of the following exhibits the largest electrical conductivity in the liquid state?
(a) F2 (b) Cl2 (c) Br2 (d) I2
39. Fluorine is highly reactive because
(a) F—F bond energy is high (b) F—F bond energy is low
(c) it is gaseous at room temperature (d) F has a smaller size
40. Which of the following molecules has minimum dipole moment?
(a) HF (b) HCl (c) HBr (d) HI
41. The strongest reducing agents amongst the halides is
(a) F– (b) Cl– (c) Br– (d) I–
42. BrF3 molecule is
(a) planar trigonal (b) tetrahedral (c) T-shaped (d) trigonal pyramidal
43. The correct order of stability of the halous acids in aqueous solution is
(a) HClO < HBrO < HIO (b) HClO > HBrO < HIO (c) HClO > HBrO > HIO (d) HClO < HBrO > HIO
44. The strongest Bronsted base is
(a) ClO– (b) ClO–2 (c) ClO–3 (d) ClO–4

(a) Cryolite (b) Feldspar (c) Fluorspar (d) Fluoroapatite


46. Which of the following contains chlorine with oxidation number +1?
(a) ICl (b) Cl2O (c) HCl (d) HClO2
47. Which of the following is not a polar molecule?
(a) HCl (b) NI3 (c) Cl2O (d) CCl4
48. Which of the following will liberate Br2 from KBr?
(a) Cl2 (b) HI (c) I2 (d) SO2
49. The order of strength of oxyacids of chlorine is
(a) HClO < HClO2 < HClO3 < HClO4 (b) HClO > HClO2 > HClO3 > HClO4
(c) HClO > HClO2 > HClO3 < HClO4 (d) HClO < HClO2 < HClO3 > HClO4
50. The shape of XeO3 is
(a) triangular planar (b) tetrahedral (c) triangle pyramidal (d) square planar
51. The salts of halogens are extensively found in seawater with the exception of that of

(a) violet light (b) red light (c) green light (d) orange light
53. The gaseous iodine molecules absorb yellow light from the visible region. Its colour would be
(a) violet (b) red (c) green (d) yellow

(a) F2 > Cl2 > Br2 (b) F2 < Cl2 < Br2 (c) F2 > Cl2 < Br2 (d) F2 < Cl2 > Br2
55. Which of the following statements regarding halogens is not true?
(a) Ionization energy decreases with increase in atomic number
(b) Electronegativity decreases with increase in atomic number

(d) Enthalpy of fusion increases with increase in atomic number


16.36 Complete Chemistry—JEE Main

56. Which of the following halogens does not show positive oxidation states?
(a) Fluorine (b) Chlorine (c) Bromine (d) Iodine
57. Which of the following orders regarding the boiling point of halogens is true?
(a) HF > HCl > HBr > HI (b) HF < HCl < HBr < HI
(c) HF > HCl < HBr < HI (d) HF > HCl > HBr < HI
58. Which of the following statements regarding halogens is not true?
(a) Halogens act as strong oxidizing agents.
(b) Fluorine oxidizes water to oxygen and ozone
(c) Chlorine oxidizes water to oxygen
(d) Bromine disproportionates in water producing the reaction
Br2 + H2O HBr + HOBr
59. Which of the following equations is not shown by the appropriate halogen?
(a) F2 + 2OH – 2F – + H2O + OF2
2% solution
hot
(b) 2F2 + 4OH – ææÆ 4F– + 2H2O + O2
conc.
hot
(c) X2 + 2OH – ææÆ H2O + X– + XO–; where X is Cl, Br and I
conc.
light
(d) Xe + F2 æææ
Æ XeF2
60. Which of the following reactions would not proceed to right hand side?
(a) F2 + Cl– Æ (b) Cl2 + Br– Æ (c) Br2 + I– Æ (d) I2 + Cl– Æ
61. Which of the following reactions regarding the preparation of hydrogen halide is not correct?
(a) CaF2 + H2SO4 Æ CaSO4 + 2HF (b) NaCl + H2SO4 Æ NaHSO4 + HCl
conc. conc.
(c) NaBr + H2SO4 Æ NaHSO4 + HBr (d) PI3 + 3H2O Æ H3PO3 + 3HI
conc.
62. Which of the following orders regarding the acid character of hydrogen halides in the aqueous medium is correct?
(a) HCl > HBr < HI (b) HCl < HBr > HI (c) HCl > HBr > HI (d) HCl < HBr < HI

(a) Fluorine (b) Chlorine (c) Bromine (d) Iodine


64. Which of the halogens is found in DDT?
(a) Fluorine (b) Chlorine (c) Bromine (d) Iodine
65. Which of the following interhalogens does not exist?
(a) ClF (b) ClF2 (c) ClF3 (d) IF7
66. When chlorine reacts with cold and dilute solution of sodium hydroxide the products obtained are
(a) Cl– + ClO– (b) Cl– + ClO–2 (c) Cl– + ClO–3 (d) Cl– + ClO–4
67. When iodine reacts with a hot and concentrated solution of sodium hydroxide, the products obtained are
(a) I– + IO– (b) I– + IO3– (c) I– + IO–4 (d) I– + I2O94–
68. Which of the following halogens is the strongest oxidizing agent?
(a) Fluorine (b) Chlorine (c) Bromine (d) Iodine
69. Which of the following interhalogen is not observed?
(a) AX (b) AX3 (c) AX5 (d) AX6
70. The structure of interhalogen AX3 is
(a) triangular planar
(b) pyramidal
(c) T-shaped with two lone pairs of electrons at the equilateral positions
(d) tetrahedral with a single electron
Study of the p-Block Elements (Groups 16, 17 and 18) 16.37

71. Which of the following is anhydride of hypochlorous acid?


(a) Cl2O (b) ClO2 (c) Cl2O6 (d) Cl2O7
72. Which of the following is anhydride of perchloric acid?
(a) Cl2O (b) ClO2 (c) Cl2O7 (d) Cl2O7

73. The hybridization of I in ICl 2 is
(a) sp (b) sp3 (c) d2sp3 (d) sp3d

74. The hybridization of I in ICl 4 is
(a) sp3 (b) dsp2 (c) sp3d2 (d) d2sp3
75. Which of the following is not a pseudohalogen?
(a) CN– (b) SCN– (c) OCN– (d) NO+
3 3
76. The hybridization sp d leads to
(a) octahedral geometry (b) hexagonal pyramid structure
(c) pentagonal bipyramid structure (d) square geometry
77. The structure of ClF3 is
F F
F F F
F Cl
(a) Cl (b) Cl (c) Cl (d)
F F
F
F F F
78. Which of the following interhalogens is colourless?
(a) ClF(g) (b) BrF(g) (c) BrCl(g) (d) ICl(s)
79. Which of the following elements does not exhibit basic properties?
(a) Fluorine (b) Chlorine (c) Bromine (d) Iodine
80. In the reaction ClF + Cl2 + SbF5 Æ [Cl3] [SbF6]
(a) Cl2 acts as a Lewis acid (b) Cl2 acts as a Lewis base
(c) Cl2 is an amphoteric species (d)Cl2 acts as a reducing agent
- 78 ∞C
81. In the reaction [Ph4As][Cl] + Cl2 æææÆ [Ph4As][Cl3]
(a) Cl2 acts as a Lewis acid (b) Cl2 acts as a Lewis base
(c) Cl2 is an amphoteric species (d) Cl2 acts as an oxidizing agent.
82. Interhalogen compounds are
(a) more reactive than halogens (b) less reactive than halogens
(c) equally reactive as halogens (d) inert substances

ANSWERS
1. (b) 2. (c) 3. (d) 4. (d) 5. (d) 6. (a)
7. (c) 8. (d) 9. (a) 10. (a) 11. (a) 12. (b)
13. (c) 14. (b) 15. (d) 16. (b) 17. (d) 18. (a)
19. (b) 20. (c) 21. (d) 22. (b) 23. (a) 24. (d)
25. (c) 26. (b) 27. (b) 28. (b) 29. (b) 30. (a)
31. (d) 32. (c) 33. (b) 34. (a) 35. (a) 36. (a)
37. (c) 38. (d) 39. (b) 40. (a) 41. (d) 42. (c)
43. (c) 44. (a) 45. (b) 46. (b) 47. (d) 48. (a)
49. (a) 50. (c) 51. (d) 52. (a) 53. (a) 54. (d)
55. (c) 56. (a) 57. (c) 58. (c) 59. (c) 60. (d)
16.38 Complete Chemistry—JEE Main

61. (c) 62. (d) 63. (a) 64. (b) 65. (b) 66. (a)
67. (b) 68. (a) 69. (d) 70. (c) 71. (a) 72. (d)
73. (d) 74. (c) 75. (d) 76. (c) 77. (c) 78. (a)
79. (a) 80. (b) 81. (a) 82. (a)

HINTS AND SOLUTIONS


2. The atomic number of astatine is 2 + 8 + 8 + 18 + 18 + (14 + 17) = 85

5. Cl— Cl bond is the strongest.


6. F is the most electronegative atom.
7. Because of hydrogen bonding, HF has higher melting point than HCl, which, in turn, has smaller melting point
than HBr.
8. Chile saltpeter is NaNO3.
13. Hydrogen halides are covalent molecules in the gaseous phase.
14. The halide in the lower oxidation state are less covalent than the one in the higher.
22. Hypohalous acids are all weak acids.
H O
27. 2HClO4 æææ
2
Æ Cl2O7
28. ClF3 contains even number of electrons whereas rest of the three contain odd number of electrons.
33. HOCl is the weakest acid, OCl– undergoes maximum hydrolysis.

56. Fluorine being most electronegative always exhibits the oxidation state of –1.
57. Because of small size and highly electronegative nature of F, there occurs extensive hydrogen bonding in HF. It
is because of this, melting and boiling points of HF are greater than those of HCl.
58. The reaction of Cl2 with water is Cl2 + H2O HCl + HOCl
59. The reaction of halogens (except F2) with hot and concentrated alkali is
hot
3X2 + 6OH – ææÆ 3H2O + 5X– + XO3–
conc.

60. F2 can replace 2Cl – to Cl2, Cl2 can replace 2Br– to Br2 and Br2 can replace 2I– to I2.
61. Sulphuric acid oxidizes HBr (or HI) to Br2 (or I2). The halides HBr and HI are prepared by hydrolysis of
phosphorus trihalides with cold water
PX3 + 3H2O H3PO3 + 3HX
(X = Br or I)
62. The order of electronegativity of halogens is F > Cl > Br > I
Hence, acid strength follows the reverse order.

73. There are 3 ¥ 7 + 1 = 22 valence electrons in ICl2–. These are distributed as Cl I Cl


3
d.

74. There are 5 ¥ 7 + 1=36 valence electrons in ICl4–. These are distributed as

There are six pairs of electrons around I. It hybridization will be sp3d2


75. NO+ is not a pseudo halogen?
76. The hybridization sp3d3 leads to pentagonal bipyramid distribution of electrons.
Study of the p-Block Elements (Groups 16, 17 and 18) 16.39

77. There are 4 ¥ 7 = 28 valence electrons in ClF3. These are distributed as F Cl F


F
Hence, its hybridization is sp3d which leads to trigonal bipyramid
structure. The actual structure is T-shaped as shown by the choice (c).
78. ClF(g) is colourless.
79. Fluorine is the most electronegative element. It does not exhibit basic properties.

MULTIPLE CHOICE QUESTIONS FROM AIEEE AND JEE MAIN

(c) hydration enthalpy (d) bond dissociation energy [2004]

2. The smog is essentially caused by the presence of


(a) O2 and O3 (b) O2 and N2
(c) oxides of sulphur and nitrogen (d) O3 and N2 [2004]
3. The correct order of the thermal stability of hydrogen halides (H—X) is
(a) HI > HBr > HCl > HF (b) HF > HCl > HBr > HI
(c) HCl < HF < HBr < HI (d) HI > HCl < HF < HBr [2005]
4. What products are expected from the disproportionation reaction of hypochlorous acid?
(a) HCl and HClO3 (b) HClO3 and Cl2O (c) HClO2 and HClO4 (d) HCl and Cl2O
[2006]
5. Which of the following reactions of xenon compounds is not feasible?
(a) 2XeF2 + 2H2O Æ 2Xe + 4HF + O2 (b) XeF6 + RbF Æ Rb [XeF7]
(c) XeO3 + 6HF Æ XeF6 + 3H2O (d) 3XeF4 + 6H2O Æ 2Xe + XeO3 + 12HF + 1.5O2
[2009]
6. Among the following oxoacids, the correct order of acid strength is
(a) HClO2 > HClO4 > HClO3 > HOCl (b) HOCl > HClO2 > HClO3 > HClO4
(c) HClO4 > HOCl > HClO2 > HClO3 (d) HClO4 > HClO3 > HClO2 > HClO [2014]
7. Shapes of certain interhalogen compounds are stated below. Which one of them is not correctly stated?
(a) IF7: pentagonal bipyramid (b) BrF5: trigonal bipyramid
(c) BrF3: planar T-shaped (d) ICl3: planar dimeric [2014, online]
8. The least number of oxoacids are formed by
(a) Nitrogen (b) Sulphur
(c) Fluorine (d) Chlorine [2015, online]
9. Chlorine water on standing loses its colour and forms: [2015, online]
(a) HCl only (b) HOCl and HOCl2 (c) HCl and HOCl (d) HCl and HClO2
10. The non-metal that does not exhibit positive oxidation state is: [2016, online]
(a) Chlorine (b) Iodine (c) Fluorine (d) Oxygen
11. Which of the following is the most reactive?
(a) Cl2 (b) Br2 (c) I2 (d) ICl
16.40 Complete Chemistry—JEE Main

ANSWERS
1. (c) 2. (c) 3. (b) 4. (a) 5. (c) 6. (d)
7. (b) 8. (c) 9. (c) 10. (c) 11. (d)

HINTS AND SOLUTIONS


1. The process F(g)– + aq Æ F(aq)– is accompanied with a high decrease in enthalpy as compared to other halogens.

Therefore, the choice c is correct.


2. The smog is caused by the oxides of sulphur and nitrogen. Therefore, the choice c is correct.
3. The thermal stability of H—X (hydrogen halides) decreases with increase in the atomic number of X.
4. The reaction is 3HClO Æ 2HCl + HClO3
5. Besides XeF2 and XeF4, the compound XeF6 also exhibit reaction with water.
XeF6 + 3H2O Æ XeO3 + 6HF
XeF6 also reacts with RbF forming Rb+[XeF7] –.
6. The larger the number of oxygen atoms attached to chlorine, greater the electron pull towards oxygen, hence, more
easy to remove hydrogen from the acids. The given acids are
Cl — OH; OCl — OH; O2Cl — OH, O3Cl — OH
(HOCl) (HClO2) (HClO3) (HClO4)

7.

8. Flourine forms least number of oxoacids.


9. Cl2 + H2O Æ HCl + HOCl
10. Fluorine is the most electronegative atom. It does not have positive oxidation state.
11. Interhalogen compound is more reactive than halogens.
Study of the p-Block Elements (Groups 16, 17 and 18) 16.41

The Group 18 Elements


Group 18 of the periodic table consists of helium (He), neon (Ne), argon (Ar), krypton (Kr), xenon (Xe) and radon
(Rn). They are collectively known as noble gases because of their extremely low reactivity.
Discovery of Noble Gases
In 1785, Henry Cavandish passed repeated electric sparks in a mixture of dry air (free from carbon dioxide) and oxygen
in order to achieve complete conversion of nitrogen of the air to its oxides. The resulting nitrogen oxides were absorbed
in caustic potash solution and the excess oxygen was removed by potassium pentasulphide. Still a very small volume of
the gas (1/120th of the original volume of air) was left behind which neither combined with oxygen nor with any other
element. This remarkable observation of Cavandish remained unnoticed for more than a century.
In 1895, Lord Rayleigh found that the density of nitrogen obtained from atmosphere was about 0.47% higher than
that prepared from the decomposition of ammonium nitrite. He concluded that the nitrogen obtained from atmosphere
contains some constituent (or constituents) which is (or are) heavier than nitrogen.
Ramsay and Rayleigh repeated Cavandish’s experiment and recorded the spectrum of the residual gas. It was found
to be entirely different from that of any other element known till then. The vapour density of the gas was found to be 20
which gives its relative molar mass as 40. This gas was found to be chemically inactive, and therefore named ‘argon*
(Greek word meaning lazy or inert). However, the new gas was soon found to be a mixture of a number of chemically

The history of discovery of helium is also very interesting. As early as 1868, Lockyer concluded that the bright
yellow line (which he called D3 line) observed in the solar spectrum during a total eclipse is due to the presence of a new
element in the sun. He called this element helium (From the Greek word helios, the sun). Ramsay, in 1895, showed that
the gas isolated by Hillebrand, from uranium mineral, had the same spectrum as helium discovered by Lockyer. Thus,

In 1898, Ramsay and Travers carried out systematic fractional evaporation of a large volume of liquid gas obtained

new). Its vapour density was found to be 10.1 and hence relative atomic mass 20.2. Its spectrum establishes it to be

krypton (from Greek word krypto meaning hidden) and xenon (from Greek word which means stranger), respectively.
The vapour densities of krypton and xenon were found to be 40 and 64 and hence their relative atomic masses were 80
and 128, respectively.
In 1900, Dorn discovered the last member of noble gases as a disintegration product of radium and named it as radon
88 Ra Æ 86 Rn + 2 He
226 222 4

Physical Properties of Noble Gases


The physical properties of the noble gases are recorded in Table 9.
Table 9. Physical Properties of Noble Gases
Property He Ne Ar Kr Xe Rn
2 2 6 2 6 2 6 2 6
(1s) (2s) (2p) (3s) (3p) (4s) (4p) (5s) (5p) (6s) (6p)6
2

Atomic number 2 10 18 36 54 86
Relative atomic mass 4.002 20.179 39.098 83.80 131.29 222.018
Abundance in dry air by volume (ppm) 5.24 18.18 93.40 1.14 0.09 traces
Melting point, T/K * 24.6 83.8 115.9 161.3 202.0
Boiling point, T/K 4.2 27.1 87.3 119.7 165.0 211.0
Atomic radius, r/pm 93 112 154 167 190 —
Ionization energy, I/kJ mol–1 2372 2080 1520 1351 1170 1037
Enthalpy of vaporization, 0.08 1.74 6.52 9.05 12.65 18.1
DvapH/kJ mol–1
* Helium is the only liquid that cannot be frozen without applying pressure.
16.42 Complete Chemistry—JEE Main

Description
1. All elements exist as monatomic gases.
ns)2(np)6, where n varies from
2
.

inactive.
4. The radii of the elements are very large and increase on descending the group as expected (Fig. 15). These
nonbonded radii should be compared with van der Waals radii of other elements and not with covalent bonded
radii.
5. The ionization energies of the noble gases are very high order (Fig. 16)—helium has the highest value of all
the elements.
6. Melting and boiling points are low indicating extremely weak interatomic forces (van der Waals types) between
the atoms of noble gases (Fig. 17). This is supported by the low values of enthalpies of vaporization.
b.pt.
200 m.pt.

240° Ionization energies


van der Waals Radii
200
160

200°
120
160
IE/kJ mol-1
r/pm

160°

T/K
80

120
120° 40

80 80°
He Ne Ar Kr Xe He Ne Ar Kr Xn Rn He Ne Ar Kr Xe Rn

Fig. 15 Molecular Radii of the Fig. 16 Ionization Energies of the Fig. 17 Melting and Boiling Points of the
Elements of Group 18 Elements of Group 18 Elements of Group 18

Special Properties of Helium


When helium is cooled below 2.178 K at 1 atmospheric pressure, a remarkable liquid with abnormal properties is
obtained. The liquid is referred to as helium II and the characteristic temperature is called the lamda (l) point. The

tension and compressibility.


Compounds of Noble Gas
No compound of the noble gases was known before 1962. As highly oxidizing PtF6 could oxidize oxygen
PtF6 + O2 æÆ O +2[PtF6]–
Æ Xe+ + e–; 1170 kJ mol–1) is very similar to oxygen (O2
Æ O +2 + e–; 1165 kJ mol–1), should also react in the same way. In 1962, N. Bartlett could actually prepare a yellow-red
powder corresponding to Xe+[PtF6]– by interaction of PtF6 with xenon.
Xe + PtF6 æ298 K
ææÆ Xe+[PtF6]–
The reaction has since been shown to be more complicated, and the product is [XeF]+ [Pt2F11]–.
Xe[PtF6] + PtF6 æ298
ææ K
Æ [XeF]+ [PtF6]– + PtF5 æ333
ææ K
Æ [XeF]+[Pt2F11]–
Soon after this, a colourless volatile solid XeF4 was obtained when Xe and F2 reacted at 400 °C.
The ionization energies of He, Ne and Ar are much higher than Xe and are high enough to enter into the chemical
Study of the p-Block Elements (Groups 16, 17 and 18) 16.43

2. The
chemistry of radon is not well studied because of its short half-life.
Compounds of Xenon

Xenon Fluorides °C in a sealed nickel vessel.


The product depends on the ratio of Xe and F.
2:1 ratio
XeF2

1:5 ratio
Xe + F2 XeF4
1:20 ratio
XeF6

Oxidation Reactions 2 to H+, Cl– to Cl2, I– to I2 and Ce(III) to Ce(IV).


XeF2 + H2 Æ 2HF + Xe
XeF2 + 2HCl Æ 2HF + Xe + Cl2
XeF2 + 2KI Æ 2KF + Xe + I2
XeF2 + Ce2(SO4)3 + SO2–4 Æ 2Ce(SO4)2 + Xe + F2
Similar reactions can be written for XeF4 and XeF6.
Fluorination Reaction
XeF4 + 2SF4 Æ Xe + 2SF6
XeF4 + 2C6H6 Æ Xe + 2C6H5F + 2HF
Reaction with Water
2XeF2 + 2H2O Æ 2Xe + 4HF + O2 (slow reaction)
3
3XeF4 + 6H2O Æ 2Xe + XeO3 + 12HF + O2 (violent reaction)
2
XeF6 + 3H2O Æ XeO3 + 6HF (violent reaction)
XeF6 undergoes partial hydrolysis reaction with small quantity of water.
XeF6 + H2C Æ XeOF4 + 2HF
XeOF4 is also obtained when XeF6 reacts with SiO2
2XeF6 + SiO2 Æ 2XeOF4 + SiF4
The reaction of XeF6 with XeO3 also produces XeOF4.
2XeF6 + XeO3 Æ 3XeOF4.
Formation of Complexes XeF2 forms complexes with PF5, AsF5. SbF5
structures of complexes are believed to be
[XeF+] [MF6]–
[XeF+] [M2F11]–
[Xe2F3]+ [MF6]–
XeF4 and XeF6 also form complexes, but their number are less.
Structure of XeF2
The number of valence electrons in XeF2 are 8 + 2 ¥ 7 = 22. These are distributed as follows.
F—Xe—F

bipyramidal orientations. Experimentally, XeF2 is found to be linear with bond distance X—F equal to 200 pm. Thus,
F atoms in XeF2 occupy apical positions while the three lone pair of electrons occupy equatorial positions.
16.44 Complete Chemistry—JEE Main

Xe

F
Note: In triangular bipyramidal, lone pairs of electrons always occupy equatorial positions.
2
are as follows.
s p d
Xe in free atom ≠Ø ≠Ø ≠Ø ≠Ø

Xe in FeX2 ≠Ø ≠Ø ≠Ø ≠ ≠

sp3d hybridization, octahedral geometry

Structure of XeF4
There are 8 + 4 ¥ 7 = 36 valence electrons in XeF4.
There are six electron pairs around Xe. According to the VSEPR theory, these will attain octahedral orientations.
Experimentally, all the four X—F bonds in XeF4 have the identical bond distance of 195 pm. Thus, the four equatorial
positions will be occupied by F atoms while the two lone pair of electrons will occupy apical positions.

F F
F Xe F
F F Xe

F F

Note: In octahedral orientation, lone pairs of electrons occupy apical positions.


2 are as follows.
s p d
Xe in free state ≠Ø ≠Ø ≠Ø ≠Ø

≠Ø ≠Ø ≠ ≠ ≠ ≠

sp3d2 hybridization, trigonal bipyramidal geometry

Structure of XeF6
There are 8 + 6 ¥ 7 = 50 valence electrons.
There are seven-electron pairs around Xe atom. Six of them attain octahedral orientations and the seventh electron
pair of Xe causes distortion in the octahedral orientations. Thus, the structure of XeF6 is the distorted octahedral.
Study of the p-Block Elements (Groups 16, 17 and 18) 16.45

F
F F
F F
F Xe F Xe
F F
F F
F
slightly distorted octahedron

Structure of XeO3
There are 8 + 3 ¥ 6 = 26 valence electrons. There are four electron pairs around Xe. Hence, these electrons acquires
tetrahedral orientations (VSEPR theory).

O Xe O Xe
O O O

O
XeOF4
Valence electron = 8 + 6 + 4 ¥ 7 = 42 Number of electron pairs around Xe = 6
Arrangement of electron pairs around
O F Xe = square bipyramidal
F Xe O

F F F F
Lewis structure
Xe

F F

square bipyramidal (octahedral with one


position unoccupied)

XeO2F2
Number of valence electron = 8 + 2 ¥ 6 + 2 ¥ 7 Number of paired electrons around Xe = 5
= 34 Orientation of electron pairs around
Xe is triangular bipyramidal.
F
F
O Xe O
O
F Xe
Lewis structure O

F
triangular
bipyramidal
16.46 Complete Chemistry—JEE Main

[XeO6]4–
Number of valence electron = 8 + 6 ¥ 6 + 4 Number of paired electrons around Xe = 6
= 48 Orientation of electron pairs around
Xe is octahedron
O 4

O O
O O
O Xe O Xe
O O
O O
Lewis structure
O
octahedral
XeO4
Number of valence electron = 8 + 4 ¥ 6 = 32 Number of paired electrons around Xe = 4
Arrangement of electron pairs around
Xe is tetrahedron
O

O
Xe
O Xe O
O O
O
Lewis structure O
Tetrahedral

MULTIPLE CHOICE QUESTIONS

1. The number of elements in Group 18 is


(a) 4 (b) 5 (c) 6 (d) 7

2. The ionization energy of Xe Xe+ + e is very close to that of
(a) N2 Æ N+2 + e– (b) O2 Æ O+2 + e– (c) C2 Æ C+2 + e– (d) B2 Æ B+2 + e–
3. When a mixture of Xe and F2 in the ratio of 2 : 1 is heated at 400 C in a sealed nickel vessel, the compound formed
is
(a) XeF (b) XeF2 (c) XeF4 (d) XeF6
4. When a mixture of Xe and F2 in the ratio of 1 : 5 is heated at 400 °C in a sealed nickel vessel, the compound formed
is
(a) XeF (b) XeF2 (c) XeF4 (d) XeF6
5. When a mixture of Xe and F2 in the ratio of 1 : 20 is heated at 400 °C in a sealed nickel vessel, the compound
formed is
(a) XeF (b) XeF2 (c) XeF4 (d) XeF6
6. The number of paired electrons around Xe in XeF2 is
(a) 3 (b) 4 (c) 5 (d) 6
Study of the p-Block Elements (Groups 16, 17 and 18) 16.47

7. The number of paired electrons around Xe in XeF4 is


(a) 3 (b) 4 (c) 5 (d) 6
8. The number of paired electrons around Xe in XeF6 is
(a) 4 (b) 5 (c) 6 (d) 7
9. The number of paired electrons around Xe in XeO3 is
(a) 4 (b) 5 (c) 6 (d) 7
10. The structure of XeO4 is
(a) square planar (b) tetrahedral
(c) pentagonal with one paired electrons (d) octahedral with two paired electrons
11. Which of the following inert gases has the largest abundance (by volume) in air?
(a) Helium (b) Neon (c) Argon (d) Krypton

experimental conditions?
(a) XeF2 (b) XeF3 (c) XeF4 (d) XeF6
13. Which of the following statements is correct?
(a) The processes Xe(g) Xe+(g) + e– and O2(g) O2+ (g) + e– involve more or less identical ionization energy
(b) XeF2 possesses an angular structure
(c) XeF2
the three lone pairs occupying equatorial position
(d) XeF4
14. The formula of freon – 12 is
(a) CFCl3 (b) CF2Cl2 (c) CF3Cl (d) CF4

ANSWERS
1. (c) 2. (b) 3. (b) 4. (c) 5. (d) 6. (c)
7. (d) 8. (d) 9. (a) 10. (b) 11. (c) 12. (b)
13. (b) 14. (b)

HINTS AND SOLUTIONS


13. XeF2 molecule has a trigonal bipyramidal structure.

MULTIPLE CHOICE QUESTIONS FROM AIEEE AND JEE MAIN

(a) XeO2F2 (b) XeOF4 (c) XeO3 (d) XeO4 [2014, online]

ANSWERS
1. (d)
16.48 Complete Chemistry—JEE Main

HINTS AND SOLUTIONS

1 3
XeF2 + H2 O2 + 2HF 3 XeF4 + 6 H2 O2 + XeO3 + 12 HF
2 2
XeF6 + 3 H2 3 + 6HF XeF6 + H 2 O 4 + 2HF
XeOF4 + XEO3 2
(
small
quantity )
17
d– and f–Block
Elements

Transition Elements

Table 1

V
Cr
Mn
6
Iron Fe
Cobalt Co
Ni
Copper
17.2

Variable Oxidation States

Table 2

V Cr Mn Fe Co Ni
I I
II II II II II II II II II
III III III III III III III III III
IV IV IV IV IV IV IV
V V V
VI VI VI
VII

Formation of Complexes

6]Cl 6 6

Size of Atoms

Y
17.3

Density

Melting and Boiling Points

Ionization Energies

Colour

Table 3

Colour absorbed Complimentary colou


Violet

Red

Table 4
Number of unpaired electrons Metal ions Colour observed
0
+

1
2 Ni
3 Co
3 Cr
4 Fe
4 Cr
5 Mn
5 Fe
17.4

Magnetic Properties

n (n + 2) B

Catalytic Activity

Table 5

Fe
Pd
Cr
Ni
PdCl
Pt
Mn

Lanthanides and Actinides

n n
17.5

MULTIPLE CHOICE QUESTIONS

General Characteristics

Mn is

6]

Oxidation State
17.6

Cr

to Cr

Magnetic Properties

B B

B n B n(n + 1) B n(n + 2) B (n + 1)(n + 2)

Mass of a substancein a magnetic field


Actual mass of the compound

Mass of a substancein a magnetic field


Actual mass of the compound

is

V is

is
B B B
is
B B B
Cr is

Fe is
17.7

V Cr Fe Co

Mn
Colour

V Fe

Characteristics of Compounds
17.8

Cr is
Cr Cl

Cr

Cl6

Cl6 is
Cl Cl
Cl Fe … Fe Cl ]+ ]
Cl Cl
Cl Cl
Cl Cl Cl
Fe Fe Cl Fe Cl F
Cl Cl Cl
Cl Cl
17.9

Cl

Inner Transition Elements

x
x
La Ce and
Nb and Mo and

Sm 66Dy is

La

+
17.10

Ce Pr Sm 66Dy

ANSWERS

HINTS AND SOLUTIONS

Mn
17.11

Mn + Mn
+7
Mn O -4 Mn
+7 +4
Mn O -4 Mn O 2
+7 +6
Mn O -4 Mn O 24-
+
Cr

B 1(1 + 2)
V B 2(2 + 2)
6
Fe
B 4( 6)
B 4( 6)
Cr B 3(5)
Fe B 5(7)

B 6(8)

V Cr Fe and Co

only V

Cr
+

æDæ
Æ
17.12

n n
Sm 66Dy

La La

and

n n
and Dy

ANNEXURE

A Few Common Elements and Their Important Salts


Iron

6 6 6

+C
Fe
17.13

Copper, Silver and Gold

n n

S to

S · Sb S

] ]

] ]
17.14

] +C N
]



exposure
 

373 K
 

423 K


Zinc and Mercury


n n

Cl

Important Salt of Manganese


Potassium Permanganate

Æ
Æ +e
Cl + Cl Æ
Æ

Æ Mn
17.15

+e Æ
Æ

by in I to

Important Salt of Chromium


Potassium Dichromate

+
Æ Na Cr +

Na Cr Æ Cr

Cr +
+ 6e Æ
to Fe to I and S

Cr Æ Cr
Æ ¥
Cr Æ Cl
Cr Æ Cl
Cl
Cl Æ
Cl Æ

Pb Æ +

Cr Æ
17.16

+
Cr
Cr
and Cr
O 2 O O 2

Cr Cr Cr
O O O O O

O O O

MULTIPLE CHOICE QUESTIONS

Iron

is
17.17

Fe

6 6

Copper, Silver and Gold

6 6
n n n n n n n n
17.18

S · Sb S

S
17.19

] +C N

]
17.20

Æ +
+e Æ +
+e Æ +
+e Æ

Zinc and Mercury

+N

smaller
17.21

Cl

Cl

]
]

is
17.22

KMnO4 and K2Cr2O7


is

is

Cr is

Cl

Cr

ANSWERS
17.23

HINTS AND SOLUTIONS



exposure
 

373 K
 

423 K


+N

+N
17.24

373 K 723 K above 1000 K


æææÆ æææÆ æææææÆ
-6 H 2 O - H2O

MULTIPLE CHOICE QUESTIONS FROM AIEEE AND JEE MAIN

[2003]

[2003]

and Cr
[2003]
Z

[2004]

[2005]

[2005]

[2005]

[2006]
mB

[2006]

a Ca
[2006]
17.25

[2007]

[2007]

[2008]

[2009]

[2009]

[2011 cancelled]

[2012]

O , heat CO, 600∞C CO, 700 °C


Fe ææææ
2
Æ Fe3O 4 ææææÆ FeO æææææ
Æ Fe
dil. H SO H SO , O heat
Fe ææææ
2 4
æÆ FeSO 4 æææææ
2 4 2
Æ Fe 2 (SO 4 )3 æææ
Æ Fe
O , heat dil. H SO heat
Fe ææææ
2
Æ FeO ææææ
2 4
Æ FeSO 4 æææ
Æ Fe
Cl , heat heat, air Zn
Fe ææææ
2
Æ FeCl3 ææææ
Æ FeCl2 ææÆ Fe [2014]
17.26

[2014, online]

[2014, online]

[2014, online]
Cr
[2014, online]

[2014]
6

[2015, online]
false

Cr Cr Cr
[2015, online]

s s* transition [2015]

+
[2016, online]

and Cr and Co and Cr and Co


[2016, online]

[2016]

and N [2016]

ANSWERS
17.27

HINTS AND SOLUTIONS


6 6 6

+
Cr

mm = n ( n 2) n
ation d s .

L n (n + 2) mB
L 2 (4) mB mB

n+l

and FeCl
e

Cr2 O72- + 14H + + 6e- Æ 2Cr 3+ + 7H 2 O


S2- Æ S + 2e- ] × 3
Cr2 O72- + 14H + + 3S2- Æ 2Cr 3+ + 7H 2 O + 3S

Cr S Æ Cr

nt n (n + )
mB
as V
17.28

and Cl
Cr Cr

Al 6

Æ +N
Æ
18
Coordination Chemistry and
Organometallics

3+
3) 6]
ligands
bound around a metal ion is called the coordination number

Mono or Unidentate Ligand – – –


2 3
– –
2

Bidentate

Tridentate

Tetradentate
Pentadentate

Hexadentate

Nomenclature of Coordination Compounds

– – – 2– –
2)

3 2
+
2 3
18.2 Complete Chemistry—JEE Main

— 2 —
— —

3) 6 3
2+
3) 5]

4 3) 4 3

2) 3 3) 3

2 4

K3 5

5 5) 2

6 6) 2

Explanation of Formation of Coordination Compounds

[Cr(NH3)6]3+
5
3
2 3
hybrid orbitals
Coordination Chemistry and Organometallics 18.3

3d 4s 4p
Cr atom

Cr3+ ion

Cr3+ in
[Cr(NH3)6]3+
d2sp3 hybridization, six pairs of electrons
from six NH3 molecules

[CoF6]3–

3d 4s 4p
Co atom

Co3+ ion
4d
Co3+in
[CoF6]3 -
sp3d2 hybridization, six pairs of electrons
from six F - ligands

3–
6]

[Ni(CN)4]2– and [NiCl4]2–

3d 4s 4p
Ni atom

Ni2+ ion

Ni2+ in
[Ni(CN)4]2 -
dsp2 hybridization, four pairs of
electrons from four CN - ligands

Ni2+ in
[NiCl4]2 -
sp3 hybridization, four pairs of electrons
from four Cl - ligands

2– 2 2–
4] 4] is tetrahedron
3 –
18.4 Complete Chemistry—JEE Main

[Ni(NH3)6]2+
3d 4s 4p
Ni atom

Ni2+ ion
4d
Ni2+in
[Ni(NH3)6]2+
sp3d2 hybridization, octahedral
2+
3) 6]

Ni(CO)4
3d 4s 4p
Ni atom

Ni in
Ni(CO)4
sp3 hybridization

Inner and Outer Orbital Octahedral Complexes


2 3
3 2
d

3+ 2 3 3– 3 2
Example 3) 6] 6] d


Note
Isomerism in Coordination Compounds

Ionization Isomerism

3) 5 4 3) 5 4

Linkage Isomerism

3) 5 2 2 3) 5 2

Coordination Isomerism

3+ 3– 3+ 3–
3) 6] 6] 3) 6] 6]
Coordination Chemistry and Organometallics 18.5

Geometrical Isomerism

cis isomer while those


trans

Geometrical Isomerism in Square Planar Complexes


4 3
2b 2 2 cis – trans

trans
2 cis trans-

Example of Geometrical Isomers in Square Planar Complexes

H3N Cl H3 N Cl

Pt Pt

H 3N Cl Cl NH3
cis-isomer trans-isomer

H3 N Br H3 N Py H3 N Py

Pt Pt Pt

Cl Py Cl Br Br Cl
1
Geometrical Isomerism in Octahedral Complexes

cis 5
2
M
4
3
trans

6
Fig. 18.1
18.6 Complete Chemistry—JEE Main

Number of Isomers of a Complex


trans

trans
4
C2
trans

2 2b 2 2b 2c 2 3b 3

6 5 3
4b 2 3b 3 4

cis
trans
and 6)

cis cis
trans trans

cis

isomer
trans

∑ 2b 2c 2
trans

Trans positions; (enantiomer)


aa; bb; cc aa; bc; bc ab; ab; cc ab; ac; bc ac; bb; ac
Coordination Chemistry and Organometallics 18.7

∑ 2b 2 six trans

(and its enantiomer) (and its enantiomer)

∑ 6
C2

∑ 2b 2 2a 2 2
cis and trans cis

∑ 3 different donor
cis– and trans
18.8 Complete Chemistry—JEE Main

Example of Geometrical Isomers in Octahedral Complexes


cis trans

Cl 2+ Cl 2+
H3N Cl H3 N NH3

Co Co

H 3N Cl H3 N NH3
NH3 Cl

cis-isomer trans-isomer

Optical Isomers

Stability of Coordination Compounds

– – –
Coordination Chemistry and Organometallics 18.9

– – 2– – – – – 2– – –
2 3 2
– –
2

2+ 2+
and
2+

2+ 2+ 2+ 2+ 2+ 2+

2+

Crystal Fields Splitting in the Complexes


xy xz yz z2
dx2–y2
xy xz and dyz x2–y2 and dz2
eg g 2g

2g
followed by eg

Colour
18.10 Complete Chemistry—JEE Main

Table 18.1 The Relationship between the Colour Absorbed and Complimentary Colour Observed

Colour Absorbed Complimentary Colour


Violet Yellow–green
Yellow
Orange to red
Green to yellow green
Orange

Table 18.2 Colour of Some Transition Metal Ions


Number of Metal Colour
unpaired electrons ions observed
+ 2+ 2+ 4+

2+ 4+

2+ 3+
2 Green
2+
3
3+
3 Green
2+
4 Green
2+
4
2+
5
3+
5 Yellow

Applications of Coordination Compounds

2+ 3+ 2+

Organometallic Compounds

s bonded 2 3) 6 6 5) 6 3) 4 2 5) 4 2 5) 2
p bonded
s and p bonded
p h2 h5 and h6

MULTIPLE CHOICE QUESTIONS

General Characteristics
Coordination Chemistry and Organometallics 18.11


2

3) 6 3

3) 4 2

3) 3 3 3) 4 2 3) 5 2 3) 6 3

molecules

3) 4 4 4]
2 4 Al2
· 4 2 4 6]

2+
3) 6] is

– – –

is
– – – – – – – – – – – –

Geometry of Complex
3) 6 3 is

2 3

3) 6 3
2 3
18.12 Complete Chemistry—JEE Main

3–
6]
2 3
3 2
d

2– 2–
4] 4]
2– 2–
4] 4]
2– 2–
4] 4] differ in the geometry
2– 2–
4] 4]

2–
4 4]
2–
4 4]
2–
4 4]
2–
4 4] differ in the nature of ligands
2+
3) 6]
3 3 2 3 3 2
d
4–
6]
3 3 2 3 3 2
d
2

Nomenclature
4] is

3 2) 6] is

2 2

3) 4 2 4 is

3) 3 2

3) 4 4]

Magnetism of Complexes

3– 2– 2+ 2–
6] 4] 3) 6] 4]
Coordination Chemistry and Organometallics 18.13

— — 3) 6 2 is
—–
÷ ÷ ÷
4–
6] is

4–
— 6] is
—– —–
÷ ÷ ÷24 bohr magneton
2– 2– 2–
4] 4] 4]

3) 6 3
2 3

2+ 2– 4–
3) 4] 4] 4 6]

3+ 2+ 2+ 2+
2O)6] 2O)6] 2O)6] 2O)6]

2 2O 7 4) 2 6 4 3 6]

Isomerism in Complexes

2– – 2–
4 3 3 2 2 2 en

2 en+2 2 en+2 2
+
3) 4 2
+
3) 4
+
2en2 is

3 3) 3 is


2 2 is

2 2en is

3) 5 4 3) 5 4

3) 5 2 2 3) 5 2

3) 6 6 3) 6 6

3) 2 2] can form

3) 2 2] can form

2–
3 2 3] ion is
18.14 Complete Chemistry—JEE Main

2 2

3 2

2a 2

Additional Problems

) ion

2– 2– 2– 2–
4] 4] 4] 4]

3+ 2+
3) 6] 3) 6]

– – –
2+ 2+
3] 4] 3) 4]

2 3) 6 3) 4 2 5) 2

3) 4

5 6 2 2
5 5) 2 6 6) 2 2 4 3 2 4) 2 2]

2– 2–
4] 4]
2– 2–
4] 4]
2– 2–
4] 4]
2– 2–
4] 4] are stable
Coordination Chemistry and Organometallics 18.15

– – – – – – – – – – – –

– – –
3–
6] 4

3– 3+
6] 3) 6]
3– 3+
6] 3) 6]
3– 3–
6] 3) 6]
3– 3–
6] 3) 6]

ANSWERS

HINTS AND SOLUTIONS

O O

3 O C CH2 CH2 C O 5
1 2
N CH2 CH2 N
4 O C CH2 CH3 C O 6

O O

– –
is strongest ligands
3+

2– 2–
4] 4]
2–
4 4]
2+ 8 3 2
d
18.16 Complete Chemistry—JEE Main

3d 4s 4p 4d

sp3d2 hybridization
2+ 4 2 3

3d 4s 4p

d2sp3 hybridization

dsp2 hybridization

2–
4]

n m= n(n + 2) m = 8m

m= n(n + 2) m = 2(2 + 2) m = 8 m
2+ 6 2+ 7 2+ 9
)

3d 4s 4p

d2sp3 hybridization
n(n + 2) m B
9 8 7
2 2 3
and d2 3
3) 4]
2+
3+ 2+ 2+ 2+ 3 6 9
6

trans trans
trans

in three ways in the trans


2 a2 cis trans
Cis

– – –
Coordination Chemistry and Organometallics 18.17

and p
2+

– – –

– –

MULTIPLE CHOICE QUESTIONS FROM AIEEE AND JEE MAIN

4 4
[2003]
3) 5 3
3
is
3) 4 2◊ 3 3) 5 2
3) 3 3] ◊ 2 3 3) 4 2 ◊ 3 [2003]
2+
3) 4]

+
4 3 molecule are not

2 [2003]

[2004]

4– 4– 3+ 2+
6] 6] 3) 6] 3) 6] [2004]

[2004]

+ 2+ 2+ +
3) 4 2] 3) 5 3) 2 2 2]
[2004]

2– 2– 4– 2– 4– 2–
4] 4] 6] 4] 6] 4]
4– 2– 2– 4– 2– 2–
6] 4] 4] 6] 4] 4]
[2004]
18.18 Complete Chemistry—JEE Main

3– 3– 3– 3–
6] 6] 6] 6]
[2005]
3 6] is

[2005]
2 3) 5 2 is

[2006]
– 2–
4]

[2006]
5
s p s and p [2006]

2– 2– 2– 2–
4] 4] 4] 4]
[2007]
Do be the

3+ 3– 3– 3+
3) 6] 6] 2O 4) 3] 2O)6] [2008]
2 2O 4 2

[2008]

3) 5 3 4 3) 5 4 3 2 3) 4 2 2 3) 4 2
3) 4 4 3) 4 4 )
3 2 2 3) 2 2]
[2009]

3+ 3+ + 2+
2O)4 2 3) 2] 3) 3 3) 2]
[2009]

2+ 2+ 3+ 3+
2] 3) 2] 3] 2O)4
[2010]
2–
4] is

[2011 (cancelled)]
3) 6 3

2 3

[2011 (cancelled)]

3
– –
3 3
– –
3 3 [2011]
Coordination Chemistry and Organometallics 18.19


3 3 2 2 4] 2 [2012]

3+ +
3] 2 2]
+
3) 3 3] 3) 2 2] [2013]
3+
2 3 4 in
gands
is
2 4 3 4 3 2 3 2 4 3 2 4
[2014]

4 Æ K2 4] + K2 4
2 Æ 2O
3) 5]
+ +
Æ 2+ +
4

excess NaOH
2O)6]
2+ 4–
æææææÆ 2+
2O [2014]

AgCl Ø + 2NH3  [Ag ( NH3 )2 ] + Cl-


+

3 3 4
[2014]

K2 4]
2-
5]

3) 2 4]

3) 4 2 4
[2014, online]
3) 6 3

3+ - 3+
3) 6] 3 [2014, online]
D
- -
3 [2014, online]

3+ 4+ 3+ 2+
2O)6] 3) 6] 3) 6] 3) 6]
[2014, online]
3 4
3 2

[2014, online]
2-
4]

3 2 2 3

[2014, online]
18.20 Complete Chemistry—JEE Main

3+
3 2 2
d d2 3 3
d [2014, online]
3- 3-
6] 6]

3- 3-
6] 6]
3- 3-
6] 6] [2014, online]

– –
4 [2015, online]

2+ 2+ 2+
2O)5 3] 2O)4 3) 2] 2O)3 3) 3] is

[2015, online]
2

4– 3– 2– 2+
6] 6] 4] 2O)6]
[2015, online]
2g and eg
3– 2+
6] 3) 6]
4– 3–
6] 6] [2015, online]
+
3 2 is

[2015]
not
2 6 3 2) 6]
4) 3 3O )4 4 [2015]
3
2 5 2 3 6 2O)5 2 2O)6 3
[2016, online]

2+ 2+ 3+ 2+
2O)6] 2O)6] 2O)6] 2O)6]
2+ 2+ 3+ 2+
2O)6] 2O)6] 2O)6] 2O)6]
2+ 2+ 3+ 2+
2O)6] 2O)6] 2O)6] 2O)6]
2+ 2+ 3+ 2+
2O)6] 2O)6] 2O)6] 2O)6] [2016, online]

3) 6 3 3) 2 2 3) 4 2 3) 5 2
[2016, online]

2+ 2+ 2+ 2+
2O)6] 2O)6] 2O)6] 2O)6]
2– 2+ 2+ 2–
4] 2O)6] 2O)6] 4] [2015]

cis 2 2 trans 2 2
3) 4 2 3) 3 3] [2016]
Coordination Chemistry and Organometallics 18.21

ANSWERS

HINTS AND SOLUTIONS


– +

3d 4s 4p
2+
26Fe ion in complex d2sp3 (inner)

25Mn
2+
ion in complex d2sp3 (inner)

3+
27Co ion in complex d2sp3 (inner)

28Ni
2+
ion in complex sp3d2 (outer)
2+
3) 6]

+
2 2]

3d 4s 4p Hybridization
2+
25 Mn sp3

2+
27 Co sp3

2+
26 Fe d2sp3

2– 2– 4–
4] 4] 6]

3+ 5 3+ 6 3+ 3 3+ 4
26 27 24 25
18.22 Complete Chemistry—JEE Main

3–
6]

8 2 2–
4]

3d 4s 4p

Paramagnetic sp3 hybridization


nature (tetrahedral arrangement)

M C O M C O

s-bonding p-bonding
2+ 2+ 2+ 6 7 8
2
3 2– 2– 2–
4] 4] 4] are
2–
4]
2 2+

Do
– – – – – – –
2 3 2

Do
2–
2O 4 ¥
2– –
2O 4 2

3+
2 3) 2]
Coordination Chemistry and Organometallics 18.23

3+ 3+
en en
NH3 H3N
Co Co
NH3 H3N
en en
2+ 2+ 2+
2] 3) 2] has 3d
3

3+ 3+
3] 2O)4en]
3+
3]

2+ 8 2–
28 4] is
3d 4s 4p
Æ
Æ
Æ
Æ
Æ
Æ
Æ
Æ

sp3 hybridization
Electrons from the four chlorines

m= n(n + 2) m = 2(2 + 2) m = 8 m m
3+ 3
24

3d 4s 4p
Æ
Æ
Æ

d2 sp3 hybridization
Electrons from the six NH3 species
2 3

3
3
3+ –

3) 3 3
18.24 Complete Chemistry—JEE Main

4 2 3 3 2 4

4 3 4
2 2O

3
2-
5]
3+

- -
3

5c3+ 22
4+
23
2 2+
)
-

È ˘ - È ˘
ÍM (NH3 ) (SO 4 )˙ Cl and ÍM ( NH3 ) Cl˙ SO 4 s
ÍÎ (A) ˙˚ Î ( B) ˚

2+
28 is 3d8
3d 4s 4p

2+
in the

3+ 6 3+
27 27

3d 4s 4p

3+
3+

d2sp3
3+ 2 3
Coordination Chemistry and Organometallics 18.25

3+
ion is 3d5
3d 4s 4p

3-
6]
3d 4s 4p

d2sp3 hybridization
3-
6]
3d 4s 4p 4d

sp3d2 hybridization

4
2+
2O)5 3]
2+ 2+
2O)4 3) 2] 2O)3 3) 3] do show geometrical isomerism
NH 3 NH3
H2O NH3 H2O OH2
Ni Ni cis
H 2O OH2 H2O OH2
OH2 NH3
cis trans

NH3 NH3
H2O NH3 H 2O NH3
Ni Ni trans
H2O NH3 H2O OH2
OH2 NH3
cis trans

2+
2O)6]

2O)6]
2+ –
Æ 4]
2–
2O

3+
27 3d6 27
2+
3d7 25
2+
3d5 26
3+
3d5
– –
and eg
2g
3 2
2g and e g 2g) g)
18.26 Complete Chemistry—JEE Main

+
trans
a c a b a b

d b d c c d

2 6

3
3

3) 6 3

24 3d4 ≠ ≠ ≠ ≠
3 2
d
6 3 2
26 3d ≠Ø ≠ ≠ ≠ ≠ d
5 3 2
25 3d ≠ ≠ ≠ ≠ ≠ d
7 3
27 3d ≠Ø ≠Ø ≠Ø ≠
2+ 2+
2O)6] 2O)6]

cis 2a 2

N – N
N Cl Cl N
Co Co
N Cl Cl N
N N
Mirror
19
Nuclear
Chemistry

The natural nuclear changes are associated with the emission of three different types of radiations namely a-rays (i.e.
2He particles), b-rays (i.e.–1e) and -rays (i.e., electro-magnetic radiations of wavelength of the order of 0.1 nm). This
4 2+ 0

phenomenon is known as radioactivity. The nuclear changes result into the following transformations.
m m-4
nA n-2 B + 42 He
m m 0
nA n +1 B + -1 e
m *
nA
m
nA +g
While expressing the nuclear reactions, the charges on the involved species are not mentioned. As a nuclear reaction
proceeds, extra electrons within an atom lose energy so as to bring the atom to a stable state; and positive ions eventually
pick up electrons. From the equations given above, it is obvious that the emission of an a-particle results in the formation
of a daughter element which occupies a position two groups to the left of that of the parent element in the periodic table,
whereas in b-particle emission, the daughter element occupies a position one group to the right of the parent element.
This fact is known as the group displacement law.
The stability of a nucleus depends upon the ratio of protons and neutrons. If this ratio of a nuclide lies away from the
corresponding stability ratio, it results in an unstable nuclide which tries to attain stability by the spontaneous emission
of an a or a b-particle along with the emission of g-radiations.
The loss of mass when a given nucleus is formed, starting from the appropriate number of neutrons and protons,
is known as mass defect. This loss of mass appears in the form of energy according to the Einstein equation E = mc2.
A radioactive disintegration series involves the transformation of the parent radioactive element into the nonradioactive
end product. There are four such series known as thorium (4n), neptunium (4n + 1), uranium (4n + 2) and actinium
(4n + 3) series. Of these, neptunium or (4n + 1) series is man-made while others occur in nature. The mass number
changes only when an -particle is emitted imparting a change of 4 units. In elements of 4n series, the mass numbers are
divisible by 4. In (4n + 1), (4n + 2), and (4n + 3) series, the mass numbers give a remainder of 1, 2 and 3, respectively,
when divided by 4. The starting and end elements of the four series are as follows.
232 208
Thorium (4n) Series 90Th 82Pb ;(– 6a, – 4b)
237 209
Neptunium (4n + 1) Series 93Np 83Bi ;(– 7a, – 4b)
238 206
Uranium (4n + 2) Series 92U 82Pb ;(– 8a, – 6b)
235 207
Actinium (4n + 3) Series 92U 82Pb ;(– 7a, – 4b)

l
log N = log N0 – t
2.303
19.2 Complete Chemistry—JEE Main

where N and N0 are the number of radioactive atoms present at time t and 0, respectively, and l is the decay constant.
The time required to reduce the initial number of atoms (or concentration) of a radioactive element to half its initial
value is known as half-life period and is given by the expression
0.693
t1/2 =
l
Nuclear reactions are processes in which a nucleus is converted into one or more different nuclei as the result of an
interaction with another nucleus or with an elementary particle. A nuclear reaction can be represented by an equation
similar to those used for ordinary chemical reactions. For example, the transmutation of nitrogen into oxygen by the
bombardment of a particles is represented as
14 4 17 1
7N + 2He 8O + 1H
In shorthand notation, the above reaction is written as 147N( , p)178O.

MULTIPLE CHOICE QUESTIONS

General Characteristics
1. Which of the following represents an alpha particle?
(a) Proton (b) Neutron (c) Electron (d) Dipositive helium ion
2. Which of the following represents a beta particle?
(a) Proton (b) Neutron (c) Electron (d) Dipositive helium ion
3. Which of the following represents gamma rays?
(a) Stream of dipositive helium ions (b) Electromagnetic radiation
(c) Stream of electrons (d) Cathode rays

(a) Alpha particles (b) Beta particles (c) Gamma rays (d) Protons
5. The emission of beta particles is from
(a) the valence shell of atom
(b) the inner shell of atom
(c) the nucleus due to the nuclear conversion proton neutron + electron
(d) the nucleus due to the nuclear conversion neutron proton + electron
6. The instability of a nucleus is due to
(a) high proton : electron ratio (b)high proton : neutron ratio
(c) low proton : electron ratio (d) low proton : neutrino ratio
7. Which of the following is expected to have a negative charge?
(a) Neutrino (b) Neutron (c) Positron (d) Antiproton
8. Muon is heavier than electron by
(a) 106 times (b) 207 times (c) 307 times (d) 407 times
9. A positron has a mass
(a) equal to proton (b) equal to electron (c) greater than electron (d) smaller than electron
10. Mesons are responsible for
(a) disintegration of the nucleus(b)repulsion between the protons within the nucleus
(c) attraction between nucleons (d) attraction between neutrons only
11. In a b decay, the ratio of p/n of the nucleus is
(a) increased (b) decreased
(c) not changed (d) changed but cannot be predicted
Nuclear Chemistry 19.3

Group Displacement Law


12. Group 15 element 21183Bi is transformed to
211
84Po. To which group does the element Po belong?
(a) 14 (b) 15 (c) 16 (d) 13
210
13. The element 84Po emits an alpha particle. The daughter element is
(a) 204
82Pb (b) 206
82Pb (c) 208
82Pb (d) 210
82Pb
14. The element of 27
12 Mg emits a beta particle. The daughter element is
(a) 23
10 He (b) 23
11Mg (c) 27
13Al (d) 23
11Na

15. In a radioactive decay 232 208


90Th is transformed into 82Pb. The number of alpha and beta particles emitted respectively
are
(a) 4, 6 (b) 4, 4 (c) 6, 6 (d) 6, 4
16. Which of the following is man made disintegration series?
(a) 4n series (b) 4n + 1 series (c) 4n + 2 series (d) 4n + 3 series
17. Which of the following is 4n series?
(a) Thorium series (b) Neptunium series (c) Uranium series (d) Actinium series
18. Which of the following is 4n + 1 series?
(a) Thorium series (b) Neptunium series (c) Uranium series (d) Actinium series
19. Which of the following is 4n + 2 series?
(a) Thorium series (b) Neptunium series (c) Actinium series (d) Uranium series
20. Which of the following is 4n + 3 series?
(a) Thorium series (b) Neptunium series (c) Actinium series (d) Uranium series
21. The end product of 4n + 1 disintegration series is
(a) 208
82 Pb (b) 209
83 Bi (c) 206
82 Pb (d) 297
82 Pb
22. For which radioactive series, lead is not the end product?
(a) 4n (b) 4n + 1 (c) 4n + 2 (d) 4n + 3
23. After the emission of one a-particle followed by one b-particle from the atom 238
92 X, the number of neutrons in the
daughter atom will be
(a) 142 (b) 143 (c) 144 (d) 146
24. Lead 207 is produced as the end product in
(a) 4n series (b) 4n + 1 series (c) 4n + 2 series (d) 4n + 3 series
Kinetics of Radioactive Decay
25. The half-life of a radioactive decay is given as
(a) t1/2 = 0.693/l (b) t1/2 = log 2/l (c) t1/2 = l /log 2 (d) t1/2 = l/0.693
where l is decay constant.
26. The disintegration rate of a radioactive element changes from an initial value of 10000 disintegrations per minute
to 2500 disintegrations per minute in 50 days. The half-life of the element is
(a) 25 days (b) 50 days (c) 75 days (d) 100 days
27. If N0 and N are the number of radioactive particles at t = 0 and at time t, then
1 N 2.303 N t N 1 N
(a) l = log 0 (b) l = log (c) l = log 0 (d) l = ln 0
2 N t N0 2.303 N t N
where l is the decay constant.
28. The activity of a sample has 40% as much radioactivity as present originally. If half life period of the radioactivity
is 5000 y, the life of the sample undergoing disintegration is
(a) 5000 y (b) 6000 y (c) 6667 y (d) 5667 y
19.4 Complete Chemistry—JEE Main

29. The activity of a sample of wood is due to the presence of 50% of 14C as compared to the original sample. If half
life of 14C is 5760 y, the life of the sample of wood is
(a) 5760 y (b) 5670 y (c) 6667 y (d) 5000 y
30. Carbon dating involves the counting of
(a) 12C isotope (b) 13C isotope (c) 14C isotope (d) 12C and 13C isotopes
31. The radioactivity due to 14C isotope (t1/2 = 5760 y) of a sample of wood from a dead tree was found to be nearly
one fourth of fresh wood. The tree died
(a) 5760 y back (b) 9090 y back (c) 11520 y back (d) 2880 y back
32. The reciprocal of radioactive decay constant is called
(a) half-life period (b) average life period
(c) natural life period (d) root mean square life period
Mass Defect and Binding Energy
33. The energy equivalent to 1 atomic mass unit is
(a) 921 MeV (b) 931 MeV (c) 941 MeV (d) 951 MeV
34. The expression of mass-energy conversion is
(a) E = m2c (b) E = mc2 (c) E2 = mc (d) E2 = mc2
35. MeV stands for
(a) milli electron volt (b) milli electron velocity (c) mill electron volume (d) mega electron volt
36. Binding energy can be calculated from the formula
931 MeV ˆ 9.31 MeV ˆ
(a) Binding energy = (Mass defect) ÊÁ (b) Binding energy = (Mass defect) ÊÁ
Ë 1amu ˜¯ Ë 1amu ˜¯

(mass defect/1amu) (Mass defect/1amu)


(c) Binding energy = MeV (d) Binding energy = MeV
931.05 9.31
37. The mass defect for the formation of 12C is 0.10242 amu. Its binding energy would be
(a) 95.35 MeV (b) 9.535 MeV (c) 95.35 eV (d) 9.535 eV
38. Energy released during the annihilation of one positron and one electron (me = 9.11 10–31 kg) is
(a) 8.2 10–14 J (b) 1.64 10–13 J (c) 2.46 10–13 J (d) 4.92 10–13 J
39. The mass defect of the nuclear reaction 104Be 10 0
5B +–1e is
(a) Dm = atomic mass of (105B – 104Be)
(b) Dm = atomic mass of (105B – 104Be) + mass of one electron
(c) Dm = atomic mass of (105B – 104Be) + mass of two electrons
(d) Dm = atomic mass of (105B – 104Be) + mass of three electrons
40. The mass defect of the nuclear reaction 58B 8
4Be ++10 e is
(a) Dm = atomic mass of (48Be – 85B)
(b) Dm = atomic mass of (48Be – 85B) + mass of one electron
(c) Dm = atomic mass of (48Be – 85B) + mass of two electrons
(d) Dm = atomic mass of (48Be – 85B) + mass of three electrons
41. The binding energy per nucleon of 37Li and 5927 Co is 6.44 and 8.57 MeV. Which of the following statements is
correct?
(a) Li is more stable than Co
(b) Li is less stable than Co
(c) Both Li and Co are equally stable
(d) The conversion of Co to Li is associated with release of energy
Nuclear Chemistry 19.5

42. If the binding energy per nucleon of 7Li and 4He are 5.60 MeV and 7.06 MeV, respectively, the energy associated
with the reaction 7Li + p æÆ 242 He is
(a) 17.3 MeV (b) – 17.3 MeV (c) 34.6 MeV (d) – 34.6 MeV
Nuclear Reactions
43. Which one of the following is an ( , n) type nuclear transformation?
(a) 75 4
33As + 2He
78 1
35Br + 0n (b) 73Li + 11He 7
4Be + 10n
(c) 45
21Sc + 01n 45
20Ca + 11H (d) 14
7N + 11H 15
8O +g
44. In the nuclear reaction
14 4 17
7N + 2He 8O + 11H
the projectile is
(a) 147N (b) 42H (c) 178O (d) 11H
45. Atom bombs are based on

(c) nuclear fusion (d) spontaneous chemical reaction


46. Hydrogen bomb is based on

(c) nuclear fusion (d) spontaneous chemical reaction


47. The material used for absorbing neutrons in a nuclear reactor is
(a) cadmium (b) radium (c) uranium (d) zinc
75 4 78 1
33As + 2He 35Br + 0n the projectile is
14
(a) 7N (b) 42He (c) 178O (d) 11H
9
4B + 11H 8 2
4Be + 1H the subsidiary particle is
(a) 49Be (b) 11H (c) 84Be (d) 21H
10
5B + 10n 7 4
3Li + 2He the target is
10 1
(a) 5B (b) 0n (c) 73Li (d) 42He
51. Which of the following is capture reaction?
(a) 199F + 11H 16 4 7 4
8O + 2He (b) 3Li + 2He
11
5B +g (c) 32
16S + 13H 34
17Cl + 01n (d) 94Be + g 8
4Be + 10n

(a) 16
8O + 21H 14
7N + 42He (b) 25
12Mg +g 24
11Na + 11H
75
(c) 33As + 24He 78
35Br + 01n (d) 235
92U + 01n 140
56Ba + 94 1
36Kr + 20n

53. Which of the following is a spallation reaction?


(a) 168O + 21H 14 4
7N + 2He (b) 63
29Cu + 24He 37
17Cl + 1601n + 1411H
(c) 21H + 21H 4
2He +g (d) 238
92U + 01n 239
92U +g
54. Which of the following reactions represents a fusion reaction?
238
(a) 92U + 01n 239
92U (b) 13H + 21H 4
2He + 10n
75
(c) 33As + 24He 78
35Br + 01n (d) 32
16S + 13H 34
17Cl + 01n
55. For the conversion 238
92U
234
90Th + 24He, Dm = – 7.639 × 10–30 kg. The energy released during the process is
(a) 4.14 109 J mol–1 (b) 4.14 1010 J mol–1 (c) 4.14 1011 J mol–1 (d) 4.14 1012 J mol–1
56. In a nuclear reactor, the speed of neutrons is slowed down by
(a) ordinary water (b) heavy water (c) zinc rod (d) mercury
19.6 Complete Chemistry—JEE Main

57. The 146 C in upper atmosphere is generated by the nuclear reaction


(a) 14
7N + 11 H æÆ 146 C + 01 e + 11 H (b) 14
7N æÆ 146 C + 01e
(c) 14
7N + 10 n æÆ 146 C + 11 H (d) 14
7N + 11 H æÆ 116C + 42 He
58. In nuclear reactors, cadmium is used to
(a) slow down neutron
(b) absorb neutron
(c) activate neutron
(d) absorbed energy released during nuclear reaction.

(a) 234U (b) 235U (c) 238


U (d) 239
U
60. In Breeder reactors, the coolant used is
(a) an alloy or Na and K (b) cadmium (c) graphite (d) heavy water

ANSWERS
1. (d) 2. (c) 3. (b) 4. (c) 5. (d) 6. (b)
7. (d) 8. (b) 9. (b) 10. (c) 11. (a) 12. (c)
13. (b) 14. (c) 15. (d) 16. (b) 17. (a) 18. (b)
19. (d) 20. (c) 21. (b) 22. (b) 23. (b) 24. (d)
25. (a) 26. (a) 27. (d) 28. (c) 29. (a) 30. (c)
31. (c) 32. (b) 33. (b) 34. (b) 35. (d) 36. (a)
37. (a) 38. (b) 39. (a) 40. (c) 41. (b) 42. (a)
43. (a) 44. (b) 45. (b) 46. (c) 47. (a) 48. (b)
49. (d) 50. (a) 51. (b) 52. (d) 53. (b) 54. (b)
55. (c) 56. (b) 57. (c) 58. (b) 59. (b) 60. (a)

HINTS AND SOLUTIONS


232 - 208
15. Number of alpha particles emitted = =6
4
Number of beta particles emitted = 82 – (90 – 6 ¥ 2) = 4
t0.5
26. 10,000 disintegrations ætæÆ 5000 disintegrations æÆ 2500 disintegrations
0.5

Obviously, 2t0.5 = 50 days. Hence, t0.5 = 25 days


0.693
28. l =
5000 y
2.303 [A] 2.303 ¥ 5000 y 2.303 ¥ 5000
t = - log = - log (0.4) = ¥ 0.401 y = 6667 y.
l [A]0 0.693 0.693
29. 50% of 14C means half of 14C has disappeared. Therefore, t = t0.5 = 5760 y.
-3
Ê 10 kg ˆ
33. E = mc2 = Á (3 ¥ 108 m s–1)2 = 1.49 ¥ 10–10 J
Ë 6.023 ¥ 1023 ˜¯
1 eV = (1e) (1 V) = (1.60 ¥ 10–19 C) (1 V) = 1.60 ¥ 10–19 J

1.49 ¥ 10–10 J Ê 1 eV ˆ (1.49 ¥ 10–10 J)


ÁË -19 ˜
1 amu 931.4 MeV
1.60 ¥ 10 J ¯
Nuclear Chemistry 19.7

Ê 931 MeV ˆ
37. Binding energy = (0.10242 amu) Á = 95.35 MeV
Ë 1 amu ˜¯
38. E = (2 me)c2 = (2 ¥ 9.11 ¥ 10–31 kg) (3 ¥ 108 m s–1)2 = 1.64 ¥ 10–13 J
10 10
39. The precise conversion is 4Be (4p, 6n, 4e) 5B (5p, 5n, 4e) + –10 e
Dm = {mass of (5p + 5n + 4e) + mass of an electron} – mass of (4p + 6n + 4e)
= mass of (5p + 5n + 5e) – mass of (4p + 6n + 4e)
= mass of (105B – 104Be)
8 8
40. The precise reaction is 5B(5p, 3n, 5e) 4Be(4p, 4n, 5e) + +10 e
Dm = {mass of (4p + 4n + 5e) + mass of a positron} – mass of (5p + 3n + 5e)
= mass of (4p + 4n + 4e) + mass of (electron + positron) – mass (5p + 3n + 5e)
= mass of (48Be – 85B) + mass of 2 electrons
7 1
42. 3Li + 1P 2 42He
(3p, 4n, 3e) (2p, 2n, 2e)
Energy of reaction = Energy released in the formation of product – Energy require to separate n ??? of reactants
= 8(7.06) MeV – 7(5.6) MeV = 17.3 MeV
55. Energy released per mole of reaction is
E = (Dm)c2 NA = (7.64 ¥ 10–30 kg) (3 ¥ 108 m s–1)2 (6.022 ¥ 1023 mol–1)
= 4.14 ¥ 1011 J mol–1

MULTIPLE CHOICE QUESTIONS FROM AIEEE AND JEE MAIN

1. The half-life of a radioactive isotope is three hours. If the initial mass of the isotope were 256 g, the mass of it
remaining undercayed after 18 hours would be
(a) 16.0 g (b) 4.0 g (c) 8.0 g (d) 12.0 g [2003]
A
2. Consider the following nuclear reactions: 238 92
X 4
Y N + 2 2 He; Y
X
B L + 2b
+

The number of neutrons in the element L is


(a) 142 (b) 144 (c) 140 (d) 146 [2004]
3. The half-life of a radioisotope is four hours. If the initial mass of the isotope was 200 g, the mass remaining after
24 hours undecayed is
(a) 1.042 g (b) 2.084 g (c) 3.125 g (d) 4.167 g [2004]
4. Hydrogen bomb is based on the principle of
[2005]
24
5. A photon of hard gamma radiation knocks a proton out of 12 Mg to form
23
(a) the nuclide 11 Na (b) the isobar of 23
11Na
(c) the isotope of parent nucleus (d) the isobar of parent nucleus [2005]
6. In the transformation of 238
92 U to 234
92 U, if one emission is an a-particle, what should be the other emission(s)?
(a) One b + and one b- (b) Two b- (c) Two b- and one b+ (d) One b- and one g
[2006]

ten times the permissible value, after how many days will it be safe to enter the room?
(a) 1000 days (b) 300 days (c) 10 days (d) 100 days [2007]
19.8 Complete Chemistry—JEE Main

8. Which of the following nuclear reactions will generate an isotope?


(a) neutron particle emission (b) positron emission
(c) a-particle emission (d) b-particle emission [2007]

ANSWERS
1. (b) 2. (b) 3. (c) 4. (a) 5. (a) 6. (b)
7. (d) 8. (a)

HINTS AND SOLUTIONS


1. 18 hour implies six half-lives. Hence, the mass remaining undecayed will be
256
m = 6 = 4.0 g
2
2. The correct nuclear reactions are
238
92 M Æ 230
88 N + 2 42 He and 230
88 N Æ 230
86 L + 2 +10 e
The number of neutrons in L is 144 (= 230 – 86).
3. Twenty four hours is equivalent to six half-lives. Mass of remaining radioisotope = (1/26) (200 g) = 3.125 g.
4. Hydrogen bomb is based on the fusion reactions involving the combination of small nuclei to form a larger nucles.
During this process, energy is released because the binding energy per nucleon of lighter elements is smaller than
those of the intermediate size.
5. The mass number and charge number both decrease due to the knocking out of proton.
6. The transformation reactions are
7. The fraction of radioactive material remaining undecayed after each successive half-life is as follows.
1 1 1 1
1 t Æ t
Æ t
Æ t
Æ
1/ 2 2 1/ 2 4 1/ 2 8 1/ 2 16
1/10th of the radioactive material will be present in between third and fourth half-lives, i.e. in between 90 days and
120 days. The minimum time after which it is safer to enter the room may be 100 days. This may be quantitatively

N0 ln 2 ln 2
ln = k t ; For a half-life; t1/2 = i.e. k =
N k t1/ 2
For N0 /N = 10, time required will be
ln N 0 / N Ê ln 10 ˆ t1/ 2 30
t= = Á ˜ t1/ 2 = = = 99.67 d
k Ë ln 2 ¯ log 2 0.3010

Thus, 100 days will be the minimum safer time to enter the room.
8. The emission of neutron does not change the atomic number of the element and thus generates an isotope.
20
Purification and
Characterization of
Organic Compounds
Quite often, one has to determine the molecular formula of a compound from the quantitative data on its elements. A
brief review of this is described in the following.
1. Carbon is estimated in terms of carbon dioxide
MC mCO2
¥ 100 = Ê 12 ˆ Ê mCO2 ˆ ¥ 100
Mass per cent of carbon in the compound = · ÁË ˜¯ Á
M CO2 mcompound 44 Ë mcompound ˜¯
2. Hydrogen is estimated in terms of water

2M H mH 2O
¥ 100 = Ê 2 ˆ Ê mH 2O ˆ ¥ 100
Mass per cent of hydrogen in the compound = · ÁË ˜¯ Á
M H 2O mcompound 18 Ë mcompound ˜¯

3. Nitrogen in the compound is estimated using either Dumas’ method or Kjeldahl’s method.
In Dumas method, nitrogen is converted into nitrogen gas. From the volume of gas collected, we get
mnitrogen ( pV / RT ) ( M N 2 )
Mass per cent of nitrogen in the compound = ¥ 100 = × 100
mcompound mcompound
If the volume of gas is measured at STP, then we can use the following expression.

{(VN 2 )STP / (22 400 cm3 mol-1 )}{M N 2 }


Per cent of nitrogen in the compound = ¥ 100
mcompound
In Kjeldahl’s method, nitrogen is converted into ammonia which is absorbed in sulphuric acid. The remaining sulphuric
acid is determined by titrating against a standard alkali solution.
If Vacid and Nacid are the volume and normality of acid solution taken to start with and Valkali and Nalkali are the volume
and normality of alkali solution used in the back titration, then we have
M N . mNH3
Mass per cent of nitrogen in the compound = × 100
M NH3 mcompound
mNH3
where = nNH3 = Vacid Nacid – Valkali Nalkali
M NH3
4. Halogen is estimated in terms of AgX
M X . mAgX
Mass per cent of X in the compound = × 100
M AgX mcompound
20.2 Complete Chemistry—JEE Main

5. Sulphur is estimated in terms of BaSO4


MS mBaSO4
Mass per cent of S in the compound = . × 100
M BaSO4 mcompound
6. Phosphorus is estimated in terms of Mg2P2O7 (magnesium pyrophosphate)
2M P mMg 2 P2O7
Mass per cent of P in the compound = . 100
M Mg 2 P2O7 mcompound
From the given percentages of elements, one can determine the empirical formula of the compound as described in
the following.
Divide each per cent by the respective relative molar mass. The resultant numbers give the relative ratio of atoms
present in the molecule. If fractions appear in the ratio, then one can multiply by the smallest number so as to get integral
ratio of atoms.
After determining empirical formula, one has to determine molecular formula, which is an integral multiple of
empirical formula. The integral multiple is given as
Molar mass of the compound
n=
Empirical molar mass of the compoound
Molar mass of the compound is determined by a suitable physico-chemical method, e.g. Victor-Meyer method,
volumetrical method, and colligative properties.
A few expressions to compute molar mass of a compound are as follows.
m2
1. From relative lowering of vapour pressure M2 =
(m1 / M1 ) (-D p1 / p1* )
K b m2
2. From elevation of boiling point M2 =
DTb m1
K f m2
3. From depression of freezing point M2 =
(-DTf ) m1
Ê m2 ˆ RT
4. From osmotic pressure M2 = Á ˜
ËV ¯ P
In all the above expressions, the subscripts 1 and 2 stand for solvent and solute, respectively.
5. From vapour density

hydrogen is 2 g mol–1, the molar mass of the gas will be


Molar mass of gas = (2 g mol–1) (vapour density)
6. From the silver salt method for cabroxylic acids
The carboxylic acid is converted into silver salt which on strongly heating produces silver. From the masses of
silver salt and silver, the molar mass of acid can be determined as follows.
(Msalt)eq = ÊÁ ¥ 108ˆ˜ g eq–1
mass of silver salt
Equivalent mass of silver salt,
Ë mass of silver ¯
Equivalent mass of acid, (Macid)eq = (Msalt)eq – 107 g eq–1
Molar mass of acid, Macid = (basicity) (Macid)eq
7. From the platinichloride method for organic bases
Organic bases form solid platinichlorides (or chloroplatinates) with chloroplatinic acid (H 2PtCl6). These, on
heating, leave behind platinum. Now, each chloroplatinic acid H2PtCl6 (==PtCl4 · 2HCl) contains 2 molecules of
HCl. Hence,
1
1 equivalent of salt 1 equivalent of H2PtCl6 1 equivalent of base (molar mass of Pt)
2
20.3

195
g eq–1
2
Equivalent mass of platinum salt
mass of platinum salt 195
(Msalt)eq = × g eq–1
mass of platinum 2
Equivalent mass of base

(Mbase)eq = (Msalt)eq – (MH2PtCl6)eq = ÈÍ( M salt )eq - g eq -1 ˘˙


410
Î 2 ˚
Mbase = (acidity) (Mbase)eq

Detection of Elements (N, S and Halogens)

extract of the given organic compound is prepared as described in the following.


A pea sized dry piece of freshly cut sodium metal is taken in an ignition tube. This piece is melted to shining globule

gently and then strongly till the lower end of the tube becomes red hot. The tube is then plunged and broken in about

is known as sodium extract.

Test For Nitrogen


To a little sodium extract (say, 2 mL), add a little solid ferrous sulphate. Boil the solution and add a little dilute sulphuric
acid. Appearance of prussian blue or green coloration indicates the presence of nitrogen in the organic compound.
Fuse
Na + C+N
 æææ
Æ Na+ + CN –
Ê from organicˆ
ÁË compound ˜¯

Fe2+ + 6CN– [Fe(CN)6]4–


3[Fe(CN)6]4– + 4Fe3+ Fe4[Fe(CN)6]3
prussian blue

Test for Sulphur


To a little sodium extract (say, 2 mL), add a few drops of acetic acid and lead acetate. Appearance of black precipitate
indicates the presence of sulphur in the organic compound.
2Na + S 2Na+ + S2–
(from organic
compound)
S2– + Pb2+ PbS
(black ppt)
Alternatively, a few drops of freshly prepared sodium nitroprusside is added to 2 mL sodium extract. Appearance of
purple coloration indicates the presence of sulphur.
S2– + [Fe(CN)5NO]2– [Fe(CN)5NOS]4–

Test for Nitrogen and Sulphur Present Together


To a little sodium extract (say, 2 mL), add a little dilute sulphuric acid and ferric chloride solution. Appearance of red
coloration indicates the presence of both nitrogen and sulphuric in the organic compound.
Na + C + N + S Na+ + CNS–
(from organic
compound)
20.4 Complete Chemistry—JEE Main

Fe3+ + 3CNS– Fe(CNS)3


red colour

Test for Halogens


To about 5 mL of sodium extract, add equivalent volume of concentrated nitric acid. Boil the solution till volume is
reduced to half. This ensures the evaporation of CN–, S2– or CNS– if present in the solution, since otherwise these ions
also show the test carried out for halogens. Cool the solution. To a portion of this solution, add silver nitrate solution.
Appearance of
(a) white precipitates soluble in NH4OH indicates the presence of chlorine.
(b) light yellow precipitates soluble in excess of NH4OH indicates the presence of bromine
(c) yellow precipitates insoluble in NH4OH indicates the presence of iodine.
Bromine and iodine may also be detected by carrying out the carbon tetrachloride (or carbon disulphide) test. To a
little solution obtained after treating with concentrated nitric acid, add about 1 mL of CCl4 (or CS2). Add a little chlorine
water or concentrated nitric acid. Shake the solution vigorously. Appearance of red (or violet) coloration in CCl4 layer
indicates the presence of bromine (or iodine) in the organic compound.

MULTIPLE CHOICE QUESTIONS

(a) simple distillation (b) fractional distillation


(c) distillation under reduce pressure (d) steam distillation

(a) distillation (b) steam distillation (c) crystallization (d) fractional distillation
4. Chromatorgraphy is based on the phenomenon of
(a) adsorption (b) absorption (c) solubility (d) chemisorption
5. Essential oils can be isolated by
(a) crystallization (b) steam distillation (c) sublimation (d) distillation
6. Sugar is decolourized by
(a) coal (b) carbon black (c) charcoal (d) coke
7. Two substances having slightly different solubility can be separated by
(a) distillation (b) factional distillation
(c) crystallization (d) fractional crystallization
8. Liebig method is used for the estimation of
(a) carbon only (b) hydrogen only (c) carbon and hydrogen (d) nitrogen
9. Per cent of carbon in an organic compound is given as
M C . mCO2 M C mcompound
(a) ¥ 100 (b) ¥ 100
M CO2 mcompound M CO2 mCO2
M CO2 mCO2 M CO2 mcompound
(c) ¥ 100 (d) ¥ 100
M C mcompound MC mCO2
where m is the mass of the substance of M stands for its molar mass.
10. Per cent of hydrogen in an organic compound is given as
MH mH 2O 2M H mH 2O
(a) ¥ 100 (b) ¥ 100
M H 2O mcompound M H 2O mcompound
20.5

M H 2O mcompound M H 2O mcompound
(c) ¥ 100 (d) ¥ 100
MH mH 2O 2M H mH 2O
11. Dumas method is used for the estimation of
(a) carbon (b) hydrogen (c) sulphur (d) nitrogen
12. The expression to compute per cent of nitrogen in an organic compound by the Dumas method is
(VN 2 )STP M N 2 (VN )STP mcompound
¥ 100 ¥ 100
2
(a) (b)
(22414 cm mol ) mcompound
3 -1
(22414 cm3 mol-1 ) M N 2

(22414 cm3mol-1 ) M N 22414 cm3 mol-1 mcompound


¥ 100 ¥ 100
2
(c) (d)
(VN )STP mcompound
2 ( )STP
VN 2
M N2

13. Kjeldahl’s method is used to estimate the element


(a) carbon (b) hydrogen (c) sulphur (d) nitrogen
14. The expression to compute per cent of nitrogen in an organic compound by the Kjeldahl’s method is
Ê MN ˆ Ê mcompound ˆ
(a) Á ˜ (Vacid Nacid – Valkali Nalkali) ¥ 100 (b) Á
Ë M N ˜¯
(Vacid Nacid – Valkali Nalkali) ¥ 100
Ë mcompound ¯

Ê MN ˆ Ê mcompound ˆ
(c) Á ˜ (Vacid – Valkali) ¥ 100 (d) Á
Ë M N ˜¯
(Vacid – Valkali) ¥ 100
Ë mcompound ¯

15. The Carius method is used for the estimation of


(a) carbon (b) hydrogen (c) halogens (d) nitrogen
16. The expression to compute the per cent of halogen X in an organic compound by the Carius method is
Ê mAgX ˆ M Ag MX mAgX
(a) Á ˜ ¥ 100 (b) ¥ 100
Ë M AgX ¯ mcompound M AgX mcompound

M AgX mcompound M X mcompound


(c) ¥ 100 (d) ¥ 100
MX mAgX M AgX mAgX

17. Molar mass of volatile compounds can be determined by


(a) volumetric method (b) Victor Meyer’s method
(c) depression in freezing point (d) elevation in boiling point
18. Molar mass and vapour density are related to each other through the expression
Ê 1 ˆ
(a) Msubstance = (2 g mol–1) (Vapour density) (b) Msubstance = Á -1 ˜
(Vapour density)
Ë 2 g mol ¯
Ê 1 g mol-1 ˆ
(c) Msubstance = Á ˜¯ (Vapour density) (d) Msubstance = Vapour density
Ë 2
19. 0.1890 g of an organic compound gives 0.287 g of silver chloride determined by Carius method. The per cent of
Cl in the compound is
(a) 18.7 (b) 37.5 (c) 56.2 (d) 75.0
20. An organic compound contains 90% C and 10% H by mass. Its empirical formula is
(a) C2H6 (b) C2H4 (c) C3H8 (d) C3H4
3
21. 0.246 g of the organic compound gave 22.4 cm of nitrogen gas at STP as determined by Dumas method. The per
cent of nitrogen in the compound is
(a) 11.38 (b) 17.07 (c) 22.76 (d) 34.14
20.6 Complete Chemistry—JEE Main

22. An organic compound contains 58.53% C, 4.06% H and 11.38% N by mass. The empirical formula of the
compound is
(a) C6H5N2O (b) C6H5NO2 (c) C5H6N2O (d) C5H7NO2
3
23. A monoacid organic base (0.0915 g) requires 15 cm of N/20 HCl for complete neutralization. The molar mass of
the base is
(a) 61 g mol–1 (b) 122 g mol–1 (c) 244 g mol–1 (d) 183 g mol–1
24. An organic compound (0.156 g) on Kjeldahl’s method for nitrogen estimation requires 16.25 cm3 of 0.1 M NaOH
for the remaining unneutralized acid in 60.25 cm3 of 0.1 N H2SO4 taken to start with. The per cent of nitrogen in
the compound is
(a) 39.26 (b) 78.52 (c) 49.26 (d) 59.26
3
25. A volatile liquid (0.146 g) displaced 30.5 cm of air at 305 K and 760 mmHg pressure. The molar mass of the
liquid is
(a) 117.8 g mol–1 (b) 118.8 g mol–1 (c) 119.8 g mol–1 (d) 120.8 g mol–1
26. A compound (0.244 g) containing sulphur yielded 0.9336 g of BaSO 4. The per cent of sulphur in the
compound is
(a) 25.46 (b) 52.46 (c) 64.25 (d) 46.52
27. An organic compound (0.45 g) on combustion yielded 1.1 g of CO2 and 0.3 g water. Besides C and H, nitrogen is
also present in the compound. The empirical formula of the compound would be
(a) CH2N (b) C2H3N (c) C3H4N (d) C2H5N
28. 0.1 g of an organic monobasic acid on complete combustion gave 0.2545 g of CO2 and 0.04428 g of H2O. For
complete neutralization 0.122 g of the acid required 10 cm3 of 0.1 M NaOH solution. The molecular formula of
the acid would be
(a) C7H6O2 (b) C6H7O2 (c) C7H7O2 (d) C6H6O2
29. Which of the following statements is true regarding the fractional distillation of a mixture of two liquids?
(a) The distillate collected is the liquid of lower boiling point
(b) The distillate collected is the liquid of higher boiling point

column
30. For which of the following compounds, Kjeldahl’s method can be used to estimate nitrogen?
(a) Aniline (b) Methyl amine (c) Urea (d) Toluidine
31. Which of the following compounds is not expected to exhibit Lassaigne’s test of nitrogen?
(a) CH3CN (b) CH3NH2 (c) CH3NO2 (d) NH2OH
32. Which of the following compounds is expected to give red colouration while performing Lassaigne’s test of
nitrogen?
(a) Urea (b) Thiourea (c) Aniline (d) Nitrobenzene

ANSWERS
1. (b) 2. (c) 3. (b) 4. (a) 5. (b) 6. (c)
7. (d) 8. (c) 9. (a) 10. (b) 11. (d) 12. (a)
13. (d) 14. (a) 15. (c) 16. (b) 17. (b) 18. (a)
19. (b) 20. (d) 21. (a) 22. (b) 23. (b) 24. (a)
25. (d) 26. (b) 27. (c) 28. (a) 29. (a) 30. (c)
31. (d) 32. (b)
20.7

HINTS AND SOLUTIONS


Ê M Cl ˆ Ê 100 ˆ Ê 35.5 ˆ Ê 100 ˆ
19. Per cent of chlorine = Á ˜ (mAgCl) Á ˜ = ÁË 143.5 ˜¯ (0.287) ÁË 0.189 ˜¯ = 37.57
Ë AgCl ¯
M Ë compound ¯
m
90 10
20. C : H :: : :: 7.5 : 10 :: 3.4 ; Empirical formula C3H4
12 1
Ê VN 2 ˆ Ê 100 ˆ 22.4 ˆ 100 ˆ
= ÊÁ ˜ (28) ÊÁ
Ë 0.246 ˜¯
21. Per cent of nitrogen = Á -1 ˜
(MN2) Á ˜ = 11.38
Ë 22400 cm mol ¯
3
Ë mcompound ¯ Ë 22400 ¯

58.53 4.08 11.38 26.01


22. C : H : N : O :: : : : :: 4.88 : 4.08 : 0.81 : 1.63
12 1 14 16
:: 6 : 5 : 1 : 2. Empirical formula = C6H5NO2
24. Initial amount of H+ = VM = (0.06025 dm3) (0.1 mol dm–3) = 0.006025 mol
Remaining amount of H+ = (0.01625 dm3) (0.1 mol dm–3) = 0.001625 mol
Amount of H+ reacted = (0.006025 – 0.001625) mol = 0.0044 mol
Mass of NH3 produced = (Amount of H+) (MNH3) = (0.0044 mol) (17 g mol–1) = 0.0748 g
Ê MN ˆ Ê 100 ˆ
= ÊÁ ˆ˜ (0.0748) ÊÁ
14 100 ˆ
Ë 0.156 ˜¯
Per cent of nitrogen= Á ˜ (mNH3) Á ˜ = 39.5
Ë M NH3 ¯ Ë mcompound ¯ Ë 17 ¯

25. n =
pV
=
(
(1 atm ) 30 . 5 ¥ 10- 3 L )
= 0.00121 mol
RT (
0.0825 L atm K - 1 mol- 1 (305 K ) )
m 0.146 g
M= = = 120.7 g mol–1
n 0.00121 mol

Ê MS ˆ Ê 100 ˆ 32 ˆ Ê 100 ˆ = 52.5


26. Per cent of sulphur = Á ˜ (mBaSO4) Á ˜ = ÊÁ ˜¯ (0.9336) ÁË ˜
Ë M BaSO4 ¯ Ë mcompound ¯ Ë 233 0.244 ¯

Ê MC ˆ Ê 100 ˆ Ê 12 ˆ Ê 100 ˆ
27. Per cent of C = Á ˜ (mCO2) Á ˜ = ÁË 44 ˜¯ (1.1) ÁË 0.45 ˜¯ = 66.67
Ë CO2 ¯
M Ë compound ¯
m

Ê 2M H ˆ Ê 100 ˆ
= ÊÁ ˆ˜ (0.3) ÊÁ
2 100 ˆ
Per cent of H = Á ˜ (mH2O) Á ˜ ¯ ˜ = 7.4
Ë M H2O ¯ Ë mcompound ¯ Ë 18 Ë 0.45 ¯

66.67 7.4 25.93


C:H:N :: : : :: 5.56 : 7.4 : 1.85 :: 3 : 4 : 1 ; Empirical formula = C3 H4 N
12 1 14
Ê MC ˆ Ê 100 ˆ
= ÊÁ ˆ˜ (0.2545) ÊÁ
12 100 ˆ
Ë 0.1 ˜¯
28. Per cent of C = Á ˜ (mCO2) Á ˜ = 69.4
Ë M CO2 ¯ Ë mcompound ¯ Ë 44 ¯

Ê 2M H ˆ Ê 100 ˆ
= ÊÁ ˆ˜ (0.04428) ÊÁ
2 100 ˆ
Ë 0.1 ˜¯
Per cent of H = Á ˜ (mH2O) Á ˜ = 4.9
Ë M H2O ¯ Ë mcompound ¯ Ë 18 ¯

69.4 4.9 25.7


C:H:O :: : : :: 5.78 : 4.9 : 1.61 :: 3.6 : 3 : 1 :: 7 : 6 : 2
12 1 16
Empirical formula = C7 H6 O2; Empirical molar mass = 122 g mol–1
Amount of acid = VM = (10 ¥ 10–3 dm3) (0.1 mol dm–3) = 10–3 mol
20.8 Complete Chemistry—JEE Main

m 0.122 g
Molar mass of compound = = -3
= 122 g mol–1
n 10 mol
Molecular formula = C7H6O2.

MULTIPLE CHOICE QUESTIONS FROM AIEEE AND JEE MAIN

1. The compound formed in the positive test for nitrogen with the Lassaigne solution of an organic compound is
(a) Fe4[Fe4(CN)6]3 (b) Na3[Fe(CN)6] (c) Fe(CN)3 (d) Na4[Fe(CN)5NOS]
[2004]
2. The ammonia evolved from the treatment of 0.30 g of an organic compound for the estimation of nitrogen was
passed in 100 mL of 0.1 M sulphuric acid. The excess of acid required 20 mL of 0.5 M sodium hydroxide solution
for complete neutralization. The organic compound is
(a) acetamide (b) benzamide (c) urea (d) thiourea [2004]
3. 1.4 g of an organic compound was digested according to Kjeldahl’s method and the ammonia evolved was
absorbed in 60 mL of M/10 H2SO4 solution. The excess sulphuric acid required 20 mL of M/10 NaOH solution
for neutralization. The percentage of nitrogen in the compound is:
(a) 3 (b) 5 (c) 10 (d) 24 [2015, online]
4. Sodium extract is heated with concentrated HNO3 before testing for halogens because:
(a) Ag2S and AgCN are soluble in acidic medium.
(b) Silver halides are totally insoluble in nitric acid
(c) S2– and CN–, if present, are decomposed by conc. HNO3 and hence do not interfere in the test.
(d) Ag reacts faster with halides in acidic medium [2016, online]

ANSWERS
1. (a) 2. (c) 3. (c) 4. (c)

HINTS AND SOLUTIONS


1. The Lassaigne test involves the reactions
Na + C + N Æ NaCN
Fe2+ + 6CN– Æ [Fe(CN)6]4–
Fe3+ + [Fe(CN)6]4– Æ Fe4[Fe(CN)6]3

2. Amount of H+ used for the neutralization of NH3


n = 2 (0.1 mol L–1) (0.1 L) – (0.5 mol L–1) (0.02 L) = 0.01 mol
Amount of N = Amount of NH3 = Amount of H+ used = 0.01 mol
Mass of N = (0.01 mol) (14 g mol–1) = 0.14 g

 0.14 g  (100) = 46.67%


Mass per cent of N in the compound =
 0.3 g 
Mass per cent of N in the given compounds are as follows.
20.9

14
Acetamide (CH3CONH2) × 100 = 23.73 Benzamide (C6H5CONH2) It is less than 23.73
59
28
Urea (NH2CONH2) × 100 = 46.67
60
The compound is urea.
3. Amount of NaOH reacted with excess of H2SO4 is
Ê1 ˆ
n1 = VM = (20 ¥ 10–3 L) Á mol L-1 ˜ = 2 ¥ 10–3 mol
Ë 10 ¯
Amount of H2SO4 reacted with NaOH is
n
n2 = 1 = 1 ¥ 10-3 mol
2
Amount of H2SO4 reacted with NH3 is
Ê1 ˆ
n3 = (60 ¥ 10–3 L) Á mol L-1 ˜ – 1 ¥ 10–3 mol = 5 ¥ 10–3 mol
Ë 10 ¯
Amount of NH3 (also amount of N in the compound) liberated is
n4 = 2n3 = 10 ¥ 10–3 mol
percentage of N in the compound is
n4 M N (1.0 ¥ 10-3 mol) (14 g mol-1 )
¥ 100 = ¥ 100 = 10%
mcompound (1.4 g )

4. Both S2– and CN– are decomposed by conc. HNO3 and hence no precipitation of these ions occur with AgNO3.
21
Some Basic
Principles

Concept of Hybridization
2 2
2p2

hybridization

sp Hybridization

diagonal or linear hybridization

Fig. 1 Structure of acetylene

sp2 Hybridization
2

trigonal
2
hybridization
21.2 Complete Chemistry—JEE Main

Fig. 2 Structure of ethylene

sp3 Hybridization
3
¢
3
tetrahedral hybridization

1s(H)

1s(H)
H

C 1s(H)
H H
H

sp3(C)

1s(H)

Fig. 3 Structure of methane

Comparison of sp, sp2 and sp3 Hybrid Orbitals of Carbon

Number of s (or p) orbitals mixed


Total number of orbitals mixed

2
3

Size of orbitals

3 2

Electronegativity of Orbitals

2 3
Some Basic Principles 21.3

Sigma and Pi bond

s
of pi (p
π bond

σ bond
Fig. 4 Illustration of s and p bonds

bonding molecular orbital


antibonding molecular orbital

σ* 1s

1s 1s

σ 1s

Fig. 5
—C in C2H6 s 2H s and one p ∫∫ C bond in
C 2H 2 s and two p

Bond Lengths
—C
∫∫ —

Molecule CH C 2H C 2H 2

3 2
< <

Homologous Series
2

IUPAC System of Nomenclature of Organic Compounds


21.4 Complete Chemistry—JEE Main

Nomenclature of Alkanes
2H 6 3H ) and butane
(C H
H 6H H
3 2H 3H )
Ê CH3CHCH 2 -- ˆ
Á | ˜,
sec Ê CH3CH 2 CH -- ˆ tert 3) 3C — Ë CH3 ¯
Á | ˜
Ë CH3 ¯

(a) Longest chain rule

(b) Lowest number rule

(c) Preference to the substituent with the least number of carbon atoms

(d) Preference to the greater number of substituents

(e) Alphabetical order of alkyl group

(f) Proper way of writing

6 7
CH3 CH2CH3
1 2| 3 4 5|
CH3—CH—CH—CH2—CHCH3
|
HC—CH3
|
CH3

Nomenclature of Alkenes

1 2 3 4 5
CH2 == CH—CH—CH == CH2
|
CH3

Nomenclature of Alkynes

CH3
5 4| 3 2 1
CH3—CH—CH2—C CH
Some Basic Principles 21.5

Nomenclature of Halogen Derivatives

5 4 3 2 1
CH3—CH—CH==CH—CH3
|
Br

Nomenclature of Alcohols

4 5 6
CH2—CH2—CH3
|
1 2 3
CH3—CH2— C—OH
|
CH2CH3

Nomenclature of Aldehydes

CH 3 O
| ||
CH3 CH CH 2— C —H
4 3 2 1

Nomenclature of Ketones

Nomenclature of Carboxylic Acid

CH3 O
| ||
CH3CH2CH2CCH2 — C — OH
|
CH2CH3

Nomenclature of Carboxylic Acid Derivatives


(a) Acid chlorides
2CH2
(b) Esters
3 2H
21.6 Complete Chemistry—JEE Main

(c) Anhydrides
O O
|| ||
2H C C C 2H
(d) Amides
3 2

Nomenclature of Ethers

6 5 4 3 2 1
CH3CHCH2CH2CHCH3
| |
CH3 OCH2CH3

Nomenclature of Amines
2
N

CH3 — CH— CH3 CH3 — CH— CH3


| |
NH2 HN—CH3
2-propanamine N-methyl-2-propanamine

Nomenclature of Cyanides or Nitriles

4 3 2 1
CH3CH2CH2CN

Nomenclature of Nitro Compounds

1 2 3 4 5
CH3—CH—CH2—CH—CH3
| |
NO2 CH3

Nomenclature of Sulphonic Acids

1 2 3 4
CH3—CH—CH2—CH—CH3
| |
SO3H CH2CH3

Nomenclature of Polyfunctional Groups


Some Basic Principles 21.7

—Cl , —Br , —I , —R , —OR , — OH


chloro bromo iodo alkyl alkoxy hydroxy
—NH2, —CO —, —NO2, —NO, — CN
amino oxo nitro nitroso cyano

OH CH 3
| |
CH3CH2 CH— CHCOOH
5 4 3 2 1
O
||
CH3CH2— C —CH2COOH
5 4 3 2 1
O
||
CH3CH2— C —CH2CH 2OH
1 2 3 4 5

CH3—CH ==CH — C CH
1 2 3 4 5
4 3 2 1
CH3— CH —CH2— C —OCH2CH3
| ||
OH O
5 4 3 2 1
CH3— C== C— CH2—COOH
| |
CH 3 CH 3
Nomenclature of Benzene Derivatives

(a) When substitution is in the benzene ring


CH3 CH3
Cl

CH3
1-chloro-2-methylbenzene 1, 3-dimethylbenzene
(b) When substitution is in one or more carbon attached to the ring

CH2Cl CH2OH CHO


OH

phenylchloromethane phenylmethanol 2-hydroxyphenyl-


methanal
(c) Some derivatives are named as per their common names
OH COCH3 COC6H5

phenol acetophenone benzophenone


21.8 Complete Chemistry—JEE Main

Isomerism

Structural Isomerism
Stereoisomerism

Structural Isomerism

(a) Chain isomerism


(b) Position isomerism

(c) Functional isomerism


(d) Metamerism
(e) Tautomerism

O O OH O
|| || | ||
CH3 — C — CH2 — C —OC2H5 CH3— C ==CH— C —OC2H5
keto form enol form


Stereoisomerism

(a) Geometrical isomerism

CH3
H3C CH3 H3C H
CH3
C == C C == C
CH3
H H H CH3 CH3
cis-2-butene trans-2-butene cis -1, 2-dimethylcyclopropane trans-1, 2-dimethylcyclopropane
In cis trans
(b) Optical isomerism

Optically active molecule

Asymmetric or chiral carbon


Enantiomers
Dextrorotatory isomer isomer

Laevorotatory isomer isomer

Racemic mixture
Some Basic Principles 21.9

Diastereomers

COOH COOH COOH


| | |
H—C—OH HO—C—H H—C—OH Plane of
| | ------|------- symmetry
HO— C—H H—C—OH H—C—OH
| | |
COOH COOH COOH
(+)-tartaric acid (–)-tartaric acid mesotartaric acid

Meso-compound

Racemisation

Resolution
Number of Optical Isomers
n

n n
N

* * 2
CH CH 2 N
n
(n
N + 2(n

* *
2CCH 2 N +2
n
(n (n
N ] + 2(n

* * *
2CCH CH CH 2 N ]+2

*
3CCH CHCH3 N *2
CH3 CH3 CH3 CH3
H OH HO H H OH HO H
H3C C CH3 H C H
H C H C
C C C C
H H
H H CH3 CH3

cis( )–isomers trans( )–isomers


21.10 Complete Chemistry—JEE Main

Fischer
projection†

CHO CHO
| |
HO—C—H H—C—OH
| |
CH2O CH2O
L-(–)-Glyceraldehyde D-(+)-Glyceraldehyde
COOH COOH
| |
HO—C—H H—C—OH
| |
CH3 CH3
L-(+)-Lactic acid D-(–)-Lactic acid

Supplementary Material
Fischer Projections

H H H
H3C COOH
C ≡≡ H3C C COOH H3C COOH

HO OH OH
(+)-Lactic acid

H OH
Rotate clockwise
H3C COOH HOOC CH3
by 180°
OH H

H H
Hold H steady
H3C COOH HO CH3
rotate the other
three clockwise
OH COOH
Sawhorse Projections


Some Basic Principles 21.11

1 2 3 4
C H3 C H 2 C H 2 C H3

2 3
C -- C

Hc
Hc Hd

1 3 Hd CH3
CH3 4 Hb
2
CH3 Ha
Ha CH3
Hb

Fig. 6a Fig. 6b
Newman Projections 3—CH3

3—CH3

H
H
H
H H

H
H H H
H H
H
Fig. 7a Fig. 7b
Flying Wedge Projection

COOH H Br
COOH H3C H3C
H
H
C C Br C C H
H H
H3 C OH HO
CH3 CH3 HO CH3
HO
(+)–Lactic acid (–)–Lactic acid

Fig. 8
Inter-Conversion of Projections
Fischer to Sawhorse to Newman and Vice Versa
21.12 Complete Chemistry—JEE Main

CH3 H H OH OH

OH H
H OH
H
H OH CH3
OH CH3
CH3 CH3 CH3
Fig. 9

CH3 HO CH3
H
H HO H
H OH H
CH3 H OH
OH OH
CH3
CH3 CH3

CH3
HO OH CH3
H OH H
H
H OH
HO H
H OH
CH3
CH3 CH3
CH3
Fig. 10
Note

Comment on the Eclipsed Form of Newmann Projection


meso
Fischer Projection to Flying Wedge Projection and Vice Versa

COOH COOH COOH

H OH
H OH
or

CH3 HO CH3 H3C H

bending of C—CH3 bond to bending of C—CH 3 bond


the right, C—OH bond is to the left, C—H bond
projected in front of the paper is projected in front of the paper
Fig. 11
Some Basic Principles 21.13

Sequence Rules for Specifying R and S

Æ2Æ clockwise
R
Æ 2 Æ
counterclockwise S R and S R from rectus
S from sinister
Assignment of R, S
R S

R or S

2 4
COOH H
1 4 Hold CH2OH 2 1
H2N H HOOC NH2
rotate the other three
3 counterclockwise 3
CH2OH CH2OH
≡≡

H H
2 1 2 1
HOOC NH2 HOOC NH2
C ≡≡ C

CH2OH CH2OH
3 3
S configuration

4
CHO 2
H
CHO
HO H 1 4 Hold R 2 1
≡≡ HO H OHC OH
H OH rotate the other
counterclockwise 3
3 R
CH2OH R S configuration
21.14 Complete Chemistry—JEE Main

2
R 4
H
4 1 Hold CH2OH 1 2
H OH HO R
rotate the other
3 clockwise CH2OH
3
CH2OH
R configuration
1
CHO
2
HO H
3 S R
H OH
4
CH2OH

Stereoisomer Enantiomer Diastereomers


(2R R) (2S S) (2R S S R)
(2S S) (2R R) (2R S S R)
(2R S) (2S R) (2R R S S)
(2S R) (2R S) (2R R S S)

Sequence Rules for E, Z Designation of an Alkene


E Z

Z Zusammen
E entgegen

CH3
O
H H3C CH Br
C == C H3C C OH
H C == CH2
H C == C
C == C H2C == C
H3C CH3 H H CH2OH
(E) -3-methyl-1, 3-pentadiene (E) -1-bromo-2-isopropyl-1, 3-butadiene (Z) -2-hydroxymethyl-2-butenoic acid

Homolytic and Heterolytic Cleavages

Homolytic Cleavage

A : B Æ A· + · B
Some Basic Principles 21.15

Heterolytic Cleavage

(CH3)3CBr (CH3)3C + + : Br –
carbocation
–
CH3CCH3 + : B CH3CC H2 + HB+
|| ||
O O
carbanion

Stability of Carbocation, Carbanion and Radicals

I
I

(CH3)3C + > (CH3)2CH + > CH3CH2+ > CH 3+


Cl3C + < Cl2CH+ < ClCH2+ < CH+3

CH2 == CH ⎯ CH2 ←→ CH2 ⎯ CH == CH2


CH2 CH2 CH2 CH2

←→ ←→ ←→

CH2 CH2 CH2 CH2

←→ ←→ ←→

+ + +
CH3CH2 (CH3)2CH (CH3)3C
primary (or 1°) secondary (or 2°) tertiary (or 3°)
21.16 Complete Chemistry—JEE Main

Electrophiles or Electrophilic Reagents

H+ + + + +
2 3C
+ +
2 3 3 2 3
Nucleophiles or Nucleophilic Reagents

Cl H2 3 2

Inductive Effect

bond moment
p d q) (d)
dq d C

—H

d+ d
CH3—CH2—

d
CH2CH3
d |d+ d
CH3CH2—Pb—CH2CH3
|
d
CH2CH3
+I and I Groups
I
I I I

I I
+I — CH3 —CH2CH3 —CH(CH3)2 —C(CH3)3
I — C 6H — — 3 — — — — — 2

Resonance Effect
Some Basic Principles 21.17

O O

R ⎯ CH ⎯ C ⎯ H ←→ R ⎯ CH == C ⎯ H
Fig. 12 Resonance stabilisation of a carbanion
— 2

O⎯H O⎯H NH2 NH2


←→ ←→

Fig. 13 Resonance donation of pi electrons

Acids and Bases


+

HA + B  A + HB+

pair and a
3 3 3 2

Strengths of Acids

HA + H2  A + H3 +

[A - ] [H 3O + ]
Ka K 2
[HA]
Ka equilibrium constant
standard euilibrium constant
Ka
Ka Ka
Ka
pKa Ka
Ka Ka Ka (or
Ka

Acids 3> > CH3 2 2H


pKa
Conjugate
bases Cl 3 < CH3 < C 2H
3
 CH3 + H+
21.18 Complete Chemistry—JEE Main

+I 3CH2 3) 2 3) 3

O O

H⎯C⎯O ←→ H ⎯ C == O

Resonance

resonating or canonical structures

N == N == O ←→ N ≡≡ N ⎯ O

resonance energy

←→ ←→ ←→ ←→

Kekule’s structures Dewar’s structures


Some Basic Principles 21.19

Types of Organic Reactions

Substitution Reactions

Free-radical substitution and Cl2


2
(a) Initiation step
Cl : Cl Æ 2 ·Cl
(b) Propagation steps
CH + ·Cl Æ CH 3· + HCl
CH 3· + Cl2 Æ CH3Cl + Cl·

(c) Termination steps


Cl· + ·Cl Æ Cl2
CH 3· + ·Cl Æ CH3Cl
Electrophilic Substitution Reactions

3
C6H6 + Br2 æææÆ C6H Br + HBr

(a) Formation of an electrophile


Br : 3 Æ Br +

(b) Electrophilic attack


H H
Br Br
+ Br+ → ←→

Resonance
hybrid
(c) Elimination of proton t
H Br

Br FeBr4
⎯⎯⎯→ + HBr + FeBr3

Nucleophilic Substitution Reactions

In S
(CH3)3 Æ (CH3)3

(CH3)3CBr ææææææÆ (CH3)3C + + Br


carbonium ion
21.20 Complete Chemistry—JEE Main

(CH3)3C + Æ (CH3)3

In S
CH3 Æ CH3

H H H
H
− rate
HO + C ⎯ Br ⎯⎯⎯⎯→ HO− C Br → HO ⎯ C + : Br −
H determining H
step
H H H
transition state

Addition Reactions

Electrophilic addition reaction


CH2 2 + HBr Æ CH3CH2Br
+

H—Br H+ + Br –
electrophile nucleophile
+
H + + CH2 == CH2 CH3—CH2
carbonium ion

CH3 ⎯ CH2 + Br− CH3—CH2 — Br


Nucleophilic addition reaction
C O
OH
|
CH3COCH3 + HCN CH3 — C — CH3
|
CN

H—CN H+ + CN :–
H3C H3C H3C H3C
C == O + CN : → C⎯O ; C⎯O + H → +
C ⎯ OH
δ+ δ− CN CN CN
H3C
CH3 CH3 CH3

Elimination Reactions
Some Basic Principles 21.21

— CH2— CH2—X ææææÆ — CH2 CH2

MULTIPLE CHOICE QUESTIONS

Hybridization

2
2 2 3

∫∫C—C 2
2

2 2 3 2

· 3


(a) H C 2 2C  3) 3 C 
3C

2 3
2 2—CH2—C∫∫ C —C
2 3 3 3 2 3

∫∫ 2
3 3 2 3 2 3

rect?
2 3 2 3 2 3 2 3

2 3 2 3 2 3 2 3

2H and C2H2
(a) CH > C2H > C2H2 (b) CH < C2H < C2H2 (c) CH > C2H < C2H2 (d) CH < C2H > C2H2
2 CH 2
p
2

s p

(a) C2H2 (b) C2H (c) C2H6 (d) C2H2Br2

2
3

2H and C2H2
21.22 Complete Chemistry—JEE Main

Nomenclature

CH3
|
H3C— C— CH CH2
|
CH3

compound CH2 3) 2

d
pound
H
CH3
C ⎯ CH
CH3
NH2

(a

CH2 CH3

CH3 CH2 CH2 CH ⎯ CH ⎯ C ⎯ CH2 CH3

CH CH3 CH2 CH3

CH3 CH3

(a) CH3CH2CH2 CH 3C∫∫C CH


| |
Cl CH3
(c) CH3CH2 3 3 CH CH2CH2
|
CH3
CH3
|
CH3CH—CHCH2CH2CH—CH2 CH —CH3
| | |
CH3 CH3 CH2CH2CH2CH3
Some Basic Principles 21.23

CH3
HO CH3

(a) CH3C(CH3)2CH2CH2CH(CH3)2 (b) CH3C(CH3)2CH2CH(CH3)CH2CH3


(c) CH3C(CH3)2CH(CH3)CH2CH2CH3 (d) CH3C(CH3)2CH2C(CH3)2CH3

(a) — C ∫∫ — C— (d) —C ==O


|| |
C H

(a) CH3CH(CH3)CH2CH2 3)3CCH2


(c) CH3(CH2)3 3CH2 3
CH3 ∫∫CCH3
6 octadien 2 octadien 6
(c) 2 6 diene (d) 6 diene
2H )2 2
N N N
N

O CN
|| |
CH3 — C — CH2—C—CH3
|
CH3

CH3
HO H
H Cl
C2H5

(a) (2R R R S
(c) (2S R S S

3 ∫∫ CCH3
NO2

Cl NO2
21.24 Complete Chemistry—JEE Main

OH
CH2 CH C(CH3)2
CH(CH3)2

3 2
|
CH3

3CH CHCH
CH3
CH2 C CO NH N
CH3 CH3
O

CH2 CH C N(CH2CH3)2

CH2 CH2
NN
CH3
|
3C — N -- C -- CH 2 CH3 NN
| |
CH C 2 H5
3

Isomerism

C 6H

cis trans
B Butanal

(a) n
H

(a) n
H

O O
|| ||
(a) H5C6—C—H (b) H5C6—C—CH3

O O O
|| || ||
(c) H5C6— C —C6H5 (d) H5C6— C —CH2— C —CH3
Some Basic Principles 21.25

3H Br3

3H Br3

CH3 — CH CH — CH3 and CH3— CH2— CH CH2

3 3 3 2 2 2

COOCH3 COOH COOH


H OH H OH H OH
H OH H OH HO H
COOH COOCH3 COOCH3
(I) (II) (III)

(a) CH == CH OH (b) O O

(c) O (d) O

O O
21.26 Complete Chemistry—JEE Main

cis and trans

(a) A meso
(b) A meso
(c) A meso

(d) A meso

(a) CH3 3CH(CH3 3CH(CH3)CH2 3CH(CH3)CH2Cl


Some Basic Principles 21.27

Cl Br H Cl H Cl H H
| | | | | | | |
(a) H—C—C—H (b) H—C—C—Cl (c) H—C—C—H (d) H—C—C—CH3
| | | | | | | |
H H H H H H Br OH

3 2

3 2 3 > CH2
3 > CH2 3 > CH2
2 ∫∫ 2 and —CH2 2

2 2 > C∫∫ 2 2 2 > C∫∫ 2 > CH2 2


2 2 > C∫∫ 2 2 2 2 > CH2 2 > C∫∫
CH3
H OH
C2H5
21.28 Complete Chemistry—JEE Main

S?
CH3 OH CH == CH2 H
H OH H CH3 CH3 OH HO CH == CH2
CH == CH2 CH == CH2 H CH3
(I) (II) (III) (IV)

(a) (I) and (II) (b) (I) and (III) (c) (I) and (IV) (d) (II) and (IV)

CH3 OH CH == CH2 H
H OH H CH3 CH3 OH HO CH == CH2
CH == CH2 CH == CH2 H CH3
(I) (II) (III) (IV)
(a) (II) and (III) (b) (I) and (IV) (c) (II) and (IV) (d) (III) and (IV)

CH3 OH CH == CH2 H
H OH H CH3 CH3 OH HO CH == CH2
CH == CH2 CH == CH2 H CH3
(I) (II) (III) (IV)
(a) (I) and (II) (b) (I) and (III) (c) (II) and (IV) (d) none

meso
meso
meso
meso
meso

(a) (R S
(c) meso racemic
S
(a) (S R
(c) (R R R
Some Basic Principles 21.29

(a) CH3CH2 2 Æ CH3CH2 2


(b) CH3CH2 3) 2 + CH3 Æ CH3CH2 3)CH(CH3)2 + HCl
(c) CH3 2 Æ CH3 2
+
+ H2
(d) CH3CH2CH2CH(Cl)CH3 Æ CH3CH2CH2 3 + Cl
S S 3CHClCH(CH3)C2H

(a) CH3 (b) CH3 (c) CH3 (d) CH3


Cl H H Cl H Cl Cl H

CH3 H CH3 H H CH3 H CH3

C2H5 C2H5 C2H5 C2H5


R R 2H CH(CH3)CH(D)CH2
(a) C2H5 (b) CH3 (c) C2H5 (d) CH3
H3C H H CH3 H CH3 H3C H
H D H D D H D H
CH2D C2H5D CH2D CH2D
S R 2H CH(CH3)CH(D)CH2
(a) C2H5 (b) C2H5 (c) C2H5 (d) C2H5
H3C H H CH3 H3C H H CH3
H D H D D H D H
CH2D CH2D CH2D CH2D
R

meso 6H Cl2

(a) Cis trans


21.30 Complete Chemistry—JEE Main

E F and G
H3C O H3C OH H3C CH3

H3C CH3 H3C CH3 H3C OH


(E) (F) (G)
(a) E F and G E F and E, G
(c) F and G F and G
Inductive and Resonance Effects

in a molecule

in a molecule
I
3 3
I
(a) —C6H (b) —CH3 (c) —CH2CH3 (d) —C(CH3)3

(a) NH2 (b) NH3 (c) OH (d) Cl

+ + + +
(a) CH3 H2 2C 3C

(a) CH 3· ·
2 2CH
· 3C
·
(a) + I I
I I
I
2 2
2 2
I
+ +
(a) — I < — Cl < — Br < — 3 <— 2
+ +
2 2
Some Basic Principles 21.31

I
3 I

2
Acid-Base Strength
K°a

(a) CH3 2 2H 3 2 2H
(c) CH3 2 2H 3 2 2H

(a) CH3 > C 2H (b) CH3 < C 2H


(c) CH3 < C 2H (d) CH3 > C 2H

2
(a) CH3 2 > (CH3)2 3) 3 3 2 < (CH3)2 3) 3
(c) CH3 2 > (CH3)2 3) 3 3 2 < (CH3)2 3) 3

(a) CH3CH2 3CH(Cl)CH2 2CH2CH2


(b) CH3CH2 3CH(Cl)CH2 2CH2CH2
(c) CH3CH2 3CH(Cl)CH2 2CH2CH2
(d) CH3CH2 3CH(Cl)CH2 2CH2CH2

(a) Cl3 2 2 3 2 2
(c) Cl3 2 2 3 2 2

(a) (CH3)3 3) 2 3CH2


(b) (CH3)3 3) 2 3CH2
(c) (CH3)3 3) 2 3CH2
(d) (CH3)3 3) 2 3CH2
21.32 Complete Chemistry—JEE Main

(a) I 2
2 3 3C 3C

3
Ka
3 CH3 + H+ 3 3

(b) +I

Types of Organic Reactions

3 3

3 2 (c) H2 3

+ +
(a (b (c) H+ (d) Ca2+

(a) In S
(b) In S

centre

(a) In S

not correct?
+
3 > CH3CH2+ > (CH3)2CH + > (CH3)3C+

. .
3) 3 C 3)2 CH

3) 3 Æ (CH3)3
3CH2 Æ CH3CH2
Some Basic Principles 21.33

PCl5
HO2 CCH2CHCO2H HO2CCH2CHCO2 H
ether
OH Cl
(–)-malic acid (+)-chlorosuccinic acid

PCl5
HO2CCH2CHCO2H HO2CCH2CHCO2H
ether
Cl OH
(+)-chlorosuccinic acid (+)-malic acid

+ +
C
(d) S

ANSWERS

HINTS AND SOLUTIONS

1 2 3 4
CH 2 == C == CH -- CH3
2 2 3
2
21.34 Complete Chemistry—JEE Main

2 3

 in (CH ) C
C  3
3 3

2 C ∫∫ CH

2 3
3

2 2 p

H3C CH3
H3C C CH2 CH2 CH3
H3C CH3

CH3
H3C C CH2 CH2 CH3
CH3

CH3

H3C C CH CH2 CH3


CH3 CH3

H3C CH CH2 CH2 CH3


CH3

CH3
CH
CH3

CH3
4 3 2 1
H3C C CH CH3
CH3

1 2 3 4
CH2 CH CH CH3
CH3

CH3
CH3 C CH2OH
CH3
Some Basic Principles 21.35

1 2 3 4 5 6 7
CH3 CH == CH CH == CH C ∫∫ CCH3

1
CN
5 4 3 2
CH3COCH2C CH3
CH3
R and R R R
N N

C
C C C C C C; C C C C C; C C C C C; C C C C; C C C C
C C C C C

3 3 cis-trans
3 2CH3 H 3CH2 2CH3 H
2 2—CH3 cis
trans 3 3 and (CH3)2 2
CH2ClCH2CH2CH3 and CH3CHClCH2CH3
CH3 2 —CH 3 and CH3 2—CH3
cis trans

O O OH O
H5 C6 C CH2 C CH3 H5 C6 C CH C CH3

2 2 and cis trans

CH 3CH 2CH 2CH 2 CH2CH 2CH3


* *
CH3CHCH2CH2CH2CH2 ; CH3CH2CHCH2CH2CH3 ; CH3CH CHCH2CH3 ;
CH3 CH3 CH3 CH3
(two optical isomers) (two optical isomers)
CH3 CH3 CH3
CH3 CHCH2CHCH3 ; CH3 CCH2CH2CH3 ; CH3CH2CCH2CH3 ; CH3C CHCH3
CH3 CH3 CH3 CH3 CH3 CH3

CH3CH2 3CHBr2 3CBr3 2CH2 2CHBr2 2CBr3 2CHCHBr2 2CHCBr3


Br3CCBr3

CH3CH2CBr3 3CHBrCHBr2 3C(Br)2CH2 2CH2CHBr2 2CH(Br)CH2Br


3 CHBrCHBr 2
21.36 Complete Chemistry—JEE Main

H H H

BrCH2 C CH2CH3 and CH3 C C CH3


CH3 H3C Br

CH3 CH3 CH3


H OH HO H H OH
plane of symmetry
HO H H OH H OH
CH3 CH3 CH3

optically active optically inactive

meso

*
3 CH

2 > CH3 S
R S and R

S and R

CH3 CH3 CH3 CH3


HO H H OH HO H H OH
H Cl Cl H Cl H H Cl
C2H5 C2H5 C2H5 C2H5
I II III IV
Some Basic Principles 21.37

n
n

CH3 CH3 CH3


H OH OH H H OH
HO H H OH H OH
CH3 CH3 CH3
(I) (II) (III)

CH3
OH H
HO H
CH3
meso

CH3CHClCH2CH3 ClCH2CHClCH2CH3 + CH3C(Cl2)CH2CH3


( ) ( ) achiral
+ CH3CHClCHClCH3 + CH3CHClCH 2CH2Cl
( ) ( )

CH3
CH3 H
H Cl
H Cl Cl CH3 configuration
CH3 H
R R
C2 H 5
CH3
R H
H Cl
CH3 H R CH3 configuration
CH3 H
C2H5 C2 H5
C2 H 5

C2 H 5
C2H5 H
H3C H
H3C H C2 H 5 CH3 configuration
H D
R R
CH2D
C2H 5
R H
H 3C H
H D D R configuration
H D
CH2 D CH2D
CH2 D
21.38 Complete Chemistry—JEE Main

CH3
CH3 H
H Cl
CH3

CH2 Cl CH3 CH3 CH3 CH3


CH3 H CH2Cl H CH3 Cl CH3 H CH3 H
H Cl H Cl H Cl Cl Cl H Cl
CH3 CH3 CH3 CH3 CH2Cl
(I) (II) (III) (IV) (V)

R R S R R
S
meso 6H Cl2
CH3
C2 H 5 CH2Cl
H Cl
H Cl H CH3
CH2
H Cl H CH3
H Cl
C2 H 5 CH2Cl
CH3

E and F E and G

cis trans

I
I

+
-- NH3
Some Basic Principles 21.39

I
I

p d· CH2

. d·
CH— CH 2 ¨Æ
æ H 2 C —CH
.
2C CH2

I
I

K°a

(b) H2 3 2

2
(d) In Cl3 p

– –
Cl C Cl ¨Æ
æ Cl C Cl ¨Æ
æ Cl C Cl
Cl Cl Cl –

3C
3C 3C
3

(b) +I

+
3) 3C > (CH3)2CH+ > CH3CH+2 > CH+3
(b) In S
21.40 Complete Chemistry—JEE Main

(i) Delocalization of electrons

X X X X X

←→ ←→ ←→ ←→

(ii) Difference in hybridization of carbon


3 2

+ +
C O C O

MULTIPLE CHOICE QUESTIONS FROM AIEEE AND JEE MAIN

[2003]

CH3 CH3 H C+ CH3

C 2H CH C 3H CH3 2H H C 2H CH C 2H
(I) (II) (III) (IV)

[2003]
nH2n 2
[2003]
3 3) 2

[2003]
Some Basic Principles 21.41

2H 3 3

[2004]
2

(a) acetone (b) acetic acid (c) acetonitrile (d) acetamide [2004]

[2004]

CH 3
|
(a) CH3—CH2—CH2—CH3 (b) CH3 CH2 CH CH3

H
(c) CH3 C (d) CH3—CH2—C∫∫ [2004]
C2H5

[2005]

(i) CH3C O – 3
O

[2005]

2 ) 3] 6] 3) ]2+ ] [2005]

[2005]]

CHO
(a) HO H (b)
SH
CH2OH
COOH
H2 N NH2
(c) H H (d) H2N H [2007]
Ph Ph
H
21.42 Complete Chemistry—JEE Main

[2007]
HOOC COOH
OH
HO H H
[2008]

(a) CH3Cl (b) (C2H )2 CHCl (c) (CH3)3CCl (d) (CH3)2 CHCl [2008]

2 3 3 2
3 2 3 2 [2008]

CH3
[2009]

[2009]
3) 3C 3C 3)2CH 6H CH2
(a) Cl3C > C6H CH2 > (CH3)2CH > (CH3)3C (b) (CH3)3C > (CH3)2CH > C6H CH2 > Cl3C
(c) C6H CH2 > Cl3C > (CH3)3 C > (CH3)2CH (d) (CH3)2CH > Cl3C > C6H CH2 > (CH3)3C
[2009]

[2013]

(a) (b)
[2014, online]

A B

(a) (b)


(c) 3C , (CH3)3 C∑ [2014, online]

[2015, online]
6H
[2015, online]

[2015]
Some Basic Principles 21.43

[2016, online]
CO2H
H OH
H Cl
CH3
[2016]

ANSWERS

HINTS AND SOLUTIONS

sp3 sp2 sp3 sp3 sp2 sp 3 sp sp3 sp2


CH3 C CH3 ; CH3 C OH ; CH3 C N; CH3 C NH2
O O O
(acetone) (acetic acid) (aceonitrile) (acetamide)

H3C CH2 CH CH3 ; H 3C CH CH CH3 ;


Cl Cl Cl
(2-chlorobutane) (2, 3-dichlorobutane)

H3C CH2 CH CH CH3 H 3C CH COOH


Cl Cl OH
(2, 3-dichloropentane) (2-hydropropanoic acid)

meso

CH3 CH
C2H5

CN– > CH3O– > CH3COO– > H3 C SO–


3
21.44 Complete Chemistry—JEE Main

2 ) 3]

CH3 CH3 CH3


H Cl H Cl Cl H
H Cl Cl H H Cl
CH3 CH3 CH3
(i) (ii) (iii)

CHO

HO H

CH2OH
meso

CH3
7 6 5 4 3 2 1
CH3 CH2 CH2 C CH CH2CH3

CH3 CH2CH3

1 1
COOH COOH

HOOC COOH HO H
2
OH HO H
2
HO H H H OH
3
COOH R′
4

H R′ H

HO R1 H OH HOOC OH
3

COOH 4 COOH R′
R-configuration R-configuration
Some Basic Principles 21.45

cis-trans

CH3

H3C C CH3

CH3

3) 3C
3C

CH3
1 2 3 4 5 6
CH3 CH2 CH CH CH2 CH3
CH2 CH3

3C 3) 3C p

CH3 -- CH -- CHO ; CH3 -- CH -- CH 2 -- CH 2 -- CH3


Ω Ω
Cl Cl
2- chloropropanal 2- chloropentane
Cl
Ω
CH3 -- CH 2 -- CH -- CH3 ; CH3 -- C -- CH 2 -- CH3
Ω Ω
Cl CH3
2- chlorobutane 2- chloro - 2- methylbutane
21.46 Complete Chemistry—JEE Main

C -- C -- C -- C -- C -- C ; C -- C -- C -- C -- C C -- C -- C -- C -- C ; C -- C -- C -- C
Ω Ω Ω Ω
C C C C C
Ω
C -- C -- C -- C
Ω
C
Ê M Br ˆ Ê mAgBr ˆ Ê 80 u ˆ Ê 141 mg ˆ
ÁM ˜Á ˜ (100) ÁË 188 u ˜¯ ÁË 250 mg ˜¯ (100)
Ë AgBr ¯ Ë mcompound ¯
CH3

H3C C CH2 — H2C C


H
CH3

CH3
5 4 3 2 1
H3C C CH2 CH CH2

CH3

3 4
CO2H H
4 1 Hold atom C-2 and 1 3
H OH rotate the other HO CO2H
2 2 S Configuration
H Cl three clockwise H Cl
CH3 CH3
Priorities follow
counter clockwise

CO2H CO2H
Hold atom C-2 and
H 2
OH rotate the other H 2
OH
R Configuration
H Cl three counter clockwise Cl CH3
4 1 1 3
3
CH3 H
4
Priorities follow
clockwise
22
Hydrocarbons

SECTION 1 Alkanes

Methods of Preparation
Decarboxylation of Sodium Salt of Fatty Acids
Sodium salt of a carboxylic acid produces alkane when it is dry distilled with sodalime (NaOH + CaO):
RCO2Na + NaOH æCaO ææ Æ RH + Na2CO3
Wurtz Reaction
An alkyl halide produces alkane when it is made to react with Na in dry ether:
RBr + 2Na + BrR æether ææ Æ R—R + 2NaBr
If R and R¢ are different alkyl groups, the product obtained is a mixture of R—R, R—R¢ and R¢—R¢.
Wurtz reaction constitutes one of the methods to ascent the alkane series.
Corey-House Alkane Synthesis
To prepare alkane of the type R— R¢, this method can be used. This involves the following three steps:
RBr + 2Li æether
ææ Æ RLi + LiBr

2RLi + CuI Æ LiR 2 Cu + LiI


Lithium dialkyl
cuperate

LiR2Cu + R´Br Æ R—R¢ + RCu + LiBr

Kolbe Electrolytic Method


Electrolysis of concentrated solution of sodium salt of a carboxylic acid produces alkane.
O
O– ææÆ R— R + 2CO2 + 2e

Anodic reaction 2R C
Cathodic reaction 2H2O + 2e– ææÆ 2OH– + H2

Hydrogenation of Alkenes and Alkynes


Reduction of alkenes and alkynes with H2 in the presence of Ni catalyst at 200–300 °C produces alkanes:
Ni
CH2 ==CH2 + H2 ææÆ H3C—CH3
200-300∞C
22.2 Complete Chemistry—JEE Main

Ni
CH CH + 2H2 ææÆ CH3— CH3
200-300∞C

Reduction of Alkyl Halide


This may be done by using any of the following reducing agents:
Zn + HCl; Zn + CH3COOH; Zn–Cu couple in ethanol; LiA1H4
The reaction may be formulated as
R— X + 2H Æ RH + HX

Hydrolysis of Grignard Reagent


RMgX + H2O Æ RH + XMgOH

Physical Properties
1. CH4 to C4H10 are gases, C5H12 to C17H36 are liquids and higher ones are solids.
2. Being nonpolar, these are insoluble in water and soluble in nonpolar solvents like benzene and carbon tetrachloride.
3. Boiling point increases with molecular mass; branched chain isomer has a lower boiling point than normal alkane
as the former involves lesser van dar Waals interactions (dispertion forces) than the latter.
4. Liquid alkanes are lighter than water.

Chemical Properties
Because of stronger C— H and C—C bonds, alkanes at room temperature are stable towards acids, bases, oxidizing and
reducing agents and active metals.
A few important reactions of alkanes are as follows.
Halogenation
This involves the substitution of hydrogen atom by halogen atom. The order of reactivity is F2 > Cl2 > Br2(> I2). Fluorine
reacts explosively, chlorination is vigorous, bormination is slow and iodination is reversible.
The mechanism of chlorination and bromination involves free radicals. The reaction is initiated by the absorption of
ultraviolet radiations or heating to about 250–400 °C. The following steps are involved in the mechanism of chlorination.
(a) Chain-Initiation Step
Cl2 æhvæÆ 2 Cl
(b) Chain-Propagation step
CH4 + Cl Æ CH3 + HCl
Cl2 + CH3 Æ CH3Cl + Cl
------------- -------------
------------- -------------
(c) Chain-Terminal Step
Cl + Cl Æ Cl2
CH3 + Cl Æ CH3Cl
CH3 + CH3 Æ CH3—CH3
The chlorination is continued to produce CH2Cl2, CHCl3 and CCl4 depending upon the initial ratio of Cl2 and CH4. Only
monochlorination takes place if methane is taken in a large excess in comparison to chlorine.
In more complex alkanes, the abstraction of each different kind of H atoms gives a different molecule. The factors
determining the relative yields of the isomeric product are as follows.
1. Probability factor This factor is based on the number of H atoms of different types. For example, CH2CH2CH3
contains six equivalent 1° H’s and two equivalent 2° H’s. Hence, the chance of removing 1° H relative to 2°H
is 6 : 2, i.e., 3 : 1.
Hydrocarbons 22.3

2. Reactivity of H The order of reactivity of H is 3° > 2° > 1°. At room temperature, the relative rates of
abstraction of hydrogen atoms by chlorine atom are 5.0 : 3.8 : 1.0. The relative rates of bromination of 1° : 2°
: 3° H¢s are 1 : 82 : 1600.
3. Reactivity of X
reactivity of H atoms. The less reactive Br
For example, the chlorination of CH3CH2CH2CH3 produces two isomers:
Cl 2
CH3CH2CH2CH3 Æ CH3CH2CH2CH2Cl + CH3CH2CHCH3
light, 25 C
n-butane n-butyl chloride Cl
sec-butyl chloride
The relative ratio of the two isomers is as follows.
n-butyl chloride number of 1∞H reactivity of 1∞H 6 1.0 6
= ¥ = ¥ =
sec-butyl chloride number of 2∞H reactivity of 2∞H
H 4 3.8 15.2
6
Percentage of n-butyl chloride = ¥ 100 = 28.3
21.2
Percentage of sec-butyl chloride = 71.7

in chain propagation steps, namely,


Cl + CH4 Æ CH3 + HCl DH° = 17 kJ mol–1
Br + CH4 Æ CH3 + HBr DH° = 75 kJ mol–1
Bromination of higher alkanes is decided by the reactivity H’s atoms and not by probability factor. For example, the
major product (90%) in the bromination of n-butane is 2-bromobutane.
Nitration RH + HNO3 æ400-500
ææææ ∞C
Æ RNO2 + H2O
Higher alkanes produce mainly nitrated products because of breaking of C— C bonds.
D
Sulphonation RH + HO— SO3H ææ
Æ R—SO3H + H2O
fuming
where R is the larger alkyl group (≥C6H13).
3n + 1ˆ
CnH2n+2 + ÊÁ O Æ nCO2 + (n + 1)H2O
Ë 2 ˜¯ 2
Oxidation

Large quantity of heat is generated in the combustion of alkanes.


Isomerization
Normal alkanes are converted into branched-chain isomer in the presence of AlCl3 and HCl.
CH3CH2CH2CH3 æAlCl
ææ 3
Æ CH3 CH CH3
HCl
CH3

Aromatization
CH3
For example, H2C CH3 Cr2O3-Al2O3
Æ + 4H2
600°C
H2C CH2
CH2
A few facts about alkanes are as follows.

Conformations
Ethane can exist in different conformations due to the rotation of methyl groups about C— C bond axis. The Newman
projection formulae for the conformations of ethane are shown in Fig. 22.1.
22.4 Complete Chemistry—JEE Main

H HH
H H

H H
H H H H
H H
Staggered Eclipsed

Fig. 22.1 Newman projections of ethane


Staggered conformation has the lowest potential energy whereas eclipsed conformation has the highest potential energy.
The difference between these two conformers is 11.3 kJ mol–1.
The conformations of butane with reference to the central C—C bond are as follows.
Anti The conformer in which methyl groups are far apart is known as anti conformer.

Gauche The conformer in which methyl groups are closest to each other is known as gauche conformer

Eclipsed The confer in which methyl group is near to either methyl or hydrogen are known as eclipsed conformers.

Isomerism
The compound having different structural formulae but the same molecular formula are known as isomers. For example,
the alkanes butane and pentane can have following carbon skeltons.
Butane C— C—C—C and C C C
C
C
Pentane C— C—C—C—C, C C C C and C C C
C C
Optical Isomerism

atom in a molecule is known as chiral atom. The spatial arrangement and its mirror image are not superimposable on
each other and thus are two different isomeric compounds. They rotate the plane of polarised light by the same angle but
in opposite directions. A mixture of equal amounts of these two isomers is known as racemic mixture. The net optical
rotation of such a mixture is zero.

MULTIPLE CHOICE QUESTIONS ON SECTION 1

Preparation and Nomenclature


1. Ethane is produced during the electrolysis of potassium salt of
(a) succinic acid (b) malonic acid (c) acetic acid (d) fumaric acid
2. Ethane is produced when C2H5MgI is treated with
(a) dilute HCl (b) aq KOH (c) CO2 (d) H2O
3. Methane is produced during the hydrolysis of
(a) CaC2 (b) Al4C3 (c) MgC2 (d) SiC
Zn - Hg
4. The reaction CH3COCH3 + 4H ææææ
Æ C3H8 + H2O is known as
conc. HCl
(a) Rosenmund’s reduction (b) Clemmensen reduction
(c) Wurtz reaction (d) Kolbe’s reaction
Hydrocarbons 22.5

5. A concentrated solution of sodium butyrate is electrolysed, the product obtained at anode is


(a) butane (b) propane (c) hexane (d) pentane
6. A hydrocarbon is obtained by heating
(a) a sodium salt of fatty acid with alcohol (b) a calcium salt of fatty acid
(c) an acid chloride of a fatty acid with sodium (d) a sodium salt of fatty acid with soda lime
Ni
7. The reaction CH2== CH2 + H2 ææÆ C2H6 is an example of
600 K

(a) Sabatier and Senderens reaction (b) Kolbe reaction


(c) Sandmeyer’s reaction (d) Wurtz’s reaction
CH3
8. The IUPAC name of the compound CH3CH CH2CH2 CH CH is
CH3
CH3 CH3
(a) 1, 1, 2, 5-tetramethylhexane (b)2, 5, 6, 6-tetramethylhexane
(c) 2, 5, 6-trimethylheptane (d) 2, 3, 6-trimethylheptane
General Characteristics
9. The compound having one isopropyl group is
(a) 2, 2, 3, 3-tetramethylpentane (b) 2, 2-dimethylpentane
(c) 2, 2, 3-trimethylpentane (d) 2-methylpentane
10. The compound with the highest boiling point is
(a) n-hexane (b) n-pentane (c) 2, 2-dimethylpropane (d) 2-methylbutane
[IIT 1982]
11. Which of the following is expected to have minimum boiling point?
(a) n-Pentane (b) n-Butane (c) 2-Methylbutane (d) 2, 2-Dimethylpropane
12. The highest boiling point is expected for
(a) isooctane (b) n-octane
(c) 2, 2, 3, 3-tetramethylbutane (d) n-butane [IIT 1986]
13. Which of the following statements is not correct?
(a) Alkanes are nonpolar in nature
(b) Alkanes are soluble in nonpolar solvents
(c) Straight-chain alkanes have lesser boiling point than the corresponding branched-chain isomers
(d) Alkanes exhibit alternations in melting point
14. Marsh gas is
(a) methane (b) ethane (c) propane (d) butane
15. Which of the following molecules contains carbon atom of 2°?
(a) CH3CH2CH2CH3 (b) (CH3)2CHCH3 (c) (CH3)3CH (d) (CH3)4C
16. The correct order of stability of the given radicals is
(a) tertiary > secondary > primary (b) tertiary > secondary < primary
(c) tertiary < secondary > primary (d) tertiary < secondary < primary
17. Which of the following facts is correct?
(a) C— D bond is slightly weaker than C—H bond
(b) C—D bond is slightly stronger than C—H bond
(c) Both C—H and C—D bonds are equally strong
(d) Replacement of D in C—D by Cl is faster than the replacement of H in C—H
18. The most strained cycloalkane is
(a) cyclopropane (b) cyclobutane (c) cyclopentane (d) cyclohexane
22.6 Complete Chemistry—JEE Main

19. Which of the following statements is correct?


(a) The bond angle HCH in methyl radical is 120°.
(b) The eclipsed conformation of ethane has lower potential energy as compared to staggered conformation.
(c) The IUPAC name of CH3CH2CH2CH ⎯ CH ⎯ CHCH2CH3 is 3-ethyl-4-methyl-5-isopropyloctane.

CH CH3 CH2CH3

CH3 CH3
(d) Lower alkanes are soluble in water while higher ones are insoluble.
20. Which of the following statements is not correct?
(a) Methane is the major constituent of natural gas.
(b) In the reaction RMgX + NH3 Æ RH + Mg(NH2)X, ammonia acts as a base.
(c) The reactivity of hydrogen atom in alkane towards replacement by a halogen follows the order 3° > 2° > 1°.
(d) The enthalpy of formation of a free radical from the corresponding alkane follows the order 1° > 2° > 3°.
21. Which of the following statements is not correct?
(a) The octane value of n-heptane is zero.
(b) The octane value of isooctane is 100.
(c) The octane number of a mixture containing 85% isooctane and 15% heptane is 15.
(d) The 3° radicals react more slowly and smoothly with oxygen than the 1° and 2° radicals.
22. Which of the following statements is not correct?
(a) Of the vinyl, allyl, 3°, 2°, 1° and benzyl radicals, the least stable one is vinyl.
(b) Of the C6H5CH3, (CH3)3CH, (CH3)2CHCH3 and CH2==CHCH3, the maximum ease of abstraction of hydrogen
atom is shown by C6H5CH3.
(c) Free radical monochlorination of tert-butyl bromide gives 1-bromo-2-chloro-2 methyl propane.
(d) Of Cl2, Br2, and I2, the most reactive halogen towards photohalogenation of alkanes is I2.
Chemical Reactions
23. Ethanol on treatment with concentrated HI and red phosphorus forms
(a) C2H5I (b) C2H4 (c) C2H6 (d) C3H8
24. Alkanes can be iodinated in the presence of
(a) HI (b) I2 and P (c) HIO3 (d) PI3
25. Isomerism in alkane can be brought about by using
(a) Al2O3 at 100 °C (b) Fe2O3 (c) Cr2O3/Al2O3 at 600 °C (d) conc. H2SO4

(a) CH3Cl (b) CH2Cl2 (c) CHCl3 (d) CCl4


27. Hexane when heated under pressure in the presence of Cr2O3 carried on Al2O3 support at 750–770 K produces
(a) cyclohexane (b) hexene (c) cyclohexene (d) benzene
–1
28. A single substitution of H atom in an alkane of molar mass 72 g mol by another substituent produces only one
product. The alkane is
(a) n-pentane (b) 2-methylbutane (c) 3-methylbutane (d) 2, 2-dimethylpropane
29. The reaction conditions leading to the best yield of C2H5Cl are
uv light dark
(a) C2H6 (excess) + Cl2 ææææ
Æ (b) C2H6 + Cl2 æææææ
Æ
room temperature
uv light
(c) C2H6 + Cl2 (excess) Æ (d) C2H6 + Cl2 ææææ
Æ
30. The relative reactivity of 1° : 2° : 3° hydrogens to chlorination is
(a) 1 : 5 : 3.8 (b) 1 : 3.8 : 5 (c) 3.8 : 1 : 5 (d) 5 : 1 : 3.8
Hydrocarbons 22.7

31. The relative reactivity of 1° : 2° : 3° hydrogens to bromination is


(a) 1 : 3.8 : 5 (b) 1 : 1600 : 82 (c) 1 : 82 : 1600 (d) 1600 : 82 : 1
32. Which of the following statements is not correct?
(a) The order of reactivity of halogen towards alkanes is F2 > Cl2 > Br2.
(b) The reaction of Cl2 and CH4 proceeds via ionic mechanism.
(c) Oxygen acts as an inhibitor in the reaction between CH4 and Cl2.
(d) Iodine does not react with CH4 at all.
33. Which of the following statements is correct?
(a) The percentage of n-propyl chloride obtained in the chlorination of propane is about 56%.
(b) The percentage of 1-chloro-2-methylpropane obtained in the chlorination of isobutane is about 64%.
(c) The percentage of n-propyl bromide obtained in the bromination of propane is 44%. The relative reactivities
of 3°, 2° and 1° H atoms are 1600:82:1.
(d) The vinyl radical is more stable than 2° radical.
34. Which of the following statements is correct?
Cl

(a) The chlorination of CH3—CH—CH3 yields CH3 C CH3 .


Br
Br
(b) The number of isomers of dimethylcyclopropane is three.
(c) The number of geometric isomers of 1,2,4-trimethylcyclohexane is three.
(d) Substituted cycloalkanes do not show geometrical isomers.
35. Which of the following statements is correct?
(a) The electrolysis of concentrated solution of sodium ethonate produces ethene.
(b) If the relative rates of abstraction of 2° and 1° hydrogen atoms are 3.8 and 1.0, respectively, then the
percentage ratios of the products 1-chloro-, 2–chloro–, and 3-chloro–, obtained in the photochlorination of n-hexane
would be 16.48 : 41.76 : 41.76.
(c) The relative rate of abstraction of 3°, 2° and 1° hydrogen atoms is maximum for 1° hydrogen atoms.
(d) The relative rate of abstraction of 3°, 2°and 1° hydrogen atoms is minimum for 3° hydrogen atoms.
Isomers and Conformations
36. The minimum number of carbon atoms in alkane molecule to show isomerism is
(a) 2 (b) 3 (c) 4 (d) 5
37. The maximum number of structural isomers for the alkane C4H10 is
(a) 2 (b) 3 (c) 4 (d) 5
38. The maximum number of structural isomers for the alkane C5H10 is
(a) 2 (b) 3 (c) 4 (d) 5
39. Which of the following is not found in alkanes?
(a) Chain isomerism (b) Geometrical isomerism
(c) Mesomerism (d) Positional isomerism
40. A molecule is said to be chiral if it
(a) contains a centre of symmetry (b) contains a plane of symmetry
(c) cannot be superimposed on its mirror image (d) exists as cis and trans-forms
41. The maximum number of structural isomers for an alkane hexane is
(a) 2 (b) 3 (c) 4 (d) 5
42. Which of the following statements is not true?
(a) In ethane, staggered conformation is more stable than eclipsed conformation
(b) Cyclohexane exists in two conformations
22.8 Complete Chemistry—JEE Main

(c) The boat conformation of cyclohexane is more stable than chair conformation
(d) The boat and chair conformations of cyclohexane do not exist as independent compounds
43. An equimolar mixture of enantiomers is called
(a) optical isomers (b) resolution (c) racemic mixture (d) dextroisomer
44. Optical rotation of a racemic mixture is
(a) positive (b) negative (c) zero (d) temperature dependent
45. The energy barrier to rotation about the C— C bond in ethane is about 12.55 kJ/mol at room temperature (298 K).
Assuming the contribution of entropy negligible, the ratio of staggered to eclipsed conforations is about
(a) 150 (b) 159 (c) 165 (d) 175
46. The ratio of anti/gauche conforations of butane is 4.6. The anti conformation is more stable than the gauche
conformation at room temperature (25 °C) by
(a) 2.71 kJ/mol (b) 3.12 kJ/mol (c) 3.78 kJ/mol (d) 4.14 kJ/mol
47. Which of the following is the stablest form of cyclohexane?
(a) Boat form (b) Chair form (c) Half-chair form (d) Half-boat form
48. Which of the following conformations of butane has the lowest energy?
(a) Anti (b) Gauche (c) Eclipsed (d) Staggered
49. Which of the following statements is not correct?
(a) Cycloheptane and cyclooctane are more stable than cyclohexane.
(b) The ring strain in cyclopropane is more than that in the cyclobutane.
(c) The C—H bond in cyclopropane is shorter than in propane.
(d) The H atoms in cyclopropane is more acidic than in propane.
50. Which of the following statements is not correct?
(a) The potential energy difference between eclipsed and staggered conformations of ethane is 12.552 kJ mol–1.
Ignoring the entropy effect, the ratio of staggered to eclipsed conformation at 25 °C is about 155.
(b) The potential energy difference between anti and gauche con formations of butane is about
3.8 kJ mol–1. Ignoring the entropy effect, the ratio of anti to gauche conformations at 25 °C is about 2.6.
(c) The activation energy for the reaction 2 Cl◊ Æ Cl2 is zero.
(d) The free radical chlorination of CH4 occurs faster than that of CD4.

ANSWERS
1. (c) 2. (d) 3. (b) 4. (b) 5. (c) 6. (d)
7. (a) 8. (d) 9. (d) 10. (a) 11. (d) 12. (b)
13. (c) 14. (a) 15. (a) 16. (a) 17. (b) 18. (a)
19. (a) 20. (b) 21. (c) 22. (d) 23. (c) 24. (c)
25. (c) 26. (d) 27. (d) 28. (d) 29. (a) 30. (b)
31. (c) 32. (b) 33. (b) 34. (b) 35. (b) 36. (c)
37. (a) 38. (b) 39. (c) 40. (c) 41. (d) 42. (c)
43. (c) 44. (c) 45. (b) 46. (c) 47. (b) 48. (a)
49. (a) 50. (b)

HINTS AND SOLUTIONS


1. Kolbe’s electrolytic method 2CH3COOK + 2H2O Æ CH3CH3 + 2CO2 + 2KOH + H2
2. C2H5MgI + H2O Æ C2H6 + HOMgI
Hydrocarbons 22.9

3. Al4C3 + 12H2O Æ 3CH4 + 4Al(OH)3


5. 2CH3CH2CH2COONa + 2H2O Æ C6H14 + 2CO2 + 2NaOH + H2
hexane
9. Isopropyl group is CH3 CH
CH3
10. The boiling point gradually increases with increase in the molecular mass. Straight-chain alkanes have higher
boiling point than the corresponding branched-chain isomer.
17. The bond with the heavier isotope is slightly stronger than that with the lighter isotope.
19. (a) The bond angle is 120° because carbon is sp2 hybridized.
(b) In eclipsed conformation, hydrogen atoms of the two carbon atoms are closer to each other, hence more
repulsive interactions resulting in the higher potential energy.
(c) The correct name is 3-ethyl-5-isopropyl-4-methyloctane. Isopropyl comes before methyl.
(d) Alkanes are souble in non-polar solvents such as benzene, ether and chloroform.
20. (a) Natural gas contains about 97% of methane.
(b) Here NH3 acts a stronger acid and RH as a weak acid.
(d) The more stable the free radical, the lesser the enthalpy of formation of the free radical from the corresponding
alkane.
21. (c) The octane value is 85.
HI P
23. C2H5OH æææ
Æ C 2H 6 + H 2O

Cr O Al O
27. CH3CH2CH2CH2CH2CH3 æææææ
2 3 2 3
Æ
750-770 K
Benzene

32. (b) The reaction proceeds via free radical mechanism.


(c) This is due to the formation CH3—O—O· radical which is less reactive than CH3· radical.
(d) It is due to the fact, that the reaction I· + CH4 Æ HI + CH3· is highly endothermic (DH° = 138 kJ mol–1).
33. (a) The chlorination of propane gives two isomeric products: 1-chloropropane and 2-chloropropane. Their relative
ratio would be given as
1-Chloropropane no. of 1∞ H reactivity of 1∞ H 1.0 6.0
= ¥ = 6 ¥ =
2-Chloropropane no. of 2∞ H reactivity of 2∞ H 2 3.8 7.6
6.0
Percentage of 1° isomer = ¥ 100 = 44%
6.0 + 7.6

(b) Isobutane is CH3 CH CH3 . Its chlorination would give two isomers.

CH3
1∞ isomer no. of 1∞ H reactivity of 1∞H 9 1 9
= ¥ = ¥ =
3∞ isomer no. of 3∞ H reactivity of 3∞H 1 50 5
9
Percentage of 1° isomer = ¥ 100 = 64.3%
9+5

(c) The propane is CH3CH2CH3. The relative ratio of the two isomers is
1∞ isomer no. of 1∞ H reactivity of 1∞ H 6 1 6
= ¥ = ¥ =
2∞ isomer no. of 2∞ H reactivity of 2∞ H 2 82 164
3
Percentage of 1° isomer = ¥ 100 4
3 + 164
22.10 Complete Chemistry—JEE Main

(d) The homolytic bond dissociation energy of vinylic H is greater than that of 2° H. Hence, vinylic radical is less
stable than 2° radical.
∑ ∑
Cl Cl
34. (a) CH3 CH CH3 CH3 CH CH2 ærearrangement
æææææ Æ CH3 C HCH 2 Br æææÆ CH3CHCH 2 Br
|
Br Br Cl
1° radical 1-bromo-2-chloropropane
Although alkyl free radicals seldom if ever rearrange by migration of hydrogen or alkyl, but they can rearrange
by migration of halogen.
(b) The number of isomers is three
Me
Me Me
H
Me H
Me H
Me
H
1,1-dimethylcyclopropane cis-1,2-dimethylcyclopropane trans-1,2-dimethylcyclopropane

Cis-and trans-isometers are due to the rigidity of the ring. The rotation about the ring C—C bonds is prohibitive.
(c) There are four geometric isomers.

Me Me Me Me
Me

Me
cis, cis-1,2,4-Trimethylcyclohexane trans-2-cis-4-Trimethylcyclohexane
Me
Me
Me Me
Me
Me
cis-2-trans-4-Trimethylcyclohexane trans-2-trans-4-Trimethylcyclohexane

35. (a) ethane is the product of electrolysis


(b) 6 ¥ 1 : 4 ¥ 3.8 : 4 ¥ 3.8 :: 16.48% : 41.76% : 41.76%
(c) 3°
(d) 1°
37. C— C—C—C; C C C
C
C
38. C—C—C—C—C; C C C C; C C C
C C
C
41. C—C—C—C—C—C ; C C C C C; C C C C C; C C C C; C C C C
C C C C C
42. The boat conformation of cyclohexane is less than stable chair conformation.
45. Keq = exp(–DG/RT) = exp(12550/8.314 ¥ 298) ~
– 159
Hydrocarbons 22.11

46. Given is the fact that Keq = 4.6 for gauche anti. Hence
~
DH – DG = –RT ln Keq = [– (8.314) (298) ln 4.6] J/mol
= –3781 J/mol
49. (a) Both are slightly less stable than cyclohexane.
(c) The observed bond angle H—C—H in cyclopropane is 114°. To account for this angle, it is postulated that the
hybrid orbital of C involves more s-character as compared to the sp3 hybrid orbital.
(d) The C—H bonds in cyclopropane involves more s-character than those of an alkane.
50. (a) Here DG° DH°. Using DG° = – RT ln K°, we get
DG∞ (12.552 ¥ 103 J mol-1 )
ln K° = - = = 5.066 fi K° = 158.57
RT (8.314 J K -1mol-1 ) (298 K)

(b) Here DG° DH°. Using DG° = – RT ln K°, we get


D G∞ 3.8 ¥ 103 J mol-1
ln K°= - = = 1.534 fi K° = 4.64
RT (8.314 J K -1mol-1 ) (298 K)
(c) In this case, no bonds are broken, they are only formed.
(d) The C—D bonds in CD4 are slightly stronger than C—H bonds in CH4. Thus, DH for abstraction of D is slightly
greater than for H. The abstraction being the slow step, the removal of H will be faster.

MULTIPLE CHOICE QUESTIONS FROM AIEEE AND JEE MAIN

1. On mixing a certain alkane with chlorine and irradiating it with ultraviolet light, it forms only one monochloroalkane.
The alkane could be
(a) neopentane (b) propane (c) pentane (d) isopentance [2003]
2. Which of the following compounds is not chiral?
(a) 1-chloropentane (b) 2-chloropentane
(c) 1-chloro-2-methylpentane (d) 3-chloro-2-methylpentane [2004]
3. 2-Methylbutane on reacting with bromine in the presence of sunlight gives mainly
(a) 2-bromo-3-methylbutane (b) 1-bromo-3-methylbutane
(c) 1-bromo-2-methylbutane (d) 2-bromo-2-methylbutane [2005]
4. Alkyl halides react with dialkyl copper reagents to give
(a) alkanes (b) alkenyl halides (c) alkenes (d) alkyl copper halides
[2005]

is
(a) Anti, Gauche, Eclipse (b) Eclipse, Gauche, Anti (c) Gauche, Eclipse, Anti (d) Eclipse, Anti, Gauche
[2006]
6. Which one of the following conformations of cyclohexane is chiral?
(a) Twist boat (b) Rigid (c) Chair (d) Boat [2007]
7. How many chiral compounds are possible on monochlorination of 2-methylbutane?
(a) 8 (b) 2 (c) 4 (d) 6 [2012]
8. The major product obtained in the photo catalysed bromination of 2-methylbutane is:
(a) 1-bromo-2-methylbutane (b) 1-bromo-3-methylbutane
(c) 2-bromo-3-methylbutane (d) 2-bromo-2-methylbutane [2014, online]
22.12 Complete Chemistry—JEE Main

ANSWERS
1. (a) 2. (a) 3. (d) 4. (a) 5. (a) 6. (a)
7. (b) 8. (d)

HINTS AND SOLUTIONS


2. The given compounds are
CH3CH2CH2CH2CH2Cl ; * * *
CH3CH2CH2CHCH3 ; CH3CH2CH2CHCH2Cl ; CH3CH2CHCHCH3
1-chloropentane
Cl CH3 Cl CH3
2-chloropentane 1-chloro-2-methylpentane 3-chloro-2-methylpentane

The compound 1-chloropentane is not chiral as other compounds contain chiral carbon atoms.
3. The reaction is
Br
H3 C CH CH2 CH3 + Br2 Æ H3C C CH2 CH3 + HBr
CH3 CH3
2-bromo-2-methylbutane

The reaction proceeds via free-radical mechanism. 3º radical being most stable is formed by the extraction of H
from 2 position.
4. The reaction is

5. Because of hydrogen bonding, the order of stability is reversed when compared to ethane. Hence, the increasing
order of stability is anti, gauche, eclipse.
7. The possible monochlorination of 2-methylbutane are as follows.
Cl
* CH3 C CH2CH3 ; *
ClCH2CHCH2CH3 ; CH3CH CH CH3 CH3 CHCH2CH2Cl
CH3 CH3 CH3 Cl CH3
(i) (ii) (iii) (iv)
Of the four compounds, the compounds (i) and (iii) are chiral compounds as each of the two has one asymmetric
carbon atom. Each of these two will exhibit two enantiomers.
8. 2- Methylbutane is .

Photocatalysed bromination proceeds through the formation of radicals. Since 3° radical is most stable, the product
obtained is 2-bromo-2-methyl butane.
Hydrocarbons 22.13

SECTION 2 Alkenes

Methods of Preparation
Dehydrohalogenation of Alkyl Halides
Heating of alkyl halides with alcoholic KOH produces alkanes:
ethanol
RCH2CH2X + KOH æææ
D
Æ R— CH==CH2 + KX + H2O
The ease of dehydrohalogenation is 3° alkyl halide > 2° alkyl halide > 1° alkyl halide.
The dehydrobromination of optical isomers of 2, 3-dibromobutane depends whether the isomer is racemic or meso.

For the detailed analysis of these reactions, see the answers of Q.18c and Q.18d, respectively.
Dehydration of Alcohols
H SO
Sulphuric acid is a dehydrating agent. R— CH2—CH2OH æææ
2
D
4
Æ R— CH==CH2 + H2O
The ease of dehydration is 3° alcohol > 2° alcohol > 1° alcohol.
Dehalogenation of Vicinal Dihalides
This is carried out by heating with zinc dust in ethyl alcohol.
alcohol
RCH CH2 + Zn æææ
D
Æ R— CH==CH2 + ZnBr2
Br Br
The debromination of optical isomers of 2, 3-dibromobutane depends whether the isomer is meso- or racemic.
CH3 CH3
H3C H H Br H3C CH3
H Br C C C C
H Br H CH3 Br H H H
CH CH 3
3
meso-2, 3-dibromobutane trans-2-butene racemic-2, 3-dibromobutane cis-2-butene
For the detailed analysis of these reactions, see the answers of Q.18a and Q.18b, respectively.
Partial Reduction of Alkynes
R H H2
R R
Na or Li
C C R C C R Lindar
C C
NH3
H R catalyst H H
trans-alkene cis-alkene

Kolbe’s Electrolytic Method


The electrolysis of an aqueous solution of potassium salt of a saturated dicarboxylic acid produces alkene.
Anodic Reaction CH 2 COO - ææ
Æ CH 2 + 2CO 2 + 2 -
Cathodic Reaction 2H2O + 2e– Æ 2OH– + H2
| -
||
CH 2 COO CH 2
22.14 Complete Chemistry—JEE Main

Saytzeff Rule
In dehydrohalogenation or dehydration, the preferred product is the alkene that has the greater number of alkyl groups
attached to the doubly bonded carbon atoms.
The alkene with the greater number of alkyl groups is the preferred product because it is formed faster than alternative
alkenes. The sequence showing the relative rates of formation of alkenes is
R2C==CR2 > R2C==CHR > R2C==CH2, RCH==CHR > RCH==CH2
The sequence of stability of alkenes also follows the above order. Hence, the more stable the alkene, the faster it is
formed.
For example,
CH3CH 2 CHCH3 æKOH(alc)
æææÆ
|
Br
A carbocation formed as an intermediate is particulary prone to rearrangement. For example, the following reaction
involves rearrangement.
CH3
CH3 CH3
EtOH
CH3 C CH CH3 Æ CH3 C C CH3 + CH3 CH C CH2

CH3 Br CH3 CH3


(main product)
This arrangement is due to the formation of more stable 3° carbocation from the lesser stable 2° carbocation:
CH3 CH3 CH3 CH3
–Br– Saytzeff rule
CH3 C CH CH3 Æ CH3 C CH CH3 Æ CH3 C CH CH3 Æ CH3 C C CH3
+ +
CH3 Br CH3 CH3 CH3
(2°) (3°)

The ease of dehydration of alcohols (3° > 2° > 1°) is in agreement with the formation of carbocation as the intermediate.
Wherever possible, rearrangement takes place. For example
H+
CH3 CH2 CH CH2OH Æ CH3CH CCH3 + CH3CH2C CH2
CH3 CH3 CH3
(main product)
CH3 CH3 CH3
|
H+
CH3CCHOHCH3 Æ CH3C CCH3 + CH2 CCHCH3
| |
CH3 CH3 CH3
(main product)
Hofmann Rule
Heating of a quaternary ammonium hydroxide mainly produces the least substituted alkene. For example
CH3CH2CHCH3 Æ CH3CH2CH CH2 + CH3CH CHCH3
150 C
+ (95%) (5%)
N(CH3)3
+ (CH3)3N: + H2O
A less substituted alkene is formed as a result of the loss of more acidic bH (which follows the trend 1° > 2° > 3°).

Chemical Properties
Electrophilic Addition Reactions
Because of loosely held p electrons, the carbon-carbon double bond serves as a source of electrons and thus addition
reactions across the double bond are initiated by an electrophilic attack. The addition reactions exhibited by alkenes
are as follows.
Hydrocarbons 22.15

1. Addition of hydrogen A solution of the alkene is shaken under a slight pressure of hydrogen gas in the presence
of a small amount of the catalyst (Pt, Pd, or Ni)
H , Ni
RCH==CH2 æææ
2
Æ RCH2CH3
The reaction is quantitative, the volume of hydrogen gas consumed provides the number of double bond in the compound.
Heats of hydrogenation provide the information regarding the relative stabilities of unsaturated compounds. Of simple
dialkylethylenes, the trans isomer is the more stable. It is also found that the greater the number of alkyl groups attached
to the doubly bonded carbon atoms, lesser the heat of hydrogenation and thus more stable the alkene—a conclusion
which is in agreement with the Saytzeff rule.
The catalytic addition of hydrogen atoms across a double bond involves syn- addition, i.e. on the same side of the
double bond. The hydrogenetaion of cis- and trans- isomers of 2, 3-dibromobutene yields different optical isomers of
2, 3-dibromobutane.
CH3 CH3
H3C CH3 Br CH3 H Br
H2/Ni H Br H2/Ni
C C C C
H Br H3C Br Br H
Br Br
CH3 CH3
cis-2, 3-dibromobutene
meso-2, 3-dibromobutane racemic-2, 3-dibromobutane
For the detailed analysis of these reactions, see the answers of Q.43a and Q.43b, respectively.
2. Addition of hydrogen halide When dry gaseous hydrogen halide is directly passed into an alkene, the corresponding
alkyl halide is formed. The addition of HX follows Markovnikov’s rule which states that the hydrogen of acid attaches
itself to the carbon that already holds the greater number of hydrogens. For example,
R— CH==CH2 + HCl Æ R CH CH3
Cl
In the presence of peroxide, the addition of HBr does not follow Markovnikov’s rule. The addition is just a reverse
of Markovnikov’s rule and this fact is known as anti-Markovnikov’s rule. For example,
peroxide
R— CH==CH2 + HBr ææææ
Æ R—CH2—CH2Br

Mechanism of Addition of HBr in the Presence of Peroxide


Anti-Markovnikov addition of HBr in the presence of peroxide occurs via free-radical mechanism.
Chain-Initiating Steps
Peroxide Æ Radical
Radical + H : Br Æ Radical : H + Br
Chain-Propagating Steps

......................... ..........................
......................... .........................
Factors that stabilize the free radical also stabilize the incipient free radical in the transition state. For example,
secondary free radical, being more stable than primary radical, is formed faster in the reaction between R—CH==CH2
and HBr and thus the product obtained is RCH2CH2Br. Only for HBr, both the steps mentioned above are exothermic

the reverse is true.


22.16 Complete Chemistry—JEE Main

3. Addition of sulphuric acid When a gaseous alkene is passed through concentrated sulphuric acid in cold or a liquid
alkene is stirred with the acid, the addition of a molecule of H2SO4 takes place:
R— CH==CH2 + H2SO4 Æ R— CH --CH3
|
OSO3 H
The addition of sulphuric acid follows Markovnikov’s rule. The resultant hydrogen sulphate when heated with water
produces the corresponding alcohol:
R— CH -- CH3 + H 2 O Æ R— CH -- CH3 + H 2SO 4
| |
OSO3 H OH

4. Addition of water In the presence of acids, water adds to the more reactive alkenes. The addition is in accordance
with the Markovnikov’s rule.
H+
R—CH==CH2 + H2O ææÆ R— CH -- CH3
|
OH
Oxymercuration-Demercuration Process Alkenes in tetrahtydrofuran react with mercury acetate in the presence of
water to give hydroxymercurial compounds which on reduction with sodium borohybride (NaBH4) yield alcohol. The
net result is the addition of H2O to the double bond and the product obtained is in agreement with Markovnikov’s rule.
For example,
+ HgOAc HgOAc
Hg(OAc)2 – OAc – NaBH4
CH3CH CH2 ¾¾¾® CH CH —– CH ¾¾®
H2O
CH3CH—CH2 ¾¾® CH3CH—CH3
3 2
OH OH
The electrophile of Hg(OAc)2 is +HgOAc. No rearrangement occurs in this reaction.
Hydroboration–Oxidation Process Alkenes undergo hydroboration with diborane in THF to yield alkylboranes which
on oxidation with alkaline H2O2 give alcohols. The net result is the addition of H2O to the double bond and the product
obtained is in accordance with the anti-Markovnikov’s rule. For example,

H2O2, OH –
== OH

B2H6 in THF exists as BH3,


5. Addition of halogens Chlorine or bromine readily adds to double bond. Iodine does not react.
R—CH==CH2 + X2 æ(æææææ
X 2 ∫∫ Cl2 , Br2 )
Æ R -- CH -- CH 2
| |
X X

Mechanism of Addition of Halogens


Two steps involved are as follows.
Step 1 Formation of halonium ion
X+

X X+ C C Æ X + C C

halonium ion
Step 2 Addition of halide ion (or any other species similar to it) to halonium ions. This addition involves anti-addition.
X+ X
C C + Y– Æ C C
Y
Hydrocarbons 22.17

For example, the addition of Br2 to ethylene in the presence of NaCl in aqueous medium, Y– may be Br–, Cl– or OH– and
thus all the three compounds given below are formed.
BrCH 2CH2Br, BrCH2CH2Cl, BrCH2CH2OH
The addition of Cl2 or Br2 to double bond involves anti-addition, i.e. halogen atoms are attached at the opposite faces
of the double bond.
If the compound containing double bond exists in two stereoisomers, namely, cis and trans, then the addition of halogen
may generate chiral carbon atoms and thus the product will exhibit optical isomers. The addition of halogen across
cis alkene yields only racemic mixture whereas trans alkene
produces meso compound.
For example,

For the detailed analysis of these reactions, see the answers of Q.42c and Q.42d, respectively.
6. Addition of hypohalous acids (Chlorine or bromine in the presence of water)
(X 2 , H 2O)
R— CH==CH2 æ(X
æææææ
2 ∫∫ Cl 2 , Br2 )
Æ R -- CH -- CH 2
| |
OH X
The reaction in which two hydroxyl groups are added to the double bond is known as
hydroxylation. This can be achieved by treating the compound containing double bond with cold alkaline potassium
permanganate (Baeyer’s reagent). Mild conditions of oxidation are the essential requirement as otherwise the resultant
compound may be oxidized further. The reaction is
3CH2==CH2 + 2KMnO4 + 4H2O Æ 3CH2 CH2 + 2MnO2+ 2KOH
OH OH
Ozone can be used to break the double bond to give two smaller molecules. The reaction
takes place in two steps.
Step 1 Formation of ozonide
O
RCH==CH2 + O3 Æ R CH CH2

O O
Step 2 Hydrolysis of ozonide in the presence of a reducing agent (frequently (CH3)2S or zinc dust and acetic acid)
O
H O
R CH CH2 æææ
2
Æ RCHO + H2CO
Zn

O O
The cleavage products are aldehydes or ketones depending upon the atom or group attached to carbons of the double bond.
The function of reducing agent is to prevent the formation of H2O2 which would otherwise react with aldehydes and
ketones.
9. Polymerization of alkenes Ethylene produces polyethylene when it is heated under pressure with oxygen.
O , heat
nCH2==CH2 ææææ
2
Æ (— CH2—CH2—)n
pressure

10. Degradation by sodium periodiate (NaIO4) in the presence of potassium permanganate (KMnO4).

| | È | | ˘
Æ Í-- C -- C -- ˙ æææ
KMnO 4
-- C == C -- ææææ Æ acids, ketones, CO 2
Í | | ˙
ÍÎ OH OH ˙˚
22.18 Complete Chemistry—JEE Main

Carboxylic acid is obtained instead of aldehydes. The terminal == CH2 group is oxidized to CO2.
CH3 CH3
KMnO 4
CH3CH C CH3 Æ CH3COOH + O C CH3
NaIO 4
KMnO
CH3CH2CH==CH2 ææææ
4
Æ CH3CH2COOH + CO2
NaIO 4

The rate of addition of an acid to a double bond depends upon the stability of the carbocation being formed. For a given
acid, the order of reactivity of alkenes is as follows.
CH3
C == CH2 > CH3CH==CHCH3, CH3CH2CH==CH2, CH3CH==CH2 > CH2==CH2 > CH2==CHCl
CH3
For the detailed analysis of these reactions, see answers of Q. 42c and Q. 42d, respectively.
Substitution Reactions
An alkyl group attached to a double bond can undergo substitution reaction involving halogen—a typical reaction
shown by alkanes. The working conditions are the high temperature (500–600°C) in the gaseous phase. These may
be compared with those of addition reaction which are low temperature in the absence of light and the medium of the
reaction is generally the liquid phase. For example,

Substitution reaction at alkyl group occurs via free-radical mechanism. It is also found that the low concentration
of halogen can be used instead of high temperature to cause substitution at the allylic position. The compound
N-bromosuccinimide has been used to brominate alkenes at the allylic position. As each molecule of HBr is formed by
the bromination, N-bromosuccinimide generates Br2 according to the reaction
O O
C C
H2C H2C
HBr + N Br Æ Br2 + N H
H2C H2 C
C C
O O
N-bromosuccinimide succinimide
Since substitution of H by X occurs only at allylic position and not at vinylic position, the following conclusions may
be drawn.

C C C
H H H
allylic
vinylic
Ease of abstraction of hydrogen atoms allylic > 3° > 2° > 1° > CH4 > vinylic
Ease of formation of free radical or Stability of free radical allyl > 3° > 2° > 1° > CH3 > vinyl
Since the allyl radical involves resonance (CH2==CH—CH2 ´ ∑CH2—CH==CH2), the allylic substitution may give
two different products. For example, 1-octene on treating with N-bromosuccinimide gives the following two products:

The resonance in allylic radical makes it more stable than other alkene.
Hydrocarbons 22.19

MULTIPLE CHOICE QUESTIONS ON SECTION 2

General Characteristics

(a) C2H2 (b) C2H4 (c) C2H6 (d) C3H6


2. Geometrical isomerism is shown by
(a) alkanes (b) alkenes (c) alkynes (d) aromatic hydrocarbons
3. The geometric isomers
(a) rotate the plane of polarised light (b) exhibit optical properties
(c) contain atleast one carbon-carbon double bond (d) contain atleast one carbon-carbon triple bond
4. Which of the following compounds exhibits cis-trans isomerism?
(a) 2-Butane (b) 2-Butyne (c) 2-Butanol (d) Butanal
5. The IUPAC name of the compound
CH3
CH3 C CH CH2
CH3
is
(a) 3, 3, 3-Trimethylpropane (b) 1, 1, 1-Trimethylprop-2-ene
(c) 3, 3-Dimethylbut-1-ene (d) 2, 2-Dimethylbut-3-ene
6. Propane and propene cannot be distinguished by using
(a) Cl2 (b) Br2 (c) KMnO4 (d) O3
7. The compound CH2==C==CH2
(a) has linear geometry (b) has p bonds in a plane
(c) has p bonds perpendicular to each other (d) has p electrons delocalized over the whole molecule
8. Which of the following order regarding relative stability of the given alkenes is correct?
(a) 1-Pentene < trans-2-pentene < 2-methyl-2-butene (b) 1-Pentene < trans-2-pentene > 2-methyl-2-butene
(c) 1-Pentene > trans-2-pentene < 2-methyl-2-butene (d) 1-Pentene > trans-2-pentene > 2-methyl-2-butene
9. Which of the following statements is not correct?
(a) cis-Alkene can be converted into trans-alkene on heating with I2.
(b) The compound shows a faster rate of catalytic hydrogenation than the compound .

(c) The larger the enthalpy of formation of an alkene, the more stable the alkene.
(d) The compound C2H5CH(Cl)CH2 CH2 on hydrogenation produces an optically inactive compound.
10. Which of the following statements is not correct?
(a) In general, cis isomer has higher boiling point and lower melting point as compared to its trans isomer.
(b) The treatment of vicinal dihalides with zinc generates carbon-carbon triple bond.

it by treating with zinc.


(d) The ease of dehydrohalogenation of alkyl halides is 3° > 2° > 1°.
Preparation of Alkenes
11. The dehydrohalogenation of an alkyl halide to produce an alkene can be brought about by using
(a) aqueous KOH (b) alcoholic KOH (c) conc. H2SO4 (d) AlCl3
22.20 Complete Chemistry—JEE Main

12. The treatment of 2, 3-dibromobutane with Zn followed by heating produces


(a) CH3C CH CH3 (b) CH3CH==CHCH3 (c) CH3C∫∫CCH3 (d) CH3C == C CH3
| |
Br Br Br
13. The electrolysis of aqueous solution of potassium succinate produces
(a) methane (b) ethene (c) acetylene (d) methyl alcohol
14. During debromination of meso-dibromobutane, the major compound formed is
(a) n-butane (b) 1-butene (b) cis-2-butene (d) trans-2-butene
15. Which of the following statements is not correct?
(a) The ease of dehydration of alcohols is 1° > 2° > 3°.
(b) The dehydrohalogenation of alkyl halides is an example of 1,2-elimination and is brought about by action of
a base.
(c) 1-2 Elimination reaction involving E2 mechanism does not involve any rearrangement of carbon skeleton.
(d) 1-2 Elimination reaction involving E1 mechanism may involve the rearrangement of carbon skeleton.
16. Which of the following statements is not correct?
(a) In dehydrohalogenation, the more stable the alkene, the faster it is formed.
(b) The order of reactivity of alkyl halides towards E2 dehydrohalogenation is 3° > 2° > 1°.
(c) Hydrogenation of an alkene is an exothermic reaction.
(d) n-propyl alcohol can be produced by treating propylene with 80% H2SO4 followed by heating with water.
17. Which of the following statements is correct?
(a) The major product in the dehydrohalogenation of 2-chloro-2,3-dimethylbutane is 2,3-dimethyl-1-butene.
(b) The order of reactivity of n-propyl bromide, isobutyl bromide and ethyl bromide towards E2 dehydrohalogenation
is ethyl > n-propyl > isobutyl.
(c) Of the simple dialkyl ethylenes, the cis isomer is more stable than the trans isomer.
(d) If enthalpy of combustion of cis-2-butene is more negative than trans-2-butene, then the conversion of cis to
trans isomer is exothermic.
18. Which of the following statements is correct?
(a) The dehalogenation of meso-2, 3-dibromobutane produces cis-2-butene
(b) The dehalogenation of (S, S)-2, 3-dibromobutane produces trans-2-butene
(c) Dehydrobromination of (R, R)-2, 3-dibromobutane produces cis-2-bromobutene
(d) Dehydrobromination of meso-2, 3-dibromobutane produces cis-2-bromo-2-butene
Reactions of Alkenes

(a) ethylene glycol (b) acetic acid


(c) ethyl alcohol (d) carbon dioxide and water
20. The Baeyer’s reagent is
(a) alkaline KMnO4 4 (c) alkaline K2Cr2O7 2Cr2O7
21. The addition of HOCl to but-1-ene produces
(a) 1-hydroxy-2-chlorbutane (b) 2-hydroxy-1-chlorobutane
(c) 1-hydroxy-3-chlorobutane (d) 3-hydroxy-1-chlorobutane
22. The ozonolysis of 2, 4-dimethylpent-2-ene produces
(a) two molecules of aldehydes
(b) two molecules of ketones
(c) one molecule of aldehyde and one molecule of ketone
(d) neither aldehyde nor ketone
23. The ozonolysis of 2, 3-dimethylpent-2-ene produces
(a) two molecules of aldehydes
Hydrocarbons 22.21

(b) two molecules of ketones


(c) one molecule of aldehyde and one molecule of ketone
(d) neither aldehyde nor ketone
24. The ozonlysis of 2, 5-dimethylhex-3-ene produces
(a) two molecules of aldehydes
(b) two molecules of ketones
(c) one molecule of aldehyde and one molecule of ketone
(d) neither aldehyde nor ketone
25. The addition of HI in the presence of peroxide does not follow anti-Markovnikov’s rule because
(a) HI bond is too strong to be broken homolytically (b) I atom is not reactive enough to add on a double bond
(c) I combines with H to give back HI (d) HI is a reducing agent
26. The addition of HCl in the presence of peroxide does not follow anti-Markovnikov rule because
(a) HCl bond is too strong to be broken homolytically
(b) Cl atom is not reactive enough to add on a double bond
(c) Cl combines with H to give HCl
(d) HCl is a reducing agent
27. Ozonlysis of 2-butene produces
(a) ethanal only (b) methanal only (c) butanal (d) propanone
28. Ozonlysis of 2-methylbut-2-ene produces
(a) ethanal only (b) acetone only (c) ethanal and propanal (d) ethanal and acetone
29. The peroxide effect involves
(a) ionic mechanism (b) free-radical mechanism

30. Identify the product in the following reaction:


heated with Cr O
n-heptane æ æææææ Æ ‘product
2 3

supported on alumina
at 600∞C
(a) benzene (b) toluene (c) xylene (d) cyclohexane
31. The treatment of ethylene with Baeyer’s reagent produces
(a) ethyl alcohol (b) acetaldehyde
(c) ethylene glycol (d) a-hydroxyl acetaldehyde
32. In which of the following reactions, addition does not take place in accordance with the Markovnikov’s rule?
RCOOR RCOOR
(a) CH3CH == CH2 + HCl ææææ
ƺ (b) CH3CH==CH2 + HBr ææææ
ƺ
RCOOR
(c) CH3CH==CH2 + HI ææææ
ƺ (d) CH3CH==CH2 + HOCl Æ …

(a) propene (b) 2-methyl-1-butene


(c) 2, 3-dimethylbut-2-ene (d) 1-butene
34. The treatment of CH3CH==CHCH3 with NaIO4 or boiling KMnO4 produces
(a) CH3CHO only (b) CH3COOH only
(c) CH3CHO and CH3COOH (d) CH3COCH3 and HCOOH
35. The treatment of CH3C == CHCH3 with NaIO4 or boiling KMnO4 produces
|
CH3
(a) CH3COCH3 and CH3COOH (b) CH3COCH3 and HCHO
(c) CH3CHO and CO2 (d) CH3COCH3 only
22.22 Complete Chemistry—JEE Main

36. When ethene reacts with bromine in aqueous sodium chloride solution, the product(s) obtained is (are)
(a) ethylene dibromide only (b) ethylene dibromide and 1-bromo-2-chloroethane
(b) 1-bromo-2-chloroethane only (d) ethylene dichloride only
37. Which of the following statements is not correct?
(a) The electophilic addition across the double bond may be accompanied with the rearrangement of carbon
skeleton.
(b) Markovnikov’s rule is in agreement with Saytzeff’s rule.
(c) The rate of addition of a hydrogen ion to a double bond depends upon the stability of the carbocation being
formed.
(d) The more stable the carbocation formed as an intermediate in the addition reaction of alkene, the slower it is
formed.
38. Which of the following statements is correct?
(a) During the addtion of halogens to alkenes, a carbocation is formed as an intermediate.
(b) The addition of aqueous Br2 to ethylene in the presence of NaCl forms only dibromoethylene.
(c) Polyethylene is essentially an alkane with a very long chain.
(d) Treatment of an alkene with cold alkaline potassium permangate breakes the double bond.
39. Which of the following statements is correct?
(a) 1,2-dichloroethene is more reactive towards addition of H2SO4 in comparison to vinyl chloride.
(b) The function of the zinc dust in the ozonolysis of an alkene is to destroy H2O2 which is formed during the
reaction.
(c) Dehydration of 1-butanol mainly gives 2-butene.
(d) Dehydrohalogenation of 2-bromopentane results in the formation of 1-pentene.
40. Which of the following statements is correct?
(a) The addition of HBr to 1,3-butadiene at – 80 °C produces 1-bromo-2-butene as the major product.
(b) The addition of HBr to 1,3-butadiene at 40 °C produces 3-bromo-1-butene as the major product.
(c) The addition of HBr to 1,3-butadiene produces both 1-bromo-2-butene and 3-bromo-1-butene which are in
equilibrium with each other. The relative amounts of the two are temperature dependent.
(d) The treatment of gaseous propene with chlorine at 500-600 °C yields 1,2-dichloropropane.
41. Which of the following statements is correct?
(a) The trans-2-butene is more stable than cis-2-butene.
(b) The acid-catalysed dehydration of an alcohol follows E2 mechanism.
(c) The dehydration of an alcohol can be carried out in the presence of a base.
CH2OH CH2

(d) The acid-catalysed dehydration of produces as the major product.

42. Which of the following statements is correct?


(a) The addition of Br2 to cyclohexene produces cis-1, 2-dibromocyclohexane
(b) The addition of Br2 to cyclohexene, in the presence of NaCl produces a mixture of trans- bromohexane, trans-
1-bromo-2-cholorohexane and trans-dibromohexane
(c) The addition of Br2 to cis-2-butene produces racemic-2, 3-dibromobutane
(d) The addition of Br2 to trans-2-butene produces racemic-2, 3-dibromobutane
43. Which of the following statements is correct?
(a) The addition of H2 to cis-2, 3-dibromobutene produces meso-2, 3-dibromobutane
(b) The addition of H2 to trans-2, 3-dibromobutene produces meso-2, 3-dibromobutane
Hydrocarbons 22.23

(c) The hydroxylation of cis-2, 3-dimethylbutene with Baeyer’s reagent produces racemic-2, 3-dihydroxybutane
(d) The hydroxylation of trans-2, 3-dimethylbutene with Baeyer’s reagent produces meso-2, 3-dihydroxybutane.

ANSWERS
1. (b) 2. (b) 3. (c) 4. (a) 5. (c) 6. (a)
7. (c) 8. (a) 9. (c) 10. (b) 11. (b) 12. (b)
13. (b) 14. (d) 15. (a) 16. (d) 17. (d) 18. (d)
19. (d) 20. (d) 21. (b) 22. (c) 23. (b) 24. (a)
25. (b) 26. (a) 27. (a) 28. (d) 29. (b) 30. (b)
31. (c) 32. (b) 33. (c) 34. (b) 35. (a) 36. (b)
37. (d) 38. (c) 39. (b) 40. (c) 41. (a) 42. (c)
43. (a)

HINTS AND SOLUTIONS


CH3
3 2 1
5. CH3 C CH CH2 ; 3, 3-Dimethylbut-1-ene
CH3
8. Increasing R groups attached to double bonds increases stability of alkene.
10. (a) In general, cis isomer has higher polarity and hence has higher boiling point. On the other hand, because of
lower symmetry, cis
(b) With zinc, dehalogenation occurs producing carbon-carbon double bond. The use of alcoholic KOH followed
by KNH2 produces carbon-carbon triple bond.
Zn
12. CH3 CH CH CH3 Æ CH3CH==CHCH 3
Br Br
13. CH 2 COOK + 2H 2 O Æ H2C==CH2 + 2CO2 + 2KOH + H2
|
CH 2 COOK
16. (b) This is due to the fact that the intermediate carbocation has the stability in the order 3° > 2° > 1°.
(d) The addition of H2SO4 to double bond follows Markovnikov’s rule
H SO H O
CH3CH == CH 2 æææ
2 4
Æ CH3CHCH3 æheat
ææ
2
Æ CH3CHCH3
| |
OSO3 H OH
Hence, n-propyl alcohol cannot be produced by this method.
17. (a) According to Saytzeff’s rule, the more substituted alkene is more stable and is thus a preferred product. Thus,
the major product is
H3C H
CH3 C C CH3 Æ CH3 C C CH3
Cl CH3 CH3 CH3
2,3-dimethyl-2-butene
(b) Reactivity in E2 dehydrohalogenation depends mainly upon the stability of the alkenes being formed.
CH 3 CH3
CH3 CH CH 2Br Æ CH 3 C CH 2 ; CH CH CH Br Æ H C—CH=CH ; CH CH Br Æ CH ==CH
3 2 2 3 2 3 2 2 2
isobutyl bromide disubstituted n-propyl bromide monosubstituted thyl bromide unsubstituted
22.24 Complete Chemistry—JEE Main

According to Saytzeff’s rule, the stability of alkenes increases with increase in the number of substituents on
the doubly bonded carbons. Hence, the order of reactivity towards dehydrohalogenation is
isobutyl > n-propyl > ethyl
(c) A trans isomer is usually more stable than a cis isomer.
18. (a) The Product is trans-2-butene.

(b) The product is cis-2-butene.

(c)

(d)

Cr O 2- H +
19. CH2==CH2 æææææ
2 7
Æ 2CO2 + 2H2O
HOCl
21. CH3—CH2—CH==CH2 æææÆ CH3CH2CH(OH)CH2Cl

Æ CHCHCHO + OC (CH3 )2
O
22. CH3CHCH == CCH 3 ææ
3

| | | (ketone)
CH3 CH3 CH3
(aldehyde)
Hydrocarbons 22.25

O3
23. CH3CH2C CCH3 Æ CH3CH2C O+O C CH3
CH3 CH3 CH3 CH3
(ketone) (ketone)

24. CH3CHCH == CHCHCH3 æOæ


3
Æ 2 (CH3 )2CHCHO
| | (aldehyde)
CH3 CH3

27. CH3CH == CHCH3 æOæ


3
Æ 2CH3CHO
ethanol

O
28. CH3CH == C(CH3)2 ææ
Æ CH3CHO + OC(CH3)2
3

ethanal acetone

31. H2C==CH2 æBaeyer’s


æææææ
reagent
Æ HOCH2CH2OH
RCOOR
32. CH3CH==CH2 + HBr ææææ
Æ CH3CH2CH2Br

33.

37. (b) Both Markovnikov and Saytzeff rules follows from the fact that there is an intermediate formation of the more
stable carbocation.
(c) The more stable the carbocation, the more quickly it is formed.
38. (a) A halonium ion is formed as the intermediate instead of a carbocation.
(b) Besides dibromo compound, the bromochloro and bromoalcohol compounds are also formed.
(c) The structure of polyethylene is (—CH2—CH2—)n.
(d) A diol is formed.
39. (a) Two chlorine atoms attached to ethylene destabilizes the intermediate carbocation more than the monochloro
substituted ethene.
40. (a) At lower temperature, 1,2-adduct predominates over the 1,4-adduct. Hence, the major component is 3-bromo-
1-butene.
(b) At higher temperature, 1,4-adduct predominates over the 1,2-adduct. Hence, the major component is
1-bromo-2-butene.
(d) At the given conditions, substitution at alkyl group takes place. This gives allyl chloride.
41. (b) The mechanism for the acid-catalysed dehydration of an alcohol (say, isopropyl alcohol) is as follows.
Step 1 CH3 CH CH3 + H+ CH3 CH CH3
from acid
+ OH2
OH
+
Step 2 CH3 CH CH3 Æ CH3CHCH3 + H2O
+
OH2

Step 3 CH3 CH CH2 + HSO4 CH3CH == CH2 + H2SO4

H
Step 1 is fast equilibrium reaction. Step 2 is the slowest step as bond breaking involves high energy intermediate.
It is also a rate determining step. Since it involves only one species, the elimination reaction follows E1
mechanism.
(c) The base catalysed reaction will involve the reaction
22.26 Complete Chemistry—JEE Main

+
CH 3 — CH —CH 3 CH3— C H—CH3 + OH –
|
OH
which is not possible, as the leaving group OH– is a strong base and is a poor leaving group. In the presence
of acid, the poor leaving group OH– is converted into a good leaving group H2O (a very weak base).
(d) The product is 1-methylcyclohexene. An :H shift converts the less stable 1° to a more stable 3° carbocation.

+ H+

42. (a) The addition of halogen to an alkene involves anti-addition. Thus, trans-1, 2-dibromocyclohexane is obtained.
(b) Only a mixture of trans-dibromocyclohexane and trans-1-bromo-2-chlorocyclohexane is obtained. Trans-
dichlorocyclohexane is not obtained as it will require the presence of Cl+ which is not available.
(c) The addition of Br 2 across a double bond involves the formation of a bromonium ion intermidiate,
followed by the anti addition of Br–. For cis-2-butene, the reaction proceeds so as to form racemic-2,
3-dibromobutane.
H H Br CH3
Br CH3 CH3 H CH3 Br H
H H Br Br
H H3C H3C H H Br
CH3 Br + CH3 Br H Br CH3
CH3 (2R, 3R)
(c) H Br2
H H Br CH3 Br CH3
CH3
CH3 CH3 H H3C H H Br
H Br
H H
Br CH3 CH3 CH3 Br H
Br
Br CH3
(2S, 3S)

CH3 CH3 Br CH3


Br H H H3C H H Br
CH3 H Br Br
CH3 H3C H3C H H Br
H
+
Br CH3 Br H Br CH3
Br2 H (2S, 3R) meso
(d) H
H Br H
H3C Br CH3
CH3
CH3 H CH3 H CH3 Br H
H Br
H H
Br CH3 CH3 CH3 Br H
Br
Br CH3
(2R, 3S) meso
43. (a) Hydrogenation of an alkene is a heterogeneous process which occurs on the surface of a solid catalyst (PtO2 or
Pd/C). The addition occurs with syn stereochemistry (both hydrogens add from the same side). For cis isomer,
the product is a meso compound.
H3C H Br H CH3
H Br CH3 Br CH3 H Br
CH3 H3C Br
H Br CH3 H Br
Br CH3
Br H H CH3
H2 (2S, 3R) meso
H3C
CH3
H CH3
Br
Br H3C Br Br H
H3C H
H Br H3C Br Br H
H CH3
(2R, 3S) meso (same product)
Hydrocarbons 22.27

(b) The product is a racemic mixture.


H3C H Br H CH3
H Br CH3 Br CH3 H Br
H3C Br
H3C H H3C Br Br H
CH3 Br
Br H H CH3
H2 (2S, 3S)
Br
H3C
H CH3
CH3
Br H3C Br Br H
Br H
H CH3 Br CH3 H Br
H CH3
(2R, 3R)
(c) The addition of two hydroxyl groups involves syn addition (on the same side of a double bond). The result is
same as that of hydrogenation. The cis isomer produce meso compound.
(d) The addition of two hydroxyl groups involves syn addition (on the same side of a double bond). The result is
same as that of hydrogenation. The trans isomer produces racemic mixture.

MULTIPLE CHOICE QUESTIONS FROM AIEEE AND JEE MAIN

1. Among the following structures


CH3 CH3

C 2H 5 CH C 3H 7 CH3COCHC2H5
(I) (II)

H
CH3
+
H C
C 2H 5 CH C 2H 5
(IV)
H
(III)
it is true that
(a) only II and IV are chiral compounds (b) all four are chiral compounds
(c) only I and II are chiral compounds (d) only III is a chiral compound [2003]
2. Reaction of one molecule of HBr with one molecule of 1, 3-butadiene at 40ºC gives predominantly
(a) 3-bromobutene under thermodynamically controlled conditions
(b) 1-bromo-2-butene under kinetically controlled conditions
(c) 3-bromobutene under kinetically controlled conditions
(d) 1-bromo-2-butene under thermodynamically controlled conditions [2005]
3. HBr reacts with CH2 = CH—OCH3 under anhydrous conditions at room temperature to give
(a) H3C—CHBr—OCH3 (b) CH3CHO and CH3Br
(c) BrCH2CHO and CH3OH (d) BrCH2—CH2–OCH3 [2006]
4. Reaction of trans 2-phenyl-1-bromocyclopentane on reaction with alcoholic KOH produces
(a) 3-phenylcyclopentene (b) 4-phenycylclopentene (c) 2-phenylcyclopentene (d) 1-phenylcyclopentene
[2006]
22.28 Complete Chemistry—JEE Main

Me D
5. The alkene formed as a major product in the elimination reaction is
Me
N
OH
-Bu Et
Me Me
(a) (b) Me (c) CH2 = CH2 (d) [2006]

6. In the following sequence of reactions, the alkene forms the compound B.


O3 H2O
CH3 CH == CHCH3 A B
Zn
The compound B is
(a) CH3CHO (b) CH3CH2CHO (c) CH3COCH3 (d) CH3CH2COCH3
[2008]
7. The alkene that exhibits geometrical isomerism is
(a) 2-Butene (b) 2-Methyl-2-butene (c) Propene (d) 2-Methylpropene
[2009]
8. The main product of the following reaction
conc. H SO
C6H5CH2CH(OH)CH(CH3)2 æææææ
2 4
Æ?
is
C6H5CH2CH2 C6H5 H
(a) C CH2 (b) C C
CH3 H CH(CH3)2

C6H5CH2 CH3 C6H5 CH(CH3)2


(c) C C (d) C C [2010]
H CH3 H H
9. One mole of a symmetrical alkene on ozonolysis gives two moles of an aldehyde having a molecular mass 44 u.
The alkene is
(a) ethene (b) propene (c) 1-butene (d) 2-butene [2010]
10. Out of the following, the alkene that exhibits optical isomerism is
(a) 2-methyl-2-pentene (b) 3-methyl-2-pentene (c) 4-methyl-1-pentene (d) 3-methyl-1-pentene
[2010]

(a) an acetylenic triple bond (b) two ethylenic double bonds


(c) a vinyl group (d) an isopropyl group [2011 cancelled]
12. Elimination of bromine from 2-bromobutane results in the formation of
(a) predominantly 1-butene (b) predominantly 2-butyne
(c) equimolar mixture of 1-butene and 2- butene (d) predominantly 2-butene [2005]
13. In allene (C3H4), the type (s) of hybridization of the carbon atoms is (are):
(a) sp and sp3 (b) sp2 and sp (c) only sp2 (d) sp2 and sp3
[2014, online]
14. The gas liberated by the electrolysis of dipotassium succinate solution is:
(a) Ethane (b) Ethyne (c) Ethene (d) Propene [2014, online]
15. In the presence of peroxide, HCl and HI do not give anti-Markownikoff’s addition to alkenes because:
(a) One of the steps is endothermic in HCl and HI
(b) Both HCl and HI are strong acids
(c) HCl is oxidizing and the HI is reducing
(d) All the steps are exothermic in HCl anHI [2014, online]
Hydrocarbons 22.29

16. What is the major product expected from the following reaction? [2015, online]
CH3

D—Cl

where D is an isotope of hydrogen


(a) CH3 (b) CH3 (c) D (d) D

H H Cl
Cl Cl CH3 CH3
D H Cl H
17. Which of the following compounds will exhibit geometrical isomerism? [2015]
(a) 1-phenyl-2-butene (b) 3-phenyl-1-butene (c) 2-phenyl-1-butene (d) 1, 1-Diphenyl-1-propane
18. Which compound would give 5-keto-2-methylhexanal upon ozonolysis? [2015]
CH3 CH3 CH3 CH3
(a) CH3 (b) (c) (d) H3C

CH3
CH3
19. Bromination of cyclohexene under conditions given below yields:

Br2/hn

Br
Br
Br Br
(a) (b) (c) (d) [2016, online]
Br Br
Br
20. The reaction of propene with HOCl (Cl2 + H2O) proceeds through the intermediate
+ +
(a) CH3 – CH – CH2Cl (b) CH3 – CH(OH) – CH 2
+ +
(c) CH3 – CHCl – CH 2 (d) CH3 – CH – CH2 – OH [2016]

ANSWERS
1. (c) 2. (d) 3. (a) 4. (a) 5. (c) 6. (a)
7. (a) 8. (b) 9. (d) 10. (d) 11. (c) 12. (d)
13. (b) 14. (c) 15. (a) 16. (a) 17. (a) 18. (b)
19. (d) 20. (a)

HINTS AND SOLUTIONS


2. At 40°C, thermodynamically controlled, 1, 4-addition product predominates.
HBr
CH2==CH — CH==CH2 CH3CH==CHCH2Br
40°C
1-Bromo-2-butene
At low temperature (0 ºC), kinetically controlled 1,2-addition product predominates.
Br
2
1
C6H5 C6H5
4. The involved reaction is 2 ethanolic 1 3
KOH
22.30 Complete Chemistry—JEE Main

5. The major product in Hofmann elimination is to give the less substituted alkene resulting from the loss of the more
acidic b H (1° < 2° > 3°).
6. The given reactions are as follows.
O
CH 3CH CHCH 3 O3 CH3 CH CHCH3
Zn/H2O
2 CH3CHO

O O
7. The given compounds are:
(a) H3C—CH == CH—CH3 (b) H3C—CH == C — CH3
CH3
(c) H3H—CH == CH3 (d) H3C—C == CH2
CH3
Only the compound (a) can exists as cis-trans isomers.
8. The dehydration of the given alcohol can give two products. These are
C6H5 H C6H5CH2 CH3
C CH and C C
H CH(CH3)2 H CH3

9. The aldehyde CH3CHO has molecular mass of 44 u. Hence, the symmetrical alkene is CH3CH CHCH3.
O3
CH3CH==CHCH3 ææÆ 2CH3 CHO
2-Butene.
10. The structures of given alkenes are as follows.
CH3 CH3 CH3 CH3
CH3C CH CH2CH3 CH3CH C — CH2 CH3 CH2 CH CH2CHCH3 CH2 CH CHCH2CH3
2-Methylpentene 3-Methyl-2-pentene 4-Methyl-1-pentene 3-Methyl-1-pentene
The molecule 3-methyl-1-pentene has one assymetrical carbon atom and thus it will exhibit optical isomerism.
11. A vinyl group will produce formaldehyde on ozonolysis.
O3
H 2C CH ææÆ HCHO + other fragment
12. The predominant product is 2-butene (a Saytzeffs products).
–HBr
CH3CHCH2CH3 CH3CH CHCH3
Br 2-butene

1 2 1
13. The allene is C H 2 = C = C H 2 . In this compound, carbon labelled as (1) is sp2 hybridized while carbon labelled
as (2) is sp hybridized.
14. The electrolysis of dipotassium succinate solution releases ethene.
CH2COO CH2
+ 2CO2 + 2e
CH2COO CH2
ethene
15. In the chain propagation steps, we have

Step 1 X+ C C C C
+
X

Step 2 C C + HX C C +X
+
X X
Hydrocarbons 22.31

With HCl, Step 2 is endothermic owing to strong H-Cl bond.


With HI, Step 1 is endothermic owing to weak HI bond. In fact, HI is oxidised to I2 by peroxide.
16. CH3 Cl
CH3 DCl H + CH3 H + H
Cl CH3
D D D
(major) (minor)

17. The compound 1-phenyl-2-butene will exhibit geometrical isomerism. The isomers are:

PhCH2 CH3 PhCH2 H


C == C and C == C
H H H CH3
6 6
CH3 CH3
5 5
18. 4 1 Ozonolysis 4 O
CHO
1
3 2 CH3 3 2
CH3
5-keto-2-methyl-hexanal

19. In the presence of ultraviolet radiation, the allylic C—H bond undergoes bromination.

Br

Br2/hn

20. The reaction proceeds as follows


+ -
CH3 CH CH2 + Cl+ CH3 CH CH2Cl OH CH3 CH CH2Cl

OH

SECTION 3 Alkynes

Methods of Preparation
Dehydrohalogenation of Vicinal Dihalides
Dehydrohalogenation takes place in two stages

First stage requires mild conditions whereas second stage requires more vigorous conditions as vinylic halide is
unreactive.
22.32 Complete Chemistry—JEE Main

Reaction of Metal Acetylides with Primary Alkyl Halides


LiNH 2 –
C C H Æ C C : Li + + RX Æ C C R + LiX
(R must be 1°)
With secondary and tertiary halides, elimination is the predominant reaction which results in the formation of alkenes.
This method can be used to generate larger alkyne from the smaller one.
Hydrolysis of CaC2 and Mg2C3
Calcium carbide gives acetylene whereas Mg2C3 gives propyne.
CaC2 + 2H2O Æ HC CH + Ca(OH)2
Mg2C3 + 4H2O Æ CH3C CH + 2Mg(OH)2

Kolbe’s Electrolytic Method


The electrolysis of an aqueous solution of potassium salt of an unsaturated dicarboxylic acid forms alkyne.
Anodic reaction CHCOO– Æ HC CH + 2CO 2 + 2e–
CHCOO–
Cathodic reaction 2H2O + 2e– Æ 2OH– + H2

Dehalogenation of vic-Tetrahalogen Compounds


C 2 H 5 OH
CH3— CBr2— CBr2— CH3 + 2Zn Æ CH3— C C— CH3 + 2ZnBr2
heat

Chemical Properties
Addition of Hydrogen
If the triple bond is not present at the end of the chain of the molecule, its reduction produces either a cis alkene or a
trans alkene depending upon the choice of reducing agent. The use of a catalyst causes syn addition resulting into cis-
alkene whereas the reduction with a reducing agent causes anti addition resulting into trans-alkene.

Na, NH3 (liq) H R


C C anti addition
R H
R C C R

Na or Li
R R
C C syn addition
Lindlar catalyst H H
(Pb-BaSO4)

Further reduction produces an alkane.


Addition of Halogens
Chlorine and bromine add on alkyne:
X X
X2 X2
R C CH ÆR C CH ÆR C C H
X X X X

Alkynes are less reactive than alkenes. The mechanism of the reaction involves the initial formation of a cyclic halonium
sulting in the lower activity of alkynes.
Hydrocarbons 22.33

Addition of Hydrogen Halides (HCl, HBr, HI)


Cl
HCl HCl
R C CH ÆR C CH2 ÆR C CH3
Cl Cl
The addition takes place in accordance with Markovnikov’s rule. Peroxides have the same effect on addition of HBr to
acetylenes as they have on alkenes.

Addition of Water
In the presence of acid and HgSO4, a molecule of water adds to the triple bond. Initially enol is formed which is rapidly
converted into an equilibrium mixture containing keto form in excess.
2 H SO
4
R C C R´ + H2O HgSOÆ R C C R´
4
H OH

Æ
Æ
Keto-enol tautomerism
R CH2 C R´
O
2COR¢ and RCOCH2R¢ ).

Formation of Metal Acetylides


A triply bonded carbon which is sp hybridized has a high electronegativity and hydrogen atom attached to it shows
appreciable acidity. For example, acetylene reacts with Na and Li liberating H2 gas:
HC C—H + Na Æ HC C–: Na+ + 12 H2
sodium acetylide
Compared to H2O, NH3 and alkane, the order of acidity of acetylene is as follows.
H2O > HC CH > NH3 > RH
The order of conjugate base will be
OH– < HC C– < NH2– < R–
Lithium acetylide has been used to convert lower alkynes into the bigger one using the reaction
R—C C—Li + R¢ X Æ R—C C—R¢ + LiX
(1° alkyl halide)
Heavy metal ions, mainly, Ag+ and Cu+, react with acetylinic hydrogen to form insoluble acetylides. This reaction can
be used to differentiate between terminal and nonterminal alkynes:
ammoniacal Ag+
R —C C —H Æ R— C C—Ag
(Tollens reagent)
precipitate

2Ag +
H —C C —H Æ Ag—C C—Ag
Cu +
R —C C —H Æ R— C C—Cu
ammoniacal
medium

Reaction with Grignard Reagent or Alkyllithium


The acetylinic hydrogen on reacting with R´MgBr or R´Li produces the alkane R´H:
RC CH + R¢ MgBr Æ R¢ H + RC CMgBr
RC CH + R¢ Li Æ R¢ H + RC CLi
22.34 Complete Chemistry—JEE Main

Ozonolysis
Alkynes produces carboxylic acids.
O
O3 H 2O
CH3CH2C CCH3 Æ CH3CH2C CCH3 Æ CH3CH2COOH + CH3COOH
O O
Polymerization
Acetylene dimerizes in the presence of cuprous chloride and ammonium chloride to give vinyl acetylene which on
reacting with HCl gives chloroprene. The latter polymerizes to give neoprene—a synthetic rubber. The reactions are:
Cu Cl
CH NH ClÆ CH2
2 2
HC CH + HC CH C CH
4 HCl
vinyl acetylene

Cl Polymerization
Æ peroxide CH2 CH C CH2
CH2CH C CH2 n
Cl
chloproprene

Cyclic Polymerization
The cyclic polymerization takes place when alkyne is passed through red hot iron tube at 400°C.

hot iron tube


3HC CH Æ
400 C
benzene

CH3

hot iron tube


3CH3C CH Æ
400 C
H3C CH3
mesitylene

Isomerization
The following reactions can be used to convert 1-alkyne to 2-alkyne and vice versa.
KOH (alc)
CH3CH2C CH Æ CH3CH==C==CH2
Æ

allene
H2O
Æ

NaNH 2 in
CH3CH2C CNa Æ CH3C CCH3
inert solvent

MULTIPLE CHOICE QUESTIONS ON SECTION 3

General Characteristics of Alkynes


1. Ethylene can be separated from acetylene by passing the mixture through
(a) fuming H2SO4 (b) pyrogallol (c) ammoniacal CuCl2 (d) charcoal powder
2. Which of the following has the highest boiling point?
(a) 1-Butyne (b) 2-Butyne (c) 1-Pentyne (d) 2-Hexyne
Hydrocarbons 22.35

3. Lindlar’s catalyst is
(a) Pt in ethanol (b) Ni in ethanol (c) Pd with BaSO4 (d) Na in liquid NH3
4. The two hydrogen atoms in acetylene
(a) are acidic in nature (b) are alkaline in nature
(c) are neutral in nature (d) are acidic and alkaline in nature, respectively.
5. Which of the following is true?
(a) Acetylene is more reactive than ethylene to an electrophilic attack
(b) Acetylene is less reactive than ethylene to an electrophilic attack
(c) Acetylene may show more reactivity or less reactivity towards electrophilic attack depending upon electrophilic
reagent
(d) Acetylene and ethylene show identical reactivities towards an electrophilic attack
6. Which of the following molecules has all the three types sp, sp2 and sp3 of carbon atoms?
(a) CH2==CH—CH==CH2 (b) CH3—CH==CH—CH3
(c) CH3—CH==CH—C CH (d) CH3 CH2 CH2 CH3
7. The molecule CH3—CH==CH—C CH contains
(a) one sp3, two sp2 and two sp carbon atoms (b) one sp3, one sp2 and three sp carbon atoms
2
(c) two sp and three sp carbon atoms (d) two sp3, two sp2 and one sp carbon atoms
8. The IUPAC name of the compound
CH3
CH3 C C CH
CH3
is
(a) 3, 3, 3-trimethylpropane (b) 1, 1, 1-trimethylpropyne
(c) 3, 3-dimethylbut-1-yne (d) 2, 2-dimethyl but-3-yne
9. The IUPAC name of the compound HC CCH2CH==CH2 is
(a) 1-penten-4-yne (b) 1-pentyn-4-ene (c) allylacetylene (d) 1-penten-5-yne
10. The compounds 1-butyne and 2-butyne can be distinguished by using
(a) bromine water (b) KMnO4 solution (c) Tollens reagent (d) chlorine gas
11. Which of the following statements is correct?
(a) Alkynes are reduced less readily than alkenes
(b) Alkynes are reduced more readily than alkenes
(c) Both alkynes and alkenes are reduced with equal speeds
(d) Alkynes cannot be reduced to the alkene stage
12. Which of the following orders regarding acid strength is correct?
(a) CH3COOH > CH3CH2OH > CH CH (b) CH3COOH > CH CH > CH3CH2OH
(c) HC CH > CH3COOH > CH3CH2OH (d) HC CH > CH3CH2OH> CH3COOH
13. Which of the following statements is correct?
(a) Acetylene is more reactive than ethylene towards the addition of halogens
(b) Acetylene is less reactive than ethylene towards the addition of halogens
(c) Acetylene and ethylene show identical reactivities towards an electrophilic attack.
(d) The reactivities of acetylene and ethylene towards electrophilic attack depend on the electrophilic reagent.
14. Which of the following statements is correct?
(a) Ethyne and its derivatives give white precipitate with ammoniacal silver nitrate solution.
(b) The carbon-carbon triple bond in acetylene has a bond length of 121 pm.
(c) Alkynes are more dense than water.
(d) The hydrohalogenation of vinylic halide requires mild conditions.
22.36 Complete Chemistry—JEE Main

15. Which of the following statements is correct?


(a) Tautomers differ in the arrangement of atoms and are also in rapid equilibrium with each other.
(b) Keto-enol tautomerism is an equilibrium between two acids favouring the formation of the stronger acid.
(c) A pair of electron in an sp hybrid orbital of carbon is less near to the nucleus as compared to that in an sp3
hybrid orbital.
(d) 2-Butyne reacts with ammoniacal silver nitrate solution.
Preparation of Alkynes
16. The hydrolysis of Mg2C3 produces
(a) acetylene (b) propyne (c) butyne (d) ethylene
17. The hydrolysis of calcium carbide produces
(a) acetylene (b) propyne (c) butyne (d) ethylene
18. Acetylene is produced when C2H4Br2 is heated with
(a) aqueous KOH (b) ethanolic KOH and NaNH2
(c) dilute HCl (d) Zn dust
19. Acetylene is produced by carrying out the electrolysis of potassium salt of
(a) acetate (b) succinate (c) fumarate (d) oxalate
20. The product(s) obtained via oxymercuration (HgSO4 + H2SO4) of 1-butyne would be
(a) CH3CH2COCH3 (b) CH3CH2CH2CHO
(c) CH3CH2CHO + HCHO (d) CH3CH2COOH + HCOOH
Reactions of Alkynes
21. The addition of H2 to CH3C∫∫CCH3 using Lindlar’s catalyst produces
(a) cis-but-2-ene (b) trans-but-2-ene
(c) a mixture of cis- and trans-but-2-ene (d) 2-methylprop-1-ene
22. The addition of H2 to CH3C∫∫CCH3 in the presence of Na or Li in liquid ammonia produces
(a) cis-but-2-ene predominantly (b) trans-but-2-ene predominantly
(c) an equal mixture of cis- and trans-but-2-ene (d) 2-methylprop-1-ene
23. Ozonolysis of propyne gives
(a) acetic acid only (b) formic acid only (c) acetic and formic acids (d) acetaldehyde
24. When propyne is passed through red hot iron, it is converted into
(a) benzene (b) toluene (c) mesitylene (d) xylene
25. The addition of hypohalous acid to acetylene produces
OH Cl OH OH
H Cl Cl
(a) C C (b) H C C H (c) H C C H (d) CHCHO
HO H Cl
OH Cl Cl Cl
26. The hydration of propyne in the presence of HgSO4/H2SO4 produces
(a) CH3CHO (b) CH3CH2CHO (c) CH3COCH3 (d) HCHO
27. Sulphuric acid reacts with acetylene in the presence of mercuric sulphate to give
(a) acetic acid (b) acetaldehyde
(c) ethyl hydrogen sulphate (d) diethyl sulphate
28. Which of the following compounds reacts with an aqueous solution of Ag(NH3)OH?
(a) 2-Butyne (b) 1-Butyne (c) Ethene (d) Ethane
29. The cyclic polymerization of dimethylacetylene produces
(a) benzene (b) o-xylene
(c) 1, 3, 5-trimethylbenzene (d) 1, 3, 5-trimethylbenzene
30. The cyclic polymerization of acetylene produces
(a) benzene (b) cyclohexane (c) toluene (d) 1, 3, 5-trimethylbenzene
Hydrocarbons 22.37

31. The order of reactivity of acetylene with halogen acids is


(a) HI > HBr > HCl (b) HI < HBr < HCl (c) HI < HBr > HCl (d) HI > HBr < HCl
32. The ozonolysis of a triple bond produces
(a) a mixture of aldehyde/ketone and carboxylic acid
(b) a mixture of aldehydes/ketones
(c) a mixture of carboxylic acids
(d) CO2 and H2O
33. The reduction of an alkyne to alkene using Lindlar catalyst results into
(a) cis addition of hydrogen atoms
(b) trans addition of hydrogen atoms
(c) a mixture obtained by cis and trans additions of hydrogen in the ratio 1 : 1
(d) a mixture obtained by cis and trans additions of hydrogen in ratio 1 : 2
34. Which of the following statements is correct?
(a) The electrophilic addition of hydrogen halides to alkyne proceeds via the formation of an intermediate
carbocation.
(b) The addition of protic acids to alkynes takes place at very much the same rate as to alkenes.
(c) The addition of halogens to alkynes takes place at very much the same rate as to alkenes.

(d) The equilibrium C C OH C— C O lies much in favour of enol form.


H
35. Which of the following statements is correct?
(a) The addition of H2 to 2-butyne using Na in liquid NH3 gives cis-1,2-dimethylethene.
(b) The addition of H2 to 2-butyne using H2 and Lindlar catalyst gives cis-1,2-dimethylethene.
(c) The cis-1,2-dimethylethene is more stable than its trans isomer.
(d) cis-trans mixture of 2-pentene cannot be converted to either cis or trans form.
36. Which of the following statements is correct?
(a) The addition of HCl to 3,3-dimethyl-1-butyne exclusively gives 2,2-dichloro-3,3-dimethylbutane.
(b) The number of isomers (including structural and stereoisomers) of alkynes C6H10 is eight.
(c) The treatment of 1,1-dibromopentane with KOH(s) at 200 °C produces 1-pentyne.
(d) The treatment of 1,1-dibromopentane with NaNH2 followed by acid gives 2-pentyne.
37. Which of the following statements is correct?
(a) 2-Butyne is not soluble in concentrated H2SO4.
(b) The reaction of one mole of HBr with 1-pentene-4-yne is
H 2C CHCH2C CH HBr Æ H C CHCH2 C
2 CH2

Br
(c) Alkynes are more reactive than alkenes toward addition of Br2.
(d) Alkynes are more reactive than alkenes toward catalytic hydrogenation.
38. Which of the following statements is correct?
(a) The treatment of 3-heptyne with KMnO4 under neutral conditions at room temperature gives CH3CH2COOH
and CH3CH2CH2COOH.
(b) The treatment of 3-heptyne with KMnO4 under alkaline or acidic conditions at higher temperatures gives
CH3CH2COCOCH2CH2CH3.
(c) The addition of HBr to 1,3-pentadiene proceeds at a slower rate than that to 1,4-pentadiene.
(d) The addition of HBr to butadiene at –80 °C to give 1,2-adduct is kinetically controlled while that at 40 °C to
give 1,4-adduct is thermodynamically controlled.
22.38 Complete Chemistry—JEE Main

39. Which of the following statements is not correct?


(a) The addition of acetylene to lithium amide dissolved in ether produces NH3 and HC C Li + .
(b) If water can generate HC∫∫CH from HC∫∫CLi then water must be a stronger acid than acetylene.
(c) Acetylide ion (which has a pair of electrons in sp orbital of carbon) is more stronger base than ethide ion,
C2H5– (which has a pair of electrons in sp3 orbital of carbon).
(d) Lindlar catalyst is Pb/BaSO4.
40. Which of the following statements is correct?
(a) The polymerization of chloroprene gives synthetic rubber known as neoprene.
(b) The ozonlysis of propyne produces acetic acid.
(c) The IUPAC name of HC∫∫C—CH2==CH2 is 4-pentyne-1-ene.
(d) The number of isomer of alkyne C6H10 is six.

ANSWERS
1. (c) 2. (d) 3. (c) 4. (a) 5. (b) 6. (c)
7. (a) 8. (c) 9. (a) 10. (b) 11. (b) 12. (a)
13. (b) 14. (b) 15. (a) 16. (b) 17. (a) 18. (b)
19. (c) 20. (a) 21. (a) 22. (b) 23. (c) 24. (c)
25. (d) 26. (c) 27. (b) 28. (b) 29. (d) 30. (a)
31. (a) 32. (c) 33. (a) 34. (a) 35. (b) 36. (b)
37. (d) 38. (d) 39. (c) 40. (a)

HINTS AND SOLUTIONS


1. Hydrogen atoms in acetylene are acidic. These can be replaced by Cu+.
2. The boiling point increases with increase in carbon number.
5. For an electrophilic attack, acetylene is less reactive than ethylene.
6. CH3—CH==CH—C CH
sp3 sp2 sp2 sp sp

8.

9. C==C has priority over C C and gets the smaller number.


11. The adsorption of alkynes on the surface of a catalyst involves less strained transition state as compared to that
of alkenes. Hence, alkynes react faster than alkenes.
14. (a) White precipitate is obtained only when —C∫∫C— functional group is present at the terminal(s) of the
molecule, since only then it will have acidic hydrogen atom. (d) A vinylic halide is very unreactive. The use
of vigorous conditions (such as the use of a strong base, NaNH2) are required to remove HX.
15. (b) Equilibrium reaction always favours the formation of weaker substances. In keto-enol tautomerism, we have

C C O H C C O
stronger acid
H
weaker acid

(c) An sp hybrid orbital has more s-character as compared to an sp3 hybrid orbital.
(d) 2-butyne does not contain acetylinic hydrogen atom.
Hydrocarbons 22.39

16. Mg2C3 + 4H2O Æ CH3C CH + 2Mg(OH)2


17. CaC2 + 2H2O Æ HC CH + Ca(OH)2
19. HCCOOK + 2H2O Æ HC CH + 2CO2 + 2KOH + H2
HCCOOK
21. The use of Lindlar’s catalyst causes cis-addition.
22. The use of Na or Li in liquid ammonia produces trans form predominantly.
23. CH3C CH O 3 Æ CH3COOH + HCOOH
–H 2 O
25. HC CH HOCl
Æ HOCH==CHCl HOCl
Æ HOCH CHCl2 Æ OCH CHCl2
OH
OH
HgSO4 H 2SO 4 –H2O
26. CH3C CH Æ CH3C CH2
HgSO 4 / H 2 SO 4
CH3COCH3 CH3C CH3
H2O H 2O
OH OH

OH
–H 2O
HgSO 4 / H 2 SO 4
27. HC CH HgSO 4 H 2SO 4
Æ CH2==CHOH CH3CHO CH3 C H
H2O H 2O
OH

28. A compound containing acidic hydrogen in —C ∫∫ CH group reacts with Ag(NH3)OH.

CH3
H3 C CH3
29. 3CH3C CH3 Æ
H3 C CH3
CH3

30. 3HC CH Æ

34. (c) Alkynes are considerably less reactive than alkenes as for as the addition of halogen is concerned. This is due

(d) It lies much in favour of keto form.


35. (a) The trans isomer is obtained.
(c) The trans isomer is more stable than its cis isomer.
(d) The conversion may be achieved as follows.

36.(a) The product is a mixture of three isomers as explained in the following.


22.40 Complete Chemistry—JEE Main

CH3 CH3 CH3 Cl CH3 Cl


HCl + Cl– HCl
CH3 C C CH Æ CH3 C C CH2 CH3 C C CH2 Æ CH3 C C CH3
CH3 CH3 CH3 CH3 Cl
vinylic cation 2,2-dichloro-
3,3-dimethylbutane

Besides this, the other products are obtained due to the methyl shift in vinylic cation.

CH3 CH3 CH3

CH3 ⎯ C ⎯ C == CH2 → CH3 ⎯ C C CH 2

CH3
Cl −

CH3 CH3 CH3 CH3

CH3 ⎯ C == C ⎯ CH2Cl CH3 ⎯ C ⎯ C == CH2

HCl Cl
HCl

CH3 CH3 CH3 CH3

CH3 ⎯ C ⎯ CH ⎯ CH2Cl CH3 ⎯ C ⎯ C ⎯ CH3

Cl Cl Cl
1, 3-dichloro-2, 3-dimethylbutane 2, 3-dichloro-2, 3-dimethylbutane

(b) There are eight isomers including stereoisomers.


HC CCH2CH2CH2CH3; CH3C CCH2CH2CH3; CH3CH2C CCH2CH3; CH 3 — CH—C ≡≡ CCH 3
1-pentyne 2-pentyne 3-pentyne |
CH 3
4-methyl-2-pentyne

H H
CH 3 — CH—CH 2 —C CH CH3CH2 C CH HC C CH2CH3
| ;
CH 3 CH3 CH3
4-methyl-1-pentyne (S)-3-methyl-1-pentyne (R)-3-methyl-1-pentyne

CH 3
|
CH 3 — C—C CH
|
CH 3
3,3-dimethyl-1-butyne

(c) The initially formed 1-pentyne is converted into 2-pentyne because the more highly R-substituted alkyne is more
stable.
(d) The very strong base NaNH2 removes the terminal proton from the dehydrohalogenation product and thus it
cannot rearrange.
H 3O
CH3CH2CH2CH2CHBr2 NaNH 2
Æ CH3CH2CH2C C– Æ CH3CH2CH2C CH
37. (a) The vinyl carbocation from 2-butyne is more stable because of the presence of electron-repelling CH 3
+
group attached to C+ (i.e. CH3—C==CH—CH3). Thus, 2-butyne is soluble in concentrated H2SO4 due to the
+
formation of the salt (CH3—C==CH—CH3)(HSO4–).
Hydrocarbons 22.41

(b) The addition of HBr proceeds through the formation of a carbocation. The alkyl carbocation formed from the
+ +
alkene group (H3C—CHCH2C∫∫CH) is more stable than the vinyl carbocation (CH2==CHCH2C==CH2) from
the alkyne group. The DH ‡ for the former is lesser than that of the latter resulting in the addition to HBr
across the double bond, i.e. the alkene reacts at a faster rate than the alkyne towards electrophilic addition.
Thus, the reaction taking place is
H2C==CHCH2C CH + HBr H3CCHCH 2 C CH
|
Br
(c) The addition of Br2 across a double or triple bond occurs via the formation of the three membered ring of
bromonium ion.
CH == CH H2C CH2

Br Br
(from alkyne) (from alkene)

The brominium ion from alkyne is more strained due to the presence of double bond (shorter bond length).
Moreover, its carbon atoms have more s-character. These factors make this brominium ion less stable than that
from the alkene makaing alkynes less reactive than alkenes.
(d) The hydrogenation proceeds via the adsorption on the surface of the catalyst. The adsorption of alkene occurs
when the plane of the p-bond approaches perpendicular to the catalyst. The alkyne, any direction (parallel to
C—C bond) can lead to the adsorption due to the cylindrical nature of the p-bonds. Thus, the transition state
in case of alkynes is less constrained leading to the more positive value of DS ‡. This makes alkynes to react
at a faster rate than alkenes.
38. (a) With neutral conditions at room temperature, a diketone is formed via the hydroxylation reaction.

CH3CH2C C(CH2)2CH3 Æ CH3CH2C C(CH2)2CH3 Æ


OH OH

CH3CH2CH—C(CH2)2CH3 CH3CH2C—CH(CH2)2CH3
[O] CH3CH2C—C(CH2)2CH3
OH O + O OH Æ
O O

(b) Under neutral or alkaline conditions at higher temperatures, the triple bond is cleaved to give carboxylic acids.
[O]
CH3CH2C C(CH2)2CH3 Æ CH3CH2COOH + HOOC(CH2)2CH3

(c) The addition of HBr proceeds via the formation of carbocation.

CH2==CH—CH==CH—CH3 H + Æ CH — C H—CH==CH—CH
3 3
allylic carbocation

CH2==CH—CH2—CH==CH2 H + Æ CH — C H—CH —CH==CH


3 2 2
2° carbocation
Allylic carbocation is more stable due to resonance

CH3 CH CH == CH CH3 +
or CH3 CH CH CH CH3

CH3 CH == CH CH CH3

Its formation requires lesser DH‡ as compared to 2° carbocation and hence is formed at a faster rate.
22.42 Complete Chemistry—JEE Main

(d) The additions may be depicted as follows.


80 C H2C CH CH == CH2
Intermediate
complex 40 C H Br
1, 2-adduct
Br ,
80 C

H+
CH2 == CH CH == CH2 CH2 CH CH CH2
H
allylic carbocation

Br ,
40 C

Intermediate H2C CH == CH CH2


complex H Br
1, 4-adduct

The conversion allylic carbocation Æ 1,2-adduct involves a lower DH‡ as compared to carbocation Æ 1,4-adduct.
At low temperature, 1,2-adduct is formed due to the lower DH‡. Thus, it formation is kinetically-controlled. As
temperature is raised, 1,2-adduct passes over to the allylic carbocation and then to thermodynamically more
stable 1,4-adduct. The 1,4-adduct is more stable as it is a more substituted alkene. Once, 1,4-adduct is formed,
it cannot be converted to 1,2-adduct on lowering temperature as the conversion of 1,4-adduct to intermediate
complex involves a larger DH‡ value.
39. (c) Acetylide is a weaker base than ethide ion. A pair of electrons is more tightly bound in acetylene than in ethide
ion.
(d) The Lindlar catalyst is Pd/BaSO4
O
40. (b) CH3C∫∫CH 3
Æ CH3COOH + HCOOH
(c) The correct name is 1-penten-4-yne. C==C has priority over C∫∫C and gets the smaller number
(d) There are seven isomers.
1-hexyne, 2-hexyne, 3-hexyne, 4-methyl-1-pentyne, 4-methyl-2-pentyne, 3-methyl-1-pentyne,
3, 3-dimethyl-1-butyne

MULTIPLE CHOICE QUESTIONS FROM AIEEE AND JEE MAIN

1. Which of the following reactions will yield 2, 2-dibromopropane?


(a) CH3—C ∫∫ CH + 2HBr æÆ (b) CH3CH == CHBr + HBr æÆ
(c) CH ∫∫ CH + 2HBr æÆ (d) CH3—CH == CH2 + HBr æÆ [2007]
2. The hydrocarbon which can react with sodium in liquid ammonia is
(a) CH3CH2C∫∫CCH2CH3 (b) CH3CH2CH2C∫∫CCH2CH2CH3
(c) CH3CH2C∫∫CH (d) CH3CH∫∫CHCH3 [2008]
3. The treatment of CH3MgX with CH3C∫∫C—H produces
(a) CH4 (b) CH3CH==CH2 (c) CH3C∫∫CCH3 (d) CH3CH==CHCH3
[2008]
4. The major organic compound formed by the reaction of 1, 1, 1– trichloroethane with silver powder is
(a) 2-butene (b) acetylene (c) ethene (d) 2-butyne [2014]
Hydrocarbons 22.43

5. The reagent needed for converting


Ph H
Ph C C Ph C C
H Ph
is:
(a) Cat. Hydrogenation (b) H2/Lindlar Cat.
(c) Li/NH3 (d) LiAlH4 [2014, online]

ANSWERS
1. (a) 2. (c) 3. (a) 4. (d) 5. (c)

HINTS AND SOLUTIONS


Br
1. The reaction yielding 2, 2–dibromopropane is CH3—C CH + 2HBr Æ CH3 C CH3

Br
2. The compound CH3CH2C∫∫CH contains hydrogen atom attached to triply bonded sp hybridized carbon which
acquires acidic nature due to high electronegativity of carbon. This acetylinic hydrogen is replaced by Na+.
– + 1
CH3CH2C CH + Na Æ CH3CH2C C : Na + H2O.
2
3. The acetylinic hydrogen on reacting with CH3MgX produces the alkane CH4.

CH3C∫∫CH + CH3MgX Æ CH4 + CH3C∫∫CMgX


4. The reaction is
Cl Cl
CH3 C Cl + 6 Ag + Cl C CH3 CH3 C C CH3
Cl Cl 2-Butyne

5. The anti addition of H2 to an acetylinic compound occurs by the use of Li/NH3. The use of catalytic hydrogenation
causes syn. addition of hydrogen.

SECTION 4 Benzene

Structure of Benzene
According to Kekule, benzene is a resonance hybrid of the following resonating structures.

The resonance energy of benzene is about 150.6 kJ mol–1, i.e. benzene is more stable than cyclohexatriene by
150.6 kJ mol–1. The above structure of benzene accounts for the following.
22.44 Complete Chemistry—JEE Main

1. Molecular formula of benzene is C6H6.


2. Benzene yields only one monosubstitution product.
3. Benzene yields three isomeric disubstitution products.
4. All carbon-carbon bond distance in benzene are equal (139 pm) and are intermediate in length between single
(154 pm) and double bonds (134 pm).
5. Benzene undergoes substitution rather than addition reactions.
For example, the addition reaction of alkenes with cold dilute/alkaline KMnO4, Br2/CCl4 and HCl are not shown
by benzene. However, under drastic conditions benzene is reduced to cyclohexane. Benzene undergoes substitution
reactions in which H is replaced by another atom or group of atoms.
For examples
(i) C6H6 + HONO2 æææ Æ C6H5—NO2 + H2O
H 2SO 4

nitrobenzene

(ii) C6H6 + Cl2 æFeæÆ C6H5—Cl + HCl


chlorobenzene

(iii) C6H6 + HOSO3H æSO


æ3 Æ C6H5—SO3H + H2O
benzenesulphonic acid

(iv) C6H6 + RCl æææ


Æ C6H5—R + HCl
AlCl3

alkylbenzene

(v) C6H6 + CH3COCl æAlCl


ææ 3
Æ C6H5—COCH3 + HCl

Reaction (iv) is known as Friedel-Crafts alkylation and reaction (v) is known as Friedel-Crafts acylation.
6. The enthalpies of hydrogenation and combustion of benzene are lower than those expected for cyclohexatriene.

Orbital Representation of Benzene


Benzene is a planar molecule where each carbon is sp2 hybridized. Of the three hybrid orbitals, two are used in s-bonding
with two other carbon atoms and the third is used in s-bonding with hydrogen atom. In addition to these orbitals, each
carbon has one p orbital occupied by one electron. Since this orbital lies perpendicular to the plane of benzene ring,
the electron in this orbital is of p-type. The p orbital of each carbon atom can overlap with the adjacent p orbitals of
carbon atoms making additional bond of p-type. But this bond is not localized between two carbon atoms but form two
continuous doughtnut-shaped electrons cloud one lying above and the other below the plane of cyclic carbon skelton.
It is the overlap of the p orbitals in both directions that corresponds to the resonance hybrid of two Kekule structures.
The delocalization of p-electrons gives rise to resonance energy and makes the molecule more stable.
Substitution Reactions in Benzene
The typical reactions of the benzene ring are those in which the p-electrons serve as a source of electrons for electrophilic
(acidic) reagents. Because of delocalization of p-electrons, benzene does not show additional reactions as in the case
of alkenes but undergoes substitution reactions in which hydrogen atom attached to carbon atom is replaced by another
atom or group of atoms.

The Hückel 4n + 2 Rule


A molecule acquires aromatic characteristics provided it has cyclic clouds of delocalized p electrons above and below
the plane of the molecule and this p clouds has a total of (4n + 2) p electrons. The requirement of 4n + 2 electrons is
known as 4n + 2 rule or Hückle rule. Examples are benzene (n = 1), naphthalene (n = 2) and anthracene (n = 3). Besides

and also there is a reasonably good overlap for p-bond formation in these two rings. In smaller or bigger rings, it is

largely offset by angle strain or poor overlap of p orbitals or both.


Hydrocarbons 22.45

Mechanism of Electrophilic Substitution Reactions


Nitration
A mixture of nitric acid and sulphuric acid is the nitrating reagent. The steps involved in the mechanism are as follows.
(a) Formation of nitronium ion

HONO2 + 2H2SO4 H3O+ + 2HSO–4 + +NO2


(b) Electrophilic attack by nitronium ion

H NO2

+ +NO2 Æ + (slow)

benzenonium ion

(c) Removal of proton


NO2
H NO2

+ + HSO4– Æ + H2SO4 (fast)

The rate determining step is the formation of carbocation, which is the resonance hybrid of the following three structures.

H NO2 H NO2 H NO2 H NO2


H H
+ +
+
+

H
The dispersal of positive charge over the molecule by resonance makes this ion more stable than with a localized positive
charge. Probably because of this, a carbocation is formed from the relatively stable benzene.
Sulphonation
The involved steps are
(a) 2H2SO4 H3O+ + HSO4– + SO3

H SO3–
(b) SO3 + (slow)
+



SO3
H SO3

(c) + + HSO–4 + H2SO4 (fast)


SO3 SO3H

(d) + H 3O + + H 2O (equilibrium far to the left)


22.46 Complete Chemistry—JEE Main

In fact, the entire sulphonation process is an equilibrium process


H+
C6H6 + H2SO4 C6H5SO3H + H2O
For sulphonation, we require excess of H2SO4 along with SO3 and for desulphonation we use excess of H2O along with
heating (100 –175 °C).
Friedel-Crafts Alkylation
The steps are:
RCl + AlCl3 AlCl–4 + R+
H R
R+ + (slow)
+

R
H R
+ AlCl–4 Æ + HCl + AlCl3 (fast)
+

Here AlCl3 acts as a Lewis acid.


Mechanism of Halogenation
The steps are
Cl2 + FeCl3 Cl3 –Fe — +Cl — Cl
H Cl
– +
Cl3 Fe— Cl—Cl + Æ + FeCl–4 (slow)
+

H Cl Cl

+ FeCl–4 Æ + HCl + FeCl3 (fast)


+

Theory of Orientation

Activating Group
A group that releases electrons to benzene ring is an activating group. It directs the incoming electrophile to ortho or
para position. Examples include
Strongly activating: — NH2, —NHR, —NR2, —OH
Moderately activating: —OCH3, —NHCOCH3
Weakly activating: —CH3, —C6H5
Deactivating Group
A group that withdraws electrons from benzene is a deactivating group. It directs the incoming electrophile to meta
position. Examples include
— NO2, —CN, —COOH, —SO3H, —CHO, —COR and —N(CH3)3+
A group attached to benzene also affects the rate of electrophilic substitution reaction. An electron-releasing group
disperses the positive charge of carbocation and thus makes it more stable causing the reaction to proceed faster as
Hydrocarbons 22.47

positive charge of carbocation and thus makes it less stable causing the reaction to proceed slower as compared to the
substitution reaction involving benzene.
An activating group activates all the positions of a benzene ring; even the positions meta to it are more reactive than
any single position in the benzene ring itself. It directs ortho and para simply because it activates the ortho and
para positions much more than it does the meta. Alternatively, only in the ortho and para attack by an electrophile,
a carbocation is generated in which positive charge is located on the carbon atom to which electron-releasing group
is directly attached making this carbocation to be much more stable than the other resonating structures. Because of
contribution from the stable structure, the hybrid carbocations resulting from an attack at ortho and para positions are
more stable than the carbocation formed during the attack at meta position and since the stable hybrid carbocation is
formed faster, it follows that the electron-releasing or activating group is ortho and para directing. The above facts are
depicted in the following structures.
CH3 CH3 CH3

+
Para attack
+ +

H Y H Y H Y
stablest
CH3 CH3 CH3
H H H
+
Ortho attack Y Y Y
+ +

stablest
CH3 CH3 CH3

+ +
Meta attack H H H
+
Y Y Y
A deactivating group deactivates all positions in the ring, even the positions meta to it. It directs meta simply because
it deactivates the ortho and para positions even more than it does the meta.
Alternatively, only in the ortho and para attack by an electrophile a carbocation is generated in which positive charge is
located on the carbon atom to which electron-attracting group is directly attached making this carbocation to be much
more unstable than the other resonating structures. Because of contribution from the unstable structure, the hybrid
resulting from attack at ortho and para positions are more unstable than the carbocation formed during the attack at
meta position and since the stable carbocation is formed faster, it follows that the electron-attracting or deactivating
group is meta directing. The above facts are depicted in the following structures.
NO2 NO2 NO2

+
Para attack
+ +

H Y H Y H Y
unstable
NO2 NO2 NO2
H H +
H
Ortho attack Y Y Y
+ +

unstable
22.48 Complete Chemistry—JEE Main

NO2 NO2 NO2

+ +
Meta attack H H H
+
Y Y Y

no unstable structure

Activation of Benzene via Resonance


The groups —NH2, —OH, and —Cl can activate benzene ring. This is shown in the following structures.
+ + +
NH2 NH2 NH2 NH2

– –

+ + +
OH OH OH OH

– –

+ + +
Cl Cl Cl Cl

– –

For — NH2 and —OH, the resonance effect is much more important than the inductive effect. In the former benzene is
activated whereas in the latter, it is deactivated. In —Cl, both the resonance and inductive effects are evenly matched.
It is because of this that —Cl occupies the unusual position of being deactivating group but ortho and para Wdirector.
Effect of Activating and Deactivating Groups on Weak Organic Acids and Bases
(a) Benzoic acid If an electron-releasing group is present at the para position (where resonance effect outweighs
the inductive effect) of the — COOH group, the tendency to release proton from —COOH group is decreased making
such a substituted benzoic acid weaker than benzoic acid itself. For example, p-toluic acid, p-hydroxybenzoic acid
and p-anthranilic acid are weaker than benzoic acid. On the other hand, if such a group is present at the meta position
(where inductive effect outweights the resonance effect), the electron-withdrawing tendency of the group makes the
meta substituted acid stronger than benzoic acid.
The presence of electron-withdrawing group present at para position also makes the substituted benzoic acid more
strong than benzoic acid itself. For example, p-nitrobenzoic acid is a stronger acid than benzoic acid itself.
(b) Phenol Phenol is a weaker acid. This is because of the resonating structures in which oxygen atom acquires positive
charge making removal of proton from —OH group more easily.

H +
H +
H +
H H
O O O O O

– –


Hydrocarbons 22.49

The electron-withdrawing group present at the para or meta position makes oxygen atom more positive and thus helps
in releasing the proton and thus such a substituted phenol is more acidic than phenol itself. For example, p-nitrophenol
is more acidic than phenol. On the other hand, electron-releasing group at the para position (where resonance effect

example, p-cresol is less acidic than phenol.


Ortho meta and para isomers. Nearly all ortho
substituents exert an effect of the same kind—acid strengthening—whether they are electron-withdrawing or electron-
releasing, and the effect is a usually large. The ortho effect undoubtedly has to do with the nearness of the groups
involved, but is more than just steric hindrance arising from their bulk.
(c) Aniline Aniline is a weaker base. It is a Lewis base, i.e. the lone pair present on nitrogen can be given to the
Lewis acids. The electron-releasing group present at the para position increases the electron density on nitrogen and
thus makes it more basic. For example, para-toluidine is more basic than aniline. On the other hand, the presence of
electron-withdrawing group decreases the electron density on nitrogen and hence makes it less basic. For example,
p-nitroaniline is less basic than aniline.

Rules of Predicting Orientation in Distributed Benzenes


The following rules are followed.

1. If the groups reinforce CH3 2. In an ortho-para director and meta CH3


each other, there is no director are not reinforcing, the
problem. ortho-para director controls the
For example, orientation. The incoming group
CH3 goes mainly to ortho to the meta NO2
director. For example,
3. A strongly activating group, competing with a 4. When two weakly (or strongly) Cl
weakly activating group, controls the orientation. activating or deactivating groups
For example, compete, substantial amounts of
both isomers are obtained for ex- or
OH
OH ample,
CH3 Cl

CH3

5. Very little substitution CH3


occurs in the sterically very little
hindered position
between meta
substituents. CH3

Arenes (Aromatic-Aliphatic Hydrocarbons)


Arenes contain both aliphatic and aromatic units. Examples include ethylbenzene, toluene, xylenes and ethyl toluenes.
These compounds show two sets of chemical properties: aromatic ring shows electrophilic substitution and side
chain undergoes free radical substitution. Each portion of the molecule affects the reactivity of the other portion and
determines the orientation of attack.
Reactions shown by alkylbenzenes are electrophilic substitution on the ring and free radical substitution at the alkyl
group. For example,
22.50 Complete Chemistry—JEE Main

CH3 CH3
X
X2, FeX3
+
CH3 (X2 = Cl2, Br2)

X
CH2Cl CHCl2 CCl3
X
Cl2 Cl2 Cl2
heat or light heat or light heat or light

benzyl chloride benzal chloride benzo-trichloride

Alkenyl Benzenes
Alkenyl benzenes contain a double bond in the side chain of benzene ring. For example,

CH CH2; CH CH CH3;

styrene 1-phenylpropene
Alkenylbenzenes show both substitution in the ring and addition to the double bond in the side chain. The reactivity of
the double bond is greater than the benzene towards electrophilic reagents. Thus, mild conditions are required for the
addition reaction across the double bond as compared to those required for the benzene ring. For example,

ÈCarbocation ˘
Æ ÍÍ
Reactant ææ or ˙ ææ
˙ Æ Product
ÍÎ free radical ˙˚
The more stability of benzyl cation or radical
+ .
CHCH2CH3 or CHCH2CH3

relative to the conjugated alkenylbenzene makes the latter more reactive than simple alkenes. Also more stability of
+ + +
benzyl cation or radical relative to other possibilities, such as C6H5CH2C HCH3 and C 6H5CH2CH2C H2, determines the
Hydrocarbons 22.51

In the presence of peroxides, the reaction taking place is

MULTIPLE CHOICE QUESTIONS ON SECTION 4

Electrophilic Substitution
1. Which of the following substituents is ortho or para directing in the electrophilic substitution reactions in the
benzene ring?
(a) — NH2 (b) —NO2 (c) —CHO (d) —COOH
2. Which of the following substituents is meta directing in the electrophilic substitution reactions in the benzene
ring?
(a) — OCH3 (b) —CH3 (c) —CHO (d) —OH
3. An ortho-para directing group
(a) activates ortho and para positions and deactivates the meta position
(b) activates meta position and deactivates ortho and para positions
(c) activates ortho and para positions more than the meta position
(d) deactivates ortho and para positions less than the meta position
4. A meta directing group
(a) deactivates ortho and para positions more than the meta position
(b) activates ortho and para positions less than the meta position
(c) activates meta position and deactivates ortho and para positions
(d) deactive meta position and activates ortho and para positions
5. Among the following, the compound that can be readily sulphonated is
(a) benzene (b) toluene (c) nitrobenzene (d) benzaldehyde
6. The reaction of toluene with chlorine in the presence of ferric chloride gives predominantly
(a) benzyl chloride (b) benzoyl chloride (c) o- and p-chlorotoluene (d) m-chlorotoluene
7. Chlorination of toluene in the presence of light and heat followed by treatment with aqueous NaOH gives
(a) o-cresol (b) p-cresol (c) 2, 4-dihydroxytoluene (d) benzoic acid
8. Which of the following reagents is weakly activating group?
(a) — NH2 (b) —NR2 (c) —C6H5 (d) —NHCOCH3
9. In aniline, the —NH2 group
(a) activates the benzene ring via both inductive and resonance effects
(b) activates the benzene ring via resonance effect and deactivates via inductive effect
(c) activates the benzene ring via inductive effect and deactivates via resonance effect
(d) deactivates the benzene ring via both inductive and resonance effects
10. In the Friedel-Crafts acylation, the electrophile is
(a) C6H5+ (b) AlCl–3 (c) CH3CO+ (d) C6H5CH+2
11. In the nitrating mixture, HNO3 acts as
(a) base (b) acid (c) reducing agent (d) catalyst
12. Which of the following species is expected to yield maximum percentage of meta substitution product?
(a) ArCH3 (b) ArCH2Cl (c) ArCHCl2 (d) ArCCl3
22.52 Complete Chemistry—JEE Main

13. A direct iodination of benzene is not possible because


(a) iodine is an oxidizing agent (b) resulting C6H5I is reduced to C6H6 by HI
(c) HI is unstable (d) the ring gets deactivated
14. Which of the following statements is correct?
(a) The principal mononitration product of o-cresol is 2-methyl-4-nitrophenol.
(b) The carbon-hydrogen homolytic bond dissociation energy for benzene is considerable smaller than for
cyclohexane.
(c) The number of isomers of mononitration of o-dichlorobenzene is 4.
(d) A tribromobenzene gives two isomers when it is mononitrated. The compound is 1,2,4-tribromobenzene.
15. Which of the following statements is correct?
(a) Monobromination of p-toluenesulphonic acid followed by treatment with acid and superheated steam gives
m-bromotoluene.
O2N

(b) The mononitration of CH2 gives the product CH2

NO2 NO2
(c) The reactivity towards ring nitration of 2,4-dinitrotoluene is greater than that of m-dinitrobenzene.
(d) The reactivity towards ring nitration of 2,4-dinitrophenol is smaller than that of 2,4-dinitrochlorobenzene.
16. Which of the following statements is not correct?
(a) The order of reactivity towards ring nitration is p-xylene > toluene > p-toluic acid > terephthalic acid
(b) The order of reactivity towards ring nitration is Ph(CH2)2NO2 > PhCH2NO2 > PhNO2
(c) The order of reactivity towards electrophilic substitution is
C6H5CH3 < C6H5CH2COOC2H5 < C6H5CH(COOC2H5)2
(d) Each ring of biphenyl, C6H5 — C6H5, is more reactive than benzene towards electrophilic substitution, and the
chief products are ortho and para isomers.
17. Which of the following statements is not correct?
(a) In electrophilic aromatic substitution reaction, the formation of intermediate arenium cation is the rate-
determining step.
(b) The C—H bond in benzene is slightly stronger than the C—D bond in deuterated benzene (C6D6).
(c) The overall rate of an electrophilic substitution reaction, except sulphonation in benzene and deuterated
benzene, are identical.
(d) The sulphonation reaction involving benzene is a reversible reaction.
18. Which of the following statements is not correct?
(a) All electrophilic substitution reactions except sulphonation involving benzene are irreversible.
(b) The rates of sulphonation reactions involving benzene and deuterated benzene are slightly different.
(c) Iodine solution in benzene is brown due to the formation of charge-transfer complex between iodine and benzene.
(d) The reaction of benzene with n-propyl chloride in the presence of AlCl3 gives 1-phenylpropane as the only
product.
Acid–Base Strengthening
19. Which of the following order regarding the acidity of aromatic acids is correct?
(a) benzoic acid > p-toluic acid > p-hydroxybenzoic acid
(b) benzoic acid > p-toluic acid < p-hydroxybenzoic acid
(c) benzoic acid < p-toluic acid > p-hydroxybenzoic acid
(d) benzoic acid < p-toluic acid < p-hydroxybenzoic acid
20. Which of the following orders regarding the base strength of aromatic bases is correct?
(a) p-nitroaniline > aniline > p-toluidine (b) p-nitroaniline > aniline < p-toluidine
Hydrocarbons 22.53

(c) p-nitroniline < aniline > p-toluidine (d) p-nitroaniline < aniline < p-toluidine
21. Which of the following statements is not correct?
(a) Aniline is a weaker base than methylamine
(b) Phenol is a stronger acid than acetic acid
(c) p-Anthranilic acid is weaker acid than benzoic acid
(d) Phenol is a weaker acid than benzoic acid
22. Which of the following groups is acid-strengthening effect on benzoic acid?
+
(a) — OH group (b) — NH2 group (c) — NH3 group (d) — CH3 group
23. Which of the following groups is acid-weakening effect on benzoic acid?
+
(a) — OH group (b) — CHO group (c) — ON group (d) — NH3 group
Additional Problems
24. Which of the following carbocations is expected to be most stable?
H + H
(a) O2N + (b) O2N (c) O2N + (d) O2N +
Y Y

H Y H Y
25. The major product when 1 mol of H2 is added to C6H5CH==CHCH==CHCH3 using Pt catalyst is
(a) C6H5CH2CH==CHCH2CH3 (b) C6H5CH2CH2CH==CHCH3
(c) C6H5CH==CHCH2CH2CH3 (d) C6H5CH2CH2CH2CH==CH2
26. Which of the following statements is correct?
(a) Although benzene contains three double bonds, normally it does not undergo addition reaction.
(b) m-chlorobromobenzene is an isomer of m-bromochlorobenzene.
(c) In benzene, carbon uses all the three p orbitals for hybridization.
(d) An electron donating substitutent in benzene orients the incoming electrophilic group to the meta position.
27. Which of the following statements is correct?
(a) Benzene readily docolourises dilute KMnO4 solution.
(b) The inductive effect is the only factor which decides the orientation effects in the monosubstituted benzene.
(c) The number of isomers of aromatic compound tribromobenzene is four.
(d) The dipole moment of p-nitrotoluene is expected to be larger than p-chloronitrobenzene.
28. Which of the following statements is correct?
(a) The compound O2N NH2 is named 4-aminonitrobenzene.

(b) The compound H3CH2C CH2CH2CH2CH3 is named 1-ethyl-4-butylbenzene.

(c) Cyclopentadiene is much more acidic than 1,3-cyclohexadiene.


(d) The two phenyl groups in biphenyl lie in the same plane.
29. Which of the following statements is correct?
(a) The oxidation of naphthalene with KMnO4 in acid gives phthalic acid.
(b) The oxidation of naphthalene with O2/V2O5 gives 1, 4-naphthaquinone.
(c) Treatment of p-chlorotoluene with NaOH(aq) at 340 °C exclusively gives p-hydroxytoluene.
(d) Chlorine in p-chlorotoluene is replaced by OH when taken in NaOH(aq) at 340 °C. No such replacement occurs
for 2,6-dimethylchlorobenzene.
30. Which of the following structures is correct?
(a) The structure of the intermediate product, formed by the oxidation of toluene with CrO3 and acetic anhydride,
whose hydrolysis gives benzaldehyde, is C6H5CH2OCOCH3.
(b) The carbon-carbon bond length in benzene is 139 pm.
22.54 Complete Chemistry—JEE Main

(c) Cyclopentadienyl anion has unusually low stability.


(d) Cycloheptatrienyl cation is also known as tropylium ion.

ANSWERS

1. (a) 2. (c) 3. (c) 4. (a) 5. (b) 6. (c)


7. (d) 8. (c) 9. (b) 10. (c) 11. (a) 12. (d)
13. (b) 14. (a) 15. (c) 16. (b) 17. (b) 18. (d)
19. (a) 20. (d) 21. (b) 22. (c) 23. (a) 24. (d)
25. (c) 26. (a) 27. (d) 28. (c) 29. (d) 30. (b)

HINTS AND SOLUTIONS

1. —NH2 group is electron-releasing group and is thus ortho or para directing.


2. —CHO group is electron-attracting group and is thus meta directing.
3. An ortho-para directing group activates all positions of benzene, the ortho and para positions are activated more
than meta position.
4. A meta directing group deactivates all positions of benzene, the ortho and para positions are deactivated more
than meta position.
5. —CH3 group in toluene activates benzene ring, its sulphonation is easier than rest of the three molecules.
6. In the presence of ferric chloride, electrophilic substitution at ortho and para positions take place.
7. In the presence of light and heat, methyl group is chlorinated.
8. Phenyl group is weakly activating group.
9. —NH2 group activates benzene ring via resonance and deactivates via inductive effect.
10. The electrophile is CH3CO+.
11. HNO3 acts as a base. It accepts proton from H2SO4.
HNO3 + 2H2 SO4 H3O+ + 2HSO4– + NO+2
12. —CCl3 has a maximum electron attracting power.
13. C6H5I is reduced by HI to C6H6.

OH
CH3
14. (a) The product is

NO2
A strongly activating group, competing with a weakly activating group, controls the orientation. The position
6 will be sterically hindered.
(b) The C—H bond in benzene involves sp2–s character while in cyclohexane it is sp3–s. The bond C—H in
benzene is expected to be shorter and thus stronger than in cyclohexane.
(c) There are only two isomers.

Cl and O2N Cl

NO2 Cl Cl
Hydrocarbons 22.55

(d) 1,2,4-tribromobenzene gives three isomers.


Br Br Br
Br Br O2N Br
and
NO2 , O2N
Br Br Br

The compound 1, 2, 3-tribromobenzene gives two isomers.


Br Br
Br Br
and
Br O2N Br
NO2

15. (a) The product is o-bromotoluene.


CH3 CH3 CH3
Br Br
Br2 H2O, heat
⎯⎯→ ⎯⎯⎯⎯→
Fe H2SO4

SO3H SO3H

(b) Substitution is faster in the ring that is not deactivated by —NO2. Thus, the compounds obtained will be
NO2

CH2 CH2 NO2

O2N O2N

(c) In 2,4-dinitrotoluene, there are two deactivating (NO2) groups and one activating (CH3) group while in
m-dinitrobenzene, there are only two deactivating groups.
(d) 2,4-dinitrophenol contains a strongly activating group (—OH) while 2,4 dinitrochlorobenzene contains weak
deactivating group (—Cl).
16. (a) p-xylene contains two activating group, toluene contains one activating group, p-toluic acid contains one
activating group (CH3) and one deactivating group (COOH) and terephthalic acid contains two deactivating
groups (COOH).
(b) The farther away a deactivating group from the ring, the less effective it is .
(c) The more electron-withdrawing groups in benzene, the greater the deactivation.
(d) The positive charge of the intermediate can be dispersed by the second phenyl group when the latter is
attached to ortho or para, but not to meta.

H
Y
Intermediate
17. (a) The formation of intermediate arenium cation is slow while the elimination of H+ from this cation is fast. The
former involves the removal of aromaticity while the latter restores aromaticity.
22.56 Complete Chemistry—JEE Main

(b) The C—D bonds are stronger than C—H bonds. In fact, the bond with the heavier isotope is slightly stronger
than that with the lighter isotope.
(c) In electrophilic substitution reactions, the rate-determining step is the formation of arenium cation.
H E E
slow fast
+ H+
+ E+

In this step, E+ bonds to the aromatic ring without cleavage of a C—H bond. The latter is broken in the second
step, which is fast as it restores the stable aromatic system. Since C—H or C—D bond is not broken in the
rate-determining step, the overall rates of reaction are identical.
18. (b) In sulphonation reaction, both steps involving the formation of arenium cation and its conversion to the
substituted benzene determine the overall rate of reaction. In the second step, the loss of D+ from the deuterated
s complex is a little slower than the loss of H+ from the ordinary s complex.
(d) The product cantains both PhCH2 CH2 CH3 and PhCH(CH3)2.
19. Electron releasing group decreases the acidity of benzoic acid. Hydroxyl group is more powerful activator than
methyl group.
20. Electron releasing group increases the basicity of aniline while electron attracting group decreases the basicity.
21. Phenol is a weaker acid than acetic acid.
24. —NO2 group is benzene deactivator, it is meta directing.
25. Double bond attached to methyl group is reduced.
26. (a) Benzene gets stabilised because of resonance.
(b) Both represent one and the same compound.
(c) Each carbon in benzene is sp2 hybridized.
(d) An electron attracting group directs the incoming electrophilic group to the meta position.
27. (c) There are three isomers.
Br Br Br
Br Br
; and
Br Br Br
Br
1, 2, 3-tribromobenzene 1, 2, 4-tribromobenzene 1, 3, 5-tribromobenzene

(d) In p-nitrotoluene, both group moments act in the same direction while in p-chloronitrobenzene they act in the
opposite directions:

CH3 ΝΟ2 Cl ΝΟ2

28. (a) Amino group has priority over nitro; the correct name is 4-nitroaniline.
(b) Substituents should be listed in alphabetical order. The correct name is 1-butyl-4-ethylbenzene.
(c) The removal of H+ from cyclopentadiene gives the resonance stabilised aromatic cyclopentadienyl anion. This
is not so in case of 1,3-cyclohexadiene.
(d) The two phenyl groups are not in the same plane. They are joined by a single bond and have free rotation about
it.
29. (a) The product is 1, 4-naphthaquinone.
(b) The product is phthalic acid.
(c) Two products namely p-hydroxytoluene and m-hydroxytoluene are obtained. This reaction proceeds through
the formation of benzyne and is known as elimination-addition reaction
Hydrocarbons 22.57

CH3 CH3 CH3 CH3 CH3


OH OH
+
OH
Cl Cl OH
(d) There is no ortho-H, thus benzyne cannot form.
30. (a) The intermediate is C6H5CH(OCOCH3)2, benzylidene acetate
(c) Cyclopentadienyl anion has high stability

MULTIPLE CHOICE QUESTIONS FROM AIEEE AND JEE MAIN

1. The reaction of toluene with Cl2 in presence of FeCl3 gives predominantly:


(a) benzoyl chloride (b) benzyl chloride (c) o-and p-chlorotoluene (d) m-chlorotoluene
[2007]
2. Presence of a nitro group in a benzene ring
(a) activates the ring towards electrophilic substitution
(b) renders the ring basic.
(c) deactivates the ring towards nucleophilic substitution
(d) deactivates the ring towards electrophilic substitution [2007]
3. The electrophile E+ attacks the benzene ring to generate the intermediate s – complex. Of the following, which
s – complex is of lowest energy?
NO2
NO2 NO2
H
(a) (b) (c) (d) H
+ + E
+ H + E
E [2008]
H E
4. Toluene is nitrated and the resulting product is reduced with Sn and hydrochloric acid. The product so obtained
is diazotised and then heated with cuprous bromide. The reaction mixture so formed contains
(a) mixture of o- and m- bromotoluenes (b) mixture of o- and p- bromotoluenes
(c) mixture of o- and p- dibromobenzenes (d) mixture of o- and p- bromoanilines. [2008]
5. The nonaromatic compound along the following is

(a) (b) (c) (d) [2011]


S

6. Which compound exhibits maximum dipole moment among the following? [2015, online]
(a) NO2 (b) NO2 (c) NO2 (d) NO2
NH2

NH2
NH2
7. In the following sequence of reaction:
KMnO4 SOCl2 H2/Pd
Toluene A B C
BaSO4
The product C is
(a) C6H5COOH (b) C6H5CH3 (c) C6H5CH2OH (d) C6H5CHO [2015]
22.58 Complete Chemistry—JEE Main

ANSWERS
1. (c) 2. (d) 3. (c) 4. (b) 5. (a) 6. (b)
7. (d)

HINTS AND SOLUTIONS


1. Toluene undergoes electrophilic substitution reaction in ortho and para positions, since the methyl group attached
to benzene, being ring activator, is ortho and para directing group.
2. Nitro group is a electron-withdrawing group, thereby it deactives the ring towards electrophilic substitution of
benzene ring.
3. The nitro group is electron withdrawal group. This decreases the tendency of electrophilic substitution reaction.
The intermediate s – complex of nitrobenzene has larger energy as compared to that of benzene.
4. The given reactions are as follows.
CH3 CH3 CH3
NO2 Sn/HCl
1. HNO3
+
2. H2SO4

NO2
CH3 CH3 CH3 CH3
Br 1. Na NO2 / HCl (0 – 5°C) NH2
+ +
2. CuBr

Br NH2

5. An aromatic compound has (4n + 2) p-electons, where n is an integer. Cyclopentadiene has only 4p electrons and
thus will be nonaromatic.
NO2 NO2 NO2 NO2
6.
NH2

NH2
NH2
The group moments of NO2 and NH2 act in the same direction, their vector addition will give maximum value.
7. The reaction are:
CH3 COOH COCl CHO
KMnO4 SOCl2 H2/Pd
BaSO4

SECTION 5 Sources of Hydrocarbons

Coal and petroleum are the two main natural sources of hydrocarbons. The main source of aromatic hydrocarbons is
coal while that of aliphatic hydrocarbons is petroleum.
Hydrocarbons 22.59

The fractional distillation of petroleum provides different fractions in different temperature ranges containing different
chain lengths of carbon atoms. Table 1 describes the composition and uses of some of the fractions.
Table 1 Composition and uses of some of the fractions of petroleum

Fraction Boiling point Composition Uses


range T/K
Gaseous 110–300 C1 to C5 As a fuel and in the manufacture of hydrogen and
hydrocarbons ammonia
Petroleum ether or naphtha 300–360 C5 to C7 As a solvent and in drycleaning
Gasoline (petrol) 340–470 C1 to C12 As fuel in motor-cars, etc.
Kerosene 450–550 C12 to C15 As fuel and for illumination
Diesel oil (gas oil or fuel oil) 520–670 C15 to C18 As fuel in trucks, buses, diesel engines, etc. fuel in
electricity generation.
Lubricating oils, greases, above 520 C16 and more For lubricating of machines
petroleum jelly
m. pt 325–330 C20 and more
canvas etc.
Petroleum coke (residue) m.pt above 330 As fuel for making electrodes and road-building.

Petrol is the most important fraction of petroleum. It may also be produced by cracking process in which larger
molecules are broken into smaller ones. This cracking may be carried out thermally or catalytically. For example,
heating lubricating oil or diesel oil at a high temperature produces petrol of low octane rating. In catalytic cracking,
silica and alumina are used. This cracking produces petrol of higher octane rating.
Coal on pyrolysis (heating to a high temperature in the absence of air) produces coke. The volatile matter in coal
volatilizes. It is passed through sulphuric acid to absorb basic compounds such as ammonia. The solution of the salt

petrol. A foul-smelling, black, sticky liquid settles down at the bottom, and is known as coal tar. The latter on distillation
produces aromatic hydrocarbons. The major fraction of coal tar distillation is shown in Table 2.
Table 2 Fractions of coal tar
Name of fraction Range of boiling point T/K Approximate percentage Main Compounds
Crude light oil up to 440 2.2 benzene, toluene, xylene
Middle oil or carbolic oil 440–550 7.5 phenol
Heavy oil or creosote oil 500–540 16.5 cresols
Green oil or anthracene oil 540–630 20.0 naphthalene, anthracene, phenanthrene
Pitch residue 56.0 carbon and other compounds

Octane Rating
The quality of petrol is judged from its knocking behaviour in an engine. It is compared with two standards, namely,
n-heptane and isooctane. The former burns explosively and has been given an octane rating of zero. Isooctane (2, 2,

the mixture with n-heptane which will have the same performance as the sample of petrol.
Cetane Number
In diesel engine, the straight-chain hydrocarbons generates more power than branched-chain ones. Cetane which is
n-hexadecane ignites spontaneously and has been given a rating of 100. Zero rating is given to a-methylnaphthalene
which ignites slowly in the diesel engines. The cetane number of diesel oil is the percentage of cetane in the mixture of
cetane and a-methylnaphthalene which has the same ignition qualities as the sample of the diesel.
22.60 Complete Chemistry—JEE Main

Gasoline Additive
The octane rating of a fuel is improved by adding tetraethyl lead to it. To avoid the more wear and tear of cylinder and
piston of the engine, ethylene bromide is also added to the petrol.

MULTIPLE CHOICE QUESTIONS ON SECTION 5

1. The constituent obtained in the temperature range 340 K to 470 K during the fractional distillation of petroleum
is
(a) gaseous hydrocarbons (b) petroleum ether (c) gasoline (d) kerosene
2. The constituent obtained in the temperature range 450 K to 550 K during the fractional distillation of petroleum
is
(a) gaseous hydrocarbons (b) petroleum ether (c) gasoline (d) kerosene
3. The constituent obtained in the temperature range 520 to 670 K during the fractional distillation of petroleum is
(a) gasoline (b) kerosene (c) diesel oil (d) lubricating oil
4. The range of number of carbon atoms in the gasoline fraction is
(a) C1 to C7 (b) C7 to C12 (c) C12 to C15 (d) C15 to C18
5. The range of number of carbon atoms in the kerosene fraction is
(a) C1 to C7 (b) C7 to C12 (c) C12 to C15 (d) C15 to C18
6. The range of number of carbon atoms in the diesel oil is
(a) C1 to C7 (b) C7 to C12 (c) C12 to C15 (d) C15 to C18
7. The range of number of carbon atoms in the lubricating oils is
(a) C7 to C12 (b) C12 to C15 (c) C15 to C18 (d) C18 and more
8. Octane number is zero for
(a) n-heptane (b) 2, 3, 3-trimethylpentane
(c) n-octane (d) 2, 2, 4-trimethylpentane
9. Octane number is 100 for
(a) n-pentane (b) 2, 3, 3-trimethylpentane
(c) n-octane (d) 2, 2, 4-trimethylpentane
10. Which of the following is not true?
(a) Petrol containing a larger amount of straight-chain hydrocarbons produces less knocking
(b) Petrol containing a larger amount of branched-chain hydrocarbons produces less knocking
(c) Octane rating is zero for n-heptane
(d) Octane rating is 100 for isooctane
11. Which of the following is used as gasoline additive?
(a) n-Heptane (b) Isooctane (c) Tetraethyl lead (d) Diethyl lead
12. Dry distillation of coal gives many valuable and important products. The solid residue left behind after dry
distillation of coal is called
(a) asphalt (b) coal pitch (c) coal tar (d) coke
13. Which of the following statements about petroleum is not correct?
(a) Petroleum is a dark coloured oily liquid with a characteristic smell
(b) Petroleum is heavier than water
(c) Petroleum is insoluble in water
(d) Petroleum is formed by decomposition of animal and plant remains inside the earth
Hydrocarbons 22.61

14. Which of the following gaseous fuels is obtained by dry distillation of coal?
(a) Coal gas (b) Oil gas (c) Producer gas (d) Water gas

ANSWERS
1. (c) 2. (d) 3. (c) 4. (b) 5. (c) 6. (d)
7. (d) 8. (a) 9. (d) 10. (a) 11. (c) 12. (d)
13. (b) 14. (a)
23
Organic Compounds
Containing Halogens
(Haloalkanes and Haloarenes)

SECTION 1 Haloalkanes

Methods of Preparation
Replacement of Hydroxyl Group in Alcohols
The reagents used are:
For primary and secondary alcohol—hydrogen chloride in the presence of zinc chloride.
For tertiary alcohol—concentrated hydrochloric acid in the presence of zinc chloride.
The reaction proceeds as follows.
SN1 mechanism;
ROH + ZnCl2 Æ R O ZnCl2 Æ R+ + [HO—ZnCl2]– HClÆ RCl + H2O + ZnCl2
H
SN2 mechanism;
ROH + ZnCl2 + HCl Æ ZnCl3– + ROH +2 Æ ZnCl2 + RCl + H2O
Tertiary alcohol follows SN1 mechanism.
Straight chain primary alcohols favour SN2 mechanism.
Secondary alcohol may react via both the mechanisms.
For alkyl bromides, constant boiling hydrobromic acid (48%) is used. For primary alcohols, a little concentrated
sulphuric acid is used as a catalyst. For secondary and tertiary alcohols, sulphuric acid is not used as these alcohols tend
to dehydrate to give alkenes. HBr can be generated in situ by the action of concentrated sulphuric acid on potassium
bromide.
For alkyl iodides, constant boiling HI(57%) is used. It may be generated in situ by the action of 95% phosphoric acid
on KI.
The convenient reagents to replace — OH by —Cl are phosphorus chloride and thionyl chloride (SOCl2). The latter
is preferred as the end products are gaseous products which escape in air giving almost pure alkyl chloride.
Addition of Halogen Acids to an Alkene
The addition occurs in accordance with Markovnikov’s rule. HBr in the presence of peroxide adds according to anti-
Markovnikov’s rule.

Physical Properties
The C —X bond in alkyl halides is a polar bond. The dipole moment of methyl halides follows the order:
CH3Cl > CH3F > CH3Br > CH3I
The bond strength of C—X decreases with increasing size of halogen atom, i.e.
23.2 Complete Chemistry—JEE Main

e(C —F) > e(C—Cl) > e(C—Br) > e(C—I)


The boiling points of alkyl halides are higher than those of the corresponding alkanes. The boiling point of alkyl
halides increases with increasing size of the halogen atom.
Alkyl chlorides are lighter than water while alkyl bromides and iodides are heavier than water.

Chemical Reactions
The alkyl halides undergo nucleophilic substitution reactions in which a stronger necleophile replaces a weaker
nucleophile. Some alkyl halide can undergo elimination reactions producing an alkene.
Nucleophilic substitution reactions A few typical nucleophilic substitution reactions are as follows.
1. CH 3— Cl + OH – D
Æ CH3OH + Cl–
from aq
KOH or Ag 2O
in boiling water

2. CH3CH2—I + CH3O– Æ CH3CH2OCH3 + I–

3. CH3CH -- CH 2 I + CN - Æ CH3CHCH 2 -- CN + I-
| From alcoholic
solution of |
CH3 KCN CH3
When silver cyanide is used, the product is isocyanide.
CH3CHCH 2 -- I + AgCN Æ CH3CHCH 2 -- N ∫∫ C + Ag I
| |
CH3 CH3
4. CH3CH2I + :NH3 Æ CH3CH2NH+3 + I–
5. RX + KNO2 Æ R —O—N==O + RX
alkyl nitrite
If silver nitrite is used, the product is mainly nitroalkane.
O
R — X + Ag—O—N==O Æ R N + AgX
O
The nucleophilic substitution reaction proceeds via either SN1 or SN2 mechanism.
SN1 Mechanism
CH3
H3 C Slow Fast CH3
... C Br C+ .. C Nu
H3 C .. –Br H3C ...
H 3C CH3
CH3 CH3
Main characteristics
1. The more stable the carbocation intermediate, the faster the SN1 mechanism.
2. The nucleophilie plays no role in the mechanism.
3. The weaker bases are the better leaving groups and thus favour SN1 mechanism.
4. Polar solvents accelerate the SN1 reaction because it favours the formation of polar transition state.
5. The SN1 reaction on a chiral starting material ends up with the recemization of the product.
SN2 Mechanism
H
H – – H
Nu– C X Nu.....C.....X Nu C
..... ..... H + X–
.

H
.
...

H H H
H
Organic Compounds Containing Halogens (Haloalkanes and Haloarenes) 23.3

Main Characteristics
1. The conversion of reactants to transition state is the rate determining step.
2. The SN2 reaction involves the inversion of stereochemistry around carbon atom of the substrate. This inversion is
known as Walden inversion.
3. The rate of reaction depends on the steric bulk of the alkyl group.
The kinetic study have shown that the methyl halides are the most reactive in SN2 reactions. The increase in the
length of chain of alkyl group decreases the rate of reaction. Alkyl branching next to the leaving group decreases
the rate drastically. The reactivity order for SN2 reactions follows the following order.
CH3 > 1° > 2° >> neopentyl > 3°
4. Aprotic solvents increase the rate of SN2 reactions.
5. The replacement of halide group follows the order I– > Br– > Cl–.
Elimination Reaction
In the presence of alcoholic KOH and heating, the elimination reaction occurs which results into a double bond. If more
than one product is possible, the major product is of more substituted alkene (Saytzeff rule).
alcoholic
CH3 CH2 CH CH3 CH3CH CHCH3 + CH3 CH2CH CH2
KOH, 80% 20%
Br

Competition Between Substitution and Elimination Reactions


The relative proportion of products depends on mainly three factors, namely, basicity of the nucleophile, hindrance in
the haloalkane, and steric bulk around the nucleophilic atom.
Factor 1. Base strength of the nucleophile
Weak bases (H2O, ROH, halides, RS–, N–3, NC–, RCOO–) lead to more substitution.
Strong bases (HO–, RO–, H2N–, R2N–) lead to more elimination.
Factor 2. Steric hindrane around the reacting carbon
Sterically unhindered (primary) haloalkanes lead to more substitution.
Sterically hindered (branched primary, secondary, tertiary) haloalkanes lead to more elimination.
Factor 3. Steric hindrane in the nucleophile
Sterically unhindered (OH–, CH3O–, CH3CH2O–, H2N–) nucleophile lead to more substitution.
Sterically hindered (CH3)3CO–, [(CH3)2CH2NH–] nucleophiles lead to more elimination.
Formation Grignard Reagent
Haloalkanes react with magnesium turnings in dry ether to form alkyl magnesium halide, known as Grignard reagent.
The latter is most versatile compound as it can be used in the preparation of many different types of compounds.

RH
H2O or dilute acid
ROH
dry oxygen
ROH
HCHO
RX + Mg RMgX RCHOHR
ether R CHO R
R C(OH)
R COR R
RCHO
Ethyl formate
RCN
Cyanogen
Cyanides can also be prepared by treating alkyl halides with alcoholic solution of potassium cyanide. These, in turn,
can be converted to amide, carboxylic acid and amine.
23.4 Complete Chemistry—JEE Main

Conc. HCl
R -- C ∫∫ N + H 2 O æoræææææ
H O -NaOH
Æ RCONH 2
2 2

Mineral acid
R -- C ∫∫ N + H 2 O æææææ
or alkali
Æ RCOOH + NH3

Na/C H OH
R -- C ∫∫ N + 4H æorææææææ 2 5
H / Ni or LiA1H
Æ RCH 2 NH 2
2 4

MULTIPLE CHOICE QUESTION FROM AIEEE AND JEE MAIN

1. The major product formed when 1, 1, 1 - trichloro - propane is treated with aqueous potassium hydroxide is:
(a) Propyne (b) 1- Propanol (c) 2- Propanol (d) Propionic acid [2004]
2. Which of the following reagents is not suitable for the elimination reaction? [2016, online]
Br

(a) NaI (b) NaOEt/EtOH (c) NaOH/H2O (d) NaOH/H2O-EtOH


3. 2-Chloro-2-methylpentane on reaction with sodium methoxide yields
CH3

(i) C2H5CH2—C—OCH3 (ii) C2H5CH2—C — CH2 (iii) C2H5CH — C—CH3



CH3 CH3 CH3


(a) (i) and (iii) (b) (iii) only (c) (i) and (ii) (d) all of these [2016]

ANSWER

1. (d) 2. (a) 3. (d)

HINT AND SOLUTION


-H O
1. CH3CH 2 CCl3 + 3KOH Æ [CH3CH 2 C (OH )3 ]+ 3KCl æææÆ CH3CH 2 COOH
2

Propionic acid
2. The reagent NaI cannot be used.

3. Strong base like CH3O– leads to more elimination.


Sterically unhindred base like CH3O– leads to more substitution.
Sterically hindrance around the reacting carbon leads to more elimination.
In elimination, major product is of more substituted alkene (Saytzeff rule).
In the given reaction CH3O– is unhindered whereas reacting carbon of 2-chloro-2-methylpentane is sterically
hindered and thus all the three given compounds will be formed.
Organic Compounds Containing Halogens (Haloalkanes and Haloarenes) 23.5

SECTION 2 Haloarenes

Methods of Preparation
These can be prepared by Friedel-Crafts halogenation. Iodination cannot be carried out by this method as the reaction
is endothermic. Aryl halides can also be prepared by Sandmeyer reaction. In this reaction, benzenediazonium chloride
is treated with cuprous chloride or cuprous bromide dissolved in the corresponding acid.

+
N NCl– Cl

CuCl/HCl
+ N2
60°C

Iodobenzene is obtained by warming diazonium salt solution with aq. KI solution.


+
N N Cl– I

+ KI + N2 + KCl
warm

Fluorination is carried out by warming diazonium salt solution with HBF4. This reaction is known as Schiemann
reaction.
+
N NCl– +
N NBF4– F

+ N2 + BF3
HBF4 120°C

Chemical Reactions
In general, aryl halides are less reactive than alkyl halides towards nucleophilic substitution reactions. This is due to
the resonance effect in which lone pair of electron on halogen atom is delocalized to benzene ring imparting a partial
double bond character to C—X bond.

+ + +
X X X X X

In alkyl halide, the C—X bond involves sp3(C) whereas in aryl halide, sp2(C) is involved. Since the sp2(C) is more
electronegative than the sp3(C), the C — X bond in aryl halide is shorter than in alkyl halides. This makes C—X bond
more strong in aryl halides.
Under normal conditions, halobenzenes are inert to nucleophiles. However, chlorobenzene can be made to react if
the experimental conditions are drastic, i.e. high temperature and high pressure.
23.6 Complete Chemistry—JEE Main

H+
O–Na+ OH
Cl aq. NaOH, 300 °C

NH2
aq. NH3, 200 °C
Cu2O

CuCN in
CN
Dimethylformamide

The presence of electron-withdrawing substituent at ortho and/or para positions is a favourable factor for the
nucleophilic substitution reaction. More such substituents, the faster the reaction.

Cl OH Cl OH
NO2 NO2
15% NaOH aq. NaOH
;
160 °C 100 °C

NO2 NO2 NO2 NO2


Cl OH
O 2N NO2 O 2N NO2
aq. NaOH
Warm

NO2 NO2

The factors responsible for the nucleophilic substitution reaction is due to (1) decrease in electron density in the
ring, thereby favouring the nucleophilic attack, and (2) the stabilisation of intermediate anion by resonance. No such
resonance is possible if the electron-withdrawing substituent occupies meta position.
Wurtz-Fittig Reaction
The combination of aryl halide with alkyl halide in an ethereal solution in the presence of sodium to give alkyl
substituted benzene is known as Wurtz-Fittig reaction

Ethereal
Cl + 2Na + ClCH3 CH3 + 2NaCl
solution

Fittig Reaction
Treatment of ethereal solution of bromobenzene with sodium gives biphenyl.

Br + 2Na + Br + 2NaBr

Chloro substituent in chlorobenzene is ortho and para director, since resonance effect of Cl predominates over its
inductive effect.
Organic Compounds Containing Halogens (Haloalkanes and Haloarenes) 23.7

MULTIPLE CHOICE QUESTIONS FOR THE ENTIRE CHAPTER

General Characteristics
1. Which of the following represents sec-butyl bromide? CH3
CH3 CH3CH2CHCH3
(a) CH3CH2CH2CH2Br (b) CHCH2Br (c) CH3 C Br (d)
CH3 Br
CH3
2. Which of the following represents tert-butyl bromide?
CH3
CH3CHCH2Br CH3CH2CHCH3
(a) CH3CH2CH2CH2Br (b) (c) CH3 C Br (d)
CH3 Br
CH3
3. Which of the following represents isobutyl bromide?
CH3
CH3 CHCH2Br
(a) CH3CH2CH2CH2Br (b) (c) CH3 C Br (d) CH3CH2CHCH3
CH3
CH3
4. Which of the following order regarding bond enthalpy e(C—X) in an alkyl halide (RX) is correct?
(a) e(C —Cl) < e(C—Br) < e(C— I) (b) e(C— Cl) > e(C—Br) > e(C— I)
(c) e(C— Cl) > e(C—Br) < e(C— I) (d) e(C— Cl) < e(C—Br) > e(C— I)
5. The ease with which an alcohol can be halogenated by treating it with HX in the presence of anhydrous ZnCl2 is
(a) tertiary alcohol < secondary alcohol < primary alcohol
(b) tertiary alcohol > secondary alcohol > primary alcohol
(c) tertiary alcohol > secondary alcohol < primary alcohol
(d) tertiary alcohol < secondary alcohol > primary alcohol
6. When toluene is treated with Cl2 at low temperature in the presence of a catalyst (FeCl3), the product obtained is
(a) only o-chlorotoluene (b) only m-chlorotoluene
(c) only p-chlorotoluene (d) a mixture of ortho-and para-chlorotoluenes
7. When toluene is treated with Cl2 at high temperature without the use of any catalyst, the product obtain is
(a) o-chlorotoluene (b) m-chlorotoluene
(c) a mixture of orhto- and para-chlorotoluenes (d) benzylchloride
8. For the same alkyl group, the order of boiling points of alkyl halides is
(a) chloride < bromide < iodide (b)chloride > bromide > iodide
(c) chloride < bromide > iodide (d)chloride > bromide < iodide

(a) CH4 (b) CH3Cl (c) CH2 = CHCl (d) CCl4


10. Chloroform is stored in dark coloured bottles because it is oxidized in the presence of light and air to a poisonous
compound
(a) CO (b) COCl2 (c) CO2 (d) CCl4
11. Which of the following is expected to have high density?
(a) CH3F (b) CH3Br (c) CH3Cl (d) CH3I
12. Which of the following is used as an antiseptic?
(a) CH3I (b) CH2I2 (c) CHI3 (d) CHCl3
23.8 Complete Chemistry—JEE Main

13. Which of the following compounds represents gammexane?


(a) Hexachlorobenzene (b) Benzene hexachloride (c) Dichlorobenzene (d) Trichlorobenzene
14. The dipole moment of CH3X (where X is a halogen) follows the order
(a) CH3F > CH3Cl > CH3Br (b) CH3F > CH3Cl < CH3Br
(c) CH3F < CH3Cl > CH3Br (d) CH3F < CH3Cl < CH3Br
15. Which of the following orders regarding the boiling points of the given alkyl chloride is correct?
(a) CH3(CH2)3Cl > CH3CH2CHClCH3 > (CH3)3CCl (b) CH3(CH2)3Cl < CH3CH2CHClCH3 < (CH3)3CCl
(c) CH3(CH2)3Cl > CH3CH2CHClCH3 < (CH3)3CCl (d) CH3(CH3)3Cl < CH3CH2CHClCH3 < (CH3)3CCl
16. The number of isomers of dibromobutane is
(a) 4 (b) 5 (c) 6 (d) 8
17. Chlorination of benzene proceeds via
(a) nucleophilic substitution mechanism (b) electrophilic substitution mechanism
(c) elimination-addition mechanism (d) addition-elimination mechanism
18. Which of the following reactions is highly exothermic?
(a) Fluorination of benzene (b) Chlorination of benzene
(c) Bromination of benzene (d) Iodination of benzene
19. Which of the following reactions is an endothermic reaction?
(a) Fluorination of benzene (b) Chlorination of benzene
(c) Bromination of benzene (d) Iodination of benzene
20. Which of the following is an allylic halide?
(a) CH3CH2X (b) CH2==CH—CH2X (c) CH2==CHX (d) C6H5Cl
21. Which of the following is a vinylic halide?
(a) CH3CH2X (b) CH2==CH—CH2X (c) CH2==CHX (d) C6H5Cl
22. Carbylamine test is performed in alcoholic KOH by heating a mixture of
(a) chloroform with silver powder (b) trihalogenated methane and a primary amine
(c) an alkyl halide and primary amine (d) an alkyl cyanide and a primary amine
23. Haloform test is shown by the compound
(a) methanol (b) methanal (c) 1-propanol (d) 2-propanol
24. Treating iodoform with silver powder gives
(a) ethylene (b) acetylene (c) methane (d) diiodomethane
25. Amongst the following, a refrigerant is
(a) CHCl3 (b) CH2F2 (c) CCl2F2 (d) CCl4
26. Heating bleaching powder with ethanol gives
(a) acetone (b) methanol (c) chloroform (d) acetic acid
Nucleophilic Substitution Reactions
27. Which of the following order regarding SN2 reaction involving replacement of halogen atom in alkyl halide by a
nucleophile is correct?
(a) CH3 > 1° < 2° (b) CH3 < 1° > 2° (c) CH3 > 1° > 2° (d) CH3 < 1° < 2°
28. In the reaction

ROH + HX 
anhydrous
 
 RX + H2O
ZnCl 2

the order of reactivity of halogen acids is


(a) HI > HBr > HCl (b) HI < HBr < HCl (c) HI > HBr < HCl (d) HI < HBr > HCl
29. Which of the following compounds gives formic acid when it is hydrolysed with boiling aqueous alkali?
(a) CH3Cl (b) CH2Cl2 (c) CH3CH2Cl (d) CHCl3
Organic Compounds Containing Halogens (Haloalkanes and Haloarenes) 23.9

30. Which of the following is an example of Sandmeyer reaction?


Cl
Cl
Cl Cl
uv light
(a) FeCl 3 (b) + Cl2 Æ
+ Cl2 Æ high temperature
low temperature
Cl Cl
Cl
CH3 N N Cl Cl
CH2Cl
light, high temp. CuCl, heat
(c) + Cl2 Æ (d) Æ + N2
no catalyst

31. The major product with 2-boromobutane is treated with ethanolic KOH is
(a) 2-hydroxybutane (b) butanol (c) 1-butene (d) 2-butene
32. Which of the following reagents is most effective in replacing —OH group of an alcohol by —Cl group?
(a) HCl gas (b) Concentrated HCl (c) SOCl (d) SOCl2
33. The order of reactivity for the SN2 reaction involving replacement of — X by a nucleophile in alkyl halide is
(a) CH3 < 1° < 2° (b) CH3 > 1° > 2° (c) CH3 < 1° > 2° (d) CH3 > 1° < 2°
34. Which of the following statements is correct?
(a) Alkyl halides are more reactive than aryl halides towards nucleophilic substitution reactions
(b) Alkyl halides are less reactive than aryl halides towards nucleophilic substitution reactions
(c) The presence of an electron-withdrawing substituent at ortho and/or para position decreases the reactivity of
nucleophilic substitution of chlorine in the substituted chlorobenzene
(d) The replacement of chlorine in chlorobenzene by strong bases proceeds via elimination-addition reaction
35. The Wurtz-Fittig reactions involves
(a) two molecules of aryl halides
(b) two molecules of alkyl halides
(c) one molecule of each of aryl halide and alkyl halide
(d) one molecule of each of aryl halide and phenol
36. Which of the following compounds undergoes replacement of —Cl by —OH by merely warming the compound
with aqueous NaOH?
Cl Cl Cl Cl
NO2 NO2 O2N NO2
(a) (b) (c) (d)

NO2 NO2
37. Which of the following has the highest nucleophilicity?
(a) F– (b) OH– (c) CH–3 (d) NH–2
38. The order of reactivities of the following alkyl halides for a SN2 reaction is
(a) RF > RCl > RBr > RI (b) RF > RBr > RCl > RI
(c) RCl > RBr > RF > RI (d) RI > RBr > RCl > RF
39. Which of the following orders about nucleophilicity is correct?
(a) MeO– > HO– > MeCOO– (b) MeO– < HO– < MeCOO–
– – –
(c) MeO > HO < MeCOO (d) MeO– < HO– > MeCOO–
40. Which of the following order about nucleophilicity is not correct?
(a) HO– > H2O (b) I– > Cl– (c) HS– < HO– (d) HOO– > HO–
23.10 Complete Chemistry—JEE Main

41. Chlorine reacts with benzaldehyde to give


(a) benzyl chloride (b) benzal chloride (c) benzoyl chloride (d) chlorobenzene
42. DDT is prepared from
(a) benzene and C6H5Cl (b) C6H5Cl and chlorine (c) Chloral and C6H5Cl (d) Chloral and benzene
43. The correct sequence of the alkyl halides towards SN2 reactions is
(a) CH3Br > CH3Cl > CH3F (b) CH3Br > CH3Cl < CH3F
(c) CH3Br < CH3Cl > CH3F (d) CH3Br < CH3Cl < CH3F
44. Which of the following factors does not favour SN1 elimination in alkyl chloride?
(a) Stability of R+ (b) Steric hindrance (c) Protic solvent (d) Aprotic solvent
45. Which of the following statements is not correct for SN1 reactions?

(b) The nucliophile plays no role in the mechanism


(c) The rate determining step is the conversion of transition state to the product.
(d) The rate of reaction depends on the steric bulk of the alkyl group.
46. Which of the following statements is not correct for SN 2 reactions?
(a) The conversion of reactant to transition state is the rate determining step.
(b) The rate depends on the concentration and nature of nucleophile.
(c) Protic solvents increase the rate of reaction.
(d) The weaker the leaving base, faster the rate of replacement.
47. Which of the following factors does not favour elimination reaction than substitution reaction.... in an alkyl halide?
(a) Low polarity of solvent (b) Strong base
(c) High concentration of base (d) Sterically unhindered haloalkene.
Aryl Halides
48. The chlorine atom in chlorobenzene is ortho and para director because
(a) resonance effect predominates over inductive effect
(b) inductive effect predominates over resonance effect
(c) both inductive and resonance effects are evenly matched
(d) only resonance effect and not inductive effect is operating
49. Aryl halides are less reactive towards nucleophilic substitution reactions as compared to alkyl halides due to
(a) the formation of less stable carbonium ion (b) resonance stabilization
(c) longer carbon-halogen bond (d) inductive effect
50. Aryl halides are less reactive towards nucleophilic substitution as compared to alkyl halides. Which of the
following factors is not responsible for this fact?
(a) the formation of more stable carbonium ion (b) resonance stabilization
(c) the inductive effect (d) sp2-hybridized carbon attached to the halogen
51. Which of the following reactions is highly exothermic?
(a) Fluorniation of benzene (b) Chloroniation of benzene
(c) Bromination of benzene (d) Iodination of benzene
52. In the reaction of p-chlorotoluene with KNH2 in liquor ammonia, the major product is
(a) o-toluidine (b) m-toluidine (c) p-toluidine (d) p-chloroaniline
53. The chlorine atom in chlorobenzene is ortho and para director because
(a) resonance effect predominates over inductive effect
(b) inductive effect predominates over resonance effect
(c) both inductive and resonance effects are evenly matched
(d) only resonance effect and not inductive effect is operating
Organic Compounds Containing Halogens (Haloalkanes and Haloarenes) 23.11

ANSWERS
1. (b) 2. (c) 3. (b) 4. (b) 5. (b) 6. (d)
7. (d) 8. (a) 9. (d) 10. (b) 11. (d) 12. (c)
13. (b) 14. (c) 15. (a) 16. (c) 17. (b) 18. (a)
19. (d) 20. (b) 21. (c) 22. (b) 23. (d) 24. (b)
25. (c) 26. (c) 27. (c) 28. (a) 29. (d) 30. (d)
31. (d) 32. (d) 33. (b) 34. (a) 35. (c) 36. (d)
37. (c) 38. (d) 39. (a) 40. (c) 41. (c) 42. (c)
43. (a) 44. (d) 45. (c) 46. (c) 47. (d) 48. (a)
49. (b) 50. (c) 51. (a) 52. (b) 53. (a)

HINTS AND SOLUTIONS


4. Bond enthalpy C—X (where X is Cl, Br, I) decreases with increase in atomic number of X.
5. The decreasing order of reactivity is 3° > 2° > 1 > MeOH.
6. At low temperature in the presence of a catalyst (FeCl3), electrophilic substitution at benzene ring takes place.
7. At high temperature, substitution at methyl group takes place.
8. The boiling point of alkyl halides increases with increasing size of halogen atom.
9. CCl4 is known as pyrene.
10. CHCl3 is oxidized to COCl2.
11. Higher the size of halogen, higher the density.
12. HCI3 is an antiseptic
13. Benzene hexachloride is known as gammexane.
14. The dipole moment order is CH3Cl >CH3F > CH3Br
15. Boiling point decreases with increasing branching of the alkyl group.
16. The skeletons of molecule are
C C C CBr ; C C C CBr ; C C C C Br ; C C C CBr ;
Br Br Br Br
Br
C C C C ; C C C C
Br Br Br
17. Chlorination of benzene proceeds via electrophilic substitution mechanism.
18. Fluorination is highly exothermic reaction.
19. Iodination is endothermic reaction.
27. Replacement of halogen atom is alkyl halide by SN2 decreases with increase in steric hindrance.
28. The stronger the acid, higher the reactivity.
CHCl3 ææÆ CH(OH)3 æææ Æ HCOOH
H2O
29.
31. The major product is 2-butene (CH3CH==CHCH3).
32. The most effective reagent is SOCl2.
33. The rate decreases with increasing bulk of alkyl group.
34. Alkyl halides are more reactive than aryl halides.
35. Wurtz-Fitting reaction involves one molecule of each of aryl halide and aryl halide.
23.12 Complete Chemistry—JEE Main

36. The reactivity of Cl increases with increasing number of —NO2 group.


37. In going from left to right across the periodic table, both basicity and nucleophilicity decrease.
38. The reactivity increases with increasing size of halogen.
39. When the nucleophilic and basic sites are the same atom, nucleophilicity parallels basicity.
40. In going down a group in a period table, nucleophilicity increases.
44. Stability of R+, steric hindrance and protic solvent favour SN1 reactions.
45. (a) The rate expression is r = k [haloalkane]
(b) Nucleophile is involved in the fast reaction.
(c) Substrate Æ Transition state is the slow step.
(d) Steric bulk favours SN1 mechanism.
Æ Transition state is the slow step.
(b) The reaction follows second-order kinetics; rate = k[haloalkane][nucleophile]
(c) The weaker the base, faster the replacement.
47. Low polarity of solvent, strong base and high concentration of base favour elimination reaction.
48. Chlorine exhibits resonance effect more predominately than inductive effect.
49. Aryl halides are less reactive due to resonance stabilization.
50. The lesser reactivity of aryl halide is due to resonance stabilization and sp2-hybridized carbon attached to the
halogen.
52. The reaction proceeds via elimination-addition mechanism. The major product is m-toluidine.

MULTIPLE CHOICE QUESTIONS FROM AIEEE AND JEE MAIN

1. The compound formed on heating chlorobenzene with chloral in the presence of concentrated sulphuric acid is
(a) gammexene (b) DDT (c) freon (d) hexachloroethane
[2004]
2. Tertiary alkyl halides are practically inert to substitution by SN2 mechanism because of
(a) inductive effect (b) steric hindrance (c) insolubility (d) instability [2005]
3. Fluorobenzene (C6H5F) can be synthesized in the laboratory
(a) by reacting bromobenzene with NaF solution
(b) by heating phenol with HF and KF
(c) from aniline by diazotisation followed by heating the diazonium salt with HBF4
2 gas [2006]
4. The decreasing order of the rate of the reaction CH3Br + Nu Æ CH3—Nu + Br with nucleophiles (Nu ) A to D
– – –

[Nu– = (A) PhO–, (B) AcO–, (C) HO–, (D) CH3O–] is


(a) B > D > C > A (b) D > C > A > B (c) D > C > B > A (d) A > B > C > D [2006]
5. The structure of the major product formed in the following reaction
CH2Cl
NaCN
DMF
I
is
CH2CN CH2Cl CH2Cl
(a) (b) (c) (d)
CN
CN I CN
[2006]
Organic Compounds Containing Halogens (Haloalkanes and Haloarenes) 23.13

6. Which of the following is the correct order of decreasing SN2 reactivity?


(a) RCH2X > R3CX > R2CHX (b) RCH2X > R2CHX > R3CX
(c) R3CX > R2CHX > RCH2X (d) R2CHX > R3CX > RCH2X
(X = a halogen) [2007]
7. Compound (A) C8H9Br, gives a white precipitate when warmed with alcoholic AgNO3. Oxidation of (A) gives an
acid (B), C8H6O4. (B) easily forms anhydride on heating. Indentify the compound (A).

[2013]
8. In SN2 reactions, the correct order of reactivity for the following compounds: CH3Cl, CH3CH2Cl, (CH3)2CHCl
and (CH3)3CCl is
(a) (CH3)2CHCl > CH3CH2Cl > CH3Cl > (CH3)3CCl
(b) CH3Cl > (CH3)2CHCl > CH3CH2Cl > (CH3)3CCl
(c) CH3Cl > CH3CH2Cl > (CH3)2CHCl > (CH3)3CCl
(d) CH3CH2Cl > CH3Cl > (CH3)2CHCl > (CH3)3CCl [2014]
9. Chlorobenzne reacts with trichloro acetaldehyde in the presence of H2SO4
O
Cl H2SO4
2 + H C CCl3

The major product formed is:


Cl Cl
(a) Cl C Cl (b) Cl C Cl
Cl CH2Cl

(c) Cl CH Cl (d) Cl CH Cl [2014, online]


CCl3 Cl
10. For the compounds
CH3Cl, CH3Br, CH3I and CH3F, the correct order of increasing C- halogen bond length is:
(a) CH3F < CH3Cl < CH3Br < CH3I (b) CH3F < CH3Br < CH3Cl < CH3I
(c) CH3F < CH3I < CH3Br < CH3Cl (d) CH3Cl < CH3Br < CH3F < CH3I [2014, online]
- DMF -
11. In a nucleophilic substitution reaction: R - Br + Cl æææÆ R - Cl + Br , which one of the following undergoes

(a) C6H5CHC6H5Br (b) C6H5CH2Br


(c) C6H5CHCH3Br (d) C6H5CCH3C6H5Br [2014, online]
[2015]

(c) Finkelstein reaction (d) Swarts reaction


23.14 Complete Chemistry—JEE Main

ANSWERS
1. (b) 2. (b) 3. (c) 4. (b) 5. (a) 6. (b)
7. (d) 8. (b) 9. (c) 10. (a) 11. (b) 12. (d)

HINTS AND SOLUTIONS


1. The reaction is

2. The reactivity order for SN2 mechanism decreases with increase in the bulkiness of alkyl group. The order is
CH3 > 1° > 2° >> neopentyl > 3°
4. The order acid-strength of the conjugate acids of the given bases is CH3OH < H2O < PhOH < AcOH
The base-strength of the given bases will be CH3O– > OH– > PhO– > AcO–
(D) (C) (A) (B)

The stronger the base, stronger the nucleophile, greater the rate of nucleophilic displacement.
6. Bulkier the alkyl group, lesser the SN2 reactivity.
7. Aliphatic Br gives precipitate with AgNO3.The Oxidation of —CH2Br gives —COOH and also —CH3 gives
—COOH. The two —COOH group must be in ortho positions to give anhydride. Hence,

8. Increasing bulkiness of alkyl group decreases the SN2 reaction. Thus, the correct choise is
CH3Cl > CH3CH2Cl > (CH3)2CHCl > (CH3)3CCl

9.

10. As the size of halogen atom increases, the bond length also increases in the same order i.e.
C - F < C - Cl < C - Br < C - I
N2 mechanism. The reactivity order is
CH3 > 1° > 2° neopentyl > 3°
C6H5CH2Br contains 1° C to which Br is attached and will undergo SN2 mechanism with complete inversion of

3, SbF3, AgF, Hg2F2, etc.


For example, C2H5Cl + AgF Æ C2H5F + AgCl
24
Organic Compounds
Containing Oxygen (Alcohols,
Phenols, Ethers, Aldehydes,
Ketones, Carboxylic Acids and
their Derivatives)
SECTION 1 Alcohols

Methods of Preparation
Oxymercuration-Demercuration Process
Alkenes react with mercury acetate in the presence of water to give hydroxymercurial compounds which on reduction
with sodium borohydride (NaBH4) yield alcohol. The net result is the addition of H2O to the double bond and the product
obtained is in agreement with Markovnikov’s rule. For example,
NaBH4
CH3CH CH2 + H2O + Hg(OAc)2 Æ CH3 CH CH2 CH3 CH CH3
OH HgOAc OH
No rearrangement occurs in this reaction.
Hydroboration-Oxidation Process
Alkenes undergo hydroboration with diborane to yield alkylboranes which on oxidation with alkaline H2O2 give
alcohols. The net result is the addition of H2O to the double bond and the product obtained is in accordance with the
anti-Markovnikov’s rule. For example,
H2O2, OH–
CH3CH CH2 + BH3 Æ CH3CH2CH2BH2 CH3CH2CH2OH
n-propyl alcohol

No rearrangement occurs during the hydroboration.


Grignard Synthesis of Alcohols
Reaction of aldehyde or ketone with RMgX produces alcohol.

H H
H – + H2O
C O + R MgX Æ H C OMgX Æ H C OH
H
R R
1° alcohol
24.2 Complete Chemistry—JEE Main

H H
H – + H2O
C O + R MgX Æ R¢ C OMgX Æ R¢ C OH

R R
2° alcohol
R≤ R≤
R≤ – + H2O
C O + R MgX Æ R¢ C OMgX Æ R¢ C OH

R R
3° alcohol

Reaction of Ethylene Oxide with RMgX

– + H2O
CH2 CH2 + RMgX Æ RCH2CH2OMgX Æ RCH2CH2OH

Reactions of Lithium Acetylides or Alkynyl Grignard Reagents with Aldehyde or Ketone

R R R
H+
HC CLi + O C R¢ Æ HC C C R¢ Æ HC C C R¢
OLi OH
R R R
H+
HC CMgBr + O C R¢ Æ HC C C R¢ Æ HC C C R¢
OMgBr OH

Reduction of Aldehydes and Ketones


Sodium borohydride, NaBH4, is the convenient reagent to carry out the reduction of aldehydes or ketones into alcohol.

1.NaBH 4 in alcohol
RCHO Æ RCH2OH
2.H 3 O
LiAlH4 in ether can also be used for reduction. It is particularly useful for reducing a, b-unsaturated ketones. The
reduction of such a ketone gives a mixture of both unsaturated and saturated alcohols. For example,

HO H HO H
1. NaBH4, alcohol
+
2. H3O+
O
2-cyclohexenol cyclohexanol
(59%) (41%)

2-cyclohexenone HO H HO H
1. LiAlH4, ether
+
2. H3O+
94% 2%
Organic Compounds Containing Oxygen (Alcohols, Phenols, Ethers, Aldehydes, Ketones, Carboxylic Acids and their Derivatives) 24.3

Physical Properties
Due to the hydrogen bonding, alcohols have higher boiling points than their structural isomers. The boiling point
decreases with increase in branching, i.e. the order of boiling point is primary > secondary > tertiary.

Chemical Properties

namely, R OH bond cleavage and RO H bond cleavage.

Reactions Exhibiting R···OH Bond Cleavage


Reaction with Hydrogen Halides
R—OH + HX Æ RX + H2O
As such —OH is a poor leaving group. But its protonation converts into a good leaving group. There is formation of
carbocation as the intermediate and thus the reaction may show rearrangement. The following are the reactivities of
HX and ROH.
HI > HBr > HCl; allyl, benzyl > 3° > 2° > 1°
The reagents, used are concentrated HBr or NaBr + concentrated H2SO4, HCl + ZnCl2 and concentrated HCl.
Dehydration

CH3 — CH2 — CH2— CH2OH H


Æ CH3CH==CHCH 3 + CH3CH2CH ==CH2
2-butene 1-butene
(main product)

The reaction may show rearrangement.


Reactivity of ROH: 3° > 2° > 1°
The primary alcohol is dehydrated using concentrated H2SO4. The secondary and tertiary alcohols are dehydrated
using dilue H2SO4 to avoid the polymerization of alkenes. Saytzeff’s rules is applicable so as to get more substituted
alkene.
Reaction with Phosphorous Trihalides

3R— OH + PX3 Æ 3R— X + H3PO3


(PX3 PBr3, PI3)

Reactions Exhibiting RO···H Bond Cleavage


Reaction with Active Metals
1
RO— H + M Æ R— OM+ + H2
(M=Na, K, Mg, Al, etc.) 2

Reactivity of alcohol CH3OH > 1° > 2° > 3°


The above reaction shows alcohol as an acid. It is worth comparing the acid strength of alcohol with other species.

H2O > ROH > HC CH > NH3 > RH

The relative order of basicity follows the reverse order, i.e.


OH– < OR– < HC C– < NH–2 < R–
24.4 Complete Chemistry—JEE Main

Reaction with Carboxylic Acid


This results in the formation of an ester.
O O
H+
CH3 C OH + H OC2 H5 CH3 C OC2H5 + H2O
ethyl acetate

Instead of acetic acid, acetic anhydride or acetyl chloride can also be used.
Oxidation of Alcohols
The oxidation of an alcohol involves the loss of one or more a-hydrogens. 1° alcohol is changed to an aldehyde by
using the reagent pyridinium chlorochromate (C5H5NH+CrO3Cl–) formed by the reaction between chromic acid and
pyridinium chloride. For example,
H
C 5 H 5 NH CrO 3 Cl
CH3CH2CH2OH Æ CH3CH2 C O
propanal
1-propanol

1° alcohol is directly converted into a carboxylic acid by the use of potassium permangnate. For example,
O
KMnO 4
CH3CH2CH2OH Æ CH3CH2 C OH
1-propanol propanoic acid

2° alcohol is changed into a ketone by the use of potassium dichromate or CrO3 in glacial acetic acid or CrO3 in pyridine.
For example,
K 2 Cr2 O 7
CH3CHOH Æ CH3C O
CH3 CH3
isopropyl alcohol acetone

3° alcohol is not oxidizable as it does not contain a-hydrogen.

Characterization of Alcohols
Oxidation Method

However, drastic oxidation will convert ketone (formed from 2° alcohol) and 3° alcohol into carboxylic acid
containing fewer carbon atoms.
When the vapours of alcohols are passed over hot metallic Cu at 570 K, limited oxidation takes place.
Organic Compounds Containing Oxygen (Alcohols, Phenols, Ethers, Aldehydes, Ketones, Carboxylic Acids and their Derivatives) 24.5

Victor-Meyer Method
The alcohols are treated with P4 and I2 to form an alkyliodide. This on treatment with AgNO2 yields nitroalkanes. On
treating the latter with aqueous NaNO2
P+12 AgNO 2 HNO2
1° alcohol CH3CH2OH Æ CH3CH2 I Æ CH3CH2NO2 Æ CH3 C NO2
NOH
nitrolic acid
(Blood red colour)

P+12 AgNO 2 HNO2


2° alcohol (CH3)2CHOH Æ (CH3)2CHI Æ (CH3)2CHNO2 Æ (CH3)2CNO2
NO
pseudo nitrol
(Blue colour)

P+12 AgNO 2 HNO2


3° alcohol (CH3)3 COH Æ (CH3)3CI Æ (CH3)3CNO2 Æ no reaction

Lucas Test
Lucas reagent is a mixture of concentrated hydrochloric acid and zinc chloride. It converts alcohol into chloride and
the formation of the latter is indicated by the appearance of cloudiness which marks the separation of chloride from
the solution.

react appreciably at room temperature.


Allyl alcohol reacts as rapidly as tertiary alcohols but it remains in the solution.
Characteristic Test of CH3CO— Group

An alcohol of the type R CH OH is oxidized to R C CH3 which gives iodoform test.


|
|

||
CH3 O
The reagent used is iodine and sodium hydroxide (sodium hypoiodite, NaOI).
The reactions involved are
H
R C CH3 + NaOI R C CH3 + NaI + H2O
OH O
R C CH3 + 3NaOI R C CI3 + 3NaOH
O O
R C CI3 + NaOH RCOO–Na+ + CHI3
O Iodoform
(Yellow ppt.)

Analysis of Molecules Containing —OH or ==O Group Attached to Adjacent Carbon Atoms
Molecules containing —OH or ==O groups attached to adjacent carbon atoms undergo oxidation with cleavage of
carbon-carbon bonds when treated with periodic acid. For example,
HIO4
RCH CH R¢ Æ RCH OH + R¢CH OH Æ RCHO + R¢COH
OH OH OH OH
24.6 Complete Chemistry—JEE Main

HIO4
RCH C R¢ Æ RCH OH + HO CR¢ Æ RCHO + R¢COOH
OH O OH O
2HIO4
R CH CH CH R¢ Æ RCHO + HCOOH + R¢CHO
OH OH OH
HIO4
R C C R¢ Æ RCOOH + R¢COOH
O O
The amount of HIO4 consumed is equal to the amount of carbon-carbon bond broken in the molecule.

MULTIPLE CHOICE QUESTIONS ON SECTION 1

Preparation and Physical Properties


1. Which of the following is not characteristic of alcohols?
(a) The boiling point increases with increasing molecular mass
(b) They are lighter than water
(c) Lower members have a pleasant smell and burning taste, higher members are odourless and tasteless
(d) Lower members are insoluble in water and organic solvents but the solubility regularly increases with molecular
mass
2. Oxymercuration-demercuration of CH3CH == CH2 produces
(a) CH3CH2CH2OH (b) CH3CH(OH)CH3 (c) CH3CH(OH)CH2OH (d) CH3COCH3
3. Hydroboration-oxidation of CH3CH = CH2 produces
(a) CH3CH2CH2OH (b) CH3CH(OH)CH3 (c) CH3CH(OH)CH2OH (d) CH3COCH3
4. Which of the following order regarding acidity of the following compounds is correct?
(a) ROH > HC ∫∫ CH > NH3 (b) ROH < HC ∫∫ CH < NH3
(c) ROH > HC ∫∫ CH < NH3 (d) ROH < HC ∫∫ CH > NH3
5. The reaction of CH3CHO with RMgX produces
(a) 1° alcohol (b) 2° alcohol (c) 3° alcohol (d) a carboxylic acid
6. The reaction CH3COCH3 with RMgX produces
(a) 1° alcohol (b) 2° alcohol (c) 3° alcohol (d) a carboxylic acid
7. An industrial method of preparation of methanol is
(a) catalytic reduction of carbon monoxide in presence of ZnO-Cr2O3.
(b) by reacting methane with steam at 900 °C with a nickel catalyst
(c) by reducing formaldehyde with lithium aluminium hydride
(d) by reacting formaldehyde with aqueous sodium hydroxide solution
8. Hydrogen bonding is maximum in
(a) ethanol (b) diethylether (c) ethyl chloride (d) triethylamine
9. Which of the following alcohols is expected to have minimum boiling point?
(a) 1-Butanol (b) 2-Butanol (c) 2-Methyl-2-propanol (d) 1-Pentanol
10. Glycerine is a/an
(a) secondary alcohol (b) tertiary alcohol (c) ester (d) trihydric alcohol
11. Power alcohol is
(a) an alcohol of 95% purity (b) a mixture of petrol, ethanol and benzene
Organic Compounds Containing Oxygen (Alcohols, Phenols, Ethers, Aldehydes, Ketones, Carboxylic Acids and their Derivatives) 24.7

12. The enzyme which converts glucose to ethanol is


(a) invertase (b) zymase (c) maltase (d) diastase
13. Which of the following statements is correct?
(a) The branched isomer of an alcohol has lower boiling point than the unbranched alcohol.
(b) Ethylene glycol boils at a temperature lower than that of ethanol.
(c) The hydroboration-oxidation process gives product corresponding to Markovnikov addition of water to the
carbon-carbon double bond.
(d) The oxymercuration-demercuration process gives products corresponding to anti-Markovnikov addition of
water to the carbon-carbon double bond.
14. Which of the following statements is not correct?
(a) The addition of water to the carbon-carbon double bond via hydroboration-oxidation process does not involve
any rearrangement of carbon skeleton.
(b) The rearrangement of carbon skeleton may occur during the conversion of alcohol into alkene.
(c) The rearrangement of carbon skeleton may occur during the conversion of alcohol into alkyl halide.
(d) The cleavage of carbon-oxygen bond in alcohols is not catalyzed in the presence of an acid.
15. Which of the following statements is not correct?
(a) The substitution of hydroxyl group by a halogen group in alcohol is an electrophilic substitution reaction.
(b) Alcohols are weak acids as well as weak bases.
(c) A secondary alcohol on oxidation gives a carboxylic acid containing the lesser number of carbon atoms.
(d) A primary alcohol on oxidation gives a carboxylic acid containing the same number of carbon atoms.
16. Which of the following statements is correct?
(a) Absolute alcohol can be obtained by distillation of ethanol and water mixture.
(b) Cyclohexanol is more soluble in water than 1-hexanol.
(c) The hydration of 3-phenyl-1-butene in dilute H2SO4 produces 3-phenyl-2-butanol.
(d) The hydration of cyclobutylethene in dilute H2SO4 gives 1-cyclobutylethanol.
Cleavage of R...OH Bond
17. The ease of dehydration of alcohols is
(a) tertiary > secondary > primary (b) tertiary < secondary < primary
(c) tertiary > secondary < primary (d) tertiary < secondary > primary
18. HBr reacts faster with
(a) 2-methylpropan-2-ol (b) propan-1-ol (c) propan-2-ol (d) 2-methylpropan-1-ol
19. Dehydration of 2-butanol gives
(a) but-1-ene (b) but-2-ene (c) propene (d) 2-methylbut-2-ene
20. The order of reactivity of dehydration of alcohol is
(a) 1° > 2° > 3° (b) 1° > 2° < 3° (c) 1° < 2° > 3° (d) 1° < 2° < 3°
21. The order of reactivity of hydrogen halides with an alcohol is
(a) HCl > HBr > HI (b) HCl < HBr < HI (c) HCl > HBr < HI (d) HCl < HBr > HI
22. The iodoform test is not shown by the compound
(a) CH3COCH3 (b) CH3COCH2CH3 (c) CH3CH2COCH2CH3 (d) (CH3)2CHOH
23. The compound which gives the most stable carbonium ion on dehydration is
CH3
|
(a) CH3 CHCH 2 OH (b) CH3 — C— OH (c) CH3CH2CH2CH2OH (d) CH3CHCH 2 CH3
| | |
CH 3 CH3 OH
24.8 Complete Chemistry—JEE Main

24. The order of reactivity of alcohols towards hydrogen halide is


(a) benzyl > 3° > 2° > 1° (b) benzyl < 3° < 2° < 1° (c) 3° > 2° > 1° > benzyl (d) 3° > 2° > benzyl > 1°
25. The dehydration of 1-butanol gives
(a) 1-butene as the main product (b) 2-butene as the main product
(c) equal amounts of 1-butene and 2-butene (d) 2-methylpropene
26. Reaction of tertiary butyl alcohol with hot Cu at 350°C produces
(a) butanol (b) butanal (c) 2-butene (d) 2-methylpropene
27. The reaction of ROH with R¢MgX produces
(a) RH (b) R¢H (c) R — R (d) R¢ — R¢
28. The product obtained when a limited amount of HI reacts with glycerol is
(a) 1, 2, 3-Triiodopropane (b) Allyl iodide (c) 1, 2-Diiodopropane (d) Isopropyl iodide
29. The product obtained when excess of HI reacts with glycerol is
(a) 1, 2, 3-Triiodopropane (b) Allyl iodide (c) 1, 2-Diiodopropane (d) Isopropyl iodide
30. HBr reacts fastest with
(a) 2-methylbutan-2-ol (b) butan-1-ol (c) propan-2-ol (d) 2-methylpropan-1-ol
Cleavage of RO...H Bond
31. The order of reactivity of alcohols towards Na or K metal is
(a) primary > secondar > tertiary (b) primary < secondary < tertiary
(c) primary < secondary > tertiary (d) primary > secondary < tertiary
32. Which of the following compounds is oxidised to prepare methyl ethyl ketone?
(a) 2-Propanol (b) 1-Butanol (c) 2-Butanol (d) tert-Butyl alcohol
33. The relative order of acidity of alcohols in comparison to H2O and HC∫∫CH is
(a) H2O > ROH > HC∫∫CH (b) H2O > HC∫∫CH > ROH
(c) ROH > H2O > HC∫∫CH (d) ROH > HCOOH > H2O
34. The relative order of basicity of conjugate bases is
(a) OH– < OR– < HC∫∫C– (b) OH– < HC∫∫C– < OR–
– – –
(c) HC∫∫C < OH < OR (d) HC∫∫C– < OR– < OH–
35. When vapours of 2-propanol is passed over hot metallic Cu at 570 K, the product formed is
(a) 1-propanol (b) propanone (c) propanal (d) propene
36. Among the following compounds, the strongest acid is
(a) HC∫∫CH (b) C6H6 (c) C2H6 (d) CH3OH
37. A mixture of methanol vapour and air is passed over heated copper. The products are
(a) CO + H2O (b) HCHO + H2 (c) HCOOH (d) CO + H2O

(a) CH3OH > CH3CH2OH > (CH3)2CHOH (b) CH3OH > (CH3)2CHOH > CH3CH2OH
(c) CH3CH2OH > (CH3)2CHOH > CH3OH (d) (CH3)2CHOH > CH3CH2OH > CH3OH
39. Which of the following statements is correct?

(a) The reaction ROH + OH– 

 RO + HOH lies far towards the left side.
(b) The reaction ROH + R¢S–Na+ æÆ RO– Na+ + R¢SH is feasible.
(c) RS–, in a protic solvent, acts as a stronger nucleophile than RO–.
(d) The bond angle R—O—H in methanol is smaller than that of R—S—H in methanethiol.
Characteristic Tests
40. In the Victor-Meyer test, blue colouration is shown by
(a) primary alcohol (b) secondary alcohol (c) tertiary alcohol (d) diol
Organic Compounds Containing Oxygen (Alcohols, Phenols, Ethers, Aldehydes, Ketones, Carboxylic Acids and their Derivatives) 24.9

41. Which of the following alcohols is most reactive with Lucas reagent?
(a) Methanol (b) Ethanol (c) Isopropyl alcohol (d) tert-Butyl alcohol
42. Lucas reagent is
(a) anhydrous AlCl3 with conc. HCl (b) anhydrous ZnCl2 with conc. H2SO4
(c) anhydrous ZnCl2 with conc. HCl (d) anhydrous CaCl2 with conc. HCl
43. The compound that does not react with Lucas reagent is
(a) n–butanol (b) sec-butyl alcohol (c) isobutyl alcohol (d) tert-butyl alcohol
44. The treatment of a compound with HIO4 produces CH3CHO and CH3CH2CHO. The probable compound is
(a) CH3CH == CHCH2CH3 (b) CH3CH(OH)CH(OH)CH2CH3
(c) CH3CH == CHCH = CH2 (d) CH3CH(OH) CH2CHO
45. A molecule consumes 2 molecules of HIO4. The number of carbon-carbon bond broken by this acid is
(a) one (b) two (c) three (d) none
46. A primary alcohol can be oxidized to an aldehyde by choosing the reagent
(a) KMnO4 (b) K2Cr2O7 (c) C5H5 NH+ CrO3 Cl– (d) O3
47. The compound which reacts fastest with Lucas reagent at room temperature is
(a) 1-butanol (b) 2-butanol (c) 2-methylpropanol (d) 2-methylpropan-2-ol
48. In the Victor-Meyer test, red colouration is shown by
(a) 1° alcohol (b) 2° alcohol (c) 3° alcohol (d) phenol
49. In the Lucas test of alcohols, appearance of cloudiness is due to the formation of
(a) aldehydes (b) ketones (c) acid chlorides (d) alkyl chlorides
50. An alcohol on oxidation gives CH3COOH and CH3CH2COOH. The alcohol is
(a) CH3CH2CH2OH (b) CH3CH(OH)CH2CH3
(c) (CH3)2C(OH)CH2CH3 (d) CH3CH(OH)CH2CH2CH3
51. Which of the following statements is correct?
(a) Tertiary butyl alcohol gives positive iodoform test.
H
(b) CH3CH2 C CH2CH3 gives positive iodoform test.
OH
H H
(c) The carbon-carbon bond in R C C R can be broken by the use of periodic acid and the product obtained
are two aldehydes.
OH OH
R H
(d) The carbon-carbon bond in R C C R can be broken by the use of periodic acid giving two aldehydes.
OH OH
52. Which of the following statements is not correct?
(a) The molecule RCHCH2CHR is cleaved by HIO4 giving RCHO and R¢CHO.
OH OH
(b) Tertiary alcohols are more readily dehydrated than the secondary alcohols.
(c) Tertiary butyl alcohol when passed over hot metallic Cu at 570 K produces isobutene.
(d) Primary alcohols do not show positive Lucas test.
53. Which of the following statements is correct?
(a) Tertiary alcohols show positive Lucas test with faster speed than in the case of secondary alcohols.
(b) The order of increasing acidity amongst 1°, 2° and 3° alcohols is 1° alcohol < 2° alcohol < 3° alcohol
24.10 Complete Chemistry—JEE Main

(c) The reaction of glycerol with small amount of HI produces 2–iodopropane.


(d) The reaction of glycerol with excess of HI produces 1, 2, 3-triiodopropane.
54. Which of the following statements is not correct?
(a) An organic compound on treating with HIO4 gives CH3COCH3 and HCHO. The compound is
CH3
CH3 C CH2
OH OH
(b) An organic compound on treating with HIO4 gives 5HCOOH and one HCHO. The compound is
H2 C(CHOH)4CH2 .
OH OH
(c) Thiols are less soluble in water as compared to the corresponding alcohols.
(d) Thiol CH3SH is more acidic than CH3OH.
55. Which of the following statements is correct?
(a) The reduction of p-O2NC6H4CH2COOH with LiAlH4 gives p-O2NC6H5CH2CH2OH.
(b) The reaction of Ph2C==CHCH3 with BH3 in tetrahydrofuran followed by H2O2/OH– gives
Ph2C(OH)CH2CH3.
(c) Cyclopentylmethylcarbinol does not give iodoform test.
(d) An alcohol containing —CH(OH)CH3 gives iodoform test.

ANSWERS
1. (d) 2. (b) 3. (a) 4. (a) 5. (b) 6. (c)
7. (a) 8. (a) 9. (c) 10. (d) 11. (b) 12. (b)
13. (a) 14. (d) 15. (a) 16. (b) 17. (a) 18. (a)
19. (b) 20. (d) 21. (b) 22. (c) 23. (b) 24. (a)
25. (b) 26. (d) 27. (b) 28. (b) 29. (d) 30. (a)
31. (a) 32. (c) 33. (a) 34. (a) 35. (b) 36. (d)
37. (b) 38. (a) 39. (c) 40. (b) 41. (d) 42. (c)
43. (a) 44. (b) 45. (b) 46. (c) 47. (d) 48. (a)
49. (d) 50. (d) 51. (c) 52. (a) 53. (a) 54. (b)
55. (d)

HINTS AND SOLUTIONS


1. Reverse of the choice d is true.
2. Addition follows Markovnikov’s rule.
3. Addition follows anti-Markovnikov’s rule.
4. Alcohol is more acidic than acetylene. The latter is more acidic than NH3.
5. CH3CHO will produce 2° alcohol with RMgX.
6. CH3COCH3 produces 3° alcohol with RMgX.
8. Compound containing —OH group will show maximum hydrogen bonding.
9. Increasing branching causes lowering of boiling point.
10. Glycerin is a trihydric alcohol (CH2OHCHOHCH2OH).
13. (b) Ethylene glycol (HOCH2CH2OH) has two centres of hydrogen bonding while ethanol has only one.
Organic Compounds Containing Oxygen (Alcohols, Phenols, Ethers, Aldehydes, Ketones, Carboxylic Acids and their Derivatives) 24.11

(c) Anti-Markovnikov addition of water takes place.


(d) Markovnikov addition of water takes place.
14. (b) This is because of the formation of carbocation as the intermediate.
(c) This is because of the formation of carbocation as the intermediate.
(d) In the presence of an acid, alcohol gives protonated alcohol. the latter readily loses the weakly basic water
molecule leaving behind the carbocation. The unprotonated alcohol would have to lose the strongly basic

15. (a) Replacement of —OH by a halogen in an alcohol is nucleophilic substitution reaction. It is the protonated
alcohol which acts as a substrate.
(b) Alcohols are acidic enough to react with active metals to liberate hydrogen gas. They are basic enough to accept
a proton from strong acids.
(c) It gives a ketone containing the same number of carbon atoms. Further oxidation will give carboxylic acid
containing lesser number of carbon atoms.
16. (a) Ethanol forms a lower boiling azeotropic mixture containing 95.6 and 4.4 by volume of alcohol and water,
respectively. Water can be removed by adding Mg metal to azeotropic mixture which reacts with water to form
Mg(OH)2 and releasing H2.
(b) The OH group of cyclohexanol is more exposed and available for H-bonding with water due to the more
compactness of the remaining R group.
(c) The product is 2-phenyl-2-butanol.

(Note: Although phenyl is a better migrator than H, yet the hydride migrates so as to give a more stable 3°
benzylic carbocation.)
(d) The product is cis- and trans-2-methylcyclopentanol. In this reaction, addition of H+ to C1 gives a 2° carbocation
which rearranges by ring expansion to give substituted cyclopentanol.
H OH
CH3
H+ H
CH == CH2 CH CH3 CH CH H Æ
2O H

17. The ease of dehydration is tertiary > secondary > primary.


18. Tertiary alcohol reacts faster with HBr.
19. But-2-ene is more stable than but-1-ene (Saytzeff’s rule).
20. The order of reactivity is 1° < 2° < 3°.
21. The order of reactivity is HCl < HBr < HI.
22. The iodoform test will be shown by a molecule containing —COCH3 group or by a molecule producing this group
on oxidation. The compound CH3CH2COCH2CH3 will not show iodoform test.
23. 3° carbonium ion is most stable.
24. The order of reactivity of HX with ROH is benzyl > 3° > 2° > 1°
25. The reaction occur with rearrangement as 2° carbocation is more stable than 1° carbocation.
CH3
Cu
26. CH3 C OH Æ CH3 C CH2
350°C
CH3 CH3
24.12 Complete Chemistry—JEE Main

27. ROH + R¢MgX æÆ ROMgX + R¢H


28.

29.

31. Reactivity of alcohol towards Na is MeOH > 1° > 2° > 3°.


[O]
32. CH3CHCH2CH3 Æ CH3COCH2CH3 .
OH
33. The relative order of acidity is H2O > ROH > HC CH.
34. The relative order of basicity of conjugate base is opposite to that of acid.
Cu
35. CH3CHOHCH3 Æ CH3COCH3.
570 K
36. The strongest acid amongst the given compounds is alcohol.
39. (a) The differences in acidities and basicities are slight, all species are present at equilibrium.
(b) ROH is a weaker acid than R¢SH and also R¢S– is a weaker base than RO–. Hence, the reaction will not be
feasible as the reactants contain weaker acid and weaker base as compared to the products.
(c) Because of the bigger size of S, the negative charge on it is more easily polarised which makes it easier for the
nucleophilic attack. The nucleophilic nature of RO– is also diminished in a protic solvent because there will
be more stronger hydrogen bonding due to the large negative charge density on the smaller sized O atom.
(d) The bond angles are 109° and 96° for C—O—H and C—S—H, respectively. The oxygen involves sp3 hydrid
orbitals while sulphur involves p orbitals for bonding with C and H atoms.
40. Blue colour is obtained with secondary alcohol.
41. Tertiary alcohol is most reactive towards Lucas reagent.
43. Primary alcohol does not react with Lucas reagent.
HIO4
44. CH3CH CHCH2CH3 Æ CH3CHO + OCHCH2CH3
OH OH
45. The number of C—C bond broken is equal to the number of HIO4 molecules used.
46. The reagent C6H5NH+ CrO3Cl– oxidizes alcohol to aldehydic stage.
47. Tertiary alcohol reacts fastest with Lucas reagent.
48. Red colouration is shown by 1° alcohol.
49. The formation of alkyl chloride produces cloudiness.
H
51. (a) tert-Butyl alcohol does not possess CH3 C OH group, hence it does not give iodoform test.

H
(b) The molecule does not possess CH3 C OH group.

H H
HIO4
(c) The reaction is R C C R¢ Æ RCHO + R¢ CHO
OH OH
Organic Compounds Containing Oxygen (Alcohols, Phenols, Ethers, Aldehydes, Ketones, Carboxylic Acids and their Derivatives) 24.13

R¢ H R¢
HIO4
(d) The products are one aldehyde and one ketone. R C C R≤ Æ R C O + R≤ CHO
OH OH
52. (a) The molecule does not contain —OH groups attached to adjacent carbon atoms.
(b) Tertiary alcohol gives 3° carbocation and secondary alcohol gives 2° carbocation. Since 3° carbocation is more
stable than 2° carbocation, it is formed more readily.
53. (a) Tertiary alcohol reacts faster than secondary alcohol with Lucas reagent.
(b) The correct order is 1° > 2° > 3°.
(c) With small amount of HI, allyl iodide is obtained. See Q. 28.
(d) With excess of HI, 2-iodopropane is obtained. See Q. 29.
CH3
HIO4
54. (a) CH3 C CH2 CH3COCH3 + HCHO
OH OH
(b) The compound is . H2C(CHOH)4CHO.
OH

H2 C CH CH CH CH CH O Æ HCHO + 5HCOOH
OH OH OH OH OH
(c) Thiols are less polar than the alcohols.
(d) The negative charge on S (which is larger in size than oxygen) is more spread on its conjugate base RS– of the
acid RSH as compared to that on O of RO–, Thus, RS– is less likely to attract H+ than RO–, making RSH more
acidic than ROH.
55. (a) Both NO2 and COOH groups are reduced. The product is p-H2NC6H4CH2CH2OH.
(b) The hydration involves anti-Markovanikov addition.
1. BH THF
Ph2C==CHCH3 2. H O3 /OHÆ Ph2CHCH(OH)CH3
2 2

(c) An alcohol containing a methyl carbinol with at least one H on the carbinol C gives iodoform test.
Cyclopentylmethylcarbinol shows iodoform test and the product is cyclopentane-carboxylic acid.

MULTIPLE CHOICE QUESTIONS FROM AIEEE AND JEE MAIN

1. During dehydration of alcohols to alkenes by heating with concentrated H2SO4 the initiation step is
(a) formation of an ester (b) protonation of alcohol molecules
(c) formation of carbocation (d) elimination of water [2003]
2. Among the following compounds which can be dehydrated very easily is
OH
(a) CH3CH2CH2CH2CH2OH (b) CH3CH2CH2 CH CH3

CH3
(c) CH3CH2 CCH2CH3 (d) CH3CH2 CH CH2CH2OH [2004]

OH CH3
24.14 Complete Chemistry—JEE Main

3. The best reagent to convert pent-3-en-2-ol into pent-3-en-2-one is


(a) chromic anhydride in glacial acetic acid (b) pyridinium chlorochromate
(c) acidic permanganate (d) acidic dichromate [2005]
4. A liquid was mixed with ethanol and a drop of concentrated H2SO4 was added. A compound with a fruity smell
was formed. The liquid was
(a) CH3COCH3 (b) CH3COOC2H5 (c) CH3OH (d) HCHO [2009]
5. From amongst the following alcohols the one that would react fastest with conc. HCl and anhydrous ZnCl2 is
(a) 1-butanol (b) 2-butanol
(c) 2-methylpropan–2–ol (d) 2-methylpropanol [2010]
6. Consider the following reaction
C2H5OH + H2SO4 Æ Product
Among the following, which one cannot be formed as a product under any conditions?
(a) Ethyl-hydrogen sulphate (b) Ethylene
(c) Acetylene (d) Diethyl ether [2011]
7. Consider thiol anion (RS–) and alkoxy anion (RO–). Which of the following statements is correct?
(a) RS– is less basic and less nucleophilic than RO– (b) RS– is less basic but more nucleophilic than RO–
(c) RS is more basic and more nucleophilic than RO (d) RS– is more basic but less nucleophilic than RO–
– –

[2011]
8. Iodoform can be prepared from all except
(a) Ethyl methyl ketone (b) Isopropyl alcohol
(c) 3-Methyl-2-butanone (d) Isobutyl alcohol [2012]
9. An unknown alcohol is treated with the Lucas reagent to determine whether the alcohol is primary, secondary or
tertiary. Which alcohol reacts fastest and by what mechanism?
(a) secondary alcohol by SN1 (b) tertiary alcohol by SN1
(c) secondary alcohol by SN2 (d) tertiary alcohol by SN2 [2013]
10. Which one of the following statements is not correct ?
(a) Alcohols are weaker acids than water
(b) Acid strength of alcohols decreases in the following order
RCH2OH > R2CHOH > R3COH
(c) Carbon-oxygen bond length in methanol, CH3OH is shorter than that of C - O bond length in phenol.
(d) The bond angle O in methanol is 108.9° [2014]
C H
CH2 CH CH2
11. on mercuration- demercuration produces the major product:

CH2 CH CH3 CH2 CH2 CH2 OH


(a) (b)
OH

CH2 CH CH2 CH2 COOH


(c) (d) [2014]
OH OH
12. In the Victor-Meyer’s test, the colour given by 1°, 2° and 3° alcohols are respectively:
(a) Red, colourless, blue (b) Red, blue, colourless
(c) Colourless, red, blue (d) Red, blue, violet [2014]
Organic Compounds Containing Oxygen (Alcohols, Phenols, Ethers, Aldehydes, Ketones, Carboxylic Acids and their Derivatives) 24.15

13. In the hydroboration - oxidation reaction of propene with diborane, H2O2 and NaOH, the organic compound
formed is:
(a) CH3CH2OH (b) CH3CHOHCH3
(c) CH3CH2CH2OH (d) (CH3)3COH [2014]
14. The product of the reaction given below is [2016]

1. NBS/hn
2. H2O/K2CO3

OH OH CO2H
(a) (b) (c) (d)

ANSWERS
1. (b) 2. (c) 3. (a) 4. (b) 5. (c) 6. (c)
7. (b) 8. (d) 9. (b) 10. (c) 11. (a) 12. (b)
13. (c) 14. (a)

HINTS AND SOLUTIONS


2. The compound CH3CH2 C (CH3)CH2CH3 produces most stable 3° carbocation and thus the compound will
OH
undergo dehydration most easily. Therefore, choice (c) is correct.
4. There is a formation of ester when ethanol reacts with a carboxylic acid in acidic medium.
H+
CH3COOH + HOC2H5 CH3COOC2H5 + H2O
Ester
(fruity smell)

5. The given alcohols are:


CH3

CH3CH2CH2CH2OH ; CH3CHCH2CH3 ; CH3 C CH3 CH3


1-Butanol
OH OH CH3 CH CH2OH
2-Butanol 2-Methylpropan-2-ol 2-Methylpropanol

3° alcohol reacts fastest with conc. HCl and anhydrous ZnCl2.


6. Acetylene is not formed in the reactions between C2H5OH and H2SO4 under different experimental conditions.
7. RSH is a stronger acid than ROH. Thus, the conjugate base of RSH, i.e. RS–, is a weaker base than the conjugate
base of ROH, i.e. RO–.
In going down a group in the periodic table, nucloephilicity increasses. Thus, RS– is more nucleophilic than RO–.
8. The given compounds are:
24.16 Complete Chemistry—JEE Main

A compound containing —COCH3 and groups show iodoform test. Hence, isobutyl alcohol will not

show iodoform test.


9. Tertiary alcohol reacts fastest with lucas reagent. Because of bulkiness at the site of reaction the alcohol reacts
with Lucas reagent via SN1 mechanism.
10. Due to the resonance effect in phenol, the C - O bond in phenol is shorter than the C - O bond in methanol.
11. Alkene on oxymercuration - demercuration produces an alcohol in which the net reaction is the addition of H2O to
the double bond in accordance with Markovnikov’s rule (i.e. -OH group adds to carbon containing lesser number
of hydrogens). Hence, we have
CH2 CH CH2 CH2 CH(OH)CH3
H2 O

Therefore, the choice (a) is correct.


12. 1° alcohol produces blood red colour. 2° alcohol produces blue colour. 3° alcohol produces no colour.
13. In the hydroboration - oxidation reaction, an alken reacts with BH3 to give alkyl boranes which on oxidation with
alkaline H2O2 gives alcohol. The product obtained in the accordance with the anti– Markovnikov’s rule.
CH3 - CH = CH2 + BH3 Æ CH3CH2CH2BH2 H2O2, OH- Æ CH3CH2CH2OH
14. N – Bromosuccinimide (NBS) causes the bromination of an alkene at the allylic position.

Br OH
NBS/n H2O/K2SO3

SECTION 2 Phenols

Methods of Preparation

N N+HSO–4 OH

H2O, H+
Hydrolysis of Diazonium Salt Æ + N2

SO3H OH

NaOH
Alkali Fusion of Sulphonates Æ
fusion
Organic Compounds Containing Oxygen (Alcohols, Phenols, Ethers, Aldehydes, Ketones, Carboxylic Acids and their Derivatives) 24.17

Synthesis from Cumene


Cumene reacts with air at high temperature by a radical mechanism to form cumene hydroperoxide which on treating
with acid gives phenol and acetone.

O OH
H3 C CH3
CH H 3C C CH3 OH

O2 H3O+
Æ Æ + CH3 COCH3
D D

Physical Properties
Because of hydrogen bonding, phenols have quite high boiling points. Phenols themselves are colourless but become
coloured due to the atmospheric oxidation.
Comparison of physical properties of o-nitrophenol with those of p-nitrophenol and m-nitrophenol revealed wild
variations. For example, o-nitrophenol has a much lower boiling point, much lower solubility in water than its isomers.
It can be steam-distilled while other isomers cannot be. These variations are due to the fact that o-nitrophenol involves
intramolecular hydrogen bonding whereas the other two isomers involve intermolecular hydrogen bonding:
O O.... H
O N O
N
O
H
O
(monomolecular)
O N
H O O
(multimolecular)

The boiling point of o-nitrophenol is low because it exists as a monomolecule. The solubility of m- and p-isomers is
due to their ability to form hydrogen bond with water. Steam distillation depends upon the appreciable vapour pressure
of the substance at the boiling point of water. Ortho isomer being monomolecular has an appreciable vapour pressure
whereas meta and para isomers being multimolecular have low vapour pressures.

Chemical Reactions
Acidic Nature of Phenols
Phenols are weak acids. These are soluble in aqueous NaOH and not in aqueous bicarbonates. Phenol are stronger acids
than water and alcohols but weaker than carboxylic acids.
Phenols behaves as a weak acid because the resultant phenoxide ion is more stable (because of resonating effect) as
compared to phenol.
O H O–

+ H+

more stable
than phenol

The resonating structures of phenols are as follows.


24.18 Complete Chemistry—JEE Main

+ + +
OH OH OH OH OH
H –
´ – ´ ´ H ´

H –
(I) (II) (III) (IV) (V)

The structures (II) to (IV) involve separation of charges and thus have much higher energy than those of structures
(I) and (V). In other words, the resonance hybrid includes major contribution from (I) and (V).
The corresponding structures of phenoxide ion are as follows:
– –
O O O O O
H –
´ – ´ ´ H ´

H
(I) (II) (III) (IV) (V)

hybrid structure is more stable than phenol.


Formation of Ethers (Williamson Synthesis)
In alkaline medium, phenols react with alkyl halides to yield ethers.

OH O–Na+ OC2H5

NaOH C2H5I
Æ Æ

OH O– Na+ OCH2COONa OCH2COOH

NaOH CICH2COOH HCl


Æ Æ Æ
heat

Formation of Esters
Phenols are usually converted into their esters by the action of acid, acid chloride or anhydride.
O
OH O C R

+ RCOCI Æ

Fries Rearrangement
When esters of phenols are heated with AlCl3, the acyl group migrates from the phenolic oxygen to an ortho or para
position of the ring producing a ketone.
Organic Compounds Containing Oxygen (Alcohols, Phenols, Ethers, Aldehydes, Ketones, Carboxylic Acids and their Derivatives) 24.19

O
OH O C C2H5 OH OH
COC2H5
C2H5COCI AlCl3
Æ Æ +
CS2
o-hydroxyphenyl
ethyl ketone (volatile COC2H5
in steam) p-hydroxyphenyl
ethyl ketone

Ring Substitution
The phenolic group is a powerful activating group and is thus ortho and para directing. The intermediates formed are
oxonium ions (instead of carbocations) which are formed faster than the formation of carbocations from benzene, hence,
phenols are more reactive towards electrophilic substitution than benzene. The structures of oxonium ions are:

+ +
OH OH
H
Y and

H Y
In an aqueous solution, bromination of phenol yields tribromophenol. In lesser polar solvent, such as CHCl3, CCl4 or
CS2, monobromination products are obtained. In aqueous medium, PhOH gives PhO– of which complete bromination
takes place. The less reactive PhOH does not ionize in nonpolar solvent and thus monobromination occurs.
OH OH OH OH
Br Br Br2 Br
Br2 CHCl3 or CCl4 or CS2
+
aqueous medium medium

Br Br
Similarly, concentrated nitric acid gives trinitrophenol (picric acid) with low yield as nitration is accompanied by
considerable oxidation. Use of dilute nitric acid gives monosubstituted nitrophenol.
Nitrous acid converts phenol to nitrosophenol.
OH OH

NaNO2 + H2SO4
Æ

N
O
p-nitrosophenol

Phenols can be sulphonated with concentrated H2SO4.


OH OH OH
SO3 H
conc. H2SO4 conc. H2SO4
100°C 15–20°C

SO3H
24.20 Complete Chemistry—JEE Main

Phenols can undergo Friedel-Crafts alkylation and acylations:


OH OH OH
CH3
AlCl3
+ CH3Cl Æ +

o-cresol
CH3
p-cresol

OH OH OH
COCH3
AlCl3
+ CH3COCl Æ +

o-hydroxy-
acetophenone COCH3
p-hydroxy-
acetophenone

Kolbe Reaction
When the salt of phenol is treated with carbon dioxide, a —COOH group is introduced in the ring:
ONa OH OH
COONa COOH
H+
+ CO2 Æ Æ

(main product) salicyclic acid

Reimer-Tiemann Reaction
When phenol is treated with chloroform and aqueous sodium hydroxide, a —CHO group is introduced in the ring:
Reimer-Tiemann reaction is an electrophilic substitution on PhO–. The electrophile is dichlorocarbene which contains
a C with only six electrons. OH – – .
HC Cl3 + Æ . CCl3 Æ Cl – + .. CCl2
–H

The treatment of phenol with carbon tetrachloride in aqueous NaOH produces salicylic acid.
Coupling Reactions
Phenol couples with benzenediazonium chloride in alkaline medium to form p-hydroxyazobenzene.

OH–
N N Cl + OH Æ N N OH

Condensation Reactions
Phenol on combining with phthalic anhydride in the presence of concentrated sulphuric acid produces phenolphthalein:
Organic Compounds Containing Oxygen (Alcohols, Phenols, Ethers, Aldehydes, Ketones, Carboxylic Acids and their Derivatives) 24.21

Phenol condenses with formaldehyde in the presence of dilute acid or alkali producing bakelite.
Oxidation in Air
Phenol on exposure to air produces a red coloured product called phenoquinone.

OH O

airÆ excess of
Æ OH....O O....HO
phenol
phenoquinone
O

Schotten-Baumann Reaction
The reaction of phenol with benzoyl chloride is known as Schotten-Baumann reaction.

OH OCOC6H5

NaOH
+ ClCOC6H5 Æ

phenylbenzoate

Test for Phenol


Phenols liberate hydrogen gas when treated with sodium. These are soluble in sodium hydroxide solution and not in
sodium bicarbonate solution. Phenols give coloured complexes (usually green, violet, blue) with ferric chloride.

MULTIPLE CHOICE QUESTIONS ON SECTION 2

General Characteristics
1. Picric acid is
(a) 2-nitrophenol (b) 4-nitrophenol (c) 2, 4-dinitrophenol (d) 2, 4, 6-trinitrophenol
2. Which of the following orders is true regarding the acidic nature of phenol?
(a) Phenol > o-Cresol > o-Nitrophenol (b) Phenol > o-Cresol < o-Nitrophenol
(c) Phenol < o-Cresol > o-Nitrophenol (d) Phenol < o-Cresol < o-Nitrophenol
3. Which of the following statements is not true?
(a) Phenol is a weak acid (b) Phenol is soluble in NaOH
(c) Phenol liberates CO2 from Na2CO3 solution (d) Phenol gives violet colouration with neutral FeCl3
4. Which of the following statements is not true?
(a) When vapours of phenol are passed over Zn dust, benzene is formed.
(b) The phenolic —OH group is meta directing group
24.22 Complete Chemistry—JEE Main

(c) The phenolic —OH group is ortho-and para directing group


(d) o-Nitrophenol has a lower boiling point as compared to that of p-nitrophenol
5. Phenol is less acidic than
(a) acetic acid (b) water (c) p-nitrophenol (d) ethanol
6. Which of the following statements regarding phenols is not correct?
(a) o-Nitrophenol has a much lower boiling point as compared to p-nitrophenol.
(b) o-Nitrophenol has much lower solubility in water as compared to p-nitrophenol.
(c) o-Nitrophenol is stronger acid than p-nitrophenol
(d) o-Nitrophenol can be steam distilled while p-nitrophenol cannot be.
7. Which of the following statements regarding phenols is not correct?
(a) Phenols are stronger acids than water and alcohols
(b) Phenols are weaker acids than carboxylic acids
(c) Phenols are soluble in both aqueous NaOH and aqueous NaHCO3.
(d) Phenoxide ions are more stable than the corresponding phenols.
8. Which of the following orders regarding the acid strength of phenols is correct?
(a) p-aminophenol > p-chlorophenol > p-nitrophenol (b) p-aminophenol < p-chlorophenol < p-nitrophenol
(c) p-aminophenol > p-nitrophenol > p-chlorophenol (d) p-nitrophenol > p-aminophenol > p-chlorophenol
9. Which of the following orders regarding acid strengths is correct?
(a) Benzyl alcohol > phenol > p-hydroxybenzoic acid (b) Benzyl alcohol < phenol < p-hydroxybenzoic acid
(c) Benzyl alcohol < p-hydrobenzoic acid < phenol (d) phenol > Benzyl alcohol > p-hydroxybenzoic acid
10. Phenol can be prepared by
(a) Dow process (b) Claisen reaction
(c) Cannizzaro reaction (d) Reimer-Tiemann reaction
11. The order of activity in (I) phenol, (II) m-nitrophenol, (III), p-nitrophenol and (IV) p-methylphenol is
(a) III > II > I > IV (b) II > III > I > IV (c) I > III > II > IV (d) IV > I > II > III
12. Which of the following statements is not correct?
(a) o-Nitrophenol is steam volatile whereas p-nitrophenol is not.
(b) Phenol is partially miscible with water. Solubility increases with increase in temperature.
(c) Phenol themselves are colourless.
(d) o-Nitrophenol has a higher boiling point than p-nitrophenol.
13. Which of the following statements is not correct?
(a) o-Nitrophenol has lower solubility in water than its para isomer.
(b) Phenols and their salts have opposite solubility characteristics in ionic and nonionic solvents.
(c) Phenols are stronger acids than carboxylic acids.
(d) The —O– group in phenolate ion is more strongly electron-releasing than the —OH group in phenol.
14. Which of the following statements is not correct?
(a) The resonance stabilization of phenol is more than that of phenoxide ion.
(b) p-Aminophenol is less acidic than phenol.
(c) Heating of an ester of a phenol with aluminium chloride causes rearrangement of acyl group from the phenolic
oxygen to an ortho or para position of the ring.
(d) If halogenation of phenol is carried out in a less polar medium such as CHCl3, CCl4 or CS2, only monohalogenated
products are obtained.
15. Which of the following statements is not correct?
(a) Unlike alkyl halide, aryl halides are not readily prepared from the corresponding hydroxyl compounds.
(b) Most ethers are inert toward bases, but 2,4-dinitroanisole is readily cleaved to methanol and 2,4-dinitrophenol
Organic Compounds Containing Oxygen (Alcohols, Phenols, Ethers, Aldehydes, Ketones, Carboxylic Acids and their Derivatives) 24.23

(c) The bromination of p-methylanisole produces 2-bromo-3-methylanisole as the principal product.


(d) The nitration of m-nitroanisole produces a mixture of 3,4-dinitro-anisole and 2,5 dinitroanisole.
16. Which of the following statements is not correct?
(a) Phenol is a stronger acid than ROH.
(b) PhO– is a stronger base than RO–.
(c) NaHCO3 does not react with phenol because it is less acidic than phenol.
(d) Na2CO3 reacts with phenol because it is more acidic than phenol.
17. Which of the following statements is not correct?
(a) Picric acid is 2,4,6-trinitrophenol.
(b) Kolbe reaction helps in converting phenol to salicyclic acid.
(c) Reimer-Tiemann reaction involving phenol and chloroform converts the former into salicyclic acid.
(d) Phenol gives characteristic colour with FeCl3. This test is due to enol structure.
Chemical Reactions
18. Salicyclic acid is produced when phenol in alcoholic KOH is treated with
(a) CHCl3 (b) CH3Cl (c) CCl4 (d) CH2Cl2
19. The reaction of phenol with chloroform in alkaline medium producing salicyldehyde is known as
(a) Cannizzaro reaction (b) Claisen reaction
(c) Reimer-Tiemann reaction (d) Hell-Volhard-Zelinsky reaction
20. The bromination of phenol in aqueous medium produces
(a) 2–bromophenol (b) 4–bromophenol
(c) 2, 4, 6-tribromophenol (d) a mixture of 2- and 4-bromophenols
21. Phenol on treating with concentrated H2SO4 at 15–20 °C mainly produces
(a) phenol-2-sulphonic acid
(b) phenol-4-sulphonic acid
(c) a 50% mixture of ortho and para phenol sulphonic acid
(d) phenol-2, 4, 6-trisulphonic acid
22. Phenol on treating with concentrated H2SO4 at 100 °C mainly produces
(a) ortho-phenolsulphonic acid
(b) para-phenolsulphonic acid
(c) a 50% mixture of ortho- and para-phenolsulphonic acid
(d) phenol-2, 4, 6-trisulphonic acid
23. The treatment of phenol with phthalic anhydride in the presence of concentrated H2SO4 produces
(a) aspirin (b) methyl red (c) methyl orange (d) phenolphthalein
24. Phenol reacts with bromine in CS2 medium at low temperature to give
(a) m-bromophenol (b) p-bromophenol (c) o- and p-bromophenol (d) 2, 4, 6-tribromophenol
25. Phenol on exposure to air produces a red coloured product known as
(a) benzoquinone (b) hydroquinone (c) phenoquinone (d) quinone
ONa OH OH
COONa COOH
120-140°C HCl
26. The reaction + CO2 Æ Æ is known as
6-7 atm –NaCl

(a) Wurtz reaction (b) Riemer-Tiemann reaction


(c) Schotten-Baumann reaction (d) Kolbe reaction
24.24 Complete Chemistry—JEE Main

OH OCOC6H5

27. The reaction + ClCOC6H5 Æ is known as

(a) Wurtz reaction (b) Riemer-Tiemann reaction


(c) Kolbe reaction (d) Schotten-Baumann reaction

OH OH OH
COCH3
28. The reaction 2 + CH3COCl Æ + is known as

COCH3
(a) Wurtz reaction (b) Friedel-Crafts reaction
(c) Kolbe reaction (d) Wurtz-Fitting reaction
29. The reaction of PhOCH2CH3 with one equivalent of HI produces
(a) PhI + CH3CH2OH (b) PhI + CH3CH3 (c) C6H6 + ICH2CH3 (d) PhOH + ICH2CH3
30. Benzenediazonium chloride on reacting with phenol in weakly basic medium gives
(a) p-hydroxyazobenzene (b) benzene (c) diphenyl ether (d) chlorobenzene
31. Phenol on distilling with zinc dust gives
(a) benzene (b) diphenol (c) diphenyl ether (d) zinc phenoxide
32. In the reaction of phenol with CHCl3 in aqueous sodium hydroxide at 70°C, the electrophile attacking the ring
is
(a) CHCl3 (b) :CCl2 (c) –:CHCl2 (d) –:CCl3
33. Which of the following statements is not correct?
(a) The relative amounts of o-phenolsulphonic acid and p-phenolsulphonic acid obtained during sulphonation of
phenol depends upon the temperature at which reaction is carried out.
(b) o-Phenolsulphonic acid is converted into the para isomer by sulphuric acid at 100 °C.
(c) Alkaline hydrolysis of CHCl3 is much more rapid than CH2Cl2.
(d) It is not possible to separate the compounds in a mixture containing ArNO2, ArNH2, ArCOOH and ArOH.
34. Which of the following statements is correct?
(a) Sulphonation of phenol at low temperature is rate-controlled to give o-HOC6H4SO3H.
(b) Sulphonation of phenol at higher temperatures is thermodynamically controlled to give o-HOC6H4SO3H.
(c) The —OH is more activating than —O– towards aromatic electrophilic substitution reactions.
(d) Bromination of phenol in aqueous medium gives monobrominated phenol while in nonaqueous medium,
tribrominated phenol is formed.
35. Which of the following statements is not correct?
(a) Phenol reacts with CCl4 in the presence of NaOH solution at 70°C to give salicyclic acid.
(b) Phenol reacts with phthalic anhydride in the presence of concentrated H2SO4 forming phenolphthalein which
is used as acid-base indicator.
(c) Phenol on exposure to air produces a red coloured product known as phenoquinone.
(d) The treatment of concentrated H2SO4 at 15–20 °C with phenol produces p-phenolsulphonic acid whereas at
100 °C, o-phenolsulphonic acid is produced.
36. Which of the following statements is not correct?
(a) When vapours of phenol are passed over Zn dust, benzene is formed.
(b) The penolic —OH group is ortho- and para- directing.
(c) o-Nitrophenol has a lower boiling point as compared to that of p-nitrophenol.
(d) Phenol is less acidic than o-cresol.
Organic Compounds Containing Oxygen (Alcohols, Phenols, Ethers, Aldehydes, Ketones, Carboxylic Acids and their Derivatives) 24.25

ANSWERS
1. (d) 2. (b) 3. (c) 4. (b) 5. (a) 6. (c)
7. (c) 8. (b) 9. (b) 10. (a) 11. (a) 12. (d)
13. (c) 14. (a) 15. (c) 16. (b) 17. (c) 18. (c)
19. (c) 20. (c) 21. (a) 22. (b) 23. (d) 24. (c)
25. (c) 26. (d) 27. (d) 28. (b) 29. (d) 30. (a)
31. (a) 32. (b) 33. (d) 34. (a) 35. (d) 36. (d)

HINTS AND SOLUTIONS

2. Methyl group is activating group (electron release to benzene) and nitro group is deactivating group (electron
withdrawal group). Methyl group will make release of H+
3. Phenol being a very weak acid does not liberate CO2 from Na2CO3.
4. Phenolic —OH group is ortho- and para- directing.
12. (a) o-Nitrophenol involves intramolecular hydrogen bond whereas p-nitrophenol involves intermolecular
hydrogen bond. Consequently, the former exists as single molecule whereas the latter exists as multimolecular
form.
(d) Because of intramolecular hydrogen bonding in o-nitrophenol, the latter has lower boiling point as compared
to p-nitrophenol (which has intermolecular hydrogen bonding).
13. (b) Phenol is essentially a covalent compound whereas its salt is ionic.

14. (a) The resonating structrues of phenol involves charge separation while those of phenoxide ion do not involve
any charge separation. Since energy is needed to separate opposite charges, the structrues of phenol involve
more energy and thus are less stable than the structures of phenoxide ion.
(b) Electron-releasing substituent destabilizes the phenoxide ion, hence, makes the such substituted phenol less
acidic.
(c) This is known as Fries rearrangement.
15. (a) The bond Ar—O in phenols is strong, due to partial double-bond character and/or sp2 hybridization of
aromatic carbon.
(b) The reaction is nucleophilic aromatic substitution of —OH for —OCH3. This is achieved due to the
deactivation of two nitro groups in the ortho and para positions.
(c) The principal directing group is —OCH3. Therefore, the obtained compound would be 1-bromo-3-methylanisole.
OCH3 OCH3
Br
Br2
⎯⎯→

CH3 CH3

(d) The principal directing group is — OCH3.

OCH3 OCH3 OCH3


O2N
HNO3
⎯⎯⎯→ +
H2SO4
NO2 NO2 NO2
NO2
24.26 Complete Chemistry—JEE Main

16. (b) The negative charge on the alkoxide anion, RO–, is completely localised, but the negative charge on PhO– is
delocalised by extended p-bonding to the ortho and para ring positions. This makes PhO– weaker base than
RO– and PhOH a stronger acid than ROH.
(c) The reaction of phenol with NaHCO3 would produce the following reaction

PhOH + HCO3– PhO– + H2CO3


pK°a = 10 pK°b = 7.6 pK°b = 4 pK°a = 6.4
(weaker) (weaker) (stronger) (stronger)

Both the reactants are weaker than the corresponding species on the right side. Thus, the reaction would not
proceed to the right side.
(d) The reaction would be

PhOH + CO32– Æ PhO– + HCO3–


pK°a = 10 pK°b = 3.7 pK°b = 4 pK°a = 10.3
(stronger) (stronger) (weaker) (weaker)

Both the products are weaker than the corresponding species on the left side. Thus, the reaction proceeds to the
right side.
17.(b) The reactions are
OH OH OH
O
COONa COOH
125°C
H+
+ C
4–7 atm
O

(c) The compound formed is salicylaldehyde.

OH O– OH
CHCl2 CHO
CHCl3
H+
Aq. NAOH, 70 °C

18.

+ H+
salicylic acid
20. In aqueous medium, trisubstituted phenol is obtained
21. At low temperature, ortho-product is obtained.
22. At high temperature, para-product is obtained.
24. In nonaqueous medium, monosubstituted ortho- and para-products are obtained.
29. Aryl ethers do not cleave on the aromatic side, but only on the alkyl side.

33. (a) C6H5OH


H2SO4 100 H2SO4
˚C ˚C
15 −20
100 ˚C
o-phenolsulphonic acid ⎯⎯⎯→ p-phenolsulphonic acid
Organic Compounds Containing Oxygen (Alcohols, Phenols, Ethers, Aldehydes, Ketones, Carboxylic Acids and their Derivatives) 24.27

(b) Sulphonation is reversible. The ortho isomer is formed more rapidly; the para isomer is more stable. At 15—20
°C, there is rate control of product composition; at 100 °C, there is equilibrium control.
(c) The hydrogen atom in CHCl3 is more acidic than in CH2Cl2.
(d) These can be separated as follows.
ArNO2, ArNH2, ArCOOH, ArOH
NaHCO3

ArCOO− Na+ residue ArNO2, ArNH2, ArOH


+
H NaOH

ArCOOH
ArO− Na+ residue ArNO2, ArNH2
H+ HCl
ArOH
ArNH3+ Cl− ArNO2

OH

ArNH2
34. (b) The reaction is reversible and thus at higher temperatures, thermodynamically controlled para isomer is
formed.
(c) PhO– reacts faster than PhOH with an electrophile because the intermediate formed with PhO– is a fairly stable
uncharged species while in PhOH, a less stable positively-charged intermediate is formed. Thus, DH‡ for the
reaction of PhO– is less than the DH‡ for the reaction of PhOH.

+ + O O O
OH OH OH H
H E+
E+ + E
+ E
H E
H E unsaturated ketone
Phenol Oxonium ions (more stable uncharged
species)

(d) In aqueous solution, some phenol is ionised to give more reactive PhO– species, which, in turn, forms a
tribrominated phenol. In nonaqueous medium, the ionisation is not possible and the monobromination of less
reactive PhOH occurs.
35. (d) At low temperature, o-phenolsulphonic acid is formed which is converted into p-phenolsulphonic acid at higher
temperature.

MULTIPLE CHOICE QUESTIONS FROM AIEEE AND JEE MAIN

1. p-cresol reacts with chloroform in alkaline medium to give the compound A which adds hydrogen cyanide to form
the compound B. The latter on acidic hydrolysis gives chiral carboxylic acid. The structure of the carboxylic acid
is
CH3 CH3
CH2COOH
(a) (b)
CH2COOH
OH OH
24.28 Complete Chemistry—JEE Main

CH3 CH3
CH(OH)COOH
(c) (d)
CH(OH)COOH
OH OH [2005]
OH O–Na+
2. + CHCl3 + NaOH
CHO
The electrophile involved in the above reaction is
≈ ≈
(a) formyl cation (CHO) (b) dichloromethyl cation (C HCl2 )
(c) dichlorocarbene (: CCl2) (d) trichloromethyl anion (C HCl2 ) [2006]
3. The structure of the compound that gives a tribromo derivative on treatment with bromine water is
CH3 CH3 CH2OH

(a) (b) (c) (d)


OH
OH

(a) nitrobenzene (b) 2, 4, 6 – trinitrobenzene


(c) o- nitrophenol (d) p- nitrophenol [2006]
5. The major product obtained on interaction of phenol with sodium hydroxide and carbon dioxide is
(a) Salicylic acid (b) Phthalic acid (c) Benzoic acid (d) Salicyladehyde [2008]
6. Phenol is heated with a solution of mixture of KBr and KBrO3. The major product obtained in the above reaction
is
(a) 2,4,6-tribromophenol (b) 2-bromophenol (c) 3-bromophenol (d) 4-bromophenol
[2009]
7. The correct order of acid strength of the following compounds
A. Phenol B. p-Cresol C. m-Nitrophenol D. p- Nitrophenol
is
(a) C > B > A > D (b) D > C > A > B (c) B > D > A > C (d) A > B > D > C
[2011 (cancelled)]
8. Arrange the following compounds in order of decreasing acidity:

(a) II > IV > I > III (b) I > II > III > IV
(c) III >I > II > IV (d) IV > III > I > II [2013]
9. Sodium phenoxide when heated with CO2 under pressure at 125 °C yields a product which on acetylation
produces C.
Organic Compounds Containing Oxygen (Alcohols, Phenols, Ethers, Aldehydes, Ketones, Carboxylic Acids and their Derivatives) 24.29

ONa
125°C H+
+ CO2 B C
5 atm Ac2O

The major product C would be


OCOCH3 OCOCH3
COOH
(a) (b)
COOH

OH OH
COCH3 COOCH3
(c) (d)

COCH3 [2014]
m π 0?
Cl CN OH SH

Cl CN OH SH
(i) (ii) (iii) (iv)
(a) (iii) and (iv) (b) only (i) (c) (i) and (ii) (d) only (iii) [2014]
11. The following
OH
OH
Anhyd.
+ HCl + HCN ZnCl2
CHO
is known as:
(a) Perkin reaction (b) Gattermann-Koch Formylation
(c) Kolbe¢s reaction (d) Gattermann reaction [2014]
O-
12. Which one of the following substituents at para-position is most effective in stablizing the phenoxide
ion ?
(a) - CH3 (b) - OCH3 (c) - COCH3 (d) - CH2OH [2014]

ANSWERS
1. (d) 2. (c) 3. (b) 4. (c) 5. (a) 6. (a)
7. (b) 8. (c) 9. (b) 10. (a) 11. (d) 12. (c)

HINTS AND SOLUTIONS


1. The main substituent which decides the position of electrohilic substitution in p-cresol is —OH group. Therefore,
the electrophilic substitution occurs at ortho position to the —OH group. The reactions involved are as follows:
24.30 Complete Chemistry—JEE Main

CH3 CH3 CH3 CH3

CHCl3 2H2O (i) HCN


OH – –2HCl (ii) H+
CHCl2 CHO CH(OH)COOH
OH OH OH OH

2. The electrophile is :CCl2 generated by the reaction OH– + CHCl3 H2O + CCl3
| Æ Cl – + : CCl
2
CH3 CH3
Br2(aq) Br Br
3. The reaction is
OH OH
Br
In other choices, meta position will be included in tribromo derivative.
4. With concentrated H2SO4, the reaction proceeds as follows.

OH OH OH
SO3H
H2SO4

SO3H
(Low temperature) (High temperature)
15–20 °C 100 °C
On treating with concentrated HNO3, —SO3H group is knocked off

SO3H NO2
;

At low temperature, o-nitrophenol is formed.


In fact, the product formed may be 2, 4, 6 – trinitrophenol.
OH OH OH
O2N NO2 O2N NO2
HNO3 HNO3

SO3H SO3H NO2


5. The main reaction is

6. In acidic medium, the reaction between Br– and BrO–3 produces Br2 which on reacting with phenol in aqueous
medium produces 2,4,6-tribromophenol.
Organic Compounds Containing Oxygen (Alcohols, Phenols, Ethers, Aldehydes, Ketones, Carboxylic Acids and their Derivatives) 24.31

7. Electron withdrawing —NO2 substituent has an acid-strengthening effect while the electron releasing —CH3
substituent has an acid-weakening effect on phenol. Thus, p-cresol is the weakest acid amongst the given
compounds. The m- and p-nitrophenols are stronger acid than phenol (p-isomer is more strong than m-isomers).
The given choice (b)
8. Electron-releasing group decreases the acidity while electron-attracting group increases the acidity. Thus, the
molecule III will have maximum acidity and the molecule IV will have minimum acidity. Thus, the choice (c) is
correct.
9. The given reaction is Kolbe reaction followed by acetylation of — OH group.
ONa OH OCOCH3
COONa H+ COOH
125°C
+ CO2 Ac2O
5 atm

10. Assuming planar structure, hydroquinone has two conformations as shown in the following.
H H
O O

and

O O
H H
(I) (II)

The structure (I) is expected to have zero dipole moment whereas the structure (II) is expected to have dipole
moment. This conclusion may be derived based on the vector addition of components of dipole moment.
The above facts are also applicable to 1, 4-benzenedithiol (structure iv).
11. Preparation of phenolic aldehyde by the treatment of phenol with hydrogen cyanide and hydrogen chloride in the
presence of Lewis acid (e. g. Zn Cl2) is known as Gattermann reaction.
12. Phenoxide ion is an anion. Electron- releasing group will destabilise this while electron- withdrawing group will
stabilise this. –COCH3 is the substituent which is electron- withdrawing, hence, will stablise the anion.

SECTION 3 Ethers

Ethers
Ethers in the common system of nomenclature are named by the alkyl groups attached to the oxygen atom followed
by the word ether, e.g., dimethyl ether, methyl ethyl ether. In IUPAC, ethers are considered to be alkoxy derivatives of
alkanes, e.g. methoxyethane (CH3OC2H5), ethoxyethane (C2H5OC2H5). If other substituents like —CHO, —OH, etc.
are present, the compound is named as the derivative of that substance, e.g., 2-ethoxyethanol (C2H5OCH2CH2OH),
methoxyethanal (CH3OCH2CHO), etc.
Ethers can be prepared by the dehydration of alcohols for which concentrated sulphuric acid at 410 K or catalytic
hydrogenation (Al2O3 at 520 K) can be used. Unsymmetrical ethers can be prepared by using Williamson synthesis in
which alkyl halide is heated with sodium alkoxide. However, tertiary halide produces alkenes with the elimination of
HCl.
Ethers are chemically inert substances. With concentrated acids, they form oxonium salts. Ether linkage is cleaved
by heating with HI producing alcohol and alkyl halide. Ether containing one aromatic and one aliphatic group produces
phenol and alkyl halide on cleaving with HI.
Ether linkage in aromatic compound is ortho and para directing group.
24.32 Complete Chemistry—JEE Main

Ether when exposed to air for a long time produces compound containing peroxide linkage. For example,
CH3
CH3 CH3 CH3
air Æ CH O C O O H
CH O CH
CH3 CH3 CH3
CH3
This perioxide linkage can be detected by treating the sample with ferrous ammonium sulphate followed by the
addition of ammonium thiocynate. Ferrous ions is oxidized to ferric ions which produces red coloration with NH4CNS.

MULTIPLE CHOICE QUESTIONS ON SECTION 3

1. Which of the following is expected to have the lowest boiling point?


(a) CH3CH2OH (b) CH3CHO (c) CH3COOH (d) CH3OCH3
2. Which of the following does not react with sodium metal?
(a) CH3CH2OCH2CH3 (b) CH3OH (c) CH3COOH (d) HCOOH
3. The heating of phenyl methyl ether with HI produces
(a) ethyl chloride (b) iodobenzene (c) phenol (d) benzene
4. The IUPAC name of CH3CH2OCH2CH2CH3 is
(a) diethyl ether (b) ethyl propyl ether (c) ethoxypropane (d) propoxyethane
5. The reaction CH3I + C2H5 3OC2H5 + NaI is an example of
(a) Wurtz synthesis (b) Clemenson reaction (c) Williamson synthesis (d) Dow reaction

(a) C2H5OC2H5 (b) CH3OCH(CH3)2 (c) CH3OCH2CH2CH3 (d) CH3CH2CH2CH2OH


7. The alkoxy group in benzene is
(a) ortho directing (b) para directing
(c) ortho and para directing (d) meta directing
8. The formation of peroxide linkage in ether due to the exposure in air can be detected by treating it with
(a) sodium
(b) dilute hydrochloric acid
(c) aqueous ferrous ammonium sulphate followed by the addition of ammonium thiocynate
(d) dilute sodium hydroxide
9. The exposure of ether in air for a long time may cause
(a) oxidation to carboxylic acid (b)the formation of peroxide linkage
(c) oxidation to produce aldehyde or ketone (d) the degradation of the molecule
10. Ethers
(a) are soluble in concentrated acids
(b) are insoluble in concentrated acids
(c) have unpleasant smell
(d) have higher boiling point in comparison to the alcohol of comparable molecular mass
11. Given are the two cleavage reactions:
(i) (CH3)3COCH3 3I + (CH3)3COH
(ii) (CH3)3COCH3 3OH + (CH3)3CI
Which of the following statements is correct?
(a) The reagent used in reaction (i) is anhydrous HI in ether and in reaction (ii) is concentrated HI
(b) The reagent used in reaction (i) is concentrated HI and in reaction (ii) is anhydrous HI in ether
(c) The reagent used both in reactions (i) and (ii) is concentrated HI
Organic Compounds Containing Oxygen (Alcohols, Phenols, Ethers, Aldehydes, Ketones, Carboxylic Acids and their Derivatives) 24.33

(d) The reagent used both in reactions (i) and (ii) is anhydrous HI in ether
O
12. The reaction of (CH3)2 C — CH2 with CH3OH in (i) acid H+, and (ii) base CH3O–, respectively, give
(a) (CH3)2C(OCH3)CH2OH and (CH3)2CH(OH)CH2OCH3
(b) (CH3)2C(OCH3)CH2OH and (CH3)2C(OCH3)CH2OH
(c) (CH3)2C(OCH3)CH2OCH3 and (CH3)2C(OH)CH2OH
(d) (CH3)2C(OH)CH2OH and (CH3)2C(OCH3)CH2OCH3

ANSWERS
1. (d) 2. (a) 3. (c) 4. (c) 5. (c) 6. (d)
7. (c) 8. (c) 9. (b) 10. (a) 11. (a) 12. (a)

HINTS AND SOLUTIONS


3. SN2 attack on a C of a benzene ring does not occur. Also high energy C6H5+ is not formed by an SN1 reaction.
Hence, ArI cannot be a product even in an excess of concentrated HI.
11. The low polarity of solvent ether favours the SN2 mechanism and high polarity of solvent water favours SN1
mechanism.
+
The mechanism is (CH3)3COCH3 + HI Æ (CH3)3C O CH3 + I–
H
+ + I +
SN1: (CH3)3C O CH3 Æ CH3OH + (CH3)3C Æ (CH3)3CI ; SN2: I– + (CH3)3C O CH3 Æ CH3I + HOC(CH3)3
H H
12. There occurs cleavage of ether linkage in oxiranes (cyclic three membered ring including oxygen). In acid-
catalyzed reaction, nucleophile –OCH3 from CH3OH attaches to the more substituted carbon where as in base-
catalyzed reaction, it is attached to the less substituted carbon.

MULTIPLE CHOICE QUESTIONS FROM AIEEE AND JEE MAIN

1. Sodium ethoxide has reacted with ethanolyl chloride. The compound that is produced in the above reaction is
(a) ethyl ethanoate (b) diethyl ether (c) 2-butanone (d) ethyl chloride
[2011 (Cancelled)]
2. Williamson synthesis of ether is an example of:
(a) Nucleophilic addition (b) Electrophilic addition
(c) Electrophilic substitution (d) Nucleophilic substitution [2014]
3. Allyl phenyl ether can be prepared by heating:
(a) C6H5Br + CH2 = CH - CH2 - ONa (b) CH2 = CH - CH2 - Br + C6H5ONa
(c) C6H5 - CH = CH - Br + CH3 - ONa (d) CH2 = CH - Br + C6H5 - CH2 - ONa [2014]
4. Consider the reaction sequence below:
24.34 Complete Chemistry—JEE Main

OCH 3

Succinic anhydride Clemmenson’s


A X
AlCl 3 reduction

OCH3 OH OH OCH3
H3CO H3CO
(a) (b) (c) (d)

OH OH [2016, online]

ANSWERS
1. (a) 2. (d) 3. (b) 4. (a)

HINTS AND SOLUTIONS

1. The reaction is CH3CH2ONa + ClCOCH3 CH3CH2 — O — C — CH3

O
Ethyl ethanoate
2. Williamson synthesis involves nucleophilic substitution.
3. To prepare an ether containing a phenyl group, the only choice is to take sodium phenoxide and not phenyl bromide
as the latter is much less reactive towards nucleophilic substitution reaction.

O O
4. H3CO H3CO H3CO
AlCl3 Zn - Hg
O
HCl

O O
Succinic anhydride

SECTION 4 Et Aldehydes and Ketones hers

Methods of Preparation of Aldehydes


H
pyridinium
Oxidation of Primary Alcohol R— CH2OH Æ R C O
chlorochromate
Organic Compounds Containing Oxygen (Alcohols, Phenols, Ethers, Aldehydes, Ketones, Carboxylic Acids and their Derivatives) 24.35

CH3 CHCl2 CHO

Cl2 H2O
Oxidation of Methylbenzene Æ Æ
heat

CH(OOCCH3 )2 CHO

CrO3 H2O
Æ
acetic H+
anhydride

LiAlH(OBu t) 3
R— COCl Æ RCHO
Reduction of Acid Chlorides H 2 Pd BaSO 4
R— COCl Æ RCHO
ethyl acetate

CH(CH3)2 CH(CH3)2

HCl, AlCl 3
Gatterman-Koch Reaction + CO Æ
CuCl

CHO
OH OH

1. ZnCl 2 , ether
Gatterman Reaction + HCN + HCl Æ + NH4Cl
2. H 2 O
OH OH
activating ring
CHO

Heating a Mixture of the Calcium Salts of Formic Acid and any One of its Homologues
(RCOO)2Ca + (HCOO)2 3

Methods of Preparation of Ketones

CrO 3
Oxidation of Secondary Alcohols R CH R¢ Æ R C R¢
K 2 Cr2 O 7
OH O

AlCl 3
Friedel-Crafts Acylation + RCOCl Æ C R + HCl
O

AlCl 3
+ (CH3CO)2O Æ C CH3 + CH3COOH
O

While preparing a ketone of the type ArCOAr¢ containing deactivating group in one of the aromatic ring, it is
important to select the proper combination of Ar and Ar¢. For example, the preparation of m-nitrobenzophenone, one
should proceeds as shown in the following
24.36 Complete Chemistry—JEE Main

NO2 NO2

AlCl3
+ Cl C Æ C
O O

The reaction O 2N + Cl C Æ no reaction


O

does not take place as the strongly deactivating nitro group prevents the acylation reaction
Heating the Calcium Salt of any Fatty Acid other than Formic Acid
(RCO2 3

Physical Properties
Because of C==O group, aldehydes and ketones are polar compounds and their boiling points are higher than those of
nonpolar compounds of comparable molar masses. However, they are not as polar as alcohols and carboxylic acids due
to the absence of hydrogen bonding and thus their boiling points are lower than those of alcohols and carboxylic acids
of comparable molar masses.
Formaldehyde is a gas, rest are liquids or solids. A 40% aqueous solution of formaldehyde is known as formalin. The
lower aldehydes and ketones are soluble in water due to hydrogen bonding between solute and solvent.

Chemical Properties
+ –
The carbonyl group, C==O, plays a dominating role in the chemistry of aldehydes and ketones. Because of the C O
polarization the addition reactions occurring across C==O bond is of the nucleophilic type.
Aldehydes generally undergo nucleophilic addition more readily than ketones as the former involves lesser crowding
at the transition state.
An alkyl group releases electrons and thus destabilises the transition state by intensifying the negative charge
developing on oxygen. On the other hand, an aryl group, which is electron-withdrawing group, is expected to stabilise
the transition state and thus speed up the reaction. However, an aryl group stabilises the reactant more than the transition
state due to resonance and thus lowers down the speed of the reaction. The resonating structures are

R R

+

C O C O

The nucleophilic reaction involving C O group is catalyzed by acids. This is because carbon acquires more positive
charge as depicted in the following.

Z
Z
R¢ H + R¢ + R¢ + R¢
C O C OH C OH C OH
R R R R
(undergoes nucleophilic
attack more readily)

Typical reactions shown by aldehydes and ketones are as follows.


Oxidation
The hydrogen atom attached to the carbonyl carbon is abstracted during oxidation. Hence, only aldehydes are oxidized
easily and not ketones:
Organic Compounds Containing Oxygen (Alcohols, Phenols, Ethers, Aldehydes, Ketones, Carboxylic Acids and their Derivatives) 24.37

KMnO 4
R— CHO Æ R— COOH
K 2 Cr2 O 7
Ketones requires vigorous conditions and the acids obtained contain fewer carbon atoms. Cleavage occurs at the
double bond of the enol form:

OH O OH
C C C C C C C C C
H H H H
enol ketone enol

Aldehydes are oxidized by even mild oxidizing reagents such as Tollens reagent (ammoniacal silver nitrate):

CH3CHO + 2Ag(NH3)2+ + 3OH– Æ 2Ag + CH3COO– + 4NH3 + 2H2O


silver
mirror
O
Ketones containing CH3C are oxidized by hypohalite to give carboxylic acid and haloform. For example,
– –
C2 H 5 C CH3 + 3OI– Æ C2H5COO + CHI3 + 2OH
O
Hypohalite does not attack carbon-carbon double bond present in the molecule. For example,

H CH3 H CH3
OCl–
C C C CH3 Æ C C COOH + CHCl3
O
Reduction
Aldehydes are reduced to 1° alcohols whereas ketones to 2° alcohols. This can be achieved either by catalytic
hydrogenation (H2, Ni) or by the use of lithium aluminium hydride LiAlH4:

H2 + Ni, Pt or Pd
C OH
C O
LiAlH4 or NaBH4; then H+

If carbon-carbon double bond is also present in the carbonyl compound, it is also reduced alongwith. However, the
use of the reagent 9-BBN (9-borabicyclo[3, 3, 1]nonane) prevents this and thus only the carbonyl group is reduced.
For example,

9-BBN HOCH2CH2NH2
CH CH CHO Æ Æ CH CHCH2OH

cinnamaldehyde cinnamal alcohol

Reduction to Hydrocarbons Aldehydes and ketones are reduced to hydrocarbons by (a) Clemmensen reduction
(amalgamated zinc and concentrated hydrochloric acid) and (b) Wolff-Kishner reduction (hydrazine, NH2NH2, and a
strong base like KOH or potassium tert-butoxide). For example,
Zn(Hg)
CH3CH2COCH 3 Æ CH3CH2CH2CH3
conc. HCl
24.38 Complete Chemistry—JEE Main

The Wolff-Kishner reduction proceeds as follows.

H
NH2
O N N N
H
C CH3 C CH3 C H
CH3 C H
H2NHH2 KOH –N2 CH3
Æ Æ
KOH KOH

H
Action of Grignard Reagents

Ketones produces 3° alcohol with Grignard reagent.


OH
Reactions with Sodium Hydrogen Sulphite C O + NaHSO3 C
SO3Na
bisulphite
Most aldehydes form bisulphite compounds. Ketones of the type CH3COR, where R is a primary alkyl group, form
bisulphite compounds. If R is secondary or tertiary alkyl group, the formation of bisulphite is slow. Ketones of the type
RCOR¢ does not form bisulphite provided R is C2H5 or higher alkyl group.
Addition of HCN (or sodium cyanide and mineral acid)

+
C O + CN– C O– H C OH
CN CN
cyanohydrin

Cyanohydrin undergoes hydrolysis to give a-hydroxy acid or unsaturated acid. For example,

H H H
CN– HCl
C O Æ C OH Æ C COOH
H+ heat
CN OH
NO2 NO2 NO2
m-nitrobenzaldehyde m-nitromandelic acid

CN COOH COOH
CN– H+
CH3CH2 CCH3 Æ CH3CH2CCH3 Æ CH3CH2CCH3 Æ CH3 CH = C CH3
H+ heat 2-methyl-2-butenoic acid
O OH OH
2-butanone

Addition of Ammonia
Aldehydes react with ammonia to give unstable aldehyde-ammonia which on eliminating water gives imines.

OH
H+ – H2O
R C HO + NH3 R CH NH2 RCH NH
an imine
Organic Compounds Containing Oxygen (Alcohols, Phenols, Ethers, Aldehydes, Ketones, Carboxylic Acids and their Derivatives) 24.39

Unsubstituted imines formed from NH3 are unstable and polymerized on standing.

R NH R
3RCH == NH
HN NH

R
If a primary amine (RNH2) is used instead of ammonia, a more stable, substituted imines (known as Schiff bases) is
formed.
Formaldehyde does not form imine but gives hexamethylenetetramine, which is used as a urinary antiseptic under
the name urotropine.
6HCHO + 4NH3 æÆ (CH2)6N4 + 6H2O
The structure of hexamethylenetetramine is
N
CH2
N
CH2 CH2
N N

Aromatic aldehydes (such as benzaldehyde) or arylamines (such as aniline) gives the most stable imines, but other
aldehydes, ketones or primary amines may be used.
Addition of Derivatives of Ammonia
The derivatives of ammonia which add on the carbonyl group are hydroxylamine, hydrazine, phenylhydrazine and
semicarbazide. Their stable forms are available in the form of salts; hydroxylamine hydrochloride (HONH +3Cl–),
phenylhydrazine hydrochloride (C6H5NHNH3+ Cl–) and semicarbazide hydrochloride (NH2CONHNH3+Cl–). The addition
of sodium acetate releases free base in solution which adds on the carbonyl group as follows.

H+
C O + NH2OH Æ C NHOH Æ C NOH + H2O
hydroxylamine oxime
OH

H+ Æ
C O + NH2NHC6H5 Æ C NHNHC6H5 C NNHC6H5 + H2O
phenylhydrazine phenylhydrazone
OH

H+
C O + NH2NHCONH2 Æ C NHNHCONH2 Æ C NNHCONH2 + H2O
semicarbazide OH semicarbazone

C O + H2N NH2 Æ C N NH2


hydrazone
The adjustment of pH (about 3 to 4) is necessary as in the low enough pH, bases may form protonated species
+
H3N—G which inhibits the addition as these are no longer nucleophilic reagents. On the other hand, protonation of
carbonyl oxygen makes the molecule more susceptible to nucleophilic attack at positively charged carbon.
The addition products, especially with 2,4–dinitrophenylhydrazine, are used to characterize the aldehyde or ketone
as they have characteristic melting points.
24.40 Complete Chemistry—JEE Main

Many aromatic aldoximes and ketoximes exist in two isomeric forms—cis- and trans-forms:

Ar R Ar R
C C
and
N N
OH OH
Aromatic ketoximes undergo Beckmann rearrangement when treated with PCl5, H2SO4, etc. The exchange occurs in
anti-position and the product obtained is N-substituted acetamide.

Ar C R O C R
PCl5
N Æ NH
OH
Ar

Addition of Alcohols
Aldehydes react with anhydrous ethanol in the presence of anhydrous acids (usually dry HCl) to form acetal. The
reaction taking place is

H H H
HCl ROH
R¢ C O + ROH R¢ C OR R¢ C OR + H2O
OH OR
hemiacetal acetal
Hemiacetals are too unstable to be isolated. Acetals are readily cleaved by acids and are stable towards bases:

H H
+
R¢ C OR + H2O HÆ R¢ C O + 2ROH
OR
Cannizzaro Reaction
This reaction is shown by aldehydes containing no a-hydrogen. In the presence of concentrated alkali, two molecules
of an aldehyde undergo self-oxidation-and-reduction to yield a mixture of an alcohol and a salt of a carboxylic acid.
For example,

50% NaOH
2HCHO Æ CH 3 OH HCOO Na +
methanol sodium formate
In the crossed Cannizzaro reaction, two different aldehydes are involved yielding all possible products. However, if
one of the reactants is formaldehyde, it is always converted into sodium formate. For example,

CHO CH2OH

conc. NaOH
+ HCHO Æ + HCOO–Na+

OCH3 OCH3
anisaldehyde p-methyoxybenzyl alcohol
(p-methyoxybenzaldehyde)
The Cannizzaro reaction involved two successive additions as depicted in the following.
Organic Compounds Containing Oxygen (Alcohols, Phenols, Ethers, Aldehydes, Ketones, Carboxylic Acids and their Derivatives) 24.41

The rate of reaction is decided by the ease with which H– is transferred.


The molecule of the type PhCOCHO shows internal crossed-Cannizzaro reaction.

Aldol Condensation
This reaction is shown by aldehydes or ketones containing a-hydrogen. In the presence of a dilute solution of a base,
a-carbon of one molecule gets attached to the carbonyl carbon of the second molecule. For example,
H H H H H H
OH–
CH3 C O H C C O Æ CH3 C C C O
H OH H
aldol
The mechanism involved is as follows.
(a) Formation of carbanion—a nuclephilic reagent 
CH3CHO + OH–  


 H2O + CH2CHO
H H
(b) Attack of carbanion to the second molecule CH3 C O + -CH2CHO CH3 C CH2CHO

O-
H H
(c) Transfer of H+ from water CH3 C CH3 C
O OH
-
The product obtained in aldol reaction is b-hydroxyaldehyde or ketone. This is very easily dehydrated to form a
double bond at a- and b-carbon atoms. For example,
H H H H
dil HCl
CH3 C CH C O Æ CH3 C CH C O + H 2O
warm crotonaldehyde
OH H
The resultant molecule contains a double bond in conjugation with the carbonyl group and it thus acquires extra
stability. If the double bond is also in conjugation with an aromatic ring, the product is so stable that the product obtained
in aldol reaction is unsaturated aldehyde or ketone instead of b-hydroxyaldehyde or ketone. For example,
24.42 Complete Chemistry—JEE Main

NaOC2H5
C CH3 + CH3 C (base)
O O
CH3 CH3
–H2O
C CH C Æ C CH C
O OH H O
1,3-diphenyl-2-butene-1-one
A crossed aldol condensation involves two different carbonyl compounds yielding a mixture of all the four possible
products. However, a single product is obtained as shown in the following.
Aldehyde containing no a-hydrogen (e.g. benzaldehyde or formaldehyde)

H H H
CH3CHO
For example, CHO + OH– Æ C C C O

cinnamaldehyde

Intramolecular Aldol Condensation


Properly constituted diketons give cyclic intramolecular aldol addition products.
O O

6 5 4 3 2 1 OH- - H2O -H2O


CH3CCH2CH3CCH3 CH3CCH2CH2CCH2
-H2O -OH
OH
O O O O CH3 CH3

O O
C CH3 COCH3
- H2O -H2O
CH3CCH2CH2CH2CHCCH3
-OH
OH
O O CH3 CH3

Keto-Enol Tautomerism
A carbonyl compound with an acidic alpha hydrogen may exist in two forms called tautomers: a keto tautomer and an
enol tautomer. The keto tautomer has the expected carbonyl group while the enol (from – ene + – ol) tautomer contains
vinylic hydroxyl group which is formed by transfer of an acidic hydrogen from the a-carbon to the carbonyl oxygen.
Most simple aldehydes and ketones exist primarily in their keto forms. However, 2-4-pentanedione exists in 80%
enol from due to the formation of six membered ring involving the internal hydrogen bonding.
Organic Compounds Containing Oxygen (Alcohols, Phenols, Ethers, Aldehydes, Ketones, Carboxylic Acids and their Derivatives) 24.43

Distinction between Aldehydes and Ketones


Aldehydes show the following characteristic tests. Ketones do not show these tests.
Tollens Test The Tollens reagent is an ammoniacal solution of silver nitrate. Aldehydes reduce Tollens reagent to a
bright silver mirror.
Fehling’s Test Fehling’s solution consists of an equimolar mixture of Fehling’s solution A and Fehling’s solution
B. Fehling’s solution A is CuSO4 solution and Fehling’s solution B is a mixture of Rochelte salt (sodium potassium
tartarate) and sodium hydroxide. Aliphatic aldehydes (but not aromatic) reduces Fehling’s solution to red brown cuprous
oxide.
Schiff’s Test Schiff’s reagent is rosaniline hydrochloride solution in water decolourized with SO2. Aldehydes when
warmed with Schiff’s reagent restore pink colour of the reagent.

MULTIPLE CHOICE QUESTIONS ON SECTION 4

General Characteristics
1. Which of the following is used in preservation of biological specimens?
(a) CH3CHO (b) CH3COOH (c) CH3OCH3 (d) HCHO
2. The dry distillation of calcium formate gives
(a) CH3CHO (b) CH3COCH3 (c) HCHO (d) HCOOH
3. The dry heating of a mixture of calcium acetate and calcium formate produces
(a) CH3CHO (b) CH3COCH3 (c) CH3COOH (d) HCHO
4. Which of the following order of boiling point is correct?
(a) Butane > Butan-1-al > Butan-1-ol (b) Butane < Butan-1-al < Butan-1-ol
(c) Butane > Butan-1-al < Butan-1-ol (d) Butane < Butan-1-al > Butan-1-ol
5. Formalin is an aqueous solution of
(a) 40% formaldehyde (b) 50% formaldehyde (c) 60% formaldehyde (d) 70% formaldehyde
6. The structural formula of metaformaldehyde is
(a) (CH2O)2 (b) (CH2O)3 (c) (CH2O)4 (d) (CH2O)n
7. Polarization of electrons in acrolein may be written as
d– d+ d– d+ d– d+ d+ d–
(a) CH ==CH—CH==O (b) CH2==CH—CH==O (c) CH ==CH—CH==O (d) CH2==CH—CH==O
2 2

8. Which of the following statements regarding aldehydes is not correct?


(a) The carbonyl functional group is planar
(b) The boiling points of aldehydes is greater than those of alkanes of comparable molar masses
(c) The boiling points of aldehydes is greater than those of alcohols of comparable molar masses
(d) Formaldehyde is gas at room temperature
9. Which of the following has the most acidic hydrogen?
(a) 3-Hexanone (b) 2, 4-Hexanedione (c) 2, 5-Hexanedione (d) 2, 3-Hexanedione
10. The enol form of acetone, after treatment with D2O, gives
(a) CH 3 —C CH 2 (b) CH 3 —C—CH 3 (c) CH 2 C—CH 2 D (d) CD 2 C—CD 3
| || | |
OD O OH OD
11. Which of the following statements regarding CO group is not correct?
(a) The carbon atom of CO group in aldehydes is sp2 hybridized
(b) The carbon atom of CO group in the transition state formed during the addition reaction across CO group is
sp3 hybridized
24.44 Complete Chemistry—JEE Main

(c) The aryl group in aromatic aldehydes speeds up the addition reaction across the CO group
(d) An aryl group stabilises an aldehyde more than the transition state
12. Which of the following statements is correct?
(a) The structure of the enol form of CH 3COCH 2COCH 3 with intramolecular hydrogen bonding
is CH3 C CH C CH3

O O
H
(b) Carbonyl carbon of an aldehyde or a ketone is sp hybridized.
(c) Fomaldehyde is a liquid at room temperature.
(d) Formaline is 30% aqueous solution of formaldehyde.
Oxidation and Reduction Reactions
Zn Hg
13. The reaction RCOR Æ RCH2R is known as
HCl
(a) Clemmensen reduction (b) Rosenmund reduction
(c) Wolff-Kishner reduction (d) Catalytic reduction
Pd-BaSO 4
14. The reaction RCOCl + H 2 Æ RCHO + HCl is known as
(a) Clemmensen reduction (b) Rosenmund reduction
(c) Wolff-Kishner reduction (d) Grignard reduction
NH 2 NH 2
15. The reaction C6H5COCH3 Æ C6H5CH2CH3 is known as
KOH
(a) Clemmensen reduction (b) Rosenmund reduction
(c) Wolf-Kishner reduction (d) Catalytic reduction
16. When acetaldehyde is heated with Fehling’s solution, it gives a precipitate of
(a) Cu (b) CuO (c) Cu2O (d) Cu + Cu2O + CuO
17. Which of the following compounds will yield methyl ethyl ketone on oxidation?
(a) 2-Propanol (b) 1-Butanol (c) 2-Butanol (d) tert-Butyl alcohol
18. The reagent which can be used to reduce —CHO to —CH2OH without affecting the double bond in the
unsaturated aldehydic molecule is
(a) Zn/HCl (b) LiAlH4 (c) 9-BBN (d) NaBH4
19. Which of the following statements is not correct?
(a) Toluene can be oxidized to benzaldehyde by using alkaline KMnO4.
(b) Phenolic aldehydes can be prepared by Reimer-Tiemann reaction.
SO–3 Na +
|
(c) NaHSO3 reacts with RCHO in ethanol to give solid adduct having structure R—C—OH.
|
H
(d) The formation of oximes is hindered at the very low pH.
Nucleophilic Addition Reactions
20. Hydroxylamine reacts with aldehyde and ketone to give
(a) aldehyde ammonia (b) oximes (c) carboxylic acid (d) alkyl cyanide
21. The reaction of formaldehyde with ammonia produces
(a) an amine (b) an amide
(c) aldehyde ammonia (d) hexamethylenetetramine
22. Which reagent on condensation with benzaldehyde produces phenylhydrazone?
(a) NH2OH (b) C6H5NH2 (c) H2NNHCONH2 (d) C6H5NHNH2
Organic Compounds Containing Oxygen (Alcohols, Phenols, Ethers, Aldehydes, Ketones, Carboxylic Acids and their Derivatives) 24.45

23. The pH of the solution for the formation of addition products of carbonyl group and ammonia derivatives should
be about
(a) 0 to 1 (b) 2 to 3 (c) 3 to 4 (d) 7 to 8
24. An oxime is formed when an aldehyde/ketone reacts with
(a) hydroxylamine (b) hydrazine (c) phenylhydrazine (d) semicarbazide
25. The semicarbazone is formed when an aldehyde/ketone reacts with
(a) H2NOH (b) H2NNH2 (c) H2NNHC6H5 (d) H2NNHCONH2
26. Which of the following is an example of Perkin condensation?
KOH
(a) C6H5CHO + CH3NO2 Æ C6H5CH = CHNO2 + H2O
dry HCl
(b) 2CH3COCH3 Æ (CH3)2C = CHCOCH3
3OH + HCOONa
CH 3 COONa
(d) C6H5CHO + (CH3CO)2O Æ C6H5CH = CHCOOH
27. Which of the following represents Claisen-Schmidt reaction?
NaOH
(a) CH3CHO + CH3CHO Æ CH3CH = CHCHO + H2O
NaOH
(b) C6H5CHO + CH3CHO Æ C6H5CH = CHCHO + H2O
NaOH
(c) CH3CHO + CH3CHO Æ CH3CH = CHCH3 + O2
NaOH
(d) C6H5CHO + C6H5CHO Æ C6H5CH = CHC6H5 + O2
28. Which of the following reactions represents Tischenko reaction?
K 2 CO 3
(a) 2CH3 3COOH + CH3CH2OH (b) 2CH3CHO Æ CH3CHCH2CHO
OH
Al(OC 2 H 5 ) 3 NaOH
(c) 2CH3CHO Æ CH3CO2C2H5 (d) 2CH3CHO Æ CH3CH = CHCHO + H2O

29. Acetone on reacting with CH3MgBr followed by hydrolysis produces


(a) n-butyl alcohol (b) sec-butyl alcohol (c) isobutyl alcohol (d) tert-butyl alcohol
30. Which of the following compounds will react with ethanolic KCN?
(a) Ethyl chloride (b) Acetyl chloride (c) Chlorobenzene (d) Methane
31. The formation of cyanohydrin from a ketone is an example of
(a) electrophilic addition (b) nucleophilic addition
(c) nucleophilic substitution (d) electrophilic substitution
32. Which of the following statements regarding aldehydes is not correct?
(a) The addition reactions occurring across the C==O bond is electrophilic type
(b) The addition reactions occurring across the C==O bond is nucleophilic type
(c) Aldehydes undergo addition reactions more readily than ketones
(d) The addition reactions shown by CO group is catalyzed by acids
33. Which of the following statements regarding CO group is not correct?
(a) The carbon atom of CO group in aldehydes is sp2 hybridized
(b) The carbon atom of CO group in the transition state formed during the addition reaction across CO group is
sp3 hybridized
(c) The aryl group in aromatic aldehydes speeds up the addition reaction across the CO group
(d) An aryl group stabilises an aldehyde more than the transition state
| | | | | |
34. The reaction —C==C—C==O + HCN Æ —C—C—C==O involves
| |
CN H
24.46 Complete Chemistry—JEE Main

(a) electrophilic addition across double bond (b) nucleophilic addition across double bond
(c) free radical addition across double bond (d) no effect of the CO group on the C==C bond
35. Among the given compounds, the most susceptible to nucleophilic attack at the carbonyl group is
(a) CH3COCl (b) CH3CHO (c) CH3COOCH3 (d) CH3COOCOCH3
36. Which of the following statements is not correct?
(a) In pinacol-pinacolone rearrangement reaction involving, PhC(CH 3 )—CPh(CH 3 ) , the phenyl group migrates
| |
in preference to methyl group. HO OH

(b) In pinacol-pinacolone rearrangement reaction involving Me2 C—CH 2 , the 3° OH is lost, followed by :H
| |
migration. OH OH
(c) Schiff base is N-substituted imine.

R OH conc. H2SO4
(d) In the Beckmann rearrangement reaction C == N A , the product A is R¢CONHR.
R
Aldol and Cannizzaro Reactions
37. Aldol condensation is shown by an aldehyde containing
(a) no a-carbon atom (b) no a-hydrogen atom (c) an a-hydrogen atom (d) two a-hydrogen atoms
38. Cannizzaro’s reaction is not shown by an aldehyde containing
(a) no a-carbon atom (b) an a-carbon atom (c) no a-hydrogen atom (d) an a-hydrogen atom
39. Base catalyzed aldol condensation occurs with
(a) trimethylacetaldehyde (b) benzaldehyde (c) acetaldehyde (d) formaldehyde
40. The Cannizzaro reaction is not given by
(a) trimethylactaldehyde (b) benzaldehyde (c) acetaldehyde (d) formaldehyde
41. Base catalysed aldol condensation occurs with
(a) propanal (b) benzaldehyde (c) 2-methylpropanol (d) 2,2-dimethylpropanal
42. m-Chlorobenzaldehyde on reaction with concentrated KOH at room temperature gives
(a) potassium m-chlorobenzoate and m-hydroxybenzaldehyde
(b) m-hydroxybenzaldehyde and m-chlorobenzyl alcohol
(c) m-chlorobenzyl alcohol and m-hydroxybenzyl alcohol
(d) potassium m-chlorobenzoate and m-chlorobenzyl alcohol
43. Which of the following compounds does not undergo aldol condensation?
(a) C6H5CH2CHO (b) C6H5CHO (c) CH3COCH3 (d) CH3CH2CHO
– OH
44. In the Cannizzaro reaction 2Ph—CHO Æ Ph—CH2OH + PhCO–2 , the slowest step is

(a) the attack of OH at the carbonyl group,
(b) the transfer of hydride to the carbonyl group,
(c) the abstraction of proton from the carboxylic acid,
(d) the deprotonation of Ph—CH2OH
45. Which of the following will not undergo aldol condensation? (1998)
(a) Acetaldehyde (b) Propanaldehyde
(c) Benzaldehyde (d) Trideuteroacetaldehyde
46. Aldol condensation will be observed in
(a) CH3CH2CH2CHO (b) CHO (c) CHO (d) CHO
Organic Compounds Containing Oxygen (Alcohols, Phenols, Ethers, Aldehydes, Ketones, Carboxylic Acids and their Derivatives) 24.47

47. Which of the following statements is not correct?


(a) Acetamide on heating with P2O5 produces methanenitrile.
(b) The structural formula of meta formaldehyde is (CH2O)3.
(c) Benzaldehyde undergoes Cannizzaro reaction whereas acetaldehyde does not.
(d) The reaction of formaldehyde with ammonia produces hexamethylenetetramine.
48. Which of the following statements is correct?
(a) Benzaldehyde undergoes aldol condensation in an alkaline medium.
(b) Hydrazones of aldehydes and ketones are prepared in highly acidic medium.
(c) The reaction of methyl magnesium iodide with acetone followed by hydrolysis gives secondary butanol.

49. Which of the following statements is not correct?


H H OH
(a) The equilibrium constant of the hydration reaction C == O + H2O C
H H OH
CH3 CH3 OH
is lower than that of the reaction C == O + H2O C
CH3 CH3 OH
Cl Cl OH
| | |
(b) The equilibrium Cl—C—C O + H2 O Cl—C—C — OH, lies more to the right side.
| | | |
Cl H Cl H
(c) H2CO is always oxidized to HCOOH in the crossed-Cannizzaro reactions.
(d) The species PhCOCHO undergoes Cannizzaro reaction to give PhCH(OH)COO–.
Distinctive Tests
50. Which of the following gives a silver mirror with Tollens reagent?
(a) CH3CH2OH (b) CH3CHO (c) CH3COOH (d) CH3CH2OCH2CH3
51. Which of the following compounds does not exhibit positive iodoform test?
(a) Acetaldehyde (b) Methanal (c) Propanone (d) Ethanol
52. Which of the following compounds undergoes the Cannizzaro reaction?
(a) Acetaldehyde (b) Acetone (c) Benzaldehyde (d) Propanal
53. Benzaldehyde differs from acetaldehyde in this respect that the former does not react with
(a) Tollens reagent (b) Fehling’s solution (c) HCN (d) NaHSO3
54. Which of the following tests distinguishes acetophenone from propionaldehyde?
(a) Baeyer test (b) Iodoform test (c) Tollens test (d) Lucas test
55. The reagent which reacts both acetaldehyde and acetone is
(a) Tollens reagent (b) Fehling’s reagent (c) Grignard reagent (d) Schiff’s reagent
56. Acetophenone differs from aliphatic ketones in that it does not react with
(a) HCN (b) NaHSO3 (c) I2 and NaOH (d) NH2OH
57. The compound that gives a positive iodoform test is
(a) 1-pentanol (b) 2-pentanone (c) 3-pentanone (d) pentanal
58. Which of the following compounds will give a yellow precipitate with iodine and alkali?
(a) 2-Hydroxypropane (b) Benzophenone (c) Methylacetate (d) Acetamide
59. The compound that will not give iodoform on treatment with alkali and iodine is
(a) acetone (b) ethanol (c) diethyl ketone (d) ethyl methyl keton
60. Which of the following compounds does not exhibit(s) positive iodoform test?
(a) Acetaldehyde (b) Methanol (c) Ethanol (d) Propanone
24.48 Complete Chemistry—JEE Main

ANSWERS
1. (d) 2. (c) 3. (a) 4. (b) 5. (a) 6. (b)
7. (d) 8. (c) 9. (b) 10. (a) 11. (c) 12. (a)
13. (a) 14. (b) 15. (c) 16. (c) 17. (c) 18. (c)
19. (a) 20. (b) 21. (d) 22. (d) 23. (c) 24. (a)
25. (d) 26. (d) 27. (b) 28. (c) 29. (d) 30. (a)
31. (b) 32. (a) 33. (c) 34. (b) 35. (a) 36. (d)
37. (c) 38. (d) 39. (c) 40. (c) 41. (a) 42. (d)
43. (b) 44. (b) 45. (c) 46. (a) 47. (a) 48. (d)
49. (a) 50. (b) 51. (b) 52. (c) 53. (b) 54. (b)
55. (c) 56. (b) 57. (b) 58. (a) 59. (c) 60. (b)

HINTS AND SOLUTIONS


1. HCHO is used as preservative.
2. Dry distillation of calcium formate gives formaldehyde.
3. (CH3COO)2Ca + (HCOO)2 3CHO + 2CaCO3
4. Boiling point of alkane is less than the corresponding aldehyde, which in turn, is less than the alcohol.
5. 40% formaldehyde is known as formaline.
6. Metaformaldehyde is (CH2O)3.
8. Aldehydes have boiling points lower than those of alcohols of comparable molar masses.
O O
9. 2,4-Hexanedione is H3C C CH2 C CH2 CH3 . Methylene hydrogens attached in between two C==O
group will have maximum acidic nature.
D2O
10. CH3 C CH3 CH3 C CH2 Æ CH3 C CH2
O OH OD
11. The aryl group in aromatic aldehyde retards the addition reaction across the CO group.
12. (b) Carbon is sp2 hybridized.
(c) Formaldehyde is a gas at room temperature.
(d) Formaline is 40% aqueous solution of formaldehyde.
13. The reduction with Zn/Hg-HCl is known as Clemmensen reduction.
14. The reduction with Pd-BaSO4 is known as Rosenmund reduction.
15. The reduction with NH2NH2/KOH is known as Wolf-Kishner reduction.
16. Red precipitates are of Cu2O.
[O]
17. CH3CHOHCH2CH3 Æ CH3COCH2CH3
18. 9-BBN (9-borabicyclo[3, 3, 1]nonane) can be used.
19. (a) With an alkaline KMnO4, toluene is converted into benzoic acid. However, the use of CrO3 in acetic
anhydride followed by hydrolysis will convert toluene to benzaldehyde.
(b) At low pH, protonated aldehydes, ketones and hydroxylamines are formed.
20. Hydroxylamine forms oximes.
21. The reaction of formaldehyde with ammonia produces hexamethylenetetramine.
6HCHO + 4NH3 æÆ (CH2)6N4 + 6H2O
Organic Compounds Containing Oxygen (Alcohols, Phenols, Ethers, Aldehydes, Ketones, Carboxylic Acids and their Derivatives) 24.49

22. Phenylhydrazine (C6H5NHNH2) produces phenylhydrazone.

24. Hydroxylamine produces oxime.


25. Semicarbazide (H2NNHCONH2) forms semicarbazone.
CH3 CH3
H2O
29. CH3COCH3 + CH3MgBr Æ CH3 C OMgX Æ CH3 C OH
CH3 CH3
tert-Butyl alcohol
30. Ethyl chloride will give CH3CH2CN.
31. Addition across C==O group occurs via nucleophilic mechanism.
32. Same as Q. 31.
33. Aryl group lowers the speed of addition reaction across the C==O group due to the stabilization of the reactant
more than the transition state via resonant effect.
34. Same as Q. 31.
35. The positive charge on carbon is enhanced due to electron-with drawing nature of cholrine.
CH 3CH 3 CH 3
| | conc H 2 SO 4 |
36.(a) The reaction is Ph—C—C— Ph Æ Ph—C—C—CH 3
| | | ||
OH OH Ph O
CH 3 CH 3
| |
(b) The reaction is CH 3 —C—CH 2 Æ CH 3 —CH—CHO
| |
OH OH
– H 2O
(c) The general reaction for the formation of a Schiff base is RCHO + R¢NH2 Æ RCH N—R¢
(d) It is the trans R¢ group which migrates

37. Aldol condensation is shown by an aldehyde containing an a-hydrogen atom.


38. Cannizzaro’s reaction is not shown by an aldehyde containing an a-hydrogen atom.
39. Acetaldehyde (CH3CHO) shows aldol condensation, as it contains a-hydrogen atoms.
40. CH3CHO contains a-hydrogen atom. It will not show Cannizzaro reaction.
41. Only propanal contains a-hydrogen atoms. It will show aldol condensation.
42. m-Chlorobenzaldehyde undergoes Cannizzaro reaction.
44. Transfer of hydride to carbonyl group is the slowest step.
45. Compounds containing a-hydrogen (or deuterium) undergo aldol condensation. Acetaldehyde, propanaldehyde
and trideuteroacetaldehyde will undergo aldol condensation.
48. (a) Benzaldehyde does not possess a-hydrogen atom. It does not undergo aldol condensation but undergoes
Cannizzaro reaction giving benzoic acid and benzyl alcohol.
(b) Hydrazine gets protonated in highly acidic medium. Due to this, its nucleophilicity is decreased.
(c) The product is tertiary butanol.

(d) Carbonyl carbon is sp2 hybridized.


24.50 Complete Chemistry—JEE Main

49. (a) The equilibrium constant for the hydration of H2CO is greater than that of (CH3)2CO. Two factors responsible
for this trend are steric and inductive. Firstly, the addition of H2O involves the change in the hybridization
of C from sp2 to sp3. The alkyl groups in (CH3)2CO will involve more steric hinderance and thus its
hydration will be less effective than that of H2CO. Secondly, the methyl group is an electron-releasing group.
The presence of two methyl groups attached to C of CO group in (CH3)2CO diminishes the positive charge on

C causing the decrease in the reactivity of nucleophilic addition of H2O across .

(b) The chloral is less stable than chloral hydrate due to d+ charge on the C adjacent to the carbonyl C.

(c) H2CO is the most reactive aldehyde. It exists in aqueous OH– solution mainly as the conjugate base of its
hydrate H2C(OH)O–. The hydride shift from the anion occurs more readily giving formic acid.
(d) PhCOCHO undergoes internal crossed-Cannizzaro reaction.
50. Aldehyde (CH3CHO in this case) reduces Ag+ in Tollens reagent to give silver mirror.
51. Iodoform test is shown by a compound CH3CO— group or by a compound which produces this group. Methanal
(HCHO) does not show iodoform test.
52. Benzaldehyde does not show Cannizzaro reaction.
53. Benzaldehyde does not react with Fehling’s solution.
54. CH3CH2CHO does not containing CH3CO— group. Thus, iodoform test can distinguish between CH3CH2CHO
and CH3COC6H5.
55. Grignard reagent react both with CH3CHO and CH3COCH3.
56. CH3COC6H5 does not react with NaHSO3.
57. 2-Pentanone contains CH3CO— group. It will show iodoform test.
58. 2-Hydroxypropane will give CH3COCH3 which, in turn, will show iodoform test.
59. Diethyl ketone will not show iodoform test as it does not contain CH3CO— group.
60. Iodoform test will be shown if the compound has CH3CO— group.

MULTIPLE CHOICE QUESTIONS FROM AIEEE AND JEE MAIN

1. Which of the following on heating with aqueous KOH produces acetaldehyde?


(a) CH2Cl CH2Cl (b) CH3CHCl2 (c) CH3COCl (d) CH3CH2Cl [2009]
2. In Cannizzaro reaction given below:

OH
2PhCHO :
Æ PhCH2OH + PhCO2–
The slowest step is
(a) The abstraction of proton from the carboxylic group
(b) The deprotonation of PhCH2OH
(c) The attack of –: OH at the carboxylic group
(d) The transfer of hydride to the carbonyl group [2009]
3. Identify the compound that exhibits tautomerism.
(a) Phenol (b) 2-Butene (c) Lactic acid (d) 2-Pentanone
[2011 cancelled]
Organic Compounds Containing Oxygen (Alcohols, Phenols, Ethers, Aldehydes, Ketones, Carboxylic Acids and their Derivatives) 24.51

4. Trichloroacetaldehyde was subjected to Cannizaro’s reaction by using NaOH. The mixture of the products contains
sodium trichloroacetate and another compound. The other compound is
(a) chloroform (b) 2,2,2-trichloroethanol (c) trichloromethanol (d) 2,2,2-trichloropropanol
[2011 cancelled]
5. In the given transformation, which of the following is the most appropriate reagent?
CH CHCOCH3
CH CHCH2CH3
Reagent

HO HO
(a) NH2NH2, OH– (b) Zn-Hg/HCl (c) Na, liq. NH3 (d) NaBH4 [2012]
6. The most stable reagent for the conversion of RCH2OH to RCHO is
(a) PCC (pyridinium chlorochromate) (b) KMnO4
(c) K2Cr2O7 (d) CrO3 [2014]
7. An organic compound A, C5H8O; reacts with H2O, NH3 and CH3CHOOH as described below:
O
H2O
CH C OH
O
NH3
A CH C NH2
O O
CH3COOH
CH C O C CH3
A is:
(a) CH3CH C CHO (b) CH2 CHCH CHO
CH3 CH3
(c) CH3 CH2 C C O (d) CH3 CH2 C C O [2014]

CH3 CH2 H

(a) Aldol condensation (b) Claisen condensation


(c) Cannizzaro reaction (d) Pinacol-pinacolon reaction [2014]
9. Which one of the following reactions will not result in the formation of carbon - carbon bond ?
(a) Reimer-Tieman reation (b) Friedel Craft,¢s acylation
(c) Wurtz reaction (d) Cannizzaro reaction [2014]
10. Which is the major product formed when acetone is heated with iodine and potassium hydroxide ? [2014, online]
(a) Iodoacetone (b) Acetic acid (c) Iodoform (d) Acetophenone
11. A compound A with molecular formula C10H13Cl gives a white precipitate on adding silver nitrate solution. A on
reacting with alcoholic KOH gives compound B as the main product. B on ozonolysis gives Cannizaro reaction
but not aldol condensation. D gives aldol condensation but not Cannizaro reaction. A is:
CH3

(a) C6H5 — CH2 — C (b) C6 H5 -- CH 2 -- CH 2 -- CH -- CH3
CH3

Ω
Cl Cl
CH2 —CH2 —CH3
CH2Cl
(c) C6H5 — CH2 — CH2 — CH2 — CH2 — Cl (d) [2015, online]
24.52 Complete Chemistry—JEE Main

12. Which of the following pairs of compounds are positional isomers?


(a) CH3 -- CH 2 -- CH 2 -- CH 2 -- CHO and CH3 -- CH 2 -- CH 2 -- C -- CH3
ΩΩ
O
(b) CH3 -- CH 2 -- CH 2 -- C -- CH3 and CH3 -- CH -- CH 2 -- CHO
ΩΩ Ω
O CH3

(c) CH3 -- CH 2 -- CH 2 -- C -- CH3 and CH3 -- CH 2 -- C -- CH 2 -- CH3


ΩΩ ΩΩ
O O
(d) CH3 -- CH 2 -- C -- CH3 and CH3 --CH -- CH 2 -- CHO [2015, online]
ΩΩ Ω
O CH3
13. In the reaction sequence
OH - D
2CH3CHO æææ Æ A ææ Æ B; the product B is:
(a) CH3 — CH2 — CH2 — CH2 — OH (b) CH3 — CH == CH — CHO
(c) CH3 -- C -- CH3 (d) CH3 — CH2 — CH2 — CH3 [2015, online]
ΩΩ
O
14. The correct statement about the synthesis of erythritol (C(CH2OH)4) used in the preparation of PETN is:
(a) The synthesis requires three aldol condensations and one Cannizzaro reaction.
(b) Alpha hydrogens of ethanol and methanol are involved in the reaction.
(c) The synthesis requires two aldol condensations and two Cannizzaro reactions.
(d) The synthesis requires four aldol condensations between methanol and ethanol [2016, online]

ANSWERS

1. (b) 2. (d) 3. (a) 4. (b) 5. (a) 6. (a)


7. (c) 8. (c) 9. (d) 10. (c) 11. (a) 12. (c)
13. (b) 14. (a)

HINTS AND SOLUTIONS

1. The reaction goes as follows.

2. The mechanism of the reaction involves the following steps.


Organic Compounds Containing Oxygen (Alcohols, Phenols, Ethers, Aldehydes, Ketones, Carboxylic Acids and their Derivatives) 24.53

H H H
– –
Ph C == O + Ph C O Ph C O + Ph C == O
OH H OH
fast + H+ fast – H
+

Ph C OH Ph C == O

H O

The shifting of hydride ion to the carbonyl group is the slowest step.
3. The tautomerism in 2-pentanone is
CH3 — C — CH2CH2CH3 CH3 — CH — CHCH2CH3

O OH
OH O
H
H
The tautomerism in phenol is . However, phenol mainly exists in hydroxy form.

4. The reaction is

5. The Wolff-Kishner reduction (which uses hydrazine, NH2NH2, and a strong base OH–) reduces only —C = O
group without affecting the —C = C— group.
6. Pyridinium chlorochromate is a suitable reagent to convert an alcoholic group to an aldehydic group. The rest of
the three are stronger oxidizing agent.
7. The reactions shown are those of a ketene.
(which have C C O group). The reactions.

CH3 CH2 C C O + H2O CH3 CH2 CH CO2H


CH3 CH3
(carboxylic acid)

CH3 CH2 C C O + NH3 CH3 CH2 CH CONH2


CH3 CH3
(amide) O O
CH3 CH2 C C O + CH3COOH CH3 CH2 CH C O C CH3
CH3 CH3
(anhydride)
8.
reaction by treatment with aluminium ethoxide. Under these conditions the acid and alcohol are combined to give
as ester. This reaction is known as Tischenko reaction. For example
aluminium
2 CH3 CHo ææææ
ethoxide
Æ CH3COOH + CH3CH 2 oH Æ CH3COOCH 2 CH3 ethyl acetate
9. Reimer - Tiemann Reaction
24.54 Complete Chemistry—JEE Main

OH O- OH
CHO CHO
CHCl3 H+
aq. NaOH 70°C

Friedal - Craft’s acylation


COCH3
CH3COCl
Al Cl3

Wurtz reaction
RBr + 2Na + BrR Æ R - R + 2NaBr
Cannizzaro reaction
50% NaCH
2 HCHO æææææÆ CH3OH + HCOO- Na
In Cannizzaro reaction, no C – C bond is formed
10. A compound containing CH3 C or CH3CH (OH)

O
- shows iodoform test. The reaction is
CH3COCH3 + 3NaOI RCOCI3 + 3NaOH
(from I2 and NaOH) (Iodoform)

CH3 CH3

alc. KOH
C6H5 — CH2 — C — CH3 C6H5 — CH — C — CH3

(B)
Cl
(A) Ozonolysis
CH3
Cl gives white precipitate

with AgNO3 C6H5CHO + O—


— C — CH3
(C) (D)
aldehyde ketone
no a-hydrogen contains a-hydrogen
shows Cannizzaro reaction shows aldol condensation

12. The compounds


CH3 -- CH 2 -- CH 2 -- C -- CH3 and CH3 -- CH 2 -- C -- CH 2 -- CH3 are positional isomers.
ΩΩ ΩΩ
O O
13. The reactions are:
2CH3 -- CHO Æ CH3 -- CH -- CH 2 -- CHO æDæ Æ CH3 -- CH == CH -- CHO
Ω (B)
OH
(A)
14. Erythritol is synthesised as follows. This requires three aldol condensations and one Cannizzaro reaction.

OH OH
HO
3HCHO HCHO
CH3CHO OHC C C
OH- OH-
OH OH
3 aldol Cannizzaro
condensation
HO reaction
HO
Organic Compounds Containing Oxygen (Alcohols, Phenols, Ethers, Aldehydes, Ketones, Carboxylic Acids and their Derivatives) 24.55

Additional Information
PETN stands for pentaerythritol tetranitrate. It is one of the most powerful explosive materials. It is produced
by treating erythritol with concentrated nitric acid.

OH ONO2
OH O2NO
HNO3
C C
OH ONO2
HO O2NO
(PETN)

SECTION 5 Carboxylic Acids

Methods of Preparation
KMnO 4
Oxidation of Primary Alcohols R — CH2OH Æ R — COOH

KMnO 4
Oxidation of Alkylbenzenes C6H 5— R Æ C6H5— COOH
or K 2 Cr2 O 7
+
Carbonation of Grignard Reagent R—MgX + O==C==O Æ R— COO — MgX HÆ R— COOH + Mg2+ + X–
RMgX may be prepared from RX by adding Mg.

acid
Hydrolysis of Nitriles R —C N + 2H2O Æ R— COOH + NH3
or base

Nitriles may be prepared by adding NaCN to the corresponding halides. For example,
H2O, H +
CH3CH2CH2Br + CN– Æ CH3CH2CH2CN Æ CH3CH2CH2COOH

The above substitution of X with CN is applicable only when the halide is a primary halide. Tertiary halides yield alkene
and for secondary halide, the yield of substitution product is poor.

CH3 CH3

CH3 C Br + CN Æ CH3 C CH2 + HCN
isobutylene
CH3
tert-butyl bromide

Aryl halides are unreactive towards substitution. Thus, aromatic nitriles are prepared via diazonium salt. For example,

NH2 N N+Cl– CN COOH

NaNO2 CuCN H+
HCl
Æ Æ Æ

Physical Properties
The molecules of carboxylic acids are polar and exhibit hydrogen bonding. The boiling point of a carboxylic acid is
higher than that of an alcohol of comparable molar mass. This is due to the fact that the carboxylic acids exist as dimer.
24.56 Complete Chemistry—JEE Main

O ....H O
R C C R
O H .... O

Chemical Reactions
Acidity
Carboxylic acids are weak acids and their carboxylate anions are strong conjugate bases. The aqueous solutions of
carboxylate salts are slightly alkaline due to the hydrolysis of carboxylate anion. Compared to other species, the orders
of acidity and basicity of corresponding conjugate bases are as follows.
Acidity RCOOH > HOH > ROH > HC CCH > NH3 > RH
Basicity –
RCOO < HO < RO < HC –
CC < NH–2 < R–
– –

The carboxylic acids react with metals to liberate hydrogen and are soluble in both NaOH and NaHCO3 solutions.
For example

2CH3 3COO Na+ + H2

CH3 3COO Na + H2O

CH3COOH + NaHCO3 3COO Na+ + H2O + CO2
The acidity of a carboxylic acid is due to the resonance stabilization of its anion:

O O– O
R C R C equivalent to R C

O O O
Because of the resonance, both the carbon-oxygen bonds in the carboxylate anion have identical bond length. In the
carboxylic acid, these bond lengths are no longer identical. For example, in formic acid, one carbon-oxygen bond length
is 136 pm (single bond) and another of 123 pm (double bond), whereas in the formate anion, both carbon-oxygen bonds
have length equal of 127 pm (in between 123 pm and 136 pm).
The acidity of carboxylic acid depends very much on the substituent attached to —COOH group. Since acidity is due
to the resonance stabilization of anion, any substituent which stabilizes the anion increases acidity whereas substituent
causing destablization of anion decreases acidity. For example, electron-withdrawing group disperses the negative
charge of the anion and hence makes it more stable causing the increase in the acidity of the corresponding acid. On the
other hand, electron-releasing group increases the negative charge on the anion and hence makes it less stable causing
the decrease in the acidity. In the light of this, the following are the decreasing order of a few substituted carboxylic
acids.
1. Increase in the number of chlorine atoms on a-position increases the acidity, e.g.
Cl3CCOOH > Cl2CHCOOH > ClCH2COOH > CH3COOH
2. Increase in the distance of Cl from —COOH decreases the acidity, e.g.
CH3CH2CHCOOH > CH3CHCH2COOH > CH2CH2CH2COOH
Cl Cl Cl
This is due to the fact that inductive effect decreases with distance.
3. Increase in the electronegativity of halogen increases the acidity.
FCH2COOH > BrCH2COOH > ICH2COOH .
The aromatic acids are similarly affected by substituent.
Electron-releasing groups (—CH3, —OH, —NH2) make benzoic acid weaker.
Electron-withdrawing groups (—Cl, —NO2) make benzoic acid stronger.
Organic Compounds Containing Oxygen (Alcohols, Phenols, Ethers, Aldehydes, Ketones, Carboxylic Acids and their Derivatives) 24.57

Conversion to Acid Chloride


This may be carried out by using thionyl chloride (SOCl2), phosphorus trichloride (PCl3) and phosphorus pentachloride
(PCl5). Thionyl chloride is more convenient as the side products are only gaseous and thus the acid chloride can be
easily separated; any excess of SOCl2 can be easily removed as its boiling point is low (79°C).
RCOOH + SOCl 2 Æ RCOCl + SO 2 + HCl
reflux
RCOOH + PCl 5 Æ RCOCl + POCl 3 + HCl
heat

SOCl 2 R¢OH
Conversion into Esters RCOOH Æ RCOCl Æ RCOOR
acid acid chloride ester

A direct reaction between acid and alcohol is a reversible one. Ester can be obtained either by using one of the
reactants in excess or by removing one of the products.
H+
RCOOH + R¢OH RCOOR¢ + H2O
+
H acts as a catalyst.

CH3OH > 1° > 2° > 3°


HCOOH > CH3COOH > RCH 2COOH > R 2CHCOOH > R3CCOOH
O OH OH
+ R¢O H +
Mechanism R C OH + H R C + R C OR
OH OH H

O OH OH
+
R C OR¢ + H R C + H2O R C OR¢
OR ¢ +
OH2
Mineral acid speeds up both processes by protonating carbonyl oxygen and thus rendering carbonyl carbon more
¢OH and leaving group is water and in the
hydrolysis, the roles are reversed. Note that all steps are reversible. Forward reaction is favoured by H+ and the reverse
reaction by excess of water.
Reduction of Acids to Alcohol
Lithium aluminium hydride, LiAlH4, is used to convert acids into alcohols. The initial product is an alkoxide which on
hydrolysis gives an alcohol.
4R— COOH + 3LiAlH 4 Æ 4H2 + 2LiAlO2 + (RCH2O)4 AlLi
H O
(RCH2O)4AlLi 2
Æ 4RCH2OH
Alternatively, an ester may be converted into alcohol by the use of sodium metal and alcohol or LiAlH4. For example,
(i) LiAlH 4
CH3(CH2)14COOC2H 5 Æ CH3(CH2)14CH2OH + C2H5OH
ethyl palmitate (ii) H

The reagent BH3/THF followed by H3O+ also reduces —COOH to —CH2OH.


If the compound contains —CO—, —NO2 and —Br groups, LiAlH4 cannot be used as these are also reduced alongwith
—COOH group. However, BH3/THF can be used as this does not affect these groups. If the compound contains C==C
group, then BH3/THF cannot be used as this is rapidly reduced. However, LiAlH4 can be used.
Halogenation of Aliphatic Acids (Hell-Volhard-Zelinsky reaction)
In the presence of phosphorus, chlorine or bromine replaces a-hydrogen of an acid by halogen atom. For example,
24.58 Complete Chemistry—JEE Main

Br
|
Br2 , P Br2 , P
CH3CH2COOH Æ CH3CHCOOH Æ CH3— C— COOH
| |
Br Br
The halogen atom in the a-position can be easily replaced by another nucleophilic reagent. This provides a method
of converting carboxylic acid into many important substituted carboxylic acids. For example,
NH3
RCHCOOH RCHCOOH
(excess)
Br NH2
an a -amino acid
NaOH H+
RCHCOONa RCHCOOH
OH OH
an a -hydroxy acid
alc. KOH H+
R¢CH CHCOO– R¢CH CHCOOH
an a , b -unsaturated acid

KCN H2O COOH


RCHCOOH + RCH
H
CN COOH
a dicarboxylic acid
Decarboxylation
Heating of sodium salt of carboxylic acid with soda lime (NaOH + CaO) produces alkane
NaOH
RCOONa Æ RH + Na2CO3
H
Heating of calcium salt of carboxylic acid produces the compound containing CO group.
RCO2(Ca/2) + R¢CO2(Ca/2) RCOR¢ + Ca2CO3
If R is H, then aldehyde R¢CHO is produced.
Functional Derivatives of Carboxylic Acids
The functional derivatives of carboxylic acids are acid chlorides (RCOCl), anhydride R C O C R , amides

O O O
(RCONH2) and esters RCOR¢ . They all contain acyl group R C .

O
Nucleophilic Substitution Reactions The acyl derivatives undergo nucleophilic substitution reactions. Their
mechanisms are as follows.
Basic medium

Acidic medium
Organic Compounds Containing Oxygen (Alcohols, Phenols, Ethers, Aldehydes, Ketones, Carboxylic Acids and their Derivatives) 24.59

Reactivity of Acyl Derivatives


The reactivity of RCOG depends on the resonance stabilization of the –CO—G group.

The greater the degree of delocalization, the lesser reactive is RCOG, NH2 has the greatest degree of delocalization,
thus RCONH2 is least reactive, while X has little or no delocalization and thus RCOX is the most reactive.
Thus, the reactivity follows the order: Acid chloride > anhydride > easter > amide
A more reactive derivative may be used to prepare a less reactive derivative by reaction with appropriate nucleophile.
Incidently, the reactivity decreases as the base strength of the leaving group increases i.e. Cl– < RCOO– < RO– < H2N–.
We describe in brief the method of preparation and the main reactions of the above functional derivatives.
Acid Chlorides

(SOCl2 is the best as the unwanted products are gases.)

LiAlH4
RCH2OH alcohol
Acid Anhydrides

O
COOH C
200°C
Æ O
COOH C
O
phthalic anhydride
24.60 Complete Chemistry—JEE Main

H2O
Æ 2RCOOH
H+
2NH3
Æ RCONH2 + RCOO–NH4+
(RCO)2 O
R¢OH
Æ 2RCOOR¢ + RCOOH

ArH
Æ 2RCOAr + RCOOH
AlCl3
Cyclic anhydrides with NH3, R¢OH and ArH/AlCl3 gives only one product.
O

C
H2C CH2CONH2 H+
CH2CONH2
O + 2NH3 Æ Æ
H2C CH2COONH4 CH2COOH
C ammonium succinamic
succinamate acid
O
O

CH3OH
C OCH3
O
COOH
C
O
O
C
C
O C6H6
AlCl3
COOH
o-benzoylbenzoic acid

Amides
O NH 3
O
R C Æ R C
Cl NH2

LiAlH4
RCH2NH2

Esters
O O
R C + R¢OH R C + H2O
OH OR¢
O O
R C + R¢— OH Æ R C + HCl
Cl OR¢
Organic Compounds Containing Oxygen (Alcohols, Phenols, Ethers, Aldehydes, Ketones, Carboxylic Acids and their Derivatives) 24.61

O
(CH3CO)2 + HO NO2 Æ CH3C O NO2 + CH3COOH

p-nitrophenyl acetate

H2O O
Æ R C + R¢OH
OH
(reversible reaction)

OH– O
Æ R C + R¢OH
O–
NH3 O
Æ R C + R¢OH
O NH2
R C
O
OR¢ R≤OH
Æ R C
+ R OH
OR≤
(reversible reaction)
transesterification
R≤
2R≤MgX
Æ R C R≤
OH
H2
Æ RCH2OH + R¢OH
CuO-CuCr2O4
or LiAlH4

The reaction of RCOOR¢ with R¢¢MgX proceeds as follows

O -O(MgX)+ O -O(MgX)+

R¢¢MgX R¢¢MgX
R C OR¢ R C OR¢ R C R¢¢ R C R¢¢

R¢¢ R¢¢ H+-(MgX)+


OH
R C R¢¢

R¢¢

The reaction of RCN with R¢MgX gives a ketone:


N -(MgX)+ O
hydrolysis
R C N + R¢MgX R C R¢ R C R¢

Here, a second equivalent of R¢MgX does not react because the intermediate imine salt bears a negative charge. Unlike
the ester intermediate, where elimination of OR¢– leads to ketone, loss of the charge by elimination of R¢ merely reverse
the reaction.
24.62 Complete Chemistry—JEE Main

MULTIPLE CHOICE QUESTIONS ON SECTION 5

General Characteristics and Preparations


1. The Hell-Volhard-Zelinsky reaction is used to synthesise
(a) aldehydes (b) a-haloacids (c) acid halides (d) nitriles
2. The reagent used in the Hell-Volhard-Zelinsky reaction is
(a) zinc (b) nickel (c) phosphorus (d) aluminium chloride
3. The —COOH group in benzene ring is
(a) ortho directing (b) para directing
(c) ortho and para directing (d) meta directing
4. Which of the following orders is true regarding the acidic nature of COOH?
(a) Formic acid > Acetic acid > Propanoic acid (b) Formic acid > Acetic acid < Propanoic acid
(c) Formic acid < Acetic acid < propanoic acid (d) Formic acid < Acetic acid > Propanoic acid
5. Which of the following orders is true regarding the acidic nature of COOH?
(a) pK°a(HCOOH) > pK°a(CH3COOH) > pK°a(CH3CH2COOH)
(b) pK°a(HCOOH) > pK°a(CH3COOH) < pK°a(CH3CH2COOH)
(c) pK°a(HCOOH) < pK°a(CH3COOH) > pK°a(CH3CH2COOH)
(d) pK°a(HCOOH) < pK°a(CH3COOH) < pK°a(CH3CH2COOH)
6. Which of the following belongs to – I group?
(a) —CH3 (b) —CH2CH3 (c) —COOH (d) —CH(CH3)2
7. Which of the following orders is true regarding the acidic nature of acetic acid?
(a) Acetic acid > Monochloroacetic acid > Dichloroacetic acid
(b) Acetic acid < Monochloroacetic acid < Dichloroacetic acid
(c) Acetic acid > Monochloroacetic acid < Dichloroacetic acid
(d) Acetic acid < Monochloroacetic acid > Dichloroacetic acid
8. Which of the following orders is true regarding the acetic nature of monosubstituted acetic acid?
(a) Fluoroacetic acid > Chloroacetic acid > Bromoacetic acid
(b) Fluoroacetic acid < Chloroacetic acid < Bromoacetic acid
(c) Fluoroacetic acid < Chloroacetic acid > Bromoacetic acid
(d) Fluoroacetic acid < Chloroacetic acid < Bromoacetic acid
9. Which of the following orders is true regarding the boiling point of organic substances?
(a) CH3COCl > CH3COOH > (CH3CO)2O (b) CH3COCl < CH3COOH < (CH3CO)2O
(c) CH3COCl > CH3COOH < (CH3CO)2O (d) CH3COCl < CH3COOH > (CH3CO)2O
10. Which of the following is present in vinegar?
(a) HCOOH (b) CH3COOH (c) CH3CHO (d) CH3OH
11. The oxidation products of 1-nitronaphthalene and a-naphthylamine respectively are
(a) phthalic acid and 3-aminophthalic acid (b) 3-nitrophthalic acid and phthalic acid
(c) phthalic acid and phthalic acid (d) 3-nitrophthalic acid and 3-aminophthalic acid
12. Which of the following orders of acid strength is correct?
(a) RCOOH > ROH > HOH > HC CH (b) RCOOH > HOH > ROH > HC CH
(c) RCOOH > HOH > HC CH > ROH (d) RCOOH > HC CH > HOH > ROH
13. Which of the following orders of base strength is correct?
– –
(a) R– > NH2– > HC C > RCOO– (b) R– > NH2– > RCOO– > HC C
– –
(c) R– > RCOO– > NH2– > HC C (d) HC C > NH2– > RCOO– > R–
Organic Compounds Containing Oxygen (Alcohols, Phenols, Ethers, Aldehydes, Ketones, Carboxylic Acids and their Derivatives) 24.63

14. Which of the following orders of acid strength is correct?


(a) CH3CH2CHCOOH < CH3CHCH2COOH < CH2CH2CH2COOH
| | |
Cl Cl Cl
(b) CH3CH2CHCOOH > CH3CHCH2COOH > CH2CH2CH2COOH
| | |
Cl Cl Cl
(c) CH3CH2CHCOOH > CH2CH2CH2COOH > CH3CHCH2COOH
| | |
Cl Cl Cl
(d) CH3CH2CHCOOH < CH2CH2CH2COOH < CH3CHCH2COOH
| | |
Cl Cl Cl

15. Acetic acid differs from formic acid in that


(a) acetic acid is stable to heat (b) formic acid is stable to heat
(c) acetic acid acts as a reducing agent (d) acetic acid shows positive test with Tollens reagent
16. Which of the following sequences is correct?
(a) pK°a(p-O2NC6H4COOH) > pK°a(C6H5COOH) > pK°a (p-HOC6H4COOH)
(b) pK°a(p-O2NC6H4COOH) < pK°a(C6H5COOH) < pK°a (p-HOC6H4COOH)
(c) pK°a(p-O2NC6H4COOH) > pK°a(C6H5COOH) < pK°a (p-HOC6H4COOH)
(d) pK°a(p-O2NC6H4COOH) < pK°a(C6H5COOH) > pK°a (p-HOC6H4COOH)
17. Which of the following statements is correct?
(a) Formic acid like acetic acid can be halogenated in the presence of red phosphorus and Cl2.
(b) Formic acid is a stronger acid than acetic acid.
(c) The boiling point of propanoic acid is less than that of n-butyl alcohol, an alcohol of comparable molar
mass.
(d) The IUPAC name of CH3CH2CH2CN is propanenitrile.
18. Which of the following statements is not correct?
(a) The melting point of a trans fatty acid is higher than its cis isomer.
(b) The IUPAC name of
CH3CH2 CH2 CH2 (CH2)7COOH
C == C C == C C == C
H H H H H H
is cis,cis,cis-3,6,9-octadecatrien-18-oic acid.
(c) Vinegar is a 5% aqueous solution of acetic acid.
(d) The C==O bond in RCOOH is longer than in RCHO.
19. Which of the following statements is not correct?
(a) The reactivity of C==O towards nucleophiles in RCOOH is less than that in RCHO.
(b) Soft soaps are K+ salts of fatty acids.
(c) Hard soaps are Na+ salts of fatty acids.
(d) Peroxy acids, R COOH , are much stronger than R COH .
|| ||
O O
20. Which of the following statements is not correct?
(a) H2C==CHCH2COOH is more acidic than CH3CH2COOH.
(b) The acid (CH3)3CCH2C(CH3)2COOH is less acidic than CH3(CH2)7COOH.
(c) Trans-4-t-butylcyclohexanecarboxylic acid is stronger than the cis-isomer
(d) The pK°a1 of a dicarboxylic acid is usually greater than the pK°a of the comparable monoalkylcarboxylic acid.
24.64 Complete Chemistry—JEE Main

21. Which of the following statements is not correct?


(a) Maleic acid is a weaker acid than fumaric acid.
(b) Maleate monoanion is a weaker acid than fumarate monoanion.
(c) Salicyclic acid, o-HOC6H4COOH, is a stronger acid than o-H3COC6H4COOH.
(d) A more reactive acid derivative may be used to prepare a less reactive one by reaction with the appropriate
nucleophile.
Chemical Reactions
22. The reaction of formic acid with concentrated H2SO4 gives
(a) CH3COOH (b) CO2 (c) HCHO (d) CO
23. Which of the following reagent solutions can be used to distinguish between methanoic acid and ethanoic acid?
(a) Ammoniacal AgNO3 solution (b) FeCl3 solution
(c) Na2CO3 solution (d) NaOH solution
24. Acetic acid on reacting with ethanol in the presence of traces of sulphuric acid produces
(a) C2H6 (b) C2H4 (c) CH3COOC2H5 (d) (CH3CO)2O

(a) 1° < 2° < 3° (b) 1° > 2° > 3° (c) 1° > 3° > 2° (d) 1° < 3° < 2°

(a) RCH2COOH > R2CHCOOH > R3CCOOH (b) RCH2COOH < R2CHCOOH < R3CCOOH
(c) RCH2COOH < R3CCOOH < R2CHCOOH (d) R3CCOOH > RCH2COOH > R2CHCOOH
27. The treatment of CH3CH2COOH with chlorine in the presence of phosphorus gives
(a) CH3CH2COCl (b) CH3CH2CH2Cl (c) CH3CH(Cl)COOH (d) CH2(Cl)CH2COOH
28. Which of the following reactants produces benzophenone on heating?
(a) Calcium formate + Calcium benzoate (b) Calcium acetate + Calcium benzoate
(c) Calcium benzoate (d) Calcium acetate + Calcium formate
29. Which of the following reactants produces an aldehyde on heating?
(a) Calcium formate + Calcium benzoate (b) Calcium acetate + Calcium benzoate
(c) Calcium acetate (d) Calcium benzoate
30. Which of the following reagent/solution can be used to distinguish between methanoic acid and ethanoic acid?
(a) Tollens reagent (b) HCl solution (c) NaOH solution (d) NaHCO3 solution
31. Which of the following reactants would produce acetophenone on heating?
(a) Calcium acetate + Calcium formate (b) Calcium formate + Calcium benzoate
(c) Calcium acetate + Calcium benzoate (d) Calcium acetate
32. When propionic acid is treated with aqueous sodium bicarbonate, CO2 is liberated. The C from CO2 comes from
(a) methyl group (b) carboxylic acid group (c) methylene group (d) bicarbonate
33. Which of the following statements is not correct?
(a) The reduction of RCOOH with LiAlH4 produces RCH3.
(b) The —COOH group attached to benzene ring is meta director.
O
||
(c) In R—C— O –, both the carbon-oxygen bond lengths are identical. It is more than carbon-oxygen double bond
length, and less than carbon-oxygen single bond length.
(d) Electron-withdrawing substituents stabilises the carboxylate anion and hence increase the acidity of
carboxylic acid.
34. Which of the following statements is correct?
(a) Alkaline hydrolysis of an ester is a reversible reaction.
(b) In the alkaline hydrolysis of an ester, the bond broken is the bond between oxygen and alkyl group.
Organic Compounds Containing Oxygen (Alcohols, Phenols, Ethers, Aldehydes, Ketones, Carboxylic Acids and their Derivatives) 24.65

(c) Alkaline hydrolysis of an ester follows second order kinetics and is thus SN2.
(d) An electron-withdrawing group G in p-GC6H4COOC2H5 decreases the rate of hydrolysis as compared to
C6H5COOC2H5.
Carboxylic Acid Derivatives
35. Acetamide on heating with phosphorus pentoxide gives
(a) CH3COOH (b) CH3NH2 (c) CH3CN (d) CH3CH2NH2
36. Acetamide reacts with Br2/NaOH to give
(a) CH3COOH (b) CH3NH2 (c) CH3Br (d) CH3CH2NH2
37. The treatment of an ester with LiAlH4 followed by acid hydrolysis produces
(a) two alcohols
(b) two acids
(c) two aldehydes
(d) one molecule of alcohol and another of carboxylic acid
38. Which of the following sequences of relative rates of alkaline hydrolysis of esters is correct?
(a) CH3COOCH3 > CH3COOC2H5 > CH3COOC3H7
(b) CH3COOCH3 < CH3COOC2H5 < CH3COOC3H7
(c) CH3COOCH3 < CH3COOC2H5 > CH3COOC3H7
(d) CH3COOCH3 > CH3COOC2H5 < CH3COOC3H7
39. Which of the following sequences of relative rates of alkaline hydrolysis of esters is correct?
(a) CH3COOC3H7 > CH3COOCH(CH3)2 > CH3COOC(CH3)3
(b) CH3COOC3H7 < CH3COOCH(CH3)2 < CH3COOC(CH3)3
(c) CH3COOC3H7 < CH3COOCH(CH3)2 > CH3COOC(CH3)3
(d) CH3COOC3H7 > CH3COOCH(CH3)2 < CH3COOC(CH3)3
40. Which of the following sequences of relative rates of alkaline hydrolysis of esters is correct?
(a) HCOOCH3 < CH3COOCH3 < (CH3)2CHCOOCH3
(b) HCOOCH3 > CH3COOCH3 < (CH3)2CHCOOCH3
(c) HCOOCH3 < CH3COOCH3 > (CH3)2CHCOOCH3
(d) HCOOCH3 > CH3COOCH3 > (CH3)2CHCOOCH3
41. Which of the following sequences of rates of alkaline hydrolysis of esters is correct?
(a) CH3CH2COOCH3 < (CH3)2CHCOOCH3 < (CH3)3CCOOCH3
(b) CH3CH2COOCH3 > (CH3)2CHCOOCH3 > (CH3)3CCOOCH3
(c) CH3CH2COOCH3 > (CH3)2CHCOOCH3 < (CH3)3CCOOCH3
(d) CH3CH2COOCH3 < (CH3)2CHCOOCH3 > (CH3)3CCOOCH3
42. Which of the following statements is not correct?
(a) Carboxylic acids and their derivatives are collectively known as acyl compounds.
(b) Acyl compounds (RCOW) undergo electrophilic substitution reaction in which W is replaced by some other
basic group.
(c) Acyl compounds undergo nucleophilic substitution reactions more readily than the compounds containing no
carbonyl group.
(d) The acyl compounds differ from aldehydes or ketones in the respect that the former undergo nucleophilic
substitution reactions while the latter undergo nucleophilic addition reactions.
43. Which of the following statements is not correct?
(a) The ease with which :W of the acyl compound RCOW is lost depends upon its basicity; the weaker the base,
the better the leaving group.
(b) Aldehydes or ketones undergo nucleophilic addition reactions instead of substitution reactions because the
leaving group (:H– or :R–) are the strongest bases of all.
24.66 Complete Chemistry—JEE Main

followed by its reaction with ammonia or an alcohol.


(d) Adipic acid HOOC(CH2)4COOH on heating produces cyclic adipic anhydride having structure.
O O O
C C

CH2 CH2
CH2 CH2
44. Which of the following statements is correct?
(a) An electron-releasing group G in p-GC6H4COOC2H5 decreases the rate of hydrolysis as compared to
C6H5COOC2H5.
(b) The rate of alkaline hydrolysis of RCOOR¢ increases with the increase in the carbon atoms in the alkyl
group R¢.
(c) The rate of alkaline hydrolysis of RCOOR¢ increases with the increase in the carbon atoms in the alkyl group R.
(d) The rate of alkaline hydrolysis of RCOOR¢ decreases with the increase in the branching of carbon skeleton of
the alkyl group R.

ANSWERS

1. (b) 2. (c) 3. (d) 4. (a) 5. (d) 6. (c)


7. (b) 8. (a) 9. (b) 10. (b) 11. (b) 12. (b)
13. (a) 14. (b) 15. (a) 16. (b) 17. (b) 18. (b)
19. (d) 20. (d) 21. (a) 22. (d) 23. (a) 24. (c)
25. (b) 26. (a) 27. (c) 28. (c) 29. (a) 30. (a)
31. (c) 32. (d) 33. (a) 34. (c) 35. (c) 36. (b)
37. (a) 38. (a) 39. (a) 40. (d) 41. (b) 42. (b)
43. (d) 44. (a)

HINTS AND SOLUTIONS

1. The Hell-Volhard-Zelinsky reaction is used to synthesise a-haloacids.


2. The reagent used is phosphorus.
3. —COOH is electron-attracting group. It is thus meta director.
4. Electron-releasing group weakens the acidic nature of COOH.
5. The stronger the acid, the lesser the value of pK°a.
6. —I group is electron-attracting group.
7. Acidity increases with increase in the number of chlorine (electron-attracting group) atoms attached to methyl
group.
8. The larger the electronegativity of halogen, the larger the acidity of substituted acetic acid.
9. CH3COCl does not involve hydrogen bonding. Its boiling point is lesser than CH3COOH. The boiling point of the
latter is lesser than that of (CH3CO)2O due to larger molar mass of (CH3CO)2O.
10. Vinegar contains acetic acid.
11. Nitro bearing benzene ring is resistant to oxidation and amino bearing benzene is more susceptible to oxidation.
The products are 3-nitrophthalic acid and phthalic acid, respectively.
Organic Compounds Containing Oxygen (Alcohols, Phenols, Ethers, Aldehydes, Ketones, Carboxylic Acids and their Derivatives) 24.67

12. Carboxylic acid is the strongest acid. R group in ROH makes it less acidic than H2O. Acetylene is the least acidic.
13. The base strengths of the conjugate bases follow the reverse order of the acid strengths of acids.
14. Farther Cl atom from COOH group, weaker the acid.
15. Acetic acid is thermally more stable than formic acid.
16. Electron-attracting group makes benzoic acid more strong acidic and electron-releasing group makes it less acidic.
The pK°a follows the reverse order.
17. (a) Formic acid does not contain a-hydrogen atom.
(c) The boiling point of carboxylic acid is higher than that of alcohol of comparable molar mass because of more
extensive hydrogen bonding.
(d) The —CN group is also counted in the carbon chain. Hence, its name is butanenitrile.
18. (a) A trans double-bond does not interfere with the uniform zig-zag and close packing is permitted. This makes
its melting point higher than its cis isomer.
(b) Counting starts with COOH group. The correct name is cis, cis, cis-9,12,15-octadecatrienoic acid.

(d) Because of resonating structures

the C==O bond acquires some single-bond character, making its length longer than in RCHO.
19. (a) In RCOOH there is positive charge on the C==O carbon causing this C to be less electrophilic and less
reactive toward nucleophiles.
(d) The conjugate base of RCOOH is stabilised due to the resonance effect while no such stabilisation occurs in
the conjugate base of peroxy acid.

20. (a) The sp2 hybridised carbon of —C C— attracts the bonded electrons more than the sp3 hybridised carbon
does. Consequently. —C C— is an acid-strengthening electron-withdrawing group.

(b) The —COO of the branched acid is shielded from solvent molecules and cannot be stabilised as effectively
as can an unhindered anion.
(c) In trans isomer, the —COO– is less shielded and is thus more stabilised by solvent.

electron-withdrawing COOH group of dicarboxylic acid. Hence, pK°a1 of dicarboxylic acid is smaller than pK°a
of monoalkylcarboxylic acid.
21. (a) The maleic acid is a stronger acid because of the stabilisation of its monoanion due to H-bonding between the
cis COOH and COO–. This is not present in the trans isomer (fumarate anion).

where H-bonding is not present.


24.68 Complete Chemistry—JEE Main

(c) H-bonding in salicylate ion makes the salicyclic acid more acidic than o-H3COC6H4COOH.

–H O
22. HCOOH
2
Æ CO.
23. Methanoic acid (i.e. formic acid) reduces ammoniacal AgNO3 solution (Tollens reagent) while ethanoic acid does
not.
24. There is formation of the ester ethyl acetate.

P4
27. CH3CH2COOH Æ
Cl2 CH3CH(Cl)COOH. This is Hell-Volhard-Zlinsky reaction.
28. Heating of calcium benzoate produces benzophenone.
29. The use of calcium formate results into an aldehyde.
30. Same as Q. 23.
31. Acetophenone is produced on heating calcium acetate and calcium benzoate.
32. CO2 comes from HCO3–.
33. (a) The product obtained is RCH2OH.
(c) This is due to the resonating effect amongst the two CO groups.
(d) The stabilization is due to the dispersal of negative charge of the carboxylate anion.
34. (a) The reaction is

The reaction is irreversible because the resonance-stabilised carboxylate anion shows little tendency to react
with an alcohol.
(b) The bond broken is the bond between oxygen and acyl carbon.
(d) An electron-withdrawing group stabilises the intermediate carboxylate anion, hence speeds up the rate of
hydrolysis.
P2O5
35. CH3CONH2 Æ CH3CN.
–H2O

OBr–
36. RCONH2 Æ RNH2 + CO32–.
LiAlH
37. Two alcohols are produced. RCOOR¢ Æ
4
RCH2OH + R¢OH.
38. Increasing bulkiness of R¢ group in RCOOR¢ decreases the rate of alkaline hydrolysis of ester.
39. Increasing crowdiness of R¢ in RCOOR¢ decreases the rate of alkaline hydrolysis of ester.
40. Increasing bulkiness of R group in RCOOR¢ decreases the rate of alkaline hydrolysis of ester.
41. Increasing crowdiness of R group in RCOOR¢ decreases the rate of alkaline hydrolysis of ester.
42. (a) The acyl compounds contain RCO group.

(b) Because of group, acyl compounds undergo nucleophilic substitution reactions because carbonyl
carbon is slightly positively charged.
Organic Compounds Containing Oxygen (Alcohols, Phenols, Ethers, Aldehydes, Ketones, Carboxylic Acids and their Derivatives) 24.69

(d)

43. (c) As both the steps are fast and essentially irreversible.

Instead of producing cyclic anhydride, adipic acid loses carbon dioxide and form cyclopentanone.
44. (a) An electron-releasing group destabilises the intermediate carboxylate anion, hence decreases the rate of
hydrolysis.
(b) The steric effect and the electron-releasing effect of alkyl group decreases the rate of hydrolysis.
(c) The steric effect will cause the decrease in the rate of hydrolysis.

MULTIPLE CHOICE QUESTIONS FROM AIEEE AND JEE MAIN

1. When CH2 CH—COOH is reduced with LiAlH4, the compound obtained will be
(a) CH3CH2CHO (b) CH3CH2COOH (c) CH2=CHCH2OH (d) CH3CH2CH2OH
[2003]
2. Rate of the reaction
O O
R C + Nu Æ R C + Z
Z Nu
is fastest when Z is
(a) Cl (b) NH2 (c) OC2H5 (d) OCOCH3 [2004]
3. Consider the acidity of the carboxylic acids:
(i) PhCOOH (ii) o-NO2C6H4COOH (iii) p-NO2C6H4COOH (iv) m-NO2C6H4COOH
Which of the following order is correct?
(a) i > ii > iii > iv (b) ii > iv > iii > i (c) ii > iv > i > iii (d) ii > iii > iv > i
[2004]
4. On mixing ethyl acetate with aqueous sodium chloride, the composition of the resultant solution is
(a) CH3COOC2H5 + NaCl (b) CH3COONa + C2H5OH
(c) CH3COCl + C2H5OH + NaOH (d) CH3Cl + C2H5COONa [2004]
5. Acetyl bromide reacts with excess of CH3MgI followed by treatment with a saturated solution of NH4Cl gives
(a) acetone (b) acetamide (c) 2-methyl-2-propanol (d) acetyl iodide [2004]
6. Which one of the following is reduced with zinc and hydrochloric acid to give the corresponding hydrocarbon?
(a) ethyl acetate (b) acetic acid (c) acetamide (d) butan-2-one [2004]
7. Which one of the following undergoes reaction with 50% sodium hydroxide solution to give the corresponding
alcohol and acid?
(a) phenol (b) benzaldehye (c) butanol (d) benzoic acid [2004]
8. Among the following acids which has the lowest pKa value?
(a) (CH3)2CHCOOH (b) CH3CH2COOH (c) CH3COOH (d) HCOOH [2005]
24.70 Complete Chemistry—JEE Main

9. The reaction
RCOX + Nu Æ RCONu + X –
is fastest when X is
(a) –OC2H5 (b) –OCOR (c) Cl– (d) –NH2 [2005]
10. The increasing order of the rate of HCN addition to compounds
(A) HCHO, (B) CH3COCH3, (C) PhCOCH3, and (D) PhCOPh is
(a) C < D < B < A (b) A < B < C < D (c) D < B < C < A (d) D < C < B < A [2006]
11. The correct order of increasing acid strength of the compounds
Me
(A) CH3CO2H, (B) MeOCH2CO2H, (C) CF3CO2H, and (D) CO2H is
Me
(a) A < D < C < B (b) B < D < A < C (c) D < A < C < B (d) D < A < B < C [2006]
12. In the following sequence of reactions,
P + I2 Mg HCHO H 2O
CH3CH2OH Æ A Æ B Æ C Æ D
ether
the compound ‘D’ is:
(a) butanal (b) n-butyl alcohol (c) n-propyl alcohol (d) propanal [2007]
13. The compound formed as a result of oxidation of ethyl benzene by KMnO4 is:
(a) benzophenone (b) acetophenone (c) benzoic acid (d) benzyl alcohol
[2007]
14. Which of the following reagents may be used to distinguish between phenol and benzoic acid?
(a) Neutral FeCl3 (b) Aqueous NaOH (c) Tollens reagent (d) Molish reagent
[2011 cancelled]
15. The strongest acid amongst the following compound is
(a) ClCH2CH2CH2COOH (b) CH3COOH (c) HCOOH (d) CH3CH2CH(Cl)CO2H
[2011 cancelled]
16. An organic compound A upon reacting with NH3 gives B. On heating B gives C. The compound C in presence of
KOH reacts with Br2 to give CH3CH2NH2. The compound A is
(a) CH3COOH (b) CH3CH2CH2COOH

(c) (d) CH3CH2 COOH [2013]

LiAlH PCl alc.KOH


17. In the reaction CH3COOH ææææ 4
Æ A æææ 5
Æ B ææææ Æ C , the product C is
(a) acetyl chloride (b) acetaldehyde (c) acetylene (d) ethylene [2014]
18. Phthalic acid reacts with resorcinol in the presence of concentrated H2SO4 to give:
(a) Phenolphthalein (b) Alizarin (c) Coumarin (d) Fluorescein [2014]
19. Among the following organic acids, the acid present in rancid butter is:
(a) Pyruvic acid (b) Lactic acid (c) Butyric acid (d) Acetic acid [2014]
20.
COOK

Electrolysis
A

COOK
A is:
Organic Compounds Containing Oxygen (Alcohols, Phenols, Ethers, Aldehydes, Ketones, Carboxylic Acids and their Derivatives) 24.71

(a) (b)

(d)
(c)
n
[2015, online]
21. In the presence of a small amount of phosphorous, aliphatic carboxylic acids react with chlorine or bromine to
yield a compound in which a-hydrogen has been replaced by halogen. The reaction is known as
(a) Wolff-kischner reaction (b) Etard reaction
(b) Hell-Volhard-zelinsky reaction (d) Rosenmund reaction [2015, online]

ANSWERS
1. (c) 2. (a) 3. (d) 4. (a) 5. (c) 6. (d)
7. (b) 8. (d) 9. (c) 10. (d) 11. (d) 12. (c)
13. (c) 14. (a) 15. (d) 16. (d) 17. (d) 18. (d)
19. (c) 20. (b) 21. (c)

HINTS AND SOLUTIONS


2. The weaker the base replaced, faster the rate of reaction. Of the given leaving species, Cl– is the weakest base and
thus show the fastest rate of reaction.
Therefore, the choice a is correct.
3. Electron withdrawal group is acid strengthening group. Due to ortho effect, o-NO2C6H4COOH is the strongest
acid.
p-NO2C6H4COOH is more acidic than m-NO2C6H4COOH, because the former generates positive charge on carbon
bearing —COOH group whereas the latter generates positive charge on the next carbon atoms.

O O O
+
N + C C

O O O
+
More effective in displacing H+
–O N+
O–
Less effective in displacing H+
o-NO2C6H4COOH > p-NO2C6H4COOH > m-NO2C6H4COOH > C6H5COOH
Thus, the correct order is
(ii) (iii) (iv) (i)
4. The choice a is correct (i.e. CH3COOC2H5 + NaCl) as no reaction occurs when ethyl acetate is mixed with
aqueous sodium chloride.
5. The reaction is
OH
(i) excess of CH 3 MgI
CH3COBr Æ CH3 C CH3
(ii) saturated solution of NH 4 Cl
CH3
2-methyl-2-propanol
24.72 Complete Chemistry—JEE Main

6. The reaction is
Zn HCl
CH3COCH2CH3 Æ CH3CH2CH2CH3
butan-2-one n-butane
7. An aldehyde containing no a hydrogen atom undergoes Cannizzaro reaction:
50% NaOH
2C6H5CHO Æ C6H5CH2OH + C6H5COOH
benzaldehyde benzyl alcohol benzoic acid
8. The lowest pKa implies the highest Ka, which, in turn, implies the strongest acid. Of the given acids, HCOOH is
the strongest acid.
9. The weaker the base-leaving group, the faster the nucleophilic displacement.
10. Increase in crowding at the transition state decreases the rate of addition of HCN
11. Electron releasing group decreases the acid strength while electron attracting group increases the acid strength.
12. The reactions are as follows.
P + I2 Mg
CH3CH2OH Æ CH3CH2I Æ CH3CH2MgI
ether
(A) (B)

H
H2O
HCHO + CH3CH2MgI Æ H C OMg I Æ CH3CH2CH2OH
CH2CH3
(C)

13. The reaction is


CH2CH3 COOH

KMnO4
[O]

(Benzoic acid)
14. Neutral FeCl3 gives violet colouration with phenol while pale dull yellow precipitates are obtained with benzoic
acid.
15. The electron-attracting Cl atom attached to carbon at a-position in carboxylic acid enhances the stability of its
conjugate base and thus increases the acidicity of carboxylic acid.
16. We have

17. The reaction are


LiAlH4 PCl5 alc. KOH
CH3COOH CH3CH2OH CH3CH2Cl H2C CH2
ethylene
Organic Compounds Containing Oxygen (Alcohols, Phenols, Ethers, Aldehydes, Ketones, Carboxylic Acids and their Derivatives) 24.73

18.
HO OH HO OH HO O OH

H H H
O O
CO
C O
CO Fluorescein

19. Butyric acid is present in the rancid butter.


20. Anodic reaction during electrolysis is
COO–

+ 2CO2 + 2e–

COO–
21. Reaction is known as Hell-Volhard-Zelinsky reaction.
25
Organic Compounds Containing
Nitrogen (Cyanides,
Isocyanides, Nitrocompounds
and Amines)
SECTION 1 Cyanides and Isocyanides

The general formula of alkyl/aryl cyanide is R—C ∫∫ N while that of isocyanide is R—N Æ C. These are known as
nitriles and isonitriles, respectively. In the IUPAC system of nomenclature, the cyanides are named as alkanenitriles.
The chain is numbered with number 1 is assigned to —C ∫∫ N carbon. Isocyanides are named according to the common
system of nomenclature by placing the word isocyanide or carbylamine after the name of akyl/aryl group.
Alkyl cyanides are prepared by SN2 reactions involving alkyl halide and an aqueous ethanolic solution of sodium or
potassium cyanide. These are also prepared by the dehydration of acid amides by using P2O5 or POCl3 or SOCl2. Aryl
cyanides are prepared via Sandmeyer reaction in which arenediazonium chloride is treated with KCN in presence of
cuprous cyanide.
Nitriles are hydrolyzed by aqueous acid or base to produce carboxylic acid. Reduction with LiAlH4 produces amine.
Treatment with diisobutylaluminium hydride produces an imine derivative which on hydrolysis produces aldehyde.
The —CN or —NC group is meta director in aromatic substitution reactions.

MULTIPLE CHOICE QUESTIONS ON SECTION 1

Cyanide and Isocyanide


1. The IUPAC name of CH3CH2CN is
(a) 1-cyanoethane (b) ethyl cyanide (c) ethylnitrile (d) propanenitrile
2. The IUPAC name of CH3CH2NC is
(a) isocyanoethane (b) ethyl isonitrile (c) ethyl isocyanide (d) propanenitrile
3. The hydrolysis of isocyanide produces
(a) acid (b) amide (c) amine (d) ester
4. The partial hydrolysis of a nitrile produces
(a) acid (b) amide (c) amine (d) ester
5. The complete hydrolysis of a nitrile produces
(a) acid (b) amide (c) amine (d) ester
25.2 Complete Chemistry—JEE Main

6. The reduction of a nitrile by LiAlH4 produces


(a) primary amine (b) secondary amine (c) tertiary amine (d) amide
7. The reduction of an isocyanide by LiAlH4 produces
(a) primary amine (b) secondary amine (c) tertiary amine (d) amide

ANSWERS
1. (d) 2. (b) 3. (c) 4. (b) 5. (a) 6. (a)
7. (b)

MULTIPLE CHOICE QUESTION FROM AIEEE AND JEE MAIN

1. Ethyl isocyanide on hydrolysis in acidic medium generates


(a) methylamine salt and ethanoic acid (b) ethylamine salt and methanoic acid
(c) propanoic acid and ammonium salt (d) ethanoic acid and ammonium salt [2003]

ANSWER
1. (b)

HINT AND SOLUTION


H O
1. C2H5NC æææ Æ C2H5NH2 + HCOOH
2
+
H

SECTION 2 Nitro Compounds

The nitro compounds are named as nitro derivatives of the corresponding alkane with the lowest number assigned to
the carbon bearing nitro group.
Nitroalkane may be prepared by treating alkyl halide with silver nitrite in aqueous ethanolic solution. This method is
useful for primary nitroalkanes. The yield is poor for sec- and tert-nitroalkanes. Aromatic nitro compounds are prepared
by nitration of benzene ring with a mixture of concentrated HNO3 and concentrated H2SO4.
The reduction of nitro group in nitroalkanes depends on the condition of reduction.
LiAlH4
RCH2NH2

Zn dust
RCH2NO2 RCH2NHOH
NH4Cl soln.

SnCl2/HCl
RCH2NHOH + RCH NOH

The reduction of nitrobenzenes proceeds through the following stages.


Organic Compounds Containing Nitrogen (Cyanides, Isocyanides, Nitrocompounds and Amines) 25.3


O
NaOH/CH3OH
[PhNO + PhNHOH] PhN = N+Ph
Fe
Zn/CH3OH/NaOH CH3CO2H
PhNO2 Ph N = N Ph SnCl2
HCl
Zn/NaOH NaIO 3
Zn/NaOH 2PhNH2
Ph NHNHPh

In strongly acidic medium, the product is p-aminophenol formed by the rearrangement of phenylhydroxylamine.

NO2 NHOH OH

metal/conc. HCl rearrangement

NH2

Nitroalkanes reacts with nitrous acid, the end product depends on the nature of nitroalkane.

Nitrolic acid

Nitro group is meta directing in aromatic substitution reactions.

MULTIPLE CHOICE QUESTIONS ON SECTION 2

1. The reduction of nitrobenzene with zinc in the presence of NaOH produces


(a) aniline (b) hydrazobenzene (c) azobenzene (d) phenylhydroxylamine
2. The reduction of nitrobenzene with zinc in the presence of methanolic alkali produces
(a) aniline (b) hydrazobenzene (c) azobenzene (d) phenylhydroxylamine
3. The reduction of nitrobenzene using zinc/tin in the presence of hydrochloric acid produces
(a) aniline (b) hydrazobenzene (c) azobenzene (d) phenylhydroxylamine
4. The reduction of nitrobenzene in a neutral medium (by a reducing agent like zinc in aqueous NH4Cl) produces
(a) aniline (b) hydrazobenzene (c) azobenzene (d) phenylhydroxylamine
5. The hydrolysis of CH3CH2NO2 with 85% H2SO4 gives
(a) CH3CH2OH (b) CH3COOH (c) CH3CH = NOH (d) C2H6
6. The major product of the reaction between m-dinitrobenzene with NH4SH is
25.4 Complete Chemistry—JEE Main
NO2 NH2 NHSH NHSH

(a) (b) (c) (d)

NH2 NH2 NO2 NHSH


7. In strongly acidic medium, phenylhydroxyl amine is converted into
(a) aniline (b) phenol (c) p-aminophenol (d) anilinium salt
8. Primary nitro compound on reacting with nitrous acid gives
(a) nitrolic acid (b) pseudonitroles (c) no new compound (a) a dintro compound

ANSWERS
1. (b) 2. (c) 3. (a) 4. (d) 5. (b) 6. (a)
7. (c) 8. (a)

SECTION 3 Amines

Methods of Preparation
Reduction of Nitro Compounds
Reduction can be done by using either molecular hydrogen and a catalyst (Ni or Pt) or a metal (usually granulated tin)
and an acid (hydrochloric acid). For example,
NO2 NH2

H2/Ni or Pt

or Sn/HCl

Reaction of Halides with Ammonia


+ –
RX + NH3 3X

RNH+3 X– + OH– 2 + H 2O + X –
Replacement of X by NH2 is a nucleophilic substitution reaction. Tertiary alkyl halide generally produces alkene.
Aryl halides show low reactivity. These can be converted into amine if a strong deactivating group, such as —NO2,
is also attached to the ring at ortho or para position to halogen.
During the ammonolysis of halides, ammonia is taken in excess so as to avoid the formation of other amines (2° and 3°)
NH3
RX Æ RNH2 RX
Æ R2NH RX
Æ R3N
(1°) (2°) (3°)

Hofmann Degradation of Amides

O OBr–
R C R NH2 + CO 2–
3
NH2 (1°)
The reaction involves migration of a group from carbonyl carbon to the adjacent nitrogen atom. The reaction is believed
to proceed by the following steps.
O O

(a) Halogenation of amide R C + OBr Æ R C + OH –
:

NH2 N Br
H
Organic Compounds Containing Nitrogen (Cyanides, Isocyanides, Nitrocompounds and Amines) 25.5

O O
(b) Abstraction of H+ by OH– R C + OH– Æ R C + H2O

:
N Br N Br
H
O O
(c) Separation of halide ion R C Æ R C + Br –
N Br N

O
(d) Rearrangement to produce isocyanate R C Æ R N C O
N

R—N==C==O + 2OH – Æ R— NH2 + CO 2–


H2 O
(e) Hydrolysis of isocyanate 3

Reduction of Nitriles and Amides


1. LiAlH4 , ether
R— X NaCN
Æ R— CN Æ RCH2NH2
2. H 2 O

O
1. SOCl 1. LiAlH 4 , ether
R — COOH Æ R
2
C NH2 Æ RCH2NH2
2. NH 2. H 2 O
3

Physical Properties
Amines are polar compounds. Except tertiary amines, primary and secondary amines form intermolecular hydrogen
bonding. Amines have lower boiling points than alcohols and carboxylic acids of comparable molar masses.

Chemical Reactions
Basic Nature of Amines
Nitrogen of amines contains lone pair of electrons which can be shared with other species and thus these act as Lewis
bases. Amines form salts with acids. For example,

NH2 + HCl NH+3 Cl–

aniline anilinium chloride


Aliphatic amines are somewhat stronger bases than ammonia. This is due to the fact that alkyl group is electron-
releasing group which, in turn, increases the availability of lone pair of electrons. On the other hand, aromatic amines
are considerably weaker than ammonia. This is due to the electron-withdrawing nature of phenyl group which decreases
the availability of lone pair of electrons.
Alternatively, the more stable the ion relative to the amine from which it is formed, the more basic the amines. For
example, the ion formed in the reaction
H H
R N + H+ R N+ H
H H
is stablized by alkyl group and hence increases basicity of RNH2 as compared to NH3.
Two factors operate in deciding the basicity of alkylamines in aqueous medium.
(i) Inductive Effect The alkyl group being electron releasing increases the charge density on nitrogen. This in
turn increases the basicity of amines. The expected order of basicity is
R3N > R2NH > RNH2 > NH3
(ii) Solvation Effect Because of the positive charge carried by the conjugate acid of an amine, it is stabilized by
the hydrogen bonding with the solvent water. The larger the number of hydrogens attached to the nitrogen in the
25.6 Complete Chemistry—JEE Main

conjugate acid, the larger is its stability and thus larger is the basicity of the corresponding base. The expected
order of basicity of the alkylamines will be
NH3 > RNH2 > R2NH > R3N
The inductive and solvent effects predict the opposite trend in the basiscity of alkylamines. In going from R2NH to
R3N, the solvation effect plays a more dominating role as compared to the inductive effect making R2NH more basic than
R3N. In solution, the order is R2NH > RNH2 > R3N > NH3.
A simple explanation is that the steric factor in R3N makes the availability of a lone pair of electrons on nitrogen poor
than in the dialkylamine, predicting R2NH a stronger base than R3N.
In aromatic amines, the resonance stabilization of ArNH2 is larger than ArNH3+ , because there are more resonating
structures of ArNH2 as compared to those of ArNH+3:
+ + + + +
NH2 NH2 NH2 NH2 NH2 NH3 NH3

– –
;

Effect of substituents on basicity of aromatic amines Electron-releasing group pushes electrons toward nitrogen
and hence increases the availability of lone pair of electrons and thus increases its basicity. On the other hand, electron-
withdrawing group decreases the availability hence makes the amine less basic. In terms of the stability of ion, electron-
releasing group stabilizes cation and hence increases basicity whereas electron-withdrawing group destabilizes cation

making the amine less basic. For example G NH2 is more basic if G is —NH2 or —OCH3 or —CH3 and

is less basic if G is —NH+3 or —NO2 or —SO–3 or —COOH or —X.


It may be pointed out that the basicity of an amine and acidity of a carboxylic acid are affected by a substituent in
the opposite manner. This is to be expected, since basicity depends upon ability to accommodate a positive charge (or
ability to share lone pair) and acidity depends upon ability to accommodate a negative charge (or to accept lone pair
of electrons).
Alkylation
The alkylation of RNH2 produces quaternary ammonium salt.
RNH2 RX
Æ R2NH RX
Æ R3N RX
Æ R4N+X –
(1°) (2°) (3°) quaternary ammonium halide

Treatment of quaternary ammonium halide with aqueous Ag2O produces 4° ammonium hydroxides (which is very
strong base like NaOH).
2R4N+X– + Ag2O + H2 4N
+
OH– + 2AgX
On heating quaternary hydroxide, less substituted alkene (Holfmann product) is obtained.
[(CH3)3 NCH(CH3)CH2CH3]+OH– æDæ
Æ (CH3)3N + H2C== CHCH2CH3 + H2O
A less substituted alkene is formed as a result of the loss of more acidic bH (which follows the trend 1° > 2° > 3°).
Conversion of Amines into Substituted Amides
Primary and secondary amines react with acid chloride to give substituted amides.
Organic Compounds Containing Nitrogen (Cyanides, Isocyanides, Nitrocompounds and Amines) 25.7

Tertiary amines do not react this way as they contain no replaceable hydrogen.
Schotten-Baumann Technique to prepare substituted amides of aromatic carboxylic acid. In this method acid chloride
is added to the amine in the presence of a base (aqueous NaOH or pyridine).

base
NH2 + Cl C Æ NH C
O O
benzanilide

Substituted amides undergo hydrolysis on heating in the presence of acid or base to give amine and carboxylic acid.
Acetylation of aniline is generally carried out by using acetic anhydride instead of acetyl chloride.

CH3COONa
NH2 + (CH3CO)2 O NHCOCH3 + CH3COOH

Ring Substitution in Aromatic Amines


The —NH2, —NHR and —NR2 are benzene activating groups through resonance effect of nitrogen where the lone pair
of nitrogen is shifted to the benzene ring making ortho and para positions available for electrophilic attack.

+ +
NH2 NH2 NH2 +
NH2
H
H

The carbocation formed as intermediate are

+ +
NH2 NH2
H
Y and

H Y
In these structures, nitrogen atom carries a positive charge and is joined to the benzene ring by a double bond. Such
structures are quite stable as each atom (except hydrogen) has a complete octet of electrons.
The group —NHCOCH3 is less powerful ortho and para director because of the electron-withdrawing characteristic
of oxygen makes nitrogen a poor source of electrons. This fact is made use in preparing mono substituted aniline. The
—NH2 group is such a powerful activator, that substitution occurs at all available ortho and para positions of aniline.
If, however, —NH2 group is converted to —NHCOCH3, the molecule becomes less powerful activator. Hence, only
mono substitution products are obtained. Finally, —NHCOCH3 is converted back to —NH2 by hydrolyzing with acid.
This technique is especially used while nitrating aniline as strong oxidizing agent destroys the highly reactive ring.

NH2 NHCOCH3 NHCOCH3 NH2

Ac2O HNO3 H2O, H+


H2SO4

NO2 NO2
25.8 Complete Chemistry—JEE Main

If acetylation is not done, nitration of aniline gives about two-thirds meta and one-third para products, besides loss
of aniline. This is due to the fact that the strong acid converts aniline to anilinium ion:
+
NH2 NH3

+ H+

Now the group —+NH3 is the meta director because of its positive charge.
Reactions of Amine with Nitrous Acid
In the reacting mixture, nitrous acid is generated by the action of mineral acid on sodium nitrite. The products obtained
depend on the type of amine:
Aliphatic primary amine It forms unstable diazonium salt which on decomposing liberates nitrogen and mixture of
alcohols and alkenes.
+ –
2 X ] æææ Æ N2 + mixture of alcohols and alkenes
H2O
R—NH2 + NaNO2
Nitrogen obtained is quantitative and hence can be used to determine the strength of amine.
Aromatic primary amine It forms stable diazonium salt.
cold + –
NH2 + NaNO2 + 2HX N N X + NaX + 2H2O

Aliphatic and Aromatic Secondary Amines Both of these produce N-nitrosoamines.

CH3 CH3
N H + NaNO2 + HCl Æ N N O + NaCl + H2O

N-nitroso-N-methylaniline
Tertiary Aromatic Amine This undergoes ring substitution, to yield a compound in which in nitroso group, —N = O,
is joined to carbon:
NaNO2, HCl
(CH3)2N (CH3)2 N N O
0 – 10° C

In this reaction, the electrophile —+NO (or H2O+—NO) replaces H+ of benzene.

Synthetic Use of Diazonium Salt


One of the best general ways of introducing F, Cl, Br, I, CN, OH and H into an aromatic ring is via diazonium salt. The
freshly prepared diazonium salt (primary aromatic amine in cold aqueous mineral acid + sodium nitrite) is treated with
other reagents to get the required product.
(a) The replacement by —Cl or —Br can be carried out by either of the following ways:
Sandmeyer Reaction The diazonium salt is treated with cuprous chloride or cuprous bromide

Ar -- N +2 X - æææ
CuX
Æ Ar -- X + N 2
Gattermann Reaction In this, copper powder and hydrogen halide are used in place of cuprous halide.
(b) The replacement by —I is achieved by simply adding KI to diazonium salt
Ar—N+2 X– + I– 2 + X–
(c) The replacement by —F is achieved as follows
Organic Compounds Containing Nitrogen (Cyanides, Isocyanides, Nitrocompounds and Amines) 25.9

Ar — N2+ X – Æ Ar — N2+ BF –4 Æ Ar — F + BF3 + N2


HBF4 heat

stable salt,
filtered and dried
(d) The replacement by —CN is carried out by adding cuprous cyanide to diazonium salt
Ar -- N +2 X - æææÆ Ar -- CN + N 2
CuCN

Hydrolysis of nitrile produces carboxylic acid.


(e) The replacement by —OH is carried out by adding water to diazonium salt.
Ar—N+2 X– + H2 2+H
+

To avoid coupling of Ar—OH with Ar—N+2 X– (which form azo compounds by coupling), the latter is slowly
added to large volume of water.
(f) The replacement by —H is achieved by the reducing agent hypophosphorous acid, H3PO2.
Ar—N+2X– + H3PO2 + H2 2 + H3PO3 + HX
Hofmann Elimination Amines can be made to undergo an elimination reaction under suitable conditions to yield
alkenes. The reaction is known as elimination reaction. The amine is methylated with excess of methyl iodide, yielding
a quaternary ammonium iodide. The latter produces an alkene when heated with silver oxide. The less substituted alkene
in contrast to the more substituted stable alkene predominates in this reaction. For example, 1-butene is formed from
butylamine.
CH I +
CH3CH2CH2CH2NH2 ⎯(excess)
⎯⎯ 3 → CH CH CH CH N (CH ) I–
3 2 2 2 3 3

Ag2O, H2O
–Agl

+
CH3CH2CH = CH2 ←⎯
Δ⎯
CH3CH2CH2CH2N (CH3)3– OH–
1-butene

Hofmann Rule In Hofmann elimination, an alkene is formed as a result of the loss of more acidic bH (which follows
the trend 1° > 2° > 3°). Since the amine is relatively poor leaving group, the transition state involves more C—Hb bond
breaking. Thus, the acidity of the bH becomes the deciding factor in the formation of the double bond.

Separation of a Mixture of Amines


The mixture of the amine salt and quaternary salt is distilled with KOH solution. The three amines distill over leaving
the quaternary salt behind. The three amines can be separated by either of the two methods given below.
Hinsberg Method
The mixture of three amines is treated with Hinsberg reagent (benzenesulphonyl chloride, C6H5SO2Cl). The primary
and secondary amines react with Hinsberg reagent while tertiary does not. The reactions are

After the reactions are over, the solution is made alkaline with NaOH. The sulphonamide of primary amine is soluble
in NaOH (due to the acidic hydrogen attached with the nitrogen) while that of secondary amine is insoluble (as there is
no acidic hydrogen attached to nitrogen).
So after treating with NaOH, two layers are formed; aqueous phase containing soluble salt of primary amine
25.10 Complete Chemistry—JEE Main

(C6H5SO2NR) and second layer containing insoluble salt of secondary amine and tertiary amine as such. The two
Na
layers are separated with the help of ether. The aqueous phase is hydrolysed with concentrated HCl when the primary
amine is regenerated.
The ether layer is distilled and tertiary amine is distilled over. The residue is hydrolysed with concentrated HCl to
recover secondary amine.
Hofmann’s Method
The mixture of amines is treated with diethyloxalate which forms a solid oxamide with primary amine, a liquid oxime

remaining liquid is treated with KOH and distilled when the secondary amine is distilled over. The reactions occurring
are as follows.
COOC2H5 Æ CONHR COOC2H5 Æ CONR2
+ 2RNH2 + 2C2H5OH; + R2NH + C2H5OH
COOC2H5 CONHR COOC2H5 COOC2H5
(dialkyloxamide) (oxamic ester)
solid liquid

Test for Primary Amine (Carbylamine Reaction)


The treatment of a primary amine with chloroform and alcoholic potash produces carbylamine (isocyanide) which has
most offensive smell:
RNH2 + CHCl3 2O
This reaction is not exhibited by secondary and tertiary amines.
Dye Test for aromatic primary amines Cool in ice 0.2 g (or 4-5 drops) of the organic compound with 1 mL
concentrated HCl. Add 2 mL of iced-cold dilute NaNO2 solution. Shake and add 2-3 mL of alkaline b-naphthol solution.
A red or orange dye indicates the presence of aromatic —NH2 group

Test for Secondary Amine (Libermann Reaction)


The secondary amine is converted into nitrosoamine by treating the amine with nitrous acid. The resultant solution is
warmed with phenol and concentrated H2SO4
to green. The colour changes to red on dilution and further changes to greenish-blue or violet on treating with alkali.

MULTIPLE CHOICE QUESTIONS ON SECTION 3

General Characteristics of Amines


1. Which of the following is a primary amine?
(a) CH3CH2NH2 (b) CH3NHCH3 (c) (CH3)3N (d) (CH3)4N+ Cl–
2. Activation of benzene by —NH2 group can be reduced by treating the compound with
(a) acetic acid (b) acetyl chloride (c) dilute HCl (d) methyl alcohol
3. Which of the following orders is true regarding the basic nature of NH2 group?
(a) o-Toluidine > Aniline > o-Nitroaniline (b) o-Toluidine < Aniline > o-Nitroaniline
(c) o-Toluidine < Aniline < o-Nitroaniline (d) o-Toluidine > Aniline < o-Nitroaniline
4. Which of the following order of basic strength is correct for the amines in aqueous phase?
(a) (CH3)2NH > CH3NH > (CH3)3N (b) (CH3)2NH > (CH3)3N > CH3NH
(c) (CH3)3N > (CH3)2NH > CH3NH2 (d) (CH3)3N > CH3NH2 > (CH3)2NH
5. Which of the following orders amongst amines in the gaseous state is true regarding the basic nature of NH2 group?
Organic Compounds Containing Nitrogen (Cyanides, Isocyanides, Nitrocompounds and Amines) 25.11

(a) CH3NH2 > (CH3)2 NH > (CH3)3 N (b) CH3NH2 > (CH3)2 NH < (CH3)3 N
(c) CH3NH2 < (CH3)2 NH > (CH3)3 N (d) CH3NH2 < (CH3)2 NH < (CH3)3 N
6. Which of the following order is true regarding the acidic nature of COOH group?
(a) Benzoic acid > o-Aminobenzoic acid > o-Nitrobenzoic acid
(b) Benzoic acid < o-Aminobenzoic acid < o-Nitrobenzoic acid
(c) Benzoic acid > o-Aminobenzoic acid < o-Nitrobenzoic acid
(d) Benzoic acid < o-Aminobenzoic acid > o-Nitrobenzoic acid
7. Among the following phenols, the most acidic is
(a) p-aminophenol (b) p-nitrophenol (c) o-chlorophenol (d) m-nitrophenol
8. The correct order of basicity of the given compounds is
(a) CH3CH2NH2 < HO(CH2)3NH2 < HO(CH2)2NH2 (b) CH3CH2NH2 < HO(CH2)3NH2 > HO(CH2)2NH2
(c) CH3CH2NH2 > HO(CH2)3NH2 > HO(CH2)2NH2 (d) CH3CH2NH2 > HO(CH2)3NH2 < HO(CH2)2NH2
9. Acetamide is treated separately with the following reagents. Which one of these would give methylamine?
(a) PCl5 (b) sodalime (c) NaOH + Br2 (d) hot concentrated H2SO4
10. Chlorobenzene can be prepared by reacting aniline with
(a) hydrochloric acid
(b) cuprous chloride
(c) chlorine in the presence of anhydrous aluminium chloride
(d) nitrous acid followed by heating with cuprous chloride
11. Amongst the following, the most basic compound is
(a) benzylamine (b) aniline (c) acetanilide (d) p-nitroaniline
12. Examine the following two structures for the anilinium ion and chose the correct statement from the ones given
below.
+ NH NH3
3

←→

(I) (II)
(a) II is not acceptable canonical structure because carbonium ions are less stable than ammonium ions
(b) II is not an acceptable canonical structure because it is nonaromatic
(c) II is not an acceptable canonical structure because nitrogen has 10 valence electrons
(d) II is an acceptable canonical structure
13. Which of the following statements is not correct?
(a) Primary amines show intermolecular hydrogen bonding.
(b) Secondary amines show intermolecular hydrogen bonding.
(c) Tertiary amines show intermolecular hydrogen bonding.
(d) Amines have lower boiling points as compared to those of alcohols and carboxylic acids of comparable molar
masses.
14. Which of the following order is correct regarding the relative basicity of amines?

(a) CH3O NH2 > NH2 > O2N NH2

(b) CH3O NH2 < NH2 < O2N NH2

(c) CH3O NH2 < O2N NH2 < NH2


25.12 Complete Chemistry—JEE Main

(d) O2N NH2 < CH3O NH2 < NH2

15. Which of the following orders is correct regarding the basic strength of substituted aniline?
(a) p–nitroaniline > p–aminobenzaldehyde > p–bromoaniline
(b) p–nitroaniline < p–bromoaniline < p–aminobenzaldehyde
(c) p–nitroaniline < p–aminobenzaldehyde < p–bromoaniline
(d) p–nitroaniline > p–aminobenzaldehyde < p–bromoaniline
16. Which of the following orders regarding the basic strength of substituted aniline is correct?
(a) p–methylaniline > p–chloroaniline > p–aminoacetophenone
(b) p–methylaniline > p–aminoacetophenone > p–chloroaniline
(c) p–aminoacetophenone > p–methylaniline > p–chloroaniline
(d) p–aminoacetophenone > p–chloroaniline > p–methylaniline
17. The number of resonating structures of arylammonium ion is
(a) 2 (b) 3 (c) 4 (d) 5
18. The number of resonating structures of aniline is
(a) 2 (b) 3 (c) 4 (d) 5
19. Among the following, the strongest base is
(a) C6H5NH2 (b) p-NO2C6H4NH2 (c) m-NO2C6H4NH2 (d) C6H5CH2NH2
20. Which of the following orders is correct regarding basicity of indicated molecules?
(a) N, N-Dimethyltoluidine > p-toluidine > aniline > p-nitroaniline
(b) Aniline > N, N-dimethyl-p-toluidine > p-toluidine > aniline
(c) p-Toluidine > N, N-dimethyl-p-toluidine > aniline > p-nitroaniline
(d) N, N-Dimethyltoluidine > aniline > p-toluidine > p-nitroaniline
21. Which of the following statements is correct?
(a) Guanidine (H2N)2CN == H is expected to be a strong base.
(b) Aniline is more basic than pyridine.
N
(c) Imidizole, , has two basic sites.

N
H

(d) Purine,

22. Which of the following statements is correct?


(a) Amines have lower boiling points than nonpolar compounds of the same molar mass.
(b) Amines have higher boiling points than alcohols or carboxylic acids of comparable molar mass.
(c) Aliphatic amines are as basic as ammonia but aromatic amines are more basic.

(d) Although nitrogen in R ⎯ N is chiral and also the mirror image of the molecule is not superimposable on
R′′ R′
the molecule itself, yet the molecule does not show optical activity.
23. Which of the following statements is not correct?
(a) Quaternary ammonium salts in which nitrogen holds four different groups show optical activity.
Organic Compounds Containing Nitrogen (Cyanides, Isocyanides, Nitrocompounds and Amines) 25.13

(b) Aliphatic amines can be prepared by ammonolysis of alkyl halides but not aromatic amines.
(c) Ammonolysis of alkyl halides gives highest yield with tertiary halides and is worthless for primary halides.
(d) The more stable the substituted ammonium ion relative to the amine from where it is formed, the more basic
is the amine.
24. Which of the following statements is correct?
(a) Base-weakening substituents are the one that activate an aromatic ring toward electrophilic substitution.
(b) Aliphatic amine forms stable diazonium salt with nitrous acid.
(c) All amino compounds can be distinguished by using the reagents chloroform and potassium hydroxide.
(d) The sulphonamide formed from secondary amine does not form sodium salt and is also insoluble in water.
25. Which of the following statements are is correct?
(a) The order of acid strength in the given compounds is NH3 > CH3CONH2 > (CH3CO)2NH.
(b) p-Aminobenzoic acid is a dipolar ion.
(c) Sulphanilic acid can exist as a dipolar ion.
(d) In Hoffmann elimination, a more substituted alkene is produced.
26. Which of the following statements is not correct?
(a) NF3 is less basic than NH3.
(b) Guanidine, HN==C(NH2)2, is a weak base.
(c) The bond angle in NH3 is greater than in NF3.
(d) The decreasing base strength of the amines CH3(CH2)2NH2, CH2==CHCH2NH2 and HC∫∫CCH2NH2 is
CH3(CH2)2NH2 > H2C==CHCH2NH2 > HC∫∫CCH2NH2.
27. Which of the following statements is not correct?
(a) Amides are much stronger bases than amines.
(b) PhCONH2 is a stronger base than CH3CONH2.
(c) N, N-Dimethylaniline is only slightly more basic than aniline whereas 2,6-dimethyl-N,N-dimethylaniline is
much more basic than 2,6-dimethylaniline.
(d) p-Nitroaniline is a weaker base than p-cyanoaniline.
28. Which of the following statements is correct?
(a) 3,4,5-Trinitroaniline is less basic than 4-cyano-3,5-dinitroaniline.

(b) The increasing order of basicities of pyrrole , pyridine and piperidine H is


Pyrrole < Pyridine < Piperidine.

(c) Pyrrole is soluble in dilute HCl.

(d) The catalytic reduction of quinoline involving two equivalents of H2 produces


N N
Chemical Reactions of Amines
29. The primary amine on treating with NaNO2 and HCl produces
(a) diazonium salt (b) nitrosamine (c) nitrite salt (d) nitrophenol
30. The secondary amine on treating with NaNO2 and HCl produces
(a) diazonium salt (b) nitrosamine (c) nitrite salt (d) isonitrite salt
31. Which of the following is Sandmeyer reaction?
Cu Cl / HCl Cu 2 Cl 2 / HCl
(a) 2C6 H5 Cl æææææ
2 2
Æ C6 H 6 + Cl2 (b) C6H5N2Cl Æ C6H5Cl + N2
Zn dust (d) C6H5NO2 + 6[H] Sn / HCl
(c) C6H5OH Æ C6H6 + ZnO Æ C6H5NH2 + 2H2O
32. Aniline reacts with bromine water to give
25.14 Complete Chemistry—JEE Main

(a) p-bromoaniline (b) o-bromoaniline


(c) mixture of o-and p-bromaniline (d) 2, 4, 6-tribromoaniline
33. Benzenediazonium salt when heated in aqueous solution the compound obtained is
(a) phenol (b) nitrobenzene (c) chlorobenzene (d) benzene
34. Aniline reacts with concentrated sulphuric acid at 450-475 K to give
(a) phenol (b) sulphanilic acid
(c) aniline hydrogen sulphate (d) azo dye
35. Heating of [CH3CH2CH2CH(CH3)N+(CH3)3]OH– produces
(a) CH2==CH2 (b) CH3CH2CH2CH==CH2
(c) CH3CH2CH==CHCH3 (d) CH2==CHCH2CH==CH2
36. The compound which on reaction with aqueous nitrous acid at low temperature produces an oily nitrosoamine is
(a) methylamine (b) ethylamine (c) diethylamine (d) triethylamine
37. Which of the following statements is not correct?
(a) Replacement of halogen by NH2 in alkyl halide is a nucleophilic substitution reaction
(b) Aryl halides show more reactivity as compared to alkyl halides in the replacements of halogen by the NH2
group
(c) During the replacement of halogen by —NH2 group, ammonia is taken in large excess so as to avoid the
formation of 2° and 3° amines
(d) Tertiary alkyl halide generally produces alkene instead of the replacement of halogen by NH2 group
38. Which of the following statements is not correct?
(a) Aliphatic amines are stronger bases than ammonia.
(b) Aromatic amines are stronger bases than ammonia.
(c) The alkyl group in alkyl ammonium ion more stabilizes the ion relative to the amine.
(d) The aryl group in aryl ammonium ion less stabilizes the ion relative to the amine.
39. Which of the following amines form N–nitroso derivative when treated with NaNO2 and HCl.
R
(a) CH3NH2 (b) NH2 (c) N⎯H (d) NH2

40. On adding KI to benzenediazonium chloride, the product obtained is


(a) benzene (b) 1-4-diiodobenzene (c) iodobenzene (d) 1,3,5-triiodobenzene
41. Which of the following reagents can convert benzenediazonium chloride into benzene?
(a) Water (b) Acid
(c) Hypophosphorous acid (d) HCl
42. Benzenediazonium chloride on reaction with phenol in weakly basic medium gives
(a) diphenyl ether (b) p-hydroxyazobenzene
(c) chlorobenzene (d) benzene
43. Towards electrophilic substitution, the most reactive is
(a) anilinium chloride (b) aniline (c) N-acetylaniline (d) nitrobenzene
44. Which of the followings undergoes diazotization?
(a) CH3NH2 (b) C6H5NH2 (c) CH3CONH2 (d) CH3N(CH3)2
45. Which of the following statements is not correct?
(a) The main product obtained in the reaction of chloroform with aniline in the presence of excess of alkali is
benzeneisonitrile.
(b) Amines are more basic than alcohols, ethers and esters.
(c) Nitrogen involves sp2 orbitals in bonding with other atoms or groups in amines.
(d) Amines are less basic than water and also less basic than hydroxide ion.
Organic Compounds Containing Nitrogen (Cyanides, Isocyanides, Nitrocompounds and Amines) 25.15

(a) Kolbe’s reaction (b) Riemer-Tiemann reaction


(c) Carbylamine reaction (d) Haloform reaction
47. Hinsberg’s reagent is
(a) phenylisocyanide (b) benzenesulphonyl chloride
(c) p-toluenesulphonic acid (d) o-dichlorobenzene
48. Which of the following compounds will dissolve in an alkali solution after it has undergone reaction with Hinsberg
reagent?
(a) (C2H5)2NH (b) (CH3)3N (c) CH3NH2 (d) C6H5NHC6H5
49. Which of the following can distinguish the three amines, viz., primary, secondary and tertiary?
(a) Azo-dye test (b) Hinsberg reagent (c) Carbylamine test (d) Acetyl chloride
50. Carbylamine test is performed in alcoholic KOH by heating a mixture of
(a) chloroform and silver powder (b) trihalogenated methane and a primary amine
(c) an alkyl halide and a primary amine (d) an alkyl cyanide and a primary amine
51. Hofmann’s method to separate amines in a mixture uses the reagent
(a) benzenesulphonyl chloride (b) diethyl oxalate
(c) benzeneisocyanide (d) p-toluenesulphonic acid
52. A positive carbylamine test is given by
(a) N, N-dimethylaniline (b) 2, 4-dimethylaniline
(c) N-methyl-o-methylaniline (d) p-methylbenzylamine
53. Which of the following statements is correct?
(a) Hinsgberg test involves the reagent benzenesulphonyl chloride, in the presence of aqueous potassium
hydroxide to distinguish primary, secondary and tertiary amines.
(b) Libermann reaction is shown only by primary amines.
(c) Primary amine gives liquid oxamide when treated with diethyloxalate.
(d) Secondary amine gives solid oxamide when treated with diethyloxalate.

ANSWERS
1. (a) 2. (b) 3. (a) 4. (a) 5. (d) 6. (d)
7. (b) 8. (c) 9. (c) 10. (d) 11. (a) 12. (c)
13. (c) 14. (a) 15. (c) 16. (a) 17. (a) 18. (d)
19. (d) 20. (a) 21. (a) 22. (d) 23. (c) 24. (d)
25. (c) 26. (b) 27. (a) 28. (b) 29. (a) 30. (b)
31. (b) 32. (d) 33. (a) 34. (b) 35. (b) 36. (c)
37. (b) 38. (b) 39. (c) 40. (c) 41. (c) 42. (b)
43. (b) 44. (b) 45. (d) 46. (c) 47. (b) 48. (c)
49. (b) 50. (b) 51. (b) 52. (d) 53. (a)

HINTS AND SOLUTIONS


1. The primary amine is RNH2.
2. Acetyl chloride converts —NH2 to —NHCOCH3, which reduces the activation of benzene ring.
3. Electron-releasing group increases the availability of lone pair of electrons on nitrogen while electron attracting
group decreases this availability.
25.16 Complete Chemistry—JEE Main

4. The solution effect makes (CH3)3N poor base as compared to CH3NH2 and (CH3)2NH.
5. More the number of electron-releasing groups attached to —NH2, the larger is its basic nature.
6. The substituent at ortho position has the effect of increasing acidity. Intramolecular hydrogen bonding in
o-nitrobenzoic acid makes it less acidic than o-aminobenzoic acid.
7. Electron-withdrawal group at para position makes phenol more acidic.
8. The —OH group causes decrease in the basicity of —NH2 group.
Br2, NaOH
9. CH3CONH2 Æ CH3NH2.
10. Chlorobenzene is prepared via diazonium salt.
11. Benzylamine is an aliphatic amine. It is most basic amongst the given compounds.
12. N has 10 electrons around it.
13. Tertiary amine does not contain H attached to N.
14. —OCH3 is electro-releasing group, it enhances the basicity. —NO2 is electron-attracting group, it decreases the
basicity.
15. —NO2 is more electron attracting then —CHO; —Br is more electron-releasing than its inductive effect.
16. —CH3 is more electron releasing than —Cl; —COCH3 is electron-attracting group.
17. There are only two resonating structures.
+ +
NH3 NH3

+ + +
NH2 NH NH NH NH2

19. C6H5CH2NH2 is an aliphatic amine. It is most basic amongst the given compounds.
20. Aniline is a weaker base. It is a Lewis base, i.e. the lone pair present on nitrogen can be given to the Lewis acids.
The electron-releasing group present at the para position increases the electron density on nitrogen and thus
makes it more basic. For example, para-toluidine is more basic than aniline. On the other hand, the presence of
electron-withdrawing group decreases the electron density on nitrogen and hence makes it less basic. For example,
p-nitroaniline is less basic than aniline.
21. (a) Guanidine is a strong base as there is a greater resonance stabilization of the cation [C(NH2)3]+. This is due to
the contribution from three equivalent structures with the accommodation of positive charge by three nitrogens.
+ + +
H2N NH2 H2N NH2 H2N NH2 H2N NH2
C C C C
+
NH2 NH2 NH2 NH2

(b) The lone pair on nitrogen in aniline is considerably delocalised to the benzene ring, making the molecule quite
stable. This stability is destroyed when H+ adds to the nitrogen, thereby decreasing the basicity of aniline. In
pyridine, lone pair is completely localized to nitrogen atom making it stronger base than aniline.
(c) The electron pair of H—N: is delocalised over the ring to provide the aromatic sextet. and thus is not a basic
site. The nitrogen of :N==C retains electron density and is a basic site.
(d) Purine has three basic sites. These are the three double-bonded nitrogen. The lone pair of NH2 and NH are
involved in delocalisation.
Organic Compounds Containing Nitrogen (Cyanides, Isocyanides, Nitrocompounds and Amines) 25.17

22. (a) Being more polar than nonpolar compounds, amines have higher boiling point. There also exist hydrogen bonds
amongst amines.
(b) Amines are less polar than alcohols and carboxylic acids.
(c) Aromatic amines are less basic than ammonia and aliphatic amines. Phenyl ring being electron attractor makes
lone pair on nitrogen less readily available to an acid. Aliphatic amines are more basic than ammonia.
(d) The molecule and its mirror image is rapidly interconvertible. The energy barrier between the two is very less.
23. (b) Aryl halides have low reactivity toward nucleophilic substitution.
(c) Ammonolysis gives good yield with primary amines as nucleophilic substitution predominates but is worthless
for tertiary amines as elimination reaction predominates to give alkenes.
CH3 CH3
NH3 – NH3
CH3CH2CH2Br Æ CH3CH2CH2NH3+ Br ; CH3 C Br Æ CH3— C ==CH2 + NH4Br
CH3

24. (a) Electron-releasing groups are aromatic-ring activators and they also push electrons toward nitrogen making
lone pair of electrons more readily available for an acid. Hence, they are base-strengthening groups. Electron-
withdrawing groups act in the opposite direction, hence, they are base-weakening group.
(b) Aliphatic amines form unstable diazonium salt which decompose to give nitrogen and aliphatic alcohols.
(c) Only primary amine can be distinguished as it produces carbylamine having most offensive smell
RNH2 + CHCl3 2O

25. (a) The correct order is NH3 < CH3CONH2 < (CH3CO)2NH
In the anion from ammonia, the negative charge is localized on nitrogen.
In the anion from acetamide, the negative charge is shared by nitrogen and one oxygen. In the anion from
diacetamide, the negative charge is shared by nitrogen and two oxygens.

most stable least stable

(b) p-Aminobenzoic acid does not exist as dipolar ion. An aromatic —NH2 group is a weak base and it cannot
neutralize the —COOH group. It can, however, neutralize the stronger acidic —SO3H group. Thus, sulphanilic
acid can exist as dipolar ion.
(d) In Hofmann elimination, a less substituted alkene is produced.
26. (a) In NF3, nitrogen acquires d+
nitrogen, the lone pair of electrons are more tightly held in NF3.
(b) In guanidine, the imino (==NH) nitrogen is sp2 hybridised while amino (—NH2) is sp3 hybridised. The proton
is likely to be attached to —NH2 as its nitrogen has less s-character. Actually the imino nitrogen is protonated
because this leads to the stable symmetrical resonating cation with very high delocalisation energy as there are
three contributing structures.

H2N H2N H2N + H2N


H+ +
C NH C NH2 C NH2 C NH2
H2N H2N H2N H2N +

Because of the large delocalisation energy, the cation is very much stabilised leading the guanidine to act as a
strong base.
(c) Ammonia involves more bonding pair-bonding pair electronic repulsion as the pairs are located near the
nitrogen atom due to its more electronegative character. In NF3, F is more electronegative and thus bonding
pair lies more near F leading to less electronic repulsion.
25.18 Complete Chemistry—JEE Main

(d) The larger the s-character of a hybrid orbital of carbon, the larger electron-withdrawing (by induction) effect
it has, consequently, the larger is its base-weakening effect.
27. (a) The adjacent C==O weakens the basicity by delocalisation of electron density from N to O.
(b) The ability of C==O to cause base-weakening is attenuated by the phenyl group, making benzamide a stronger
base than acetamide.
(c) Phenyl group has a base-weakening effect because of its ability to form extended p-bonding between the amino
N and the phenyl ring.
H
NH2 N
H
(I) (II)

In the structure II, N—H bonds are in the plane of benzene ring. In 2,6-dimethyl-N,N-dimethylaniline, the
formation of extended p-bonding is sterically hindered and thus interfere with the base-weakening extended
p-bonding making this compound much more basic than 2, 6-dimethylaniline.
(d) The nitro group is more effective than cyano group in weakening the basic nature of aniline.

O H H
N N N == C N
O H H
In nitroaniline, electron delocalisation ends up with the negative charge on the more electronegative oxygen
atom whereas in cyanoaniline, the negative charge ends up in the nitrogen atom (which is less electronegative
than oxygen).
28. (a) The base-weakening via electron delocalisation is achieved when N—O bonds of nitro group in nitroaniline are
in the same plane of benzene ring. In 3,4,5-trimethylaniline, the attainment of coplanarity is sterically hindered
and is thus not effective in base-weakening compared to 4-cyano-3-5-dinitroaniline. Cyano group, being linear,
does not encounter this hinderance.
(b) In pyrrole, the lone pair on nitrogen is involved in the delocalisation over the ring and thus there is
N ,
K°b = 2 ¥ 10–3)

because it has sp3 hybrid orbital (less s-character) while the N of pyridine (K°b = 2.3 ¥ 10–9) has sp2 hybrid orbital
(more s-character).

withdrawing N. Thus, the product is


N
29. A diazonium salt is produced. H
30. A nitrosamine is produced.
Cu2Cl2/HCl
31. Sandmeyer reaction is C6H5N2Cl Æ C6H5Cl + N2
32. In aqueous medium, tribromo derivative is obtained.
H2 O
33. C H N Cl
6 5 2 Æ C6H5OH + HCl + N2.
34. Sulphanilic acid is produced.
35. In Hofmann elimination, a less substituted alkene is formed as a result of the loss of more acidic BH (which follows
the trend 1° > 2° > 3°). The reaction is
CH3CH2CH2CH(CH3) N+(CH3)3 æDæ Æ CH3CH2CH2CH == CH2
36. 2° amine produces oily nitrosoamine.
37. Aryl halides is less reactive than alkyl halide.
Organic Compounds Containing Nitrogen (Cyanides, Isocyanides, Nitrocompounds and Amines) 25.19

38. Aromatic amines are lesser basic than ammonia due to resonance effect.
39. 2° Amine produces N-nitroso derivative.
40. Treating with KI produces iodobenzene.
41. Hypophosphorous acid is used.
42. The product is p-hydroxyazobenzene.
43. —NH +3Cl, —NHCOCH3 and —NO2 groups deactivate benzene ring towards electrophilic substitution reaction.
44. Aromatic Amine undergoes diazotization.
45. (d) Amines are more basic than water.
RNH2 + CHCl3 + 3KOH ææ Æ R—NC + 3KCl + 3H2O.
47. Hinsberg reagent is benzenesulphonyl chloride (C6H5SO2Cl).
48. RNH2 gives RSO2NHR. Due to H atom attached to N, it dissolves in alkali solution.
49. Hinsberg reagent can distinguish the three amines.
50. Carbylamine test is given by primary amine by using CHCl3 in alcoholic KOH medium.
51. Hofmann’s method uses diethyl oxalate as a separating reagent.
52. Primary amine exhibits carbylamine test.
53. (b) Secondary amines show Libermann reaction. (c) Solid oxamide is obtained.
(d) Liquid oxamide is obtained.

MULTIPLE CHOICE QUESTIONS FROM AIEEE AND JEE MAIN

1. The reaction of chloroform with alcoholic KOH and p-toluidine forms

[2003]
2. The correct order of increasing basic nature for the bases NH3, CH3NH2 and (CH3)2NH is
(a) CH3NH2 < (CH3)2NH < NH3 (b) CH3NH2 < NH3 < (CH3)2NH
(c) (CH3)2NH < NH3 < CH3NH2 (d) NH3 < CH3NH2 < (CH3)2NH [2003]
3. Which of the following is the strongest base?

(a) NH2 (b) NHCH3 (c) NH2 (d) CH2NH2

CH3 [2004]
4. Which one of the following methods is neither meant for the synthesis nor for separation of amines?
(a) Wurtz reaction (b) Curtius reaction (c) Hinsberg method (d) Hofmann method
[2005]
5. An organic compound having molecular mass 60 amu is found to contain 20% C, 6.67% H and 46.67% N, while
rest is oxygen. On heating it gives NH3 alongwith a solid residue. The solid residue gives violet colour with
alkaline copper sulphate solution. The compound is
(a) (NH2)2CO (b) CH3CH2CONH2 (c) CH3NCO (d) CH3CONH2
[2005]
6. In the chemical reaction,
25.20 Complete Chemistry—JEE Main

CH3CH2NH2 + CHCl3 + 3KOH æÆ (A) + (B) + 3H2O, the compounds (A) and (B) respectively are:
(a) C2H5CN and 3KCl (b) CH3CH2CONH2 and 3KCl
(c) C2H5NC and K2CO3 (d) C2H5NC and 3KCl [2007]
7. Which one of the following is the strongest base in aqueous solution?
(a) Trimethylamine (b) Aniline (c) Dimethylamine (d) Methylamine [2007]
8. A compound with molecular mass 180 u is acylated with CH3COCl to get a compound with molecular mass 390 u.
The number of amino groups present per molecule of the former compound is
(a) 2 (b) 5 (c) 4 (d) 6 [2013]
9. On heating an aliphatic primary amine with chloroform and ethanolic potassium hydroxide, the organic compound
formed is
(a) an alkyl isocyanide (b) an alkanol
(c) an alkanediol (d) an alkyl cyanide. [2014]
10. Considering the basic strength of amines in aqueous solution, which one has the smallest pKb value?
(a) C6H5NH2 (b) (CH3)2NH (c) CH3NH2 (d) (CH3)3N [2014]
11. Complete reduction of benzene-diazonium chloride with Zn/HCl gives:
(a) Aniline (b) Phenylhydrazine (c) Azobenzene (d) Hydrazobenzene
[2014]
12. Conversion of benzene diazonium chloride to chloro benzene is an example of which of the following reactions?
(a) Claisen (b) Friedel-craft (c) Sandmeyer (d) Wurtz [2014]
2 and HCl is:
(a) Diazomethane (b) Methylalcohol (c) Methylcyanide (d) Nitromethane [2014]
14. Which one of the following compounds will not be soluble in sodium bicarbonate ?
(a) 2, 4, 6-Trinitrophenol (b) Benzoic acid
(c) o-Nitrophenol (d) Benzene sulphonic acid [2014]
15. In a set of reactions p-nitrotoluene yielded a product E
CH3

Br2 Sn/HCl NaNO2 CuBr


B C D E
FeBr3 HCl HBr

NO2
The product E would be:
CH3 CH3 CH3 CH2Br
Br Br Br Br
(a) (b) (c) (d) [2014]
Br
Br Br Br
16. The major product of the reaction
NaNO2/H2SO4

NH2 OH
is:
(a) (b)
H OH O
Organic Compounds Containing Nitrogen (Cyanides, Isocyanides, Nitrocompounds and Amines) 25.21

(c) (d) [2014]


N
H
17. Arrange the following amines in the order of increasing basicity.
(a) NH2 NH2 NH2 (b) NH2 NH2 NH2




< < < CH3NH2 < < < CH3NH2



OCH3 NO2 OCH3 NO2

(c) NH2 NH2 NH2 (d) NH2 NH2 NH2





CH3NH2 < < < < < < CH3NH2



OCH3 NO2 NO2 OCH3

[2015, online]
18. In the reaction
NH2
NaNO2/HCl CuCN/KCN
D E + N2
0 – 5 °C D

CH3
The product E is:
COOH

(a) (b) H3C CH3

CH3

CN
CH3

(c) (d) [2015]

CH3
19. The test to distinguish primary, secondary and tertiary amines is:
(a) Sandmeyer’s reaction (b) Carbylamine reaction
(c) Mustard oil test (d) C6H5SO2Cl [2016, online]
20. The “N” which does not contribute to the basicity for the compound is.
6 7
1 5 N
N
8
2
4
N N
3 9
H
(a) N9 (b) N3 (c) N1 (d) N7 [2016, online]
25.22 Complete Chemistry—JEE Main

21. In the Hofmann bromamide degradation reaction, the number of moles of NaOH and Br2 used per mole of amine
produced are
(a) Four moles of NaOH and two moles of Br2 (b) Two moles of NaOH and two moles of Br2
(c) Four moles of NaOH and one mole of Br2 (d) One moles of NaOH and one mole of Br2 [2016]
22. Fluorination of an aromatic ring is easily accomplished by treating diazonium salt with HBF4. Which of the
following conditions is correct about this reaction?
(a) NaF/Cu (b) Cu2O/H2O (c) only heat (d) NaNO2/Cu

ANSWERS
1. (a) 2. (d) 3. (d) 4. (a) 5. (a) 6. (d)
7. (a) 8. (b) 9. (a) 10. (b) 11. (a) 12. (c)
13. (b) 14. (c) 15. (b) 16. (b) 17. (d) 18. (c)
19. (d) 20. (a) 21. (c) 22. (c)

HINTS AND SOLUTIONS


3. Aliphatic amine is more basic than aromatic amine.
4. Wurtz reaction deals with the production of alkane from alkyl halide by using sodium in dry ether.
5. The ratio of atoms is
C : H : N : O :: 20 : 6.67 : 46.67 : 26.26 :: 1.67 : 6.67 : 3.19 : 1.64 :: 1 : 4 : 2 : 1
12 1 14 16
Hence, empirical formula is CH4N2O
The reactions shown by the compound on heating is that of urea.
H2NCONH2 H—N==C==O + NH3
H2NCONH2 2NCONHCONH2
biuret
Biuret gives a violet colour with alkaline copper sulphate solution. Thus, the given compound is urea
H2NCONH2.
6. The involved reaction is CH3CH2NH2 + CHCl3 + 3KOH Æ C2H5NC + 3KCl + 3H2O
7. In aqueous solution, the base strength of aliphatic amines is decided by the following two factors.
Induction factor Increasing replacement of H by R (electron-releasing group) increases electron density on N, thus
making it more strong.
Solvation factor Stabilization of conjugate acid through H-bonding with water. More the number of H atoms,
more the stability and thus more basic the amine. Thus, induction and solvation operate in opposite directions.
Induction dominates to make all three alkylamines more basic than ammonia. However, adding a third R as in
R3N does not further enhance the basicity because the opposing solvation effect assumes more importance. The
electron withdrawing nature of phenyl makes aniline weaker acid than ammonia.
8. Let the compound be R(NH2)x. Its acetylation will give R(NHCOCH3)x. Their molecular masses will be
m1(R(NH2)x) = mR + (14u)x
m2 (R(CHCOCH3)x) = mR + (56 u)x
Increase in molecular mass = (56 u – 14 u)x = (42 u)x
Given increase in the molecular mass = 210 u
210 u
Hence, x = =5
42 u
Organic Compounds Containing Nitrogen (Cyanides, Isocyanides, Nitrocompounds and Amines) 25.23

9. There is formation of isocyanide. This is known as carbylamine test of primary amine.



RNH2 + CHCl3 + 3KOH Æ R — N ∫ C : + 3 KCl + 3H2O
10. Smallest pKb implies largest value of Kb. Aliphate amines are stronger than aromatic amine.
Among the given aliphatic amines, the order of basicity is.
(CH3)2NH > CH3NH > (CH3)3N
The solution effects makes (CH3)3N poor base as compared to CH3NH2 and (CH3)2NH.
11. Benzene -diazonium salt is converted to aniline when treated with Zn/HCl.
ArN +2 Cl- ææææ Æ [ArNHNH 2 ] ææææ
Zn HCl Zn HCl
Æ ArNH 2 + NH3 aniline
12. The reaction is known as Sandmeyer reaction. The diazonium compound is treated with cuprous chloride.
Ar - N +2 X - æææ
CuX
Æ Ar - X + N 2
13. Aliphate amines on reacting with NaNO2 and HCl produces an alcohol.
14. 2, 4, 6-Trinitrophenol is as acidic as a carboxylic acid or sulphonic acid. All these three are soluble in sodium
bicarbonate solution.
o-Nitrophenol will not be soluble in sodium bicarbonate solution
CH3 CH3 CH3 CH3 CH3
Br Br NaNO Br CuBrHBr Br
Br2 Sn/HCl 2
15. Fe Br3 HCl

NO2 NO2 NH2 N2+ Cl- Br


(B) (C) (D) (E)

CH3 CH3 CH3 CH3 CH3 CH3


NaNO2/H2SO4 -N2
16. CH3 C C OH CH3 C C OH CH3 C C OH
NH2 CH3 N2 CH3 CH3

CH3 CH3 CH3 CH3


CH3 C C O CH3 C C O H
CH3 CH3
17. Aliphatic amine is more basic than aromatic amine.
Electron-attracting group decreases the charge on N of aromatic amine.
Electron-releasing group enhances the charge on N of aromatic amine.
Hence, the increasing basicity will be

O2 N — — NH2 < — NH2 < CH3O — — NH2 < CH3NH2

18. The reaction are:


NH2 N +2 Cl – CN
NaNO2/HCl CuCN/KCN
0 – 5 °C

CH3 CH3 CH3


25.24 Complete Chemistry—JEE Main

19. Hinsberg reagent (benzene sulphonyl chloride, C6H5SO2Cl) may be used to distinguish amines.
Primary amine reacts to given a product which is soluble in KOH solution.
Secondary amine also reacts to give product which is insoluble in KOH solution.
Tertiary amine does not react.
Mustard oil test uses CS2 and HgCl2. Primary amine forms isothiocyanate which has pungent smell like mustard
oil.
Secondary and tertiary amine do not react.
Carbylamine test uses CHCl3 and alcoholic KOH. Only primary amine reacts to give isothiocynide having bad
smell.
20. The lone pair of electron on N – 9 is involved in delocalization and does not contribute in the basicity for the
compound.

21. The net reactions is RCONH2 + Br2 + 4NaOH Æ RNH2 + 2NaBr + Na2CO3 + 2H2O
22. Only heating of the solution is required.

Ar - N +2 X - æææ Æ Ar - N +2 BF4- æææ


HBF heat
4
Æ ArF + BE3 + N 2
26
Synthetic and Natural
Polymers

A polymer is simply a large molecule (i.e. macromolecule) built up by repetitive binding together of many smaller units
called monomers. Some of the natural polymers are polysaccharides (from glucose units), proteins (from amino acids)
and nucleic acids (from nucleotide units).

Homopolymer and Copolymer


Synthetic polymers derived from a single repeating monomer are known as homopolymers while those derived from
two or more type of repeating monomer units is known as copolymer.

Average Molar Mass of Polymer

the following.
 Ni M i  Ni M i2
i i
Number Average Molar Mass, Mn = Mass Average Molar Mass, Mm =
 Ni  Ni M i
i i

Based on the method of preparation, the following two types of polymers may be distinguished.

Chain-growth polymers (or addition polymers) are produced by chain-reaction polymerization in which an initiator
adds to a carbon-carbon double bond to yield a reactive intermediate. The polymer is built as more monomers add
successively to the reactive end of the growing chain.
H 2 C CH 2
In• + CH2==CH2 Æ [In — CH2CH2•] Æ [In — CH2CH2CH2CH•2]
Æ

Repeated many times to give a polymer

Step-growth polymers (or condensation polymers) are prepared by reaction between two functional groups and thereby
eliminating small molecules during polymerization.
26.2 Complete Chemistry—JEE Main

Polymers which have ability to return to their original shape after the removal of force which causes stretching in them
are known as elastomers. Natural rubber is one of the examples of an elastomer. Elastomers involve weak intermolecular
forces. Examples are buna–S, buna–N, neoprene, etc.
Fibre
Certain polymers in molten state when passed through a small hole in a die or spinneret can be made in thin thread which

A polymer which is hard at room temperature but becomes soft and viscous when heated is known as thermoplastic.
Examples are polyethylene, poly(vinyl chloride), polystyrene, etc. Thermoplastics are linear or slightly branched long

A polymer which becomes highly cross-linked and solidify into a hard, insoluble mass when heated is known as
thermosetting polymer(or resin). Examples are bakelite and urea-formaldehyde. It is widely used for moulded parts,
for adhesives and for coatings.

Table 1 describes some of the polymers formed from ethylene and substituted ethylenes. These are formed via free
radical mechanism initiated by free radicals generated by benzoyl peroxide, acetyl peroxide, tert-butyl peroxide, etc.

Table 1 Polymers formed from ethylene and substituted ethylens


Monomer Polymer Uses
H2C == CH2 Polyethylene
(Ethylene) sheets
H2C == CHCH3 Polypropylene Automotive molding, carpet, rope
(Propylene)
H2C == CHCl Poly(vinyl chloride)
(Vinyl chloride)
H2C == CHC6H5 Polystyrene
handles, combs, television coil, cups, bottle
(Styrene) (styron)

H2C == CHCN Orlon, Acrilan


(Acrylonitrile)
H2C == CHOH Poly(vinyl alcohol) Paints, adhesive
(Vinyl alcohol)
H2C == CHOCOCH3 Poly(vinyl a cetate)
(Vinyl Acetate)
H2C C (CH3) CO2CH3 Plexiglas, Lucite Aircraft windows, dentures, molded articles,
(Methyl methacrylate) paints, optical lenses, transparent domes
Values and gaskets, coatings
2 2
Synthetic and Natural Polymers 26.3

The polymerization reaction corresponds to 1, 4 addition to growing chains to each conjugate diene monomer. The
resultant polymer has a double bond after every four carbon atoms along the chain. These double bond may be either
cis or trans and the proper choice of Zeigler-Natta catalyst (an organometallic transition-metal complexes preparted by
treating a trialkyl aluminium with a titanium compound) allows the preparation of either of these two choices.
Examples

CH3
2n CH3 H CH3 H
C CH2 C == C C == C
H2C CH CH2 CH2 æ CH2 CH2 n
Isoprene Natural rubber
(2-Methyl-1, 3-butadiene)
CH3 CH3
C CH2 C CH2
CH2 C CH2 C n
H H
Gutta-percha

Cl Cl Cl
2n CH2 CH2
C CH2 C C
H2C CH CH2 C CH2 C n
H H
Chloroprene Neoprene (a synthetic rubber)

Both natural and synthetic rubbers are soft. They can be hardened by the process of vulcanization discovered by Charles
Goodyear (1834). In this process, rubber is heated with a per cent by mass of sulphur. This results into the formation of
sulphur bridges between polymer chains.
Examples of Copolymers
1. H2C==CHCl + H2C==CCl2 Æ Saran
Vinyl chloride Vinylidene
chloride

2. H2C==CHC6H5 + H2C==CHCH==CH2 Æ Buna-S


Styrene (25%) Butadiene (75%) (a rubber)
26.4 Complete Chemistry—JEE Main

3. H2C==CHCN + H2C==CHCH==CH 2 Æ Nitrile rubber


Acrylonitrile Butadiene

4. H2C==C(CH3)2 + H2C==C(CH3)CH==CH2 Æ Butyl rubber


Isobutylene Isoprene

seat belts, sails, packing for food, etc.

H
O N æ OH
N O
[O] NH2OH H2SO4

Cyclohexane Cyclohexanone Caprolactam


H
N O
O -H O
n 2
C æ NH æ (CH2)5 n

Because of six carbon atoms in the monomer, the present polymer is called nylon-6. It is used for the manufacture of
tyre cords and ropes.

H H O O
nH2N — (CH2)6— NH2 + nHOOC— (CH2)4— COOH N (CH2)6 N C (CH2)4 C n + n H2 O
Hexamethylene diamine Adipic acid
Nylon-66
Both the monomers have six carbon atoms, hence the name of the polymer is nylon-66. It is used as sheets, bristles for
brushes and in textiles.

HOOC COOH O O
n (HO æ CH2CH2 æ OH) + O æ CH2CH2OC C + n H2O
Ethylene glycol
Phathalic acid n
Gyptal resin

The starting materials are phenols and formaldehyde. In the presence of a basic catalyst, these two substances combine
to give ortho and para hydroxymethylphenol. These materials undergo condensation to produce cross-linked polymer
bakelite involving methylene bridges in ortho, para or both ortho and para positions.
Synthetic and Natural Polymers 26.5

It is formed by the condensation polymerization of melamine and formaldehyde.


It is used in the manufacture of unbreakable crockery.

Many polymers are not biodegraded in the environment and remain as such causing
accumulation of solid waste materials. However, a certain polymers are biodegraded in the environment. Examples
include (i) a copolymer (known as PHBV) formed between 3-hydroxybutanoic acid and 3-hydroxypentanoic acid:
26.6 Complete Chemistry—JEE Main

and (ii) a copolymer (known as nylon 2-nylon 6) formed between glycine and aminocaproic acid:
nH2N—CH2—COOH + nH2N (CH2)5COOH
Glycine Aminocaporic acid

⎯⎯
O


NH CH2 C NH C
n
O

MULTIPLE CHOICE QUESTIONS

1. Buna-S is a
(a) monomer (b) polymer (c) copolymer (d) dimer
2. Glucose is a monomer of
(a) proteins (b) rubber (c) plastics (d) starch and cellulose
3. Oligomer is a
(a) monomer (b) short-chain polymer (c) long-chain polymer (d) linear polymer
4. Which of the following is natural polymer?
(a) Bakelite (b) Nylon (c) Proteins (d) PVC
5. Which of the following is a synthetic polymer?

6. Which of the following is an example of additional polymerization?

7. Which of the following is an example of condensation polymerization?


(a) PVC (b) BUNA rubber (c) Dacron (d) Lutrex
8. Which of the following is an example of elastomers?

10. Which of the following is an example of thermoplastic polymer?

11. Which of the following is an example of thermosetting polymer?

12. The repeating unit of polystyrene is


(a) CH2 CH (b) CH2 CH (c) CH2 CH 2 2—

Cl C6 H5 CN
Synthetic and Natural Polymers 26.7

(a) CH2 CH (b) CH2 CH (c) CH2 CH 2 2—

Cl C6H5 CN
14. The monomer of neoprene is
(a) chloroprene (b) styrene (c) vinyl chloride (d) adipic acid
15. The repeating unit of polyacrylonitrile is
(a) CH2 CH (b) CH2 CH (c) CH2 CH (d) CH2 CH
OH CN C6H5 CH3
16. The repeating unit of polyethylacrylate is
CH3
(a) CH2 CH (b) CH2 CH (c) CH2 CH (d) CH2 C
OH C6 H 5 COOC2H5 COOCH3
17. Which of the following structures represents tereylene?
O O O
(a) C NH (CH2)5 n (b) OCH CH2 O C C 6H 4 C n

(c) CH2 CH CH CH2 CH CH2 (d) (—CH2—CH==CH—CH2—)n


C 6H 5 n
18. Which of the following structures represent nylon-66?
O O O
(a) C NH (CH2)5 n (b) OCH CH2 O C C 6H 4 C n

(c) (—NH—(CH2)6—NH—CO—(CH2)4CO—)n (d) CH2 CH CH CH2 CH CH2


C 6H 5 n
19. Glyptal resins is formed from the monomers
(a) adipic acid and hexamethylene diamine (b) ethylene glycol and phthalic acid
(c) phenol and formaldehyde (d) ethylene glycol and terephthalic acid
20. Nylon-66 is formed from the monomers
(a) adipic acid and hexamethylenediamine (b) ethylene glycol and phthalic acid
(c) phenol and formaldehyde (d) ethylene glycol and terephthalic acid
21. Bakelite is formed from the monomers
(a) adipic acid and hexamethylenediamine (b) ethylene glycol and phthalic acid
(c) phenol and formaldehyde (d) ethylene glycol and terephthalic acid
22. Tereylene is formed from the monomers
(a) adipic acid and hexamethylenediamine (b) ethylene glycol and phthalic acid
(c) phenol and formaldehyde (d) ethylene glycol and terephthalic acid
23. Melamine formaldehyde resin is
(a) dimer of melamine and formaldehyde (b) an additional polymer

24. The monomer of nylon-6 is


(a) cyclohexane (b) caprolactam (c) ethylene glycol (d) amino acid
25. Which of the following polymers has ester linkages?
(a) Nylon (b) Bakelite (c) Terylene (d) PVC
26. Which of the following polymers has amide linkages?
(a) Nylon (b) PVC (c) Terylene (d) Bakelite
26.8 Complete Chemistry—JEE Main

27. The process of vulcanization makes rubber


(a) soluble in water (b) hard (c) soft (d) less elastic
28. Isoprene is a monomer of
(a) starch (b) synthetic rubber (c) natural rubber (d) PVC
29. Which of the following expressions represents the mass average molar mass?
S Ni Mi2 S Ni Mi2 S N i Mi S N i Mi
(a) M = (b) M = (c) M = (d) M =
S Ni S Ni Mi S Ni S Mi
30. Which of the following expressions represents the number average molar mass?
S Ni Mi2 S Ni Mi2 S N i Mi S N i Mi
(a) M = (b) M = (c) M = (d) M =
S Ni S Ni Mi S Ni S Mi
31. Which of the following methods will yield mass average molar mass?
(a) Elevation in boiling point (b) Osmotic pressure
(c) Depression in freezing point (d) Light scattering
32. Which of the following polymers does not involve cross linkages?
(a) Bakelite (b) Melamine (c) Polyethene (d) Vulcanized rubber
33. PVC stands for
(a) Polyvinyl carbinol (b) Polyvinyl chloral (c) Polyvinyl chloride (d) Polyvinyl chloroform

35. Which of the following is isoprene?


CH2 C CH CH2
(a) CH2==CH—CH==CH2 (b)
CH3
CH2 C C CH2 CH2 C CH CH2
(c) (d)
CH3 CH3 Cl
36. One of the constituents in the preparation of thiokol is
(a) 1, 2-dichloroethane (b) isoprene (c) chloroprene (d) sulphur
37. Which of the following statements is not correct?
(a) High pressure polymerization of ethylene in the presence of a free radical initiator gives highly branched
polymer
(b) The density of high pressure polymerized ethylene is more than that of low pressure polymerized ethylene
(c) Polymerization of ethylene at low temperature and pressure results into a linear polymer with much less
branched chains
(d) Low pressure polymerization of ethylene can be done at a lower temperature using a Ziegler catalyst
38. The repeating unit of melamine polymer is
(a) HN N NH CH2 n (b) HN N NH CH2 n

N N N N

NH2

(c) HN N CH2 NH n (d) HN N CH2 NH n

N N N N

NH2
Synthetic and Natural Polymers 26.9

39. Thiokol is a

40. Which of the following is chloroprene?


(a) CH CH2 CH CH2 (b) CH2 C CH CH2
Cl Cl
(c) CH3 C CH CH3 (d) CH3 CH2 CH CH3
Cl Cl
41. The repeating unit of thiokol is
(a) (—CH2—CH2—S—)n (b) (—CH2—CH2—S—S—CH2—)n
(d) (—CH2—S—S—CH2—)n (d) (—CH2—CH2—S—S—CH2—CH2—)n

2 2 (b) Cl C C F (c) H2C C H 2C==CCl2


F F Cl
43. Ziegler catalyst is
(a) chromium oxide supported over silica
(b) triethyl aluminium and titanium tetrachloride dispersed in an inert solvent
(c) alumina
(d) platinum/pladium
44. Which of the following polymers has the empirical formula identical with that of its monomer?

45. Which of the following polymers has molecular mass an integral multiple of molecular mass of its monomer?
(a) Buna rubber (b) Glytpal (c) Tereylene (d) Nylon 66

ANSWERS
1. (c) 2. (d) 3. (b) 4. (c) 5. (d) 6. (b)
7. (c) 8. (a) 9. (b) 10. (c) 11. (d) 12. (b)
13. (d) 14. (a) 15. (b) 16. (c) 17. (b) 18. (c)
19. (b) 20. (a) 21. (c) 22. (d) 23. (c) 24. (b)
25. (c) 26. (a) 27. (b) 28. (c) 29. (b) 30. (c)
31. (d) 32. (c) 33. (c) 34. (a) 35. (b) 36. (d)
37. (b) 38. (a) 39. (c) 40. (b) 41. (d) 42. (a)
43. (b) 44. (a) 45. (a) 46. (c) 47. (d)

HINTS AND SOLUTIONS


1. Buna-S is a copolymer. It involves two monomers styrene (H2C==CHC6H5) and butadiene (H2C==CHCH==CH2).
2. Starch and cellulose involves monomers glucose.
3. Oligomer is a short-chain polymer.
4. Proteins is a natural polymer.
5. Terelyne is a synthetic polymer.
26.10 Complete Chemistry—JEE Main

7. Dacron is an example of condensation polymerization.

10. PVC is an example of thermoplastic polymer.


11. Bakelite is an example of thermosetting polymer.
12. —CH2—CH(C6H5)— is the repeating unit in polystyrene.
2 2
14. Chloroprene is the monomer of neoprene.
15. —CH2—CH(CN)— is the repeating unit of polyacrylonitrile.
16. —CH2—CH(COOC2H5)— is the repeating unit of polyethylacrylate.
17. —OCHCH2OCOC6H4CO— is the repeating unit of tereylene.
18. —NH(CH2)6NHCO(CH2)4CO— is the repeating unit of nylon-66.
19. Ethylene glycol and phthalic acid.
20. Adipic acid and hexamethylene diamine.
21. Phenol and formaldehyde
22. Ethylene glycol and terephthalic acid.
23. Melamine formaldehyde resin is a copolymer.
24. Caprolactam is monomer of nylon-6.
25. Terylene has ester linkages.
26. Nylon has amide linkages.
27. Vulcanization makes rubber hard.
28. Isoprene is a monomer of natural rubber.
M = Si Ni M 2i /Si Ni Mi.
M = Si Ni Mi/Si Ni.
31. Light scattering.
32. Polyethene is a linear polymer.
33. PVC stands for polyvinyl chloride.

35. H2C==C(CH3)—CH==CH2.
36. One of the constituents in the preparation of thiokol is sulphur.
37. It is the reverse of the fact given in choice (b).
38. HN N NH CH2 is the repeating unit of melamine polymer.

N N

NH2
39. Thiokol is rubber.
40. CH2==C(Cl)CH==CH2 is chloroprene.
41. —CH2CH2SSCH2CH2— is the repeating unit of thiokol.
2 2
43. Ziegler catalyst is teriethyl aluminium and titanium tetrachloride dispersed in an inert solvent.

45. Buna rubber.

47. Nylon-66.
Synthetic and Natural Polymers 26.11

MULTIPLE CHOICE QUESTIONS FROM AIEEE AND JEE MAIN

1. Complete hydrolysis of cellulose gives


(a) L-glucose (b) D-fructose (c) D-ribose (d) D-glucose [2003]
2. Nylon threads are made of
(a) polyethylene polymer (b) polyvinyl polymer
(c) polyester polymer (d) polyamide polymer [2003]
3. Which of the following is a polyamide?
[2005]
4. The term anomers of glucose refers to

(c) a mixture of (D)-glucose and (L)-glucose


(d) enantiomers of glucose [2006]
5. a-D-(+)-glucose and b-D-(+) glucose are
(a) enantiomers (b) conformers (c) epimers (d) anomers [2008]
6. Bakelite is obtained from phenol by reacting with
(a) HCHO (b) (CH2OH)2 (c) CH3CHO (d) CH3COCH3 [2008]
7. Buna-N synthetic rubber is a copolymer of
(a) H2C == CH — CN and H2C == CH — CH == CH2
(b) H2C == CH — CN and H2C == CH — C == CH2
CH3
(c) H2C == CH — C == CH2 and H2C == CH — CH == CH2
Cl
(d) H2C == CH — CH == CH2 and H5C6 — CH == CH2 [2009]
8. The two functional groups present in a typical carbohydrate are
(a) — C == O and —OH (b) —OH and —CHO (c) —OH and —COOH (d) —CHO and —COOH
[2009]
9. The polymer containing strong intermolecular forces e.g. hydrogen bonding is
[2010]
10. Biuret test is not given by
(a) proteins (b) carbohydrates (c) polypeptides (d) urea [2010]
11. The change in the optical rotation of freshly prepared solution of glucose is known as
[2011]
12. Thermosetting polymer bakelite is formed by the reaction of phenol with
(a) CH3CH2CHO (b) CH3CHO (c) HCHO (d) HCOOH [2011]
13. The species which can best serve as an initiator for the cationic polymerization is
(a) LiAlH4 (b) HNO3 (c) AlCl3 (d) BuLi [2012]

(a) Acrylonitrile (b) Dacron (c) Neoprene (d) Teflon [2014]


15. Which one of the following is an example of thermosetting polymers ?
(a) Neoprene (b) Buna -N (c) Nylon 6, 6 (d) Bakelite [2014]
26.12 Complete Chemistry—JEE Main

16. Structure of some important polymers are given. Which one represents Buna-S ?
CH3
(a) ( CH2 C CH CH2 )n
(b) ( CH2 CH CH CH2 CH CH2 ) n
C6H5
(c) ( CH2 CH CH CH2 CH CH2 ) n
CN
Cl
(d) ( CH2 C CH CH2 ) n

17. Which one of the following class of compounds is obtained by polymerization of acetylene ?
(a) Poly-yne (b) Poly-ene (c) Poly-ester (d) Poly-amide

18. Match the polymers in with their main uses in and choose the correct answer:

(c) Polyvinyl chloride (iii) Manufacture of toys


(d) Bakelite (iv) Computer discs
(a) (A)-(ii), (B)-(i), (C)-(iii), (D)-(iv) (b) (A)-(iii), (B)-(i), (C)-(ii), (D)-(iv)
(c) (A)-(ii), (B)-(iv), (C)-(iii), (D)-(i) (d) (A)-(iii), (B)-(iv), (C)-(ii), (D)-(i)
19. Which one of the following structures represents the neoprene polymer?
(a) ( CH 2 -- C == CH -- CH 2 ) n (b) ( CH 2 -- CH ) n
Ω Ω
Cl CN
(c) ( CH 2 -- CH )n (d) ( CH 2 -- CH 2 )n
Ω Ω
Cl C6H5
[2015]
(a) Bakelite (b) Glyptal (c) Polypropene (d) Polyvinyl chloride
21.
Mechanical and aesthetic properties of cellulose can be improved by acetylation.
(a) Both assertion and reason are correct, but the reason is not the correct explanation for the assertion.
(b) Both assertion and reason are correct, and the reason is the correct explanation for the assertion.
(c) Assertion is incorrect statement, but the reason is correct.
(d) Both assertion and reason are incorrect.

23. Which of the following statements about low density polythene is false?
(a) It is a poor conductor of electricity

(c) It is used in the manufacture of buckets, dust-bin, etc.


[2016]
Synthetic and Natural Polymers 26.13

ANSWERS
1. (d) 2. (d) 3. (d) 4. (a) 5. (d) 6. (a)
7. (a) 8. (a) 9. (c) 10. (b) 11. (d) 12. (c)
13. (c) 14. (b) 15. (d) 16. (b) 17. (b) 18. (b)
19. (a) 20. (b) 21. (a) 22. (d) 23. (c)

HINTS AND SOLUTIONS


3. (d)
Nylon-66 contains amide linkages.

formaldehyde.
5. Two isomers which are mirror image of each other are known as enantiomers.
epimers.
Examples are D-ribose and L-ribose.
The formation of hemiacetal in a sugar generates a asymmetrical carbon with the generation of two epimers in
anomers.
The anomer with the — OH group above the plane of ring is called b anomer and the other as a anomer.
6. Bakelite is formed by the union of phenol and formaldehyde.
7. The repeating monomer units of Buna-N rubber are
H2C == CH — CN and H2C == CH — CH == CH2
Acrylonitrile Butadiene

8. A typical carbohydrate such as glucose contains —CHO and —OH groups.


The choice (a) is also correct since fructose contains C == O and —OH groups.

styrene monomers Nylon-6, 6 involves hexaethylene diamine and adipic acid as monomers. Nylon-6, 6 involves
polar units —NH—CO— which will have hydrogen bondings.
10. The biuret is NH2CONHCONH2
copper sulphate solution, a violet colour is produced. This is known as biuret reaction, which is characteristic of
all compounds containing the grouping —CONH—, e.g., proteins, polypeptide and urea (which on gently heated
form biuret).

12. Bakelite is formed by the union of phenol and formaldehyde.


13. Lewis acid such as AlCl3 is used to initiate cationic polymerization.
Lewis base such as BuLi is used to initiate anionic polymerization.
14.
Dacron is a step-growth polymer which is formed by the condensation of ethylene glycol and terephthalic acid.
15. Bakellite is a thermosetting polymer.
26.14 Complete Chemistry—JEE Main

16. Buna-S is formed from H2C = CH C6H5 (styrene) and H2C = CHCH = CH2 (butadiene)
n H 2C CHCH CH2 + n CH CH2 ( H2C CH CH CH2 CH CH2 ) n

C6H5 C6H5
Buna-S
H H
17. n H C C H C C
n
Poly-ene
18. Polystyrene is used in the manufacture of toys (matching: (A) – (iii))

Polyvinyl chloride is used in making rain coats (matching: (C) – (ii))


Bakelite is used for making computer discs.
Thus, choice (b) is the correct matching.
19. Neoprene is copolymer of chloroprene
n[CH 2 == C -- CH == CH 2 ] - ( CH 2 -- C == CH -- CH 2 )n
Ω Ω
Cl Cl

HOOC COOH
O O
n (HO – CH2CH2OH) + O CH2CH2 O C C + nH 2 O

Ethylene glycol Phthalic acid


n
Glyptal resin
21. Choice (a) holds good.
22. Chain - growth polymer is produced by chain-reaction polymerization in which an initiator (a free radical) adds
to a carbon-carbon double bond to give a reactive intermediate. The polymer is built as more monomers add suc-

chain-growth polymer.
Terylene, melamine and nylon 6, 6 are the examples of step-growth polymers.
23. Low-density polythene is not used in the manufactur of buckets, dust-bins, etc. it is used for manufacturing
27
Biomolecules and
Biological Processes
Carbohydrates
As the name implies, carbohydrates means “hydrates of carbon.” These are, in fact, polyhydroxylated aldehydes and
ketones.

Monosaccharides
These cannot be broken into smaller carbohydrates by hydrolysis. The general formula is CnH2nOn n varying from

Oligosaccharides

Polysaccharides
These include more than ten monosaccharides linked together.

Monosaccharides

CH2OH
CHO C O
(H C OH)n (H C OH)n
CH 2OH CH2OH
A polyhydroxy A polyhydroxy
aldehyde ketone

polyhydroxy ketones.

Examples
CHO CHO CH2OH CH2OH
CHOH (CHOH)2 CO CO
CH2OH CH2OH CH2OH (CHOH)2
CH2OH
Aldotriose Aldotetrose Ketotriose Ketopentose
27.2 Complete Chemistry—JEE Main

Fischer Projection Formula

1
CHO
1
CHO 2
H 2 OH C
H OH
HO 3 H 3
4 C
CH2OH HO H
4
CH2OH

Note The numbering of carbon atoms is done from the top to bottom of the vertical line containing carbon atoms.
Stereoisomers

2n n is the number of asymmetric carbon atoms in the molecule of carbohydrate.


If the hydroxyl group at the asymmetric carbon placed at the bottom of Fischer projection lies to the right, the

structures.
CHO CHO
H OH HO H
CH2OH CH2OH
D (+)-Glyceraldehyde L (–)-Glyceraldehyde

Cyclic Structures

properly positioned in the same molecule, there can occur an intramolecular rearrangement forming a cyclic hemiacetal
structure.

hydroxyl group due to the geometric constraints and the stability


4
of the product. Note that –OH at C lies to the right in a
1
b-structure.
Haworth Projection
Biomolecules and Biological Processes 27.3

it is the carbonyl carbon in the open-chain structure.

4. The terminal CH2


Examples

Chair Conformational Formula

Example
CH2OH H
O CH2OH
HO O
OH
H H
HO OH HO H
OH HO
H OH

Mutarotation
27.4 Complete Chemistry—JEE Main

When either a-form or b


+ 52.6o is obtained. Thus a-form changes to b a-form
b
Reducing and Nonreducing Sugars

to red precipitate of cuprous oxide.

reagent even it contains no aldehydic group. This occurs because fructose is readily isomerized to an aldose in basic solution:
CH2OH CHOH CHO COOH
C == O C OH CHOH CHOH
HO H -OH/H HO H –OH/H HO H Ag+ HO H
2O 2O
H OH H OH H OH H OH
NH4OH
H OH H OH H OH H OH
CH2OH CH2OH CH2OH CH2OH
D-Fructose An enediol An aldohexose An aldonic acid

dilute nitric acid oxidizes both the adehydic group at C1 and the terminol — CH2OH group to —COOH groups giving
aldaric acid.
Other Characteristic Reactions
Aldehydic or keto group in sugars can be reduced to alcohol by using sodium borohydride. The hydroxyl groups in

acidic solution, only the methylated methyl glycoside can by hydrolyzed and the ring is opened
Note If anomeric — OH is above the plane of ring, it is said to form b
a-glycoside.
Degradation with HIO4
4
—CHO and CHOH are converted into HCOOH
—CH2OH is converted into HCHO
—CO— is converted into CO2
For example,
HIO4
HOCH2(CHOH)4CHO 5HCOOH + HCHO
HIO4
HOCH2(CHOH)3COCH2OH 2HCHO + 3HCOOH + CO2
Biomolecules and Biological Processes 27.5

Reaction with Phenylhydrazine

H— C NNHPh H—C NNHPh H C NNHPh


H C O
OH H PhNHNH2 HO H PhNHNH 2 C O PhNHNH2 C NNHPh

(H OH)3 –PhNH2, –NH 3 (H OH)3


(H OH)3 (H OH)3

CH2OH CH2OH CH2OH CH2OH


an osazone

Via osazone, an aldose may be converted into ketose.


H C O CH2OH
H C NNHPh
C O C O
C NNHPh 2 PhCHO
H OH)3 Zn/HAc (H OH)3
(H OH)3 – PhCH NNHPh
CH2OH CH2OH
CH2OH
(Osone)
The more reactive aldehyde group of the osone is reduced, not the less reactive keto group.
Oxidation Reactions

more vigorous oxidizing agent nitric acid brings about the oxidation of both —CHO and —CH2OH giving aldaric acid.

COOH CHO COOH


Br2 + H2O HNO 3
(CHOH) n (CHOH)n (CHOH)n
CH2OH CH2 OH COOH
Aldonic acid Aldose Aldaric acid

Disaccharides

other sugar. Common disaccharides contain C1— C4' linkage. The glycosidic bond can be either alpha or beta. Some

Cellobiose Maltose

'-b '-a
27.6 Complete Chemistry—JEE Main

Lactose Sucrose

'-b '

+66.5o o

and fructose are linked via 1, 2' glycosidic bond.

Polysaccharides
Polysaccharides contain more than ten simple sugars linked together through glycosidic bonds. These are nonreducing

Cellulose
Formed from D-glucose via 1, 4'-b-glycoside bonds.

CH2OH
O
CH2OH O
CH2OH O OH
O
O OH
O OH
OH
O OH
OH

Starch
Formed from D-glucose via 1, 4'-a-glycoside bonds.

CH2OH CH2OH CH2OH


O O O
OH OH OH
O O O O

OH OH OH

units via 1, 6'-a


Biomolecules and Biological Processes 27.7

Amino Acids and Peptides


Amino acids contain an amino group attached to a-carbon of a carboxylic acid. These are formulated as
O
H2N CH C OH
R

Table 1 Twenty amino acids


Name Abbreviation Structure Isoelectric point
Amino Acids with Nonpolar R Group
Glycine G or Gly H2N CH COOH
H
Alanine A or Ala H2N CH COOH
CH3
Valine V or Val H2 N CH COOH
CH(CH3)2
Leucine H2N CH COOH
CH2CH(CH3)2
Isoleucine I or Ile H2N CH COOH
CH(CH3)CH2CH3
Methioine M or Met H2N CH COOH 5.7

CH2CH2SCH3
Phenylalanine F or Phe H2N CH COOH 5.5

CH2C6H5
Proline P or Pro HN CH COOH 6.3

CH2 CH2
CH2
Tryptophan W or Trp H2N CH COOH 5.9

H2C

N
H
Amino Acids with Polar R Group
Serine S or Ser H2N CH COOH 5.7

CH2OH
Threonine T or Thr H2N CH COOH 5.6

CH(CH3)OH

Contd
27.8 Complete Chemistry—JEE Main

Name Abbreviation Structure Isoelectric point


Tyrosine Y or Tyr H2N CH COOH 5.7

H2 C OH

Cysteine C or Cys H2N CH COOH


CH2SH
Glutamine Q or Gln H2N CH COOH 5.7
CH2CH2CONH2
Asparagine N or Asn H2N CH COOH 5.4

CH2CONH2
Amino Acids X with Charged R Groups
Aspartic acid D or Asp H2N CH COOH
CH2COO–
Lysine H2N CH COOH 9.7

CH2(CH2)3NH3+
Histidine H or His H2N CH COOH 7.6
H
N
CH2
N H
+

Glutamic acid E or Glu H2N CH COOH 3.2

CH2CH2COO–
Arginine H2N CH COOH NH2
+
CH2(CH2)2NH C NH2

Essential Amino Acids

Chiral Nature of Amino Acids

Acid-Base Behaviour of Amino Acids


@
+
H3N CH CO2H
R
On increasing pH of the solution, the carboxyl group ionizes to produce a dipolar ion:
+
H3N CH COO–
R
Biomolecules and Biological Processes 27.9

If the pH of the solution is further increased, the amino group is deprotonated to give
H2N CH COO–
R

+ OH– + OH–
H3N CH CHCOOH H3N CHCOO– H2 NCHCOO–
H+ H+
R R R
Low pH Intermidiate pH High pH

+
[ H 3N CH(R)COO ][H ] [ H 2 N CH(R)COO ][H + ]
Kal = +
; Ka2 = +
[ H 3N CH(R)COOH] [ H 3 N CH(R)COO ]

[ H2 N CH(R)COO ][H + ]2
Thus Ka1Ka2 = +
[ H 3N CH(R)COOH]
– +
At the isoelectric point, [H2 ] = [H3N
Thus Ka1 Ka2 = [H+]2 K°a1 + pK°a2
K°a 2.5 and pK°a2 = 9.5. Thus, at isoelectric
point, pH

+ OH – +
– OH – OH –
H3NCHCOOH H3NCHCOO H2NCHCOO– H2NCHCOO–
+ + +
+ H + H + H
(CH2)4NH3 (CH2)4NH3 (CH2)4NH3 (CH2)4NH2
If the side chain contains acidic group, then

+ OH – + OH – + OH –
H3NCHCOOH H3NCHCOO– H3NCH COO– H2NCHCOO–
+ + +
H H H
(CH2)2COOH (CH2)2COOH (CH2)2COO– (CH2)2COO–

Acidic side chain < neutral chain < basic side chain

Peptide Linkage
2O produces CO —

–H2O
H2NCHCOOH + H NH CH2COOH Æ H2NCHCO NHCH2COOH
Gly
CH3 CH3
Ala Ala · Gly

right-end carries a carboxyl group.


27.10 Complete Chemistry—JEE Main

Geometry of a Peptide Linkage


Due to the resonance

Proteins and Enzymes


Proteins are polyamides formed from aminoacids and are essential constituents of all living organisms. The relative

and its biological function.

The secondary structures include a-helix, triple helix, and b-pleated sheet structures. The tertiary structure is the shape

collagens
elastins
keratins
enzymes
hormones antibodies

Nucleic Acids—DNA and RNA

from generation to generation and also control and direct the biosynthesis of all proteins.


and thus
is called polynucleotides.

heterocylic base heterocylic base


n Æ
— sugar — phosphate — — sugar — phosphate —n

† Heterocyclic base is attached to sugar at 1¢ position. This unit is called nucleoside. When the latter is linked to
phosphoric acid at 5¢ position of sugar, the resultant unit is called nucleotide.
Biomolecules and Biological Processes 27.11

These are

NH2 O NH2

N H N
N N N

N N H 2N N N O N
H H H
Adenine (A) Guanine (G) Cytosine (C)
(6-Aminopurine) 2-Amino-6-oxypurine (2-Oxy-4-aminopyrimidine)

O O
H CH3 H
N N

O N O N
H
Thymine (T) Uracil (U)
(5-Methyl)-2,4-dioxypyrimidine 2,4-Dioxypyrimidine

DNA polymers include A, G, C and T bases.

' carbon of one sugar ring and the 5' carbon of

rings. The secondary structure describes the actual arrangements of chains of nucleotides in three dimensions. DNA

Biological Functions of Nucleic Acids—Protein Synthesis and Replication

Translation
DNA æTranscription
ææææææ Æ ææææææ Æ Protein
Step 1 Step 2

amino acids.

cell is passed on to the daughter cells.


Lipids—Structure, Membrates and their Functions

3 2 n—
27.12 Complete Chemistry—JEE Main

unit for the membranes that surrounds all the cells in the human body and in other animals.

MULTIPLE CHOICE QUESTIONS

Carbohydrates

nH2nOn

and starch

compounds

2 n

a-D-glucose is +11.2°
b-D-glucose is +52.7°

a-Glucose is readily digested by human beings and not that of b-glucose


b-Glucose is readily digested by human being and not that of a-glucose
a- and b-glucoses are readily digested by human beings
a- and b-glucose are not readily digested by human beings
Biomolecules and Biological Processes 27.13

b o

a o

13. Maltose is formed by the union of

14. Sucrose is formed by the union of

17. Carbohydrates are the compounds of

a-glucose
a-amylose and amylopectin
27.14 Complete Chemistry—JEE Main

Amino acids, proteins, etc.

— NH2 and —COOH groups are attached to different carbon atoms

29. The number of polypeptide chains in insulin is

a-helix is more stable than left-handed a-helix

a-helix are perpendicular to the axis of the helix

the surface resulting into a spheroidal shape

36. The protein molecule present in haemoglobin is


Biomolecules and Biological Processes 27.15

— CO—NH—.

41. Nucleic acids are


H
2O
CH

O H
CH H C

HC C OH
H H

base base

— sugar — phosphate — sugar — phosphate —


27.16 Complete Chemistry—JEE Main

NH2 O

C H C N
N N C
N C
CH CH
C C C C
N H N N
H N
H H
NH2 O
C H C CH3
N CH N C
C CH C C
O N O N H
H H

NH2 O

C H C N
N N C
N C
CH CH
C C C C
N H2 N N N
H N
H H
NH2 O
C H C CH3
N CH N C
C CH C C
O N O N H

NH2 O O O

C H C H C H H C CH 3
CH3 N C N C
N CH N C

C CH C C C C C C
O N O N H O N H O N CH 3

H H H H

NH2 O O O
C H C CH3 H C H H C CH3
N CH N C N C N C
C CH C C C C C C
O N O N H O N H O N CH3
H H H H
Biomolecules and Biological Processes 27.17

NH2 O O O

C C C C CH3
CH3 HN C
N CH HN C HN CH

C CH C CH C CH C C
O N O N O N O N CH3

H H H H
52. Structure of a DNA molecule is

Fates, Waxes, Enzymes, Vitamins, etc.

56. Waxes are the esters of

57. A cell membrane is made up of

61. The constituents of phospholipids are

62. The hydrolysis of triglycerides produces


27.18 Complete Chemistry—JEE Main

amino acids by peptidases


Biomolecules and Biological Processes 27.19

2 and H2O takes place in the mitochondria of cells

fermentation

1 12

1 12
27.20 Complete Chemistry—JEE Main

1, D2 and D3.

ANSWERS
Biomolecules and Biological Processes 27.21

HINTS AND SOLUTIONS

nH2nOn.
The formula of glycerol is C3H O6.
3. Monosaccharides do not have free carbonyl groups and are involved in acetal or ketal formation. The hydrolysis
of the latter produces hydroxy carbonyl compounds.

5. Deoxyribose is a pentose

b o

9. a b
digestable.
a-D-glucose or b- D-glucose attains is + 52.7o
a-methyl glycoside and b-methyl glycoside.
12. Dextrose is monosaccharide.
13. Maltose is f
14. Sucrose is formed by the union of one molecule of each of glucose and fructose.

19. Amylose is a linear polymer.

22. Same as Q. 19.


23. Amylopectin is a branched polymer of glucose.
24. Glycogen is mainly present in liver and muscles of the animals and serves as a reserve carbohydrate.
25. In amino acids,—NH2 and—COOH groups are attached to the same carbon atom.

synthesized by human body.


27. Essential amino acids are not synthesized by human body. These have to be supplied from outside in the diet. The

34. The hydrogen bonds are parallel to the axis of the helix.
35. Many type of forces are involved in the tertiary structure of proteins. Examples include hydrogen bonds, disulphide
bridges, ionic or salt bridges and hydrophobic interactions.
37. Denaturation makes the protein less active.
27.22 Complete Chemistry—JEE Main

42. Deoxyribonucleic acid is a polymer of deoxyribonucleotides.


43. The structure of ribose sugar is

H
2O
CH
O OH
CH H C

HC C H
OH OH

63. Waxes are not examples of phospholipids.

65. Catabolic reactions are usually accompanied by the release of energy.

71. Aerobic reactions involve the oxygen.


72. The pH in the stomach is acidic.
73. The major source of energy for cells is the oxidation of glucose.
74. Cellulose is not digested by human beings.
75. Energy is released during the catabolism of glucose.

92. The vitamin D group consists of vitamin D2 and D3.

96. Adrenaline helps in converting liver glycogen into blood glucose.

MULTIPLE CHOICE QUESTIONS FROM AIEEE AND JEE MAIN

1. The reason for double helical structure of DNA is operation of

[2003]
2. Identify the correct statement regarding enzymes.
Biomolecules and Biological Processes 27.23

[2004]

[2004]
4. The pyrimidine bases not present in DNA are

[2006]
5. The secondary structure of a protein refers to: [2007]
a
a

[2011 (Cancelled)]
7. Synthesis of each molecule of glucose in photosynthesis involves:

[2013]
not
[2014]
[2014, online]

[2014, online]

11. Complete hydrolysis of starch gives: [2015, online]

[2015, online]

[2016, online]

CO2H CO2- CO2- CO2-


+ pK1 + pKR + pK2
H3N H H3N H H3N H H2N H
1.88 3.65 9.60

CH2CO2H CH2CO2H CH 2COO- CH 2COO-

[2016, online]
[2016, online]

16. Thiol group is present in


[2016]

ANSWERS
27.24 Complete Chemistry—JEE Main

HINTS AND SOLUTIONS

temperature. They are homogeneous catalysts and are poisoned.

4. The bases present in DNA are adenine, guanine, cytosine and thymine.

secondary structures include a-helix, triple helix and b-pleated sheet structures.
6. The structures are
HOH2C O OH HOH2C O OH
4 1 4 1
3 2
3 2
HO OH HO H
D-ribose in D-deoxyribose in
RNA DNA
7. Eighteen molecules of ATP are involved in the synthesis of one molecule of glucose.

11. Starch on hydrolysis gives glucose only.

+ +
[H3NCH(CH 2 COOH )CO 2 H] = [H3 N CH(CH 2 COO - )CO 2- ]
1 1
Hence pI = K1 + pK
2 2

16. The amino acid cysteine contains thiol group.


H2N CH COOH

CH2SH
The structures of the remaining three compounds are:
NH2
NH2 N
Cystine HO S COOH Cytosine
C S O N
NH2
O H
Methionine CH3 S CH2CH2 CH COOH
NH2
28
Chemistry in
Everyday Life

Dyes
Dyes are organic compounds with characteristic colours. These are used to impart colour to fabrics, food stuffs and
many other substances.

Acid Dyes

Examples include

Orange I Na+ –O3S N N OH

Orange II Na+ –O3S N N

OH

Basic Dyes
These dyes are salts of a colour base. Examples include

Aniline yellow N N NH2

Butter yellow N N N(CH3)2

Direct Dyes
These dyes can be applied to fabric in an aqueous solution. Examples include
28.2 Complete Chemistry—JEE Main

Congo red NH2 NH2


N N N N

SO3H SO3H
OH
NO2
Martius yellow

NO2
Disperse Dyes

presence of stabilizing agent such as phenol or cresol or benzoic acid.


Examples include
O NH2

Celliton fast pink B

O OH
O NHCH3

Celliton fast blue B

O NHCH3

Vat Dye

Example includes
O H
C N
Indigo dye C C
N C
H O
Mordant Dye
This dye requires a metal ion (known as mordant) for dyeing a fabric. Example includes
O OH
OH
Alizarin

O
Chemistry in Everyday Life 28.3

Fibre Reactive Dye

the derivative of 2, 4-dichloro-1, 3, 5-triazine.


Azo Dye
This dye is produced in situ on the surface of fabric by means of coupling reaction between phenol, naphthol or
resorcinol adsorbed on the surface of fabric and a diazonium salt.

(e.g. orange-I), phthalein dyes (e.g. phenolphthalein), triphenylmethane dye (e.g. malachite green), indigoid dye (e.g.
indigo) and anthraquinone dye (e.g. alizarin).

Chemicals in Medicines
Medicines are chemicals that are used to take care of human sufferings caused by various types of diseases and ill health.

Antacids The acidity in the stomach is treated with a mixture of aluminium and magnesium hydroxides.
The drug cimetidine (Tegamet) was used to treat hyperacidity due to the secretion of pepsin and hydrochloric acid
due to histamine in the stomach. These days, this drug is largely replaced by ranitidine (Zantac).
Antihistamine Histamine has various functions in the body. One of its effect is the contraction of the smooth muscels
in the bronchi and gut and also cause relaxation of other muscels. The nasal congestion in common cold and allergic
response to pollens are also due to histamine.
The antihistamine brompheniramine (Dimetapp) and terfenadine (Seldane) are used to complete for the binding sites
of histamine and thus eliminate its ill affects.

Neurologically Active Drugs


(a) Tranquilizers These are used to relieve anxiety, tension, stress and depression of a person.
Antidepressants Iproniazid, phenelzine (Nardil) and equanil
Stress relieving Chlordiazepoxide and meprobamate
Barbiturates These drugs, derivatives of barbituric acid, are sleep producing agents.
(b) Analgesics These are used to reduce pain or abolish pain in the human body. Two types of analgesic are used.
Narcotic analgesic These are morphine and its homologues (also known as opiates as these are obtained from
opium poppy) and are mainly used to relieve postoperative, cardic, terminal cancer and child-birth pain.
Non-narcotic analgesics Aspirin and paracetamol are the examples of non-narcotic analgesics. Unlike
narcotic analgesic, these are non-addictive drugs. These are used in relieving skeletal pain and reducing fever
(antipyretic). Aspirin is a blood thinner and is used for the prevention of heart attack.
Antimicrobials To treat the effects of bacteria, virus, fungi and other pathogens, antimicrobials are used.
(a) Antibiotics These are mainly used to inhibit the growth or destruction of invading bacteria in human beings.
Two types of antibiotics may be distinguished.
Bactericidal These have killing effect on the microbes. Examples are: penicillin, aminoglycosides and

Bacteriostatic These have inhibitory effect on the microbes. Examples are: erythomycin, tetracycline and
chloramphenicol.
Broad spectrum Antibiotics These can kill or inhibit wide range of Gram-positive and Gram-negative

is rapidly absorbed from the gastrointestinal tract and is thus given orally to treat typhoid, dysentery, acute
fever, urinary infection, meningitis and pneumonia.
Narrow spectrum antibiotics These can effect only Gram-positive or Gram-negative bacteria. Examples:
Pencillin G
28.4 Complete Chemistry—JEE Main

(b) Antiseptics These are applied to living tissues (such as wounds and cuts) to kill or prevent the growth of
micro-organisms. Examples are furacine, soframycine, dettol (a mixture of chloroxylenol and terpineol),
iodoform and tincture of iodine (2-3 per cent of iodine in alcohol-water mixure). A dilute solution of boric acid
is used as weak antiseptic for eyes. Bithional is added to soaps to impart antiseptic properties.
(c) Disinfectants
Examples are one per cent solution of phenol and dilute solution of chlorine (0.2 to 0.4 ppm) and sulphur
dioxide.

Antifertility Drugs
The antifertility drugs are the birth-control pills and contain a mixture of hormones, estrogen and progesterone
derivatives. The latter helps suppressing ovulation.
The most commonly used derivatives of progesterone and estrogen are norethindrone and ethynylestradiol (novestrol),
respectively.
Antipyretics These are used to reduce body temperature due to fever. Examples are aspirin, paracetamol and
phenacetin.
Anaesthetics These drugs has the ability to bring about a reversible loss of consciousness. Examples are diethyl ether,
divinyl ether and chloroform.
Antimalarial Examples are quinine, chloroquine, paraquine and primaquine.

Sweetening Agents
Some of the sweeteness, which can be used in place of sucrose to control intake of calories, are as follows. Their
sweetness value with respect to that of sucrose are enclosed within paraenthesis.
Aspartame (100), Saccharin (550), Sucralose (600) and Alitame (2000). Of these, commonly used sweetener is
aspartame which is a methyl ester of dipeptide formed from aspartic acid and phenylalanine.

Food Preservatives
To prevent spoilage of food due to microbial growth, preservatives are used. The common preservatives are salt, sugar,
vegetable oils, sodium benzoate and salts of sorbic and propanoic acids.

Soaps and Detergents


Soaps and detergents are used to clean dirty clothes. In fact, these remove fats adhered to clothes along with dirt etc.
Soaps These are sodium or potassium salts of long chain fatty acids such as stearic (C17H35COOH), oleic (C17H33COOH)
and palmitic acids (C16H33
fat). This is done by treating fact with NaOH (or KOH) solution.
Chemistry in Everyday Life 28.5

Soap obtained in the above reaction is precipitated out by adding sodium chloride.
Soaps are effective as cleaning agents in aqueous medium and in alkaline medium.
Soap is not effective in hard water which contains salts of calcium and magnesium. This is due to the formation of
insoluble stearate.
2 C17H35COONa + CaCl2 (C17H35COO)2 Ca + 2NaCl
Insoluble calcium stearate
Detergents These have all the properties of soaps but do not contain any soap. These can be used in soft and hard
water.

(i) Anionic Detergents These are sodium salts of sulphonated long chain of hydrocarbon. Examples are
CH3(CH2)10CH2OSO3– Na
Sodium lauryl sulphate

- +
SO3Na

Sodium dodecylbenzene sulphonate


(ii) Cationic Detergent These are quarternary ammonium salts of amines with acetate, chloride or bromide as
anion. Example is
È + ˘ -
ÍCH3 (CH 2 )15 - N (CH3 )3 ˙ Br
Î ˚
Cetyltrimethyl ammonium bromide
(iii) Non-Ionic Detergent These carry no positive or negative charge. Example is the detergent formed by
reacting setaric acid with polyethyleneglycol.
CH3(CH2)16COOH + HO(CH2CH2O)nCH2CH2OH
Stearic acid Polyethylene glycol

O
C O (CH2CH2O)n CH2CH2OH
Polyethylene glycol stearate
Liquid dishwashing detergents are non-ionic type
The cleansing action of all detergents is due to long chain of hydrocarbon. The anionic or cationic or alcoholic portion
is oriented at the surface of water and the hydrocarbon chain is pointed outwardly. This helps dissolving grease or fat
attached to the object being washed
28.6 Complete Chemistry—JEE Main

MULTIPLE CHOICE QUESTIONS

Dyes
1. Which of the following is a natural dye?
(a) Aniline blue (b) Crystal violet (c) Phenolphthalein (d) Alizarin
2. Which of the following dyes has a nitro group?
(a) Indigo (b) Phenolphthalein (c) Martius yellow (d) Melachite
3. Which of the following is a vat dye?
(a) Indigo (b) Malachite green (c) Orange-I (d) Azo dye
4. Which of the following structures represents phenolphthalein?
OH
NO2

(a) Na O3S N N OH (b)

NO2

O
C O OH
O OH
(c) C (d)

O
HO OH
5. Which of the following structures represents alizarin?
O H OH
C N NO2
(a) C C (b)
N C
H O NO2

O OH
OH

(c) Na O3S N N OH (d)

O
Chemistry in Everyday Life 28.7

6. Which of the following represents an example of direct dye?


(a) Alizarin (b) Martius yellow (c) Aniline yellow (d) Orange-I
7. Which of the following represents an example of acid dye?
(a) Alizarin (b) Martius yellow (c) Aniline yellow (d) Orange-I
8. Which of the following represents an example of basic dye?
(a) Alizarin (b) Martius yellow (c) Malachite green (d) Orange-I
9. Which of the following represents an example of mordant dye?
(a) Martius yellow (b) Malachite green (c) Aniline yellow (d) Alizarin
10. Which of the following represents an example of azo dye?
(a) Martius yellow (b) Alizarin (c) Orange-I (d) Phenolphthalein
11. The structure of indigo dye is
OH O H
NO2 C N
(a) (b) C C
N C
NO2 H O
O
O OH C
OH O
(c) (d) C

O
12. The structure of malachite green is
OH O OH
NO2 OH
(a) (b)

NO2 O

+ O H
C N(CH3)2 N
C
(c) (d) C C
N C
H O
N(CH3 )2

(a) metal ion (b) acid (c) alkali (d) wooden vat
Chemicals in Medicines, Propellant, etc.
14. Which of the following is antipyretic?
(a) Aspirin (b) Narcotics (c) Phenol (d) Chloroform
28.8 Complete Chemistry—JEE Main

15. Which of the following may be used as antipyretic as well as analgesic?


(a) Aspirin (b) Alkaloid codein (c) Morphine (d) Chloroform
16. Which of the following is a non-narcotic analgesic?
(a) Morphine (b) Codein (c) Heroin (d) Aspirin
17. Which of the following substances may be used as antiseptic as well as disinfectant?
(a) Formaldehyde (b) Chlorine (c) KMnO4 (d) Phenol
18. Which of the following analgesics is not habit-forming?
(a) Morphine (b) Aspirin (c) Codein (d) Heroin
19. Which of the following antiseptic is a dye?

20. Which of the following statements is not correct?


(a) Antipyretics are substances which are used to reduce the body temperature
(b) Analgesics are substance which are used to relieve pain
(c) Antiseptics and disinfectants can be used for the same purposes
(d) Antiseptics can be safely applied on living beings whereas disinfectants are not safe to apply
21. Which of the following is commonly used as disinfectant?

22. Which of the following substances is added to soap to make it antiseptic?


(a) Iodine (b) KMnO4 (c) Bithional (d) Cl2
23. Detol is a mixture of
(a) chloroxylenol and terpeneol in a suitable solvent (b) formaldehyde and phenol in the solvent water
(c) tincture of iodine and chloroform (d) KMnO4 and iodoform
24. Which of the following is a general anaesthetics?
(a) Procaine (b) Cocaine (c) Nitrous oxide (d) Xylocaine
25. Which of the following is not a local anaesthetic?
(a) Diethyl ether (b) Cocaine (c) Procaine (d) Xylocaine
26. The anaesthetic which is administered by injection is
(a) diethyl ether (b) divinyl ether
(c) liquid nitrous oxide (d) morphine
27. Which of the following diseases is not caused by bacteria?
(a) Pneumonia (b) Dysentery (c) Tuberculosis (d) Diphtheria
28. Which of the following diseases is caused by bacteria?
(a) Tuberculosis (b) Dysentery (c) Malaria (d) Syphilis
29. Which of the following diseases is caused by protozoas?
(a) Tuberculosis (b) Pneumonia (c) Diphtheria (d) Malaria
30. Which of the following diseases is not caused by protozoas?
(a) Malaria (b) Dysentery (c) Smallpox (d) Syphilis
31. Which of the following is viruses born disease?

32. The drug used to cure tuberculosis is


(a) quinine (b) piperazine
(c) sulphanilamide (d) para-aminosalicyclic acid
33. The drug used for the treatment of throat infection is
(a) quinine (b) piperazine
(c) sulpha drug like sulphanilamide (d) isonicotin hydrazide
Chemistry in Everyday Life 28.9

34. Which of the following is not tranquilizer?


(a) Luminal (b) Seconal (c) Reserpine (d) Piperazine
35. Which of the following is not true for reserpine?
(a) Reserpine is an alkaloid
(b) Reserpine is also known as serpasil

(d) Reserpine is not a tranquilizer


36. Penicillin was discovered by
(a) Graham Bell (b) Alexander Fleming (c) Paul Elrich (d) Lister
37. Which of the following is used for lowering blood pressure?
(a) Reserpine (b) Morphine (c) Diethyl ether (d) Cocaine
38. Which of the following is not an antibiotic?
(a) Piperazine (b) Penicillin (c) Streptomycin (d) Chloramphenicol
39. Which of the following antibiotics is used to cure typhoid?
(a) Penicillin (b) Chloramphenicol (c) Tetracycline (d) Streptomycin
40. Which of the following antibiotics is used to cure tuberculosis?
(a) Penicillin (b) Chloramphenicol (c) Tetracycline (d) Streptomycin
41. Which of the following is not regarded as antibiotics in true sense?
(a) Tetracycline (b) Penicillin (c) Sulpha drugs (d) Chloramphenicol
42. The substance used in the birth control pills is
(a) tetracycline (b) sulphadiazine (c) mestranol (d) piperazine
43. The substance used to attract the gypsy moth is
(a) mestranol (b) norethindrone (c) disparlue (d) sulphadiazine
44. The structure of aspirin is

NHCOCH3 OC2H5 CH3


OCOCH3
COOH Cl

(a) (b) (c) (d)


HO CH3
OH NHCOCH3

45. The structure of paracetamol is

OCOCH3 NHCOCH3 OC2 H5 CH3


COOH Cl
(a) (b) (c) (d)
HO CH3
OH NHCOCH3
46. The structure of phenacetin is
OCOCH3 NHCOCH3 OC2H5 CH3

COOH Cl
(a) (b) (c) (d)
HO CH3
OH NHCOCH3
28.10 Complete Chemistry—JEE Main

47. Which of the following structures is not correct?


OCOCH3
COOH H2C CH2
(a) Aspirin (b) Piperazine HN NH
H2C CH2

(c) Isonicotin hydrazide N NHNH2 (d) Paracetamol C2H5O NHCOCH3

48. A hybrid propellant contains


(a) solid fuel and liquid oxidizer (b) liquid fuel and solid oxidizer
(c) solid fuel and solid oxidizer (d) liquid fuel and liquid oxidizer
49. Which one of the following is a composite propellant?
(a) Cellulose nitrate and ammonium perchlorate
(b) Nitroglycerine and cellulose nitrate
(c) Polybutadiene, ammonium perchlorate and aluminium powder
(d) Kerosene and oxygen
50. Which of the following is an example of a double-base propellant?
(a) Polyurethane, ammonium perchlorate and magnesium
(b) Nitroglycerine and nitrocellulose
(c) Liquid N2O4 and acrylic rubber
(d) Nitromethane
51. Which of the following is an example of monopropellant?
(a) Polybutaidene, ammonium perchlorate and aluminium
(b) Alcohol and oxygen
(c) Liquid N2O4 and acrylic rubber
(d) Nitromethane
52. Which of the following is not a food preservative?
(a) Sodium benzoate (b) Sodium sulphite (c) Sorbic acid (d) Salts of propanoic acid
53. Which of the following is used as antioxidant in food preservation?
(a) Sodium benzoate (b) Sodium sulphite (c) Sorbic acid (d) Salts of propanoic acid
54. Boric acid used in talcum powder serves as
(a) perspiration absorber (b) masking enlarge pores
(c) antiseptic (d) cooling agent
55. Dish washing detergents commonly contain
(a) sodium alkyl benzene sulphonate (b) cationic detergent
(c) anionic detergent (d) nonionic detergent
56. Allergy in the body is caused by the production of
(a) vitamin (b) hormone (c) histamine (d) enzymes
57. 2-Acetoxybenzoic acid is
(a) an antiseptic (b) known as aspirin (c) an antibiotic (d) a dye
58. Which is used in Cancer therapy?
(a) Co (b) Ni (c) Pt (d) Pd
59. Penicilline is
(a) a hormone (b) an analgesic (c) an antibiotic (d) an antibody
Chemistry in Everyday Life 28.11

60. Which of the following is an antipyretic?


(a) Luminal (b) Seconal (c) Phenacetin (d) Equanil
61. Which of the following is antihistamine?
(a) Cimetidine (b) Ranitidine (c) Brompheniramine (d) Aspirin
62. Which of the following is used to treat hyperacidity?
(a) Ranitidine (b) Aspirin (c) Terfenadine (d) Phenelzine
63. Which of the following is not a bactericidal?

64. Which of the following is a narrow spectrum antibiotics?

65. Tincture of iodine is a mixture of iodine in alcohol and water. The percentage of the former is about
(a) 2–3 percent (b) 8–10 percent (3) 20–25 percent (d) 50 percent
66. Which of the following sweetening agent has a maximum sweetness relative to sucrose?
(a) Aspartame (b) Saccharin (c) Sucralose (d) Alitame
67. Which of the following sweetening agent has a minimum sweetness relative to sucrose?
(a) Aspartame (b) Saccharin (c) Sucralose (d) Alitame
68. Which of the following is stearic acid?
(a) C17H35COOH (b) C17H33COOH (c) C16H33COOH (d) C16H31COOH
69. Soap is obtained from glyceryl ester of fatty acid by the process of

(c) boiling under reduced pressure (d) heating to its boiling point
70. Liquid dishwashing detergent are
(a) soaps (b) anionic detergent (c) cationic detergent (d) non-ionic detergent
71. Polyethyleneglycol stearate is a/an
(a) anionic detergent (b) cationic detergent (c) non-ionic detergent (d) example of soap

ANSWERS
1. (d) 2. (c) 3. (a) 4. (c) 5. (d) 6. (b)
7. (d) 8. (c) 9. (d) 10. (c) 11. (b) 12. (c)
13. (a) 14. (a) 15. (a) 16. (d) 17. (d) 18. (b)
19. (d) 20. (c) 21. (d) 22. (c) 23. (a) 24. (c)
25. (a) 26. (d) 27. (b) 28. (a) 29. (d) 30. (c)
31. (d) 32. (d) 33. (c) 34. (d) 35. (d) 36. (b)
37. (a) 38. (a) 39. (b) 40. (d) 41. (c) 42. (c)
43. (c) 44. (a) 45. (b) 46. (c) 47. (d) 48. (a)
49. (c) 50. (b) 51. (d) 52. (b) 53. (b) 54. (c)
55. (d) 56. (c) 57. (b) 58. (a) 59. (c) 60. (c)
61. (c) 62. (a) 63. (b) 64. (a) 65. (a) 66. (d)
67. (a) 68. (a) 69. (b) 70. (d) 71. (c)

HINTS AND SOLUTIONS


1. Alizarin is a natural dye. 2. Martius yellow contain a nitro group.
3. Indigo is a vat dye. 4. Structure C is phenolphthalein.
28.12 Complete Chemistry—JEE Main

5. Structure d is alizarin. 6. Martius is a direct dye.


7. Orange-I is an acid dye. 8. Malachite green is a basic dye.
9. Alizarin is a mordant dye. 10. Orange-I is an azo dye.
11. Structure b is indigo dye. 12. Structure c is malachite green.
13. Metal ion is used for binding a mordant dye. 14. Aspirin is antipyretic.
15. Aspirin is antipyretic as well as analgesic. 16. Aspirin is a non-narcotic analgesic.
17. Phenol is antiseptic as well as disinfectant. 18. Aspirin is not habit-forming.
19. 20. Statement c is not correct.
21. Phenol is commonly used as disinfectant. 22. Bithional is added to soap to make it antiseptic.
23. Detol is a mixture of chloroxylenol and terpeneol. 24. Nitrous oxide is a general anaesthetics.
25. Diethyl ether is not a local anaesthetic. 26. Morphine is anaesthetic administered by injection.
27. Dysentery is not caused by bacteria. 28. Tuberculosis is caused by bacteria.
29. Malaria is caused by protozoas. 30. Small pox is not caused by protozoas.
31. 32. para-Aminosalicyclic acid is used to cure
tuberculosis.
33. Sulpha drug is used for the treatment of throat infection. 34. Piperazine is not tranquilizer.
35. Statement d is not correct. 36. Alexander Fleming discovered Penicillin.
37. Reserpine is used for lowering blood pressure. 38. Piperazine is not an antibiotic.
39. Chloroamphenicol is used to cure typhoid. 40. Streptomycin is used to tuberculosis.
41. Sulpha drugs. 42. Mestranol is used in birth control.
43. Disparlue is used to attract the gypsy moth. 44. Structure a represents aspirin.
45. Structure b is paracetamol. 46. Structure c is phenacetin.
47. Structure d is not correct.

MULTIPLE CHOICE QUESTIONS FROM AIEEE AND JEE MAIN

1. Insulin production and its action in human body are responsible for the level of diabetes. This compound belongs
to which of the following categories?
(a) a co-enzyme (b) a hormone (c) an enzyme (d) an antibiotic [2004]
2. Which one of the following types of drugs reduces fever?
(a) Antibiotic (b) Tranquiliser (c) Analgesic (d) Antipyretic [2005]
3. Aspirin is known as
(a) Acetylsalicylic acid (b) Phenylsalicylate (c) Acetylsalicylate (d) Methylsalicylic acid
[2012]
4. What is DDT among the following?
(a) Green house gas (b) A fertilizer
(c) Biodegradable pollutant (d) Non-biodegradable pollutant [2012]
5. Which one of the following is used as Antihistamine ?
(a) Omeprazole (b) Chloranphenicol (c) Diphenhydramine (d) Norethindrone
[2014, online]
6. Aminoglycosides are usually used as:
(a) antibiotic (b) analgesic (c) hypnotic (d) antifertility [2014]
7. OCOCH3

COOH

is used as

(a) Insecticide (b) Antihistamine (c) Analgesic (d) Antacid [2015, online]
Chemistry in Everyday Life 28.13

(a) Aspartame (b) Saccharin (c) Sucralose (d) Alitame [2015, online]
9. Which of the vitamins given below is water soluble?
(a) Vitamin C (b) Vitamin D (c) Vitamin E (d) Vitamin K [2015]
10. Which of the following compounds is not an antacid?
(a) Aluminium hydroxide (b) Cimetidine (c) Phenelzine (d) Ranitidine [2015]
11. Which of the following is bactericidal antibiotic? [2016, online]

12. Which of the following is anionic detergent? [2016]


(a) Sodium lauryl sulphate (b) Cetyltrimethyl ammonium bromide
(c) Glyceryl oleate (d) Sodium stearate
13. The distillation technique most suited for separating glycerol from spent-lyne in the soap industry is [2016]
(a) Fractional distillation (b) Steam distillation
(c) Distillation under reduced pressure (d) Simple distillation

ANSWERS
1. (b) 2. (d) 3. (a) 4. (d) 5. (c) 6. (a)
7. (c) 8. (c) 9. (a) 10. (c) 11. (a) 12. (a)
13. (c)

HINTS AND SOLUTIONS


1. Insulin is a proteinaceous hormone produced by pancreas in our body.
2. Antipyretic is used to reduce fever.
OCOCH3
COOH
3. The structure of aspirin is , which is acetylsalicylic acid.

4. DDT is non-biodegradable pollutant.


5. Dipheny hydramine is an antihistamine.
6. Aminoglycosides are antibiotics.
7. The given compound is aspirin which is used as analgesic.
8. Sucralose contains chlorine. It is trichloro derivative of sucrose.
9. Vitamin C is water soluble while vitamins D, E and K are fat soluble
10. Phenelzine is not an antacid

Bacteriostatic has inhibiting effect on the microbes. Tetracycline, chloroamphenicol and erythromyacin are the
examples of bacteriostatic.
12. Sodium lauryl sulphate is an anionic detergent. This contains sulphate substituent attached to long chain of hydro-
carbon:

CH3(CH2)10CH2 O S O-Na+
O

13. Glycerol is separated from spent-lyne by using the technique of distillation under reduced pressure.
29
Principles Related to
Practical Chemistry

I. Organic Compound
Detection of Elements (N, S and Halogens)

Test for Nitrogen

Na + C
+
N ⎯⎯⎯
Fuse
→ Na+ + CN –
Ê from organicˆ
Á ˜
Ë compound ¯

Fe2+ + 6 CN – Æ [Fe(CN)6]4–
3 [Fe(CN)6]4– + 4Fe3+ Æ Fe4[Fe (CN)6]3
(prussian blue)

Test for Sulphur

Na + S Æ Na2+ + S2–
Êfrom organicˆ
ÁË compound ˜¯
S2– + Pb2+ Æ PbS
(black ppt)
29.2 Complete Chemistry—JEE Main

S2– + [Fe (CN)5NO]2– Æ [Fe(CN)5NOS]4–


(purple colour)

Test for Nitrogen and Sulphur Present Together

+
Na + C N +S Na+ + CNS –
Êfrom organicˆ
Á ˜
Ë compound ¯

Fe3+ + 3 CNS – Fe(CNS)3


red colour

Tests for Halogens

– 2– –

4
4
4

4 2

Detection of Functional Group


Test for Alcoholic Hydroxyl Group
Sodium Test

Ceric Ammonium Nitrate Test

Molybdate Test
2

Test for Phenolic Hydroxyl Group


Ferric Chloride Test

Ceric Ammonium Nitrate

Phthalein Test
2 4
Principles Related to Practical Chemistry 29.3

Tests for Aldehydic and Ketonic Groups


2, 4-Dinitrophenylhydrazine Test

Sodium Bisulphite Test

Tests for Aldehydic Group


Tollens reagent test

Fehling’s Solution Test

Schiff’s Reagent Test

Test for Ketonic Group

Tests for Carboxylic Group


Bicarbonate Test 3
2
Ester Test 2 4

Tests for Primary Amine Group


Isocyanide Test
2

Dye Test for aromatic primary amines


2 b
2

Test for Secondary Amino Group


Liebermann’s Nitroso Reactions
2
2 4

Test for Nitro Group


Dye Test

2 b
2
Mulliken and Baker’s Test
4
2
29.4 Complete Chemistry—JEE Main

II INORGANIC COMPOUND
Action of Heat on a Salt

2HgO Æ 2Hg + O2 2Ag2CO3 Æ 4Ag + 2CO2 + O2


(red) (silvery deposit) 2Pb(NO3)2 Æ 2PbO + 4NO2 + O2
2Pb3O4 Æ 6PbO + O (white) (yellow) (brown)
(red) (yellow) (NH4)2Cr2O7 Æ N2 + Cr2O3 + 4H2O
2PbO2 Æ 2PbO + O2 (orange) (green)
(brown) (black) 2NaHCO3 Æ Na2CO3 + CO2 + H2O
CuCO3 Æ CuO + CO2 NH4HCO3 Æ NH3 + CO2 + H2O
(green) (black) CaCO3 Æ CaO + CO2
ZnCO3 Æ ZnO + CO2 2NaNO3 Æ 2NaNO2 + O2
(white) yellow (hot) MgCO3 Æ MgO + CO2
white (cold)
2Mg(NO3)2 Æ 2MgO + 4NO2 + O2
CuSO4 5H2O Æ CuSO4 + 5H2O 2Ca(NO3)2 Æ 2CaO + 4NO2 + O2
(blue) (white) 2CaSO4 2H2O Æ 2CaSO4 H 2O + 3H2O
CuSO4 Æ CuO + SO3 (plaster of paris)
2FeSO4 Æ Fe2O3 + SO2 + SO3 2AlCl3.6H2O Æ Al2O3 + 6HCl + 3H2O
2Zn(NO3)2 Æ 2ZnO + 4NO2 + O2 2NH3 Æ N2 + 3H2
(white) yellow(hot) (brown) red heat
white(cold) NH4NO2 Æ N2 + 2H2O
2AgNO3 Æ 2Ag + 2NO2 + O2 NH4NO3 Æ N2O + 2H2O
2Cu(NO3)2 Æ 2CuO + 4NO2 + O2 2KClO3 Æ 2KCl + 3O2
(black) (brown) 4K2Cr2O7 Æ 4K2CrO4 + 2Cr2O3 + 3O2
2KMnO4 Æ K2MnO4 + MnO2 + O
2Ag2O Æ 4Ag + O2 (COO)2Fe Æ FeO + CO + CO2
(black)

Characteristic Tests of Anions


Carbonate 2

CO2– +
3 + 2H Æ CO2 + H2O
Ca(OH)2 + CO2 Æ CaCO3 + H2O
(milky)
CaCO3 + H2O + CO2 Æ Ca(HCO3)2
(soluble)
Sulphide 2 4 2

Na2[Fe(CN)5NO] + Na2SÆ Na4[Fe(CN)5NOS]


(purple)
Pb(CH3COO)2 + H2SÆ PbS + 2CH3COOH
(black)

Sulphite 2 4 2

Cr2O 2– + 3+ 2–
7 + 2H + 3SO2 Æ 2Cr + 3SO4 + H2O
green
Nitrite 2 4
4 ◊
Principles Related to Practical Chemistry 29.5

Chlorides 2 4

Cl– + Ag+ Æ AgCl

Æ
AgCl + 2NH 4OH Æ [Ag(NH3)2]Cl + 2H2O

K2Cr2O7 + 4Cl– + 6H2SO4 Æ 2CrO2Cl2 + 2KHSO4 + 4HSO–4 + 3H2O


(red vapours)
CrO2Cl2 + 4NaOH Æ Na2CrO4 + 2NaCl + 2H 2O
(yellow soln)
Na2CrO4 + (CH3COO)2P b Æ PbCrO4 + 2CH3COONa
(yellow ppt)
Bromides 2 4 2

2
– –
2 2
Iodide 2 4 2

2
– –
2 2
Nitrate 2 4 2
Sulphate 4 2

Thiosulphate 2

S2O 2– +
3 + 2Ag Æ Ag2S 2O3
(white)
Ag2S2O3 + H2O Æ Ag2S + H2SO4
(black)
Oxalate 2 4 2

2
– 2+
4 4

Ox2– + Ca2+ Æ CaOx


Æ

(white ppt)
MnO–4 + C2O42– + 8H+Æ Mn2+ + 2CO2 + 4H2O
Fluoride 2 4 2 4
– 2–
2 4 4

2 4 2O
Phosphate 3
4 3PO4 ◊ 3

PO3–
4 3

3 3PO4

3PO4 4 2 4 3 4 3PO4 ◊ 3 2 4NO3


29.6 Complete Chemistry—JEE Main

Borate 2 4 2

2BO33– + 3H2SO4 Æ 3SO42– + 2H3BO3


H3BO3 + 3C2H5OH Æ B(OC2H5)3 + 3H2O
(Ethyl borate)

Analysis of Cations

Solution A
Dilute HCl

Precipites of Filtrate (solution B)


cations of Gp. I
H2S in acidic medium

Precipitates of Filtrate (solution C)


cations of Gp. II
NH4OH in the presence of NH4Cl

Precipitates of Filtrate (solution D)


cations of Gp. III
H2S in NH4OH

Precipitates of Filtrate (solution E)


cations of Gp. IV (NH4) 2CO3 in NH4OH

Precipitates of Filtrate (solution F)


cations of Gp. V rest of cations

Fig. 29.1

Details of Scheme
Cations of Group I 2 2 2 2
2

PbCl2(aq) + K2CrO4 Æ PbCrO4 + 2KCl


(yellow ppt.)

PbCl2(aq) + KI Æ PbI2 + 2KCl


(yellow ppt.)

AgCl + 2NH4OH Æ [Ag(NH3)2]Cl + 2H2O


(soluble)

Hg2Cl2 + 2NH4OH Æ 2[Hg(NH3)]Cl + 2H2O


(black ppt.)
Principles Related to Practical Chemistry 29.7

[Ag(NH3)2]Cl + KI Æ AgI + 2NH3 + KCl


(yellow ppt.)

3 2

2HgCl2 + SnCl2 Æ Hg2Cl2 + SnCl4


(white ppt)
Hg2Cl2 + SnCl2 Æ 2Hg + SnCl 4
(black)

Cations of Group II 2
2 3 2 3 2 3
2
2+ 2+ 3+ 2+ 2+
Cations of Group II A
Cations of Group IIB As3+ 3+ 2+

Fig. 29.2

2 HgCl2 + SnCl2 Æ Hg2Cl2 + SnCl4


(white ppt.)
Hg2Cl2 + SnCl2 Æ 2Hg + SnCl4
(black)

3+

BiCl3 + H2O Æ BiOCl + 2HCl


2BiCl3 + 6NaOH + 3Na2[SnO2] Æ 2Bi + 3Na2[SnO3] + 3H2O + 6NaCl
(black)
2+
2 4
29.8 Complete Chemistry—JEE Main

2+
4
2+ 4–
2Cu + [Fe(CN)6] Æ Cu2[Fe(CN)6]
(chocolate-brown precipitate)
2+
2

4 3 4 4 3 4 4 2 3

2(NH4)3AsS4 + 6HCl Æ As2S5 + 6NH4Cl + 3H2S


2(NH4)3SbS4 + 6HCl Æ Sb2S3 + 6NH4Cl + 3H2S + 2S
(NH4)2SnS3 + 2HCl Æ SnS2 + 2NH4Cl + H2S

2 3 3 2

2 4 2

3
3As2 3 2 3AsO4

H3AsO4 + 12(NH4)2MoO3 + 21HNO3 Æ (NH4)3AsO4 12MoO2 + 21NH4NO3 + 12H2O


(yellow ppt.)
3+
4+
2 2 3
2
4 2 4 2 2O 4 2O

SbCl3 + 3(NH4)2C2O4 Æ (NH4)3[Sb(C2O4)3] + 3NH4Cl


(unstable)
SnCl4 + 4(NH4)2C2O4 Æ (NH4)4[Sn(C2O4)4] + 4NH4Cl
(stable)
2(NH4)3[Sb(C2O4)3] + 6H2S Æ Sb2S3 + 6H2C2O4 + 3(NH4)2S
(orange ppt.)
2+
Cations of Group III 3
3+
4 4
3 3 3
3+ 3+
2O 2 2
2 4 3
4 3
2 and Na2 4 4 3
2 4
3
4

2 4 4
2+ 2+ 2+ 2+
Cations of Group IV
2 4 4
2+ 2+ 2+
and Ni2+
Principles Related to Practical Chemistry 29.9

2+ 2+ 2+ 2+
2
2 2
Mn2+ 2 3 2

4
3 3 2 3 4 3 3 3 2O

3 2 2 3 4 2 3 2

4 when white
2
2+
and Ni2+

3+ – 3+
2 2
+ 3+ 3
2 2

H
O O
CH3 C == N N == C CH3
Ni
CH3 C == N N == C CH3
O O
H
Cations of Group V 4 4 2 3
2+ 2+ 2+

2 4 4
2+
4
2+

2+ +
Cations in the Solution F (see Fig. 1)
2+
4PO4
+

3K+ + Na3[Co(NO2)6] Æ K3[Co(NO2)6] + 3Na+


(yellow ppt.)
+

CH(OH)COOH CH(OH)COOH
K+ + | Æ | + H+
CH(OH)COOH CH(OH)COOK
(white ppt.)
+
Test of 4
+ 3–
4 2 4 3 2
+
4
CH(OH)COOH CH(OH)COOH
NH+4 + | Æ Na+ + |
CH(OH)COONa CH(OH)COO(NH 4 )
(white ppt.)
29.10 Complete Chemistry—JEE Main

3
K2HgI4 Æ 2KI + HgI2
HgI2 + 2NH3 Æ IHgNH2 + NH4I
H2NHgI + H2NHgI + H2OÆ H2NHgOHgI + NH4I
(reddish brown ppt.)

Interfering Anions

3
2

3 3

3 2 4

Characteristic Tests of Gases


Nitrogen 3N 2
Oxygen
Carbon dioxide 2
Sulphur dioxide
Chlorine
Nitric oxide
Hydrogen chloride
Hydrogen sulphide
Ammonia

MULTIPLE CHOICE QUESTIONS

Tests for Extra Elements and Organic Compounds

3 2 2 2 2 2 2

2– 4– 2– 3–
4

2 2

3 2 2 3 3 3
Principles Related to Practical Chemistry 29.11

3 3 3 2

2 3

3 2 3 2

Action of Heat on Salts


2
4NO2 3 2 3
2
4 2 2O 7 3 2 3 4NO3

3 2 3 2 3

4 4 3 2 4 2 2O 7
2 3
4 4 4 2 4

Test of Anions
2 2O 7 2 4
3 2 4

3 2 4 2 2O 3 2 3

2– 2– 3– –
4 3 3 3

2 3 2 3 3

3
4 3 3O 4 4 3 PO4 ◊ 3

4 3 PO4 ◊ 3 3 4 2
29.12 Complete Chemistry—JEE Main

Test of Cations
2
2 2 2 3

3+ 3+ 3+ 2+ 2+ 2+ 2+ 2+

2 3 2 3
3

2 3

2+ 2+ 2+ 3+

2 4 2 4
2 4

+ 2+ 2+ 2+

4
2+ + 2+ 2+
4

+
is
CH(OK)COOK CH(OH)COOH CH(OH)COOH CH(OH)COOK
| | | |
CH(OH)COOK CH(OH)COOK CH(OK)COOH CH(OH)COOK

salt is
4 3AsO4 ◊ 2 4 3AsO4 ◊ 2
4 3AsO4 ◊ 2 4 3AsO4 ◊ 2

3+ 3+ 2+ 3+

2+ 3+ 3+ 2+ 2+ 3+ 2+ 2+

2+ 2+ 2+ 2+

2 4
2+ 2+ 3+ 2+

2+ 2+ 2+ 3+

2+ + 2+ 2+
4
Principles Related to Practical Chemistry 29.13

– 3+

2O 3 3 2

2+ 2+ 2+ 3+

2+ 2+ 3+ 2+

2
2+ 3+ 2+ 3+

3 2

ANSWERS

MULTIPLE CHOICE QUESTIONS FROM AIEEE AND JEE MAIN

3 3 3

3 2 3 2 [2003]

3O 4 4 2 2O 7 2 [2003]

+ 2+ –
3 2 3 2

[2003]
29.14 Complete Chemistry—JEE Main

4 2 2O 3

2 2 2 2 2O 3 2
[2004]

[2004]
4
2 4

+
2+
[2008]

[2010]
3
2 + O2 2O + NO2 + O2 3N + O2 2O + NO + O2
[2011]
2

2 3 4 7 [2013]
2 2 2 4

[2014 online]
2
2+ 2+ 3+ 2+
[2015, online]
Column–I Column–II
Column-I Column-II
3

[2015, online]

2+ 2+ 2+ 2+
[2015, online]

ANSWERS
Principles Related to Practical Chemistry 29.15

HINTS AND SOLUTIONS

4 2 2 2 2 4 2 2O 3 2 2 4O

2

– 2+ +
4
– –
2 2 while MnO–4 2+

n VM ¥ –3
¥ –4

¥ –3
¥ –4

¥ –4

3 nM ¥ –4
¥ –4

Ê M N ˆ Ê mNH3 ˆ Ê 14 g mol - 1 ˆ Ê 85 ¥ 10-4 g ˆ


ÁË M ˜ Á ˜ ¥ ÁË 17 g mol -1 ˜¯ ÁË 29.5 ¥ 10 - 3 g ˜¯
NH3 ¯ Ë mcompound ¯

3 Æ 2O + 4NO2 + O2
Ê 2M H ˆ 2
ÁM ˜ mH 2O = 18 ¥ 0.72 g = 0.08 g
Ë H2O ¯
mH 0.08 g
nH = = = 0.08 mol
M H 1g mol-1

Ê MC ˆ 12
is ÁM ˜ mCO2 = 44 ¥ 3.08 g = 0.84 g
2
Ë CO2 ¯
mC 0.84 g
nC = = = 0.07 mol
M C 12 g mol-1

H 2 SiO3
2 2 2 4 Æ 4 4 2 4 2O Æ 2
silicic acid

2+


3
4
2–

2+
Annexure

EXERCISES ON INCREASING/DECREASING CHARACTERISTICS

Organic Chemistry
1. Increasing reactivity of halogens towards alkanes.
2. Increasing enthalpy of reaction.
F + CH4 Æ HF + CH 3 DH1 Br + CH4 Æ HBr + CH 3 DH3
Cl + CH4 Æ HCl + CH 3 DH2 I + CH4 Æ HI + CH 3 DH4
3. Increasing stability of free radicals: CH3, 1°, 2°, 3°, allyl, vinyl
4. Increasing stability of carbocations: CH+3, 1°, 2°, 3°
5. Increasing enthalpy of reactions:
CH3Br Æ CH+3 + Br– DH1
CH3CH2Br Æ CH3CH+2 + Br– DH2
+
CH 3CHCH 3 Æ CH3CHCH3 + Br DH3

|
Br
CH 3 CH3
|
CH 3 —CH—CH 3 CH3 C CH3 + Br
– DH4
| +
Br
6. Increasing order of reactivity towards SN2 displacement:
1-bromopentane, 2-bromopentane, 2-bromo-2-methylbutane
7. Decreasing order of reactivity towards SN2 displacement:
n-Butyl bromide, isobutyl bromide, sec-butyl bromide, tert-butyl bromide
8. Increasing order of reactivity towards SN1 displacement:
1-bromopentane, 2-bromopentane, 2-bromo-2-methylbutane
9. Increasing reactivity sequence of SN1 displacement of halogen: CH3X, 1° X, 2° X, 3° X,
10. Inceasing reactivity sequence of SN2 displacement of halogen: CH3X, 1° X, 2° X, 3° X.
11. Increasing reactivity of alcohols towards gaseous HBr:
2-butanol, 2-methyl-1-propanol, 2-methyl-2-propanol
12. Increasing order or reactivity towards E2 dehydrohalogenation of the bromides:
ethyl bromide, n-proyl bromide, isobutyl bromide, neopentyl bromide
13. Increasing stability of alkenes: R2C==CR2, R2C==CHR, R2C==CH2, RCH==CH2, CH2==CH2
14. Increasing dehydration of alcohol in the presence of H2SO4:
ethyl alcohol, isopropyl alcohol, tert-butyl alcohol
AN.2 Complete Chemistry—JEE Main

15. Decreasing order of reactivity towards E2 dehydrohalogenation:


2-bromo-2-methylbutane, 1-bromopentane, 2-bromopentane, 3-bromopentane
16. Increasing order of dehydrohalogenation
1-bromo-3-methylbutane, 2-bromo-2-methylbutane, 3-bromo-2-methylbutane
17. Decreasing reactivity towards SN1 substitution: 1-chloropropene 3-chloropropene, n-propylchloride
18. Decreasing acidity: H2O, HC∫∫CH, NH3, RH, ROH
19. Decreasing basicity: R–, HC∫∫C–, NH–2, OH–, OR–
20. Decreasing order of reactivity towards the addition of HCl:
styrene, p-chlorostyrene, p-methylstyrene, p-nitrostyrene
21. Decreasing order of reactivity towards dehydration:
a-phenyl ethyl alcohol, a-(p-nitrophenyl) ethyl alcohol, a-(p-aminophenyl) ethyl alcohol
22. Increasing reactivity towards SN1 solvolysis:
benzyl chloride, p-chlorobenzyl chloride, p-methoxybenzyl chloride, p-methylbenzyl chloride,
p-nitrobenzyl chloride
23. Increasing order of reactivity towards elimination by alcoholic KOH:
1-phenyl-2-bromopropane, 1-phenyl-3-bromopropane
24. Decreasing order of reactivity towards aqueous HBr: Isomeric pentyl alcohols
25. Increasing order of reactivity towards aqueous HBr:
1-phenyl-1-propanol, 3-phenyl-1-propanol, 1-phenyl-2-propanol
26. Decreasing order of reactivity towards aqueous HBr:
benzyl alcohol, p-cyanobenzyl alcohol, p-hydroxybenzyl alcohol
27. Increasing order of reactivity towards aqueous HBr:
benzyl alcohol, diphenylmethanol, methanol and triphenylmethanol
28. Decreasing order of reactivity towards bromination:
anisole, benzene, toluene, chlorobenzene, nitrobenzene, phenol
29. Increasing order of reactivity towards bromination:
hydroquinone, p-methoxyphenol, p-methylphenol, p-chlorophenol, p-nitrophenol, sym-trihydroxybenzene
30. Decreasing order of acidity of carboxylic acids:
butanoic acid, 2-bromobutanoic acid, 3-bromobutanoic acid, 4-bromobutanoic acid
31. Decreasing order of acidity of substituted benzoic acids:
p-chlorobenzoic acid, 2, 4-dichlorobenzoic acid, 2,4,6-trichlorobenzoic acid.
32. Decreasing order of acidity of carboxylic acids:
a-chlorophenylacetic acid, p-chlorophenylacetic acid, phenylacetic acid, a-phenylpropionic acid
33. Decreasing order of acidity of carboxylic acids:
p-nitrobenzoic acid, p-nitrophenylacetic acid, b-(p-nitrophenyl) propionic acid
34. Increasing order of basicity : ammonia, aniline, cyclohexylamine
35. Decreasing order of basicity: ethylamine, 2-aminoethanol, 3-amino-1-propanol
36. Decreasing order of basicity : aniline, p-methoxyaniline, p-nitroaniline
37. Increasing order of acidity: benzene sulphonic acid, benzoic acid, benzyl alcohol, phenol
38. Decreasing order of acidity: m-bromophenol, m-cresol, m-nitrophenol, phenol
39. Decreasing order of acidity of substituted phenol:
p-chlorophenol, 2, 4-dichlorophenol, 2, 4, 6-trichlorophenol
40. Decreasing order of reactivity towards KCN: benzyl chloride, chlorobenzene, ethyl chloride
41. Increasing order of nitration: benzene, chlorobenzene, nitrobenzene, toluene
42. Increasing order of reactivity towards alcoholic silver nitrate:
1-bromo-1-butene, 3-bromo-1-butene, 4-bromo-1-butene
Annexure AN.3

43. Decreasing order of reactivity towards alcoholic silver nitrate:


2-bromo-1-phenylethene, a-phenylethyl bromide, b-phenylethyl bromide
44. Increasing order of reactivity towards aqueous NaOH:
chlorobenzene, m-chloronitrobenzene, o-chloronitrobenzene, 2,4- dinitrochlorobenzene,
2,4,6-trinitrochlorobenzene
45. Increasing reactivity towards HCN: CH3CHO, CH3COCH3, HCHO, C2H5COCH3
46. Increasing basicity: p-toluidine, N, N-dimethyl-p-toluidine, p-nitroaniline, aniline.
47. Increasing ease of hydrolysis: CH3COOC2H5, CH3COCl, (CH3CO)2O, CH3CONH2
48. Increasing order of acid strength: ClCH2COOH, CH3CH2COOH, ClCH2CH2COOH, (CH3)2CHCOOH, CH3COOH
49. Increasing reactivity in nucleophilic substitution reactions: CH3F, CH3I, CH3Br, CH3Cl
50. Increasing order of expected enol content: CH3COCH2CHO, CH3COCH3, CH3CHO, CH3COCH2COCH3
51. Decreasing inductive effect: F, NO2, CH3, Cl, C2H5, OCH3, (CH3)2CH, (CH3)3C, C6H5
52. Increasing reactivity of the carbonyl group: HCHO, CH3COCH3, CH3CHO, CH3COC2H5, C2H5COC2H5
53. Increasing reactivity towards HX: CH2==CH2, (CH3)2C==CH2, CH3CH==CHCH3
54. Increasing reactivity towards HBr:
CH2==CHC2H5, CH3CH==CHCH3, CH2==CHCH==CH2, CH3CH==CHCH==CH2,
CH 2 ==C—C==CH 2
| |
CH 3 CH 3
55. Increasing order of m-orientation: C6H5CH3, C6H5CH2Cl, C6H5CHCl2, C6H5CCl3
56. Increasing order of acidity: CH3OH, n-butanol, sec-butanol, tert-butanol
57. Increasing order of acidity:
CH3
CH3CH2COOH; H ⎯ COOH, CH3COOH, HCOOH, C6H5COOH, ClCH2COOH
CH
CH3
58. Decreasing order of acidity: CH3COOH, ClCH2COOH, Cl2CHCOOH, Cl3CCOOH
59. Decreasing order of acidity: FCH2COOH, ClCH2COOH, BrCH2COOH, ICH2COOH
60. Increasing order of acidity: CH3COOH, ClCH2COOH, ClCH2CH2COOH
61. Increasing order of acidity: benzoic acid, p-bromobenzoic acid, p-chlorobenzoic acid, p-nitrobenzoic acid,
p-toluic acid, p-hydroxybenzoic acid
62. Increasing order of acidity: HCl, HCOOH, C6H5COOH, C2H5OH, H2O
63. Increasing order of acidity: H2CO3, phenol, p-nitrophenol, benzoic acid.
64. Increasing order of basicity: NH3, CH3NH2, (CH3)2NH, C6H5NH2
65. Increasing order of basicity: CH3NH2, (CH3)2NH, (CH3)3N
66. Increasing reactivity towards Lucas reagent: 1-butanol, 2-butanol, 2-methyl-2-propanol
67. Increasing reactivity towards sodium: 1-butanol, 2-butanol, 2-methyl-2-propanol
68. Increasing boiling point: n-pentane, isopentane, neopentane
69. Increasing solubility in water: n-butanol, isobutyl alcohol, sec-butyl alcohol, tert-butyl alcohol
3COOH, CH3CH2COOH, (CH3)2CHCOOH, (CH3)3CCOOH

COOH COOH COOH COOH COOH


H3C CH3 CH3 CH3
, , , ,

CH3 CH3 CH3


AN.4 Complete Chemistry—JEE Main

72. Increasing alkaline hydrolysis: HCOOCH3, CH3COOCH3, (CH3)2CHCOOCH3, (CH3)3CCOOCH3


73. Increasing alkaline hydrolysis: CH3COOCH3, CH3COOC2H5, CH3COOCH(CH3)2, CH3COOC(CH3)3
74. Increasing alkaline hydrolysis:

COOCH3 COOCH3 COOCH3 COOCH3

, , ,

Cl NO2 OCH3
75. Increasing acidity: oxalic acid, malonic acid, succinic acid, adipic acid

ANSWERS
1. Br2 < Cl2 < F2
2. DH1 < DH2 < DH3 < DH4
3. Vinyl < CH3 < 1° < 2° < 3° < allyl
4. H3C +< 1°+ < 2°+ < 3°+. The dispersal of charge stabilizes the carbocation. The greater the number of alkyl groups,
the more stable the carbocation.
5. DH4 < DH3 < DH2 < DH1. The more stable the carbocation being formed, lesser the enthalpy of reaction.
CH 3
|
6. CH 3 CH 2 —C—CH 3 < CH 3CH 2 CH 2 —CH— CH 3 < CH 3 CH 2 CH 2 CH 2 CH 2
| | |
Br Br Br
The usual SN2 order is 3° < 2° < 1°.
CH 3 CH 3 CH 3
| | |
7. CH3CH2CH2CH2Br > CH 3CHCH 2 CH 2 Br > CH 3CH 2 CHCH 2 Br > CH 3 —C—CH 2 Br
n-butyl isobutyl sec-butyl |
CH 3
tert-butyl
SN2 is slow for bulky G in G–CH2X as it hinder nuclophilic attack.
CH 3
|
8. CH 3 CH 2 CH 2 CH 2 CH 2 < CH 3 CH 2 CH 2 CHCH 3 < CH 3CH 2 CHCH 3
| | |
Br Br Br
In SN
the faster the reaction. The stability of carbocations is 1° < 2° < 3°.
9. CH3X < 1° < 2° < 3° Stability of carbocation governs the reactivity.
10. 3° < 2° < 1° < CH3X. Steric factor governs the reactivity.
CH 3 CH 3
| |
11. CH 3 CHCH 2 < CH 3 CH 2 CHCH 3 < CH 3 —C—CH 3
| | |
OH OH OH
The order of reactivity is 3° > 2° > 1° > CH3OH. Electronic factors are important except for 1° alcohols for which
steric factors are controlling. Electronic factors are the stability of carbocations.
Annexure AN.5

CH 3
|
12. CH3CH2Br < CH3CH2CH2Br < CH 3CHCH 2 Br
Reactivity in E2 dehydrohalogenation depends mainly on the stability of the alkenes being formed which is
governed by Saytzeff’s rule.
Neopentyl bromide does not undergo dehydrohalogenation as carbon bearing bromine does not contain hydrogen.
13. CH2==CH2 < RCH==CH2 < R2C==CH2 < R2C==CHR < R2C==CR2
The alkene with larger number of alkyl group (i.e. the more substituted alkene) attached to carbon-carbon double
bond is more stable.
CH 3
|
14. C2H5OH < CH3 —CH—OH < CH 3 —C—OH
| |
CH 3 CH 3
The dehydration proceeds via the formation of carbocation. Hence, the greater the stability of carbocation being
formed from alcohol, more easily the latter dehydrates.
CH 3
|
15. CH 3CH 2 CCH 3 > CH 3CH 2 CHCH 2 CH 3 > CH 3 CH 2 CH 2 CHCH 3 > CH3CH2CH2CH2CH2Br
| | |
Br Br Br
The stability of alkenes being formed decides the rate of dehydrohalogenation.
CH 3 CH 3 CH 3
| | |
16. CH 3CCH 2 CH 3 > CH 3CHCHCH 3 > CH CHCH CH
3 2 2
| | |
Br Br Br
The E2 dehydrohalogenation of isomers follows the reactivity order 3° > 2° > 1°.
17. ClCH2CH==CH2 > CH3CH2CH2Cl > CH3CH==CHCl
The reactivity is SN1 substitution depends on the stability of carbocation being formed. In the present case, we
will have allylic cation > 1° cation > vinylic cation
18. H2O > ROH > HC∫∫CH > NH3 > RH
19. R– > NH–2 > HC– ∫∫ C > OR– > OH–
Strong acid produces weak conjugate base and vice versa. The relative order of acids is H2O > ROH > HC∫∫CH
> NH3 > RH. Hence, the conjugate bases follow the reverse order.

20.

The reactivity of addition across a carbon-carbon double bond mainly depends upon the stability of carbocation.
Electron-releasing group causes more electron density on carbon-carbon double bond thereby increases the
reactivity. Electron-withdrawing group withdraws electron density causing decrease in the reactivity.

21. H2N CH ⎯ CH3 > CH ⎯ CH3 > O2N CH ⎯ CH3


OH OH OH
Same principle as give in Q. 20.
22. O2N CH2Cl < Cl CH2Cl < CH2Cl

Solvolysis means replacement of Cl by solvent anion. In SN1 mechanism, a carbocation is formed as the
intermediate. The stability of carbocation decides the reactivity. Electron-releasing group increases stability (and
hence reactivity) while electron-withdrawing group decreases the stability (and hence reactivity).
AN.6 Complete Chemistry—JEE Main

23. CH2CH2CH2Br < CH2CH(Br)CH3

The more stable conjugated alkene is formed faster than the alkene containing isolated double bond.
CH 3 CH 3
| |
24. CH 3 CH 2 —C—CH 3 > CH 3CH —CH—CH 3 , CH 3 CH 2 CHCH 2 CH 3 , CH CH CH CHCH
3 2 2 3
| | | |
OH OH OH OH
CH 3 CH 3 CH 3
| | |
> CH 3CH 2 CH 2 CH 2 — CH 2 > CH 3 CHCH 2 CH 2 > CH 3 CH 2 CH—CH 2 > CH 3 —C—CH 2 OH
| | | |
OH OH OH CH 3

In general, the reactivity of an alcohol toward aqueous HBr follows the order.
allylic, benzylic > 3° > 2° > 1° < CH3OH.
For primary alcohols, steric factors dominate the substitution reaction. The substituted primary alcohol has lesser
reactivity. For alcohols, other than primary, electronic factors are dominating.

25. C 6 H 5 —CH 2 CH 2 CH 2 < C 6 H 5 —CH 2 —CH—CH 3 < C 6 H 5 —CH—CH 2 CH 3


| | |
OH OH OH
The reactivity follows the order 1° < 2° < benzylic alcohol

26. HO CH2OH > CH2OH > N ≡≡ C CH2OH

Electron-releasing group increases reactivity while electron-withdrawing group decreases reactivity


C6 H 5 C6 H 5
| |
27. CH3OH < C6H5CH2OH < C 6 H 5 —CH—OH < C 6 H 5 —C—OH
|
C6 H 5

The reactivity follows the order. CH3OH < 1° benzylic < 2° benzylic < 3° benzylic alcohol.
28. OH OCH3 CH3 Cl NO2

> > > > >

The decreasing order of activating effect of substituent is


—OH > —OCH3 > —CH3 > —H > —Cl > —NO 2 .
deactivator
HO
29. O2N OH < Cl OH < H3C OH < H3CO OH < HO OH < OH
HO

Ring activator increases the rate of bromination while ring deactivator decreases the rate of bromination.
30. CH 3CH 2 CHCOOH > CH3CHCH2COOH > CH 3CH 2 CH 2 COOH > CH3CH2CH2COOH
| |
Br Br
Br
Electron-withdrawing substituent increases acidity. The closer the substituent to —COOH, the greater its effect.
Annexure AN.7

COOH COOH COOH


31. Cl Cl Cl
> >

Cl Cl Cl
Chlorine is a deactivator of benzene ring. More chlorine atoms attached to ring, the greater the deactivating effect.
The ring deactivator increases the acidity of benzoic acid.

32. CHCOOH
H > Cl CH2COOH > CH2COOH > CH3 ⎯ CH ⎯ COOH
Cl C6H5

Cl, and Cl are the electron-withdrawing groups. They increase the acidity of carboxylic

acid. a-chlorophenylacetic acid contains two withdrawing groups. The chlorophenyl group is more electron-
withdrawing than the phenyl group. The weakest acid is phenylacetic acid as it is a little farther away from the
—COOH group.

33. O2N COOH > O2N CH2COOH > O2N CH2CH2COOH

Nitrophenyl is electron-withdrawing group and it increases the acidity of carboxylic acid. The more near the
deactivator to —COOH, the greater the effect.
34. C6H5NH2 < NH3 < cyclohexylamine
Aliphatic amines are more basic than NH3 and aromatic amines.
35. CH3CH2NH2 > HOCH2CH2CH2NH2 > HOCH2CH2NH2
Electron-withdrawing group lowers the basicity. The more near the substituent, the greater its effect.
36. p-CH3OC6H4NH2 > C6H5NH2 > p-O2NC6H4NH2.
Electron-releasing group increases the basicity while electron-attracting group decreases the basicity.

37. CH2OH < OH < COOH < SO3H

OH OH OH OH

38. > > >


NO2 Br CH3
Electron-releasing group decreases acidity of phenol while electron-withdrawing group increases the acidity.
39. 2, 4, 6-trichloro- > 2, 4-dichloro- > p-chlorophenol
Chlorine is ring-deactivator which causes increase in acidity. The larger the number of chlorine in the ring, the
greater its effect.
40. benzyl chloride > ethyl chloride > chlorobenzene
The order of reactivity is benzylic > 1° alkyl > aryl
41. PhNO2 < PhCl < PhH < PhCH3
Ring activator increases the rate while ring deactivator decreases the rate.
42. CH3CH2CH CH < CH 2 CH 2 CH CH 2 < CH 3CHCH CH 2
| |
Br Br Br
The order of reactivity toward generating double bond is allylic > 1° alkyl > vinylic.
AN.8 Complete Chemistry—JEE Main

43. PhCHCH 3 > PhCH 2 CH 2 > PhCH==CH


| | |
Br Br Br
The order of reactivity toward generating double bond is benzylic > 1° alkyl > vinylic
44. Chlorobenzene < m-nitrochlorobenzene < o-chloronitrobenzene < 2, 4- dinitrochlorobenzene < 2, 4,
6-trinitrochlorobenzene.
45. C2H5COCH3 < CH3COCH3 < CH3CHO < HCHO
The nucleophilic addition of HCN across carbonyl group is mainly governed by steric factors. Aldehydes generally
undergo nucleophilic addition more readily than ketones because alkyl or aryl group causes more crowding in the
transition state. Also, an alkyl group releases electrons causing destabilization of the transition state.
46. p-nitroaniline < aniline < p-toluidine < N, N-dimethyl-p-toluidine.
Electron-releasing group enhances the availability of a pair of electrons on nitrogen while electron-withdrawing
group lowers the availability.
47. CH3CONH2 < CH3COOC2H5 < (CH3CO)2O < CH3COCl
48. (CH3)2CHCOOH < CH3CH2COOH < ClCH2CH2COOH < CH3COOH < ClCH2COOH
49. CH3F < CH3Cl < CH3Br < CH3I
50. CH3CHO < CH3COCH3 < CH3COCH2CHO < CH3COCH2COCH3
51. NO2 > F > Cl > OCH3 > C6H5 > CH3 > C2H5 > (CH3)2CH > (CH3)3C.
52. C2H5COC2H5 < C2H5COCH3 < CH3COCH3 < CH3CHO < HCHO
The smaller the alkyl group, the more reactive the carboynl group.
53. CH2==CH2 < CH3CH==CHCH3 < (CH3)2C==CH2
The more stable the intermediate carbocation, the greater the reactivity.
54. CH2==CHC2H5 < CH3CH==CHCH3 < CH2==CH—CH==CH2 < CH3—CH==CH—CH==CH2
< CH 2 C—C CH 2
| |
CH 3 CH 3
Conjugated dienes form the more stable allyl carbocations and are thus more reactive than alkenes. Alkyl groups
on the unsaturated carbon atoms increase reactivity.
55. C6H5CH3 < C6H5CH2Cl < C6H5CHCl2 < C6H5CCl3
56. tert-butanol < sec-butanol < n-butanol < CH3OH
Alkyl group makes an alcohol less acidic. Bigger the alkyl group, the less acidic the alcohol. Methanol is the
strongest and teritary alcohols are the weakest.
57. (CH3)2CHCOOH < C2H5COOH < CH3COOH < C6H5COOH < HCOOH < ClCH2COOH
58. Cl3CCOOH > Cl2CHCOOH > ClCH2COOH > CH3COOH
The more the number of electron-withdrawing group, the stronger the acid.
59. FCH2COOH > ClCH2COOH > BrCH2COOH > ICH2COOH
The more the electronegativity of halogen, the stronger the acid.
60. CH3COOH < ClCH2CH2COOH < ClCH2COOH

61. HO COOH < H3C COOH < COOH < Br COOH

< Cl COOH < O2N COOH

62. H2O < C2H5OH < C6H5COOH < HCOOH < HCl
63. phenol < p-nitrophenol < H2CO3 < C6H5COOH
64. C6H5NH2 < NH3 < CH3NH2 < (CH3)2NH
Annexure AN.9

65. (CH3)3N < CH3NH2 < (CH3)2NH


(CH3)3N is the weakest because of steric factors.
66. 1-butanol < 2-butanol < 2-methyl-2-propanol

not.
67. 2-methyl-2-propanol < 2-butanol < 1-butanol. Reactivity of ROH is CH3OH > 1° > 2° > 3°.
68. neopentane < isopentane < n-pentane
A branched chain isomer has a lower boiling point than a straight chain isomer. The more numerous the branches,
the lower the boiling point. The branching lowers the boiling point for all families of organic compounds.
Branching causes the shape of molecule to approach towards a sphere and thus surface area decreases, with the
result that the intermolecular forces become weaker and are overcome at a lower temperature.
69. n-butyl alcohol < isobutyl alcohol < sec-butyl alcohol < tert-butyl alcohol. Polarity of O—H bond increases in
the same order.
70. (CH3)3CCOOH < (CH3)2CHCOOH < CH3CH2COOH < CH3COOH < HCOOH

71. COOH COOH COOH COOH COOH


H3C CH3 CH3 CH3
< < < <

CH3 CH3 CH3


72. (CH3)3CCOOCH3 < (CH3)2CHCOOCH3 < CH3COOCH3 < HCOOCH3
73. CH3COOC(CH3)3 < CH3COOCH(CH3)2 < CH3COOC2H5 < CH3COOCH3
74. COOCH3 COOCH3 COOCH3 COOCH3

< < <

OCH3 Cl NO2
75. adipic acid < succinic acid < malonic acid < oxalic acid
Acidity decreases with increase in the intervening CH2 groups.

Inorganic Chemistry
1. Decreasing ionic size Mg2+, O 2– Na+, F –
2. Increasing acidic property ZnO, Na2O2, P2O5, MgO

4. Increasing bond length F2, N2, Cl2, O2


5. Increasing size Cl –, S2 –, Ca2+, Ar
6. Increasing acid strength HClO3, HClO4, HClO2, HClO
7. Increasing bond strength HCl, HBr, HF, HI
8. Increasing oxidation number of iodine I2, HI, HIO4, ICl
9. Increasing thermal stability HOCl, HOClO2, HOClO3, HOClO
10. Increasing bond enthalpy N2, O2, F2, Cl2
11. Increasing acidic character CO2, N2O5, SiO2, SO3
12. Increasing ionic size N3 –, Na+, F–, O2 –, Mg2+
13. Increasing basic character MgO, SrO, K2O, NiO, Cs2O
AN.10 Complete Chemistry—JEE Main

14. Increasing extent of hydrolysis CCl4, MgCl2, AlCl3, PCl5, SiCl4


15. Increasing strength of hydrogen bonding O, S, F, Cl, N
(X H - X)
16. Increasing ionic radii in water Li+, Na+, K+, Rb+, Cs+
17. Increasing molar conductivity in water Li+, Na+, K+, Rb+, Cs+
18. Increasing reactivity with water Li, Na, K, Rb, Cs
19. Increasing reactivity with hydrogen Li, Na, K, Rb, Cs
20. Increasing melting point Li, Na, K, Rb, Cs
21. Increasing basic nature of hydroxides LiOH, NaOH, KOH, RbOH, CsOH
22. Increasing thermal stability of hydroxides LiOH, NaOH, KOH, RbOH, CsOH
23. Inreasing covalent character LiCl, LiBr, LiI
24. Increasing ionic character CaCl2, BeCl2, MgCl2, BaCl2, SrCl2
25. Increasing solubility BeCO3, MgCO3, CaCO3, BaCO3
26. Increasing solubility BeF2, MgF2, CaF2, BaF2
27. Increasing solubility Be(OH)2, Mg(OH)2, Ca(OH)2, Ba(OH)2
28. Increasing basicity Be(OH)2, Mg(OH)2, Ca(OH)2, Ba(OH)2
29. Increasing hydration of ions Be2+, Mg2+, Ca2+, Sr2+, Ba2+
30. Increasing reactivity with water Be, Mg, Ca, Sr, Ba
31. Increasing reactivity towards air Be, Mg, Ca, Sr, Ba
32. Increasing solubility BeSO4, MgSO4, CaSO4, SrSO4, BaSO4
33. Increasing ionic character BCl3, AlCl3, GaCl3
34. Increasing strength of Lewis acid BF3, BCl3, BBr3
35. Increasing strength of Lewis acid AlCl3, GaCl3, InCl3
36. Increasing reducing power GeCl2, SnCl2, PbCl2
37. Increasing oxidizing power GeCl4, SnCl4, PbCl4
38. Increasing number of hybrid orbitals of C, Si, Sn
39. Increasing basic character NH3, AsH3, SbH3, PH3
40. Increasing thermal stability NH3, AsH3, SbH3, PH3
41. Increasing acidic strength HNO3, H3PO4, H3AsO4, H3SbO4
42. Increasing solubility in water HNO3, H3PO4, H3AsO4, H3SbO4
43. Increasing order of +5 oxidation state N, P, As, Sb and Bi
44. Increasing extent of hydrolysis NCl3, PCl3, AsCl3, SbCl3, BiCl3
45. Increasing stability of hydrides H2O, H2S, H2Se, H2Te
46. Increasing poisonous nature H2S, H2Se, H2Te, H2Po
47. Increasing acidic strength H2O, H2S, H2Se, H2Te
48. Increasing strength of oxoacids H2SO3, H2SeO3, H2TeO3
49. Increasing stability of oxoacids H2SO3, H2SeO3, H2TeO3
50. Increasing strength of oxoacids H2SO4, H2SeO4, H2TeO4
51. Increasing stability of oxoacids H2SO4, H2SeO4, H2TeO4

53. Increasing reducing power HF, HCl, HBr, HI


2, Cl2, Br2, I2
55. Increasing acidity HF, HCl, HBr, HI
56. Increasing melting point HF, HCl, HBr, HI
57. Increasing boiling point HF, HCl, HBr, HI
Annexure AN.11

58. Increasing stability HFO3, HClO3, HBrO3, HIO3


59. Increasing covalent character TiCl2, TiCl3, TiCl4
60. Increasing magnetic moment Ti3+, Ni2+, Cr2+, Co2+, Zn2+
61. Increasing ionic character VCl2, VCl3, VCl4
62. Increasing electropositivity Hg, Na, Cu, Li
63. Increasing density Fe, Pb, Al, Au
64. Increasing basic characteristics Li2O, BeO, B2O3, CO2
65. Increasing electronegativity As, P, S, Cl
66. Increasing ionization energy N, O, F
67. Increasing atomic size S, O, Se, C
68. Increasing acidity HOCl, HOBr, HOI
69. Increasing density H2S, O2, CO2, NH3, H2
70. Increasing thermal stability HF, HCl, HBr, HI
71. Increasing bond enthalpy N2, O2, F2, Cl2
72. Increasing melting point CaF2, CaCl2, CaBr2, CaI2
73. Increasing oxidizing power O, S, Se, Te
74. Increasing oxidizing power F, Cl, Br, I
75. Increasing size B, Be, Li, Na
76. Increasing single bond strength N—N, O— O, F—F
77. Increasing stability of hydrides LiH, NaH, KH, CsH
78. Decreasing pH of aqueous solution of LiCl, BeCl2, MgCl2, AlCl3
79. Increasing acidic oxide Al2O3, MgO, SiO2, P4O10
80. Increasing basicity F–, Cl–, Br–, I –
81. Increasing basic strength F–, OH –, NH 2–, CH –3
82. Increasing boiling point NH3, PH3, AsH3, SbH3
83. Increasing ionization energy B, C, N, O
84. Increasing thermal stability BeCO3, MgCO3, CaCO3, BgCO3
85. Increasing paramagnetism Ca, Al, N, O
86. Increasing ionic character LiBr, NaBr, KBr, RbBr, CsBr
87. Increasing hydration energy Be2+, Mg2+, Ca2+, Ba2+, Sr2+
88. Increasing bond angle NH3, PH3, AsH3
89. Increasing bond angle NF3, PF3, AsF3
90. Increasing bond angle H2O, H2S, H2Se
91. Increasing bond angle NF3, NCl3
92. Increasing bond angle NO2+, NO2, NO2–
93. Increasing bond angle NH3, NF3
94. Increasing bond angle PH3, PF3
95. Increasing electronegativity O+, O, O–

ANSWERS
1. O–2 > F – > Na+ > Mg2 +
All the four species are isoelectronic (1s2 2s2 2p6 ). The number of positive charges in the nucleus decreases in the
order 12Mg > 11Na > 9F > 8O. Hence, O2 – involved minimum nucleus-electrons attraction and maximum electron-
electron repulsion while Mg2 + involves maximum nucleus-electrons attraction and minimum electron-electron
repulsion. These factors make the size of anion greater than the corresponding neutral atom and that of cation
lesser than the corresponding atom.
AN.12 Complete Chemistry—JEE Main

2. Na2O2 < MgO < ZnO < P2O5


Oxides of electropositive elements are alkaline while those of electronegative elements are acidic. Alkaline
property will increase with increase in electropositive character of metal and acidic characteristics increase with
increase in electronegative characteristics of nonmetals. Since the electronegativity increases in the order Na <
Mg < Zn < P, the acidic character of oxide will also increase in the same order.
3. Na < Al < Mg < Si

11Na (1s)2(2s)2(2p)6(3s)1 ; 13Al (1s)2(2s)2(2p)6(3s)2(3p)1


12Mg (1s)2(2s)2(2p)6(3s)2 ; 14Si (1s)2(2s)2(2p)6(3s)2(3p)2
Aluminium will have lower ionization potential than magnesium as the removal of one electron leads to the

4. N2 < O2 < F2 < Cl2

C
5. Ca2+ < Cl– < S2– < Ar
The given species are isoelectronic. The size of cation will be the smallest. The mononegative anion will have
smaller size than the dinegative anion. The size of the noble gas Ar will be maximum.
6. HClO < HClO2 < HClO3 < HClO4
These acids are better represented as Cl—OH, OCl — OH, O2Cl—OH, O3Cl—OH.
The larger the number of oxygen atoms attached to chlorine, greater the electron pull towards oxygen, hence, more
easy to remove hydrogen from the acid.
7. HI < HBr < HCl < HF
As the size of the halogen atom increases, the strength of HX bond decreases. Besides this, decreasing per cent
ionic character from HF to HI makes the bond less stable.
8. HI < I2 < ICl < HIO4
The oxidation states of iodine in HI, I2, ICl and HIO4 are – 1, 0 +1 and +7, respectively.
9. HOCl < HOClO < HOClO2 < HOClO3
The stability is explained by the increasing number of electrons involved in the formation of s and p bonds in
going from HOCl to HOClO3. In ClO–4 ion all the valence orbitals and electrons of chlorine are involved in the
formation of bonds.
10. F2 < Cl2 < O2 < N2
N2 involves a triple bond, O2 involves a double bond, F2 and Cl2 involve a single bond each. F2 has a lower bond
enthalpy than Cl2. This is due to more repulsion of nonbonding electrons in F2. Besides this, there is a possibility
of multiple bonding in Cl2 involving d orbitals.
11. SiO2 < CO2 < N2O5 < SO3
Increasing electronegativity of an element makes its oxide more acidic.
12. Mg2+ < Na+ < F– < O2– < N3– See Q. 1.
13. NiO < MgO < SrO < K2O < Cs2O
Increasing electropositive nature of the element makes its oxide more basic.
14. CCl4 < MgCl2 < AlCl3 < SiCl4 < PCl5
In covalent halides, hydrolysis occurs as a result of coordination of a water molecule to the less electronegative
element. CCl4 does not undergo hydrolysis as carbon cannot expand its octet to accommodate water molecules.
15. S < Cl < N < O < F
The negative charge on X in HX increases with increasing electronegativity of X. This makes the hydrogen
bonding more strong.
16. Cs+ < Rb+ < K+ < Na+ < Li+
The ions in solution are present as hydrated ions. The smaller the size of the ion, the greater the extent of hydration.
So the size of hydrated ions becomes larger for smaller sized ion and vice versa.
Annexure AN.13

17. Li+ < Na+ < K+ < Rb+ < Cs+


Li+ ion being heavily hydrated has the lowest mobility and Cs+ ion being less hydrated has the highest mobility.
18. Li < Na < K < Rb < Cs
The reactivity increases on descending the Group 1.
19. Cs < Rb < K < Na < Li.
The ease of formation of hydrides decreases on descending the Group 1.
20. Cs < Rb < K < Na < Li
The melting (or boiling) point decreases on descending the group.
21. LiOH < NaOH < KOH < RbOH < CsOH
The basic nature of hydroxides of elements of Group 1 increases on descending the group.
22. LiOH < NaOH < KOH < RbOH < CsOH
Thermal stability of hydroxides increases on descending the group.
23. LiCl < LiBr < LiI
The smaller sized Li+ ions polarizes the larger anion more predominantly giving larger covalent character.
24. BeCl2 < MgCl2 < CaCl2 < BaCl2 < SrCl2
25. BaCO3 < CaCO3 < MgCO3 < BeCO3
On moving down the group, the lattice energies of carbonates do not decrease much while the degree of hydration

26. BeF2 < MgF2 < CaF2 < BaF2


Lattice energy variation is more dominating than the variation in hydration energy.
27. Be(OH)2 < Mg(OH)2 < Ca(OH)2 < Ba(OH)2 Same as Q. 26.
28. Be(OH)2 < Mg(OH)2 < Ca(OH)2 < Ba(OH)2
29. Ba2+ < Sr2+ < Ca2+ < Mg2+ < Be2+
The extent of hydration of ion decreases with increase in ionic size.
30. Be < Mg < Ca < Sr < Ba
The reaction of alkaline - earth metals becomes increasingly vigorous with increasing atomic number.
31. Be < Mg < Ca < Sr < Ba
32. BaSO4 < SrSO4 < CaSO4 < MgSO4 < BeSO4
Hydration of ion plays a dominating role as compared to lattice energy.
33. BCl3 < AlCl3 < GaCl3
Increase in the electropositivity of element increases its ionic character.
34. BF3 < BCl3 < BBr3
Besides s bond between boron and halogen atoms, there exist additional pp-pp bond between the two atoms

The tendency to form pp-pp bond is maximum in BF3 (2pp—2pp back bonding) and falls rapidly on passing
to BCl3 (2pp—3pp back bonding) and BBr3 (2pp —4pp back bonding). The tendency to accept electron pair,
therefore, increases from BF3 to BBr3.
35. InCl3 < GaCl3 < AlCl3
With increase in size of elements of Group 13, the tendency to accept electron pair is decreased.
36. PbCl2 < SnCl2 < GeCl2.
The stability of element in +II oxidation state increases on ascending the Group 14. This is due to inert-pair effect.
37. GeCl4 < SnCl4 < PbCl4
The stability of element in +IV oxidation state decreases on ascending the Group 14. This is due to inert pair effect.
38. Sn < Si < C
The number of hybrid orbitals and ease with which these are formed decreases from carbon to lead.
39. SbH3 < AsH3 < PH3 < NH3
The decrease in electronegativity and increase in size of element cause the decrease in tendency to accept proton.
40. SbH3 < AsH3 < PH3 < NH3
41. H3SbO4 < H3AsO4 < H3AsO3 < HNO3
AN.14 Complete Chemistry—JEE Main

42. H3SbO4 < H3AsO4 < H3AsO3 < HNO3


43. Bi < Sb < As < P < N
44. NCl3 < PCl3 < AsCl3 < SbCl3 < BiCl3
45. H2Te < H2Se < H2S < H2O
46. H2S < H2Se < H2Te < H2Po
47. H2O < H2S < H2Se < H2Te
Larger the size of X (= O, S, Se, Te) weaker its bonds with hydrogen and more easily H+ gets lost in aqueous
solution.
48. H2TeO3 < H2SeO3 < H2SO3
Decreasing size and increasing electronegativity from Te to S withdraws electrons from O—H bond towards itself,
thus, facilitating the release of proton.
49. H2TeO3 < H2SeO3 < H2SO3
50. H2TeO4 < H2SeO4 < H2SO4
51. H2TeO4 < H2SeO4 < H2SO4
52. Cl > F > Br > I
53. HF < HCl < HBr < HI
54. I2 < Br2 < Cl2 < F2
55. HF < HCl < HBr < HI
56. HCl < HBr < HF < HI
Anomalous behaviour of HF is due to hydrogen bonding.
57. HCl < HBr < HI < HF
Anomalous behaviour of HF is due to hydrogen bonding.
58. HFO3 < HClO3 < HBrO3 < HIO3
Ions of these acids are stabilized due to strong pp-pp bonding between full 2p orbitals on oxygen and empty
orbitals on the halogen atom. Fluorine has no d orbitals and cannot form pp-dp
are not known.
59. TiCl2 < TiCl3 < TiCl4
Increasing oxidation state of Ti increases charge density on the metal leading to increase in the polarization of the
anionic charge clound and thus covalency increases.
60. Zn2+ < Ti3+ < Ni2+ < Co2+ < Cr2+
Increasing number of unpaired electrons increases magnetic moment. The number of unpaired electrons in the
given species are as follows.
Ti3+ one, Ni2+ two, Co2+ three, Cr2+ four and Zn2+ zero.
61. VCl4 < VCl3 < VCl2
Decreasing oxidation state of element increases the ionic character.
62. Hg < Cu < Na < Li
63. Al < Fe < Pb < Au
64. CO2 < B2O3 < BeO < Li2O
65. As < P < S < Cl
66. O<N<F
67. O < C < S < Se
68. HOI < HOBr < HOCl
69. H2 < NH3 < O2 < H2S < CO2
Increasing molecular mass increases the density.
70. HI < HBr < HCl < HF
71. F2 < Cl2 < O2 < N2
72. CaI2 < CaBr2 < CaCl2 < CaF2
Increasing size of anion increases its polarization by the cation making the compound more covalent.
73. Te < Se < S < O
74. I < Br < Cl < F
Annexure AN.15

75. B < Be < Li < Na


76. N — N < O—O < F—F
77. CsH < KH < NaH < LiH
78. LiCl > MgCl2 > BeCl2 > AlCl3
Hydrolysis of cations depends on two factors; larger charge and smaller size favour more hydrolysis, hence more
free H+ (i.e. lesser pH).
79. MgO < Al2O3 < SiO2 < P4O10
80. I – < Br – < Cl – < F–
Stronger the acid, weaker its conjugate base.
81. F – < OH – < NH–2 < CH3–
More electronegative the atom, lesser its tendency to give a lone pair of electrons.
82. PH3 < AsH3 < NH3 < SbH3
Boiling point increases with increase of molecular mass with the exception of NH3 which exhibits hydrogen
bonding
83. B<C<O<N

84. BeCO3 < MgCO3 < CaCO3 < BaCO3


Increasing size of cation decreases its polarizing ability towards carbonate making the compound more stable.
85. Ca < Al < O < N
Paramagnetism increases with increase of number of unpaired electrons.
86. LiBr < NaBr < KBr < RbBr < CsBr
The larger the difference between the electronegativities, greater the ionic character.
87. Ba2+ < Sr2+ < Ca2+ < Mg2+ < Be2+
The smaller the size, more the hydration energy.
88. AsH3 < PH3 < NH3
The increasing size and lower electronegativity of the central atom permit the bonding electrons to be drawn out
further, thus decreasing repulsion between bonding pairs.
89. AsF3 < PF3 < NF3
90. H2Se < H2S < H2O
91. NF3 < NCl3
The bonding pair repulsion in NF3 is less than that in NCl3
92. NO2+ > NO2 > NO2–
There is maximum repulsion between free electron(s) on nitrogen and bonding pairs.
93. NF3 < NH3
There is lesser repulsion in bonding pairs in NF3.
94. PH3 < PF3

P) increases the electron density and electronic repulsion among the bonding electron
95. O– < O < O+
The positive charge on atom increases its electronegativity while negative charge decreases its electronegativity.
Practice Test Paper–I
Practice Test Paper–I
Select the Correct Choice
1. Which of the following molecules has minimum bond angle?
(a) Ni(CO)4 (b) [Ni(CN)4]2– (c) CCl4 (d) NH4+
2. KCl when heated in K-vapours attains a magenta colour to produce
(a) Frenkel defect (b) Schottky defect (c) F-centers (d) Interstitial defect
3. The general formula of an ionic compound having zinc blende structure is
(a) MX (b) MX2 (c) M2X (d) MX3
4. The addition of a little of mercuric iodide to the aqueous solution of potassium iodide results into
(a) increase in freezing point
(b) decrease in freezing point
(c) no change in freezing point
(d) increase or decrease in freezing point depending upon the concentration of KI
5. Given: DionizH°(HCN) = 45.2 kJ mol–1 and DionizH°(CH3COOH) = 2.1 kJ mol–1. Which of the following facts is
correct?
(a) pK°a(HCN) = pK°a(CH3COOH) (b) pK°a(HCN) > pK°a(CH3COOH)
(c) pK°a(HCN) < pK°a(CH3COOH) (d) pK°a(HCN) = (45.2/2) K°a(CH3COOH)
6. The mass of Cl2 produced in the electrolysis of molten sodium chloride when 1 A of current is passed for 10 min
is
(a) 3.55 g (b) 1.775 g (c) 0.95 g (d) 0.22 g
7. For the cell Ag|Ag+| |Cu2+|Cu, which of the following is correct?
(a) Cu electrode is anode (b) Cu electrode is the positive terminal
(c) Ag electrode is cathode (d) Ag electrode is positive terminal
8. A substance (initial concentration a) reacts according to zero-order kinetics. What will be the time for the reaction
to go to completion?
(a) a/k (b) k/a (c) a/2k (d) 2k/a
9. The relative nucleophilicity of NH3, H2O and HF towards bromoethane is
(a) NH3 < H2O < HF (b) NH3 > H2O > HF (c) NH3 > H2O < HF (d) NH3 < H2O > HF
10. Aryl halides are less reactive towards nucleophilic substitution reactions as compared to alkyl halides due to
(a) the formation of less stable carbonium ion (b) resonance stabilization
(c) longer carbon-halogen bond (d) inductive effect
11. The composition of azeotropic mixture of alcohol and water at 1 atm pressure is
(a) 85% ethanol and 15% water (b) 90% ethanol and 10% water
(c) 95% ethanol and 5% water (d) 98% ethanol and 2% water
12. When HBr (one equivalent) adds to butadiene at 40 °C, the major product is
(a) CH3CHBrCH==CH2
(b) BrCH2CH==CHCH3
(c) BrCH2CH2CH==CH2
(d) a mixture of CH3CHBrCH==CH2 and BrCH2CH2CH==CH2
13. Which of the following dienes is the most stable?
(a) BrCH2CH==CHCHBrCH==CH2 (b) BrCH2CHBrCH==CHCH==CH2
(c) BrCH2CH==CHCH==CHCH2Br (d) H2C==CH—CHBrCHBrCH==CH2
PI.2 Complete Chemistry—JEE Main

14. Which of the following facts is correct?


(a) Sulphonation of toluene is a reversible reaction
(b) At higher temperature (100 °C), the percentage of p-toluene sulphonic acid is the major product
(c) At higher temperature (100 °C) desulphonation of sulphonated product occurs
(d) The percentage composition of o-toluenesulphonic acid, p-toluenesulphonic varies with temperature
15. Which of the following order regarding the rate of alkaline hydrolysis of esters is correct?
(a) CH3COOCH3 > CH3COOC2H5 > C2H5COOC2H5
(b) CH3COOCH3 < CH3COOC2H5 < C2H5COOC2H5
(c) CH3COOCH3 < CH3COOC2H5 > C2H5COOC2H5
(d) CH3COOCH3 > CH3COOC2H5 < C2H5COOC2H5
16. The correct order of reactivity of hydrolysis of acid chlorides is
(a) PhCOCl > p-O2NC6H4COCl > p-CH3OC6H4COCl
(b) PhCOCl > p-CH3OC6H4COCl > p-O2NC6H4COCl
(c) p-O2NC6H4COCl > PhCOCl > p-CH3OC6H4COCl
(d) p-O2NC6H4COCl > p-CH3OC6H4COCl > pHCOCl

of
(a) alcohol (b) aldehyde (c) ketone (d) carboxylic acid
18. The reduction of nitrobenzene with Sn/HCl produces
(a) azoxybenzene (b) azobenzene (c) hydrazobenzene (d) aniline
CH3 CH3
Conc. H 2 SO 4
19. In the reaction Ph C C Ph Æ A . The product A is
OH OH
CH3 CH3 CH3 CH3
CH3 CH3
(a) (b) Ph C C Ph (c) Ph C COCH3 (d) Ph C COPh
Ph C C Ph
O Ph CH3
20. The correct sequence regarding base strength of aliphatic amines in gaseous phase is
(a) R3N > R2NH > RNH2 > NH3 (b) R3N > RNH2 > R2NH > NH3
(c) R2NH > R3N > RNH2 > NH3 (d) R2NH > RNH2 > R3N > NH3
21. Which of the following statements is not correct?
(a) The bond angle in NH3 is greater than in NF3
(b) All ortho-substituted anilines are more basic than para-substituted anilines regardless of the electronic effect
of the substituent
(c) Amides are much weaker bases than amines
(d) The increasing order of basicities of pyrrole, pyridine and piperidine is pyrrole < pyridine < piperidine
22. Natural starch contains
(a) only amylopectin (b) 50% each of amylose and amylopectine
(c) 85% amylose and 15% amylopectine (d) 15% amylose and 85% amylopectine
23. Which of the following is hexabasic acid?
(a) Pyrophosphoric acid (b) Tripolyphosphoric acid
(c) Tetrapolyphosphoric acid (d) Polymetaphosphoric acid
24. Which of the following structures of TeCl4 is correct?
Cl Cl Cl
Cl Cl Cl Cl Cl
(a) Te (b) Te (c) Te (d) Te
.....

Cl Cl Cl Cl Cl Cl
Cl Cl
25. Which of the following acids contains S—S bond?
(a) H2S2O3 (b) H2S2O6 (c) H2S2O7 (d) H2S2O8
26. Which of the following molecules is not observed?
(a) BrF (b) BrF3 (c) BrF5 (d) BrF7
Practice Test Paper–I PI.3

27. Which of the following facts regarding the densities of 5d-series elements is correct?
(a) increase progressively (b) decrease progressively
(c) initially increase followed by decrease (d) initially decrease followed by increase
28. A substance which is not paramagnetic is
(a) Cr(ClO4)3 (b) KMnO4 (c) TiCl3 (d) VOBr2
29. On heating 24

(c) decreases with increase in temperature (d) remains unchanged


30. Which of the following orders regarding the electronegativity of the following species is correct?
(a) O+ > O > O– (b) O+ > O < O– (c) O+ < O > O– (d) O+ < O < O–

ANSWERS
1. (b) 2. (c) 3. (a) 4. (a) 5. (b) 6. (d)
7. (b) 8. (a) 9. (b) 10. (b) 11. (c) 12. (b)
13. (b) 14. (c) 15. (a) 16. (c) 17. (b) 18. (d)
19. (c) 20. (a) 21. (b) 22. (d) 23. (c) 24. (d)
25. (b) 26. (d) 27. (c) 28. (b) 29. (d) 30. (a)

HINTS AND SOLUTIONS


1. [Ni(CN)4]2– is a square planar while the other three species have tetrahedron structure.
4. The reaction occurring in the solution is 2K+ + 2I– + HgI2 æÆ 2K+ + HgI42–
There is decrease in the number of species (from 4 to 3), hence, – DTf decreases. This results into increase in
freezing point.
5. Larger the value of DionizH°, the weaker the acid, hence, larger the value of pK°a.
6. Since m = (Q/F) (M/|ve|), for the oxidation of Cl– we get
 1 × 10 × 60 C   35.5 mol  ~ 1 g.
1
m=
 96500 C mol 1
 1  5
7. The right-hand electrode is cathode which carries positive charge.
8. For zero-order reaction, [A]0 – [A]t = kt. For the reaction to go to completion [A]t = 0. Hence, t = [A]0/k = a/k.
15. With increase in the bulkiness of R and R¢ group in RCOOR¢, the alkaline hydrolysis becomes slower.
16. The electron-withdrawal group increases hydrolysis while the electron-releasing group decreases hydrolysis of
—COCl group.
19. The reaction proceeds through the migration of phenyl group.
20. In the gaseous phase, only inductive effect operates.

28. Mn in KMnO4 does not carry unpaired electron.


30. The positive species has the maximum electronegativity while negative species has the minimum electronegativity.
Practice Test Paper–II
Practice Test Paper–I
Select the Correct Choice
1. The expression converting molality (m) into molarity (M) is
(a) M = (1 + mM2)/mr (b) M = mr/(1 + mM2) (c) M = mM2/r (d) M = mr/M2
where M2 is the molar mass of solute and r is the density of the solution.
2. Which of the following units corresponds to the physical quantity entropy?
(a) kg m2 s–2 K–1 (b) kg m s–1 K–1 (c) kg m s–2 K–1 (d) kg m K
3. Which of the following conversions is correctly formulated?
(a) 1 fm = 10–18 m (b) 1 am = 10–15 m (c) 1 Tm = 1015 m (d) 1 Em = 1018 m
4. The van der Waals constant b s related to the radius of molecules (assume to be spherical) by the expression
(a) b = NA(4 p r3/3) (b) b = (4 p r3)NA (c) b = 4NA(4 p r3/3) (d) b = 16NA(4 p r3/3)
5. Which of the following liquids is expected to have maximum viscosity?
(a) Ethyl alcohol (b) Benzene (c) Ethylene glycol (d) n-Butyl alcohol
6. Which of the following dimensions of a unit cell represents a tetragonal unit cell?
(a) a = b = c; a = b = g = 90° (b) a = b a = b = g = 90°
(c) a a = b = g = 90° (d) a = b = c; a = b g
7. Which of the following statements is not correct?

helium positive ion

positive ion

helium positive ion.


8. The state of hybridization of oxygen in OF2 is
(a) sp (b) sp2 (c) sp3 (d) dsp2
9. Which of the following characteristics will always lead to a spontaneous chemical reaction?
(a) DH = +ve; DS = +ve (b) DH = +ve; DS = –ve (c) DH = –ve; DS = –ve (d) DH = –ve; DS = +ve
10. A solution contains 0.1 mol of each of Cl–, Br–, I– and CrO2– 4 ions per litre of the solution. If solid AgNO3 is
gradually added, the correct order of precipitation of ions is
(a) Cl–, Br–, I–, CrO2–
4 (b) I–, Br, Cl–, CrO2–
4 (c) CrO2– – – –
4 , Cl , Br , I (d) CrO2– – –
4 , I , Br , Cl

11. If
CO2 + H2O   
+
 H + HCO 3;

K1 = 4.2 × 10–7 M

HCO3– 

+ 2–
 H + CO3 ; K2 = 4.8 × 10–11 M
The pH of 0.1 M NaHCO3 would be about
(a) 6.0 (b) 7.0 (c) 8.4 (d) 10.5

(a) Li+ > Na+ > K+ (b) Li+ > Na+ < K+ (c) Li+ < Na+ > K+ (d) Li+ < Na+ < K+
13. Which of the following statements is correct?
(a) Cathode is negative terminal in an electrolytic cell and is positive in a galvanic cell
(b) Cathode is positive terminal in an electrolytic cell and is negative in a galvanic cell
PII.2 Complete Chemistry—JEE Main

(c) Cathode is positive in both electrolytic and galvanic cells


(d) Cathode is negative in both electrolytic and galvanic cells
14. The rate constant of a reaction is found to be 1.5 × 10–2 mol L–1 s–1. The reaction follows

15. The correct order of stability of isomers of pentanes is


(a) CH3(CH2)3CH3 > (CH3)2CHCH2CH3 > (CH3)4C
(b) CH3(CH2)3CH3 > (CH3)2CHCH2CH3 < (CH3)4C
(c) CH3(CH2)3CH3 < (CH3)2CHCH2CH3 > (CH3)4C
(d) CH3(CH2)3CH3 < (CH3)2CHCH2CH3 < (CH3)4C
16. The addition of HBr (1 equivalent) to 1-pentene-4-yne is
(a) H2C==CHCH2C(Br)==CH2 (b) H2C==CHCH2CH==CHBr
(c) H3CCHBrCH2C∫∫CH (d) BrH2CCH2CH2C∫∫CH
17. The addition of HBr to butadiene at –80°C produces
(a) 3-bromobut-1-ene
(b) 4-bromobut-1-ene
(c) a mixture of 3-bromobut-1-ene and 4-bromobut-1-ene
(d) 1-bromobut-2-ene
18. The order of reactivity of hydrogen halide acids to convert ROH to RX is
(a) HI < HBr < HCl (b) HI < HBr > HCl (c) HI > HBr < HCl (d) HI > HBr > HCl
19. Which of the following substituted benzoic acids is expected to have maximum value of pK°a?
(a) p-Nitrobenzoic acid (b) p-toluic acid (c) p-chlorobenzoic acid (d) p-hydroxybenozic acid
20. The correct order of basicity of the following compounds is
(a) CH3CH2CH2NH2 > H2C==CHCH2NH2 > HC CCH2NH2
(b) CH3CH2CH2NH2 > H2C==CHCH2NH2 < HC CCH2NH2
(c) CH3CH2CH2NH2 < H2C==CHCH2NH2 > HC CCH2NH2
(d) CH3CH2CH2NH2 < H2C==CHCH2NH2 < HC CCH2NH2
21. The correct order of pK°a of given acids is
(a) pK°a(RCOOH) > pK°a(H2CO3) > pK°a(PhOH)
(b) pK°a(RCOOH) < pK°a(H2CO3) > pK°a(PhOH)
(c) pK°a(RCOOH) > pK°a(H2CO3) < pK°a(PhOH)
(d) pK°a(RCOOH) < pK°a(H2CO3) < pK°a(PhOH)
22. Ten grams of a sample of starch in 1 L of aqueous solution produces osmotic pressure of 0.5 kPa at 25 °C. The
average number of glucose units in the sample is
(a) 100 (b) 200 (c) 300 (d) 400
23. The G–C base pairing involves
(a) One H-bonds (b) Two H-bonds (c) Three H-bonds (d) Four H-bonds
24. The number of P—O coordinate bonds in P4O10 is
(a) one (b) two (c) three (d) four
25. Which of the following structures of XeO2F2 is correct?
F
O O O
F F
Xe
(a) (b) Xe

O
F
F O O
(c) Xe (d) Xe
O F O F F
26. Which of the following minerals is a silicate mineral?
(a) Kainite (b) Diaspore (c) Feldspar (d) Alunite
27. In diborane, boron involves
(a) no hybridization (b) sp hybridization (c) sp2 hybridization (d) sp3 hybridization
Practice Test Paper–II PII.3

28. Which of the following relations between bond enthalpies is correct?


(a) e(P P) > 3 e(P – P)
(b) e(P P) < 3 e(P – P)
(c) e(P P) < 2 e(P – P)
(d) e(P P) > 2 e(P = P) + e(P – P)
29. Lead sesquioxide is
(a) PbO (b) PbO2 (c) Pb2O (d) Pb2O3
30. Which of the following salts does not liberate oxygen or nitrogen when heated strongly?
(a) K2Cr2O7 (b) NH4NO3 (c) AgNO3 (d) (NH4)2Cr2O7

ANSWERS
1. (b) 2. (a) 3. (d) 4. (c) 5. (c) 6. (b)
7. (d) 8. (c) 9. (d) 10. (b) 11. (c) 12. (d)
13. (a) 14. (a) 15. (d) 16. (c) 17. (a) 18. (d)
19. (d) 20. (a) 21. (d) 22. (c) 23. (c) 24. (d)
25. (a) 26. (c) 27. (d) 28. (b) 29. (d) 30. (b)

HINTS AND SOLUTIONS


1. We have
n2 n2 ( n2 m1 )r mr mr
M= =
V ( m1 m2 ) / r 1 ( m2 m2 ) 1 {( m2 M1 ) m1} M2 1 m M2
2. We have J K–1 = (N m) K–1 = (kg m s–2)m K–1 = kg m2 s–2 K–1.
3. 1 am = 10–18 m; 1 Tm = 1012 m; 1 Em = 1018 m and 1 fm = 10–15 m.
4. The van der Waals constant b is four times the molar volume occupied by molecules of a gas.
5. Ethylene glycol involves larger hydrogen bondings.
6. For tetragonal, a = b c and a = b = g = 90°.
7. E μ – Z2/n2; r μ n2/Z; u μ Z/n and PE μ – Z2/n2.
8. There are four unpaired electrons around oxygen atom. Hence, it involves sp3 hybridization.
9. DG must be negative for a spontaneous reaction. Since DG = DH – TDS, a reaction having DH negative and DS
positive will have DG negative at all temperatures.
11. pH = (1/2) (pK°a1 + pK°a2) = – (1/2) [log K°a1 + log K°a2) (1/2) (6.4 + 10.3) 8.4.
12. Because of hydration of ions in water, the size varies as Li+ > Na+ > K+. Hence, conductivity follows reverse
order.
14. From the unit of rate constant, the order of the reaction is zero.
19. p-Hydroxybenzoic acid is the weakest of the given acids. Hence, it has maximum value of pK°a.
20. The larger the s-character of a hybrid orbital of carbon, the larger electron-withdrawing effect it has, consequently,
the larger is its base-weakening effect.
21. Since Ka(RCOOH) > Ka(H2CO3) > Ka(PhOH), pK°a values follow reverse order.
22. Since P = cRT, we have
 10 g M  (8.314 kPa dm
 1 dm 
3
0.5 kPa = 3
K–1 mol–1) (298 K)

This gives M50 000 g mol–1


M 50000 g mol 1
n= = 300
M ( C 6 H 6O 6 ) 174 g mol 1
PII.4 Complete Chemistry—JEE Main

30. On heating given salt, we observe the following


4K2Cr2O7 Æ 4K2CrO4 + 2Cr2O3 + 3O2
NH4NO3 Æ N2O + 2H2O
2AgNO3 Æ 2Ag + 2NO2 + O2
(NH4)2Cr2O7 Æ N2 + Cr2O3 + 4H2O
Practice Test Paper–III
Practice Test Paper–I
Select the Correct Choice
1. Amongst the following, identify the species with an atom in +6 oxidation state.
(a) MnO–4 (b) Cr(CN)63– (c) CrO2Cl2 (d) NiF62–
2. The temperature of a given mass of a gas is increased from 19°C to 21°C at constant pressure. The volume V of
the gas is increased
(a) to V(21/19) (b) by 2/273.15 of its volume at 0 °C
(c) by 2/273.15 of its volume at 0 K (d) by a factor of 2/273.15
3. Which of the following expressions is correct for CsCl unit cell (edge length a)?
(a) rc + ra = a (b) rc + ra = a/ 2 (c) rc + ra = 3 a/2 (d) rc + ra = a/2
4. Which of the following statements is correct?

dipositive ion

ion
-
tive ion

5. The type of hybrid orbitals used by chlorine atom in ClO–2 is


(a) sp3 (b) sp2 (c) dsp2 (d) sp
6. Molar heat capacity of water in equilibrium with ice at constant pressure is
(a) zero (b) 40.45 kJ K–1 mol–1 –1
mol–1
7. For a concentrated solution of a weak electrolyte Ax y, the degree of dissociation is given by
(a) a = K eq / c ( x y) (b) a = Keq / c ( xy)
x+y–1 y x x+y x +y–1 x y 1/(x +y)
(c) a = (Keq/c x y ) (d) a = (Keq/c x y)
2+
|Cu and Cu2+|Cu+ are 0.337 V and 0.153 V, respectively. The standard potential
+
of Cu |Cu would be

9. The hydrolysis of an ester was carried out separately with 0.05 M HCl and 0.05 M H2SO4. Which of the following
expressions is correct?
(a) kHCl > kH2SO4 (b) kHCl < kH2SO4 (c) kHCl = kH2SO4 (d) kH2SO4 = 2kHCl
10. The correct order of ionization energy is
(a) Si > P > S (b) Si > P < S (c) Si < P > S (d) Si < P < S
11. The bond angle P—P—P in P4 is about
¢ (d) 120°
12. The thermal stability of carbonates of alkaline earth metals follows the order
(a) MgCO3 < CaCO3 3 3 < CaCO3 < MgCO3
3 < MgCO 3 < CaCO 3 (d) CaCO 3 < MgCO3 3
13. The number of oxygen atoms attached to each P in P2O5 is
(a) 2 (b) 2.5 (c) 3 (d) 4
PIII.2 Complete Chemistry—JEE Main

14. Which of the following is a reducing agent?


(a) Orthophosphorous acid (b) Hypophosphoric acid
(c) Orthophosphoric acid (d) Pyrophosphoric acid
15. Which of the following statements is correct?
(a) XeF4 is tetrahedral while XeO4 is square planar
(b) XeF4 is square planar while XeO4 is tetrahedral
4 and XeO4 are square planar
4 and XeO4 are tetrahedral
16. Molecular shapes of SF4, CF4 and XeF4 are
(a) the same with 2, 0 and 1 lone pairs of electrons, respectively.
(b) the same with 1, 1 and 1 lone pairs of electrons, respectively.
(c) different with 0, 1 and 2 lone pairs of electrons, respectively.
(d) different with 1, 0 and 2 lone pairs of electrons, respectively.
17. The spin magnetic moment of 24Cr3+ is
(a) 1.73 m m m (d) 4.90 m

(a) SO42– (b) PO43– (c) NO–3 (d) I–


19. Ammonia can be dried by
(a) conc. H2SO4 (b) P4O10 (c) CaO (d) anhydrous CaCl2

(a) XeF2 (b) XeF3 (c) XeF4 (d) XeF6


21. The major product obtained in the photobromination of 2-methylbutane is
(a) 1-bromo-2-methylbutane (b) 3-bromo-2-methylbutane
(c) 2-bromo-2-methylbutane (d) 2-bromo-3-methylbutane
22. An alkene on oxidative ozonolysis gives adipic acid. The alkene is
(a) cyclohexene (b) 1-methylcyclopentene
(c) 1, 2-dimethylcyclobutene (d) 3-hexene
23. Which of the following statements is not correct?
(a) The number of isomers (including structural and stereoisomers) of alkyne C6H10 is eight
(b) The treatment of 1, 1-dibromopentane with KOH(s) at 200°C produces 2-pentyne
(c) Acetylene, like ethylene, is soluble in concentrated H2SO4
2
24. Which of the following statements is correct?
(a) The reaction of benzene with n-propyl chloride in the presence of AlCl3 gives 1-phenylpropane as the only
product
(b) Though —N(CH3)2 is an ortho- and para-director yet it gives meta nitrated product with the more powerful
electrophilic reagent HNO3/H2SO4
(c) Phenyl cation is more stable than cyclohexyl cation

25. Which of the following statements is not correct?


(a) Cyclohexanol is more soluble in water than 1-hexanol
(b) Alcohol is slightly more acidic than water
N1 mechanism
(d) SN1 nucleophilic substitution in ROH may lead to the rearrangement of carbon skeleton
CH3
conc. H2SO4
26. In the reaction CH3 C CH2 A the product A is
OH OH

CH3 H H CH3 CH3


(a) CH3 C CH2 (b) CH3 C C CH3 (c) CH3CH2 C O (d) CH3 CH CHO
Practice Test Paper–III PIII.3

27. Which of the following statements is correct?


(a) 1° amide liberates N2 when treated with HNO2
(b) 2° amide liberates N2 when treated with HNO2
(c) 3° amide liberates N2 when treated with HNO2
(d) None of 1°, 2° and 3° amides liberate N2 when treated with HNO2

(a) Phenol is stronger acid than an alcohol


(b) Decreasing order of acidic character of chlorophenols is
p-ClC6H4OH > o-ClC6H4OH > m-ClC6H4OH
(c) Sulphonation of phenol is a reversible reaction.
(d) p-Hydroxyacetophenone can be converted to o-hydroxyacetophenone on increasing the temperature from
25°C to 165°C.
29. The correct sequence regarding base strength of aliphatic amines in aqueous solution is
(a) R3N > R2NH > RNH2 (b) R2NH > RNH2 > R3N
(c) R2NH > R3N > RNH2 (d) RNH2 > R2NH > R3N
30. Which of the following is a linear polymer?
(a) Amylopectin (b) Glycogen (c) Starch (d) Amylose

ANSWERS
1. (c) 2. (b) 3. (c) 4. (b) 5. (a) 6. (c)

19. (c) 20. (b) 21. (c) 22. (a) 23. (c) 24. (b)

HINTS AND SOLUTIONS


1. The oxidation states are as follows.
MnO4– x + 4(– 2) = – 1 fi x=+7
Cr(CN)63– x + 6(– 1) = – 3 fi x=+3
CrO2Cl2 x + 2(– 2) + 2(– 1) = 0 fi x=+6
NiF62– x + 6(– 1) = – 2 fi x=+4
2. Since Vt = V0 + (V0/237.15 °C) t, we have
V0 2 V0
Vt2 – Vt1 = (t2 – t1) =
273.15 ° C 273.15
Here V0 is the volume of gas at 0°C.
3. The unit cell of CsCl is body centred. Hence 2(r+ + r–) = 3 a
4. We have E μ – Z2/n2; r μ n2/Z and u μ Z/n
5. There are four unpaired electrons around Cl in ClO2–. Hence, Cl involves sp3 hybrid orbitals.
6. Heat added to the system does not increase its temperature.
7. We have
Ax y


y+
 xA + y
x–

c(1 – a) c(xa) c(ya)

[ A y + ] x [ B x - ] y (c x a ) x (c y a ) y
Keq = = cx + y – 1 xx yy ax + y
[A x B y ] c(1 - a )
Hence, a = (Keq/cx + y – 1 xx yy)1/(x + y)
PIII.4 Complete Chemistry—JEE Main

Cu2+ + 2e– = Cu; E°1 = 0.337 V; DG°1 = – 2FE°1


Cu2+ + e– = Cu+; E°2 = 0.153 V; DG°2 = – FE2°
Cu+ + e– = Cu; E°3 = ? DG°3 = DG°1 – DG°2

DG3
E°3 = = 2E°1 – E°2 = [2(0.337) – 0.153]V = 0.521 V
-F
9. We will have [H+]0.05M H2SO4 > [H+]0.05M HCl
Since the rate constant is directly proportional to [H+], it follows that kHCl < kH2SO4.
10. Within a period, there is a tendency to increase the ionization energy with increase in atomic number of the

11. The bond angle in P4 is 60°. This makes the tetrahedron arrangement of phosphorus atoms less stable.
12. Thermal stability increases on descending the group.
13. Each phosphorus atom is surrounded by four oxygen atoms.
14. The orthophosphorous acid contains hydrogen atom directly attached to phosphorus atom.
15. The structure of XeF4 and XeO4 are
O

F F
Xe Xe
O O
F F
O
Square planar Tetrahedran
16. The shapes are different. SF4 has one lone pairs of electrons, CF4 has none, and XeF4 has two line pairs of
electrons.
5
(4s)1. Thus, Cr3+ has (3d)3
Its spin magnetic moment is n( n + 2 ) μ B , i.e. 15 μB = 3.87 μ B.

19. Ammonia does not react with CaO.


20. XeF3 is not known.
21. Tertiary radical is most stable. Hence, the product will be 2-bromo-2- methylbutane.

COOH O3
22. COOH
Adipic acid Cyclohexene
24. In the presence of HNO3/H2SO4, —N(CH3)2 is converted into electron-seeking cation which directs the incoming
group to meta position.
CH3 CH3 CH3
conc. H2SO4
26. CH3 C CH2 CH3 C CHOH CH3 CH CHO
OH OH

increasing separation of Cl and OH, predominates, making all the chlorophenols more acidic than phenol. The
correct sequence of acidic character is
o-chlorophenol > m-chlorophenol > p-chlorophenol
29. The base strength of aliphatic amines is decided by the following two factors.
Induction factor Increasing replacement of H by R (electron-releasing group) increases electron density on N,
thus making it more basic.
Practice Test Paper–III PIII.5

Solvation factor Stabilization of conjugate acid through H-bonding with water. More the number of H atoms,
more the stability and thus more basic the amine.
Thus, induction and solvation operate in opposite directions. Induction dominates to make all three alkylamines
more basic than ammonia. However, adding a third R as in R3N does not further enhance the basicity because
now the opposing solvation effect assumes more importance.
Practice Test Paper–IV
Practice Test Paper–I
Select the Correct Choice
1. The compound having +2 as the oxidation state of oxygen is
(a) H2O2 (b) CO2 (c) F2O (d) MnO2
2. Which of the following is not SI unit of surface tension?
(a) N m–1 (b) J m–2 (c) kg s–2 (d) J m–1
3. Which of the following conversions is not correctly represented?
(a) 1 fm = 10–12 m (b) 1 Gm = 109 m (c) 1 Em = 1018 m (d) 1 nm = 10–9 m
4. The ratio Cp,m/CV,m for linear triatomic molecules including vibration contribution is given by
(a) 1.667 (b) 1.286 (c) 1.154 (d) 1.167
5. The SI unit of viscosity is
(a) N m–1 s (b) N m–2 s (c) N m–2 s–1 (d) N m–1 s–2
6. Which of the following statements is not correct?
(a) The orbital 3dx2 – y2 x- and y-axes
(b) The energy of 3d orbital may be greater than or less than or equal to that of 4s orbital depending upon the
atomic number of the atom
b a-particles in spite of a-particles having larger
charge
(d) Pauli exclusion principle is followed by fermiones
7. Which of the following statements is correct?
(a) Electronegativity is the property of an isolated atom
(b) The third period of periodic table contains 18 elements because third quantum shell includes 3s, 3p and 3d
orbitals
(c) Inert pair effect refers to the two relatively stable and unreactive outer s electrons

8. Which of the following resonating structures for CO2 is not correctly represented?
+ –
(a) O C O (b) – O C O + (c) + O C O – (d) O C O
9. Which of the following substances does not crystallise in the rock-salt structure?
(a) NaCl (b) KCl (c) MgO (d) CsCl
–1
aqueous solution of formic acid is –0.21°C. If Kf(water) is 1.86
K kg mol–1, the degree of dissociation of formic acid in the solution would be
(a) 0.0132 (b) 0.0264 (c) 0.0396 (d) 0.0528
11. The enthalpy of hydrogenation of a double bond is –119 kJmol–1
–1
–1
(a) –152 kJ mol (b) –172 kJ mol–1 (c) 152 kJ mol–1 (d) 172 kJ mol–1

increased?
(a) continuous increase (b) continuous decrease

13. A dilute aqueous solution of Na2SO4 is electrolysed using platinum electrodes. The products at the anode and
cathode are
(a) O2, H2 (b) S2O2–
8, Na (c) O2, Na (d) H2, O2
PIV.2 Complete Chemistry—JEE Main

14. Which of the following statements is correct?


(a) The half-life period of a reaction is independent of temperature
(b) A catalyst decreases the enthalpy change of a chemical reaction
(c) Truely speaking, the acid hydrolysis of an ester is a third order reaction

-
rations?
CH3 OH CH CH2 H
H OH H CH3 CH3 OH HO CH CH2
CH CH2 CH CH2 H CH3
(I) (II) (III) (IV)

(a) (II) and (III) (b) (I) and (IV) (c) (II) and (IV) (d) (III) and (IV)
16. Which of the following compounds is expected to have maximum enol content in ethanol?
(a) CH3COCH2CO2CH3 (b) CH3COCH2CO2C2H5 (c) CH3COCH2COCH3 (d) CH3COCH(CH3)COCH3
17. Which of the following orders of reactivity of acid derivatives towards a nucleophile is correct?
(a) acid chloride > anhydride > ester (b) acid chloride < anhydride < ester
(c) acid chloride > ester > anhydride (d) anhydride > acid chloride > ester
18. Which of the following dicarboxylic acids gives cyclic ketone on heating?
(a) CH2(COOH)2 (b) (CH2COOH)2 (c) HOOC(CH2)3COOH (d) HOOC(CH2)4COOH
19. The correct order of base strength of the following amines is
(a) o-nitroaniline > p-nitroaniline > m-nitroaniline (b) m-nitroaniline > p-nitroaniline > o-nitroaniline
(c) m-nitroaniline > o-nitroaniline > p-nitroaniline (d) o-nitroaniline > m-nitroaniline > p-nitroaniline
20. In alkaline medium, fructose is
(a) a reducing sugar (b) a nonreducing sugar (c) an aldose (d) a furanose
21. Which of the following is an essential amino acid?
(a) Glycine (b) Proline (c) Cysteine (d) Lysine
22. Which of the following polymers contains ester linkages?
(a) Terylene (b) Nylon-66 (c) Bakelite (d) Styron
23. Which of the following dyes belongs to mordant dye?

24. Which of the following statements is correct?


(a) Red phosphorus is less volatile than white phosphorus
(b) Silicon does not form Si==O double bonds

(d) The structure of N(SiH3)3 is pyramidal


25. Which of the following statements is correct?
(a) Anhydrous MgCl2 is obtained by heating MgCl2 ◊ 6H2O
(b) PbO2 on reacting with dilute acid produces hydrogen peroxide

(d) Burning of sodium in air produces sodium monoxide


26. Which of the following statements is not correct?
(a) Silver nitrate gives white precipitate with concentrated solution of sodium thiosulphate
(b) A current of chlorine displaces bromide from silver bromide
(c) Tin(II) chloride is a reducing agent
(d) Ferric chlorides exists as a dimer in gaseous phase.
27. Which of the following statements is not correct for hypophosphoric acid?

air
(b) It is a tribasic acid
(c) It has no reducing characteristics
(d) In warm water it undergoes hydrolysis producing phosphorous acid and phosphoric acid
Practice Test Paper–IV PIV.3

28. Which of the following statements is correct?


(a) Both potassium ferrocyanide and potassium ferricyanide are diamagnetic
(b) Both potassium ferrocyanide and potassium ferricyanide are paramagnetic
(c) Potassium ferrocyanide is paramagnetic while potassium ferricyanide is diamagnetic
(d) Potassium ferrocyanide is diamagnetic while potassium ferricyanide is paramagnetic
29. Which of the following is epsom salt?
(a) FeSO4 ◊ 7H2O (b) MgSO4 ◊ 7H2O (c) ZnSO4 (d) Na2SO4 ◊ 10H2O
30. The compound [Pt(NH3)2Cl2] can form
(a) geometrical isomers (b) coordination isomers (c) linkage isomers (d) optical isomers

ANSWERS
1. (c) 2. (d) 3. (a) 4. (c) 5. (b) 6. (a)
7. (c) 8. (c) 9. (d) 10. (b) 11. (a) 12. (c)
13. (a) 14. (c) 15. (c) 16. (c) 17. (a) 18. (d)
19. (b) 20. (a) 21. (d) 22. (a) 23. (b) 24. (b)
25. (c) 26. (a) 27. (b) 28. (d) 29. (b) 30. (a)

HINTS AND SOLUTIONS


1. Fluorine is more electronegative than oxygen.
3. 1 fm = 10–15 m
4. For a triatomic molecule
3 13 13 15
CV, m = R + R + 4R = R; Cp, m = R+R= R
2 2 2 2
Hence, Cp, m /CV, m = 15/13 = 1.154
8. The structure +
O C O – is not consistent with the charges and number of electrons possessed by the
atoms.

10. HCOOH  


 HCOO + H ;
+

m(1 – a) ma ma Total molality = m (1 + a)


Hence, m(1 + a) Kf = DTf . Thus
Tf 0.21 K
= 1 1 = 0.0264
mK f (0110
. mol kg 1 ) (186
. kg mol 1 )
11. The resonance energy is – 3 (119 kJ mol–1) + 205 kJ mol–1 = – 152 kJ mol–1.
12. Zn(OH)2 is amphoteric. Its solubility is more both in acidic and alkaline solution.
14. The acid hydrolysis of an ester follows the rate law r = k[ester] [H+] [H2O]
15. The Fischer projection may be moved in the following ways without changing the meaning of the projection.
(i) Rotating the Fischer projection by 180°
(ii) Hold any group steady and rotate the other three in either a clockwise or a counter clockwise direction.
The projection (iv) can be converted into the projection (ii) by carrying out the second operation (listed above)
twice.
24. Silicon forms —O—Si—O—Si— linkages. It does not form pp–pp double bonds with oxygen due to its large

25. Solubility of sliver halides decreases in the order AgF > AgCl >AgBr > AgI
26. Silver nitrate gives eventually black precipitates with sodium thiosulphate.
PIV.4 Complete Chemistry—JEE Main

OH OH
27. Hypophosphoric acid is O P P O . . So, it is tetrabasic acid.
OH OH
4–
6 and Fe(CN)63– respectively are

No unpaired electrons
(diamagnetic)

d2sp3 hybridization
One unpaired electron
(paramagnetic)

d2sp3 hybridization
Practice Test Paper–V
Practice Test Paper–I
Select the Correct Choice
1. The amount of KMnO4 that will be needed to react with one mole of sulphite ion in acidic solution is
(a) (2/5) mol (b) (3/5) mol (c) (4/5) mol (d) 1 mol
2. At a given temperature if r(X) = 3 r(Y) and and M(Y) = 2M(X), where r and M stand for density and molar mass
of the gases X and Y, respectively, then the ratio of their pressures would be
(a) p(X)/p(Y) = 1/4 (b) p(X)/p(Y) = 4 (c) p(X)/p(Y) = 6 (d) p(X)/p(Y) = 1/6
3. The edge length of unit cell of sodium chloride is 564 pm. If the size of Cl– ion is 181 pm, the size of Na+ ion
would be
(a) 101 pm (b) 167 pm (c) 202 pm (d) 383 pm
4. Which of the following statements for crystals having Schottky defect is not correct?
(a) Schottky defect arises due to the absence of a cation or anion from the position which it is expected to occupy.
(b) Schottky defects are more common in ionic compounds with high coordination number.
(c) The density of the crystals having Schottky defect is larger than that of the perfect crystal.
(d) The crystal having Schottky defect is electrical neutral as a whole.
5. For a d-electron, the orbital angular momentum is
(a) 6 (h/2p) (b) 2 (h/2p) (c) (h/2p) (d) 2(h/2p)

(a) O > S > Se (b) O < S > Se (c) O > S < Se (d) O < S < Se
8. The hybridization of atomic orbitals of nitrogen in NO+2, NO–3 and NH+4 respectively are
(a) sp, sp3 and sp2 (b) sp, sp2 and sp3 (c) sp2, sp and sp3 (d) sp2, sp3 and sp
3
9. The reaction NH4CN(s) + O2 2(g) + CO2(g) + H2O(l) was carried out in a bomb calorimeter. The
2
heat released was 743 kJ mol–1. The value of DH300K for this reaction would be
(a) –740.5 kJ mol–1 (b) –741.75 kJ mol–1 (c) –743.0 kJ mol–1 (d) –744.25 kJ mol–1
10. When NaNO3 is heated in a closed vessel, oxygen is liberated and NaNO2 is left behind. At equilibrium
(a) addition of NaNO2 favours reverse reaction
(b) addition of NaNO3 favours forward reaction
(c) Increasing temperature favours forward reaction
(d) increasing pressure favours forward reaction
11. Which of the following aqueous solution will be acidic in nature at 40 °C? Given: pK°w at 40 °C = 13.54.
(a) 6.6 (b) 6.8 (c) 7.0 (d) 7.2
12. The solubility of sparingly soluble electrolyte MmAa in water is given by the expression
(a) [Ksp/(mm aa)]m + a (b) [Ksp/(ma am)]m+a (c) [Ksp/(ma am)]1/(m+a) (d) [Ksp/(mm aa)]1/(m+a)
13. The density of Cu is 8.94 g cm–3. The quantity of electricity needed to plate an area of 100 cm2 to a thickness of
10–2 cm using CuSO4 solution is
(a) 13586 C (b) 20379 C (c) 27172 C (d) 40758 C
PV.2 Complete Chemistry—JEE Main

14. The reduction potential of hydrogen half-cell will be negative if


(a) p(H2) = 1 atm and [H+] = 1 M (b) p(H2) = 1 atm and [H+] = 2 M
+
(c) p(H2) = 2 atm and [H ] = 2 M (d) p(H2) = 2 atm and [H+] = 1 M
15. The freezing point of equimolal aqueous solutions will be highest for
(a) C6H5NH3Cl (anilinium chloride) (b) Ca(NO3)2
(c) La(NO3)3 (d) C12H22O11 (sucrose)
16. For the chemical equations
1
N 2O 5 2+ O2 rate constant = k
2
2N2O5 2 + O2 rate constant = k¢
which of the following expressions is correct?
(a) k = k¢ (b) k = 2k¢ (c) 2k = k¢ (d) k = 3k¢
17. The highest bond enthalpy is shown by halogen
(a) F2 (b) Cl2 2 (d) I2
18. In the dichromate ion
(a) 3Cr—O bonds are equivalent (b) 4Cr—O bonds are equivalent
(c) 6Cr—O bonds are equivalent (d) 7Cr—O bonds are equivalent
19. Hypophosphorous acid is
(a) monobasic (b) dibasic (c) tribasic (d) neutral
20. Which of the following statements is not correct?
(a) The complexes [Ni(CO)4] and [NiCl4]2– differ in the state of hybridization.
(b) The complexes [Ni(CO)4] and [NiCl4]2– differ in the magnetic properties
(c) The complexes [Ni(CO)4] and [NiCl4]2– differ in the primary valencies of nickel
(d) The complexes [Ni(CO)4] and [NiCl4]2– have nickel in different oxidation states
21. Which of the following statements is not correct?
(a) Ti(NO3)4 is a colourless compound
(b) [Cu(NCCH3)4]+ –4 is a colourless compound
(c) [Cr(NH3)6]3+ 3Cl– is a coloured compound
(d) K3 6] is a colourless compound
(Given:
22. Dien is an example of
(a) monodentate ligand (b) bidentate ligand (c) tridentate ligand (d) hexadentate ligand
23. Which of the following is the 4n + 1 radioactive series?
(a) Thorium series (b) Neptunium series (c) Uranium series (d) Actinium series
24. Which of the following statements regarding alkanes is not correct?

(b) Chlorine has a much lesser tendency to replace tertiary hydrogen in preference to secondary or primary
(c) More reactive reagent shows less selectivity in reaction
(d) Chlorination of methane is retarted in the presence of oxygen
25. An alkene on reductive ozonolysis gives 2 molecules of CH2(CHO)2. The compound is
(a) 2,4-hexadiene (b) 1,3-cyclohexadiene
(c) 1,4-cyclohexadiene (d) 1-methyl-1, 3-cyclopentadiene
26. Which of the following orders regarding base strength is correct?
(a) CH3COO– > CH3CH2O– > HC ∫∫ C– (b) CH3COO– < CH3CH2O– < HC ∫∫ C–
(c) HC ∫∫ C– > CH3COO– > CH3CH2O– (d) HC ∫∫ C– > CH3CH2O– > CH3COO–
27. Which of the following sequences regarding activating effects of the given o, p-directors is correct?
(a) —O– < —OH < —OCOCH3 (b) —O– < —OH > —OCOCH3

(c) —O > —OH < —OCOCH3 (d) —O– > —OH > —OCOCH3
28. Which of the following alcohols does not give iodoform test?
(a) (CH3)2CH(OH)CH3 (b) PhCH(OH)CH2CH3
(c) 1-methylcyclohexanol (d) CH3CH2 CH(OH)CH3
Practice Test Paper–V PV.3

29. The correct sequence regarding the activity of the given acid derivatives is
(a) RCOCl > RCON3 > (RCO)2O (b) RCOCl > (RCO)2O > RCON3
(c) RCON3 > RCOCl > (RCO2)2O (d) RCON3 > (RCO)2O > RCOCl
30. Which of the following polymers contains amide linkages?

ANSWERS
1. (a) 2. (c) 3. (a) 4. (c) 5. (a) 6. (a)
7. (b) 8. (b) 9. (b) 10. (c) 11. (a) 12. (d)
13. (c) 14. (d) 15. (d) 16. (b) 17. (b) 18. (c)
19. (a) 20. (a) 21. (d) 22. (c) 23. (b) 24. (b)
25. (c) 26. (b) 27. (d) 28. (b) 29. (a) 30. (c)

HINTS AND SOLUTIONS


1. The reaction is 2MnO4– + 5SO32– + 6H+ ææ
Æ 2Mn2+ + 5SO42– + 3H2O
Hence, 1 mol SO32– ∫∫ (2/5) mol MnO42–
2. We have pV = nRT =(m/M)RT fi pM = (m/V) RT = rRT
p(X) r (X) M (Y)
Hence = = (3) (2) = 6
p(Y) r (Y) M (X)
3. In NaCl crystal, r (Na+) + r (Cl–) = a/2. Hence r (Na+) = (564 pm/2) – 181 pm = 101 pm
4. In Schottky defect, lesser number of ions as compared to a perfect crystal are present. Hence, the density of the
crystal exhibiting Schottky defect will be less as compared to that of the perfect crystal.
5. The expression of orbital angular momentum is L = l (l + 1) (h/2p). For d-electron, l = 2. Hence L = 6 (h/2p).
2
6. The energy expression is En = – RH /n . For n = 2, E

8. NO2+ is linear, NO3– is planar and NH4+ is tetrahedral.


9. We have DH = DU + (Dvg) RT = [– 743 + (1/2) (8.314 × 10–3) (300)] kJ mol–1 = – 741.75 kJ mol–1
10. The reaction is endothermic. Hence, increasing temperature favours forward direction of the reaction.
11. pH < (1/2) pK°w for acidic solution.
12. For the electrolyte MmAa, we have
a+ m–

M mA a  
 mM + a A
Ksp = [Ma+]m [Am–]a = (ms)m (as)a . Hence s = (Ksp/mm aa)1/(m + a)
13. Since, m = (Q/F) (M/|ve|), we have
Q = mF|ve|/M = (rV) F|ve|/M = [(8.94 × 1) (96500 × 2)/63] C = 27172 C
RT pH
14. For the reaction 2H+ + 2e– = H2, we have E = - In + 2 2
2F [H ]
For pH 2 = 2 atm and [H+] = 1 M, E is negative.
DTf will be least, hence Tf will be highest.
d[N 2 O5 ] 1 d[N 2 O 5 ]
16. We will have = k [N 2 O 5 ] and k [N 2 O 5 ]
dt 2 dt
To have the same value of – d [N2O5]/dt, we will have k = 2k¢.
2, the bond acquires partial double character, hence the highest bond enthalpy.
PV.4 Complete Chemistry—JEE Main

2–
O O
18. The structure of Cr2O72– ion is Cr Cr
O O O O O
OH
19. The structure of hypophosphorous acid is P . It contains one —OH group. Hence, it is monobasic.
H
H O
2– 3
4 and [NiCl 4 ] involve sp hybridization of Ni orbitals.
4+ + 3+ 3+
21. The ions Ti , Cu , Cr have (3d) , (3d) , (3d)3 and (3d)1
0 10

be colourless, colourless, coloured and coloured, respectively.


H2 H2
N N
22. The dien is H2C CH2 . It involves three coordination centres.
H2 C N CH2
H
23. Thorium, Neptunium, Uranium and Actinium series are 4n, 4n + 1, 4n + 2 and 4n + 3 radioactive series, respec-
tively.
24. The order of reactivity is 3° > 2° > 1°.
25. We have
CHO OHC CH CH
O3
CH2 CH2 CH2 CH2
CHO OHC CH CH
26. The order of acid strength is CH3COOH > CH3CH2OH > HC ∫∫ CH
Their negative ions will follow reverse base strength.
JEE (Main) Chemistry
Offline Solved Paper—2017
1. Which of the following compounds will form significant amount of meta product during mono-nitration reaction?

OH OCOH3 NH2 NHCOCH3

(a) (b) (c) (d)

2. DU is equal to
(a) isochoric work (b) isobaric work (c) adiabatic work (d) isothermal work
3. The increasing order of the reactivity of the following halides for the SN1 reaction is

CH3 -- CH -- CH 2 CH3 CH3CH2CH2Cl p – H3CO — C6H4 — CH2Cl



Cl
(I)    (II) (III)
(a) (III) < (II) < (I) (b) (II) < (I) < (III) (c) (I) < (III) < (II) (d) (II) < (III) < (I)
4. The radius of the second Bohr orbit for hydrogen atom is:
(Planck constant, h = 6.6262 ¥ 10–34 J s; mass of electron, me = 9.1091 ¥ 10–31 kg; charge of electron, e = 1.60210
¥ 10–19 C; and permittivity of vacuum, e = 8.854185 ¥ 10–12 kg–1 m–1 A2.)
(a) 1.65 Å (b) 4.76 Å (c) 0.529 Å (d) 2.12 Å
5. pKa of a weak acid (HA) and pKb of a weak base (BOH) are 3.2 and 3.4, respectively. The pH of their salt (AB)
solution is
(a) 7.2 (b) 6.9 (c) 7.0 (d) 1.0
6. The formation of which of the following polymers involves hydrolysis reaction?
(a) Nylon 6 (b) Bakelite (c) Nylon 6, 6 (d) Terylene
7. The most abundant elements by mass in the body of a healthy human adults are:
Oxygen (61.4%); Carbon (22.9%); Hydrogen (10.0%) and Nitrogen (2.6%). The mass which a 75 kg person would
gain if all 1H atoms are replaced by 2H atoms is
(a) 15 kg (b) 37.5 kg (c) 7.5 kg (d) 10 kg.
8. Which of the following, upon treatment with tert-BuONa followed by addition of bromine water, fails to decolourize
the colour of bromine?

JEE (Main) Chemistry ofline Paper I.indd 1 2/7/2018 4:05:42 PM


JEEC.2  Complete Chemistry—JEE Main

O O C6H5
(a) (b)
Br Br
O
O
(c) (d)
Br Br
9. In the following reactions, ZnO is respectively acting as a/an
I. ZnO + Na2O Æ Na2ZnO2 II. ZnO + CO2 Æ ZnCO3
(a) base and acid (b) base and base (c) acid and acid (d) acid and base
10. Both lithium and magnesium display several similar properties due to the diagonal relationship. However, the one
which is incorrect is:
(a) Both form basic carbonates
(b) Both form soluble bicarbonates
(c) Both form nitrides
(d) Nitrates of both Li and Mg yield NO2 and O2 an heating.
11. 3-Methylpent-2-ene on reaction with HBr in presence of peroxide forms an addition product. The number of
possible stereoisomers for the product is:
(a) Six (b) Zero (c) Two (d) Four
12. A metal crystallises in a face-centred cubic structure. If the edge length of its unit cell is a, the closest approach
between two atoms in the metallic crystal will be
(a) 2a (b) 2 2  a (c) 2  a (d) a / 2
13. Two reactions R1 and R2 have identical pre-exponentital factors. Activation energy of R1 exceeds that of R2 by
10 kJ mol–1. If k1 and k2 are rate constants for the reactions R1 and R2, respectively at 300 K, then ln (k2/k1) is equal to:
(R = 8.314 J K–1 mol–1)
(a) 8 (b) 12 (c) 6 (d) 4
14. The correct sequence of reagents for the following conversion will be
O
HO CH3

CHO HO CH3
CH3

(a) [Ag(NH3)2]+OH–,  H+/CH3OH,  CH3MgBr (b) CH3MgBr,  H+/CH3OH,  [Ag(NH3)2]+OH–


(c) CH3MgBr,  [Ag(NH3)2]+OH–,  H+/CH3OH (d) [Ag(NH3)2]+OH–,  CH3MgBr,  H+/CH3OH
15. The Tyndall effect is observed only when following conditions are observed:
(I) The diameter of the dispersed particles is much smaller than the wavelength of the light used.
(II) The diameter of the dispersed particles is not much smaller than the wavelength of the light used.
(III) The refractive indices of the dispersed phase and dispersion medium are almost similar in magnitude
(IV) The refractive indices of the dispersed phase and dispersion medium differ greatly in magnitude
(a) (I) and (IV) (b) (II) and (IV) (c) (I) and (III) (d) (II) and (III)

JEE (Main) Chemistry ofline Paper I.indd 2 2/7/2018 4:05:43 PM


Chemistry Offline Solved Paper—2017  JEEC.3

16. Which of the following compounds will behave as a reducing sugar in aqueous KOH solution?
HOH2C CH2OH HOH2C CH2OH
O O
(a) (b) HO
HO
OCOCH3

OH OH
HOH2C CH2OH HOH2C
O
O CH2OCH3
(c) HO (d)
OH
OCH3
HO
OH OH
17. Given:
C(graphite) + O2(g) Æ CO2(g); DrH° = – 393.5 kJ mol–1

H2(g) + 1
2 O2(g) Æ H2O (l); DrH° = – 285.8 kJ mol–1

CO2(g) + 2H2O (l) Æ CH4(g) + 2O2(g); DrH° = 890.3 kJ mol–1


Based on the above thermochemical equations, the value of DrH° at 298 K for the reaction
C(graphite) + 2H2(g) Æ CH4(g)  will be
(a) +74.8 kJ mol–1 (b) +144.0 kJ mol–1 (c) –74.8 kJ mol–1 (d) –144.0 kJ mol–1
18. Which of the following reactions is an example of a redox reaction?
(a) XeF4 + O2F2 Æ XeF6 + O2 (b) XeF2 + PF5 Æ [XeF]+ PF –6
(c) XeF6 + H2O Æ XeOF4 + 2HF (d) XeF6 + 2H2O Æ XeO2F2 + 4HF
19. The product obtained when chlorine gas reacts with cold and dilute aqueous NaOH are:
(a) ClO– and ClO–3 (b) ClO–2 and ClO–3 (c) Cl– and ClO– (d) Cl– and ClO–3
20. The major product obtained in the following reaction is:
Br BuOK
H C H
6 5 æææÆ D
C6H5
(a) (±) C6H5CH(OBu)CH2C6H5 (b) C6H5CH == CHC6H5
(c) (+) C6H5CH(OBu)CH2C6H5 (d) (–) C6H5CH(OBu)CH2C6H5
21. Sodium salt of an organic acid ‘X’ produces effervescence with conc. H2SO4. X reacts with acidified aqueous
CaCl2 solution to give a white precipitate which decolourises acidic solution of KMnO4. The compound X is
(a) C6H5COONa (b) HCOONa (c) CH3COONa (d) Na2C2O4
22. Which of the following species is not paramagnetic?
(a) NO (b) CO (c) O2 (d) B2
23. The freezing point of benzene decreases by 0.45 °C when 0.2 g of acetic acid is added to 20 g of benzene. If
acetic acid associates to form a dimer in benzene, percentage association of acetic acid in benzene will be (Given:
Kf(benzene) = 5.12 K kg mol–1):
(a) 64.6% (b) 80.4% (c) 74.6% (d) 94.6%
24. Which of the following molecule is least resonance stabilized.

(a) (b) (c) (d) O


N
O

JEE (Main) Chemistry ofline Paper I.indd 3 2/7/2018 4:05:44 PM


JEEC.4  Complete Chemistry—JEE Main

25. On treatment of 100 mL of 0.1 M solution of CoCl3 ◊ 6H2O with excess AgNO3, 1.2 ¥ 1022 ions are precipitated.
The complex is
(a) [Co(H2O)4Cl2]Cl ◊ 2H2O (b) [Co(H2O)3Cl3] ◊ 3H2O
(c) [Co(H2O)6]Cl3 (d) {Co(H2O)5Cl]Cl2 ◊ H2O
26. The major product obtained in the following reaction is
O
O

DIBAL-H
ææææÆ

COOH
OH OH

(a) CHO (b) CHO

COOH CHO

(c) CHO (d) CHO

COOH CHO
27. A water sample has ppm level concentrations of following anions
F– = 10;  SO2– –
4 = 100;  NO 3 = 50
The anion/anions that make/makes the water sample unstable for drinking is/are
(a) only NO–3 (b) both SO2– –
4 and NO 3 (c) only F– (d) only SO2–4
28. One gram of a carbonate (M2CO3) on treatment with excess HCl produces 0.01186 mol of CO2, the molar mass
of M2CO3 in g mol–1 is
(a) 1186 (b) 84.3 (c) 118.6 (d) 11.86
29. Given:
E°Cl2 | Cl– = 1.36 V E°Cr3+ | Cr = – 0.74 V
E°Cr2O 7 |Cr3+ = 1.33 V
2–
E°MnO– | Mn2+ = 1.51 V
4
Among the following, the strongest reducing agent is
(a) Cr (b) Mn2+ (c) Cr3+ (d) Cl–
30. The group having isoelectronic species is
(a) O2–, F–, Na+, Mg2+ (b) O–, F–, Na, Mg+ (c) O2–, F–, Na, Mg2+ (d) O–, F–, Na+ , Mg2+

 ANSWERS
1. (c) 2. (a) 3. (b) 4. (d) 5. (b) 6. (a)
7. (c) 8. (a) 9. (d) 10. (a) 11. (d) 12. (d)
13. (d) 14. (a) 15. (b) 16. (a) 17. (c) 18. (a)
19. (c) 20. (b) 21. (d) 22. (b) 23. (d) 24. (d)
25. (d) 26. (b) 27. (c) 28. (b) 29. (a) 30. (a)

JEE (Main) Chemistry ofline Paper I.indd 4 2/7/2018 4:05:44 PM


Chemistry Offline Solved Paper—2017  JEEC.5

  HINTS AND SOLUTIONS

1. Nitration is carried out in the presence of conc. H2SO4 and conc. HNO3. Aniline being a base forms anilinum ion
+
with acid. The -- NH3 ion being a deactivating group results in the formation of meta mono-nitration product.
+NH NH3
NH2 3

H+ conc. HNO
æææææ Æ
ææÆ conc.H SO
2
3
4
NO2
47%

2. Adiabatic condition implies q = 0. Thus, from the first law of thermodynamics, we have
DU = q + w = 0 + w = w
3. The SN1 reaction proceeds through ionic mechanism. The larger the stability of the resultant carbocation, larger
the reactivity of the halide.
The stability of the carbocations formed from the given compounds is
+ + +
CH3CH 2 C H 2 < CH3 -- C H -- CH 2 CH3 < p - H3CO -- C6 H 4 -- CH 2
(II) (I) (III)

The carbocation from the compound (III) is most stable due to conjugation with phenyl group. The primary car-
bocation (II) is lesser stable than the secondary carbocation (I).
4. Recall that the Bohr radius (i.e. radius of n = 1 orbit of hydrogen atom) is 52.9 pm ( = 52.9 ¥ 10–12 m = 0.529 Å).
Also,
rn = n2 r1
Hence r2 = (2)2 (0.529 Å) = 2.116 Å  2.12 Å

5. The pH of a solution containing a salt of weak acid and weak base is given by the expression
1
pH = (pKw + pKa – pKb)
2
Hence, at 25 °C, we have
1
pH =
(14 + 3.2 – 3.4) = 6.9
2
6. Nylon 6 is obtained by hydrolysis of caprolactam:

O
N O
ΩΩ
(i) hydrolysis
ææææææÆ C -- NH -- (CH 2 )5
(ii) polymerization n

Caprolactam Nylon-6

10
7. Mass of 1H in 75 kg of the person = ¥ 75 kg = 7.5 kg. Since the replacement of 1H by 2H causes doubling
100
the mass, the gain in mass of the person would be 7.5 kg.
8. Treatment with tert-BuONa will cause elimination reaction resulting into the formation of alkene. The compound
of choice (a) does not undergo elimination reaction and thus will not decolourize bromine water.

JEE (Main) Chemistry ofline Paper I.indd 5 2/7/2018 4:05:46 PM


JEEC.6  Complete Chemistry—JEE Main

9. Oxygen acceptor is an acid and oxygen gainer is a base.


In the reaction I, ZnO gains O while in reaction II, ZnO loses O.
10. Magnesium forms basic carbonate (4MgCO3 ◊ Mg(OH)2 ◊ 5H2O) whereas Li does not. All the other choices
(namely, b, c and d) are correct.
11. HBr in the presence of peroxide follows anti-Markownikov addition which states that the hydrogen of acid at-
taches to the carbon containing lesser number of hydrogen. The addition follows free-radical mechanism. Tertiary
radical is formed by the addition of Br ∑ followed by the addition of H.
CH3 Br CH3 Br CH3
Br • H+
CH3 CH C CH2CH3 æææ
Æ CH3 CH C CH2CH3 æææ
Æ CH3 CH CH CH2CH3
* *
3-methyl pent-2-ene

There are two asymmetric carbon leading to 22 (i.e. four) optical isomers.
12. In a face-centred cubic unit cell, atoms touch each other along the face diagonal of the cube. There are atoms at
the corners and one at the face of the cube. Thus, the face-diagonal 2 a will be equal to 4r. The closest approach
between two atoms is 2r and thus will be equal to a/ 2 .
–1)/RT
13. k2 = A e– E2/RT and k1 = A e– (E2 + 10 kJ mol
k2 e- E2 / RT
Hence =
k1
-1
e- ( E2 + 10 kJ mol )/ RT
Êk ˆ E ( E + 10 kJ mol-1 ) 10 ¥ 103 J mol-1
ln Á 2 ˜ = – 2 + 2 = =4
Ë k1 ¯ RT RT (8.314 J K -1 mol-1 ) (300 K )

14. [Ag(NH3)2]+OH– is Tollens reagent. This causes oxidation of — CHO to — COOH.


H+/CH3OH causes esterification of — COOH group.
CH3MgBr adds to C O and ester groups.
The reactions proceed as follows.
O O O

H + / CH OH
[Ag(NH ) ]OH
ææææææ
3 2
Æ æææææ
3
Æ

CHO COOH C
O OCH3
HO CH3

CH3MgBr

H 3C CH3
OH

15. The choice (b) is correct.


16. The compound of choice (a) contains an ester group which is hydrolysed by KOH resulting into — OH group.
Due to hemiacetal group, the resulting compound acts as a reducing sugar.

JEE (Main) Chemistry ofline Paper I.indd 6 2/7/2018 4:05:47 PM


Chemistry Offline Solved Paper—2017  JEEC.7

CH2OH CHO
HOH2C CH2OH HOH2C CH2OH C O H C OH
O O
aq. KOH HO H HO H
HO
ææææ
Æ HO H OH H OH
H OH H OH
OCOCH3 OH
CH2OH CH2OH
OH HO Reducing sugar

17. The following manipulations gives the desired reaction.


C(graphite) + O2 (g) Æ CO2(g); DrH° = – 393.5 kJ mol–1

2[H2(g) + 1
2
O2(g) Æ H2O (l)]; DrH° = – 2 ¥ 285.8 kJ mol–1

CO2(g) + 2H2O (l) Æ CH4(g) + 2O2(g); DrH° = 890.3 kJ mol–1


Add
C(graphite) + 2H2(g) Æ CH4(g); DrH° = (– 393.5 – 2 ¥ 285.8 + 890.3) kJ mol–1

= – 74.8 kJ mol–1.
18. Only in the choice (a), there occurs change in oxidation number of Xe and O.
+4 +1 +6 0
XeF4 + O 2F Æ Xe F6 + O 2
Here Xe is oxidised whereas O is reduced.
19. With cold and dilute aqueous NaOH, Cl2 reacts as  Cl2 + 2OH– Æ Cl– + ClO– + H2O
20. Elimination of HBr occurs resulting into the formation of double bond.
Br BuOK
H C6H5 æææ
D
Æ -- CH -- C6 H5
C6 H5 -- CH --
C6H5

21. The compound X is Na2C2O4. The given reactions are:


Na2C2O4 + H2SO4 Æ Na2SO4 + CO­­≠ + CO2­≠ + H2O
conc.
Na2C2O4 + CaCl2 Æ CaC2O4­Ø + 2NaCl
white ppt.
2MnO–4 + 16H + 5C2O2–
+ 2+
4  Æ 2Mn + 8H2O + 10 CO2
(pink) (colourless)

22. The electronic configurations of given species are:


NO 11 valence electrons   (s 2s)2 (s*2s)2 (p2px)2 (p2py)2 (s 2pz)2 (p*2px)1
CO 10 valence electrons   (s2s)2 (s*2s)2 (p2px)2 (p2py)2 (s2pz)2
O2 12 valence electrons   (s2s)2 (s*2s)2 (s2pz)2 (p2px)2 (p2py)2 (p*2px)1 (p*2py)1
B 2 6 valence electrons   (s2s)2 (s*2s)2 (p2px)1 (p2py)1
The species CO does not have unpaired electron(s) and thus will not be paramagnetic.
23. Molality of acetic acid in benzene is
n2 (m2 /M 2 ) (0.2 g / 60 g mol-1 ) 1
m= = = = mol kg -1
m1 m1 0.02 kg 6

JEE (Main) Chemistry ofline Paper I.indd 7 2/7/2018 4:05:48 PM


JEEC.8  Complete Chemistry—JEE Main

In the solution, we have


2CH3COOH   (CH3COOH)2
m(1 – a) m(a/2)

Total molality of solution, mtotal = m(1 – a) + m(a/2) = m(1 – a/2)


Since – DTf = Kfm, we have
(0.45 K) = (5.12 K kg mol–1) {(1 mol kg–1)/6} (1 – a/2)
Ê 6 ¥ 0.45 ˆ
This gives    a = 2 Á1 - ˜ = 0.9453
Ë 5.12 ¯
0.9453
The percentage association of acetic acid is  ¥ 100 = 94.53%
1

24. The molecule O is least-resonance stabilized as it does not involve 4n + 2 electrons (aromatic

requirement).
25. Amount of complex, n1 = MV = (0.1 mol L–1)(0.1 L) = 0.01 mol
N 1.2 ¥ 1022
Amount of chloride precipitated,  n2 = = = 0.02 mol
N A (6.022 ¥ 1023 mol-1 )
The complex of choice (d) will produce twice the precipitable chloride ions.
26. DIBAL-H is diisobutylaluminium hydride. It is an electrophilic reducing agent as it reacts more quickly with
electron-rich compound. It reduces carboxylic acids, their derivatives and nitriles to aldehyde. By contrast, the
use of LiAlH4 reduces esters and acyl chloride to primary alcohol and nitriles to primary amines.
In the given reaction, the product is given by the choice (b).
27. The maximum concentrations of the given ions in the drinkable water are
NO–3 = 50 ppm;  SO42– = 500 ppm;  F– = 2 ppm
28. The reaction is  M2CO3 + 2HCl Æ 2MCl + CO2 + H2O
The stoichiometric numbers of M2CO3 and CO2 are identical, the amount of M2CO3 will also be equal to 0.01186
mol. If M is the molar mass of M2CO3, then
1g 1g
= 0.01186 mol  i.e  M = = 84.32 g mol–1
M 0.01186 mol
29. Lesser the E° of the couple, lesser the reduction tendency of the species being reduced and larger the oxidation
tendency of the species being oxidized. Larger oxidation tendency implies the strongest reducing agent.

E°Cr3+|Cr has the least value and thus Cr is the strongest reducing agent.
30. The species in the choice (a) have the same number of valence electrons, namely, 10 electrons.

JEE (Main) Chemistry ofline Paper I.indd 8 2/7/2018 4:05:49 PM


JEE (Main) Chemistry
Online Solved Paper 1—2017
1. If the shortest wavelength in Lyman series of hydrogen atom is A, then the longest wavelength in Paschen series
of He+ is
(a) 5 A/9 (b) 36 A/7 (c) 36 A/5 (d) 9 A/5
2. Among the following, the essential amino acid is
(a) Valine (b) Alanine (c) Serine (d) Aspartic acid
3. Identify the pollutant gases largely responsible for the discoloured and lustreless nature of marble of the Taj Mahel.
(a) SO2 and O3 (b) O3 and CO2 (c) SO2 and NO2 (d) CO2 and NO2
4. Which of the following compounds will not undergo Friedel-Craft’s reaction with benzene?
Cl COCl
(a) (b) Cl (c) (d)
Cl
O
5. Which of the following is paramagnetic?
(a) CO (b) NO+ (c) O2–2 (d) B2
6. The rate of a reaction A doubles on increasing temperature from 300 K to 310 K. By how much the temperature
of reaction B should be increased from 300 K that rate doubles if activation energy of the reaction B is twice that
of a reaction A?
(a) 4.92 K (b) 9.84 K (c) 19.67 K (d) 2.45 K
7. A solution containing a group-IV cation gives a precipitate on passing H2S. A solution of this precipitate in dilute
HCl produces a white precipitate with NaOH solution and bluish-white precipitate with potassium ferrocyanide.
The cation is
(a) Mn2+ (b) Zn2+ (c) Ni2+ (d) Co2+
8. Which of the following statements is not true about partition chromatography?
(a) Stationary phase is a finely divided solid adsorbent.
(b) Separation depends upon equilibrium of solute between a mobile and a stationary phase
(c) Paper chromatography is an example of partition chromatography
(d) Mobile phase can be a gas
9. Excess of NaOH(aq) was added to 100 mL of FeCl3(aq) resulting into 2.14 g of Fe(OH)3. The molarity of FeCl3(aq)
is:
(a) 0.3 M (b) 0.2 M (c) 0.6 M (d) 1.8 M
10. 5 g of Na2SO4 was dissolved in the mass x of H2O. The change in freezing point was found to be 3.82 °C. If Na2SO4
is 81.5% ionised, the value of x is (Kf (water) = 1.86 °C kg mol–1.)
(a) 25 g (b) 65 g (c) 15 g (d) 45 g

JEE (Main) Chemistry Online Paper 1_2017.indd 1 2/7/2018 4:04:15 PM


JEEC.2  Complete Chemistry—JEE Main

11. Consider the following standard electrode potentials in aqueous solutions:


Element M3+|M M+|M
Al –1.66 V + 0.55 V
Tl +1.26 V – 0.34 V
Based on these data, which of the following statements is correct?
(a) Tl3+ is more stable than Al3+ (b) Al+ is more stable than Al3+
(c) Tl+ is more stable than Al3+ (d) Tl+ is more stable than Al+
12. The major product expected form the following reaction is:
HOH2C O
HO2C
NH2
HCl(g)/CCl
ææææææ
4
Æ
OH
O O
HO2C O HO2C
(a) (b) O

Cl OH
O O HOH2C O
O HO2C
(c) NH2 (d) NH2

OH Cl
13. Among the following, the incorrect statement is
(a) At low pressure, real gases show ideal behaviour
(b) At very low temperature, real gases show ideal behaviour
(c) At Boyle’s temperature, real gases show ideal behaviour
(d) At very large volume, real gases show ideal behaviour
14. The pair of compounds having metals in their highest oxidation state is:
(a) MnO2 and CrO2Cl2 (b) [Fe(CN)6]3– and [Cu(CN)4]2–
2– 2–
(c) [NiCl4] and [CoCl4] (d) [FeCl4]– and Co2O3
15. The IUPAC name of the following compound is

(a) 2-Ethyl-1, 1-dimethylcyclohexane (b) 1, 1-Dimethyl-2-ethylcyclohexane


(c) 2,2-Dimethyl-1-ethylcyclohexane (d) 1-Ethyl-2, 2-dimethylcyclohexane
16. A mixture containing the following four compounds is extracted with 1 M HCl. The compound that goes to
aqueous layer is
O
H
S N O


(I) (II) (III) (IV)
(i) (IV) (ii) (II) (iii) (I) (iv) (III)

JEE (Main) Chemistry Online Paper 1_2017.indd 2 2/7/2018 4:04:16 PM


Chemistry Online Solved Paper 1—2017  JEEC.3

17. Consider the following ionization enthalpies of two elements A and B.


Element Ionization enthapy/kJ mol–1
1st 2nd 3rd
A 899 1757 14847
B 737 1450 7731
Which of the following statements is correct?
(a) Both A and B belong to Group 1 where A comes below B
(b) Both A and B belong to Group 2 where A comes below B
(c) Both A and B belong to Group 1 where B comes below A
(d) Both A and B belong to Group 2 where B comes below A.
18. sp3d2 hybridization is not behaved by:
(a) SF6 (b) BrF5 (c) PF5 (d) [CrF6]3–
19. The number of S == O and S — OH bonds present in peroxodisulphuric acid and pyrosulphuric acid, respectively,
are
(a) (2 and 4) and (2 and 4) (b) (4 and 2) and (4 and 2)
(c) (4 and 2) and (2 and 4) (d) (2 and 2) and (2 and 2)
20. Among the following correct statement is:
(a) Sol of metal sulphides are lyophilic
(b) Brownian movement is more pronouned for smaller particles
(c) One would expect charcoal to absorb chlorine more than hydrogen sulphide
(d) Hardy-Schulze law states that bigger the size of the ion is, the greater is its coagulating power
21. The major product of the following reaction is:
CH3
Ω C2 H5ONa
C6 H5 CH 2 -- C -- CH 2 -- CH3 æææææ C2 H5OH
Æ

Br

C6 H5 CH --
(a) C6 H5 CH 2 -- C --
-- C -- CH 2 CH3 (b) -- CHCH3
Ω Ω
CH3 CH3

CH3

C6 H5 CH 2 -- C -- CH 2 CH3 (d)
(c) C6 H5 CH 2 -- C --
-- CH 2
Ω Ω
OC2H5 CH2CH3

22. What is the standard reduction potential (E°) for Fe3+ + 3e– Æ Fe?
Given that:
Fe2+ + 2e– Æ Fe;  E°Fe2+|Fe = – 0.47 V
Fe3+ + e– Æ Fe2+;  E°Fe3+, Fe2+|Pt = 0.77 V
(a) – 0.057 V (b) + 0.30 V (c) – 0.30 V (d) + 0.057 V
23. The reason for drug induced poisoning is:
(a) Binding irreversibly to the active site of the enzyme
(b) Binding at the allosteric sites of the enzyme
(c) Binding reversibly at the active site of the enzyme
(d) Bringing conformational change in the binding site of the enzyme

JEE (Main) Chemistry Online Paper 1_2017.indd 3 2/7/2018 4:04:17 PM


JEEC.4  Complete Chemistry—JEE Main

24. The major product of the following reactions is:


OH 1. K CO
ææææææ2 3
2. CH I (1aq)
Æ
3

OH
OCH3 OH

(a) (b)
OH OCH3
O OCH3
(c) (d)

25. The major product of the following reaction is


KOH, CH OH
3
CH3 -- CH CH 2 CH CH 2 CH3 ææææææ
heat
Æ
Ω Ω
Br Br
(a) CH3CH == CH — CH == CHCH3 (b) CH2 == CHCH == CHCH2CH3
(c) CH3CH == C == CHCH2CH3 (d) CH2 == CHCH2CH == CHCH3
26. In which of the following reactions, hydrogen peroxide acts as an oxidizing agent?
(a) HOCl + H2O2 Æ H3O+ + Cl– + O2 (b) I2 + H2O2 + 2OH– Æ 2I – + 2H2O + O2
(c) PbS + 4H2O2 Æ PbSO4 + 4H2O (d) 2MnO–4 + 3H2O2 Æ MnO2 + 3O2 + 2H2O + 2OH–
27. For a reaction, A(g) Æ A(l); DH = –3RT. The correct statement for this reaction is
(a) DH = DU = 0 (b) |DH| < |DU| (c) |DH| > |DU| (d) DH = DU π 0
28. The enthalpy change on freezing of 1 mol of water at 5 °C to ice at –5 °C is:
(Given: DfusH = 6 kJ mol–1 at 0 °C,   Cp(H2O, l) = 75.3 J mol–1 K–1,  Cp(H2O, s) = 36.8 J mol–1 K–1.)
(a) – 6.0 kJ mol–1 (b) –5.81 kJ mol–1 (c) –5.44 kJ mol–1 (d) – 6.56 kJ mol–1
29. Addition of sodium hydroxide solution to a weak acid (HA) results in buffer of pH = 6. If ionization constant of
HA is 10–5, the ratio of salt to acid concentration in the buffer solution will be
(a) 10 : 1 (b) 4 : 5 (c) 5 : 4 (d) 1 : 10
30. A meal M reacts with nitrogen gas to afford M3N. The compound M3N on heating at high temperature gives back
M and on reaction with water produces a gas B. The gas B reacts with aqueous solution of CuSO4 to form a deep
blue compound. The species M and B, respectively, are
(a) Li and NH3 (b) Na and NH3 (c) Ba and N2 (d) Al and N2

 ANSWERS
1. (b) 2. (a) 3. (c) 4. (a) 5. (d) 6. (a)
7. (b) 8. (d) 9. (b) 10. (d) 11. (d) 12. (c)
13. (b) 14. (a) 15. (a) 16. (b) 17. (d) 18. (c)
19. (b) 20. (c) 21. (a) 22. (a) 23. (b) 24. (a)
25. (a) 26. (c) 27. (c) 28. (d) 29. (a) 30. (a)

JEE (Main) Chemistry Online Paper 1_2017.indd 4 2/7/2018 4:04:18 PM


Chemistry Online Solved Paper 1—2017  JEEC.5

  HINTS AND SOLUTIONS

1. The energy difference between two Bohr orbits is


Ê 1 1ˆ 1 Ê 1 1ˆ
DE = R•hc Z2 Á - 2 ˜ .  This gives  = R •Z 2 Á 2 - 2 ˜
2
Ë n1 n2 ¯ l Ë n1 n2 ¯
For the shortest wavelength (i.e. maximum energy difference) in the Lyman series of hydrogen atom (Z = 1), we
have n1 = 1 and n2 = •. Thus
1
= R•
l1
For the longest wavelength (i.e.  minimum energy difference) in the Paschen series of He+ ion (Z = 2), we have
n1 = 3 and n2 = 4
1 Ê 1 1ˆ 7 R• 36 36 Ê 36 ˆ
Hence = R•(2)2 Á 2 - 2 ˜ = . Obviously, l2 = = l1 Á = A
l2 Ë3 4 ¯ 36 7 R• 7 Ë 7 ˜¯
2. The essential amino acids are not synthesized by human body. These have to be supplied in the human diet.
Valine is an example of essential amino acid.
3. The gases are SO2 and NO2.
4. The compound H2C == CH — Cl will not undergo Friedel-Craft reaction as it is unable to form carbocation
(i.e. electrophile).
5. The electronic configurations of the given species are as follows.

Species Valence electrons Electronic configuration


CO 10 (s 2s)2 (s *2s)2 (p 2px)2 (p 2py)2 (s 2pz)2
NO+ 10 same as CO
O22– 14 (s 2s)2 (s 2s)2 (s 2pz)2 (p 2px)2 (p 2py)2 (p *2px)2 (p *2py)2
B2 6 (s 2s)2 (s *2s)2 (p 2px)1 (p 2py)1
The species B2 contains unpaired electrons, and is thus paramagnetic.
6. The variation of rate constant with change in temperature is given by the expression
Êk ˆ E Ê 1 1ˆ
ln Á 2 ˜ = – a Á - ˜
Ë k1 ¯ R Ë T2 T1 ¯
From the given information, we write
1 1 Ê 1 1 ˆ 1 1 Ê 1 1 ˆ
- = 2Á - ˜  i.e.  - = 2Á - ˜
T2,A T1,A Ë T2, B T1, B ¯ 310 K 300 K Ë T2, B 300 K ¯
From this, we get
1 1Ê 1 1 ˆ (305 K) Ê 310 ¥ 300 ˆ
= Á + ˜ =  i.e. T2, B = Á K ˜ = 304.9 K
T2, B 2 Ë 310 K 300 K ¯ (310 K) (300 K) Ë 305 ¯
DT2, B = 304.9 K – 300 K = 4.9 K
7. The cation is Zn2+. The given reactions are as follows.
NH Cl / NH OH
Zn2+ + S2–  ææææææææ
4 4 Æ   ZnS ZnS + 2HCl  æÆ ZnCl2 + H2S
(precipitate) dil. (dissolves)

ZnCl2 + 2NaOH  æÆ Zn(OH)2 + 2NaCl Zn2+ + [Fe(CN)6]4–  æÆ Zn2[Fe(CN)6]


(white ppt.) (bluish-wihte ppt.)

JEE (Main) Chemistry Online Paper 1_2017.indd 5 2/7/2018 4:04:21 PM


JEEC.6  Complete Chemistry—JEE Main

8. The choice (d) is not true


9. The reaction is  FeCl3 + 3NaOH æÆ Fe(OH)3 + 3NaCl
Molar mass of Fe(OH)3, M = (56 + 3 ¥ 17) g mol–1 = 107 g mol–1
m 2.14 g
Amount of Fe(OH)3 obtained,  n = = = 0.02 mol
M 107 g mol-1
From the stoichiometry of chemical equation, we find that
Amount of FeCl3 = Amount of Fe(OH)3 = 0.02 mol
n 0.02 mol
Molarity of ferric chloride is  M = = = 0.2 mol L–1
V 0.1 L
10. Molar mass of Na2SO4, M = (2 ¥ 23 + 32 + 4 ¥ 16) g mol–1 = 142 g mol–1
m 5g 5
Amount of Na2SO4, n = = -1
= mol
M 142 g mol 142
5
Molality of Na2SO4, m = mol
142 ¥ x
Since Na2SO4 is 81.5% ionized, we will have
Na2SO4    2Na+  +  SO2–
4
m(1 – 0.815) 2(0.815 m) 0.815 m

Total molality in the solution


Ê 5 ˆ
mtotal = m(1 – 0.815) + 2(0.815 m) + 0.815 m = 2.63 m = (2.63) Á mol˜
Ë 142 ¥ x ¯
Now, since, – DTf = Kf mtotal, we have
Ê 5 ¥ 2.63 ˆ
(3.82 °C) = (1.86 °C kg mol–1) Á mol˜
Ë 142 ¥ x ¯
Ê 1.86 ¥ 5 ¥ 2.63 ˆ
This gives  x = Á kg = 0.045 kg = 45 g
Ë 3.82 ¥ 142 ˜¯
11. Larger the negative reduction potential, lesser the tendency of reduction of ion and is thus more stable towards
reduction. From the given reduction potentials, the stability of ions follows the order:
Al3+ > Tl+ > Al+ > Tl3+
The choice (d) satisfies the above order.
12. The reaction is:
HO
HO CH2 O O O
O
O HCl/CCl NH2
NH2 æææææ
-H O
4
Æ
2

OH OH

13. The choice (b) is incorrect. Real gases behave ideally at high temperature and low pressure (i.e. high volume).
14. The oxidation states of metals are:
+IV +VI +III +II
(a) MnO2 and CrO2Cl2 (b) [Fe(CN)6]3– and [Cu(CN)4]2–
+II +II +III +III
(c) [NiCl4]2– and [CoCl4]2– (d) [FeCl4]– and Co2O3
Of the given metals, the choice (a) has the highest oxidation states.

JEE (Main) Chemistry Online Paper 1_2017.indd 6 2/7/2018 4:04:22 PM


Chemistry Online Solved Paper 1—2017  JEEC.7

15. The numbering in the given compound goes as follows.

1
2
3

The compound is named as  2-ethyl-1, 1-dimethylcyclohexane.


16. The compound (II) goes into aqueous layer because it contains basic NH group.
17. The expected electronic configurations of Groups 1 and 2 are:
Group 1 (Inert gas) (ns)1 Group 2 (Inert gas) (ns)2
For Group 1, 2nd ionization enthalpy is expected to be large while for Group 2, 3rd ionization enthalpy is expected
to be larger. Moreover, ionization enthalpy decreases on descending a group.
Thus, both A and B belong to Group 2 with element B comes below A (choice d).
18. The molecule PF5 does not have sp3d2 hybridization as there are five pairs of electrons around P leading to sp3d
hybridization
19. The given acids are:
Peroxodisulphuric acid Pyrosulphuric acid
O O O O
ΩΩ ΩΩ ΩΩ ΩΩ
HO -- S -- O -- O -- S -- OH HO -- S -- O -- S -- OH
ΩΩ ΩΩ ΩΩ ΩΩ
O O O O
Four S == O bonds Four S == O bonds
Two S — OH bonds Two S — OH bond
20. Sols of metal sulphides are lyophobic. Brownian movement is more pronounced for bigger particles such as col-
lidal particles.
Hardy-Schulze rule states that the larger is the charge on ion, larger is the coagulation power.
The choice (c) is correct. Charcoal is expected to absorb chlorine more than hydrogen sulphide.

-- C -- CH 2 CH3 as the double bond is in conjugation with C6H5 group.


21. The product is C6 H5 CH --

CH3

22. Fe2+ + 2e– Æ Fe DG°1 = –2F (–0.47 V)


3+ – 2+
Fe + e Æ Fe DG°2 = –F(0.77 V)
Add
Fe3+ + 3e– Æ Fe DG° = –2F(– 0.47 V) – F(0.77 V) = (0.17 V)F

DG∞
E°Fe3+|Fe = – = – 0.057 V
3F
23. The choice (b) is correct. Binding of the drug at the allosteric site changes the shape of the active site making the
enzyme to be inactive.

JEE (Main) Chemistry Online Paper 1_2017.indd 7 2/7/2018 4:04:23 PM


JEEC.8  Complete Chemistry—JEE Main

24. The reaction is


OH OCH3
1. K 2CO3
ææææææ
2. CH3I (1aq)
Æ

OH OH

Phenolic hydrogen is more acidic than hydroxyl hydrogen and is thus extracted by base K2CO3.
25. The product is  CH3CH == CH — CH == CHCH3.  It contains extended configuration.
26. H2O2 in the reaction of choice (c) acts as an oxidizing agent
increase in oxidation state
reducing agent
-II -I +VI -II
PbS + 4H2O2 PbSO4 + 4H2O
decrease in oxidation state
oxidizing agent

27. For the given reaction, Dng = –1


Hence DH = DU + (Dng)RT gives
DH | > | DU |
DU = DH – (Dng)RT = –3RT – (–1)RT = –2RT. Obviously, | 
28. Water at 5 °C (i)
Water at 0 °C (ii)
Ice at 0 °C (ii)
Ice at -5 °C

The enthalpy changes in the above processes are:


Step (i)
DH = Cp(l)DT = (75.3 J mol–1 K–1) (–5 K) = – 376.5 J mol–1
Step (ii)
DH = DfreezH = –DfusH = – 6 kJ mol–1
Step (iii) DH = Cp(s)DT = (36.8 J K–1 mol–1) (–5 K) = –184.0 J mol–1
The total enthalpy change is
DH = (–376.5 – 6000 – 184.0) J mol–1 = – 6560.5 J mol–1 = – 6.56 kJ mol–1

29. Using Henderson equation


Ê [salt] ˆ Ê [salt] ˆ
pH = pKa + log Á  gives  log Á = pH – pKa = 6 – 5 = 1
Ë [acid] ˜¯ Ë [acid] ˜¯
Hence [salt] : [acid] :: 10 : 1
30. Species M and B, respectively, are Li and NH3 as is evident from the following reactions.
6Li + N2 æÆ 2Li3N
(M)
Li3N + 3H2O æÆ 3LiOH + NH3
 (B)
Cu2+ +4NH3 æÆ [Cu(NH3)4]2+

JEE (Main) Chemistry Online Paper 1_2017.indd 8 2/7/2018 4:04:24 PM


JEE (Main) Chemistry
Online Solved Paper 2—2017
1. A compound of molecular formula C8H8O2 reacts with acetophenone to form a single cross-aldol product in the
presence of base. The same compound on reaction with conc. NaOH forms benzyl alcohol as one of the products.
The structure of the compound is:
O
CHO

(a) (b) OCH3

CH3O
O O

OH
(c) (d) CH3

H 3C HO

2. Which of the following ions does not liberate hydrogen gas on reaction with dilute acids?
(a) V2+ (b) Ti2+ (c) Mn2+ (d) Cr2+
3. The rate of a reaction quadruples when the temperature changes from 300 K to 310 K. The activation energy of the
reaction is: (Assume activation energy and pre-exponential factor are independent of temperature; ln 2 = 0.693;
R = 8.314 J K–1 mol–1.)
(a) 53.6 kJ mol–1 (b) 214.4 kJ mol–1 (c) 107.2 kJ mol–1 (d) 26.8 kJ mol–1
4. The group having triangular planar structure is:
(a) BF3, NF3, CO2– 3 (b) CO2– –
3, NO3, SO3 (c) NH3, SO3, CO2– 3 (d) NCl3, BCl3, SO3
5. The electronic configuration with the highest ionization enthalphy is:
(a) [Ar] (3d)10 (4s)2 (4p)3 (b) [Ne] (3s)2 (3p)1 (c) [Ne] (3s)2 (3p)3 (d) [Ne] (3s2) (3p)2
6. The electron in the hydrogen atom undergoes transition from higher orbitals to orbital of radius 211.6 pm. The
transition is associated with
(a) Lyman series (b) Balmer series (c) Brackett series (d) Paschen series
7. Which one of the following is an oxide?
(a) SiO2 (b) KO2 (c) BaO2 (c) CsO2
8. 50 mL of 0.2 M ammonia solution is treated with 25 mL of 0.2 M HCl. If pKb of ammonia solution is 4.75, the
pH of the mixture is:
(a) 8.25 (b) 9.25 (c) 3.75 (d) 4.75

JEE (Main) Chemistry Online Paper 2_2017.indd 1 2/7/2018 4:05:14 PM


JEEC.2  Complete Chemistry—JEE Main

9. Which of the following compounds is most reactive to an aqueous solution of sodium carbonate?

(a) (b) (c) (d)

10. The number of P—OH bonds and the oxidation state of phosphorus atom in pyrophosphoric acid (H 4P2O7)
respectively are:
(a) five and four (b) four and five (c) five and five (d) four and four
11. The following reaction occurs in the Blast furnace where iron ore is reduced to iron metal:
Fe2O3(s) + 3CO(g)  2Fe(l) + 3CO2(g)
Using the Le Chatelier’s principle, predict which one of the following will not disturb the equilibrium?
(a) Addition of CO2 (b) Removal of CO2 (c) Addition of Fe2O3 (d) Removal of CO
12. A solution is prepared by mixing 8.5 g of CH2Cl2 and 11.95 g of CHCl3. If vapour pressure of CH2Cl2 and CHCl3
at 298 K are 415 mmHg and 200 mmHg, respectively, the mole fraction of CHCl3 in vapour form is: (Molar mass
of Cl = 35.5 g mol–1.)
(a) 0.162 (b) 0.675 (c) 0.325 (d) 0.486
13. The increasing order of the boiling points for the following compounds is:
C2H5OH   C2H5Cl  C2H5CH3  C2H5OCH3
(I) (II) (III) (IV)

(a) (IV) < (III) < I < (II) (b) (III) < (II) < (I) < (IV) (c) (III) < (IV) < (II) < (I) (d) (II) < (III) < (IV) < (I)
14. An ideal gas undergoes isothermal expansion at constant pressure. During the process:
(a) enthalpy remains constant but entropy increases (b) enthalphy decreases but entropy increases
(c) enthalpy increases but entropy decreases (d) Both enthalpy and entropy remains constant.
15. [Co2(CO)8] displays:
(a) no Co—Co bond, six terminal CO and two bridging CO
(b) no Co—Co bond, four terminal CO and four bridging CO
(c) one Co—Co bond, six terminal CO and two bridging CO
(d) one Co—Co bond, four terminal CO and four bridging CO
16. The correct sequence of decreasing number of p-bonds in the structure of H2SO3, H2SO4 and H2S2O7 is:
(a) H2S2O7 > H2SO3 > H2SO4 (b) H2S2O7 > H2SO4 > H2SO3
(c) H2SO4 > H2S2O7 > H2SO3 (d) H2SO3 > H2SO4 > H2S2O7
17. At 300 K, the density of a certain gaseous molecules at 2 bar is double to that of dinitrogen (N2) at 4 bar.
The molar mass of gaseous molecules is:
(a) 224 g mol–1 (b) 112 g mol–1 (c) 56 g mol–1 (d) 28 g mol–1
18. A gas undergoes change from state A to state B. In this process, the heat absorbed and work done by the gas is
5 J and 8 J, respectively. Now the gas is brought back to A by another process during which 3 J of heat is evolved.
In this reverse process B to A:
(a) 6 J of the work will be done by the gas
(b) 6 J of the work will be done by the surrounding on gas
(c) 10 J of the work will be done by the surrounding on the gas
(d) 10 J of the work will be done by the gas

JEE (Main) Chemistry Online Paper 2_2017.indd 2 2/7/2018 4:05:15 PM


Chemistry Online Solved Paper 1—2017  JEEC.3

19. Which of the following is a biodegradable polymer?


O O
È Ω
Ω˘
(a) C COO (CH2)2 O Í NH -- (CH 2 )5 CONH -- CH 2 -- C ˙
(b)
Î ˚n
n
O O
È Ω
Ω ˘ È Ω
Ω˘
Í NH -- (CH 2 )6 NHCO -- (CH 2 ) 4 -- C ˙ (d)
(c) Í NH -- (CH 2 )5 -- C ˙
Î ˚n Î ˚n

20. XeF6 on partial hydrolysis with water produces a compound X. The same compound X is formed where XeF6
reacts with silica. The compound X is
(a) XeF4 (b) XeF2 (c) XeO3 (d) XeOF4
21. In the following structure, the double bonds are marked as I, II, III and IV. Geometrical isomerism is not present
at site(s):
II

III

IV
I

(a) I and II (b) III (c) I (d) III and IV


22. Adsorption of a gas on a surface follows Freundlich adsorption isotherm. Plot of log (x/m) versus log p gives a
straight line with slope equal to 0.5, then: (x/m is the mass of gas adsorbed per gram of adsorbent.)
(a) Adsorption is proportional to the square root of pressure
(b) Adsorption is proportional to the square of pressure
(c) Adsorption is proportional to the pressure
(d) Adsorption is independent of pressure
23. The incorrect statement among the following is:
(a) a-D-glucose and b-D-glucose is enantiomers
(b) The penta acetate of glucose does not react with hydroxylamine
(c) a-D-glucose and b-D-glucose are anomers
(d) Cellulose is a straight chain polysaccharide made up of only b-D-glucose units.
24. In the following reaction sequence:
anhy. ZnCl2
+
KOH(aq) (i) CH MgBr conc. HCl
I ææææ
Æ II ææææææ
3
+
Æ III gives turbidity
(C3H6Cl2 ) (ii) H 2O/H immediately

The compound I is:


Cl

(a)
CH3 -- C -- CH3 (b) Cl -- CH -- CH 2 -- CH3
Ω Ω
Cl Cl
(c)
CH 2 -- CH -- CH3 (d) CH 2 -- CH 2 -- CH 2
Ω Ω Ω Ω
Cl Cl Cl Cl

JEE (Main) Chemistry Online Paper 2_2017.indd 3 2/7/2018 4:05:16 PM


JEEC.4  Complete Chemistry—JEE Main

25. The major product of the following reaction is:

Br /hn
æææÆ
2

Br
Br
Br
(a) (b) (c) (d)
Br
Br
26. Among the following compounds, the increasing order of their base strength is:
NH2 CH3
N
(I) (II) (III) (IV)
N
H NH2
(a) (II) < (I) < (III) < (IV) (b) (II) < (I) < (IV) < (III)
(c) (I) < (II) < (IV) < (III) (d) (I) < (II) < (III) < (IV)
27. What quantity (in mL) of a 45% acid solution of a mono-protic strong acid must be mixed with a 20% solution of
the same acid to produce 800 mL of a 29.875% acid solution?
(a) 316 (b) 320 (c) 325 (d) 330
28. To find the standard potential of M3+|M electrode, the following cell is constituted:
Pt | M |M3+(0.001 mol L–1) || Ag+ (0.01 mol L–1) | Ag.
The cell potential is found to be 0.421 V at 298 K. The standard potential of half reaction M3+ + 3e– Æ M at
298 K will be: (Given: E°Ag+|Ag = 0.80 V at 298 K.)
(a) 0.38 V (b) 1.28 V (c) 0.32 V (d) 0.66 V
29. Which of the following is a set of green house gases?
(a) O3, NO2, SO2, Cl2 (b) CH4, O3, N2, SO2 (c) CO2, CH4, N2O, O3 (d) O3, N2, CO2, NO2
30. Which of the following compounds will show highest dipole moment?
O

O
(I) O (II) (III) O (IV)

O
(a) (II) (b) (IV) (c) (III) (d) (I)

 ANSWERS
1. (a) 2. (c) 3. (c) 4. (b) 5. (b) 6. (b)
7. (a) 8. (b) 9. (c) 10. (b) 11. (c) 12. (c)
13. (c) 14. (a) 15. (c) 16. (b) 17. (b) 18. (b)
19. (b) 20. (d) 21. (c) 22. (a) 23. (a) 24. (a)
25. (a) 26. (a) 27. (a) 28. (c) 29. (c) 30. (d)

JEE (Main) Chemistry Online Paper 2_2017.indd 4 2/7/2018 4:05:17 PM


Chemistry Online Solved Paper 1—2017  JEEC.5

  HINTS AND SOLUTIONS

1. The reactions are


O
O
CHO CH CH C Ph
ΩΩ
+ OH- H3C -- C-- Ph
ææææææ
Æ
CH3O CH3O
(cross-addol product)

Cannizzaro
reaction
CH2OH COONa

conc. NaOH +

CH3O CH3O

2. Mn2+ does not librate hydrogen gas on reaction with dilute acids.
3. We have
Êk ˆ E Ê 1 1ˆ Ea Ê 1 1 ˆ
ln Á 2 ˜ = – a Á - ˜   i.e.  ln 4 = –  -
Ë k1 ¯ R Ë T2 T1 ¯ (8.314 J K mol ) Ë 310 K 300 K ˜¯
-1 -1 Á

This gives
(ln 4) (8.314 J K -1mol-1 ) (310 K) (300 K) È (2 ¥ 0.693) (8.314 ¥ 310 ¥ 300) ˘ -1
Ea = = Í ˙ J mol
(10 K) Î 10 ˚
= 107165.8 J mol–1 = 107.2 kJ mol–1
4. Each species of choice (b) has 24 valence electrons. Each of them has three paired electrons around the central
atom. According to VSEPR, each of them will have triangular planar structure.
5. Half-filled 3p orbitals will have higher ionization enthalpy than the (3p)1 and (3p)2 configurations. The con-
figuration [Ne] (3s)2 (3p)1 will have higher ionization enthalpy than [Ar] (3d)10 (4s)2 (4p)3 configuration as the
former belongs to 2nd period while the latter belongs to 3rd period. In general, ionization enthalpy decreases on
decending the group.
6. The radius of Bohr orbits has the following dependency.
n2

Z
The Bohr radius (n = 1, Z = 1) is 52.9 pm. The value of n for orbit of radius 221.6 pm will be 2.
Hence, the given transitions will belong to Balmer series (n1 = 2, n2 = 3, 4, …).
7. SiO2 is silicon dioxide KO2 is potassium superoxide
BaO2 is barium peroxide CsO2 is ceasium superoxide
8. The solution will contain equal amounts of ammonia and ammonium chloride. According to Henderson equation
Ê [salt] ˆ
pOH = pKb + log Á
Ë [base] ˜¯

JEE (Main) Chemistry Online Paper 2_2017.indd 5 2/7/2018 4:05:18 PM


JEEC.6  Complete Chemistry—JEE Main

we will have pOH = 4.75  and  pH = 14 – 4.75 = 9.25.


9. Cyclopentadiene on treating with a base produces cyclopentadienyl anion. The resultant anion is planar and has
6p electrons and is thus an aromatic compound— a stable species.

aq.Na CO
æææææ
2 3
Æ
_

H
O O
ΩΩ ΩΩ
10. The pyrophosphoric acid is  HO -- P -- O -- P -- OH
Ω Ω
OH OH
There are four P — OH bonds and the oxidation state of P is five.
Note: All acid in phosphoric acid has +V oxidation state of phosphorus. Choice (b) is correct because there can-
not be five O—H bonds as there are only four hydrogen atoms.
11. The equilibrium constant of the reaction does not involve the concentration of condensed phases (i.e. solid and
liquid). Hence, addition of Fe2O3 will not disturb the equilibrium.
12. Molar mass of CH2Cl2, M1 = (12 + 2 + 2 ¥ 35.5) g mol–1 = 85 g mol–1
m1 8.5 g
Amount of CH2Cl2, n1 = = 0.1 mol
M1 85 g mol-1
Molar mass of CHCl3, M2 = (12 + 1 + 3 ¥ 35.5) = 119.5 g mol–1
m2 11.95 g
Amount of CHCl3, n2 = = = 0.1 mol
M 2 119.5 g mol-1

n2 0.1 mol
Mole fraction of CHCl3 = = = 0.5
n1 + n2 0.2 mol
Partial vapour pressure of CHCl3.  p2 = x2 p*2 = (0.5) (200 mmHg) = 100 mmHg.
Partial vapour pressure of CHCl2 p1 = (1 – x2) p1* = (0.5) (415 mmHg) = 207.5 mmHg
p2 100 mmHg
Mole fraction of CHCl3 in vapour phase is   y2 = = = 0.325
p1 + p2 307.5 mmHg
13. Ethanol will have maximum boiling point due to the existence of hydrogen bondings.
Next lower boiling point will be that of C2H5Cl due to dipole-dipole interactions.
The above order is satisfied by the Choice (c). From this choice, it may be concluded that the boiling point of
C2H5CH3 will be lesser than that of C2H5OCH3.
14. Enthalpy of an ideal gas does not change due to constant temperature. Entropy will increase because of increase
in volume imparting larger randomness.
OC CO CO
15. The structure of [Co2(CO)8] is OC Co Co CO
OC CO CO
There is one Co — Co bond, six terminal CO bonds and two bridging CO bonds.

O O O
ΩΩ ΩΩ ΩΩ
16. H2S2O7 HO -- S -- O -- S -- OH H2SO4 HO -- S -- OH
ΩΩ ΩΩ ΩΩ
O O O

JEE (Main) Chemistry Online Paper 2_2017.indd 6 2/7/2018 4:05:20 PM


Chemistry Online Solved Paper 1—2017  JEEC.7

H2SO3 HO -- S -- OH
ΩΩ
O
H2S2O7 has four p–bonds (i.e. four S == O bonds), H2SO4 has two while H2SO3 has one.
17. From ideal-gas equation of state, we have
m m rRT
pV = nRT  fi  pV = RT  fi  pM = RT  fi  M =
M V p
M 2 Ê r2 ˆ Ê p1 ˆ Ê 4 bar ˆ
Thus = Á ˜ Á ˜ = (2) Á =4
M1 Ë r1 ¯ Ë p2 ¯ Ë 2 bar ˜¯
Hence M2 = 4M1 = 4 (28 g mol–1) = 112 g mol–1
+5 J - 8 J
18. A B

-5 J + w
For the cyclic process DU = 0, we will have  +5 J – 8 J – 3 J + w = 0
This gives  w = 6 J, that is, 6 J of work is done by the surroundings on the gas
19. Polymer of choice (b) is biodegradable. It is known as nylon 2-nylon 6 polymer. It is a polyamide copolymerisa-
tion of glycine (H2N — CH2 — COOH) and amino caporic acid (H2N — (CH2)5 — COOH).
20. The reactions are: XeF6 + H2O æÆ XeOF4 + 2HF  and  2XeF6 + SiO2 æÆ 2XeOF4 + SiF4
21. The site I contains two identical CH3 groups attached to double bond. This site will not show geometrical isomer-
ism.
x
22. The Freundlich adsorption isotherm is  = kp1/ n
m
Its logarithm form is
Ê xˆ Ê kˆ 1
log Á ˜ = log Á ˜ + log (p/p°); (where p° unit pressure and k° unit value of k.)
Ë m¯ Ë k∞¯ n
x
It is given that 1/n = 0.5. Hence  = k p
m
23. a-D-Glucose and b-D-glucose are anomers. The choice (a) is incorrect.
24. Since the compound III gives turbidity immediately with anhyd. ZnCl2 + conc. HCl, the compound must be a
tertiary alcohol. Its explanation from the compound of choice (a) is

Cl OH O
aq.KOH
CH3 C CH3 CH3 C CH3 CH3 C CH3
(II)
Cl OH
(I)

anhyd. ZnCl2 + HCl OH


Turbidity (Lucas test) (i) CH3Mg Br
instantaly CH3 C CH3
(ii) H2O/H+
CH3

JEE (Main) Chemistry Online Paper 2_2017.indd 7 2/7/2018 4:05:21 PM


JEEC.8  Complete Chemistry—JEE Main

25. The reaction proceeds via free radical mechanism.

hn
Br2 ææÆ 2Br
CH3
H 3C H

Br
ææÆ + HBr

(Tertiary radical most stable)

CH3
H 3C Br

Br
ææÆ
2 + Br

26. Aliphate amine is more basic than aromatic amine


Primary aliphatic amine is more basic than tertiary aliphatic amine.
In pyrrole, lone pair is involved in the delocalisation over the ring and is thus least basic.
Based on the above facts, the order is (II) < (I) < (III) < (IV) — Choice (a).
27. Let V be the volume of 45% acid solution. Since 800 mL is the final volume, the volume of 20% acid solution
will be (800 mL – V). Equating the amount of acids before and after mixing, we will have
Ê 45 ˆ Ê 20 ˆ Ê 29.875 ˆ
V¥ Á + (800 mL – V) ˜¯ = (800 mL) ÁË ˜
Ë 100 ˜¯ ÁË
100 100 ¯
or simply  45 V + (800 mL – V) (20) = (800 mL) (29.875)
Solving for V, we get
(800 mL) (9.875)
(45 V – 25 V) = (800 mL) (29.875 – 20)  i.e.  V = = 316 mL
20
28. The cell reaction are:
left-half cell M æÆ M3+ + 3e–
right-half cell Ag+ + e– æÆ Ag
cell reaction M + 3Ag+ æÆ M3+ + Ag
Its Nernst equation is
RT È [M 3+ ]/c∞ ˘
Ecell = E°cell – ln Í + 3˙
3F Î ([Ag ]/c∞) ˚
0.059 V 0.001 0.059 V
0.421 V = E°cell – log 3
= E°cell – log (1000) = E°cell – 0.059 V
3 (0.01) 3
Hence E°cell = (0.421 + 0.059)V = 0.480 V
Now E°cell = E°Ag+|Ag – E°M3+|M
This gives E°M3+|M = E°Ag+|Ag – E°cell = (0.80 – 0.48) V = 0.32 V

JEE (Main) Chemistry Online Paper 2_2017.indd 8 2/7/2018 4:05:22 PM


Chemistry Online Solved Paper 1—2017  JEEC.9

29. The green house gases are  CO2, CH4, N2O and O3.
30. In cyclopropenone, there is a strong polarization of the carbonyl group, which gives a partial positive charge with
aromatic stabilization on the ring and partial negative charge on oxygen
+
O + O- O- O- + O-
+

The resonance stabilization of charges give larger dipole moment to cyclopropenone. It is considered to be an
aromatic molecules.

JEE (Main) Chemistry Online Paper 2_2017.indd 9 2/7/2018 4:05:23 PM

Você também pode gostar